You are on page 1of 473

Contents

1. Complex Numbers and De-Moivre's Theorem 1-19


2. Quadratic Expressions 20-24
3. Theory of Equations 25-38
4. Functions 39-49
5. Mathematical Induction 50-53
6. Partial Fractions 54-57
7. Binomial Theorem 58-69
8. Permutations and Combinations 70-80
9. Matrices and Determinants 81-101
10. Measures of Dispersion 102-110
11. Probability 111-132
12. Trigonometric Functions and Identities 133-148
13. Trigonometric Equations 149-153
14. Properties of Triangles 154-168
15. Inverse Trigonometric Functions 169-173
16. Hyperbolic Functions 174-178
17. Rectangular Cartesian Coordinates 179-190
18. Straight Line and Pair of Straight Lines 191-221
19. Circle and System of Circles 222-252
20. Conic Sections 253-276
21. Vector Algebra 277-303
22. Three Dimensional Geometry 304-320
23. Limits and Continuity 321-335
24. Differentiation 336-352
25. Applications of Derivatives 353-370
26. Indefinite Integrals 371-394
27. Definite Integrals and Its Applications 395-412
28. Differential Equations 413-426
29. Miscellaneous 427-428

Practice Sets (1-3) 429-472


1
Complex Numbers and
De-Moivre’s Theorem
1. If 1, a , a 2 , ...., a n − 1 are the nth roots of unity. Sol. (d)
n −1 2α = −1 − i 3, 2 β = −1 + i 3
1
Then ∑ − ai
is equal to (5α 4 + 5β 4 + 7α −1β −1)
i = 12 [17 Sep. 2020, Shift-I] 7
= 5(α 4 + β 4) +
(n − 2 ) 2 n −1 + 1 αβ
(a) (n − 2 )2 n (b)
2n − 1 7
= 5[(α 2 + β 2)2 − 2α 2β 2] +
(n − 2 ) 2 n − 1 1 α ⋅β
(c) (d)
2n − 1 (n − 2 ) 2 n  1 2 
= 5  (2α + 2 β)2 − 2αβ  − 2 α 2 β 2  +
7
Sol. (b)   4   α ⋅β
n −1 1
∑ 1, α , α 2 , ......, α n−1 are the nth root  1 2 
i =1 2− αi = 5  × 4 − 2 − 2 + 7
4 
of unity  
Q x n −1 = 0 = 5 [1 − 2] + 7 = 2
⇒ x n − 1 = (x − 1)(x − α)(x − α 2)......(x − α n −1) i
3. If a + bi = , then (a , b) =
⇒ log(x n − 1) = log(x − 1) + log(x − α) 1− i [17 Sep. 2020, Shift-I]
+ ......log(x − α n −1) −1 −1
(a)  ,  (b)  , 
1 1
Differentiating w.r.t ‘x’, we get 2 2 2 2
1 −1 −1 1
nx n −1 1 1 1 (c)  ,  (d)  , 
⇒ = + + ...... + 2 2   2 2
x −1
n
( x − 1) ( x − α) (x − α n −1)
At (x = 2) Sol. (d)
n ⋅ 2n −1 1 1 1 a + bi =
i
⇒ =1+ + + ...... + 1− i
2n − 1 2− α 2− α2 2 − α n −1
i(1 + i) 1 i 1 1
⇒ a + bi = =− + ⇒ a=− ,b=
n −1 1  n ⋅ 2n −1  (n − 2)2n −1 + 1 2 2 2 2 2
⇒ ∑ = n − 1 =
i =1 2− α  2 −1
i
 2n − 1 4. Let z1, z 2 be two complex numbers such that

2. If 2α = − 1 − i 3 and 2β = − 1 + i 3, then z1 − iz 2 = 0 and arg(z1 z 2) = , then arg(z1) =
4
5α 4 + 5β 4 + 7 α −1β −1 is equal to [17 Sep. 2020, Shift-II]
[17 Sep. 2020, Shift-I] π − π π π
(a) (b) (c) (d)
(a) −1 (b) −2 (c) 0 (d) 2 4 8 8 3
2 AP EAMCET Chapterwise Mathematics

Sol. (c)  1
7. Let the complex numbers α and   lie on
Given z1 − i z2 = 0 α
⇒ z1 = i z2 ⇒ z1 = iz2 ⇒ z1 = − iz2 circles (x − x 0)2 + (y − y 0)2 = r 2 and
π
Clearly argument of z1 = argument of z2 − (x − x 0)2 + (y − y 0)2 = 4 r 2 respectively.
2
π If z 0 = x 0 + iy 0 satisfies the equation 2 | z 0 |2 =
or argument z1 = argument z2 −
2 r 2 + 2, then| α | = [18 Sep. 2020, Shift-I]
π
⇒ argument z2 = argument z1 + (a)
1
(b)
1
(c)
1
(d)
1
2 2 2 7 3
Let argument z1 = α
3π Sol. (c)
Then, given argument ( z1 z2 ) = As point α lies on the circle
4
3π (x − x 0)2 + (y − y0)2 = r 2
⇒ argument z1 + argument z2 =
4 ∴ |α − z0|2 = r 2, where z0 = x 0 + iy0
π 3π ⇒ |α|2 + |z0|2 −(α z0 + α z0) = r 2
α+α+ = …(i)
2 4 1
3π π Q lies on the circle (x − x 0)2 + (y − y0)2 = 4r 2
2α = − α
4 2 1
2

π ∴ − z0 = 4r 2
⇒ α= α
8
1  αz αz 
(3 + 2 i) (4 − 7 i) (12 + 13 i) ⇒ + |z0|2 −  02 + 02  = 4r 2
5. If x + iy = , then |α|2  |α| |α| 
(13 − 12 i) (2 − 3 i) (11 + 3 i)
⇒ 1 + |z0|2|α|2 − (αz0 + α z0) = 4r 2|α|2 …(ii)
x 2 + y2 = [17 Sep. 2020, Shift-II]
By subtracting Eqs. (i) and (ii), we get
1
(a) 1 (b) 2 (c) (d) 3 1−|α|2 −|z0|2 (1−|α|2) = r 2(4|α|2 −1)
2
⇒ (|α|2 −1) (|z0|2 −1) = r 2(4|α|2 −1)
Sol. (c)
Given, r2 + 2
Q |z0|2 = , we get
(3 + 2i) (4 − 7i)(12 + 13i) 2
z = x + iy =
(13 − 12i) (2 − 3i) (11 + 3i) r2
(|α|2 −1) = r 2(4|α|2 −1)
|3 + 2i|⋅ |4 − 7i|⋅ |12 + 13i| 2
|z| =
|13 − 12i|⋅ |2 − 3i|⋅ |11 + 3i| ⇒ |α|2 −1 = 8|α|2 −2
1
( 32 + 22) ( 42 + 72) ( (122 + 132) ⇒ 7|α|2 = 1 ⇒ |α|=
= 7
( 132 + 122) ⋅ ( 22 + 32) ( 112 + 32)
8. If α and β are non-real roots of
42 + 72 65 1
= = = x 3 − x 2 − x − 2 = 0, then
112 + 32 130 2
α 2020 + β 2020 + α 2020 ⋅ β 2020 =
1 1
⇒ x + y =
2 2
or x + y =
2 2
[18 Sep. 2020, Shift-I]
2 2
(a) 1 (b) 2020
6. What is the modulus of the complex number (c) 1 + α + β (d) − 1
(1 + 2 i) (− 2 + i)? [17 Sep. 2020, Shift-II]
Sol. (c)
(a) 5 (b) 5 (c) 5 5 (d) 35 Given equation, x 3 − x 2 − x − 2 = 0
Sol. (b) ⇒ (x − 2)(x 2 + x + 1) = 0
|($i + 2$i) ⋅ (−2 + i$)| = |$i + 2i||
⋅ −2 + i| −1 ± 3i
∴ α and β are or we can say
= ( 12 + 22) ⋅ ( (−2)2 + 12) 2
α and β are non-real complex roots of unity.
= 5× 5 = 5.
Complex Numbers and De-Moivre’s Theorem 3

So, let α = ω and β = ω2, where ω3 = 1 and So, x 2 = 16 and y 2 = 9


ω2 + ω + 1 = 0. Therefore vertices of the rectangle are (± 4, ± 3),
∴ α 2020 + β 2020 + α 2020 β 2020 so area of the rectangle is 4 × 4 × 3 = 48.
= ω2020 + ω4040 + ω2020 ω4040 Hence, option (c) is correct.

= (ω3)673ω + (ω3)1346ω2 + (ω3)673ω(ω3 )1346 ω2 11. If m and n are the least and greatest values
of| z | respectively and| z − 4 + 3 i |≤ 1. Let k
= ω + ω2 + ω3 = 1 + ω + ω2 = 1 + α + β.
x4 + x2 + 4
be the least value of on the
9. The equation of any ............ in the complex x
plane is of the form zz + bz + bz + c = 0 where interval (0 , ∞). Then k = [18 Sep. 2020, Shift-II]
(b ∈C, c ∈R) [18 Sep. 2020, Shift-II] (a) n (b) m (c) m + n (d) mn
(a) Circle (b) Straight line Sol. (a)
(c) Parabola (d) Hyperbola Z – 4+ 3i ≤ 1
Sol. (a) Z – (4 – 3i) ≤ 1
Let Equation of circle in cartesian system be It represent a circle of radius less than or equal
x 2 + y 2 + 2gx + 2 fy + c = 0 … (i) to 1 unit
Let Z = x + iy Minimum value of Z = OP = OC − CP = 5 − 1
Z = x − iy m= 4
Z + Z = 2x Maximum value of Z = OQ = OC + CQ = 5 + 1
Z − Z = 2iy n= 6
x4 + x2 + 4 4
Z−Z = x3 + x +
2y = = i(Z − Z) x x
i
Z ⋅ Z = (x + iy)(x − iy) x4 + x2 + 4 1 1 1 1
= x + x+ + + +
3
x x x x x
Z ⋅ Z = x 2 + y2
Since, AM ≥ GM
∴From Eq. (i) 1 1 1 1
x3 + x + + + +
Z Z + g(Z + Z ) + i (Z − Z) f + c = 0 x x x x ≥ 6 x3 ⋅ x ⋅ 1 ⋅ 1 ⋅ 1 ⋅ 1 ≥ 6 1
Z Z + (g − if ) Z + (g + if ) Z + c = 0 6 x x x x
Z Z + b Z + b Z + c = 0 [Let b = g+if] 1 1 1 1
x3 + x + + + + ≥ 6
Hence, option (a) is correct. x x x x
x4 + x2 + 4x
10. Let z = x + yi, where, x, y are integers and ≥6
4
i = − 1 the area of the rectangle whose x4 + x2 + x
∴ Minimum Value of =6
vertices are the roots of the equation 4
zz 3 + z(z)3 = 700 is ........ [18 Sep. 2020, Shift-II] k=6 [given]
(a) 32 (b) 40 (c) 48 (d) 80 k=n [Q 6 = n]
Hence, option (a) is correct.
Sol. (c)
It is given, for a complex number Z = x + iy, 12. Geometrically, the set { z ∈ C :| z − 2 − 2i |≤ 1}
where x , y are integers such that represents ........... [21 Sep. 2020, Shift-I]
Z Z 3 + Z (Z)3 = 350 (a) a closed circular disc with center at (− 2, − 2 ) and
⇒ ZZ (Z 2 + (Z)2) = 350 with radius 1
⇒ (x 2 + y 2) 2(x 2 − y 2) = 350 [∴Z Z = Z ]
2 (b) a closed circular disc with center at (2, 2 ) and
with radius 1
⇒ (x 2 + y 2) (x 2 − y 2) = 175 (c) a closed circular disc with center at (11
, ) and with
∴ x 2 + y 2 = 25 radius 0.5
(d) a closed circular disc with center at (− 1, − 1) and
and x 2 − y2 = 7 [Q x , y ∈Integer] with radius 0.5
4 AP EAMCET Chapterwise Mathematics

Sol. (b) = (− 2w2)300 + (2w)300


Given inequality is |z − 2 − 2i|≤ 1 {where w is cube root of unity}
Let z = x + iy, then we get = 2300 [(w2)300 + w300 ] = 2300 × 2 = 2301
(x − 2)2 + (y − 2)2 ≤ 1 Hence, option (b) is correct.
⇒ (x − 2) + (y − 2) ≤ 1
2 2
z−i
16. The locus of z satisfying = 2 is a
The above inequality represents a closed circular z − 2i
disc with center at (2, 2) and with radius 1. [21 Sep. 2020, Shift-II]
Hence, option (b) is correct. (a) Hyperbola (b) Circle
(c) Straight line (d) Ellipse
13. If 2 + 4 i is one of the roots of x 2 + bx + c = 0
with b, c ∈R then (b, c) = [21 Sep. 2020, Shift-I] Sol. (b)
(a) (4, − 20) (b) (4, 20) It is given that,
(c) (− 4, − 20) (d) (− 4, 20) z−i
=2
Sol. (d) z − 2i
It is given that 2 + 4i is one of the roots of Let z = x + iy, then
x 2 + bx + c = 0 with b , c ∈ R, so other root will |x + i(y − 1)|= 2|x + (y − 2)i|
be 2 − 4i. ⇒ x 2 + (y − 1)2 = 4[x 2 + (y − 2)2]
Now, the sum of roots = − b ⇒ 3x 2 + 3y 2 − 14 y + 16 = 0 represent a circle, so
⇒ (2 + 4i) + (2 − 4i) = − b ⇒ b = − 4 locus of z is a circle.
and the product of roots = c Hence, option (b) is correct.
⇒ (2 + 4i) (2 − 4i) = c
17. For how many natural numbers ‘n’ such that
⇒ 4 + 16 = c ⇒ c = 20 n
1 + i 
∴ (b , c) = (− 4, 20) 1 ≤ n ≤ 2021 is   =1?
1 − i  [21 Sep. 2020, Shift-II]
Hence, option (d) is correct.
(a) 504 (b) 505 (c) 506 (d) 503
14. If (2 + i) is a root of the equation
Sol. (b)
x 3 − 5 x 2 + 9 x − 5 = 0 , then the other roots
It is given that,
are [21 Sep. 2020, Shift-I] n
 1 + i  = 1 ⇒  1 + i + 2i  = 1
n 2
(a) 1 and (2 − i ) (b) − 1and (3 + i )    
1 − i   1− i
2

(c) 0 and 1 (d) − 1and (− 2 + i ) n
⇒  2i  = 1 ⇒ i n = 1
Sol. (a)  
 2
It is given that 2 + i is the root of the equation
x 3 − 5x 2 + 9 x − 5 = 0, so another non-real If n is a integral multiple of 4, then i n = 1
complex root will be 2 − i. Q1 ≤ n ≤ 2021, so the possible values of n are
4, 8, 12, 16, .... , 2020 and there are 505 values
Now, let the third root is α, so by product of
of n.
roots, we have
Hence, option (b) is correct.
(2 + i) (2 − i) α = 5 ⇒α = 1
Hence, option (a) is correct. 3
18. If x + iy = , then x 2 + y 2 =
2 + cos(θ) + i sin(θ)
15. (− i + 3)300 + (− i − 3)300 =
……… . [21 Sep. 2020, Shift-II]
[21 Sep. 2020, Shift I]
(a) 4 x − 3 (b) 4 x + 3 (c) 0 (d) 1
(a) 2 300
(b) 2 301
(c) 2 100
(d) − 2 300

Sol. (a)
Sol. (b) It is given that,
(− i + 3)300 + (− i − 3)300 = (i − 3)300 3 3(2 + cosθ − i sinθ)
x + iy = =
+ (− i − 3)300 2 + cosθ + i sinθ (2 + cosθ)2 + sin2 θ
= i 300 (1 + i 3)300 + i 300 (− 1 + i 3)300 3 (2 + cosθ) 3sinθ
⇒ x + iy = −i
5 + 4 cosθ 5 + 4 cosθ
Complex Numbers and De-Moivre’s Theorem 5
9
Q x 2 + y 2 = (x + iy)(x − iy) = Sol. (a)
(2 + cosθ)2 + sin2 θ Given complex number
9 10 10
=  2 + i 5  2 − i 5
5 + 4cosθ Z=  +  
12(2 + cosθ)  2 − i 5  2 + i 5
Q 4x − 3 = −3
5 + 4 cosθ Let 2 = r cos θ and 5 = r sin θ ⇒ r = 3
24 + 12cosθ − 15 − 12cosθ 9 10 10
= =  cos θ + i sin θ   cos θ − i sin θ 
5 + 4 cosθ 5 + 4 cosθ Then, Z =   +  
 cos θ − i sin θ   cos θ + i sin θ 
∴ x 2 + y2 = 4 x − 3
= cos(20θ) + i sin (20θ) + cos (20θ) − i sin(20θ)
Hence, option (a) is correct.
= 2 cos 20θ
19. If ω is a complex cube root of unity, then  2 
∴| Z | = 2| cos 20θ | = 2cos  20 cos− 1   
 π   3 
sin (ω10 + ω 23) π −  =
 4 [22 Sep. 2020, Shift-I]
22. Cube roots of unity are the vertices of a/an
1 1 3 ……… , which is inscribed in a circle of unit
(a) (b) (c) 1 (d)
2 2 2 radius, with its centre at origin
Sol. (a) [22 Sep. 2020, Shift-II]
It is given that w is a complex cube root of unity, (a) Right angled triangle (b) Equilateral triangle
so w3 = 1 and w2 + w + 1 = 0. (c) Scalene triangle (d) Isosceles triangle
π π Sol. (b)
So, sin (w10 + w23) π −  = sin (w + w2) π − 
 4  4 We have, cube roots of unity (1, ω, ω2) are the
π π
= − sin π +  = sin =
1 vertices of an equilateral triangle, which is
 4 4 2 inscribed in a circle of unit radius and having
Hence, option (a) is correct. centre at origin.
Hence, option (b) is correct.
20. If “2i’’ is a root of
5i
f (z) = z 4 + z 3 + 2 z 2 + 4 z − 8 = 0, then which 23. Find the conjugate of
7+i
among the following cannot be a root of
f (z) = 0? [22 Sep. 2020, Shift-I] [22 Sep. 2020, Shift-II]
1 1
(a) −2i (b) 1 (c) −2 (d) 2 (a) (1 − 7 i ) (b) (7 i − 1)
10 10
Sol. (d) (c)
1
(1 + 7 i ) (d)
1
(1 − 7 i )
It is given that, f (z) = z 4 + z 3 + 2 z 2 + 4z − 8 10 50
have a root 2i, so one more root will be − 2i, so Sol. (a)
(z 2 + 4) is the factor of z 4 + z 3 + 2 z 2 + 4z − 8. 5i 5i 7−i
= ×
So, z 4 + z 3 + 2z 2 + 4z − 8 7+ i 7+ i 7−i
= (z 2 + 4) (z 2 + z − 2 ) 5 i (7 − i) 5(7i − i 2)
= = {Q i 2 = −1}
and z + z − 2 = (z + 2) (z − 1)
2 (7)2 − ()i2 50
Therefore, the roots of f (z) are 2i, − 2i, − 2 and 1. 5i 1 + 7i
=
7+i 10
21. The modulus of the complex number 5i 1 − 7i
10 10 ∴Conjugate of is
2 + i 5 2 − i 5 7+ i 10
  +  is
2− i 5 2 + i 5 [22 Sep 2020, Shift I] Hence, option (a) is correct.
 2   2  z − 25
(a) 2 cos 20cos −1    (b) 2 sin  10cos −1    24. If  = 5, then|z|=
  3    3 
 z − 1 [22 Sep. 2020, Shift-II]
 2   2 
(c) 2 cos  10 cos −1    (d) 2 sin  20 cos −1    (a) 5 (b) 3 (c) 4 (d) 10
  3    3 
6 AP EAMCET Chapterwise Mathematics

Sol. (a) 26. Suppose Z ∈C has argument θ such that


z − 25 π
=5 0 <θ< and satisfy the equation|z − 3 i| = 3.
z −1 2
Let z = x + iy 6
Then what is the value of cotθ − ?
x + iy − 25 z
=5
x + iy − 1 [23 Sep. 2020, Shift-I]
( x − 25) + iy (a) 2i (b) i (c) −i (d) −2i
=5
(x −1) + iy Sol. (b)
It given for a complex number z,|z − 3i| = 3
(x − 25)2 + y 2
=5 Q arg(z) = θ, so ∠OAB = θ
(x − 1)2 + y 2 Q OB = OAsinθ
Squaring on both sides, ⇒ z = 6 (sinθ) e i θ
(x − 25)2 + y 2 = 25 [(x − 1)2 + y 2]
Imaginary axis
⇒ x 2 + 625 − 50 x + y 2 = 25 [x 2 + 1 − 2x + y 2]
A (0, 6i)
⇒ x 2 + y 2 − 50 x + 625 = 25x 2 + 25y 2 − 50 x + 25
θ
⇒ 24 x 2 + 24 y 2 = 600
(0, 3i)
⇒ x 2 + y 2 = 25 B (z)
∴ |z| = x 2 + y2 = 25 θ
O Real axis
|z| = 5
Hence, option (a) is correct. 6 1
⇒ =
20 z sinθ(cosθ + i sinθ)
1 + cos(3θ) + i sin (3θ)
25.   =? cosθ − i sinθ
 1 + cos(3θ) − i sin(3θ)  ⇒ = = cot θ − i
[23 Sep. 2020, Shift-I] sinθ
(a) cos(60 θ) + i sin (60 θ) (b) cos(60 θ) − i sin (60 θ) 6
⇒ cotθ − = i
(c) cos(20 θ) + i sin (20 θ) (d) cos(20 θ) − i sin (20 θ) z
Sol. (a) 27. Solve ix 2 − 3 x − 2i = 0 [23 Sep. 2020, Shift I]
20
 1 + cos 3θ + i sin 3θ  (a) −i only (b) −2i only
 
 1 + cos 3θ − i sin 3θ  (c) −i and −2i (d) i only

 3θ 3θ 3θ 
20 Sol. (c)
2cos2   + i  2sin ⋅ cos 
  2  2 2 Given, ix 2 − 3x − 2i = 0
=
3θ 
 2cos2  3θ  − i  2 sin ⋅ cos  
3θ ⇒ − ix 2 + 3x + 2i = 0
  2  2 2 Multiply by i on both sides
20
3θ 3θ 3θ x 2 + 3xi + 2i 2 = 0
2cos   cos + i sin 
 2   2 2 
= 20
x 2 + 3ix − 2 = 0
3θ  3θ 3θ 
2cos cos − i sin  −3i ± (3i)2 − 4 ⋅1 ⋅ (−2)
2  2  2   x=

21
20
 i  3θ   20 − 3i ± − 9 + 8 − 3i ± i − 3i − i
 i  3θ  3θ
+ i    = = (or)
 e  2 
=  = e  2   2
 2 2 2
 3θ    −3i + i
 − i  2     =
e  2
= [e i( 3θ) ]20 = e i60θ = cos 60θ + i sin 60θ x = − i (or) −2i
Hence, option (a) is correct. Hence, option (c) is correct.
Complex Numbers and De-Moivre’s Theorem 7

28. If z = x + iy , x , y ∈ R, (x , y) ≠ (0 , − 4) and Arg 30. If ω represents a complex cube root of unity,


 2z − 3 π  1  1  1  1 
  = , then the locus of z is then 1 +  1 + 2  + 2 +  2 + 2 
 z + 4 i 4  ω  ω   ω  ω 
[20 April 2019, Shift-I]  1  1
(a) 2 x2 + 2 y2 + 5 x + 5 y − 12 = 0 + … + n +  n + 2 =
 ω  ω 
(b) 2 x2 − 3 xy + y2 + 5 x + y − 12 = 0
[20 April 2019, Shift-I]
(c) 2 x2 + 3 xy + y2 + 5 x + y + 12 = 0
n(n2 + 1) n(n2 + 2 )
(d) 2 x2 + 2 y2 − 11x + 7 y − 12 = 0 (a) (b)
3 3
Sol. (a) n(n2 − 2 ) n2 (n − 1)
(c) (d)
For z = x + iy, x , y ∈ R, (x , y) ≠ (0, − 4) 3 6
2z − 3 (2x − 3) + 2iy x − i(y + 4) Sol. (b)
= ×
z + 4i x + i(y + 4) x − i(y + 4) As ω is complex cube root of unity, then
(2x 2 − 3x + 2y 2 + 8 y) + i (2xy − 2xy + 3y − 8 x + 12) 1 + ω + ω2 = 0 and ω3 = 1 …(i)
=
x 2 + (y + 4)2
Q  r +   r + 2  = r 2 + r  + 2  + 3
1 1 1 1 1
(2x 2 − 3x + 2y 2 + 8 y) + i (12 + 3y − 8 x)  ω  ω  ω ω  ω
=
x 2 + (y + 4)2  ω2 + ω  1
= r2 +   r + 3 = r − r + 1 [from Eq. (i)]
2

−1  12 + 3y − 8 x  π  ω  ω
3
2z − 3
So, arg   = tan  2  =
 z + 4i   2x − 3x + 2y + 8 y  4
2
Now, ∑  r +   r + 2 
n 1 1
(given) r =1  ω  ω 
12 + 3y − 8 x n n(n + 1)(2n + 1) n(n + 1)
⇒ =1 = ∑ (r 2 − r + 1) = − + n
2x 2 − 3x + 2y 2 + 8 y r =1 6 2
n(n + 1) n(n + 1)
⇒ 2x 2 + 2y 2 + 5x + 5y − 12 = 0 = [2n + 1 − 3] + n = (2n − 2) + n
6 6
Hence, option (a) is correct.
n n n(n2 + 2)
= (n2 − 1) + n = [n2 − 1 + 3] =
29. If z = x + iy , x , y ∈ R and the imaginary part 3 3 3
z −1 Hence, option (b) is correct.
of is 1, then the locus of z is
z−i 31. If ω is a complex cube root of unity, then
[20 April 2019, Shift-I]
9
(a) x + y + 1 = 0 ∑ r(r + 1 − ω) (r + 1 − ω 2) =
(b) x + y + 1 = 0, ( x, y) ≠ (0, − 1) r =1 [20 April 2019. Shift-I]
(c) x2 + y2 − x + 3 y + 2 = 0 (a) 5025 (b) 4020
(d) x2 + y2 − x + 3 y + 2 = 0, ( x, y) ≠ (0, − 1) (c) 2016 (d) 3015

Sol. (d) Sol. (d)


If z = x + iy, then As, ω is complex cube root of unity, then
z − 1 x − iy − 1 x + i(y + 1) 1 + ω + ω2 = 0
= ×
z−i x − iy − i x + i(y + 1) and ω3 = 1 …(i)
[x(x − 1) + y(y + 1)] + i[(y + 1)(x − 1) − xy] Q r(r + 1 − ω)(r + 1 − ω ) 2
=
x 2 + (y + 1)2
= r [(r + 1)2 − (ω + ω2)(r + 1) + ω3]
 z − 1  xy − y + x − 1 − xy
∴Im   = =1 (given) = r [(r + 1)2 + (r + 1) + 1] [from Eq. (i)]
 z − i x 2 + (y + 1)2
= r(r + 3r + 3) = r + 3r + 3r
2 3 2

⇒ x 2 + y 2 − x + 3y + 2 = 0, (x , y) ≠ (0, − 1) 9 9
∴ ∑ r(r + 1 − ω)(r + 1 − ω2) = ∑ (r 3 + 3r 2 + 3r)
Hence, option (d) is correct. r =1 r =1
8 AP EAMCET Chapterwise Mathematics

2
9 × 10 Sol. (a)
= 
9 (10)  9 (10) (19)
 + 3 + 3
 2  6 2 As it is given, |z1|=|z2|= 1,
= (45)2 + (45 × 19) + (3 × 45) = 45[45 + 19 + 3] so let z1 = a + ib = cosα + i sinα.
and z2 = c + id = cosβ + i sinβ.
= 45 × 67 = 3015
Hence, option (d) is correct. Now, Re(z1 z2) = ac + bd = cosα cosβ + sinα sinβ
= cos(α − β) = 0 [given]
32. z1 , z 2 are two complex numbers with π π
So, α − β = or −
|z1 − z 2| < k. If the complex number z satisfies 2 2
the condition|z − z1| + |z − z 2| = k, then z lies Now, for the pair of complex numbers
on [20 April 2019, Shift-II] w1 = a + ic = cosα + i cosβ
(a) a parabola (b) an ellipse and w2 = b + id = sinα + i sinβ.
(c) a circle (d) a hyperbola The Re(w1 w2) = ab + cd = cosα sinα + cosβ sinβ
1
Sol. (b) = [sin 2α + sin 2β]
2
According to the given information, 1
|z − z1|+ |z − z2| = K > |z1 − z2| = × 2sin(α + β) cos(α − β) = 0
2
⇒ Sum of the distances of the point z from z1 Q α − β = π or − π 
and z2 is constant (K) and more than the  2 2 
distance between points z1 and z2. Hence, option (a) is correct.
So, locus of point ‘z’ is an ellipse.
35. If (1 + x)n = a 0 + a1 x + a 2 x 2 + … + a n x n and
Hence, option (b) is correct.

33. In the complex plane C, the set a 0 − a 2 + a 4 − a 6 + …= k cos , then k =
4
  z − 1 π 
 z ∈ C : arg   =  represents [20 April 2019, Shift-II]
  z + 1 4  2n
n
n 2n
[20 April 2019, Shift-II] (a) 2 (b) 2 (c) (d) 2 2
2
(a) a straight line (b) a circle
(c) a parabola (d) an ellipse Sol. (d)
Sol. (b) If (1 + x) n = a 0 + a1 x + a 2 x 2 + a 3 x 3 + … + a n x n
Let a complex number z = x + iy, Put x = i, we get
z − 1 (x − 1) + iy (x + 1) − iy (1 + i)n = (a 0 − a 2 + a 4 − a 6 + …)
then = ×
z + 1 (x + 1) + iy (x + 1) − iy + i (a1 − a 3 + a 5 − a 7 + …)
n
(x 2 + y 2 − 1) + iy(x + 1 − x + 1)   π π 
= ⇒ 2 cos + i sin  = (a 0 − a 2 + a 4 − a 6 + …)
(x + 1)2 + y 2   4 4  
(x 2 + y 2 − 1) 2y + i (a1 − a 3 + a 5 − a 7 + …)
= + i
(x + 1)2 + y 2 (x + 1)2 + y 2 On comparing the real and imaginary parts, we
get
According to the given information, nπ
2 n/ 2 cos = a0 − a2 + a4 − a6 + …
 z − 1 π 2y 4
arg   = ⇒ 2 =1

 z + 1  4 (x + y 2 − 1) and 2n/ 2 sin = a1 − a 3 + a 5 − a 7 + …
⇒ x 2 + y 2 − 2y − 1 = 0 4
n
∴The complex plane C represents a circle. nπ
Q a 0 − a 2 + a 4 − a 6 + … = k cos ⇒ k = 22
Hence, option (b) is correct. 4
Hence, option (d) is correct.
34. For a , b, c , d ∈ R , if z1 = a + ib, z 2 = c + id are
 x y
such that|z1| = |z 2| = 1 and Re (z1 z 2) = 0 , then 1  + 
the pair of complex numbers w1 = a + ic and  a b
36. If z = x − iy and z3 = a + ib, then =
w 2 = b + id satisfy [20 April 2019, Shift-II] a 2 + b2
(a) Re(w1 w2 ) = 0 (b) Re(w1 w2 ) = 1 [21 April 2019, Shift I]
(c)|w1| ≠ |w2| (d)|w1| = |w2| = 0 (a) − 2 (b) − 1 (c) 1 (d) 2
Complex Numbers and De-Moivre’s Theorem 9

Sol. (a) On comparing both sides, we get


1 ⇒ x2 + y = − 3 …(i)
Given, z = x − iy and z3 = a + ib
and 4=− x y
2

z1 / 3 = a + ib 4 4
⇒ y= =− 2 …(ii)
Take cube on both sides, we get − x2 x
(z1 / 3)3 = (a + ib)3 On puting the value of y in Eq. (i), we get
⇒ z = a 3 + (ib)3 + 3a 2ib + 3a(ib)2 4
∴ x 2 − 2 = − 3 ⇒ x 4 − 4 = − 3x 2
⇒ z = a 3 + i 3b 3 + 3a 2ib + 3ab 2i 2 x
⇒ z = a 3 − ib 3 + 3a 2bi − 3ab 2 [Q i 2 = − 1] ⇒ x 4 + 3x 2 − 4 = 0

⇒ z = (a − 3ab ) − i(b − 3a b)
3 2 3 2 ⇒ (x 2 + 4) (x 2 − 1) = 0

⇒ x − iy = (a 3 − 3ab 2) − i(b 3 − 3a 2b) [Q z = x − iy] ⇒ x2 + 4 = 0

Here, x = a 3 − 3ab 2 and y = b 3 − 3a 2b ⇒ x =−4


2
(not possible)
x −1 = 0 ⇒ x =1 ⇒ x = ± 1
2 2
 x + y   a − 3ab + b − 3a b 
3 2 3 2

  Put the value of x = ± 1 in Eq. (ii), we get


   a b 
Now, a2 b2 = −4
a + b a2 + b2 y= 2 =−4
()
1
a 2 − 3b 2 + b 2 − 3a 2
= ∴ x = ± 1, y = − 4
a2 + b2
Hence, (| x | + | y |)2
− 2a 2 − 2b 2 − 2(a 2 + b 2)
= = =−2 = | x |2 + | y |2 + 2| x || y |
a2 + b2 a2 + b2
= ()
1 2 + |(− 4) |2 + 2|1 || − 4 |
37. The equation whose solutions are the = 1 + 16 + 8 = 25
non-zero solutions of the equation z = iz 2 , is 6
 2 πk 2 πk 
[21 April 2019, Shift-I] 39. ∑ sin 7
− i cos
7 
 =
(a) z + i = 0
3
(b) z + z + 1 = 0
3 k=1 [21 April 2019, Shift-I]
(c) z3 − i = 0 (d) z3 + iz + 1 = 0 (a) − 1 (b) 0 (c) i (d) − i
Sol. (a) Sol. (c)
2πk 2πk 
∑  sin
6
z = iz 2 − i cos 
k =1  7 7 
⇒ z = − iz 2 ⇒ z = − i[i z 2]2
 2πk  1  2πk  
= − i ∑ cos
6
− sin
⇒ z = − i i 2z 4 ⇒ z = iz 4   7  i  7  
k =1 
1
⇒ z4 = z ⇒ z 4 = − iz  2πk   2πk  
= − i ∑ cos
6
i  + i sin 
k =1 
  7   7  
⇒ z 4 + iz = 0 ⇒ z(z 3 + i) = 0
 6 i 2πk 
⇒ z3 + i = 0 (Q z ≠ 0) = − i  ∑ e 7  …(i) {Q e iθ = cosθ + i sinθ}
 k =1 
38. If x , y ∈ R and x 2 + y + 4 i and − 3 + x 2 yi are
Now,
conjugates to each other, then (| x | + | y |)2 =  i 2π i4π i 6π i8π i10 π i12π

[21 April 2019, Shift-I] 1 + e 7 + e 7 + e 7 + e 7 + e 7 + e 7 = 0



(a) 17 (b) 16
i2π i4π i12π
(c) 25 (d) 9
{Q z 7 = 1 then, roots 1, e 7 , e 7 ... e 7 }
Sol. (c) 6 6
x 2 + y + 4i and − 3 + x 2 yi are conjugate. = 1 + ∑ e i( 2πk / 7) = 0 = ∑ e i( 2πk / 7) = − 1 … (ii)
k =1 k =1
Therefore, x 2 + y + 4i = − 3 − x 2 yi From Eqs. (i) and (ii), we get
⇒ (x 2 + y) + 4i = (− 3) − x 2 yi = − i(− 1) = i
10 AP EAMCET Chapterwise Mathematics

40. If a , b are the least and the greatest values Sol. (d)
respectively of|z1 + z 2|, where z1 = 12 + 5 i and If z = x + iy, x , y ∈ R.
2z + i 2(x + iy) + i 2x + i(2y + 1)
|z 2| = 9 , then a 2 + b2 = [21 April 2019, Shift-II] then = =
z−2 (x + iy) − 2 (x − 2) + iy
(a) 468 (b) 500 (c) 250 (d) 450
2x + i(2y + 1) (x − 2) − iy
= ×
Sol. (b) (x − 2) + iy (x − 2) − iy
Given, z1 = 12 + 5i  2z + i 
∴ |z1| = 122 + 52 = 144 + 25 = 169 = 13 and Q Re  =0
 z − 2
|z2| = 9
⇒ 2x(x − 2) + y(2y + 1) = 0
Now, greatest value of |z1 + z2| = 13 + 9 = 22 = a
⇒ 2x 2 + 2y 2 − 4 x + y = 0
and least value of |z1 + z2| = 13 − 9 = 4 = b
y
Hence, a 2 + b 2 = (22)2 + (4)2 = 484 + 16 = 500 ⇒ x 2 + y 2 − 2x + = 0 …(i)
2
41. If a complex number z is such that z + i x + i(y + 1) (x + 1) − iy
and = ×
(7 + i)(z + z) − (4 + i)(z − z) + 116 i = 0 , then z + 1 (x + 1) + iy (x + 1) − iy
z⋅z = [21 April 2019, Shift-II]  z + i
∴ arg 
(a) 400 (b) 300 (c) 200 (d) 100  z + 1
Sol. (c)  (x + 1)(y + 1) − xy  π
= tan−1  =
Let z = x + iy  x(x + 1) + y(y + 1)  2
Then, z = x − iy
⇒ x 2 + y2 + x + y = 0 …(ii)
Now, (7 + i)(z + z) − (4 + i)(z − z) + 116i = 0
⇒ (7 + i)(2x) − (4 + i)(2iy) + 116i = 0 Equation of common chord of circles (i) and (ii)
⇒ 14 x + 2ix − (8iy − 2y) + 116i = 0
is S1 − S2 = 0
y
⇒ (14 x + 2y) + i(2x − 8 y + 116) = 0 ⇒ −3x − = 0
2
∴ 14 x + 2y = 0 or 2x − 8 y + 116 = 0
⇒ y + 6x = 0 …(iii)
⇒ y = − 7x
For the intersection of the circles on solving
∴ 2x − 8(−7 x) + 116 = 0
chord (iii) and circle (ii), we get
⇒ 58 x = − 116 ⇒ x = − 2 and y = 14
x 2 + 36 x 2 + x − 6 x = 0
∴ |z| = x 2 + y 2 = (−2)2 + (14)2 5
⇒ 37 x 2 − 5x = 0 ⇒ x = 0,
= 4 + 196 = 200 37
There, zz = |z| = ( 200) = 200
2 2 30
So, y-coordinate is 0, − .
37
42. Let the point P represent z = x + iy, x , y ∈ R ∴The point of intersection of the curves C1 and
C2, other than the origin is  , −  .
in the argand plane. Let the curves C1 and C 2 5 30
be the loci of P satisfying the conditions  37 37 
2z + i 20
(i) is purely imaginary and 43. If z = cos 6 ° + i sin 6 °, then ∑ Im(z 2n − 1) =
z−2 n =1
 z + i π [21 April 2019, Shift-II]
(ii) Arg   = respectively. Then the −3 3
 z + 1 2 (a) 0 (b) −1 (c) (d)
4sin 6° 4sin 6°
point of intersection of the curves C1 and
C 2 , other than the origin, is Sol. (d)
20
[21 April 2019, Shift-II] Q ∑ z 2n − 1 = z + z 3 + z 5 + … + z 39
n =1
(b)  , − 
2 5
(a) (1, 2)
7 7 Now, z = cos 6° + i sin 6° = e i 6°

(d)  , −
5 30  20
(c) (−3, 4)  ∴ ∑ z 2n − 1 = e i 6° + e i18 ° + e i 30 ° + … + e i( 39 × 6°)
 37 37  n =1
Complex Numbers and De-Moivre’s Theorem 11

= e i 6° [1 + e i12 + e i 24 + … + e i 228 ] 2 2
 x − 1  +  y − 3 =  10 
2
⇒      
 1 3   2  2  2 
e i 6 − − i − 1
 e i 240 − 1   Which is a circle with centre  ,  and
2 2  1 3
= e i 6  i12 =  2 2
 e −1  e i 6(2i sin 6°)
10
3 3 radius .
− − i 2
2 2 3 3
= =− + i 1 + 3i 10
2isin 6° 4sin 6° 4sin 6° ∴z ∈ C : (3 − i)z + (3 + i)z − 6 > 0, z − =
20 2 2
3
∴ ∑ Im(z 2n − 1) =
n =1 4sin 6° 45. If P , Q and R are points, respectively

44. If z = x + iy , x , y ∈ R and if the point P in the
representing the complex numbers z , ze 3
argand plane represents z, then the locus of
 iπ 
 z −1  π
P satisfying the condition arg   = , is and z 1 + e 3  in argand plane, then the area
 z − 3i  2  
 
[22 April 2019, Shift-I] of the triangle PQR, is [22 April 2019, Shift-I]
 1 + 3i 10 
(a)  z ∈ C /z − =  (a) 3| z |2 (b)
3 2
| z| (c)
3 2
| z| (d) 2 3| z|2
  2  2  2 4
(b) { z ∈ C / (3 − i )z + (3 + i )z − 6 = 0} Sol. (c)
(c) { z ∈ C / ( 3 − i ) z + ( 3 + i ) z − 6 > 0, Given that,
z − 1 + 3i = 10  PQ = |ze iπ / 3 − z| = |z||e iπ / 3 − 1|
 2  2 
π π
= |z|cos + i sin −1
(d) { z ∈ C / ( 3 − i ) z + ( 3 + i ) z − 6 < 0,  3 3 
z − 1 + 3i = 10  2 π π π
 
= |z| −2sin + 2i sin cos 
 2  2   6 6 6
Sol. (c)  π  π π 1
=|z| 2sin sin − i cos = |z|⋅|2 × ||1|
 z −1  π  6  6 6 2
We have, arg   = PQ = |z|
 z − 3i  2
π Now, QR = [z(1 + e iπ / 3) − 2e iπ / 3]
⇒ arg (z − 1) − arg (z − 3i) =
2 QR = |z|
π Similarly, PR = |z|
⇒ arg[(x − 1) + iy] − arg [x + (y − 3)i] =
2 So, PQ = QR = PR
y y −3 π ∴∆PQR is equilateral triangle with side length
⇒ tan−1 − tan−1 =
x −1 x 2 ≥ 1.
− 3 2
 y

y 3  Now, area of ∆PQR = |z|
−1
 x −1 x  π 4
⇒ tan  =
y y − 3 2 46. A(z1) and B(z 2) are two points in the argand
1 + . 
 x −1 x  plane. Then, the locus of the complex
xy − (x −1)(x − 3) π  z − z1 
⇒ = tan
x(x −1) + y(x − 3) 2 number z satisfying arg   = 0 or π, is
 z − z2 
xy − (x − 1)(y − 3) 1
⇒ = [22 April 2019, Shift-I]
x(x − 1) + y(y − 3) 0 (a) the circle with AB as a diameter
⇒ x(x −1) + y(y − 3) = 0 (b) the ellipse with A, B as extremities of the major
axis
⇒ x 2 + y 2 − x − 3y = 0
2 2
(c) the perpendicular bisector of AB
⇒  x − 1  +  y − 3 = 1 + 9 (d) the straight line passing through the points
   
 2  2 4 4 A and B
12 AP EAMCET Chapterwise Mathematics

Sol. (d) Sol. (*)


Let the point z(x , y), z1 (x1 , y1) and z2(x 2 , y2). Given,
∴ z − z1 = x + iy − x1 − iy1 = x − x1 + i(y − y1) z−
6
=5
and z − z2 = x + iy − x 2 − iy2 = x − x 2 + i(y − y2) z
z − z1 6
Now, Now, − 5≤ |z |− ≤5
z − z2 |z|
[(x − x1) + i(y − y1)] [(x − x 2) − i(y − y2)] ⇒ − 5| z |≤ | z |2 − 6
= ×
[(x − x 2) + i(y − y2)] [(x − x 2) − i(y − y2)]
⇒ | z |2 − 6 + 5| z |≥ 0
(x − x1)(x − x 2) + (y − y1)(y − y2) +
⇒ |z | + 6|z |− |z |− 6 ≥ 0
2
i[(x − x 2)(y − y1) − (x − x1)(y − y2)]
= ⇒ | z |(| z | + 6) − 1(| z | + 6) ≥ 0
(x − x 2)2 + (y − y2)2
⇒ (| z | − 1) (| z | + 6) ≥ 0
 z − z1 
= Arg   = 0 or π ⇒ | z |≥ 1 (Q| z | ≠ − 6)
 z − z2  and |z|2 −6 ≤ 5|z|⇒ |z|2 −5|z|− 6 ≤ 0
 [(x − x 2)(y − y1)] − [(x − x1)(y − y2)] ⇒ (|z|− 6)(|z| + 1) ≤ 0
⇒  =0
 [(x − x1)(x − x 2) + (y − y1)(y − y2)]  ⇒ |z|≤ 6
⇒ (x − x 2)(y − y1) = (x − x1)(y − y2) So, 1 ≤ |z|≤ 6
⇒ xy − xy1 − x 2 y + x 2 y1 = xy − xy2 − x1 y + x1 y2 So, maximum value of (z) = 6
⇒ x(y2 − y1) + y(x1 − x 2) + (x 2 y1 − x1 y2) = 0
It represents a straight line passing through the 49. If a = 3 + 4 i, z1 and z 2 are two complex
points A of B. numbers such that| z1 | = 3 and| z 2 − a | = 2,
47. If x is a cube root of unity other than 1, then then the maximum value of| z1 − z 2 | is
[22 April 2019, Shift-II]
2 2 2
 1  2 1  12 1  (a) 5 (b) 10 (c) 15 (d) 20
 x +  +  x + 2  + ... +  x + 12  =
 x  x   x  Sol. (b)
[22 April 2019, Shift-I] Given,
(a) 12 (b) 64 (c) 24 (d) 0 | z1 | = 3, represents a circle centred at 0 and
Sol. (c) radius 3.
Given, x is a cube root of unity other than 1 | z2 − 3 − 4i| = 2, represents a circle centred at
i.e. x =ω or ω2 (3, 4) and radius = 2. [Q a = 3 + 4i]
2 2 2 This circles passes through origin. Distance of
Now,  x +  +  x 2 + 1  + K +  x12 + 1 
1
    diametrically opposite end is 10.
 x  x2   x12 
2 2 2 So, maximum value of | z1 − z2 | is 10.
=  ω +  +  ω2
 1
+ 2  + K +  ω12 + 12 
1 1
 ω  ω   ω  50. If α is the real root and β, γ are the complex
2 2 2
=  ω +  +  ω2 + 2  + K +  ω11 + 11  roots of the equation x 3 + 3 x 2 + 3 x + 28 = 0 ,
1 1 1
 ω  ω   ω  then 2α + 3β + 3 γ = [22 April 2019, Shift-II]
= (ω + ω2)2 + (ω2 + ω)2 + (1 + 1)2 + (ω + ω2)2
(a) − 5 (b) 0 (c) 5 (d) − 23
+ (ω2 + ω)2 + (1 + 1)2 + (ω + ω2)2
Sol. (a)
= 8(ω + ω2)2 + 41 ( + 1)2
Given equation is
= 8(−1) + 4(2) = 8 + 16 = 24
2 2
x 3 + 3x 2 + 3x + 28 = 0 …(i)
48. If z is a complex number such that ⇒ x + 3x + 3x + 1 + 27 = 0
3 2

6 ⇒ (x + 1)3 + 27 = 0
z− = 5, then the maximum value of| z |
z ⇒ (x + 1)3 + 33 = 0
is [22 April 2019, Shift-II]
⇒(x + 1 + 3) {(x + 1)2 − 3(x + 1) + 32} = 0
3
(a) 3 (b) 2 (c) 1 (d)
2 ⇒ (x + 4) (x 2 + 1 + 2x − 3x − 3 + 9) = 0
Complex Numbers and De-Moivre’s Theorem 13

⇒ (x + 4) (x 2 − x + 7) = 0 53. If 1, α1 , α 2 ,...., α n − 1 are the nth roots of


∴ x=−4 ⇒α=−4 unity and n is an even natural number, then
(Q x 2 − x + 7 ≠ 0 real roots) (1 + α1) (1 + α 2) .... (1 + α n − 1) =
Given that, α, β and γ are roots of Eq. (i) [23 April 2019, Shift-I]
α+β+ γ=−3 (a) 1 (b) −1
⇒ −4+β+ γ =−3 (c) 0 (d) 2
⇒ β + γ =1 Sol. (c)
Now, 2α + 3β + 3γ = 2(− 4) + 3(β + γ) Since,1, α1 , α 2 ..........., α n −1 are the nth roots of
= − 8 + 3= − 5 unity, then x n − 1 = (x − 1) (x − α1) (x − α 2) ........
51. For a complex number Z = a + ib, let Z$ (x − α n −1)
xn − 1
= b + ia. If Z1 , Z 2 are such complex numbers, ⇒ (x − α1) (x − α 2) ....... (x − α n −1) =
x −1
then Z1Z 2 = [23 April 2019, Shift-I]
On putting x = − 1, we get
$ Z
(a) Z $ $ Z$
(b) Z
1 2 1 2 (−1)n − 1
$ $ Z (− 1 − α1) (− 1 − α 2) ......... (− 1 − α n − 1) =
(c) Z Z
1 2 (d) Z1 2 −1 − 1
Sol. (c) (1 + α1) (1 + α 2) ......... (1 + α n −1) = 0
For given complex number Z = a + ib, it is given [as n is an even natural number]
that Z$ = b + ia Hence, option (c) is correct.
Let Z1 = a + ib and Z2 = c + id 1 1
54. If x + = 2 sin α and y + = 2 cos β, then
Then, Z1 Z2 = (ac − bd) + i (ad + cb) x y
⇒ Z1 Z2 = (ad + cb) + i(ac − bd) 1
x y + 3 3 =
3 3

Q Z1 Z$ 2 = (a + ib) (d + ci) = (a − ib) (d + ic) x y [23 April 2019, Shift-I]


= (ad + cb) + i(ac − bd) (a) 2 cos 3 (β − α ) (b) 2 cos 3 (β + α )
(c) 2 sin 3 (β − α ) (d) 2 sin 3 (β + α )
∴ Z1 Z2 = Z1 Z$ 2
Hence, option (c) is correct. Sol. (c)
It is given that
52. The points in the argand plane represented 1
x + = 2sin α ⇒ x = sin α + i cosα
by the complex conjugates of x
1 + 2i , 2 − 3i , 3 − 4 i [23 April 2019, Shift-I] π π
= cos  − α  + i sin  − α 
(a) are collinear 2  2 
(b) form an equilateral triangle π
i  − α 
(c) form an obtuse angled triangle ⇒ x=e2 
(d) form an acute angled triangle
1
and y + = 2cosβ
Sol. (c) y
The complex conjugate of given points are in ⇒ y = cos β + i sinβ = e iβ
argand plane A(1 − 2i), B(2 + 3i), C(3 + 4i)
3π 3π
So, a = BC = 1 + 1 = 2 i  − 3α  i  + ( 3β − 3α) 
Now, x 3 y 3 = e  2 
⋅ e i 3β = e  2 
b = CA = 4 + 36 = 40
3π 3π
c = AB = 1 + 25 = 26 = cos  + 3β − 3α  + i sin  + 3β − 3α 
 2   2 
a 2 + c 2 − b 2 2 + 26 − 40
Q cos B = = = sin(3β − 3α) − i cos (3β − 3α)
2ac 2 2 26 1
12 3 and = sin (3β − 3α) + i cos (3β − 3α)
=− =− <0 x 3 y3
4 13 13
1
So, point A, B and C represents the vertices of an So, x 3 y 3 + = 2 sin (3β − 3α) = 2sin 3 (β − α)
obtuse angled triangle. x 3 y3
Hence, option (c) is correct. Hence, option (c) is correct.
14 AP EAMCET Chapterwise Mathematics

55. Let z = x + iy and a point P represent z in the 57. If z1 = 1 − 2i; z 2 = 1 + i and z 3 = 3 + 4 i, then
z −1  1 3  z3
Argand plane. If the real part of is 1,  +  =
z+i  z1 z 2  z 2 [22 April 2018, Shift-I]
then a point that lies on the locus of P is 13 13
[22 April 2018, Shift-I] (a) 13 − 6 i (b) 13 − 3 i (c) 6 − i (d) − 3i
2 2
(a) (2016, 2017)
(b) (− 2016, 2017) Sol. (d)
(c) (−2016, −2017) We have,
(d) (2016, −2017) z1 = 1 − 2i, z2 = 1 + i,
Sol. (d) z3 = 3 + 4i
z −1 1 3  z3  1 3   3 + 4i 
We have, Now,  +  = +   
z+ i  z1 z2  z2  1 − 2i 1 + i   1 + i 
x + iy − 1 (x − 1) + iy (1 + i) + 31 ( − 2i) 3 + 4i 4 − 5i 3 + 4i
= = = × = ×
x + iy + i x + (y + 1)i (1 − 2i) (1 + i) 1+ i 3− i 1+ i
(x − 1) + iy x − (y + 1)i 12 + 16i − 15i + 20 32 + i
= × = =
x + (y + 1)i x − (y + 1)i 3 + 3i − i + 1 4 + 2i
x(x − 1) + ixy − (x − 1) (y + 1)i + y(y + 1) (32 + i) (2 − i) 64 + 2i − 32i + 1
= = =
x 2 + (y + 1)2 2(2 + i) (2 − i) 2(4 + 1)
x(x − 1) + y(y + 1) [xy − (x − 1)(y + 1)]i 65 − 30i 13
= + = = − 3i
x 2 + (y + 1)2 x 2 + (y + 1)2 10 2
 z − 1 58. If 1, ω , ω 2 are the cube roots of unity, then
Since, Re  =1
 z + i 1 1 1
+ − =
∴ x(x − 1) + y(y + 1) = x + (y + 1) 2 2 1 + 2ω 2 + ω 1 + ω [22 April 2018, Shift-I]
⇒ x − x + y + y = x + y + 2y + 1
2 2 2 2 (a) 1 (b) ω (c) ω2 (d) 0
⇒ − x + y = 2y + 1 Sol. (d)
⇒ x + y+1= 0 Given,
1
+
1

1
∴(2016, − 2017) lies on 1 + 2ω 2 + ω 1 + ω
x + y+1= 0 2 + ω + 1 + 2ω 1
= −
−1
5 (1 + 2ω) (2 + ω) (1 + ω)
56. If 13e i tan
12
= a + ib, then the ordered pair 3 + 3ω 1
= −
(a , b) = [22 April 2018, Shift-I] (1 + 2 ω) (2 + ω) (1 + ω)
(3 + 3ω) (1 + ω) − (1 + 2ω) (2 + ω)
(a) (12, 5) (b) (5, 12) (c) (24, 10) (d) (10, 24) =
(1 + 2ω) (2 + ω) (1 + ω)
Sol. (a)
3 + 3 ω + 3 ω + 3 ω2 − (2 + ω + 4 ω + 2 ω2)
We have, =
i tan − 1
5 (1 + 2 ω) (2 + ω) (1 + ω)
13e 12 = a + ib 3 + 6 ω + 3 ω2 − 2 − w − 4 ω − 2 ω2
=
 5  (1 + 2 ω)(2 + ω )(1 + ω)
⇒ 13, cos tan− 1  + i sin tan− 1  = a + ib
5
  12  12  1 + ω + ω2
=
 5  (1 + 2 ω) (2 + ω) (1 + ω)
⇒ 13 cos cos− 1  + i sin sin− 1  = a + ib
12
  13  13  =0 [Q 1 + ω + ω2 = 0]
13 + i  = a + ib
12 5
⇒ π  π
13 13 59. The amplitude of sin + i 1 − cos  is
5  5
⇒ 12 + 5i = a + ib
∴ a = 12, b = 5 [22 April 2018, Shift-II]
∴ (a , b) = (12, 5) π π π 2π
(a) (b) (c) (d)
15 10 5 5
Complex Numbers and De-Moivre’s Theorem 15
15
Sol. (b)  73 + 87ω + 51ω2 ω 87 + 51ω + 73ω2 ω2 
= × 3+ × 3
Given complex number is,
 73 + 87ω + 51ω ω 87 + 51ω + 73ω ω 
2 2
π π
= sin + i 1 − cos  = (ω + ω2 )15 = (−1)15 = − 1
5  5
= 2 sin
π
cos
π
+ i  2sin2 
π 62. If z ∈ C and iz 3 + 4 z 2 − z + 4 i = 0, then a
10 10  10  complex root of this equation having
π  cos π + i sin π  minimum magnitude is [22 April 2018, Shift-II]
= 2 sin
10  10 10  1− i 3+ i 1+ i
π (a) 4i (b) (c) (d)
So, amplitude of given complex number is . 2 2 2
10
Sol. (b)
60. If a point P denotes a complex number Given complex equation,
z +1 iz 3 + 4z 2 − z + 4i = 0
z = x + iy in the argand plane and if is a
z+i ⇒ z (iz + 4) + i(iz + 4) = 0
2

purely real number, then the locus of P is ⇒ (iz + 4)(z 2 + i) = 0


[22 April 2018, Shift-II] ⇒ z = 4i or z 2 = i
(a) x + y + 1 = 0 ⇒ z = 4i
(b) x2 + y2 + x + y = 0  1 1 
or ±  − i
(c) x2 + y2 + 2 y + 1 = 0, ( x, y) ≠ (0, − 1)  2 2 
(d) x + y + 1 = 0,( x, y) =/ (0, − 1) So, the complex root having minimum
Sol. (d) magnitude is
z + 1 x + iy + 1 (x + 1) + iy 1−i −1 + i
Since, = = or
z + i x + iy + i x + i(y +1) 2 2
x − i(y + 1) 3+i
× 63. If z = , then (z101 + i103)105 =
x − i(y + 1)
2
z+1 [23 April 2018, Shift-I]
If is a purely real number, z ≠ − i
z+ i (a) z (b) z2 (c) z3 (d) − z
 z + 1
Then, Im   = 0, z = − i Sol. (c)
 z + i
3+ i 3 1
⇒xy − (x + 1) (y + 1) = 0, (x , y) ≠ (0, − 1) We have, z = ⇒ z= + i
2 2 2
⇒ xy − xy − x − y − 1 = 0
π π
⇒ x + y + 1 = 0 , (x , y) ≠ (0, − 1) ⇒ z = cos + i sin
6 6

61. If ω is a complex cube root of unity, the
⇒ z=e 6
51 + 73ω + 87 ω 2 51 + 73ω + 87 ω 2  101 π  5π
+ 
 = Now, z101 = e i  6 

= e i16π . e i 6
 73 + 87 ω + 51ω 87 + 51ω + 73ω 2 
2

− 3 i
[22 April 2018, Shift-II] = +
2 2
(a) 1 (b) − 1 (c) 0 (d) 2
and i103 = i .i = i3 = − i
100 3
Sol. (b) 105
15  3 i 
 51 + 73ω + 87ω2 51 + 73ω + 87ω2  Now, (z101 + i103)105 =   − +  − i
 73 + 87ω + 51ω2 + 87+ 51ω + 73ω2    2 2 
 
105
 51ω2 + 73ω3 + 87ω4 1 
15   3 i 
= −  +  = (− z)105
 73 + 87ω + 51ω2 × ω2    2 2 
=  105 3
 51ω + 73ω2 + 87ω3 1   iπ  i3 π  iπ 
 + ×  = − e 6  = −e i17π
.e 6 =  e 6  = z3
 87 + 51ω + 73ω2 ω    
   
16 AP EAMCET Chapterwise Mathematics

3i 1
64. If u + iv = , then y = and slope of line = m2 =
x + iy + 2 2
1
[23 April 2018, Shift-I] Now, m1 m2 = − 2 × = − 1
9u 3u 6u 12u 2
(a) (b) (c) (d) So, line −2x + 4 y + 5 is perpendicular to AB.
u2 + v2 u2 + v2 u2 + v2 u2 + v2
Hence, locus of point p is the perpendicular
Sol. (b) bisector of AB.
3i
We have, u + iv = 8 π 8 π
x + iy + 2 66. If a = cos   + i sin   , then
 11   11 
3i
⇒ x + iy + 2 = Re(a + a 2 + a 3 + a 4 + a 5) =
u + iv
3i u − iv 3ui − 3v(i 2) [23 April 2018, Shift-I]
⇒ (x + 2) + iy = + = 2 1 1
u + iv u − iv u − (iv)2 (a) 0 (b) − (c) (d) 1
3ui + 3v 2 2
⇒ (x + 2) + iy = 2 [Q i 2 = − 1]
u + v2 Sol. (b)
i8π
3v 3u 8π   8 π  ⇒ a = e 11
⇒ (x + 2) + iy = 2 + i a = cos   + i sin  
u + v2 u2 + v2  11   11 
by comparing Imaginary parts ⇒ a is 11th root of unity and all roots are
3u
y= 2 1, a , a 2 , ..., a10
u + v2
a10 . a a11 1
Now, a10 = = = =a
65. Let A(3 − i), B(2 + i) be two points in the a a a
argand plane. If the point P represents the Similarly, a 9 = a 2 , a 8 = a 3 , a 7 = a 4 , a 6 = a 5 ,
complex number z = x + iy, which satisfies We know that,
| z − 3 + i | = | z − 2 − i |, then the locus of the Sum of n roots of unity = 0
point P is [23 April 2018, Shift-I] 1 + a1 + a 2 + a 3 + ... + a10 = 0
(a) the circle with AB as diameter ⇒ a + a2 + a3 + a4 + a5 + a6 + a7 + a8
(b) the line passing through A and B
(c) the perpendicular bisector of AB a 9 + a10 = − 1
(d) the ellipse with AB as major axis ⇒(a + a) + (a 2 + a 2) + (a 3 + a 3)
Sol. (c) + (a 4 + a 4) + (a 5 + a 5) = − 1
We have a complex number z = x + iy ⇒ 2Re (a) + 2Re (a 2) + 2Re (a 3) +2Re (a 4) + 2Re (a 5)
and |z − 3 + i | = |z − 2 − i| = − 1[z + z = 2 Re (z)]
⇒ | x + iy − 3 + i | = | x + iy − 2 − i | ⇒ 2Re (a + a 2 + a 3 + a 4 + a 5) = − 1
⇒ (x − 3)2 + (y + 1)2 = (x − 2)2 + (y − 1)2 1
⇒ Re (a + a 2 + a 3 + a 4 + a 5) = −
⇒ x 3 − 6 x + 9 + y 2 + 2y + 1 2
= x 2 − 4 x + 4 + y 2 − 2y + 1 67. If a complex number z satisfies
⇒ − 2x + 4 y + 5 = 0 … (i) 2
z + 1 = z 2 − 1, then the locus of z is
So, it represent a line
Point A(3, − 1) and B (2, 1) [23 April 2018, Shift-II]
(a) a circle (b) the real axis
So, mid-point of AB =  , 0
5
2  (c) the imaginary axis (d) the straight line y = x
1 − (−1) Sol. (c)
m1 = slope of AB = =−2
2− 3 Let z = x + iy, so
| z |2 + 1 = | z 2 − 1 |
Point  , 0 satisfies the equation
5
2  ⇒ x 2 + y 2 + 1 = (x 2 − y 2 − 1)2 + 4 x 2 y 2
− 2x + 4 y + 5 = 0
⇒(x 2 + y 2 + 1)2 = (x 2 − y 2 − 1)2 + 4 x 2 y 2
Complex Numbers and De-Moivre’s Theorem 17

⇒ (x 2 + y 2 + 1)2 − (x 2 − y 2 − 1)2 = 4 x 2 y 2 Sol. (a)


⇒ [(x 2 + y 2 + 1) + (x 2 − y 2 − 1)] a + ib
Given that, α=
1−c
[(x 2 + y 2 + 1) − (x 2 − y 2 − 1)] = 4 x 2 y 2
a2 + b2
⇒ (2x 2) (2y 2 + 2) = 4 x 2 y 2 ⇒ |α |2 = … (i)
(1 − c)2
⇒ x y + 2x = x y
2 2 2 2 2
⇒ x = 0 ⇒ x = 0,
2
And a 2 + b 2 + c 2 = c
So, locus is a imaginary axis.
⇒ a 2 + b 2 = c(1 − c) … (ii)
68. P is a point denoting z in the argand diagram From Eqs. (i) and (ii),
z−i
and if is always purely imaginary, then c 1 + |α |2 1
z −1 |α |2 = ⇒ =
(1 − c) 1 1−c
locus of P is [23 April 2018, Shift-II]
1
⇒ 1−c=
(a) the circle with centre  ,  and radius
1 1 1 … (iii)
2 2 1 + |α |2
2
 1 1
(b) the circle with centre  − , −  and radius
1 From Eqs. (i) and (iii), we get
 2 2 2 a2 + b2 |α |2
|α |2 = ⇒ a2 + b2 = .
(c) the points on the circle with centre  ,  and
1 1  1 
2
(1 + |α |2)2
2 2  
1 + |α | 
2
1
radius , excluding the points (1, 0) and (0, 1)
2 70. For n ∈ Z + ,
(d) the points on the circle with centre
(1 + sin θ + i cos θ)n + (1 + sin θ − i cos θ)n =
 − 1 , − 1  and radius 1 , excluding the origin [23 April 2018, Shift-II]
 
 2 2 2 π θ nπ θ
n
(a) 2 n+1
⋅ cos  −  cos − 
4 2  4 2
Sol. (c)
π θ nπ θ
z − i x + i(y − 1) (b) 2 n + 1 ⋅ cos n  −  . sin  − 
Let z = x + iy, then = 4 2   4 2
z − 1 (x − 1) + iy
π θ nπ nθ 
 z − i  x(x − 1) + y(y − 1) (c) 2 n + 1 ⋅ cos n  −  cos  − 
So, Re  = 4 2  4 2 
 z + 1 (x − 1)2 + y 2 π θ nπ nθ 
(d) 2 n + 1 ⋅ cos n  −  sin  − 
z−i 4 2  4 2 
Q is purely imaginary,
z+1
Sol. (c)
 z − i (1 + sinθ + i cosθ)n + (1 + sinθ − i cosθ)n
 = 0 ⇒ x + y − x − y = 0,
2 2
So, Re 
 z + 1 n
 π π 
= 1 + cos  − θ + i sin − θ
and it is a circle with centre  ,  and radius
1 1  2  2  
 2 2
n
 π π 
+ 1 + cos − θ − i sin − θ
1
, excluding the points (1, 0) and (0, 1). 
2 2  2  
n
69. If a , b, c are non-zero real number with c ≠ 1  π θ π θ π θ 
= 2cos2  −  + 2 i sin −  cos − 
a + ib   4 2  4 2  4 2 
such that a 2 + b2 + c 2 = c and if α = ,
1− c  π θ π θ π θ 
n
+ 2cos2  −  − 2i sin −  cos − 
then a + b =
2 2
[23 April 2018, Shift-II]   4 2  4 2  4 2 
2 4
α α π θ 
n
π θ π θ 
(a) (b) = 2n cosn  −   cos −  + i sin − 
(1 + α )2
2
(1 + α )2
2  4 2   4 2  4 2 

α α
π θ  
n
 π θ
(c) 2
(d)
1+ α + cos −  − i sin −  
1+ α   4 2  4 2  

18 AP EAMCET Chapterwise Mathematics

π θ  nπ nθ   nπ − nθ 
= 2n cosn  −  cos −  + i sin  73. If C r = nC r , then C 0 + C 4 + C 8 + C12 + K =
 4 2   4 2  4 2
[24 April 2018, Shift-I]
nπ nθ   nπ − nθ  
+ cos −  − i sin  n  nπ
n
− 1
 4 2  4 2   22 sin + 22 
 4 
π θ nπ nθ 
= 2n + 1 cosn  −  cos − . (a)
 4 2  4 2 2
n
2 nπ
71. The maximum value of the modulus of e z on (b) 2 2 sin
4
the set { z ∈ C / 0 ≤ Re (z) ≤ 1, 0 ≤ Im (z) ≤ 1} is n −1 nπ
(c) 2 cos
[24 April 2018, Shift-I] 4
2 n  n
− 1
(a) (b) e (c) e + 1 (d) e 2 n π
e 2 2 cos + 22 
 4 
Sol. (b) (d)
2
Let z = x + iy 0 ≤ x ≤ 1, 0 ≤ y ≤ 1
2 2
Sol. (d)
− y 2 + 2ixy
⇒ ez = ex
In given series difference of suffies is 4
2 2
− y2
⇒ ez = ex ⋅ e i( 2xy) 0 − 4 = 8 − 4 = 12 − 4 = K = 4
2 2 1 1
− y2
⇒ | ez | = ex [| e i( 2xy) | = 1] 1 4 = (cos 0 + i sin 0) 4
Now, ()
0 ≤ x 2 ≤ 1, 0 ≤ y 2 ≤ 1 ⇒ 0 ≤ x 2 ≤ 1 1

− 1 ≤ − y2 ≤ 0 ⇒ − 1 ≤ x 2 − y2 ≤ 1 = (cos 2rπ + i sin 2rπ) 4


rπ rπ
= cos + i sin
2
⇒ maximum of | e z | = e
2 2
72. If Z ≠ ± 1 is a complex number and where r = 0, 1, 2, 3
 Z − 1 π Four roots of unity are 1, i , − 1, − i
Arg   = , then the locus of Z in the = 1,α ,α ,2 α 3 (say)
 Z + 1 4 n
Arg and plane is [24 April 2018, Shift-I] and (1 + x) = ∑ Cr x
n n r
r =0
(a) x + y − 2 y − 1 = 0 (b) x + y + 2 y − 1 = 0
2 2 2 2
Putting x = 1, α, α 2, α 3
(c) x2 + y2 − 2 x + 1 = 0 (d) x2 + y2 + 2 x + 1 = 0 n
2n = ∑ n Cr … (i)
Sol. (a) r =0
n
Let z = x + iy (1 + α)n = ∑ n Crα r … (ii)
z − 1 x + iy − 1 r =0
=
z + 1 x + iy + 1 n
(1 + α 2)n = ∑ n Crα 2r … (iii)
(x − 1) + iy (x + 1) − iy r =0
= × n
(x + 1) + iy (x + 1) − iy (1 + α 3)n = ∑ n Crα 3r … (iv)
r =0
x 2 + y 2 − 1 + 2iy
= On adding Eqs. (i), (ii), (iii), (iv), we get
(x + 1)2 + y 2
2n + (1 + α)n + (1 + α 2)n + (1 + α 3)n
 z − 1 −1 2y  n
arg   = tan  2  = π/4 = ∑ n Cr (1 + α r + α 2r + α 3r) … (v)
 z + 1   x + y 2
− 1  r =0
2y For r = 0, 4, 8, 12 K R.H.S of Eq. (v)
⇒ = tan π / 4 = 1
x 2 + y2 − 1 n
C0 (1 + 1 + 1 + 1) + n C4 (1 + α 4 + α 8 + α12)
x 2 + y 2 − 1 = 2y + n C8 (1 + α 8 + α16 + α 24) + ... +
⇒ x + y − 2y − 1 = 0.
2 2
= 4( C0 + C4 + n C8 K )
n n
(Q α 4 = 1).
Complex Numbers and De-Moivre’s Theorem 19

L.H.S of Eq. (v) 74. One of the complex roots of the equation
= 2n + (1 + i)n + (1 + i 2)n + (1 + i 3)n x 11 − x 6 − x 5 + 1 = 0 is [24 April 2018, Shift-I]
= 2n + (1 + i)n + 0 + (1 − i)n 3π π
(a) C is (b) C is
n n 5 3
π π
Since, (1 + i)n = 22  cos + i sin  (c) C is

(d) C is

 4 4 6 5
n
nπ nπ
= 22  cos + i sin 
Sol. (b)
 4 4  x11 − x 6 − x 5 + 1 = 0
n
nπ nπ ⇒ (x 6 − 1) (x 5 − 1) = 0
(1 − i)n = 22  cos − i sin 
 4 4  ⇒ x 6 = 1 or x 5 = 1
⇒ 2 + (1 + i) + (1 − i)
n n n 1 1

n
⇒ x = ()1 6 or x = ()
15
nπ 2kπ 2kπ
= 2n + 2⋅ 22 cos x = cos + i sin
4 6 6
⇒ 4 ⋅ (n C0 + n C4 + n C8 + n C12 ...) 2rπ 2rπ
or x = cos + i sin
n  n
−1
5 5

= 2⋅ 22  cos + 22  where, k = 0, 1, 2, 3, 4, 5 where r = 0, 1, 2, 3, 4
 4  kπ kπ
x = cos + i sin
n n
−1 3 3

22  cos + 22  When k = 1,
 4 
=  π π
cos + i sin π / 3 = C is .
2 3 3
2
Quadratic Expressions
1. If [ a , b] is the range of the function ⇒ x 2(y − 1) + x(2y − 34) + (71 − 7 y) = 0
x +2 As x is complex, directriminant of above
for x ∈R, then equation,
2x2 + 3x + 6
[17 Sep. 2020, Shift-I] D≤ 0
(a) a < 0, b < 0 (b) a < 0, b > 0 ⇒ (2y − 34)2 − 4(y − 1) (71 − 7 y) ≤ 0
(c) a > 0, b > 0 (d) a > 0, b < 0 ⇒ 5< y < 9
Sol. (b) ⇒ (a , b) ≡ (5, 9)
x+2
[a , b] is range of and x ∈ R 3. Solve (8 − t)2 < (t2 − 3t − 10) [21 Sep. 2020, Shift-II]
2x 2 + 3x + 6
(a)  , 8 (b)  ,∞  (c) (8, ∞ )
74 74
x+2 (d) [8, ∞ )
Let y= 2  13   13 
2x + 3x + 6
⇒ 2yx 2 + 3xy + 6 y = x + 2 Sol. (b)
⇒ 2yx + (3y − 1) x + 6 y − 2 = 0
2 It is given that,
x ∈ R So, D ≥ 0 (8 − t)2 < t 2 − 3t − 10
⇒ (3y − 1)2 − 4(6 y − 2)(2y) ≥ 0 ⇒ 64 − 16t + t 2 < t 2 − 3t − 10
⇒ −39 y 2 + 10 y + 1 ≥ 0 ⇒ 13t > 74
74
⇒ 39 y 2 − 10 y − 1 ≤ 0 ⇒ t>
13
(3y − 1)(13y + 1) ≤ 0 ⇒ y ∈  − , 
1 1
⇒ t∈ , ∞ 
74
 13 3 ⇒
 13 
1 1
So, a = − , b = Hence, option (b) is correct.
13 3
x
∴ a < 0, b > 0 4. If x ∈ R, then the range of is
x − 5x + 9
2
x 2 + 34 x − 71
2. If x is complex, the expression [20 April 2019, Shift-II]
x2 + 2x − 7 −1
(a)  − , 1 (b)  −∞,  ∪ (1, ∞ )
1
takes all values which lie in the interval (a , b),  11   11 
find the value of a and b. [17 Sep. 2020, Shift-II] −1
(c)  , 1 (d)  −1, 
1
(a) a = − 1, b = 1 (b) a = 1, b = − 1  11   11
(c) a = 5, b = 9 (d) a = 9 , b = 5
Sol. (c)
Sol. (c) x
Let = y
Here we have to find range of x 2 − 5x + 9
x 2 + 34 x − 71 ⇒ yx 2 − (5y + 1) x + 9 y = 0
= y (Let)
x 2 + 2x − 7
Q x ∈ R ⇒ D ≥ 0 ⇒(5y + 1)2 − 36 y 2 ≥ 0
Then, x 2 + 34 x − 71 = x 2 y + 2xy − 7 y
Quadratic Expressions 21

⇒ 25y 2 + 10 y + 1 − 36 y 2 ≥ 0 ⇒ x 2 + 2x + 2 = − yx 2 + 2xy − y
⇒ 11 y − 10 y − 1 ≤ 0
2
⇒ x + yx 2 + 2x − 2xy + 2 + y = 0
2

⇒ 11 y 2 − 11 y + y − 1 ≤ 0 ⇒ x 2(1 + y) + (2 − 2y) x + 2 + y = 0 ⇒ D ≥ 0

⇒ (11 y + 1)(y −1) ≤ 0 ⇒ y ∈  − , 1


1 ∴ (2 − 2y)2 − 4(2 + y) (1 + y) ≥ 0
 11 
4 + 4 y 2 − 8 y − 4(2 + y) (1 + y) ≥ 0
Hence, option (c) is correct.
1 + y 2 − 2y − (2 + y) (1 + y) ≥ 0
5. The set of all real numbers satisfying the 1 + y 2 − 2y − (2 + 2y + y + y 2) ≥ 0
inequation x − | x + 2 | + x > 0 is
2
1 + y 2 − 2y − 2 − 3y − y 2 ≥ 0
[21 April 2019, Shift-I] 1
− 5y − 1 ≥ 0 ⇒ y ≤ −
(a) [− 2, − 2 ) ∪ ( 2 , ∞ ) (b) (− ∞, − 2 ) ∪ (2, ∞ ) 5
(c) (− ∞, − 2 ) ∪ ( 2 , ∞ ) (d) (− ∞, − 2 ) ∪ ( 2 , ∞ )
(x) ∈  −∞ , − 
f 1
So,
Sol. (c) g  5
x 2 − | x + 2| + x > 0 1
So, a = 1, b = 0 and c = −
Case I If x + 2 ≥ 0 5
Hence, a + 2b + 5c + 4 = 1 + 0 + 5  −  + 4
1
  x, x≥ 0
x 2 − (x + 2) + x > 0 Q| x | =    5
 − x , x< 0
=1 −1 + 4 = 4
⇒ x2 − 2> 0
x−P
⇒ (x − 2)(x + 2) = 0 7. If takes all real values for x ∈ R,
x2 − 3x + 2
+ − +
then the range of P is [21 April 2019, Shift-II]
−2 − 2 2
(a) 1 ≤ P ≤ 2 (b) 1 < P < 2
x ∈ [− 2, − 2] ∪ [ 2, ∞] (c) P < 1or P > 2 (d) P ≥ 2 or P ≤ 1
Case II x + 2 < 0 Sol. (a)
x 2 + (x + 2) + x > 0 ⇒ x 2 + 2x + 2 > 0 x−P
Let = y, provided x 2 − 3x + 2 ≠ 0
⇒ (x + 1)2 + 1 > 0 ⇒ x < − 2 x 2 − 3x + 2
Hence, the solution set is ⇒ x ≠ 1, 2
x ∈ (− ∞ , − 2) ∪ ( 2, ∞) ⇒ yx 2 − (3y + 1) y + (2y + P) = 0
Q x ∈ R, so D ≥ 0
6. Let f (x) = x 2 + 2 x + 2, g(x) = − x 2 + 2 x − 1 ⇒ (3y + 1)2 − 4 y(2y + P) ≥ 0
and a , b be the extreme values of f (x), g(x)
⇒ y 2 + (6 − 4P) y + 1 ≥ 0
f
respectively. If c is the extreme value of (x) Q y ∈ R, so D ≤ 0
g
⇒ (6 − 4P)2 − 4 ≤ 0
(for x ≠ 1), then a + 2 b + 5 c + 4 =
[21 April 2019, Shift-I]
⇒ (4P − 6 + 2)(4P − 6 − 2) ≤ 0
⇒ (P − 1)(P − 2) ≤ 0 ⇒ P ∈[1, 2]
(a) 2 (b) 1
(c) 4 (d) 3  6 + x − x2 6 + x − x 2 
Sol. (c) 8.  x ∈ R : ≥ =
 2x + 5 x+4 
Given, f (x) = x 2 + 2x + 2
[21 April 2019, Shift-II]
= x 2 + 2x + 1 + 1 = (x + 1)2 + 1
−5
(a) [−2, 3] (b) (−∞, − 4] ∪  , − 1
Here, f (x) ∈ [1, ∞) and g(x) = − x 2 + 2x − 1  2 
= − (x 2 − 2x + 1) = − (x − 1)2 (c) [−2, − 1] ∪ {3} (d) (−∞, − 4] ∪ [−2, − 1]
Here, g(x) ∈ (− ∞ , 0] Sol. (c)
f x 2 + 2x + 2 6 + x − x2 6 + x − x2
Now, (x) = = y Given, ≥
g − x 2 + 2x − 1 2x + 5 x+ 4
22 AP EAMCET Chapterwise Mathematics

 1 1  (a) (− 2, 7 ) (b) φ
⇒ 6 + x − x2  − ≥0
 2x + 5 x + 4   2 − 14 2 + 14 
(c)  ,  (d) R
 5 5 
 x + 4 − 2x − 5 
⇒ 6 + x − x2  ≥0
 (2x + 5)(x + 4)  Sol. (c)
We have,
 −(x + 1) 
⇒ 6 + x − x2  ≥ 0 x 2 − 5x − 14 > 0 ⇒ (x − 7)(x + 2) > 0
(2x + 5)(x + 4) 
+ – +
–1 –2 7
–4 –5
2 ∴ x ∈ (−∞ , − 2) ∪ (7, ∞)
⇒ (x + 1)(2x + 5)(x + 4) ≤ 0 ∴ A = (−∞ , − 2) ∪ (7, ∞)
x ∈ (−∞ , − 4] ∪  − , − 1 −6 x 2 + 2x − 3 < 0
5 Again,
⇒ …(i)
 2 
⇒ 6 x 2 − 2x + 3 > 0
Now, far expression to be exist
which is always true for all x ∈ R
6 + x − x2 ≥ 0
∴ B=R
⇒ x2 − x − 6 ≤ 0 Also, 4 x − 5x 2 + 2 > 0
⇒ (x − 3)(x + 2) ≤ 0 ⇒ x ∈ [−2, 3] …(ii) ⇒ 5x 2 − 4 x − 2 < 0
By Eqs. (i) and (ii), we get  2 + 14   2 − 14 
x ∈ [−2, − 1] ∪ {3} ⇒ x − x − <0
 5  5 
3 x 2 −7 x + 8 – +
9. If x ∈ R and 1 ≤ ≤ 2, then the –
x2 + 1
 2 − 14   2 + 14 
minimum and maximum values of x are    
respectively. [22 April 2019, Shift-I]  5   5 
(a) 1, 2 (b) 5, 12 (c) 6, 10 (d) 1, 6   2 − 14   2 + 14  
∴ x ∈ ,  
Sol. (d)  5   5 
3x 2 − 7 x + 8  2 − 14 2 + 14 
Given, 1≤ ≤ 2 [Q x 2 + 1 ≥ 0 ] ∴ C= , 
x2 + 1  5 5 
x2 − 7x + 8  2 − 14 2 + 14 
Now, 1≤ ⇒ 0 ≤ 2x 2 − 7 x + 7 ∴ A∩B∩C=
x2 + 1 , 
 5 5 
∴ f (x) = 2x 2 − 7 x + 7
∆ = (7)2 − 4(2)(0) ⇒ ∆ < 0
11. The complete solution set of the inequation

⇒ f (x) = 2x 2 − 7 x + 7 > 0 x 2 − 3 x + 2 > (3 − x) is [22 April 2019, Shift II]


(a)  , 3
7
3x − 7 x + 8
2 (b) (3, ∞ )
⇒ x ∈ R and ≤2  3 
x2 + 1
(d)  , ∞ 
7
(c) (− ∞, 1] ∪ [2, ∞ )
⇒ x2 − 7x + 6 ≤ 0 3 

⇒ (x − 1)(x − 6) ≤ 0 ⇒ x ∈[1, 6] Sol. (d)


Hence, minimum and maximum values are 1 Given,
and 6. x 2 − 3x + 2 > (3 − x)
10. If A , B, C are the sets of all values of x, for ⇒ x 2 − 3x + 2 > x 2 + 9 − 6 x
which x 2 − 5 x − 14 is positive, − 6 x 2 + 2 x − 3 ⇒ 3x > 7 ⇒ x >
7
is negative and 4 x − 5 x 2 + 2 is positive 3
respectively, then A ∩ B ∩ C = ∴  7 
x ∈  , ∞
[22 April 2019, Shift-II] 3 
Quadratic Expressions 23

12. If a and b are the maximum and minimum and 2x 2 + (3 − k) x + 2 > 0, ∀ x ∈ R


values of the quadratic expressions Then D < 0 ⇒ (3 − k)2 − 16 < 0
1 − 2 x − 5 x 2 and x 2 − 2 x + 5 respectively, ⇒ (k − 3 − 4) (k − 3 + 4) < 0
then the set of all values of x for which the ⇒ k ∈ (−1, 7) …(ii)
expression 5 ax 2 + bx + 7 is positive, is From intervals (i) and (ii), we get
[23 April 2019, Shift-I] k ∈ (−1, 5)
(a) (a, b ) (b) (−∞, 7 ) Hence, option (b) is correct.
(c) (5, ∞ ) (d) (−∞, ∞ ) 14. The number of integral vaules of x satisfying
Sol. (d) 5 x − 1 < (x + 1)2 < 7 x − 3 is [22 April 2018, Shift-I]
The maximum value of the expression (a) 0 (b) 1 (c) 2 (d) 3
1 − 2x − 5x 2
4 + 20 6
Sol. (d)
a=− = We have, 5x − 1 < (x + 1)2
4(−5) 5
and minimum value of the expression ⇒ 5x − 1 < x 2 + 2x + 1
x 2 − 2x + 5, ⇒ x 2 − 3x + 2 > 0
4 − 20 ⇒ (x − 1) (x − 2) > 0
b=− =4
4 ⇒ x ∈ (− ∞ , 1) ∪ (2, ∞)
Now, the given quadratic expression Again, (x + 1)2 < 7 x − 3
6
5ax 2 + bx + 7 at a = and b = 4 must be ⇒ x 2 + 2x + 1 < 7 x − 3
5
positive, then ⇒ x 2 − 5x + 4 < 0
6x2 + 4x + 7 > 0 ⇒ (x − 1) (x − 4) < 0
Q Discriminant D = 16 − 4(6) (7) < 0 and ⇒ x ∈(1, 4)
coefficient of x 2 term is positive. ∴ x=3 [Q x is integer]
∴ 6 x 2 + 4 x + 7 > 0, ∀ x ∈ R = (− ∞ , ∞).
15. Given that a, b and c are real numbers such
Hence, option (d) is correct. that b2 = 4 ac and a > 0. The maximal possible
x + kx + 1
2 set D ⊆ R on which the function f :D → R
13. If < 3 for all real numbers x, given by
x2 + x + 1
f (x) = log{ ax 3 + (a + b) x 2 + (b + c) x + c} is
then the range of the parameter k is
[23 April 2019, Shift-I] defined, is [22 April 2018, Shift-I]
(a) R − − 
b
(a) (0, 4) (b) (− 1, 5) (c) (− 4, 0) (d) (− 5, 1)
 2a
Sol. (b)
(b) R −  −  ∪ (−∞,−1)
b
It is given, for all x ∈ R
  2a 
x 2 + kx + 1
<3 
(c) R −  −  ∪ { x: x ≥ 1}
 b
x2 + x + 1   2a 
x 2 + kx + 1 (d) R − ({− b / 2 a} ∪ (−∞,−1])
⇒ − 3< <3
x2 + x + 1
Sol. (d)
⇒ −3x 2 − 3x − 3 < x 2 + kx + 1 < 3x 2 + 3x + 3 Given function,
[Q x 2 + x + 1 > 0, ∀ x ∈ R] f (x) = log{ax 3 + (a + b) x 2 + (b + c) x + c}
So, 4 x 2 + (k + 3) x + 4 > 0, ∀ x ∈ R = log{(ax 2 + bx + c)(x + 1)}
Then D < 0 ⇒ (k + 3) − 4(4) (4) < 0
2
The function f (x) will be define, if
⇒ (k + 3)2 − 82 < 0 (ax 2 + bx + c)(x + 1) > 0
⇒ (k + 3 − 8) (k + 3 + 8)< 0 ⇒ x + 1 > 0 {Q a > 0 and b 2 = 4ac}
⇒ k ∈ (− 11, 5) …(i) and x≠−
b
2a
24 AP EAMCET Chapterwise Mathematics

So, D = x : x ∈ (−1, ∞) and x ≠ −  f (x) = a  x +  (3 − x), a > 0


b 5
 2a   3
g(x) = b(x − 3)  x −  , b > 0
9
D = R − −  ∪ (−∞ , − 1].
b and
or 
 2a  2
Now, f (x) ⋅ g(x) = ab
16. For real number x, if the minimum value of  x + 5 (3 − x) (x − 3)  x − 9 
   
f (x) = x 2 + 2 bx + 2 c 2 is greater than the  3  2
maximum value of g(x) = − x 2 − 2 cx + b2 , then According to wavy curve method,
[22 April 2018, Shift-I] + +
(a) c 2 > 2 b 2 (b) c 2 < 2 b 2 –5 3 9 –
(c) b 2 = 2c 2 (d) c 2 = 2 b 2 3 2
So, f (x) ⋅ g(x) is positive in [0, 3) ∪  3, 
9
Sol. (a)  2
We have,
and negative in  , 5.
9
f (x) = x 2 + 2bx + 2c 2  2 
= (x + b)2 + 2c 2 − b 2 x2 + 2x + 1
∴ Minimum value of
18. If x is real, then the range of is
x2 + 2x + 7
f (x) = 2c 2 − b 2
[23 April 2018, Shift-II]
Again, g(x) = − x 2 − 2cx + b 2 (a) [0, 1) (b) (−∞, 0) ∪ (1, ∞ )
= − [x 2 + 2cx − b 2] (c) (0, 1) (d) R
= − [(x + c)2 − b 2 − c 2] Sol. (a)
= − (x + c) + b + c
2 2 2 x 2 + 2x + 1
Let = y,
x 2 + 2x + 7
∴ Maximum value of
g(x) = b 2 + c 2 Q y ≠1 …(i)
⇒ (y − 1) x + 2(y − 1) x + (7 y − 1) = 0
2
Now, according to the question.
2c 2 − b 2 > b 2 + c 2 ⇒ c 2 > 2b 2 Q x ∈ R,
so, D≥ 0
17. f (x) is a quadratic expression such that f (x) ⇒ 4(y − 1)2 − 4(y − 1) (7 y − 1) ≥ 0
 5 ⇒ (y − 1) [y − 1 − 7 y + 1] ≥ 0
is negative when x ∈  − ∞ , −  ∪ (3 , ∞) and
 3 ⇒ y(y − 1) ≤ 0
 5  ⇒ y ∈[0, 1] …(ii)
positive when x ∈  − , 3 . g(x) is another From Eqs. (i) and (ii), we are getting
 3 
y ∈[0, 1)
quadratic expression such that g(x) is negative
 9  9 19. The solution set of the inequation
when x ∈ 3 ,  and positive when x ∈ R − 3 , .
 2  2 x 2 + 6 x + 5 > (8 − x) is [24 April 2018, Shift-I]
Then, the sign of f (x) g(x) in [0, 5] is
(b)  , 8 (c)  , ∞  (d) (− 1, ∞ )
59 59
(a) (8, ∞ )
[22 April 2018, Shift-II]  22   22 
(a) positive in  0,  and negative in  , 5
9 9
 2 
Sol. (c)
2 
x 2 + 6 x + 5 > (8 − x)
(b) positive in [0, 3) ∪  3,  and negative in  , 5
9 9
 2  2  On squaring both side,
(c) positive in [0, 3) ∪  3,  ∪  , 5
9 9 ( x 2 + 6 x + 5)2 > (8 − x)2
 2   2 
x 2 + 6 x + 5 > 64 − 16 x + x 2
(d) negative in [0, 3) ∪  3,  ∪  , 5
9 9
6 x + 16 x > 64 − 5 ⇒ 22x > 59
 2   2 
⇒ x ∈  , ∞ 
59 59
x>
Sol. (b) 22  22 
3
Theory of Equations
1. The roots of the equation| x 2 − x − 6| = x + 2 Sol. (b)
are [17 Sep. 2020, Shift-I] Given,
(a) −2, 1, 4 (b) 0, 2, 4 f (x) = x 5 − 5x 3 − 5x − 1 = 0
(c) 0, 1, 4 (d) − 2, 2, 4 Now by inspection,
Sol. (d) x = 1 is a zero of f (x)
Given, |x 2 − x − 6|= x + 2 So, we can divide f (x)
by x − 1 to get,
⇒ x2 − x − 6 = x + 2
f (x) = (x − 1) (x 4 + x 3 − 4 x 2 + x + 1)
Or x2 − x − 6 = − x − 2
⇒ x 2 − 2x − 8 = 0 or x 2 = 4 We similarly proceed to get
f (x) = (x − 1)(x − 1) (x 3 + 2x 2 − 2x − 1)
⇒ (x = −2, 4) or (x = ±2) ⇒ x = {−2, 2, 4}
f (x) = (x − 1) (x − 1) (x − 1) (x 2 + 3x + 1)
2. If x ∈R, then one of the solutions of
So, f (x) has 3 equal roots.
x + 1 − | x − 1| = 4 x − 1 among the
following is [17 Sep. 2020, Shift-I] 4. Roots of the equation x 3 − 3 x 2 + 3 x − 9 = 0
5 −5 are ...... [17 Sep. 2020, Shift-II]
(a) x = (b) x =
4 4 (a) 3, 1 + 2ω , 1 + 2ω2 (b) 3, − 1 + 2ω , − 1 − 2ω2
(c) x = 0 (d) x = 1 (c) 3, 2 − ω , 2 − ω2 (d) 1, 1 + 2ω , 1 + 2ω2
Sol. (a) Sol. (a)
x + 1 −| x − 1|= 4x −1
Given equation is,
⇒ x + 1 − 4 x − 1 =| x − 1| x 3 − 3x 2 + 3x − 9 = 0
Squaring on both sides ⇒ (x − 3) (x 2 + 3) = 0
⇒ x + 1 + 4 x − 1 − 2 (x + 1)(4 x − 1) = x − 1
So one of root is 3.
⇒ 5x − 2 (x + 1)(4 x − 1) = x − 1 Now, x 2 + 3 = 0
⇒ 4 x + 1 = 2 (x + 1)(4 x − 1) taking x = 1 + 2w, we have
Squaring on both sides (1 + 2ω)2 + 3
⇒ 16 x 2 + 1 + 8 x = 4(4 x 2 + 3x − 1) = 1 + 4ω 2 + 4ω + 3
⇒ 16 x 2 + 8 x + 1 = 16 x 2 + 12x − 4 = 1 + (4ω2 + 4ω + 4) − 1
5
⇒ 4x = 5 ⇒ x = = 1 + 4(ω2 + ω + 1) − 1
4
= 0; or ω2 + ω + 1 = 0
3. Equation x 5 − 5 x 3 + 5 x 2 − 1 = 0 has ..........
Similarly,1 + 2ω2 also satisfies given equation
equal roots [17 Sep. 2020, Shift-II]
So roots are; 3, 1 + 2ω and 1 + 2ω2
(a) 2 (b) 3 (c) 4 (d) 5
26 AP EAMCET Chapterwise Mathematics

5. If the roots of the equation ax 2 + ax + c = 0 So, α will satisfy both the equations
Now, α 3 + α 2 − 2α − 2 = 0
p q
are in the ratio p : q, then + = and α 3 − α 2 − 2α + 2 = 0
q p
[17 Sep. 2020, Shift-II] On subtraction, we get
2 2 2α 2 − 2 = 0 ⇒ α = ± 1.
a a a a
(a) (b) (c) (d) So, there are 2 common roots.
c 2c c 2c
Sol. (c) 8. Find α 4 + β 4 if α, β are the roots of equation
Equation given be, x 2 + x + 1 = 0. [18 Sep. 2020, Shift-I]
ax 2 + ax + c = 0 1 2
(a) (b) (c) αβ (d) − αβ
a αβ αβ
∴Sum of roots = − = −1
a
Sol. (d)
c
and product of roots = . The roots of the equation x 2 + x + 1 = 0 are α
a
p and β and α 3 = β 3 = 1 and α + β + 1 = 0
Now roots are given in ratio ,
q {Qα , β are non-real cube roots of unity}
α p ∴ α + β4 = α + β
4
i.e., =
β q = −1 = −αβ {Q product of roots αβ = 1}
p q α β
So, + = + 9. Which among the following equations has
q p β α roots which are negatives of the roots of the
=
α+β
=
−1
=−
a
or
a equation x 3 − x 2 + x − 4 = 0 ?
αβ c c c [18 Sep. 2020, Shift-I]
±
a (a) x − x + x − 4 = 0
3 2
(b) x + x2 + x + 4 = 0
3

6. If the sum of the roots of the quadratic (c) x − x + x − 4 = 0


3 2
(d) x3 − x2 − x − 4 = 0
equations is 1 and sum of the squares of the Sol. (b)
roots is 13, then find that equation. Let α is the root of equation
[18 Sep. 2020, Shift-I]
x3 − x2 + x − 4 = 0
(a) x2 + x − 6 = 0 (b) x2 − x + 6 = 0
Now, put (−α) = x , so we get the required
(c) x − x − 6 = 0
2
(d) x2 + x + 6 = 0 equation, and it is
Sol. (c) (− x)3 − (− x)2 + (− x) − 4 = 0
Let the roots of the quadratic equation are α and ⇒ − x3 − x2 − x − 4 = 0
β then it is given that ⇒ x3 + x2 + x + 4 = 0
α + β = 1 and α 2 + β 2 = 13
1 1
10. If a, b, c are in Arithmetic Progression (AP),
∴ αβ = [(α + β)2 − (α 2 + β 2)] = [1 − 13] = −6 then the roots of the equation
2 2
ax 2 − 2 bx + c = 0 are [18 Sep 2020, Shift-II]
So, equation of required quadratic is
c −1
x 2 − (α + β) x + αβ = 0 (a) 1, (b) , −c
a a
⇒ x 2 − x − 6 = 0. −c −c
(c) − 1, (d) − 2,
7. Number of roots common to the equations a 2a
x 3 + x 2 − 2 x − 2 = 0 and x 3 − x 2 − 2 x + 2 = 0 Sol. (a)
is [18 Sep. 2020, Shift-I] Given, a , b , c are in AP
(a) 1 (b) 2 (c) 3 (d) 0 ⇒ 2b = a + c
ax 2 − 2bx + c = 0
Sol. (b)
Let x is the common root of the equations ax 2 − (a + c) x + c = 0
x 3 + x 2 − 2x − 2 = 0 and x 3 − x 2 − 2x + 2 = 0 ax 2 − ax − cx + c = 0
Theory of Equations 27

ax(x − 1) − c (x − 1) = 0 1 3  π π
x = 2 + i  = 2 cos + i sin 
(x − 1)(ax − c) = 0 2 2   3 3
c
x = 1, iπ
a α = 2.e
3

Hence, option (a) is correct. − iπ

11. For how many values a ∈C, the equations Similarly, β = 2⋅ e 3

x 2 − 8 x + 7 = 0 and x 2 − 2 ax + 49 = 0 have a  iπ 
n
 − iπ   inπ
n
− inπ 

common root? [18 Sep. 2020, Shift-II] α + β =  2e 3  +  2e 3  = 2n  e 3 + e 3 


n n
     
     
(a) 1 (b) 3 (c) 2 (d) 0
n nπ nπ nπ nπ 
Sol. (c) = 2  cos + i sin + cos − i sin 
 3 3 3 3
Let α be the common root of given Equations nπ n+1 nπ
= 2 2cos
n
= 2 ⋅ cos
⇒ α 2 − 8α + 7 = 0 …(i) 3 3
α 2 − 2a α + 49 = 0 …(ii) Hence, option (b) is correct.
From Eq (i). − Eq. (ii) 13. Solve the equation, 3x
2
−x
= 25 − 4 x
2
−x
⇒ (−8 + 2a)α − 42 = 0
[21 Sep. 2020, Shift-I]
2(− 4 + a)α = 42
21 (a) − 1only (b) 2 only
α= (c) Both − 1and 2 (d) No solution
a−4
On putting value of α in Eq. (i), Sol. (c)
2 2
−x −x
2
 21  − 8  21  + 7 = 0 Given equation 3x = 25 − 4x
    2 2
 a − 4  a − 4 ⇒ 4x − x + 3x − x = 25 = 42 + 32
441 − 168(a − 4) + 7(a − 4) = 0 2
⇒ x2 − x = 2 ⇒ x2 − x − 2 = 0
441 − 168a + 672 + 7(a 2 + 16 − 8a) = 0 ⇒ (x − 2)(x + 1) = 0 ⇒ x = −1, 2
Hence, option (c) is correct.
7a 2 − 224a + 1225 = 0
Here, D > 0 14. If α, β, γ are the roots of
∴ Above quadratic equation have two distinct 1 1 1
real roots. f (x) = x 3 − 9 x 2 + 26 x − 24, then , , are
α β γ
∴ Number of possible value of a are 2
the roots of [21 Sep. 2020, Shift-I]
Hence, option (c) is correct.
(a) 24 x3 + 26 x2 + 9 x − 1 (b) 24 x3 − 26 x2 + 9 x − 1
12. If α, β are the roots of x 2 − 2 x + 4 = 0, for (c) 24 x3 + 26 x2 − 9 x − 1 (d) 24 x3 − 26 x2 + 9 x + 1
n ∈N, what is the value of α + β = n n
Sol. (b)
[18 Sep. 2020, Shift-II] It is given that roots of
nπ nπ
(a) 2 n + cos  
2
(b) 2 n + 1 cos   f (x) = x 3 − 9 x 2 + 26 x − 24 are α , β , γ , then the
 3  3 1 1 1
equation, where roots are , , . We can
nπ nπ
(c) 2 n + 1 cos   (d) 2 n + 2 cos   α β γ
 6  6 obtain by dividing f (x) by x 3, so required
Sol. (d) equation is 24 x 3 − 26 x 2 + 9 x −1.
Given, Hence, option (b) is correct.
α , β are roots of x 2 − 2x + 4 = 0
15. Let α and β be the roots of the equation
2± 4 − 4 ⋅1 ⋅ 4
x= px 2 + qx + r = 0 , p ≠ 0 . If p , q, r are in AP and
2⋅ 1
2 ± −12
1 1
x= + = 4, then the value of|α − β| is
2 α β
[21 Sep. 2020, Shift-II]
x =1± 3i
61 2 17 34 2 13
(a) (b) (c) (d)
x =1+ 3 i (or)1 − 3 i 9 9 9 9
28 AP EAMCET Chapterwise Mathematics

Sol. (d) 17. The number of solutions for the equation


Since, α and β are roots of quadratic equation x 2 − 5 | x| + 6 = 0 is ……… [22 Sep. 2020, Shift-I]
px 2 + qx + r = 0, p ≠ 0 and p , q , r are in AP such
(a) 4 (b) 3 (c) 2 (d) 1
that
1 1 Sol. (a)
+ =4 …(i)
α β Given equation
Let the p = q − d and r = q + d, where d is the x 2 − 5| x | + 6 = 0
common difference of AP ⇒ | x |2 − 5| x | + 6 = 0
−q
∴Sum of the roots α + β = ⇒ (| x | − 3) (| x | − 2) = 0
q−d
⇒ |x | = 2 or 3
q+ d
and product of the roots αβ = ⇒ x = ± 2, ± 3
q−d
∴ There are 4 solutions.
Q From Eq. (i), we have
α+β −q 18. If α , β , γ and δ are the roots of the equation
=4 ⇒ = 4 ⇒ 5q + 4 d = 0 …(ii)
αβ q+ d x 4 + 3 x 3 − 6 x 2 + 2 x − 4 = 0 , then find the
Now, |α − β|= (α + β)2 − 4αβ 1 1 1 1
equation having roots , , and
α β γ δ
q2 q+ d q 2 − 4(q 2 − d 2) 4d 2 − 3q 2
= −4 = = [22 Sep. 2020, Shift I]
(q − d)2 q−d (q − d)2 (q − d)2
(a) 4 x4 − 2 x3 + 6 x2 − 3 x − 1 = 0
2
4 − q  − 3q 2
5 25 (b) 4 x4 + 2 x3 − 6 x2 + 3 x + 1 = 0
−3
 4  4 2
= 2
= 2
= 13 (c) 4 x4 − 2 x3 + 6 x2 − 3 x + 1 = 0
 q + 5 q  9 9
(d) 4 x4 − 2 x3 + 6 x2 + 3 x − 1 = 0
   
 4   4
Hence, option (d) is correct.
Sol. (a)
It is given that the equation
16. If α , β are the roots of x 2 + px + q = 0 , then x 4 + 3x 3 − 6 x 2 + 2x − 4 = 0 having roots α, β, γ
the values of α 3 + β 3 and α 4 + α 2β 2 + β 4 are 1 1 1 1
and δ, then equation having roots , , and
α β γ δ
respectively …… and …… [22 Sep. 2020, Shift-I]
1
(a) (3 pq − p3 ) and ( p4 − 3 p2q + 3q 2 ) can be obtained by replacing x by in given
x
(b) − p(3q − p2 ) and ( p2 − q ) ( p2 + 3q ) equation, so required equation is
(c) ( pq − 4) and ( p4 − q 4 ) 4 x 4 − 2x 3 + 6 x 2 − 3x − 1 = 0
(d) (3 pq − p3 ) and ( p2 − q ) ( p2 − 3q )
19. If α , β are the roots of the equation
Sol. (d)
x 2 − 2 x + 4 = 0 , then α n + β n = ……
Since α and β are roots of equation
 nπ 
x 2 + px + q = 0, so × cos   for any n ∈N
 3 [22 Sep. 2020, Shift-I]
α + β = − p and αβ = q
As we know that, α 3 + β 3 = (α + β)(α 2 + β 2 − αβ) (a) 2 n (b) 2 n+ 1 (c) 2 n−1 (d) 2 n− 2

= (α + β) [(α + β)2 − 3 αβ] Sol. (b)


Given quadratic equation x 2 − 2x + 4 = 0 having
= (− p) [p 2 − 3q] = 3pq − p 3
roots α and β.
and α 4 + α 2β 2 + β 4 2 ± 4 − 16 2 ± 2 3i
So, α, β = = = 1 ± 3i
= (α 2 + β 2 )2 − α 2β 2 = [(α + β)2 − 2αβ]2 − (αβ)2 2 2
π π
= ((− p)2 − 2q)2 − q 2 = 2  cos + i sin  and
 3 3
= (p 2 − 2q)2 − q 2 = (p 2 − 2q − q) (p 2 − 2q + q)
 π π 
2  cos  −  + i sin  −  
= (p 2 − q) (p 2 − 3 q)   3  3
Theory of Equations 29

 π π
n
22. If the roots of x 3 − px 2 + qx − r = 0 are in AP.
∴ α n + β n = 2n   cos + i sin 
 3 3
 Then, [22 Sep. 2020, Shift-II]
π   (a) 2 p3 − 9 pq + 27 r = 0 (b) 2 p3 + 9 pq − 27 r = 0
n
 π
+  cos  −  + i sin  −   
  3  3   (c) 2 p3 − 8 pq + 27 r = 0 (d) 2 p3 − 9 pq + 28 r = 0

n  nπ  n+1  nπ  Sol. (a)
= 2 2 cos =2 cos 
 3   3 Let a − d , a , a + d are roots of
x 3 − px 2 + qx − r = 0 …(i)
20. If the roots of the equation b
x 3 − 6 x 2 + 11 x − 6 = 0 are α , β and γ. Then Sum of roots = −
a
the equation whose roots are α 2 , β 2 , γ 2 (− p)
a − d + a +a + d = −
among the following is ……… 1
[22 Sep. 2020, Shift-I] 3a = p
(a) x + 14 x + 49 x − 36 = 0
3 2
a=
p
(b) x3 − 14 x2 + 49 x − 36 = 0 3
p
(c) x3 − 14 x2 − 49 x + 36 = 0 Since, a = should be satisfied by given
3
(d) x3 − 14 x2 − 49 x − 36 = 0 equation.
Sol. (b) p
So, put x = in Eq. (i)
It is given that roots of the equation 3
3 2
x 3 − 6 x 2 + 11 x − 6 = 0 are α, β, γ.  p − P  p + q  p − r = 0
     
Now, to find the equation whose roots are  3  3  3
α 2, β 2, γ 2, put α 2 = x ⇒α = x. p 3 p 3 pq
− + −r=0
Since, α is the root of the given equation, so 27 9 3
x 3/ 2 − 6 x + 11 x1 / 2 − 6 = 0. p − 3p + 9pq − 27r = 0
3 3

⇒ x1 / 2(x + 11) = 6(x + 1) −2 p 3 + 9pq − 27r = 0


On squaring both sides, we get 2 p 3 − 9pq + 27r = 0
x(x + 11)2 = 36(x + 1)2 Hence, option (a) is correct.
⇒ x[x 2 + 22x + 121] = 36[x 2 + 2x + 1]
23. For which value of ‘k’, the roots of equation
⇒ x 3 + 22x 2 + 121 x = 36 x 2 + 72x + 36 2 x 2 + 5 x + k = 0 are rational?
⇒ x − 14 x + 49 x − 36 = 0
3 2
[22 Sep. 2020, Shift-II]
5 25 25 5
21. If α , β , γ are the roots of the equation (a) (b) (c) (d)
8 8 4 4
3 x 3 − 9 x 2 + 5 x − 7 , then what is the value of
Sol. (b)
α + β + γ? [22 Sep. 2020, Shift-II]
Given, quadratic equation is
(a) 3 (b) −3
2x 2 + 5x + k = 0
(c) 9 (d) −9
Quadratic equation have rational roots when D
Sol. (a) is perfect square,
(a) Given, D = b 2 − 4 ac = 25 − 4 ⋅ 2 k
α , β , γ are roots of 3x 3 − 9 x 2 + 5x − 7 = 0
D = 25 − 8 k
b Q a = 3
∴ α + β +γ = −  b = − 9 from options checking k =
25
makes D is
a   8
−(−9) perfect square
=
3 25
∴ k=
α+β+ γ=3 8
Hence, option (a) is correct. Hence, option (b) is correct.
30 AP EAMCET Chapterwise Mathematics

24. If the roots of the equation Consider,


b 3 Σαβ(α + β) = αβ (α + β) + βγ (β + γ) + γα(γ + α)
x 3 − ax 2 + bx − c = 0 are in GP, then = α 2β + αβ 2 + β 2γ + βγ 2 + γ 2α + γα 2
a3
[22 Sep. 2020, Shift-II] = (α + β + γ) (αβ + βγ + γα)
(a) 1 (b) −1 (c) c (d) − c = 2 × − 3= − 6
Hence, option (b) is correct.
Sol. (c)
p 27. If the bi-quadratic equation
Let roots be , p , pr
r f (x) = x 4 + 2 x 3 − 16 x 2 − 22 x + 7 = 0 has
p
∴ + P + pr = a … (i) 2 + 3 as one of its roots, then which of the
r
p2 following is not a root of f (x)?
+ P 2r + p 2 = b … (ii) [23 Sep. 2020, Shift-I]
r
p3 = c … (iii) (a) 3 − 2 (b) 2 − 3 (c) −3 + 2 (d) −3 − 2
Sol. (a)
p 2  + r + 1
1
Eq. (ii) r  b Given bi-quadratic equation,
⇒ ⇒ =
Eq. (i)  1  a f (x) = x 4 + 2x 3 − 16 x 2 − 22x + 7 = 0 has one of
p  + r + 1
r  the root is 2 + 3, then one more root will be
⇒ p=
b 2 − 3.
a Now equation whose roots are 2 + 3 and 2 − 3
3
 b
from Eq. (iii)   = c is
 a x 2 − 4 x + 1 = 0 and so, x 2 − 4 x + 1 is the factor of
b3 bi-quadratic equation
=c
a3 x 4 + 2x 3 − 16 x 2 − 22x + 7 = 0
Hence, option (c) is correct. ⇒ (x 2 − 4 x + 1)(x 2 + 6 x + 7) = 0
25. Which of the following is not a root of So, (x − 4 x + 1) and (x 2 + 6 x + 7) are factors of
2

f (x) = x − 11 x + 36 x − 36?
3 2
given bi-quadratic equation.
[23 Sep. 2020, Shift-I] Now, the roots of quadratic equation
(a) 2 (b) 4 (c) 3 (d) 6 −6 ± 36 − 28
x 2 + 6 x + 7 = 0 are x = = − 3± 2
Sol. (c) 2
∴ The roots of given bi-quadratic equations are
Given, f (x) = x 3 − 11 x 2 + 36 x − 36
2 + 3, 2 − 3, − 3 + 2 and −3 + 2.
By option verification, x = 4 is not a root of
Hence, option (a) is correct.
f (x) = 0
i.e., f(4) = (4)3 − 11(4)2 + 36(4) − 36 28. If α and β are the roots of x 2 + 7 x + 3 = 0 and
f(4) = 4 ≠ 0 2α 2β
, are the roots of
∴ f(4) ≠ 0 3 − 4α 3 − 4β
∴ x = 4 is not a root of f (x) = 0. ax 2 + bx + c = 0 and GCD of a , b, c is 1, then
26. If α , β , γ are the roots of a + b+ c= [20 April 2019, Shift-I]
x 3 − 2 x 2 + 3 x − 4 = 0 , then find Σα β (α + β) (a) 11 (b) 0
[23 Sep. 2020, Shift-I] (c) 243 (d) 81
(a) −2 (b) −6 (c) 6 (d) 2 Sol. (d)

Sol. (b) Let = y ⇒ 2α = 3y − 4αy
3 − 4α
Given, α , β, γ are roots of x 3 − 2x 2 + 3x − 4 = 0
3y
∴ α+β+ γ=2 ⇒α(2 + 4 y) = 3y ⇒α =
2 + 4y
αβ + βγ + γα = − 3
αβγ = 4 Qα is root of quadratic equation x 2 + 7 x + 3 = 0,
Theory of Equations 31
2
 3y   3y  30. Let a , b and c be the sides of a scalane
So,   + 7  + 3= 0
 2 + 4y  2 + 4y triangle. If λ is a real number such that the
⇒ 9 y 2 + 84 y 2 + 42y + 48 y 2 + 48 y + 12 = 0 roots of the equation
⇒ 141 y 2 + 90 y + 12 = 0 x 2 + 2(a + b + c) x + 3 λ (ab + bc + ca) = 0 are
⇒ 47 y 2 + 30 y + 4 = 0 real, then the interval in which λ lies is
2α [20 April 2019, Shift-I]
Q y= is root of quadratic equation
(a)  −∞,  (b)  , ∞  (c)  ,  (d)  , ∞ 
3 − 4α 4 5 1 5 4
 3 3   3 3 3 
ax 2 + bx + c = 0.
∴ a = 47, b = 30 and c = 4 and GCD of 47, 30, 4 Sol. (a)
is 1. It is given that roots of given quadratic equation
∴ a + b + c = 47 + 30 + 4 = 81 x 2 + 2(a + b + c) x + 3λ(ab + bc + ca) = 0 are real,
Hence, option (d) is correct. so D≥ 0
⇒ 4(a + b + c)2 − 4 × 3λ(ab + bc + ca) ≥ 0
29. If α , β are the roots of x 2 + bx + c = 0, γ, δ are
⇒ (a + b + c)2 − 3λ(ab + bc + ca) ≥ 0
the roots of x 2 + b1 x + c1 = 0 and
(a + b + c)2
γ < α < δ < β, then (c − c1)2 < ⇒ λ≤
3(ab + bc + ca)
[20 April 2019, Shift-I]
Now, for scalane triangle.
(a) (b1 − b ) (bc1 − b1c ) (b) 1
Q For ∆ ABC|b − c|< a ,|c − a|< b and |a − b|< c
(c) (b − b1 )2 (d) (c − c1 )(b1c − b1c1 )
⇒(b − c)2 + (c − a)2 + (a − b)2 < a 2 + b 2 + c 2
Sol. (a) ⇒ a 2 + b 2 + c 2 < 2(ab + bc + ca)
According to given informations after drawing
figure. ⇒ a 2 + b 2 + c 2 + 2(ab + bc + ca) < 4(ab + bc + ca)
x2 +b1x+c1=0 x 2 +bx+c=0 (a + b + c)2 4
⇒ <
3(ab + bc + ca) 3
4
∴λ <
γ α δ β 3
X–axis
Hence, option (a) is correct.
P
31. The polynomial equation of degree 4 having
aλ.γ<α<δ<β
real coefficients with three of its roots as
For x-coordinate of point P, on subtracting given 2 ± 3 and 1 + 2 i, is [20 April 2019, Shift-I]
quadratic equations, we get
x 2 + b1 x + c1 = 0
(a) x4 − 6 x3 − 14 x2 + 22 x + 5 = 0
(b) x4 − 6 x3 − 19 x + 22 x − 5 = 0
x 2 + bx + c = 0
(c) x4 − 6 x3 + 19 x − 22 x + 5 = 0
 c − c1 
(b1 − b) x + (c1 − c) = 0 ⇒ x =   (d) x4 − 6 x3 + 14 x2 − 22 x + 5 = 0
 b1 − b 
Now, with respect to quadratic expression Sol. (d)
f (x) = x 2 + bx + c It is given that, the polynomial equation of
 degree 4 having real coefficients with three of
c − c1 
fx = <0 its roots as 2 ± 3 and 1 + 2i, so the remaining
 b1 − b 
2
root is1 − 2i. Now, the quadratic equation
 c − c1   c − c1  whose roots as 2 ± 3 is
⇒  + b  + c<0
 1
b − b   b1 − b  x 2 − 4 x + 1 = 0, and
⇒(c − c1) < b(c1 − c)(b1 − b) − c(b1 − b)
2 2
the quadratic equation whose roots as1 ± 2i, is
⇒(c − c1)2 < (b1 − b)[bc1 − bc − cb1 + cb] x 2 − 2x + 5 = 0
⇒(c − c1)2 < (b1 − b)(bc1 − cb1) So, the required polynomial equation is
Hence, option (a) is correct. (x 2 − 4 x + 1)(x 2 − 2x + 5) = 0
32 AP EAMCET Chapterwise Mathematics

⇒ x 4 − 6 x 3 + 14 x 2 − 22x + 5 = 0 1    
⇒ 3 x +  2 x +  − 5 − 5 2 x +
1 1
 − 5 = 0
Hence, option (d) is correct.  x    x    x 
    
2 x +  − 5 3  x +
1 1
32. If α , β , γ are the roots of the equation ⇒  − 5 = 0
  x    x 
x 3 − ax 2 + bx − c = 0 , then Σ α 2 (β + γ) =
⇒ Either 2 x +  = 5 or 3 x +  = 5
1 1
[20 April 2019, Shift-II]  x  x
ab − 3c b 2 − 2 ac a2 − 2 b ⇒ 2x 2 − 5x + 2 = 0 or 3x 2 − 5x + 3 = 0
(a) ab − 3c (b) (c) (d)
c c2 c2 ⇒ 2x 2 − 4 x − x + 2 = 0
Sol. (a) Q 3x 2 − 5x + 3 = 0 has no solution because D < 0.
If α , β , γ are the roots of the equation. ⇒ 2x(x − 2) − 1(x − 2) = 0
x 3 − ax 2 + bx − c = 0 1
⇒ x = 2,
Then, α + β + γ = a, αβ + βγ + γα = b 2
and αβγ = c. So, sum of all the rational roots
Now, Σα 2(β + γ) = Σα 2(a − α) [as α + β + γ = a] 1
λ = 1 − 1 + 2 + = 2. 5
2
= a Σα 2 − Σα 3
Hence, option (d) is correct.
= a[(α + β + γ)2 − 2(αβ + βγ + γα)] − [(α + β + γ)
(α 2 + β 2 + γ 2 − αβ − βγ − γα) + 3αβγ] 34. If α satisfies the equation
x 2x + 1
= a[(α + β + γ) − 2(αβ + βγ + γα)] − (α + β + γ)
2
+ = 2, then the roots of the
2x + 1 x
{(α + β + γ)2 − 3(αβ + βγ + γα) + 3αβγ]
equation α 2 x 2 + 4 αx + 3 = 0 are
= a[a − 2b] − [a {a 2 − 3b} + 3c]
2
[21 April 2019, Shift-I]
= a 3 − 2ab − a 3 + 3ab − 3c = ab − 3c
(a) 1, 3 (b) − 1, 1 (c) 2, − 3 (d) 3, 4
Hence, option (a) is correct.
Sol. (a)
33. The sum of all the rational roots of the x 2x + 1
+ =2 … (i)
equation 2x + 1 x
6 x 6 − 25 x 5 + 31 x 4 − 31 x 2 + 25 x − 6 = 0 is
x
[20 April 2019, Shift-II] Let = y
2x + 1
25
(a) 3 (b) 3.5 (c) (d) 2.5 1
6 Then, y+ =2
y
Sol. (d)
⇒ y 2 − 2y + 1 = 0 ⇒ (y − 1) (y − 1) = 0
Given, equation
6 x 6 − 25x 5 + 31 x 4 − 31 x 2 + 25x − 6 = 0 ⇒ (y − 1)2 = 0 ⇒ y −1 = 0

⇒ 6(x 6 − 1) − 25x(x 4 − 1) + 31 x 2(x 2 − 1) = 0 ⇒ y =1 ⇒ y =1


x x
⇒(x − 1)[6(x + x + 1) − 25x(x + 1) + 31 x ] = 0
2 4 2 2 2 ⇒ =1 ⇒ =1
2x + 1 2x + 1
⇒ (x − 1)[6 x − 25x + 37 x − 25x + 6] = 0
2 4 3 2
⇒ 2x + 1 = x ⇒ x = −1
Either x 2 − 1 = 0 ⇒ x = + 1, − 1 Qα satisfies the Eq. (i), therefore α = − 1
or 6 x 4 − 25x 3 + 37 x 2 − 25x + 6 = 0 Now, α 2 x 2 + 4αx + 3 = 0
⇒ 6(x 4 + 1) − 25x(x 2 + 1) + 37 x 2 = 0 Put the value of α
(− 1)2 x 2 + 4(− 1) x + 3 = 0
6 x 2 + 2  − 25 x +  + 37 = 0
1 1

 x   x x2 − 4x + 3 = 0
2
6  x +  − 25 x +  + 25 = 0 (x − 1) (x − 3) = 0
1 1

 x  x x = 1, 3
2
Hence, the roots of the equation
⇒ 6  x +  − 15 x +  − 10 x +  + 25 = 0
1 1 1
 x  x  x α 2 x 2 + 4αx + 3 = 0 are 1, 3.
Theory of Equations 33

35. If α , β , γ are the roots of 37. Let θ be an acute angle such that the
x − 6 x + 11 x − 6 = 0 , then the equation
3 2 equation x 3 + 4 x 2 cos θ + x cot θ = 0 has
having the roots α 2 + β 2 , β 2 + γ 2 and γ 2 + α 2 multiple roots. Then the value of θ (in
radians) is [21 April 2019, Shift-II]
is [21 April 2019, Shift-I]
π π π 5π π 5π
(a) x3 − 28 x2 + 245 x − 650 = 0 (a) (b) (c) or (d) or
3 8 12 12 6 12
(b) x3 − 28 x2 + 245 x + 650 = 0
Sol. (c)
(c) x3 + 28 x2 − 245 x − 650 = 0
Given,
(d) x3 + 28 x2 + 245 x − 650 = 0 x 3 + 4 x 2 cosθ + x cot θ = 0
Sol. (a) ⇒ x(x 2 + 4 x cosθ + cot θ) = 0
x − 6 x + 11 x − 6 = 0
3 2
…(i) ⇒ x = 0 is root of equation.
⇒ (x − 1) (x − 2) (x − 3) = 0 ⇒ x = 1, 2, 3 ⇒ x 2 + 4 x cosθ + cot θ = 0 has multiple roots
Qα , β , γ are the roots of the Eq.(i), so So, ∆=0
α = 1, β = 2 , γ = 3 ⇒ (4 cosθ)2 − 4()(cot
1 θ) = 0
Therefore, α 2 + β 2 = ()1 2 + (2)2 = 5 = α ′ (say) ⇒ 16 cos2 θ = 4 cot θ
β + γ = (2) + (3) = 13 = β ′ (say)
2 2 2 2
cosθ
⇒ 4 cos2 θ = cot θ ⇒ 4 cos2 θ =
and γ 2 + α 2 = (3)2 + 1 = 10 = γ ′ (say) sinθ
1
Equation of the having the roots α′ , β′ and γ ′, ⇒ cosθ = 0 as 4 cosθ =
sinθ
x 3 − (α ′ + β ′ + γ ′) x 2 + (α ′β ′ + β ′ γ ′ + γ ′α ′) x 1 1
⇒ 2sinθ cosθ = ⇒sin 2θ =
− α ′β ′ γ ′ = 0 2 2
⇒ x 3 − (5 + 13 + 10) x 2 + (5 × 13 + 13 × 10 + 10 × 5) x π 5π π 5π
⇒ 2θ = or ⇒θ = or
− 5 × 13 × 10 = 0 6 6 12 12
⇒ x 3 − 28 x 2 + 245x − 650 = 0
38. If 3 x 2 − 7 x + 2 = 0 and 15 x 2 − 11x + a = 0
36. If α , β are the real roots of x 2 + px + q = 0 have a common root and a is a positive real
and α , β are the roots of x − rx + s = 0 ,
4 4 2 number, then the sum of the roots of the
equation 15 x 2 − ax + 7 = 0 , is
then the equation x 2 − 4 qx + 2 q2 − r = 0 has
[22 April 2019, Shift-I]
always [21 April 2019, Shift-II] 76 38 2 36
(a) two positive roots (a) (b) (c) (d)
15 15 15 15
(b) two negative roots
(c) one positive root and one negative root Sol. (c)
(d) two real roots Given equations are
Sol. (d) 3x 2 − 7 x + 2 = 0 …(i)
Given, α , β are roots of x + px + q = 0.
2 and 15x − 11 x + a = 0
2
…(ii)
∴ α + β = −p Here, a1 = 3, b1 = − 7, c1 = 2
and αβ = q a 2 =15, b2 = − 11, c 2 = a
Since, α 4 , β 4 are roots of x 2 − rx + s = 0 Let α is a common root of the Eqs. (i) and (ii),
Then, α will satisfy both the equations.
∴ α 4 + β 4 = r and α 4β 4 = s ∴Common root is given by
Now, x 2 − 4qx + 2q 2 − r (2 × 15 − a × 3)2 = (−7a + 22)(−33 + 105)
D = (4q)2 − 4(2q 2 − r) ⇒ (30 − 3a)2 = (22 − 7a)(72)
= 16q 2 − 8q 2 + 4r = 8q 2 + 4r ⇒ ( − a)2] = (22 − 7a)(72)
[910
Here, r = α 4 + β 4 ≥ 0 and 8q 2 ≥ 0 ⇒ (10 − a)2 = (22 − 7a)8
So, D ≥ 0 ⇒ 100 + a 2 − 20a = 176 − 56a
So, given equation has two real roots. ⇒ a 2 + 36a − 76 = 0
34 AP EAMCET Chapterwise Mathematics

⇒ a 2 + 38a − 2a − 76 = 0 Also, given that a , b and c are roots of cubic


⇒ a(a + 38) − 2(a + 38) = 0 equation x 3 − 3x + λ = 0.
⇒ (a + 38)(a − 2) = 0 ab + bc + ca = − 3 … (i)
⇒ a =2 [Q a > 0] a + b + c =0 … (ii)
Now, for equation15x 2 − ax + 7 = 0
and abc = − λ … (iii)
– Coefficient of x Squaring Eq. (ii), we get, (a + b + c)2 = 0
Sum of roots =
Coefficient of x 2
−(− a) a 2 a 2 + b 2 + c 2 + 2(ab + bc + ca) = 0
= = =
15 15 15 a2 + b2 + c 2 = 6

39. Let α ,β be the roots of the equation Similarly, a 2b 2 + b 2c 2 + c 2a 2 = 9 (by solving)

x 2 −|a| x − |b| = 0 such that|α | < |β| . If Put these values in φ(a) φ(b) φ(c) , we get
−λ
|a| < β −1, then the positive root of
 x2 (1 + 0 − 3 − λ)(1 +`6 + 9 + λ 2)
log|α|  2  − 1 = 0 , is −λ λ
β  [22 April 2019, Shift-I] =
(−2 − λ)(λ 2 + 16) (λ + 2)(λ 2 + 16)
(a) < |α| (b) < α (c) < β (d) > β
Sol. (d) 41. The greatest real root of the equation
x 2 6 x 4 − 35 x 3 + 62 x 2 − 35 x + 6 = 0 is
Given, log|α|  2  = 1 [22 April 2019, Shift-II]
β 
5 7
x2 (a) 2 (b) (c) 3 (d)
⇒ =|α|⇒ x 2 = β 2|α| ...(i) 2 2
β2
Sol. (c)
and α , β are roots of equation
Given equation is
x 2 −|a|x −|b|= 0, so, α + β = −|a|< 0
6 x 4 − 35x 3 + 62x 2 − 35x + 6 = 0
so, β < 0 as |α|<|β|
⇒ x 2 6 x 2 + 2  − 35 x +  + 62 = 0
1 1
From Eq. (i), we get x = ± |β||α|1 / 2   
 x x 
So, the positive root is 1
Let x + = y, therefore,
|β||α|1 / 2 > β [as β < 0] x
x (6(y 2 − 2) − 35y + 62) = 0
40. Let φ(x) = . If a , b and c are the
(x +1)(x +1)
2 ⇒ (6 y 2 − 12 − 35y + 62) = 0

roots of the equation x 3 − 3 x + λ = 0 , (λ ≠ 0). ⇒ (6 y 2 − 35y + 50 ) = 0


Then, φ(a) φ(b) φ(c) = [22 April 2019, Shift-I] ⇒ (6 y − 15y − 20 y + 50 ) = 0
2

−λ ⇒ (2y − 5) (3y − 10 ) = 0
(a) λ (b)
(λ + 2 ) (λ 2 + 16) 1
Now, put back y = x + and multiply each
λ λ x
(c) (d) factors by x, we get
(λ + 2 ) (λ + 2 ) (λ 2 + 16)
⇒ (2x 2 − 5x + 2) (3x 2 − 10 x + 3) = 0
Sol. (d)
⇒ (x − 2)(2x − 1)(x − 3)(3x − 1) = 0
x
Given, φ(x) = 1 1
(x + 1)(x + 1)
2 ⇒ x = 2, , 3,
2 3
abc Hence, greatest value is 3.
∴φ(a) φ(b) φ(c) =
(1 + a)(1 + b)(1 + c)(1 + a 2)(1 + b 2)(1 + c 2)
42. If the product of the roots of the equation
abc
= x 2 + 4 kx + 12 e 3 log k − 1 = 0 , (k > 0) is 323, then
(1 + a + b + c + ab + bc + ca + abc)(1 + a 2 + b 2
the sum of its roots is [23 April 2019, Shift-I]
+ c 2 + a 2b 2 + b 2c 2 + c 2a 2 + (abc)2)
(a) 9k (b) 12 (c) −12 (d) −16k
Theory of Equations 35

Sol. (c) ⇒ a 2 + b 2 + c 2 = − 2q …(i)


Given quadratic equation is Now, (a − b)2 + (b − c)2 + (c − a)2
x 2 + 4kx + 12e 3log k − 1 = 0 = a 2 + b 2 − 2ab + b 2 + c 2 − 2bc + c 2 + a 2 − 2ca
According to given information ,
= 2a 2 + 2b 2 + 2c 2 − 2(ab + bc + ca)
Product of the roots = 323
= 2(a 2 + b 2 + c 2) − 2(ab + bc + ca)
⇒ 12e 3log k −1 = 323
= 2(− 2q) − 2(q) [by Eq. (i)]
⇒ 12k3 = 324 ⇒ k3 = 27 ⇒ k = 3
= − 4q − 2q = − 6q.
So, sum of roots = − 4k = − 12
Hence, option (c) is correct. 45. If the sum of two roots of the equation
x 3 − 2 px 2 + 3 qx − 4 r = 0 is zero, then the
43. Let a , b, c , d ∈ R. If the equations
value of r is [22 April 2018, Shift-I]
2 bx 2 + 3 cx − d = 0 and 2 ax 2 + 3 bx + 4 c = 0 3 pq 3 pq
(a) (b) (c) pq (d) 2 pq
have a common root and 2 4
4 bc + ad bd + 4 c 2 Sol. (a)
= , then k =
k(b2 − ac) 4 bc + ad Let the two roots of equation is m, − m.
[23 April 2019, Shift-I] Now, sum of three roots = 2p
9 2 1 1 Hence, third root will be 2p.
(a) (b) (c) (d)
2 9 9 3 Now, m × (− m) + m × (2p) + (− m) × 2p = 3q
Sol. (a) ⇒ − m2 = 3q
It is given that the given quadratic equations Now, m × (− m) × 2p = 4r
2bx 2 + 3cx − d = 0 and ⇒ 3q × 2p = 4r
3pq × 2 3pq
2ax 2 + 3bx + 4c = 0 have a common root, ⇒ r= ⇒ r=
4 2
(6b − 6ac)(12c + 3bd) = (8bc + 2ad)
2 2 2

46. If α and β are the roots of the equation


Then, 6(b 2 − ac) 3(4c 2 + bd) = 4(4bc + ad)2
x 2 − 4 x + 5 = 0 , then the quadratic equation
4bc + ad bd + 4c 2
⇒ = whose roots are α 2 + β and α + β 2 is
(b − ac) 4bc + ad
9 2
2 [22 April 2018, Shift-II]
9 (a) x + 10 x + 34 = 0
2
(b) x2 − 10 x + 34 = 0
On comparing, we get k =
2 (c) x − 10 x − 34 = 0
2
(d) x2 + 10 x − 34 = 0
Hence, option (a) is correct.
Sol. (b)
44. If a, b and c are the roots of x 3 + qx + r = 0, Since, α and β are roots of the quadratic
then (a − b)2 + (b − c)2 + (c − a)2 = equation
[22 April 2018, Shift-I] x2 − 4x + 5 = 0
(a) −6q (b) −4q (c) 6q (d) 4q So, α + β = 4 and αβ = 5 …(i)
Now, (α 2 + β) + (α + β 2) = (α 2 + β 2) + (α + β)
Sol. (a)
Given, a , b and c are the roots of equation = (α + β)2− 2αβ + (α + β)
x 3 + qx + r = 0, = 16 − 10 + 4 = 10
∴ a+ b+ c=0 and (α 2 + β) (α + β 2) = α 3+ α 2β 2 + βα + β 3
ab + bc + ca = q = α 3 + β 3+ αβ(αβ + 1)
and abc = − r = (α + β) (α 2 + β 2 − αβ) + αβ (αβ + 1)
Q a+ b+ c=0
= (α + β) [(α + β)2 − 3αβ] + αβ (αβ + 1)
⇒ (a + b + c)2 = 0
= 4 [16 − 15] + 5 (5 + 1)
⇒ a 2 + b 2 + c 2 + 2ab + 2bc + 2ca = 0
= 4 + 30 = 34
⇒ a 2 + b 2 + c 2 + 2(ab + bc + ca) = 0 So, the quadratic equation whose roots are
36 AP EAMCET Chapterwise Mathematics

(α 2 + β) and (α + β 2) is are real, so


2 [2(a + b + c)]2 − 41
() [3λ (ab + bc + ca)] ≥ 0
x − (α + β + α + β ) x + (α + β) (α + β ) = 0
2 2 2 2

a 2 + b 2 + c 2 + 2(ab + bc + ca)
⇒ x 2 − 10 x + 34 = 0 ⇒ λ≤
3(ab + bc + ca)
47. If a , b, c ∈ R are such that 4 a + 2b + c > 0 and a2 + b2 + c 2 2
⇒ λ≤ + … (i)
ax + bx + c = 0 has no real roots, then the
2
3(ab + bc + ca) 3
value of (c + a) (c + b) is [22 April 2018, Shift-II] Now, sum of two roots is greater than thirds
(a) greater than ab (b) less than bc So, a < b + c ⇒(a − c) < b
(c) greater than ca (d) less than ab + bc + ca ⇒ (a − c)2 < b 2
Sol. (a) ⇒ a 2 + c 2 − 2ac < b 2 …(ii)
Since, the equation ax + bx + c = 0 have no real
2
Similarly, a 2 + b 2 − 2ab < c 2 …(iii)
roots and 4a + 2b + c > 0 and c 2 + b 2 − 2bc < a 2 …(iv)
∴ a+ b+ c>0 Adding Eqs. (ii), (iii) and (iv), we get
{Q ax 2 + bx + c > 0, ∀ x ∈ R}
a 2 + b 2 + c 2 < 2 (ab + bc + ca)
So, c + a > − b and c + b > − a
a2 + b2 + c 2
⇒ (c + a)(c + b) > ab ⇒ <2
ab + bc + ca
48. The minimum degree of a polynomial a2 + b2 + c 2
equation with rational coefficients having ⇒ 2> …(v)
ab + bc + ca
3 + 27 , 2 + 5 i as two of its roots is [from Eqs. (i) and (v)]
[22 April 2018, Shift-II] 2 2 4
λ< + ⇒ λ<
(a) 8 (b) 6 3 3 3
(c) 4 (d) 2 50. If a , b and c are the roots of x 3 + 4 x + 1 = 0,
Sol. (b) 1 1 1
The polynomial equation with rational then + + =
a + b b+ c c+ a
coefficients having 3 + 27 , 2 + 5i as two
[23 April 2018, Shift-I]
roots. Then, other roots are
(a) 2 (b) 3 (c) 4 (d) − 4
3 − 27 and 2 − 5i
Sol. (c)
− 3+ 27 and − 3 − 27
a , b , c are roots of x 3 + 4 x + 1 = 0
Therefore, number of roots are 6 and the
⇒ x3 + 0 ⋅ x2 + 4x + 1 = 0
minimum degree of a polynomial equation
having 6 distinct roots is 6. So, a + b + c = 0 ⇒ abc = −1
and ab + bc + ca = 4
49. Let a , b and c be three positive real numbers 1 1 1 1 1 1
Now, + + = + +
such that the sum of any two of them is a+ b b+ c c+ a −c −a −b
greater than the third.
bc + ac + ab   4 
= −  + +  = − 
1 1 1
All the values of λ such that the roots of the =− =4
 a b c  abc   − 1
equation
x 2 + 2(a + b + c) x + 3 λ (ab + bc + ca) = 0 are 51. If the sum of any two roots of the equation
real, are given by [23 April 2018, Shift-I] x 3 + px 2 + qx + r = 0 is zero, then
2 2 [23 April 2018, Shift-I]
(a) λ < (b) λ ≥
3 3 (a) r = pq (b) pq 2 = r
4 1 2
(c) λ < (d) < λ< (c) r 2 = pq (d) pqr = 1
3 3 3
Sol. (c) Sol. (a)
Roots of the equation Given equation, x 3 + px 2 + qx + r
x 2 + 2(a + b + c) x + 3λ (ab + bc + ca) = 0 Let roots of given equation are α , β and γ
Theory of Equations 37

α+β=0 …(i) 53. If [ x ] denotes the greatest integer not


Now, α+β+ γ=−p …(ii)
exceeding x , then the values of x satisfying
αβ + βγ + γα = + q …(iii)
[ x ]2 − 7 [ x ] + 12 ≤ 0 are
and αβγ = − r …(iv)
[23 April 2018, Shift-II]
[From Eqs. (i) and (ii)]
γ=−p (a) 1 ≤ x < 4 (b) 3 ≤ x < 5
Substituting this value in Eq. (iv) (c) −5 < x ≤ − 3 (d) 2 ≤ x ≤ 4
αβ (− p) = − r Sol. (b)
r Given equation,
⇒ αβ =
p [x]2 − 7[x] + 12 ≤ 0
Substituting this value in Eq. (iii) ⇒ [x]2 − 4[x] − 3[x] + 12 ≤ 0
r
+ r (α + β) = + q ⇒ ([x] − 4) ([x] − 3) ≤ 0 ⇒ [x] ∈ [3,4]
p
r
=+ q [Q α + β = 0] ⇒ x ∈[3, 5).
p 54. If the equation whose roots are p times the
r = pq
roots of x 4 − 2 ax 3 + 4 bx 2 + 8 ax + 16 = 0 is a
52. If the quadratic equation formed by reciprocal equation, then p =
eliminating x from x 2 + αx + β = 0 and [23 April 2018, Shift-II]
xy + l(x + y) + m = 0 has the same roots as 1 1
(a) 3 (b) 2 (c) (d)
that of the given quadratic equation, then 2 3
the set of values of β is [23 April 2018, Shift-II]
Sol. (c)
(a) {m, αl − m} (b) {m, l + m}
Let the roots are α , β , γ , δ for the equation
(c) {m, αl + m} (d) {m, l − m}
x 4 − 2ax 3 + 4bx 2 + 8ax + 16 = 0
Sol. (a)
So, α × β × γ × δ = 16 …(i)
The new quadratic equation formed by after
For the reciprocal equation roots are pα, pβ, pγ
elimination of ‘x’ is,
2
and pδ, so product of roots = 1
 ly + m   ly + m  ⇒ p 4αβγδ = 1 …(ii)
  − α  +β=0
 y+ l   y+ l  From Eqs. (i) and (ii), we are getting
⇒(l 2 y 2 + m2 + 2lmy) − α(ly + m) (y + l) |p|= 1 / 2
+ β(y + l)2 = 0 55. If the quadratic equation
⇒(l − αl + β) y + (2lm − αl − αm + 2βl)
2 2 2 2
α  1
4 sec . x 2 + 2 x + β 2 − β +  = 0 has real
y + (m − αml + βl ) = 0
2 2
…(i)  2
Q Eq. (i) and x 2 + αx + β = 0 has same roots, so roots, then the value of cos 2 α + cos −1 β is
[24 April 2018, Shift-I]
l 2 − αl + β 2lm − αl 2 − αm + 2βl
= π π π π
1 α (a) (b) + 1 (c) (d) −1
3 3 2 2
m2 − αml + βl 2
= Sol. (b)
β
The quadratic equation,
So, βl 2 − βαl + β 2 = m2 − αml + βl 2
4sec α x 2 + 2x +  β 2 − β +  = 0 have real roots
2 1
⇒ β 2 − β(lα) − m(m − αl) = 0  2
⇒ β 2 − βm + β(m − αl) − m(m − αl) = 0 ⇒ discriminant = 4 − 4 ⋅ 4sec α  β 2 − β +  ≥ 0
2 1
 2
⇒ β(β − m) + (m − αl) (β − m) = 0
2  1 
⇒ β = m, αl − m ⇒ 4sec α  β 2 − β +  ≤ 1
 2
38 AP EAMCET Chapterwise Mathematics

1 
2
3± 5 1± i 3
=  β −  + ≥
2
α 1 1 1 1 1
But 4sec ≥ 4, β 2 − β + (a) − 1, (b) (c) (d) 1, −
2  2 4 4 3 2 2 3
So, the equation will be satisfied only Sol. (a)
2 1 1
When 4sec α = 4 and β 2 − β + = Given equation,
2 4 x 5 − 5x 4 + 9 x 3 − 9 x 2 + 5x − 1 = 0
2
sec2 α = 1 and  β −  = 0
1 x = 1 is one root of equation.
 2
So, (x − 1) (x 4 − 4 x 3 + 5x 2 − 4 x + 1) = 0
1
cos2 α = 1 and β =
2 ⇒ x 4 − 4 x 3 + 5x 2 − 4 x + 1 = 0
∴ cos2 α + cos− 1 β = 1 + cos− 1 (1 / 2) = 1 + π / 3 On dividing by x 2, we get,
4 1
⇒ x2 − 4x + 5 − + 2 = 0
56. The sum of the non-real roots of x x
(p 2 + p − 3) (p 2 + p − 2) − 12 = 0 is ⇒  x 2 + 2  − 4 x +  + 5 = 0
1 1
[24 April 2018, Shift-I]  x   x
2
(a) 1 (b) − 1  x + 1  − 4 x + 1  + 3 = 0
⇒    
(c) 6 (d) − 6  x  x
Sol. (b)  1  1
2 
 as x + 2 =  x +  − 2
2
Given equation is x x
 
(P 2 + P − 3)(P 2 + P − 2) − 12 = 0 1
Now, let x + = y
Let y = P 2 + P − 2 x
∴ y(y − 1) − 12 = 0 y2 − 4 y + 3 = 0
⇒ y 2 − y − 12 = 0 (y − 1) (y − 3) = 0 ⇒ y = 1, 3
⇒ (y − 4)(y + 3) = 0 ⇒
1
x + = 1 and x + = 3 .
1
⇒ y − 4 = 0 as y + 3 = 0 x x
⇒ P 2 + P − 6 = 0 as P 2 + P + 1 = 0 ⇒ x 2 + 1 = x and x 2 + 1 = 3x
For imaginary (non-real roots) ⇒ x 2 − x + 1 = 0 and x 2 − 3x + 1 = 0
P2 + P + 1 = 0
Equation x 2 − 3x + 1 = 0 gives irrational roots
∴ Sum of non-real roots = −1.
Let α , β are roots then,
57. If α, β are the irrational roots of the equation α + β = 3, αβ = 1 putting these values in
x 5 − 5 x 4 + 9 x 3 − 9 x 2 + 5 x − 1 = 0, then the (α + β) x 2 + 2αβx − αβ = 0
roots of the equation ⇒ 3x 2 + 2x − 1 = 0
(α + β) x 2 + 2αβx − αβ = 0 are ⇒ (3x − 1) (x + 1) = 0
[24 April 2018, Shift-I] ⇒ x = −1, 1 / 3
4
Functions
1. Find the domain of the real valued function Sol. (d)
−1 / 2 f : Z→ Z
f (x) = ([ x ] − [ x ] − 2) , where [ ⋅ ] is the
2

greatest integer function. [17 Sep. 2020, Shift-I] f (x + y) = f (x) + f (y); x , y ∈ Z


(a) R − (−1, 3] (b) R − [−1, 3) ∴ f (x) = kx
(c) R − (−1, 3) (d) R − [−1, 3] So, there are infinitely many bijections.

Sol. (a) 4. The function f : R → R defined by


−1 / 2
Let y = f (x) = ([x] − [x] − 2)
2
x
f (x) = is ........
1 1 + x2
⇒ y2 = [17 Sep. 2020, Shift-II]
[x]2 − [x] − 2
(a) Surjective but not injective
For real valued (b) Bijective
[x]2 − [x] − 2 > 0 (c) Injective but not surjective
⇒ {[x] − 2}{[x] + 1} > 0 (d) Neither injective nor surjective
[x] ∈ R − (−1, 2) Sol. (c)
So, x ∈ R − (−1, 3] x
f (x) =
2. Let a , b, c be real numbers. A set formed with 1 + x2
a , b, c whose order of occurrence is x1 x2
f (x1) = f (x)2 ⇒ =
preassigned is called ………… 1+ x12 1 + x 22
(a) An ordered triad [17 Sep. 2020, Shift-I] ⇒ x12 (1 + x 22) = x 22(1 + x12)
(b) An ordered pair ⇒ x12 = x 22
(c) Both ordered triad and ordered pair
(d) None of these options are correct x1 = x 2
∴F is injective in nature
Sol. (a) x
A set formed with real numbers a , b , c whose Also, Let f (x) = y =
order of occurrence is preassigned is called an 1 + x2
ordered triad. ⇒ y 2(1 + x 2) = x 2
3. How many bijections f : Z → Z are there ⇒
y2
= x2
such that f (x + y) = f (x) + f (y) for all 1 − y2
x, y ∈ Z ? [17 Sep. 2020, Shift-I] y
⇒ x=
(a) One 1 − y2
(b) Two
(c) Three As y 2 ≤ 1 ⇒ −1 ≤ |y|≤ 1
(d) Infinitely many So, f is non surjective.
40 AP EAMCET Chapterwise Mathematics

5. How many functions f : Z → Z are there such x6


8. If f : R → R is defined as f (x) = ,
that f (x + y) = f (x) + f (y) for all x, y ∈ Z? x + 2020 6

[17 Sep. 2020, Shift-II] ∀ x ∈ R, then the range of f is .......


(a) 1 (b) 2 [18 Sep. 2020, Shift-II]
(c) 3 (d) Infinitely many
(d)  0,
1 
(b) [0, ∞ )
 2020 
(a) [0, 1] (c) [0, 1)
Sol. (d)
F: z → z
Sol. (c)
f (x + y) = f (x) + f (y)
We have,
As sum of z integers always gives an integer so,
x 6 + 2020 > x 6
there is no restriction and hence infinite such
functions are possible. x6
⇒ <1
sin π[ x ] x x + 2020 6

6. For f (x) = + , where [ x ] ∴ Range = [0, 1)


1 + [ x] 2 + 3 x
Hence, option (c) is correct.
denotes the greatest integer function, the
domain and range in R are respectively ......... 9. Exactly how many functions f : Q → Q exist
[17 Sep. 2020, Shift-II] such that f (x + y) = f (x) + f (y) and f (xy)
− 2
(a) R − − 1,  1
 and R −   = f (x) f (y) for all x , y ∈ Q? [18 Sep. 2020, Shift-II]
 3   3
− 2 (a) One (b) Two

(b) R − − 1,  and [− 1, 1] (c) Three (d) Infinitely many
 3 
Sol. (b)
(c) R − [− 1, 0) and R −  
1
 3 These functions are simultaneously satisfy for
(d) R − [− 1, 0) and [−1, 1] f (x) = x and f (x) = 0 ∀ x ∈ Q
Hecne, option (b) is correct.
Sol. (d)
As [x] = −1 when 10. If f : R → R is defined as
x ∈ [−1, 0). This makes denominator of first part 2020 x
1 + [x] = 0. Hence, f (x) = , ∀ x ∈ R, then
2020 x + 2020
Interval [−1, 0) must be excluded from domain 4039
 r 
set.
∴D ( f) = R − [−1, 0)
∑ 2 f  4040  =
r =1 [21 Sep. 2020, Shift-I]
Also at x = 0 (which is part of domain), value of (a) 4040 (b) 4039 (c) 2020 (d) 1010
function is zero.
i.e. f(0) = 0 Sol. (b)
So option (d) is correct. The given function f : R → R is define as
2020x
39 f (x) = , ∀ x∈ R
7. Find ∑ f ()t if f : R → R is defined as 2020x + 2020
t=1 2020
∴ f (1 − x) + f (x) =
f (x + y) = f (x) + f (y), x , y ∈R and f ()
1 =7 2020 + 2020x
[18 Sep. 2020, Shift-I]
2020x
(a) 5187 (b) 5460 (c) 5740 (d) 5407 + =1
2020x + 2020
Sol. (b)
∴ 2 Σ f 
4039 r 
(b) It is given that, f (x + y) = f (x) + f (y)  =2
r = 1  4040 
and 1 =7
f()
∴ f (x) = 7 x   1   4039     2   4038  
  f  4040  + f  4040   +  f  4040  + f  4040  
39    
So, Σ f ()
t = 7 [1 + 2 + 3 + ... + 39]
t =1   2019   2021    2020  
39 × 40 + .... +  f   + f  + f 
=7× = 5460.   4040   4040    4040  
2
Functions 41

 1 
= 2 2019 + f    = 2 2019 +  Q f   = 
1 1 1 15. For equality of functions f and g ………
  2  
 2 
   2  2
(i) domain of f = domain of g
= 4039
Hence, option (b) is correct. (ii) f (x) = g(x)
(iii) x ∈ domain of f [22 Sep. 2020, Shift-I]
11. If f (x) = e x ; g(x) = ln(x) for all x ∈[1, ∞), then (a) Both (i) and (ii) necessary
fog is ....... [21 Sep. 2020, Shift-I] (b) Both (ii) and (iii) are necessary
(a) A one-one function (b) An onto function (c) Both (i) and (iii) are necessary
(c) Not a function (d) Bijective (d) All of the above
Sol. (d) Sol. (d)
For x ∈ [1, ∞) , f (x) = e x and g(x) = ln x For equality of functions f and g it is necessary
that
∴( fog)(x) = e = x, x ∈ [1, ∞) is a bijective
ln x
(i) domains of f = domain of g
function.
(ii) f (x) = g(x)
Hence, option (d) is correct.
and (iii) x ∈domain of f.
12. The domain of | x|− x is [21 Sep. 2020, Shift-II] So, all are necessary.
(a) (− ∞, 0) (b) (0, ∞ ) (c) (− ∞, ∞ ) (d) R − {0} 16. Let f : N × N → N be a function such that
Sol. (c) f ((1, 1)) = 2 and f ((m + 1, n)) = f ((m , n))
Given function |x|− x is define, if |x|≥ x + 2(m + n) and f ((m , n + 1)) = f ((m , n)) + 2
(m + n − 1), ∀ m , n ∈ N , then find f (2 , 2)
⇒ x ∈R
[22 Sep. 2020, Shift-I]
Hence, option (c) is correct.
(a) 8 (b) 7 (c) 9 (d) 10
13. Let S be a finite set. Then a non-identity Sol. (d)
function f : S → S can be ………
Given function f :N × N → N, such that
[21 Sep. 2020, Shift-II]
f (m + 1, n) = f (m, n) + 2(m + n)
(a) Injective but not surjective
and f (m, n + 1) = f (m, n) + 2 (m + n − 1) , where
(b) Surjective but not injective
f(1, 1) = 2 .
(c) Bijective but it does not have an inverse function
∴ f (2, 2) = f (1 + 1, 2) = f (1, 2) + 2 (1 + 2)
(d) Data insufficient
= f(1, 1 + 1) + 6
Sol. (d) = f(1, 1) + 2 (1 + 1 − 1) + 6
It is given that for a finite set S, f : S → S is = f( 1, 1) + 8 = 2 + 8 = 10
non-identity function, then it can be injective
only or surjective only or both or none.  1 
17. For f (x) = sin   the domain and

So, data is insufficient to answer.
| x| x 2 − 1 
Hence, option (d) is correct.
range of f (x) in R are [22 Sep. 2020, Shift-I]
14. If ( f (x))2 = f (x 2) + f ()
1 holds good, then find (a) R − {0, ± 1} and [−1, 1,] respectively
f (x) [21 Sep. 2020, Shift-II] (b) R − [−1, 1] and [−1, 1] respectively
1 1 1 1 (c) R − {0, ± 1} and [0, 1,] respectively
(a) x + (b) x − (c) x2 + (d) x −
x x x x2 (d) R − [−1, 1] and [0, 1], respectively

Sol. (a) Sol. (b)


 1 
Given functional relation Given function f (x) = sin 
( f (x))2 = f (x 2) + f ()
1 | x | x 2 − 1 
 
1 For the domain,| x | ≠ 0 ⇒ x ≠ 0
On putting f (x) = x + , the RHS is
x and x 2 − 1 > 0 ⇒ x ∈ R − [− 1, 1]
2
+ 2 =  x +  = ( f (x))2 = LHS.
1 1
x2 + So domain of f is R − [− 1, 1]
x2  x
and we know the range of sine function is
Hence, option (a) is correct. [− 1, 1].
42 AP EAMCET Chapterwise Mathematics

1 ⇒ 2 a−1 (2b − 1) = 2 c −1 (2d − 1)


18. The range of the function f (x) = x 2 +
x +1
2
⇒ 2 . b − 2 a−1 = 2 c . d − 2 c −1 ⇒ (a , b) = (c , d)
a

is [22 Sep. 2020, Shift-I] ∴ f is a one-one function.


(c)  , ∞ 
3 Now as 2 m−1 is a even number for ∀m ∈ N − {1}
(a) [1, ∞ ) (b) [2, ∞ ) (d) (0, 1]
 2 
and (2n − 1) is a odd number for ∀N ∈ N, so the
Sol. (a) every natural number can be obtain by the for
1 2m −1 .(2n − 1) for some combination of (m, n), so f
Given function f (x) = x 2 + = y (let) is an onto function.
x2 + 1
Hence, option (d) is correct.
⇒ x 4 + x 2 + 1 = y(x 2 + 1) and y > 0
⇒ x 4 + (1 − y) x 2 + (1 − y) = 0 21. In the following statements______ .
Q x ∈ R, so discriminant ≥ 0 (i) Relation is a special case of function
⇒ (1 − y)2 − 4(1 − y) ≥ 0 (ii) Function is special case of relation
(iii) Both relation and function are same
⇒ (1 − y) (1 − y − 4) ≥ 0
[23 Sep. 2020, Shift-I]
⇒ (1 − y)(− 3 − y) ≥ 0
(a) (iii) is true, (i) and (ii) are false
⇒ ( y + 3) ( y − 1) ≥ 0
(b) (i) is true, (ii) and (iii) are false
⇒ y ∈ (− ∞ , − 3] ∪ [1, ∞)
(c) (ii) is true, (i) and (iii) are false
but y > 0, so y ∈ [1, ∞) (d) All (i), (ii) and (iii) are true
Therefore range of f is [1, ∞)
Sol. (c)
19. The function f (x) = sin x − cos x is ……… By definition, (c) option is true.
[22 Sep. 2020, Shift-II]
22. If f : R → R is defined as f (x) = x − [ x ] + 3,
(a) Odd function
(b) Even function
∀ x ∈ R , then f is [23 Sep. 2020, Shift-I]
(c) Neither even nor odd function (a) Not a function
(d) f( x) is not a function (b) A periodic function with period π
(c) A periodic function with period 1
Sol. (c) (d) An invertible function
f (x) = sin x − cos x
Sol. (c)
f (− x) = sin (− x) − cos (− x)
f (x) = x − [x] + 3
f (− x) = − sin x − cos x
= x − (x − {x}) + 3 [Q[x] = x − {x}]
f (− x) = − (sin x + cos x)
f (x) = {x} + 3
f (− x) ≠ f (x)
∴ f (x) is neither even nor odd function. ∴ f (x) is periodic function with period 1.
Hence, option (c) is correct. Hence, option (c) is correct.

20. If f : N × N → N is defined by 23. If f : R → R is defined as


f ((m , n)) = 2 m−1
(2 n − 1), ∀ (m , n) ∈ N × N , then f (x) = (2020 − x 2019)1 / 2019 , ∀ x ∈ R , find
f is [22 Sep. 2020, Shift-II]  2019 
( f 0 f 0 f 0 f)  
(a) One-one but not onto  2020  [23 Sep. 2020, Shift-I]
(b) Onto but not one-one (a) 1 (b) 0
(c) Neither one-one nor onto 2019 2020
(c) (d)
(d) Both one-one and onto 2020 2019
Sol. (d) Sol. (c)
The given function f :N × N → N is defined by The given real function,
f (m, n) = 2 m −1(2n − 1), ∀(m, n) ∈ N × N. 1

Now, let f ((a , b)) = f ((c , d)), where a , b , c , d ∈ N f (x) = (2020 − x 2019) 2019
Functions 43
1
 1  2019  2019 26. Consider the following lists.

∴   2019 2019 
fof (x) = 2020 − (2020 − x )  [20 April 2019, Shift-I]
   
  
 List I List II
1
| x + 2|  1 , 1
= (2020 − 2020 + x 2019) 2019 = x f ( x) = , x ≠ −2
(A) x+2 1.  3 
∴ fofofof (x) = fof (x) = x
(B) g ( x) = [ x], x ∈ R 2. Z
fofofof 
∴ 2019  2019
 = (C) h( x) = | x − [ x]|, x ∈ R 3.
 2020  2020 W
Hence, option (c) is correct. 1
(D) f ( x) = , x∈R 4. [0, 1)
2 − sin 3 x
24. If f is a continuous real valued function,
5. {−1, 1}
then the range of the function is
[23 Sep. 2020, Shift-I] A B C D A B C D
(a) [0, 1] (a) 5 3 2 1 (b) 3 2 4 1
(b) [Minimum (f ), Maximum (f )] (c) 5 3 4 1 (d) 1 2 3 4
(c) [0, ∞)
Sol. (c)
(d) (− ∞, 0]
|x + 2|
(A) Q f (x) = ,x≠−2
Sol. (b) x+ 2
Given,
 x + 2, x>−2
f is continuous real valued function  x + 2  1, x>−2
∴Range f = [minimum ( f), maximum ( f)] = =
x+ 2
− , x < −2 −1, x < −2
Hence, option (b) is correct.  x + 2
x x x
25. f (x) = + + 2 cos 3 on R − {0 } is So, range of f (x) is {−1, 1}.
e −1
x
2 2 (B) Q g(x) =|[x]|, x ∈ R
[20 April 2019, Shift-I] As [x] ∈ I ⇒|[x]|∈ W
(a) one one function (b) bijection So, range of g(x) is W.
(c) algebraic function (d) even function (C) Q h(x) = |x − [x]|, x ∈ R = |{x}|∈ [0, 1)
[Q {x} = x − [x] and {x} ∈ [0, 1)]
Sol. (d)
x x x So, range of h(x) is [0, 1).
Given function f (x) = + + 2cos3 on 1
ex − 1 2 2 (D) Q f (x) = , x ∈R
2 − sin 3x
R − {0}.
−x − 1 ≤ sin 3x ≤ 1, ∀ x ∈ R
− + 2cos3  −
x x Q
Q f (− x) = 
e −x − 1 2  2 ⇒ −1 ≤ − sin 3x ≤ 1
⇒ 2 − 1 ≤ 2 − sin 3x ≤ 2 + 1
− x ex x x
= − + 2cos3 1 1 1
1 − ex 2 2 ⇒ ≤ ≤
3 2 − sin 3x 1
x
xe x x
= − + 2cos3
So, range of f (x) is  , 1.
1
ex − 1 2 2  3 
x(e x − 1 + 1) x x
= − + 2cos3 Hence, option (c) is correct.
ex − 1 2 2
 3  3
= x+ x
x x
− + 2cos 3 x 27. If f : R −   → R −   is given by
e −1 2 2 7  7 
3x + 5
= x
x x
+ + 2cos 3 x
= f (x) f (x) = , then the statement which is
e −1 2 2 7x −3
Q f (− x) = f (x), ∀ x ∈ R − {0} not true, is [20 April 2019, Shift-II]
∴ f (x) is an even function. (a) f −1( x) = f( x) (b) (fof )( x) = x
Hence, option (d) is correct. (c) (fofof )( x) = x (d) (fofofof )( x) = x
44 AP EAMCET Chapterwise Mathematics

Sol. (c) 30. If the number of elements in the sets G and


Given, function f : R −   → R −   is define
3 3 A are 3 and 4 respectively, then match the
 
7  
7 items of List I with those of List II
3x + 5
by f (x) = . List I List II
7x − 3
Let f (x) = y A.
The number of non bijective
I. 24
3x + 5 functions from G × G to G
⇒ = y
7x − 3 B. The number of bijective
II. 0
3y + 5 functions from A to A
⇒ x= , so f (x) is a bijective function and
7y − 3 C. The number of functions from
III. 1728
f −1 (x) = f (x). G to G × A
( fof)(x) = x and ( fofofof)(x) = x D. The number of surjective
IV. 12
But ( fofof)(x) ≠ x. functions from A to A × A
Hence, option (c) is correct. V. 19683
28. If f : A → B, g : B → C are functions such that
The correct match is [21 April 2019, Shift-I]
gof : A → C is onto, then a necessary A B C D A B C D
condition is [20 April 2019, Shift-II] (a) V I III II (b) V III IV II
(a) f is onto (b) g is onto (c) III IV V II (d) I II III IV
(c) both f, g are onto (d) neither f nor g is onto
Sol. (a)
Sol. (b) Given that, number of elements in the sets G
If functions f : A → B and g : B → C, such that and A are 3 and 4 respectively.
gof : A → C is onto, then it is necessary that ‘g’ is (A) The number of non bijective function from
onto. Hence, option (b) is correct.
G × G to G = 39 = 19683
29. Let D = R − {0 , 1} and f : D → D, g : D → D G G
and h : D → D be three functions defined by 1 1
1 1 2 2
f (x) = ; g(x) = 1 − x and h(x) = . If
1− x
...

x 3
9
j : D → D is such that (gojof ) (x) = f (x) for all
x ∈ D, then which one of the following is (B) A to A A to A
j (x)? [21 April 2019, Shift-I] a a
(a) (fog ) ( x) (b) f( x) b b
(c) g ( x) (d) (goh) ( x) c c
Sol. (c) d d
We have, Here, a has 4 choice, b has 3, c has 2 and d has 1
(goj of)(x) = f (x) for all x ∈D choice.
1 1 So, number of bijective functions from A to A is
⇒ g(( jof) (x)) = ⇒ 1 − ( jof) (x) =
x x = 4 × 3 × 2 × 1 = 24
1
⇒ 1 − j ( f (x)) = (C) G to G × A
x G to G × A
⇒ 
1− j   =
1 1
a 1
 x x
b 2
j   = 1 −
1 1 3
⇒ c
 x
...

x
⇒ j(x) = 1 − x = g(x) 12
Functions 45

So, number of function from G to G × A is ∴ 4 − 16 y 2 ≥ 0 and y ≠ 0


= 123 = 1728 ⇒ 1 − 4 y 2 ≥ 0 and y ≠ 0
(D) A to A × A
y ∈  − ,  − {0}
1 1

The number of surjective from G to G × A  2 2

∴Range of g(x) =  − , 
i.e. 4 to 16 is 0 1 1
So, A → V, B → I, C → III, D → II.  2 2

∴D = R+ − {all integers} and C =  − , 


1 1
31. Let A and B be finite sets and PA and PB
 2 2
respectively denote their power sets. If PB
D ∩ C =  0, 
1
has 112 elements more than those in PA , ∴
 2
then the number of functions from A to B
which are injective is [21 April 2019, Shift-II] 33. If f : R → R is defined by
(a) 224 (b) 56 (c) 120 (d) 840
|[ x − 5]|, for x < 5
Sol. (d) f (x) = 
[| x − 5|], for x ≥ 5
Let n(A) = m and n(B) = n. Then, according to the
question,  7
Then, (fof)  −  =
n(P(B)) − n(P(A)) = 112  2
⇒ 2n − 2m = 112
(here, [x] is the greatest integer not
⇒ 2m (2n − m − 1) = 16 × 7 exceeding x) [22 April 2019, Shift-I]
⇒ 2m (2n − m − 1) = 24 (23 − 1)
(a) (fof )  −  (b) (fof )  − 
11 9
∴ m = 4 and n − m = 3  2  2
(d) (fof )  
⇒ m = 4, n = 7 9
(c) (fof )(3)
∴Number of injective functions from A to B 2
7! Sol. (d)
= n( B) Pn( A) = 7P4 = = 840
3!
fof  −  = f  − − 5 = f (9) = |[19 − 51]| = 4
7 7
   2  2 
x − | x|
32. Let D =  x ∈ R : f (x) = is defined By checking option
 x − [ x]   
(A) → f   − − 5  = 6
11
and C be the range of the real function 
  2 

2x
g(x) = . Then D ∩ C = 
(B) → fof  −  = 5
9
4 + x2  2
[21 April 2019, Shift-II]
(C) → fof(3) = 3
(a)  − ,  (b)  0, 
1 1 1
(c) R + (d) R + − Z + (D) → fof   = f ()
9
 2 2   2  1 =4
 2
Sol. (b) Hence, option (d) is correct.
We have, 34. If f : A → B is an onto function such that
x − |x|
f (x) = 1
x − [x] f (x) = | x| − x + , then A and B are
| x|− x
∴ x − |x|≥ 0 and x − [x] > 0
⇒ x > |x|and x > [x] respectively. [22 April 2019, Shift-I]
∴ x ∈ R+ − {all integers} (a) (−∞, ∞ ), (0, ∞ ) (b) (−∞, 0), [2, ∞ )
2x (c) (0, ∞ ), (2, ∞ ) (d) (−∞, 0], (0, ∞ )
Again, g(x) =
4 + x2 Sol. (b)
2x We have,
Let, y= ⇒ 4 y + x 2 y = 2x
4 + x2 1
f (x) = |x| − x +
|x| − x
2± 4 − 16 y 2
⇒ yx 2 − 2x + 4 y = 0 ⇒ x = We know that, f (x) will be defined when
2y
46 AP EAMCET Chapterwise Mathematics

|x| − x > 0 ⇒ |x|> x x2 − x


Again, let y=
∴ x ∈ (−∞ , 0) x 2 + 2x
1
Now, f (x) = |x| − x + ⇒ yx 2 + 2xy = x 2 − x
|x| − x
⇒ (y − 1) x 2 + x(2y + 1) = 0
1
= −2x + [Q x ∈ (−∞ , 0)] ⇒ x[(y − 1) x + 2y + 1] = 0
−2x
(2y + 1)
1 ⇒ x=− [Q x ≠ 0]
∴ fmin = 2 −2x . = 2 [Q AM ≥ GM] y −1
−2x
∴Range of f (x) = R − {1}
∴ f (x) ∈ [2, ∞)
∴B = R − {1}
∴ A = (−∞ , 0) and B = [2, ∞)
(2x + 1)
f (x) Also, f −1 (x) = − = g(x)
35. Let f be a function such that f (xy) = for (x − 1)
y  2(x − 1) − (2x + 1) 
all positive real numbers x , y. If f (20) = 15, ∴ g′(x) = −  
 (x − 1)2 
then f (50) = [22 April 2019, Shift-II]
75  2x − 2 − 2x − 1  3
(a) (b) 12 (c) 6 (d) 75 = − = >0
2  (x − 1)2  (x − 1)
2

Sol. (c) ∴ g(x) = f −1 (x) is an increasing function.


f ( x)
Given, f( xy) = and f(20) = 15
y 37. Match the following
f(5)
⇒ f(5 × 4) = = 15 List I List II
4
f : R → R is such that
⇒ f(5) = 4 × 15 = 60 f is neither one-
A. f( x) = px + q , I.
f(5) 60 one nor onto
Now, f (50) = f (5 × 10) = = =6 ( p ≠ 0), ∀ x ∈ R
10 10
f : R → R + ∪ {0} is such f is both one-one
B. that f( x) = x2 , ∀ x ∈ R II.
36. If f : A → B is a function defined by and onto
x2 − x , f : N → N is such that
f (x) = then which one of the C. III.
f is one-one but
x2 + 2x f(n) = n2 + 2 n + 3, ∀ n ∈ N not onto
following is true? [22 April 2019, Shift-II]
f : R → R is such that
(a) A = R − {0, − 2}, B = R and f( x) is decreasing f is onto but not
D. f( x) = 2 (cos 2 5 x + sin2 5 x), IV.
function one-one
∀ x∈R
(b) A = R − {− 2}, B = R − {1} and f − 1( x) is
decreasing function f is a constant
(c) A = R − {0, − 2}, B = R − {1} and f − 1( x) is V. function and also
a bijection
increasing function
(d) Both f( x) and f − 1( x) are increasing functions The correct answer is [23 April 2019, Shift-I]
Sol. (c) A B C D A B C D
We have, (a) II IV III I (b) II IV V I
(c) II I III V (d) III II I IV
x2 − x
f (x) = Sol. (a)
x 2 + 2x
(A) For function f : R → R is defined as
∴ f (x) is defined when x 2 + 2x ≠ 0 f (x) = px + q, (p ≠ 0) is a linear function.
Now, x 2 + 2x = 0 And linear functions are one-one and onto in
⇒ x(x + 2) = 0 set of real numbers (R).
⇒ x = 0, − 2 So, A → II
∴Domain of f (x) = R − {0, − 2} (B) For function f : R → R+ ∪ {0} is defined as
∴ A = R − {0, − 2} f (x) = x 2
Functions 47

Q f (− 1) = f ()
1 = 1, so f (x) is not one-one function 39. If f :R → R is defined by f (x) = [2 x ] − 2[ x ] for
but range of f (x) = x 2 is [0, ∞),
x ∈ R, then the range of f is (Here [x] denotes
Q x 2 ≥ 0, ∀ x ∈ R. the greatest integer not exceeding x)
So, f is onto but not one-one. [22 April 2018, Shift-I]
So, B → IV (a) Z, the set of all integers
(C) For f : N → N is defined as (b) N, the set of all natural numbers
f (x) = n2 + 2n + 3is one-one but not onto (c) R. the set of all real numbers
because there is not value of n, for which (d) {0, 1}
f (n) = 3. Sol. (d)
So, C → III Since, x = [x] + {x}
(D) For f : R → R is defined as f (x) = 2
⇒ 2x = 2[x] + 2{x}
(cos2 5x + sin2 5x) = 21
() = 2.
[2x] = 2 [x] + (2[x])
Q f is define for every value of x ∈ R, but range
2[x] + 0, 0 < {x} < 1
of f is {2}. 
[2x] =  2
So, f is neither one-one nor onto. 1
 2[x] + 1, ≤ {x} < 1
Hence, option (a) is correct.  2
a − | x| 0, 0 ≤ {x} < 1
38. The range of f (x) = , (a > 0) is  2
(a + 1) − | x| ∴ [2x] − 2[x] = 
1
1, ≤ {x} < 1
[23 April 2019, Shift-I]  2
(a) [0, a] Hence, Range of f is {0, 1}
 a a 
(b) [0, ∞ ) −  − ,  40. If f : R → [ − 1, 1] and g : R → A are two
 a + 1 a + 1
surjective mappings and
 a 
(c)  0,  ∪ (1, ∞ )  π f (x)
 a + 1 sin  g(x) −  = 4 − f 2 (x), then A =
 3 2
 a 
(d)  0, + 1 [22 April 2018, Shift-II]
 a + 1  2π − π π
(a)  0, (b) [− 1, 1] (c)  ,  (d) (0, π )
 3   2 2
Sol. (c)
Given function is Sol. (a)
a − |x| Let f (x) = y, then
f (x) = , (a > 0)
(a + 1) − |x| π
sin g(x) −  =
f (x) y
4 − f 2(x) = 4 − y2 = t
 3 2 2
Q f (x) ≥ 0, ∀ x ∈ domain of f (x).
a − |x| (let)
Now, let = y
(a + 1) − |x| y4
⇒ y2 − = t2 ⇒ 4 y 2 − y 4 = 4t 2
⇒ a − |x| = y (a + 1) − y|x| 4
1
[Q assuming |x| ≠ a + 1] ⇒ (y 2− 2)2 = − 4t 2 + 4 ⇒ t 2 = 1 − (y 2 − 2)2
4
⇒(y − 1) |x| = y(a + 1) − a
y(a + 1) − a Q f (x) = y ∈ [−1, 1] ⇒ y 2 ∈[0, 1]
⇒|x| = ≥ 0, ∀ x ∈ domain of f (x)
y −1  3 3
t 2 ∈  0, 
3
∴ ⇒ t ∈ − , 
 4  2 2
 a  
∴ y ∈  − ∞,  ∪ (1, ∞) (as a > 0)
 a + 1 π  3 3
So, sin g(x) −  ∈  − , 
  3  2 2 
a 
So, range of f (x) = y ∈  0,  ∪ (1, ∞) π π π
 a + 1 ⇒ g ( x) − ∈  − , 
3  3 3 
[as y ≥ 0]
2π
⇒ g ( x) ∈  0,
Hence, option (c) is correct.  3 
48 AP EAMCET Chapterwise Mathematics

4 − x2 For a , y ∈ A, f (y) ∈ B, let f (y) = x.


41. The domain of the function f (x) = , For every element Z ∈ C, we have at X ∈ B.
[ x] + 2
Such that g(x) = Z.
where [ x ] denotes the greatest integer not ∴ g is onto
more than x, is [22 April 2018, Shift-II]
Hence, f is an injection and g is a surjection.
(a) (− ∞, − 2 ) ∪ (1, 2 ) (b) (− ∞, − 2 ) ∪ (− 1, 2 )
(c) (− ∞, − 2 ), ∪ [− 1, 2 ] (d) (− ∞, − 1) ∪ (1, 2 ) 43. If f : [R → [R is such that
Sol. (c) f (x + y) = f (x) + f (y) for all x, y ∈ [R. f ()
1 =7
n
4− x
∑ f (r) = 14112, then n =
2
Given function f( x) = , is define if and
[ x] + 2 r =1
4 − x2 x2 − 4 [23 April 2018, Shift-I]
≥0 ⇒ ≤0 (a) 9 (b) 13 (c) 63 (d) 62
[ x] + 2 [ x] + 2
So, either x2 − 4 ≤ 0 Sol. (c)
We have, f (x + y) = f (x) + f (y) and f()1 =7
and [x] + 2 > 0 …(i)
Q 1 =7
f()
or x2 − 4 ≥ 0
∴ f (2) = f ()
1 + f ()
1
and [x] + 2 < 0 …(ii) f(2) = 7 + 7 = 14
From Eq. (i), Similarly, f (x) = f ()
1 + f (2) = 7 + 14 = 21
x ∈ [−2, 2] and x ∈ [−1, ∞) f (4) = f ()
1 + f (3) = 7 + 21 = 28
So, x ∈ [−1, 2] …(iii) ∴ f (n) = 7n
From Eq. (ii), n
x ∈ (−∞ , − 2] ∪ [−2,− ∞] and x∈ (−∞ , − 2) Now, ∑ f (r) = 14112
r =1
So, x ∈ (−∞ , − 2) …(iv) ⇒ f ()
1 + f (2) + f (3) + ... + f (n) = 14112
From intervals Eqs. (iii) and (iv), ⇒ 7 + 14 + 21 + ... + 7n = 14112
x ∈ (−∞ , − 2) ∪ [−1, 2] ⇒ ( + 2 + 3 + ... + n) = 14112
71
42. If f : A → B and g : B → C are two functions 7n (n + 1)
⇒ = 14112
such that gof : A → C is a bijection, then 2
which one of the following is always true? ⇒ n(n + 1) = 4032
[23 April 2018, Shift-I] ⇒ n(n + 1) = 63 × 64 ⇒ n = 63
(a) f and g are bijections 1
44. If g(x) = x 2 + x − 2 and (gof)
(b) f is an injection and g is a surjection 2
(c) f is a surjection and g is an injection (x) = 2 x − 5 x + 2 , then one such function
2
(d) f is a bijection but g is not a injection
f (x) = [23 April 2018, Shift-II]
Sol. (b)
(a) 2 x − 3 (b) 2 x + 3
If f : A → B and g : B → C are two function such
(c) 2 + 2 x (d) 2 x2 − 3 x − 1
that go f : A → C is bijection.
As, gof is one-one for x , y Sol. (a)
Let f (x) = f (y) Given, g(x) = x 2 + x − 2
go( f (x)) = go ( f (y)) 1
and (gof)(x) = 2x 2 − 5x + 2
⇒ g f (x) = g f (y) 2
⇒ x= y ⇒ g( f (x)) = 4 x 2 − 10 x + 4
∴ f is one one. ⇒ ( f (x))2 + ( f (x)) − 2 = 4 x 2 − 10 x + 4
Now, Let Z ∈ C
Now, it is necessary that f (x) should be linear
As gof is onto for every element Z ∈ C polynomial expression, so let f (x) = ax + b, then
there exist y ∈ A that is in domain of gof.
(ax + b)2 + (ax + b) − 2 = 4 x 2 − 10 x + 4
gof (y) = Z
⇒ a 2 x 2 + (2ab + a) x + (b 2 + b − 2)
g ( f (y)) = Z
f lies in range of B as f : A → B. = 4 x 2 − 10 x + 4
Functions 49

Then, f (x) ⋅ f   = f (x) + f  


On comparing the coefficient of different kind 1 1
of terms, we are getting  x  x
a 2 = 4, ⇒ (a 0 x n + a1 x n − 1 + K + a n)
⇒ 2ab + a = − 10  a 0 + a1 + K+ a 
 n n
and b2 + b − 2 = 4 x xn − 1 
So, a = ± 2, On comparing the coefficient of x n , we have
−3; if a = 2 a0 an = a0 ⇒ an = 1
then b=
 2 ; if a − 2′ Comparing the coefficient of x n − 1 , we have
and these value satisfy the all above relations, ⇒ a 0 a n − 1 + a n a1 = a1
so
f (x) = 2x − 3 or − 2x + 2. ⇒ a 0 a n − 1 + a1 = a1 [as a n = 1]
1 ⇒ a0 an − 1 = 0
45. If f : R → A defined by f (x) = ,
x2 + 2x + 2 ⇒ an − 1 = 0 [as a 0 ≠ 0]
∀x ∈ R is surjective, then A = Similarly, a n − 1 = a n − 2 = ... = a1 = 0
[23 April 2018, Shift-II] and a0 = ± 1
(a) [1, ∞ ] (b) (1, ∞ ) (c) [0, 1] (d) (0, 1] ∴ f (x) = 1 ± x n
Sol. (d) f (4) = 1 ± 4n = 257 ⇒ 4n = 256
Since, the quadratic expression ⇒ 4n = 256
x 2 + 2x + 2 = (x + 1)2 + 1 ∈ [1, ∞), ∀x ∈ R ⇒ 4=4
1 f ( x) = 1 + x 4
⇒ ∈ (0, 1]
(x + 1)2 + 1
So, f(3) = 1 + 34 = 82
1
For f (x) = 2 , ∀x ∈ R is surjective, then
x + 2x + 2 47. The set of all real values of x for which the
x
set A = (0, 1]  1
real valued function f (x) = 1 +  is
46. If f (x) is a polynomial function satisfying  x
 1  1 defined, is [24 April 2018, Shift-I]
f (x) ⋅ f   = f (x) + f   and f (4) = 257 , (a) (0, ∞ ) (b) R − {0}
 x  x
(c) (− ∞, − 1) ∪ (0, ∞ ) (d) R − {0, − 1}
then f (3) = [24 April 2018, Shift-I]
(a) 28 (b) 65 (c) 82 (d) 244 Sol. (c)
x
f (x) = 1 +  ⇒ 1 +  > 0
1 1
Sol. (c)
 x  x
Let f (x) = a 0 x n + a1 x n − 1 + a 2 x n − 2
x +1
+ K + an − 1 x + an > 0 ⇒ x ∈ (− ∞ , − 1) ∪ (0, ∞).
x
5
Mathematical Induction
k5 k3 7 3. a n + bn is divisible by .........., if n is any odd
1. + + is ……… if k ∈N
5 3 15 k positive integer. [18 Sep. 2020, Shift-I]
[17 Sep. 2020, Shift-I] (a) a − b (b) a2 − b 2
(a) A natural number (b) An integer (c) a2 + b 2 (d) a + b
(c) A positive fraction (d) Equal to zero
Sol. (d)
Sol. (a)
5 3 For any odd positive integer ‘n’ a n + b n is
k k 7k
Let P(K) = + + , k∈N divisible by (a + b).
5 3 15
3k5 5k3 (15 − 5 − 3)k 4. Let P(n) :12 + 22 + 32 + K + n2
= + +
15 15 15 6 (n − 1) (n − 2) K(n − 2020) + 2 n3 + 3 n2 + n
= ,
3 5 5 3 6
= (k − k) + (k − k) + k
15 15 for all n ∈N. Then which of the following is
1 1 correct? [21 Sep. 2020, Shift-I]
= k(k + 1)(k + 1)(k − 1) + k(k + 1)(k − 1) + k
2
5 1444 24443 3 14
divisible by 5
4244 3
divisible by 3 (a) P(n) is true for all n ∈N
(b) P(n) is true for all h > 2020
So, overall it is a natural number
(c) P(n) is true for all n ≤ 2020
2. Let P(n) : 2 + 22 + 23 + ...+ 2n = 2n + 1, n ∈N. (d) P(n) is not true for any n ∈N
Then, [17 Sep. 2020, Shift-II] Sol. (c)
(a) P(m) is true ⇒ P(m + 1) is true Given statement
(b) P(n) is true for all n ∈N P(n) =12 + 22 + 32 + K + n2 =
(c) P(n) is true for all n ≥ 20 6(n − 1) (n − 2) ...(n − 2020) + 2n3 + 3n2 + n
(d) P(n) is true for all n ≤ 10
6
Sol. (a) Q We know that,
Given, p(n) = 2 + 22 + 23 + ....+2n = 2n + 1 n(n + 1) (2n + 1)
12 + 22 + 32 + ... + n2 =
Where n ∈N. 6
Let p(m) is true then, 6(n − 1) (n − 2) ... (n − 2020) + 2n3 + 3n2 + n

P (m) = 2 + 22 + 23 + .... + 2m = 2m + 1 6
n(n + 1) (2n + 1)
So, P(m + 1) = 2 + 22 + 23 + ....+2m + 2m + 1 = (n −1) (n − 2) ... (n − 2020) +
6
= (2 + 22 + 23 + .... + 2m) + 2m + 1 n(n + 1) (2n + 1)
will be if n =1, 2, 3,...., 2020 only.
= 2( m + 1) + 2m + 1 = 2. 2m + 1 = 2m + 2
6
Clearly, if P(m) is true then p(m + 1) is also true. Therefore, P(n) is true for all n ≤ 2020.
∴ P(m) is true ⇒ P (m + 1) is true. Hence, option (c) is correct.
Mathematical Induction 51

5. The sum of the cubes of three consecutive (a) Both (A) and (R) are true and (R) is the correct
natural numbers is divisible by explanation of (A)
[21 Sep. 2020, Shift-I]
(b) Both (A) and (R) are true but (R) is not the correct
explanation of (A)
(a) 26 (b) 25 (c) 9 (d) 7
(c) (A) is true but (R) is false
Sol. (c) (d) (A) is false but (R) is true
Let the three consecutive natural numbers Sol. (a)
(n − 1), n, n + 1 where n ≥ 2
Since,1 + (1 + 2 + 4) + (4 + 6 + 9) + (9 + 12 + 16)
∴Sum of cubes of the three consecutive natural
+ … + (81 + 90 + 100)
numbers is
= 1 + (12 + (2 × 1) + 22) + (22 + (2 × 3) + 32) + (32
(n − 1)3 + n3 + (n + 1)3 = 3n3 + 6n = 3n(n2 + 2)
+ (3 × 4) + 42) + … + (92 + (9 × 10) + 102)
For each value of n ≥ 2, the expression 3n(n2 + 2)
10
is divisible by 9. Hence, option (c) is correct. = ∑ [(r − 1)2 + r(r − 1) + r 2]
r =1
6. The ten’s digit in 10
= ∑ [r − (r −`)]
1 [(r − 1)2 + r(r − 1) + r 2]
1! + 4 ! + 7 ! + 10 ! + 12 ! + 13 ! + 15 ! + 16 ! + 17 ! is r =1
divisible by [22 Sep. 2020, Shift-II] 10
= ∑ [r 3 − (r − 1)3]
(a) 4! (b) 3! (c) 5! (d) 7 r =1

Sol. (b) = (13 − 03) + (23 − 13) + (33 − 23) + … + (103 − 93)
1! + 4! + 7! = 1 + 24 + 5040 = 103 − 03 = 1000
1! + 4! + 7! = 5065 So, both (A) and (R) are true and (R) is the
(10! + 12! + .... + 17!) this value correct explanation of (A).
have last 2 digits as zero es Hence, option (a) is correct.
So, 10’s digit of given question is same as 10’s
digit of (1! + 4! + 7!) 9. The sum of first n terms of the series
∴Required 10’s digit is 6 = 3! 3 21 117
+ + + … is
Hence, option (a) is correct. 5 25 125 [20 April 2019, Shift-II]
2n + 1 2 2n + 1 2
7. For which value of n ∈N, n! has 13 trailing (a) n + − (b) n − −
3 × 5n 3 3 × 5n 3
zeros? [23 Sep. 2020, Shift-I]
2n + 1 2 2n + 1 2
(a) 51 (b) 54 (c) 57 (d) 60 (c) n + + (d) n − +
3 × 5n 3 3 × 5n 3
Sol. (c)
If n! ends with p zeros, then n >,4p. Sol. (a)
As n! ends with 13 zeros, so n ≥, 52. Given, series
Let us examine for n = 54. 3 21 117
+ + + …+ upto n terms
We have,   +  2  +  3  = 10 + 2 + 0 = 12
54 54 54 5 25 125
 5   5   5 
= 1 −  + 1 −  + 1 −
2 4 8 
 + … + upto
 5  25  125
and   +  2  +  3  = 11 + 2 + 0 = 13, where
57 57 57
 5   5   5  n terms
[x] represents greatest integer function. 2 2 3 
= n −  +   +   + … + upto n terms
2 2
Hence, option (c) is correct.
 5  5  5 
8. Assertion (A)
2 2 
n
1 + (1 + 2 + 4) + (4 + 6 + 9)
() 1 −   
5  5 
+ (9 + 12 + 16) + … + (81 + 90 + 100) = 1000 = n−
n 2
1−
Reason (R) ∑(r3 − [ r − 1]3) = n3 for any 5
r =1  n −1 a(1 − r n) 
Q a + ar + ar + … + ar =
2
natural number n [20 April 2019, Shift-I] 
 1−r 
52 AP EAMCET Chapterwise Mathematics

2  2  n2(n + 1)2 + n(n + 1) (2n + 1) + n(n + 1)


n
1 −    =
5  5  n+1 12
= n−  = n+ 2 2
− n(n + 1) [n(n + 1) + (2n + 1) + 1]
3/ 5 3× 5n
3 =
12
Hence, option (a) is correct. n(n + 1) 2
= [n + n + 2n + 2]
12
10. If 15k divides 47 ! but 15k + 1 does not divide it, n(n + 1) 2 n(n + 1)
then k = [20 April 2019, Shift-II] = [n + 3n + 2] = [(n + 1) (n + 2)]
12 12
(a) 15 (b) 12 (c) 10 (d) 5 n(n + 1) (n + 1) (n + 2) n(n + 1)2 (n + 2)
= =
Sol. (c) 12 12
Since, 15 = 3 × 5, so to find the exponent of 15 in
12. Which of the following is divisible by
47! It is enough to find the exponent of 5 in 47!.
x 2 − y 2 , ∀x ≠ y? [21 April 2019, Shift-II]
So, exponent of 5 in 47!.
(a) xn − yn , ∀ n ∈ N
=   +  2 + …,
47 47
 5   5  (b) xn + yn , ∀ n ∈ N
(c) ( xn − yn )( x2 n + 1 + y2 n + 1 ), ∀ n ∈ N
[where, [x] is greatest integer of x].
= 9+1 (d) ( xn − yn )( xm + ym ), ∀ m, n ∈ N
So, required value of k is 10. Sol. (c)
Hence, option (c) is correct. Q x n − y n is divisible by (x + y)(x − y), if
n = even
11. 12 + (12 + 22) + (12 + 22 + 32) + K +
Similarly, x n − y n is divisible by (x − y), if n =
(12 + 2 2 + K + n2) = odd and (x 2n + 1 + y 2n + 1) is divisible by (x + y).
[21 April 2019, Shift-I] Hence, option (c) is correct.
n(n + 1)(n + 2 ) n(n + 1) (2 n + 1)
(a) (b)
1 1 1
12 6 13. + + + K16 terms =
n(n + 1)2 (n + 2 ) n(n + 1)(n + 2 )(n + 3) 2.5 5.8 8 .11
(c) (d)
12 12 [22 April 2019, Shift-I]
4 8 16 1
Sol. (c) (a) (b) (c) (d)
25 25 25 25
1 + (1 + 2 ) + (1 + 2 + 3 ) + K
2 2 2 2 2 2

+ (12 + 22 + K + n2) Sol. (a)


1 1 1
n n i Given, + + + .... 16 terms
= ∑ (12 + 22 + 32 + K + i 2) = ∑ ∑ j 2 2⋅ 5 5⋅ 8 8 ⋅11
i =1 i =1 j =1
=  + ....
1 3 3 3
i(i + 1) (2i + 1)  n(n + 1) (2n + 1)  + +
3  2⋅ 5 5⋅ 8 8 ⋅11 
n
= ∑ Q Σn =
2

i =1 6  6 
=  − + − + −
1 1 1 1 1 1 1 1 1 
2i 3 + 3i 2 + i 1 n 3 1 n 2 1 n + .... + −
3  2 5 5 8 5 11 47 50 
n
= ∑ = ∑i + ∑i + ∑i
i =1 6 3 i =1 2 i =1 6 i =1
=  − = 
1 1 1 1 48  1  24  8 4
= = =
3  2 50  3 100  3  50  50 25
2
1  n(n + 1)  1 n(n + 1) (2n + 1)  1
 n(n + 1) 
= +  +

3 2   2  6  6
 2 
 14. The statement “n5 − 5n3 + 4 n is divisible by
 n(n + 1)  ;
2
Q Σn = 
3

  2   120” is true for [22 April 2019, Shift-II]
  (a) n = 1only (b) n = 10 only
 n(n + 1)  (c) n = 100 only
 Σn =  (d) All positive integer values of n
 2 
1 n2(n + 1)2 1 n(n + 1) (2n + 1) 1 n(n + 1) Sol. (d)
= ⋅ + ⋅ + ⋅
3 4 2 6 6 2 We have,120 = 3 × 5 × 8
n2(n + 1)2 n(n + 1) (2n + 1) n(n + 1) Therefore to show that given expression is
= + +
12 12 12 divisible by 120, it is sufficient to show that 3, 5
and 8 are the factors of n5 − 5n3 + 4n.
Mathematical Induction 53

Let us factorise the given expression (a) an > 7, ∀n ≥ 1 (b) an > 3, ∀n ≥ 1


n5 − 5n3 + 4n = n[n4 − 5n2 + 4] (c) an < 4, ∀n ≥ 1 (d) an < 3, ∀n ≥ 1
= n[n4 − 4n2 − n2 + 4] = n[n2(n2 − 4) − 1(n2 − 4)] Sol. (c)
= n(n2 − 4) (n2 − 1) = n(n2 − 22) (n2 − 12) On limiting case a n < a ∞
= n(n − 2) (n + 2) (n − 1) (n + 1) Now, a∞ = 7 + a∞
= (n − 2) (n − 1) n (n + 1) (n + 2) ⇒ a ∞2 − a ∞ − 7 = 0
Since, (n − 2,) (n − 1), n, (n + 1) are 4 consecutive
1+ 29
integers. One of it must be divisible by 4. And at ⇒ a∞ = [Q a ∞ > 0]
least two out of 5 consecutive integers must be 2
even. Thus, it must be divisible by 8. 1+ 29
So, an < , ∀ n≥1
Now, again out of three consecutive integers 1 2
must be divisible by 3, and out of 5 consecutive ⇒ a n< 4, ∀ n ≥ 1.
integers 1 must be divisible by 5.
 3  5  7  2 n + 1
Thus, the given expression is divisible by 3, 5, 8. 18. If 1 +  1 +  1 +  ... 1 +  = 121,
 1  4  9  n2 
Hence, it clear that n5 − 5n3 + 4n is divisible by
120 for all positive integer values of n.
then n = [23 April 2018, Shift-I]
(a) 11 (b) 10
15. If 2 ⋅ 4 2k +1 + 33k +1 = 11t and (c) 9 (d) 8
2 ⋅ 4 2 k + 3 + 3 3 k + 4 = 11 (pt + 3 q) , where k , t ∈ Z + , Sol. (b)
then (p , q) = [23 April 2019, Shift-I] We have,
(a) (16, 3k + 1) (b) (16, 3k + 4) 1 + 3 1 + 5 1 + 7  ... 1 + 2n + 1  = 121
       
(c) (32, 3k + 1) (d) (32, 3k + 4)  1  4  9  n2 
Sol. (a)  n2 + 2n + 1 
(4)     ... 
9 16
⇒  = 121
(a) By verification method, if k = 0  4  9   n2 
Then, 2⋅ 41 + 31 = 11t ⇒ 8 + 3 = 11t ⇒ t = 1
⇒ n2 + 2n + 1 = 121
Now, from the second given relation on putting
⇒ (n + 1)2 = 121
the values of (k, t) = (0, 1), we get
2⋅ (64) + 81 = 11(p + 3q) ⇒ 128 + 81 = 11 (p + 3q) ⇒ n + 1 = 11
⇒ n = 10
⇒ 11(p + 3 ) = 209 ⇒ p + 3 = 19
q q

Now, from the option p must be 16 and q = 1 19. If 2(4 2n + 1) + 33n + 1 is divisible by k, k > 1 for
= (3k + 1)k = 0 . all n ∈ N , then the value of k is
Hence, option (a) is correct. [23 April 2018, Shift-II]
16. For any natural number n, (15 × 5 ) + (2 × 2 ) 2n 3n (a) 19 (b) 17
(c) 11 (d) 13
is divisible by [22 April 2018, Shift-I]
(a) 7 (b) 11 (c) 13 (d) 17 Sol. (c)
The given expression
Sol. (d)
2(42n + 1) + 33n + 1 = P(n) (let)
We have, (15 × 5 ) + (2 × 2 )
2n 3n

Q P(n) = 8(42n) + 3(33n)


For n = 1,
we get15 × 52 + 2 × 23 = 15 × 25 + 2 × 8 For n = 1, Q n ∈ N
P(n = 1) = 128 + 81 = 209
= 375 + 16 = 391
is divisible by 19 and 11.
which is divisible by 17
Similarly, P(n = 2) = 2048 + 2187 = 4235
∴(15 × 52n) + (2 × 23n) is divisible by 17, ∀ n ∈ N.
is divisible by 11 but not divisible by 19.
So, the given expression 2(42n + 1) + 33n + 1 is
17. If a n = 7 + 7 + 7 + ... n times, then which
divisible by
one of the following is true? k = 11
[22 April 2018, Shift-II] For all n ∈ N.
6
Partial Fractions
x4 Sol. (b)
1. is a ……… .
x − 3x + 2
3 x2
[17 Sep. 2020, Shift-I]
(x − a)(x − b)
(a) Proper fraction (b) Improper fraction
(c) Mixed fraction (d) Not a fraction Here, degree of both Numerator and
denominator are same. Then the rational
Sol. (b) function is called
 x4  Improper rational function. That way the given
 3  is a improper fraction since
 x − 3 x + 2 Partial fraction is improper.
(degree of number ≥ degree of denominator) for Here, option (b) is correct.
a improper fraction. f (x)
4. Reduction of proper fraction into a sum
x2 + x + 1 B C g(x)
2. If =A+ + , then of partial fraction depends up on
x2 + 2x + 1 x + 1 (x + 1)2
factorization of ......... [21 Sep. 2020, Shift-I]
A − B is equal to [18 Sep. 2020, Shift-I]
(a) f( x) alone (b) g ( x) alone
(a) 4C (b) 4C + 1 (c) 3C (d) 2C (c) both f( x) and g ( x) (d) factors of f( x) and g ( x)
Sol. (d) Sol. (b)
It is given that, f (x)
The reduction of proper fraction into a sum
x2 + x + 1 B C g(x)
= A+ +
x + 2x + 1
2
x + 1 (x + 1)2 of partial fraction depends up on factorization of
denominator g(x) only.
⇒ x 2 + x + 1 = A(x 2 + 2x + 1) + B(x + 1) + C
Hence, option (b) is correct.
⇒ x 2 + x + 1 = Ax 2 + (2A + B) x + (A + B + C)
x +1 A B
On comparing the coefficient of different terms, 5. If = + , then
(2 x − 1) (3 x + 1) 2 x − 1 3 x + 1
we get A = 1, 2A + B = 1 and A + B + C = 1
16 A + 9 B is equal to [22 Sep. 2020, Shift-II]
∴ B = −1, and C = 1
(a) 4 (b) 5 (c) 6 (d) 8
∴ A − B = 2 = 2C.
Sol. (c)
x2 x +1 A B
3. The fraction is = +
(x − a) (x − b) (2x −1)(3x + 1) 2x − 1 3x + 1
[18 Sep. 2020, Shift-II] x + 1 = A(3x + 1) + B(2x − 1) … (i)
1
(a) always a proper partial fraction Put, x = in Eq. (i)
(b) always an improper partial fraction 2
= A  3. + 1 + B(0)
3 1
(c) a proper partial fraction for certain values of a, b
2  2 
only
3 5 3
(d) an improper partial fraction for certain values of = A. ⇒ A=
a, b only 2 2 5
Partial Fractions 55

x =−
1 x +1  D 
Put, in Eq. (i) and = Af1 (x) +  B + 
3 (x − 1)2(x 2 + x + 1)  x − 1
= A(0) + B 2⋅  −  − 1
2 1 C
3   3  f2(x) +
(x − 1)2
2  5
= B −  ⇒ B = −
2
From above informations, it is necessary, that
3  3 5
1 x+ 2
Consider f1 (x) = and f2(x) = 2
x −1 x + x +1
16 A + 9B = 16  + 9  −  = 6
3 2
x +1 A (Bx − B + D)(x + 2)
 5  5 So, = +
(x − 1)2 (x 2 + x + 1) x − 1 (x 2 + x + 1)(x − 1)
Hence, option (c) is correct.
C
+
x2 + 5x + 7 A B C (x − 1)2
6. If = + +
(x − 3)3 (x − 3) (x − 3)2 (x − 3)3 ⇒ x + 1 = A(x − 1)(x 2 + x + 1) + B(x − 1)
then 9 A − 3 B + C = [23 Sep. 2020, Shift-I] (x + 2)(x − 1) + D(x + 2)(x − 1) + C(x 2 + x + 1)
(a) 2 (b) 5 (c) 7 (d) 9 On comparing the coefficient, we get
Sol. (c) coefficient of x 3 = 0 ⇒ A + B = 0
x 2 + 5x + 7 A B C coefficient of x 2 = 0 ⇒ A − A + 2B − 2B + D + C = 0
Given, = + +
(x − 3)3 x − 3 (x − 3)2 (x − 3)3
⇒ D+ C=0
x 2 + 5x + 7 A(x − 3)2 + B(x − 3) + C ∴ A+ B+ C+ D=0
=
(x − 3)3 (x − 3)2 Hence, option (c) is correct.
x 2 + 5x + 7 = Ax 2 + (−6 A + B) x + (9 A − 3B + C) x4 A B
8. If = P(x) + +
∴On comparison of coefficients, we get (x − a)(x − b)(x − c) x−a x−b
−6 A + B = 5 9 A − 3B + C = 7 C
+ , then P(0) + A(a − b)(a − c) =
A = 1 −6(1) + B = 5 9(1) − 3 (11) + C = 7 x−c
B = 11 9 − 33 + C = 7 [20 April 2019, Shift-II]
C = 31 (a) a4 + b 4 + c 4 + a (b) a + b + c
∴Consider 9 A − 3B + C = 9 (1) − 3 (11) + 31 (c) a4 − a − b − c (d) a + b + c + a4
= 40 − 33 = 7
Hence, option (c) is correct. Sol. (d)
If is given that
3 1
7. If = x4 A B C
(x −1)(x 2 + x + 1) x −1 = P(x) + + +
(x − a)(x − b)(x − c) x−a x−b x−c
x +2
− 2 = f1 (x) − f 2 (x) and ⇒ x 4 = (x − a)(x − b)(x − c) P(x) + A(x − b)(x − c)
x + x +1
+ B(x − c)(x − a) + C ((x − a)(x − b)
x +1  D  At x = 0, abc P(0) = bc A + ca B + ab C
= Af1 (x) +  B + 
(x − 1) (x + x + 1)
2 2
 x − 1 A B C
P(0) = + + …(i)
C a b a
f 2 (x) + , A+ B+C + D=
(x − 1)2 a4
At x = a, A = , similarly
[20 April 2019, Shift-I]
(a − b)(a − c)
−1 1 b4
(a) 1 (b) (c) 0 (d) At x = b, B = and
3 3 (b − c)(b − a)
Sol. (c) c4
At x = c, C = …(ii)
It is given that (c − a)(c − b)
3 1 x+ 2
= − a3 b3 c3
(x − 1)(x 2 + x + 1) x − 1 x 2 + x + 1 So, P(0) = + +
(a − b)(a − c) (b − c)(b − a) (c − a)(c − b)
= f1 (x) − f2(x)
56 AP EAMCET Chapterwise Mathematics

a 3(c − b) + b 3(a − c) + c 3(b − a) Sol. (c)


=
(a − b)(b − c)(c − a) Given,
a 3(c − b) + bc(c 2 − b 2) + a(b 3 − c 3) 8 Ax + B C
= = +
(a − b)(b − c)(c − a) (x + 3)2(x − 2) (x + 3)2 x −2
a 3 + bc(c + b) − a(b 2 + c 2 + bc) 8 (Ax + B)(x − 2) + C(x + 3)2
= ⇒ =
(a − b(a − c) ( x + 3) ( x − 2)
2
( x + 3) 2 ( x − 2)
a(a − b ) − c (a − b) − bc(a − b)
2 2 2 ⇒ 8 = (Ax + B)(x − 2) + C(x + 3)2
=
(a − b)(a − c) At x = 2,
(a 2 + ba) − c 2 − bc 8
= =a+ b+ c ⇒ 8 = 0 + C(5)2 ⇒ 8 = 25C ⇒ C =
(a − c) 25
At x = 0,
So, P(0) + (a − b)(a − c) A = a + b + c + a 4
⇒ 8 = (B)(−2) + C(9)
Hence, option (d) is correct.
⇒ 8 = − 2B + 9C
x4 72 Q C = 8 
9. If ⇒ 8 = − 2B +  
(x − 1) (x − 2) (x − 3) 25  25
A B C 72
= x+k+ + + , then ⇒ 2B = −8
x −1 x − 2 x − 3 25
72 − 200 −128 −64
k+ A− B+C = [21 April 2019, Shift-I] ⇒ 2B = = B=
127 25 25 25
(a) 104 (b) 52 (c) 63 (d) At x = 1,
2
⇒ 8 = (A + B)(−1) + C(4)2
Sol. (c)
⇒ 8 = − A − B + 16C
x4
Given, 64 128
(x − 1)(x − 2)(x − 3) ⇒ 8 =− A + +
25 25
A B C −8
= x+ k+ + + ⇒ A=
192
−8 ⇒ A=
x −1 x − 2 x − 3 25 25
⇒ x 4 = x(x − 1)(x − 2)(x − 3) + k(x − 1)  −64 64 8
Now, 25(B + 8C − A) = 25 + + 
(x − 2)(x − 3) + A(x − 2)(x − 3) + B(x − 3)  25 25 25
(x − 1) + C(x − 1)(x − 2) = 25 ×
8
=8
1 25
On putting x = 1, we get A =
2
x2 + 5x + 7 A B C
On putting x = 2, we get B =
16
= − 16 11. If + + + ,
−1 (x − 3) 3
(x − 3) (x − 3) (x − 3)3
2

81 then the equation of the line having slope A


On putting x = 3, we get C =
2 and passing through the point (B, C) is
On putting x = 0, we get [22 April 2018, Shift-I]
1 (a) x + y − 20 = 0 (b) x − y + 20 = 0
0 = k(−1 )(−2)(−3) + (−2)(−3) − 16
2 (c) x + y + 20 = 0 (d) x − y − 20 = 0
81
(−3)(−1) + (−1)(−2) Sol. (b)
2
⇒ 6k = 3 − 48 + 81 = 36 ⇒ k = 6 We have,
1 81 x 2 + 5x + 7 A B C
∴ k + A − B + C = 6 + + 16 + = + +
2 2 ( x − 3) 3 x − 3 ( x − 3) 2 ( x − 3) 3
= 6 + 16 + 41 = 63 ⇒ x 2 + 5x + 7 = A(x − 3)2 + B(x − 3) + C …(i)
8 Ax + B
C Put x = 3 in Eq. (i), we get
10. If = + , then
(x + 3) (x − 2) (x + 3)
2 2
x −2 9 + 15 + 7 = C ⇒ C = 31
25(B + 8 C − A) = [22 April 2019, Shift-I] Put x = 0 in Eq. (i), we get
(a) 25 (b) 1 (c) 8 (d) −8 7 = 9 A − 3B + 31
Partial Fractions 57

⇒ 9 A − 3B = − 24 ⇒ 3A − B = − 8 …(ii) Now, consider


Put x = 1 in Eq. (i), we get 1 5x 2 − 4 x + 1 
 3 
1 + 5 + 7 = 4 A − 2B + 31 2  2x − 5x 2 + 4 x − 1 
⇒ 13 = 4 A − 2B + 31
1 B C D 
⇒ 4 A − 2B = − 18 ⇒ 2A − B = − 9 …(iii) =  + + 
2  2x − 1 x − 1 (x − 1)2 
On solving Eqs. (ii) and (iii), we get
A = 1 and B = 11 1 5x 2 − 4 x + 1 
 3 
∴Equation of line having slope A and passing 2  2x − 5x 2 + 4 x − 1 
through the points (B, C) is 1  B(x − 1)2 + C(x − 1) (2x − 1) + D(2x − 1) 
=  
y − 31 = 1(x − 11) ⇒ y − 31 = x − 11 2 (2x − 1) (x − 1)2 
x − y + 20 = 0
⇒ 5x 2 − 4 x + 1
3x + 1 2
Ax + B Cx + D = B(x − 1)2 + C(x − 1) (2x − 1) + D(2x − 1)
12. If = +
(x + 1) (x + 2)
2
x +12
x2 + 22 2 Put x = 1, we get; D = 2
Ex + F Put x = 1 / 2, we get B = + 1
+ 2 , then A + C + E = Now put x = 0, we get
(x + 2)2 [22 April 2018, Shift-I] 1= B+ C− D ⇒ 1= +1+ C− 2 ⇒ C= 2
7 4
(a) 0 (b) (c) 1 (d) 1  5x 2 − 4 x + 1 
3 3 Now,  
2  (2x − 1) (x − 1)2 
Sol. (a)
1  +1 2 2 
If  = + + 
2  2x − 1 x − 1 (x − 1)2 
3x + 1 2
Ax + B Cx + D Ex + F
= 2 + 2 + 1 2
(x 2 + 1) (x 2 + 2)2 x +1 x + 2 (x 2 + 2)2 +
2 1
3x 2 + 1 = + 2 +
⇒ 2x − 1 x − 1 (x − 1)2
(x + 1)(x 2 + 2)2
2
Put in Eq. (i), we get
Ax + B(x 2 + 2)2 + (Cx + D)(x 2 + 1)  1 
1  2 1 1 
(x 2 + 2) + (Ex + F)(x 2 + 1) = +  + + 
= 2  2x − 1 x − 1 (x − 1)2 
(x 2 + 1)(x 2 + 2)2  
⇒ 3x 2 + 1 = (Ax + B) (x 2 + 2)2 + (Cx + D) By comparing A = 1 / 2 , B = 1 / 2, C = 1, D = 1.
(x 2 + 1) (x 2 + 2) + (Ex + F) (x 2 + 1) Now, 2A − 3B + 4C + 5D
1 1
On comparing the coefficient of different term, = 2× − 3× + 4 ×1 + 5×1
A = 0, C = 0 and E = 0 2 2
3 3 17
So, A+ C+ E=0 = 1 − + 4 + 5 = 10 − =
2 2 2
x3 B C x 4 + 24 x 2 + 28 A B
13. If = A+ + 14. If = + +
(2 x − 1) (x − 1) 2
2x − 1 x − 1 (x + 1)
2 3
(x + 1)
2
(x + 1)2
2

D
+ , then 2 A − 3 B + 4 C + 5 D = C
, then A + C =
(x − 1)2 (x + 1)3
2
[23 April 2018, Shift-II]
[23 April 2018, Shift-I]
(a) 12 (b) 10 (c) 9 (d) 6
21 23 17 19
(a) (b) (c) (d)
2 2 2 2 Sol. (d)
x 4 + 24 x 2 + 28 A B C
Sol. (c) If = 2 + +
(x 2 + 1)3 x + 1 (x 2 + 1)2 (x 2 + 1)3
x3 x3
We have, = 3 ⇒ x 4 + 24 x 2 + 28 = A(x 2 + 1)2 + B(x 2 + 1) + C
(2x − 1) (x − 1) 2
2x − 5x 2 + 4 x − 1
On comparing the coefficient of different terms
1 1 5x 2 − 4 x + 1 
= +  3  …(i) A = 1; 2A + B = 24 and A + B + C = 28
2 2  2x − 5x 2 + 4 x − 1  ⇒ A = 1, B = 22, so A + C = 6.
7
Binomial Theorem
1. The constant term in the expansion of 16 − 20 x + 6 x 2 16 − 20 x
= = {Q x 2 negligible}
x 1  a 2
5 16 − 9 x 2 16
 + + 2  is , then a = 5x
2 x  2 =1−
[17 Sep. 2020, Shift-I] 4
(a) 7 (b) 69 (c) 63 (d) 65 3. If [ x ] denotes the greatest integer function
Sol. (c) on x, then the number of positive integral
x+1 +
5
divisors of [(2 + 3)5] is
2
[17 Sep. 2020, Shift-I]

2 x  (a) 6 (b) 4 (c) 2 (d) 8
10 10
 x 1   x + 2 (x + 2) 10
Sol. (b)
⇒  +  =  ⇒
 2 x  2x  32x 5 (2 + 3)5 = 5C0 ⋅ 25 ⋅ ( 3)0 + 5 C1 ⋅ 24 ⋅ ( 3)
Forms constant term in the expansion 5
C2 ⋅ 23( 3)2 + 5 C3 ⋅ 22 ⋅ ( 3)3
Coefficient of x 5 in (x + 2)10 + 5 C4 ⋅ 2⋅ ( 3)4 + 5 C5 ⋅ 2° ( 3)5
=
32
= 32 + 80 3 + 240 + 120 3 + 90 + 9 3
10
C5 × 4 2 a 2
⇒ = ⇒ (a = 63) (2 + 3)5 = 362 + 209 3 = 723. 99
32 2
⇒ [(2 + 3)5] = 723 ⇒ 723 = 31 × 2411
2. Assuming| x| to be so small, that x 2 and Now, positive integral divisors of
higher powers of x can be neglected, then 723 = (1 + 1)(1 + 1) = 4
1 + x + (1 − x)3 / 2
= 4. If (1 − x + x 2)10 = a 0 + a1 x + a 2 x 2 + ... + a 20 x 20 ,
(1 + x) + 1 + x [17 Sep. 2020, Shift-I] then 2 a 2 + 3 a 3 + 4 a 4 + ... + 20 a 20 =
5x 5x [17 Sep. 2020, Shift-I]
(a) 1 + (b) 1 −
4 4 (a) 0 (b) 10 (c) 20 (d) −20
4x 4x
(c) 1 + (d) 1 − Sol. (c)
5 5
(1 − x + x 2)10 = a 0 + a1 x + a 2 x 2 + ...... + a 20 x 20
Sol. (b)
On differentiating
Given, |x| is very small, x 2 is negligible
( − x + x 2)9 ⋅ (−1 + 2x) = a1 + 2a 2 x + 3a 3 x 2
101
1 3
1+ x +1− x
1 + x + (1 − x)3/ 2 2 2 + ...... + 20a 20 x19
=
(1 + x) + 1 + x 1+ x +1+
x At x = 1
2 ⇒ a1 + 2a 2 + 3a 3 + ...... + 20a 20 = 10
2− x 4 − 2x (4 − 2x)(4 − 3x)
= = = At x = 0 ⇒ a1 = −10
3x 4 + 3x (16 − 9 x 2)
2+ So, 2a 2 + 3a 3 + ...... + 20a 20 = 20
2
Binomial Theorem 59

5. If the mth term is the middle term in 7. The coefficient of x 50 in the expansion of
 1
20
(1 + x)1000 + x (1 + x)999
expansion of  x 2 −  . Find coefficient of
 2x + x 2 (1 + x)998 + K + x 1000 is
Tm + 3 . [17 Sep. 2020, Shift-II] [17 Sep. 2020, Shift-II]
(a) 20
C13 2 − 13
(b) − 20
C13 2 13 (a) 1000 C 50 (b) 999
C 50 (c) 1000 C 51 (d) 1001C 50
(c) − C13 2 − 13
20
(d) 20
C13 213 Sol. (d)
Sol. (a) Expression given is,
20 f (x) = (1 + x)1000 + x (1 + x)999 + x 2(1 + x)998
Expansion of  x 2 −  contains ‘21’ terms.
1
 2x  + .......+ x1000 ,
20 This is a Geometric progression,
So middle term is, + 1 = 11 term x
2 common ratio = and number of terms
⇒ m = 11 1+ x
Now, 1001.
 1001 
(1 + x)1000 1 − 
13 x 
C13 (x 2)20 −13  − 
1  
Tm+ 3 = T11 + 3 = T13 + 1 = 20
 1 + x  
 2x   
So, f (x) =
= 20
C13 x 7(−1)13 1 − x 
 
 1+ x
213 , x13
= (1 + x)1001 − x1001
−13
So coefficient is, ‘‘ C13 ⋅ 2 20
’’
So, coefficient of x 50
1 1×3 1×3 ×5 = coefficient of x 50 in (1 + x)1001
6. If x = + + + ...., then
5 5 × 10 5 × 10 × 15 = n C50 = 1001
C50
3x2 + 6 x = [17 Sep. 2020, Shift-II]
8. In the expansion of (5 3 + 3 2)15
(a) 1 (b) 2 (c) − 1 (d) − 2
[18 Sep. 2020, Shift-I]
Sol. (b)
(a) Number of rational terms is 3
we have, (b) Sum of all rational terms is 58
1 1⋅ 3 1 ⋅ 3⋅ 5
+ x= + + ... (c) Sum of all rational terms is greater than the sum
5 5⋅10 5⋅10 ⋅15 of all irrational terms
Comparing expression with, (d) Sum of all irrational terms is greater than the sum
1 + ny n(n − 1) 2 of all rational terms
(1 + y)n = + y + ....
1! 2! Sol. (d)
we get, Given binomial is (5 3 + 3
2)15
1 n (n − 1) 2 1 ⋅ 3
ny = , .y = 15− r r
5 2⋅ 1 5⋅10 Q The general term Tr + 1 =
15
Cr 3 5 23
Solving we get,
= Cr 33− r / 52r / 3
15
2 1
y = − and n = − For rational terms r must be multiple of 15, so
5 2
1 possible values of r = 0 and 15

So, x = (1 + y)n − 1 ⇒ x = 1 −
2 2
−1=
5
−1 (Q 0 ≤ r ≤ 15)

 5 3 ∴Sum of rational terms = 15C0 33 + 15C15 25
So, 3x 2 + 6 x = 27 + 32 = 59.
2
 5    5  ∴The sum of all irrational terms is greater than
= 3 − 1 + 6    − 1
 3  the sum of all rational terms.
  3 
5 5 5 9. Find the coefficient of x 5 in (1 + x + x 2)8.
= 3 + 1 − 2  + 6 −6
3 3 3 [18 Sep. 2020, Shift-II]
= 5+ 3− 6 = 2 (a) 405 (b) 508 (c) 404 (d) 504
60 AP EAMCET Chapterwise Mathematics

Sol. (d) Q It is given that, the coefficients x 9 and x10 are


Given, Expansion is (1 + x + x ) 2 8 equal,
3n − 9 3n −10
Coefficient of x 5 in above Expansion is So, n C9 9 = n C10 10
8! 2 2
∑ 1 n1 ⋅ (x)n2 (x 2)n3
() n! × 3 n!
n1 ! n2 ! n3 ! ⇒ =
9!(n − 9)! 10!(n − 10)! × 2
∴ (n1 + n2 + n3) = 8
3 1
and n2 + 2n3 = 5 ⇒ = ⇒ n − 9 = 60
n − 9 10 × 2
n1 n2 n3
⇒ n = 69
5 3 0 Hence, option (a) is correct.
5 1 2
4 3 1 12. By neglecting x 4 and higher powers of x,
find approximate value of
8! 8! 8!
Coefficient of x 5 = + +
5! 3! 0! 5! 1! 2! 4! 3! 1! 3
x 2 + 64 − 3
x 2 + 27 [21 Sep. 2020, Shift-I]
= 56 + 168 + 280 = 504 7 2 7
(a) 1 − x (b) 1 − x2
Hence, option (d) is correct. 234 432
7 2 7 2
 1
n (c) 1 − x (d) 1 − x
10. In the expansion of  a + 1 +  , where n ∈N 32 42
 a
Sol. (b)
there are 2029 terms. Then n = Given expression
[18 Sep. 2020, Shift-II] 3
x 2 + 64 − 3 x 2 + 27
(a) 1015 (b) 1013 1/ 3 1/ 3
(c) 1014 (d) 1012  x2   x2 
= 4 1 +  − 31 + 
Sol. (c)  64   27 
n
 a + 1 + 1  = 1 (a 2 + a + 1)n  x2   x2 
  = 4 1 +  − 31 +
 a an  3 × 64   3 × 27 
∴Number of terms = 2n + 1 {on neglecting x 4 and higher powers of x}
2029 = 2n + 1
x2 x2 27 − 48 2
2n = 2028 = 4+ − 3− =1+ x
48 27 48 × 27
n = 1014 21 7 2
Hence, option (c) is correct. =1− x2 = 1 − x
48 × 27 432
11. If the coefficients of x 9 and x 10 in the Hence, option (b) is correct.
n
 x 13. The coefficients of x 50 in (1 + x)101
binomial expansion of 3 +  are equal,
 2
(1 − x + x 2)100 is ...... [21 Sep. 2020, Shift-I]
then n = [21 Sep. 2020, Shift-I]
(a) 1 (b) − 1 (c) 0 (d) 2
(a) 69 (b) 96
(c) 66 (d) 99 Sol. (c)
Sol. (a) Given expression is
n (1 + x)101 (1 − x + x 2)100
Given binomial is  3 +  and the general term
x
 2 = (1 + x) [(1 + x) (1 − x + x 2)]100
in the expansion is = (1 + x) (1 + x 3)100
n 3n − r So, coefficient of x 50 in (1 + x)101 (1 − x + x 2)100
Cr r x r
2
= coefficient of x 50 in (1 + x) (1 + x 3)100
∴Coefficient of x 9 and x10 are
= coefficient of x 50 in (1 + x 3)100
n 3n − 9 3n −10
C9 9 and n C10 10 respectively
2 2 + coefficient of x 49 in (1 + x 3)100
Binomial Theorem 61

Q In the expansion of (1 + x 3)100 , the power of x 1 1 1 


=  2 − x  + 10 −  
is multiple of 3, and while 50 and 49 are not 2 3 32 
multiple of 3. 32 + 960 − 3 989
∴Coefficient of x = − =−
So, coefficient of x 50 and 49 in the expansion of 192 192
(1 + x 3)100 is zero.
16. If the sum of all the coefficients of
∴Coefficient of x 50 in (1 + x)100 (1 − x + x 2)101 is
(αx 2 − 2 x + 1)2019 is equal to the sum of all
zero.
Hence, option (c) is correct. the coefficients of(x − αy)2019 , then α =
[22 Sep. 2020, Shift-I]
y3
14. What is the coefficient of 8
in (x + y)−5, (a) −1 (b) 0 (c) 1 (d) 2
x
y Sol. (c)
when <1? The sum of the coefficient of (αx 2 − 2x + 1)2019 is
x [21 Sep. 2020, Shift-II]
(α − 1)2019. (On putting x = 1)
(a) − 35 (b) − 30 (c) −25 (d) 10
and similarly the sum of the coefficients of
Sol. (a) (x − αy)2019 is (1 − α)2019 (on putting x = y = 1)
−5
1 1  y y
Since (x + y)−5 = = 1 +  , < 1 Now according to the question,
(x + y)5 x 5  x x (α − 1)2019 = (1 − α)2019
y3
Q Coefficient of 8 in(x + y)−5 ⇒ α −1 =1 − α ⇒ α=1
x 9
y3
−5  2 p 3 q
= coefficient of 8 in 5 1 + 
1 y 17. If the 6th term in  +  is “ap b qc ”, then
x x  x 3 2
3 −5
a , b and c respectively are [22 Sep. 2020, Shift-I]
= coefficient of 3 in1 + 
y y
x  x (a) 189, 5, 4 (b) 189, 4, 5 (c) 212, 4, 5 (d) 212, 5, 4
−5(−5 − 1)(−5 − 2)
= = −35 Sol. (b)
3! 9
The sixth term in the expansion 
2p 3q 
Hence, option (a) is correct. +  is
 3 2
15. Assuming x to be so small that x 2 and 4 5
T6 = T5 + 1 = 9C5    
2p 3q
higher powers of x can be neglected, the
 3  2
(1 − x)1 / 3 + (1 − 5 x)2
coefficient of x in is 9 × 8 × 7 × 6  2  3 4 5
4 5
(16 − x)1 / 4 = ×    p q
4 × 3× 2  3  2
equal to [22 Sep. 2020, Shift-I]
989 989 989 989 = 189 p 4 q 5 = ap bq c (given)
(a) (b) (c) − (d) −
96 192 96 192 On comparing, we get
Sol. (d) a = 189, b = 4 and c = 5
Given expression 18. What is the constant term in the binomial
(1 − x)1 / 3 + (1 − 5x)2  1
n

(16 − x)1 / 4 expansion of (1 + 3 x)n 1 +  ?


 3x
−1 / 4 − 1/ 4 
1 1/ 3  x
+ (1 − 5x)2 1 − 
x [22 Sep. 2020, Shift-II]
= (1 − x) 1 −  
2 16  16   2n  2n 
 (a)   (b)  
1  1   x  x   n  n − 1
= 1 − x  1 +  + (1 − 10 x) 1 + 
2   3   64   64    2n 
(c)   (d) No such term exists
(On ignoring the higher degree terms)  n + 1
1 x x x Sol. (a)
= 1 − + + 1 − 10 x + 
2 3 64 64  n n
n  3x +1 
(1 + 3x)n 1 +
1 
(On ignoring the higher degree terms)  = (1 + 3x)  
 3x   3x 
62 AP EAMCET Chapterwise Mathematics

(1 + 3x)n (1 + 3x)n (1 + 3x)2 n ⇒ (125 − 121)2n + 1 = Rf {Q f ′ = f }


= =
(3x)n (3x)n ⇒ Rf = 42n + 1
In (1 + 3x)2n general term is Hence, option (d) is correct.
Tr + 1 = 2n
Cr (3x)r
21. If the term independent of x in the
∴ Coefficient of x n in (1 + 3x)2n is Cn 3n and 2n
 k
10
2n
Cn 3n 2n expansion of  x − 2  is 405, then k =
coefficient of x n in (3x)n is 3n = = Cn  x  [23 Sep. 2020, Shift-I]
3n
(a) 3 only (b) −3 only (c) ± 3 (d) 0
Hence, option (a) is correct.
Sol. (c)
19. (102)4 = ? [22 Sep. 2020, Shift-II] 10
Given,  x − 2 
k
(a) 108242316 (b) 108423216  x 
(c) 102843216 (d) 108243216 Let rth be the independent term
Sol. (d) r
Cr ( x)10 − r ⋅  − 2 
k
∴ Tr + 1 = 10
(102)4 = (100 + 2)4  x 
= 4 C0 (100)4 ⋅1 + 4 C1 (100)3 ⋅ 21 10 − r
= 10
Cr ⋅ x 2 ⋅ (− k)r ⋅ x −2 r
+ 4 C2(100)2 ⋅ 22 + 4 C3(100)1 ⋅ 23 + 4C4 (100)0 24
10 − r
−2 r
= (100)4 + 4100
( )3 ⋅ 2 + 6100
( )2 ⋅ 4 Tr + 1 = 10
Cr . x 2 (− k)r
+ 4100
( )8 + 1 ⋅1 ⋅16 10 − r
To get independent term, we put − 2r = 0
= 100000000 + 8000000 + 240000 + 3200 + 16 2
= 108243216 10 − r
= 2r ⇒ 5r = 10 ⇒ r = 2
Hence, option (d) is correct. 2
∴Given independent term = 405
20. Let R = (5 5 + 11)2n+1 and f = R −[ R], where 10 × 9 2
[ x ] denotes the greatest integer less than or
10
C2(− k)2 ⋅ x ° = 405 ⇒ ⋅ k = 405
2 ×1
equal to x , then Rf = [23 Sep. 2020, Shift-I]
⇒ k2 = 9 ⇒ k=±3
(a) 2 n+ 1 (b) 2 2 n+ 1 (c) 4n+ 1 (d) 42 n+ 1
Hence, option (c) is correct.
Sol. (d)
It is given that, 22. The index of the power of x occurring in the
(5 5 + 11)2n + 1 = R = [R] + f
5th term from the end in the expansion of
12
 x3 2
and (5 5 − 11)2n + 1 = f ′, where f , f ′∈ (0, 1)  − 2  is
∴ [R] + f = 2n + 1
C0 (5 5)2n + 1 + 2n + 1 C1 (5 5)2n (11)1  2 x  [23 Sep. 2020, Shift-I]
+ 2n + 1 C2(5 5)2n −1 (11)2 + … (a) 3 (b) −3 (c) 4 (d) −4
2n + 1 2n + 1 2n + 1
and f ′ = C0 (5 5) − 2n
C1 (5 5) .(11) 1 Sol. (d)
2n + 1
+ C2(5 5)2n −1 (11)2 –.... The fifth term from the end in the expansion of
12
On subtracting both, we get  x3 2
 − 2  is same as fifth term from begining
[R] + f − f ′ = 2 [2n + 1 C1 (5 5)2n (11)1  2 x 
12
+ 2n + 1 C3(5 5)2n − 2 (11)3 + ......]  2 x3 
in the expansion of  2 −  and
⇒[R] + f − f ′ = even integer x 2
Q [R] is an integer so, f − f ′ must be integer 12− 4
 x3 
4
C4  2 
2
and as f ∈(0, 1) and f ′(0, 1) T5 = T4 + 1 = 12
− 
x   2
∴−1 < f − f ′< 1, so possible integral value of
f − f ′ is zero. 28 x12 12
= 12
= C4 24 x −4
C4
∴ f = f′ 24 x16
Now, (5 5 + 11)2n + 1 (5 5 − 11)2n + 1 = Rf ′ ∴ The index of the respective term is −4
Hence, option (d) is correct.
Binomial Theorem 63

23. The sum of all the coefficients in the and


(1 − qx)−1 = 1 + qx + q 2 x 2 + q 3 x 3 + … + q n x n + …
binomial expansion of (1 + 2 x) is 6561. Let
n

Now, coefficient of x n in the expansion of


R = (1 + 2 x)n = I + F, where I ∈ N and
(1 − px)−1 (1 − qx)−1
1 F
0 < F < 1. If x = , then 1 − = = p n + p n − 1 q + p n − 2q 2 + p n − 3q 3 + … + q n
2 1 + ( 2 −1)4 n + 1
  q
[20 April 2019, Shift-I] p n 1 −   
  p 
(a) (3 2 − 4) (b) 4(3 2 + 4)   p n (p n + 1 − q n + 1) p
an = =
(c) ( 2 − 1)4 (d) 1 1−
q (p − q) p n + 1
p
Sol. (c)
pn + 1 − q n + 1
It is given that sum of all the coefficients in the =
binomial expansion of (1 + 2x)n is 6561 = (1 + 2)n p−q
on putting x = 1. pn + 1 − q n + 1
So, an =
⇒ 3n = 6561 ⇒ n = 8 p−q
1 Hence, option (b) is correct.
Now, at x = , then R = (1 + 2x)n = I + F
2
25. If (1 + x + x 2)n = c0 + c1 x + c2 x 2 + …, then the
⇒ R = ( 2 + 1)8 = I + F, where I ∈ N and 0 < F < 1
value of c0 c1 − c1 c2 + c2 c3 − … is
∴( 2 − 1)8 = F ′, where 0 < F ′ < 1 [20 April 2019, Shift-II]
∴( 2 + 1)8 + ( 2 − 1)8 = I + (F + F ′) (a) (−1)n (b) 0 (c) 2 n (d) 3n
⇒ 2[( 2)8 + 8 C2 ( 2)6 + 8 C4 ( 2)4 + 8 C6( 2)2 + 8 C8 ] Sol. (b)
= I + (F + F ′) If (1 + x + x 2)n = C0 + C1 x + C2 x 2 + … …(i)
⇒ Even integer = I + (F + F ′) 1
On replacing x by − , we get
⇒ F + F ′ ∈Integer x
Q 0 < F < 1 and 0 < F ′ < 1 ⇒ 0 < F + F ′ < 2 (1 − x + x 2)n
So, F + F ′ = 1 x 2n
⇒ F = 1 − F ′ = 1 − ( 2 − 1)8 C x 2n − C1 x 2n − 1 + C2 x 2n − 2 − C3 x 2n − 3 + …
= 0
F 1 − ( 2 − 1)8 x 2n
So, 1 − = 1 −
1 + ( 2 − 1)4 1 + ( 2 − 1)4 ⇒(1 − x + x ) = C0 x − C1 x 2n − 1 + C2 x 2n − 2
2 n 2n

[1 + ( 2 − 1)4 ] [1 − ( 2 − 1)4 ] − C3 x 2n − 3 + … …(ii)


=1 −
1 + ( 2 − 1)4 Q C0 C1 − C1 C2 + C2C3 − … = Coefficient of x 2n + 1
= 1 − [1 − ( 2 − 1)4 ] = ( 2 − 1)4 in (C0 x 2n − C1 x 2n − 1 + C2 x 2n − 2 − C3 x 2n − 3 + …)
Hence, option (c) is correct. × (C0 + C1 x + C2 x 2 + C3 x 3 + … )
= Coefficient of x 2n + 1 in (1 + x + x 2)n (1 − x + x 2)n
(1 − px)−1
24. If = a 0 + a1 x + a 2 x 2 + a 3 x 3 + … , = Coefficient of x 2n + 1 in ((1 + x 2)2 − x 2)n
(1 − qx)
= Coefficient of x 2n + 1 in (1 + x 4 + x 2)n = 0
then a n = [20 April 2019, Shift-I]
pn + 1 − q n + 1 pn + 1 − q n + 1 Q Coefficient of x k in the expansion of
(a) (b) (1 + x 2 + x 4)n exists if k is even, otherwise it will
q− p p−q
be zero and (2n + 1) is odd.
pn − q n pn − q n
(c) (d) Hence, option (b) is correct.
q− p p−q
26. If x is so small that x 2 and higher powers of
Sol. (b) x can be neglected, then the approximate
(1 − px)−1 1
Since, = a 0 + a1 x + a 2 x 2 + a 3 x 3 + … −2
(1 − qx)  3 2  2x
value of 1 + x  1 −  is
Q(1 − px)−1 = 1 + px + p 2 x 2 + p 3 x 3 + … + p n x n + …
 4   3
[20 April 2019, Shift-II]
64 AP EAMCET Chapterwise Mathematics

41 + 24 x 41 − 24 x 24 + 41x 24 − 41x  − 3  − 3 − 1  − 3 − 2 ...  − 3 − r + 1  − 2


r
(a) (b) (c) (d)
         
41 41 24 24  2  2   2   2   5
=
Sol. (c) r!
If x is so small that x 2 and higher powers of x Comparing with general term of (1 + x)n , n ∈ R
can be neglected, then n(n − 1) (n − 2) K (n − r + 1) x r
(1 + x)n = 1 + nx. ∴
r!
1/ 2 −2
So, 1 + x  1 − x  = 1 + x  1 +
3 2 3 4x   − 3  − 3 − 1 ...  − 3 − r + 1  − 2
r
        
 4   3   8  3  2  2   2   5
3 4x =
=1 + x + [on neglecting the x 2 term] r!
8 3 3 2
24 + 41 x ⇒n = − , x = −
= 2 5
24 − 3/ 2 − 3/ 2 3/ 2
∴ x + = 1 −  =   =   =
Hence, option (c) is correct. 8 2 3 5 5 5
5  5  5  3 3 3
27. Let a 0 , a1 , a 2 ,K a n ∈ R be in an arithmetic 5x + 8 5 5 25 5
⇒ = ⇒ 5x + 8 =
progression and let C 0 , C1 , C 2 , K ,C n be the 5 3 3 3 3
n
binomial coefficients. Then ∑ ak ⋅C k = 29. If ‘a’ is the middle term in the expansion of
k=0
(2 x − 3 y)8 and b, c are the middle terms in the
[21 April 2019, Shift-I]
1 expansion of (3 x + 4 y)7 , then the value of
(a) (a0 + an ) (b) (a0 + an ) ⋅ 2 n − 1 b+ c
2 , when x = 2 and y = 3, is
(c) (a0 + an ) (d) 0 a
[21 April 2019, Shift-II]
Sol. (b) 1 2
n (a) (b) (c) 1 (d) 2
∑ a k ⋅ Ck = a 0 C0 + a1 C1 + a 2C2 + K + a n Cn 2 3
k =0
= a 0 C0 + (a 0 + d)C1 + (a 0 + 2d)C2 Sol. (d)
+ K + (a 0 + nd)Cn In the expansion of (2x − 3y)8 middle term is 5th
Where d is an common difference of an AP term.
= a 0 (C0 + C1 + K + Cn) + ⇒ a = 8 C4 (2x)4 (3y)4 (−1)4
d(C1 + 2C2 + 3C3 + K + n Cn) =
8!
× 24 × 34 × x 4 y 4
= a 0 ⋅ 2n + d(n ⋅ 2n − 1) 4! 4!
= 70 × 24 × 34 × 24 × 34 (Q x = 2 ,y = 3 )
= 2n − 1 [2a 0 + nd] = (a 0 + a n) 2n − 1
Similarly, in the expansion of (3x + 4 y)7, 4th
3 3 ⋅7 3 ⋅7 ⋅9
28. If x = + + + K , then and 5th terms are middle terms.
10 10 ⋅15 10 ⋅15 ⋅ 20 ∴ b = 7C3(3x)4 (4 y)3 = 35 × 37 × 210
5x + 8 = [21 April 2019, Shift-I]
5 5 5 5 3 3 25 5 and c = 7C4 (3x)3(4 y)4 = 35 × 37 × 211
(a) (b) (c) (d) Now, b + c = 35 × 37 × 210 + 35 × 37 × 211
3 3 3 5 3 3
Sol. (d) = 35 × 37 × 210 (1 + 2) = 35 × 38 × 210
Given, b + c 35 × 38 × 210 210
∴ = = 9 =2
3 3⋅ 7 3⋅ 7 ⋅ 9 a 70 × 28 × 38 2
x= + + +K
10 10 ⋅15 10 ⋅15⋅ 20
3⋅ 5 3⋅ 5⋅ 7 3⋅ 5⋅ 7 ⋅ 9 30. The first negative coefficient in the terms
x= + + + ... 21
5⋅10 5⋅10 ⋅15 5⋅10 ⋅15⋅ 20
3⋅ 5⋅ 7 ... (2r + 1) occurring in the expansion of (1 + x) 5 is
∴ Tr + 1 = r
5 ⋅1 ⋅ 2⋅ 3 ... r [21 April 2019, Shift-II]
−6160 −416 −616 −616
7 
...  r + 
1 (a) (b) (c) (d)
r 3 / 2⋅ 5 / 2⋅ 15625 3125 57 56
 2 2  2
= 
 5 r! Sol. (c)
Binomial Theorem 65
n(n − 1) 2 Now, coefficient of x 4 is
Q (1 + x)n = 1 + nx + x
2!  1 1   1 1  
n(n − 1)(n − 2) 3  2  2 −1  x 2    −1  2  2  
2
+ x + …… 
2 2  x
3! 2   − 3   
 2!  4  2!  9  
21 21 × 16 2  
∴(1 + x)21 / 5 = 1 + x+ 2 x    
5 5 × 2!
= x 4 − + 3 × ×  = x 4 −
1 1 1 1 1 
21 × 16 × 11 3 21 × 16 × 11 × 6 4 +
+ x + x  4 × 16 8 81   
 64 216 
53 × 3! 54 × 4!
∴Coefficient of x 4 is  −
21 × 16 × 11 × 6 × 1 5 1 1 
+ x + 
 64 216 
55 × 5!
21 × 16 × 11 × 6 × 1 × 4 6 −216 + 64 −152 −19
− x + …… = = =
56 × 6! (64)(216) (64)(216) 1728

∴The first negative coefficient in the terms 33. If the coefficients of r th , (r + 1)th and (r + 2)th
occurring in the expansion of (1 + x)21 / 5 is
terms in the expansion of (1 + x)14 are in an
21 × 16 × 11 × 6 × 1 × 4 88704 616
− =− 7 =− 7 arithmetic progression, then r =
56 × 720 5 × 144 5
[22 April 2019, Shift-I]
5 (a) 4 or 10 (b) 5 or 9 (c) 8 or 6 (d) 7
31. The coefficient of x in the expansion of
(x + 2 x + 3) , is
2 5
[22 April 2019, Shift-I] Sol. (b)
(a) 1052 (b) 540 (c) 480 (d) 1020 We know that, if coefficients rth ,(r + 1)th , (r + 2)th
Sol. (a) terms in the expansion of (1 + x)n are in AP, then
Given, (3 + 2x + x 2)5 n2 − (4r + 1)n + 4r 2 − 2 = 0 ⇒ n = 14


n!
(3)p (2x)q(x 2)r …(i) ∴ 196 − 56r − 14 + 4r 2 − 2 = 0
(p !)(q !)(r !) ⇒ 4r 2 − 56r + 180 = 0
Where, p+ q + r = n= 5 ⇒ r 2 − 14r + 45 = 0
For x 5, we have, q + 2r = 5 ⇒ r − 9r − 5r + 45 = 0
2

∴ r = 0, q = 5, p = 0 ⇒ r =1, q = 3, p = 1 ⇒ r(r − 9) − 5(r − 9) = 0


r = 2, q =1, p = 2 ⇒ (r − 9) (r − 5) = 0 ⇒ r = 5 or 9
Putting the above values in Eq. (i), we have 5 5 ⋅7 5 ⋅7 ⋅9
5! 5!
34. If x = + + + ..., then
(3)0 (2)5(1)0 + (3)1 (2)3(1)1 2! 3 3! 32 4 ! 33
0! 5! 0! 1! 3! 1!
x2 + 4 x = [22 April 2019, Shift-I]
5!
+ (3)2(2)1 (1)2 (a) 17 (b) 23 (c) 27 (d) 39
2! 1! 2!
= 32 + 20 × 3 × 8 + 30 × 9 × 2 Sol. (b)
5 5⋅ 7 5⋅ 7 ⋅ 9
= 32 + 480 + 540 = 1052 Given, x= + + + ...
2! 3 3! 32 4! 33
32. If x is so small, that x 5 and higher power of 3
2
 1  = 1 + 3  1  + 3⋅ 5  1  + K
x may be neglected, then the coefficient of x +1+      
1!  3 1!  3 2!  3
x 4 in the expansion of x 2 + 4 − x 2 + 9 , is − 3/ 2
2 + x = 1 − 
2
⇒ ⇒ 2 + x = (3)3/ 2
[22 April 2019, Shift-I]  3
19 −19 43 −43 ⇒ x 2 + 4 x + 4 = 27 ⇒ x 2 + 4 x = 23
(a) (b) (c) (d)
1728 1728 1728 1728
Sol. (b) 35. The coefficient of x 3 in the expansion of
x
Given, x2 + 4 − x2 + 9 is
1 1
(x + 1) (x − 2)2 [22 April 2019, Shift-I]
1 1
 
x2  2 x2  2 9 15 77 15
= (x +
2
4) 2 − (x +2
9) 2 = 21 +  − 31 +  (a) (b) (c) (d)
 4  9 48 48 324 16
66 AP EAMCET Chapterwise Mathematics

Sol. (*) Sol. (d)


The given expression,
x Let x = (1 + y)n − 1 = ny
(x + 1)(x − 2)2 n(n − 1) 2 n( n − 1) (n − 2) 3
+ y + y + ..........
−2 2! 3!
(1 + x)−1 1 − 
x x x
= = 2 1.3  2
2
1 ⋅ 3⋅ 5  2
3
2
4  2 = +   +   + ...........
(1 + x)22 1 − 
x
 5 2!  5 3!  5
2
On comparing first three terms, we get
x  x (+2)(+3)  x  
2
2
= (1 − x + x 2)1 + 2 +    2 n(n − 1) 2 1 ⋅ 3  2
  2  ny = , y =  
4  2 2 5 2! 2!  5
3
[On neglecting the higher degree terms] n(n − 1) (n − 2) 3 1 ⋅ 3⋅ 5  2
and y =  
= (1 − x + x 2)1 + x + x 2 
x 3 3! 3!  5
4  4  From first two relations, we get
Now, coefficient of x 3 in the expansion of 22 
x  − y 2
ny(ny − y) 5  5  1 ⋅ 3  2
(x + 1)(x − 2)2 = =  
2! 2! 2!  5
× coefficient of x 2 in (1 − x + x 2)1 + x + x 2 
1 3  2  2
= ⇒   − y = 1⋅ 3  
4  4   5  5
=  − 1 + 1 =
1 3 3 2 6 4 1
⇒ y = − = − and n = −
44  16 5 5 5 2
1 4
Now, on putting n = − and y = − in LHS of
36. If the coefficients of rth and (r + 1)th terms in 2 5
the expansion of (1 + x)24 are in the ratio third relation, we get
12 : 13, then r is the root of the quadratic  − 1   − 1 − 1  − 1 − 2
      3
 2  2   2   4
equation [23 April 2019, Shift-I] = − 
3!  5
(a) x2 − 5 x + 6 = 0 (b) x2 − 11x + 30 = 0
1  1 + 2  1 + 4 
(c) x − 14 x + 13 = 0
2
(d) x2 − 14 x + 24 = 0     3 3
 2   2  3  2 1 ⋅ 3⋅ 5  2
= 2 2   =   =
Sol. (d) 3!  5 3!  5
According to given information, RHS
24! −
1
5+1
So, x = 1 − 
24 4 2 1
Cr − 1 12 (r − 1)! (25 − r)! 12 −1 = 5 − 1 so, =
= ⇒ =  5 x 4
24
Cr 13 24! 13
r ! (24 − r)! 1 5+1 5 5− 3
So, x + = 5 −1 + =
r 12 x 4 4
⇒ = ⇒ r = 12
25 − r 13 Hence, option (d) is correct.
and since, x 2 − 14 x + 24 = 0 38. The coefficient of x 4 in the expansion of
⇒ (x − 12) (x − 2) = 0 1
is
⇒ x = 2, 12 (1 − x)(1 − 2 x)(1 − 3 x) [23 April 2019, Shift-I]
Hence, option (d) is correct. 601
(a) 602 (b) 301 (c) (d) 302
2 3 2
2 1⋅ 3  2 1⋅ 3 ⋅ 5  2
37. If x = +   +   + ..., then Sol. (b)
5 2!  5 3!  5
1
1 Since,
x+ = (1 − x)(1 − 2x)(1 − 3x)
x [23 April 2019, Shift-I]
= (1 − x)−1 (1 − 2x)−1 (1 − 3x)−1
1+ 5
(a) (b) 3 = (1 + x + x 2 + x 3 + x 4)
4
5 5+ 3 5 5−3 (1 + 2x + 4 x + 8 x 3 + 16 x 4)
2
(c) (d)
4 4 (1 + 3x + 9 x 2 + 27 x 3 + 81 x 4)
Binomial Theorem 67

[Expand till x 4 -term because coefficient of x 4 is   3b


(20)
required and (1 − ax)− n    20 
× 4 Q b = , a = 
2a 2 1
∴ r=  =
= 1 + ax + a 2 x 2 + a 3 x 3 + ............] 3b 1 + 4 
 3 4 
 1+   
So, coefficient of x 4 is  2a
81 + 54 + 36 + 24 + 16 + 27 + 18 + 12 + 8 + 9 + r = 16
6 + 4 + 3 + 2 + 1 = 301 ∴greatest term of (2a − 3b)19 is
Hence, option (b) is correct. 16
C16 × 219 ⋅ a19  
3b
= 19
39. The number of rational terms in the  2a 
binomial expansion of (4 5 + 5 4 )100 is 19
C3 × 219 ×   × (4)16 [Q 19C16 =
1
= 19 19
C3]
[22 April 2018, Shift-I]  4
(a) 50 (b) 5 (c) 6 (d) 51 1
= 19C3 × 219 × 38 × 232 = 19C3 × 213
Sol. (c) 2
We have, 41. If a > 0 and the coefficient of x 2 in the
100
100 − r
( 5+
4 5 100
4) = ∑ 100 4
Cr ( 5) 5
( 4) r
 c
6
r =0 expansion of  ax 3 +  is 60, then ac 2 =
100
100 − r r
100
 x
= ∑ 100
Cr 5 4 4 5 = ∑ Tr + 1 [22 April 2018, Shift-II]
r =0 r =0
100 − r r (a) 2 (b) 3 (c) 4 (d) 5
Where, Tr + 1 = 100
Cr 5 4 45 Sol. (a)
100 − r r
General term, Tr + 1 = 6Cr (ax 3)6 − r  
c
Clearly, Tr + 1 will be an integer if and
4  x
r
are integers. This is possible when = 6Cr a 6 − r c r x18 − 3r − r
5
= 6Cr a 6 − r c r x18 − 4 r .
100 − r is a multiple of 4 and r is a multiple of 5
⇒100 − r = 0, 4, 8, 12, K , 96, 100 and According to the question,
r = 0, 5, 10, ..., 100 18 − 4r = 2 ⇒ 4r = 16
⇒ r = 0, 4, 8, 12, K , 100 and r = 0, 5, 10, 100 r = 4.
⇒ r = 0, 20, 40, 60, 80, 100 So, 6 C4 a 6 − 4 c 4 = 60 ⇒ 15 a 2c 4 = 60
Hence, there are 6 rational terms. ⇒ a 2c 4 = 4 ⇒ ac 2 = ± 2
Q a> 0
40. The numerically greatest term in the
1 ∴ ac 2 = 2
binomial expansion of (2 a − 3 b)19 when a =
4 3 3 ⋅5 3 ⋅5 ⋅ 7
2
and b = is 42. If x = + + + ..., then
3
[22 April 2018, Shift-I] 4 ⋅ 8 4 ⋅ 8 ⋅12 4 ⋅ 8 ⋅12 ⋅16
1 2x + 5x =
2
[22 April 2018, Shift-II]
(a) 19 C 5 ⋅ 211 (b) 19 C 3 ⋅
211 7 7 7
(a) (b) 7 (c) (d)
1 8 16 4
(c) 19 C 4 ⋅ (d) 19 C 3 ⋅ 213
213 Sol. (a)
Sol. (d) 3 3⋅ 5 3⋅ 5⋅ 7
x= + + + ........
19 4 ⋅ 8 4 ⋅ 8 ⋅12 4 ⋅ 8 ⋅12⋅16
We have, (2a − 3b)19 = 219 ⋅ a19 1 − 
3b
 n(n − 1) 2 n(n − 1) (n − 2)
2a  Let, (1 + y)n = 1 + ny + y +
2! 3!
We know that, the r th term of greatest term of n(n − 1)(n − 2)(n − 3) 4
+ y + .....
 (n + 1) | x | 4!
(1 + x)n =  
 1 + |x|  If we compare with third term on words, the
n(n − 1) 2 3 n(n − 1)(n − 2)
3b y = ,
Here, n = 19, x = − 2! 4.8 n!
2a
68 AP EAMCET Chapterwise Mathematics

3⋅ 5 1  3 3⋅ 5 3⋅ 5⋅ 7
y3 = =  − + − ...
4 ⋅ 8 ⋅12 3  4 4 ⋅ 8 4 ⋅ 8 ⋅12 
n(n − 1) (n − 2) (n − 3) 4 3⋅ 5⋅ 7  3 5 3 5 7 
and y = . . .
1  3 1 2 2 1  
2 3
4! 4 ⋅ 8 ⋅12⋅16 2 2 2  1
=  × −   +   .... … (i)
On solving, we are getting. So, 32 2 2!  2 3!  2 
x = (1 + y)n − 1 − ny  
−1 / 2 Now, we know that
1 −1
= 1 −  − 1 −  −   
1
n(n + 1) 2
 2  2  2  (1 + x)− n = 1 − nx + x .....
2!
1 5
⇒ 2 −1 − = 2 − n(n + 1) 2
4 4 ⇒ nx − x + .... = 1 − (1 + x)− n … (ii)
2!
2x 2 = 2 2 +
25 5 25
Now, − =4+ −5 2 3 1
 16 2  8 Put n = and n =
2 2
25
and 5x = 5 2 − 3 5
. 2
4 3 1 2 2  1  + ....
25 25 ⇒ . −  
So, 2x + 5x = 4 +
2
−5 2+ 5 2− 2 2 2!  2
8 4 −3/ 2 − 3/ 2
= 1 − 1 +  = 1 −  
1 3
32 + 25 − 50 7
= =  2  2
8 8 3/ 2
2 2 3 3−2 2
= 1 −  
2
=1 − =
43. The number of rational terms in the  3 3 3 3 3
144
 1 1
Put in Eq. (i), we get
expansion of 3 4 + 7 6  is
  1 3 3 −2 2 3 3 −2 2
  S=   ⇒S=
[23 April 2018, Shift-I] 3 3 3  9 3
(a) 33 (b) 23 (c) 12 (d) 13
45. The coefficient of x 50 in the expansion of
Sol. (d) (1 + x)100 + 2 x (1 + x)99 + 3 x 2(1 + x)98 +
In the expansion of (31 / 4 + 71 / 6)144
.....+101 x 100 , is [23 April 2018, Shift-II]
General term is
(a) 100 C 50 (b) 101C 50 (c) 102 C 50 (d) 103 C 50
Tr + 1 = 144
Cr (31 / 4)144 − r (71 / 6)r
144 − r r Sol. (c)
= 144
Cr (3) 4 76 Let S = (1 + x)100 + 2x(1 + x)99 + 3x 2(1 + x)98
For rational terms 144 − r is divisible of 4. + K + 101 x100
∴ r = 0, 4, 8, .... x
S = x(1 + x)99 + 2x 2(1 + x)98 + K
and r is divisible by 6 1+ x
∴ r = 0, 6, 12, ..... x101
+ 100 x100 + 101
So, common value of r will be multiple of 12. 1+ x
∴ i = 0, 12, 24, ... 144 ⇒
S
= (1 + x) + x(1 + x) + x (1 + x)
100 99 2 98
total values of r = 13 1+ x
∴Number of rational terms = 13 x101
+ K + x100 − 101
1 5 5 ⋅7 1+ x
44. − + − ... =
4 4 ⋅ 8 4 ⋅ 8 ⋅12 [23 April 2018, Shift-I]   x  101 
(1 + x)100    − 1
3 3 −2 5 2 3−3 2   1 + x  
(a) (b) x101
9 3 9 3 = − 101
x
−1 1+ x
3 3 −2 2 2 3−3 5
(c) (d) 1+ x
9 3 9 3
⇒ S = (1 + x)102 − x102 − 102x101
Sol. (c)
So, coefficient of x 50 in the expansion of
1 5 5⋅ 7
Let S = − + − ... S = 102C50 .
4 4 ⋅ 8 4 ⋅ 8 ⋅12
Binomial Theorem 69

5 5×7 5×7 ×9 Sol. (b)


46. If α = + + + .... , then
2! × 3 3! × 3 2
4 ! × 33 Using binomial expansions and neglecting x 2
and higher powers, we get
α 2 + 4α = [23 April 2018, Shift-II]  1 1

(a) 21 (b) 23 (c) 25 (d) 27  21 + x  2 + 21 − x  3 


  4  8 
Sol. (b)  
3
Since, 1 − 2x  2
n(n − 1) 2 n(n − 1) (n − 2) 3  
(1 + x)n = 1 + nx + x + x  3
2! 3! −
3

= 2 1 + ⋅ + 1 − ⋅  1 −
n(n − 1) (n − 2) (n − 3) 4 1 x 1 x 2x  2
+ x + ....  
4! 2 4 3 8   3
x 3 − 2x  
1 − ⋅ 
On comparing
= 2 2 + −
x
n(n − 1) 2 n(n − 1) (n − 2) 3 5× 7 
8 24   2  3  
5 
x = and x =
2! 2! × 3 3! 3! × 32
= 2 2 +
2x 
2 [1 + x]
So, x = − and n = − 3 / 2  24 
3
= 2 2 +  [1 + x] = 2 2 + 2x + 
∴ (1 − 2 / 3)− 3/ 2 = 1 + 1 + α ⇒ 33/ 2 = 2 + α x x
 12  12
⇒ α + 4α + 4 = 27
2
⇒ α + 4α = 23.
2

= 2 2 +
25x 
47. The coefficient of x 4 in the expansion of  12 
6
(1 + x − x 2 − x 3)11 is [24 April 2018, Shift-I] When, x =
25
(a) 990 (b) 220 (c) − 220 (d) − 385
= 2 2 +
25 6 
= 2  2 +  = 2⋅ = 5
1 5

Sol. (c)  12 25  2 2
(1 + x − x 2 − x 3)11
49. The coefficient of x 4 in the power series
⇒ ( + x) − x 2(1 + x)]11
[11
x2 − 1
⇒ [(1 − x ) (1 + x)]
2 11 expansion of is
(x 2 + 1) (x 2 + 2)
11
⇒ [(1 − x) (1 + x)2] [24 April 2018, Shift-I]
⇒ (1 − x) ⋅ (1 + x)
11 22 15 15 13 77
(a) (b) (c) − (d)
4 16 4 8 324
The coefficient of x can be given,
4 Sol. (c)
= ∑ (− 1)r ⋅11 Cr ⋅22 C4 − r
r =0 Using binomial expansions,
−1
⇒ 11 C0 × 22
C4 − 11
C1 ⋅22 C3 (x 2 − 1) 1  x2 
= (x 2 − 1) (1 + x 2)− 1 1 + 
+ 11
C2 ⋅22 C2 − 11
C3 ⋅22 C1 + 11
C4 ⋅22 C0 (x + 1) (x + 2) 2
2 2
 2
⇒ 7315 − 16940 + 12705 − 3630 + 330 1 2  x2 x4  
= (x − 1) (1 − x + x ...) 1 −
2 4
+ K 
⇒ 20350 − 20570 = − 220. 2  2 4 
48. If| x| is so small that x 2 and higher powers 1   x 2
x 4
x 4  
= (x 2 − 1) 1 − + − x2 + + x 4 ... 
of x may be neglected, then the approximate 2  2 4 2 
4 + x + 38 − x 6 1 2 
(x − 1) 1 − x 2 + x 4 K
3 7
value of when x = is =
3
25 2   2 4  
 2x 2
1 −  =  −1 + x 2 − x 4 + x 2 − x 4 ...
1 3 7 3
 3
[24 April 2018, Shift-I] 2  2 4 2 
(a) 6 (b) 5 taking coefficient of x 4 ,
1 − 13 − 13
=  − −  = 
2 5 1 7 3
(c) (d)  = .
3 6 2  4 2  2 4  8
8
Permutations and
Combinations
1. Find the number of ways of selecting 4 pens = (n + 1)! − 1! = 11! − 1! (given)
and 3 pencils from a packet of 8 pens and So, n = 10
5 pencils, [17 Sep. 2020, Shift-I] Now, maximum value of 10 Cr occurs when
(a) 700 (b) 8 P4 × 5 P3 r= =
n 10
=5
(c) 8 P4 + 5 P3 (d) 700 × 8 P4 2 2
10!
Now, 10 C5 = = 252
Sol. (a) 5!(10 − 5)!
Number of ways = 8 C4 × 5C3 = 700 n

2. In how many ways one can send 6 new-year


5. If n is a positive integer, then ∑r ⋅ Cr =
r =1
greeting cards to 4 people? [17 Sep. 2020, Shift-I]
[17 Sep. 2020, Shift-II]
(a) 360 (b) 180 (c) 720 (d) 90
(a) 2 n − 1 (b) n2 n − 1 (c) n2 n + 1 (d) 2 n + 1
Sol. (a)
Number of ways = 6P4 = 360
Sol. (b)
n
Σ r ⋅ Cr = 1C1 + 2C2 + 3C3 + ...+ ...+ nCn = n.2n −1
3. Find the number of different garlands that r =1

can be prepared using 5 different coloured


flowers. [17 Sep. 2020, Shift-I]
6. Letters of the word ‘GOVIND’ are permuted
and arranged as in a dictionary, then the
(a) 120 (b) 60 (c) 119 (d) 59
number of words that appear after the word
Sol. (*) ‘GOVIND’ is .......... [17 Sep. 2020, Shift-II]
4!
Number of different garlands = = 12 (a) 519 (b) 510 (c) 506 (d) 511
2
Sol. (c)
4. If 1 × 1! + 2 × 2! + 3 × 3! + K + n × n! = 11! − 1, Word is ‘GOVIND’
then the maximum value of nC r is total words possible = 6! = 720 words
[17 Sep. 2020, Shift-II] Now we find rank of word GOVIND,-
(a) 462 (b) 252 (c) 162 (d) 512 Number of words starting with
D = 5! = 120 words
Sol. (b)
Number of words starting with
1 × 1! + 2 × 2! + K + n × n!
G and second letter D
= (2 − 1) × 1! + (3 − 1) × 2! + K+ [(n + 1) − 1] × n!
= {2 × 1! + 3 × 2! + 4 × 3! + K(n + 1) × n!} GD = 4!
− {1 × 1! + 1 × 2! + 1 × 3! + K + 1 × n!}
= 24 words
Permutations and Combinations 71

G and second Letter I Sol. (d)


Since,
GI = 4!= 24words
(1 + x)n = C0 + C1 x + C2 x 2 + C3 x 3 + …… + Cn x n
G and second Letter N On putting x = 1 and x = −1 respectively, we get
2n = C0 + C1 + C2 + C3 + ...... + Cn
G = 4!= 24words
N and 0 = C0 − C1 + C2 − C3 + ...... + (−1)n Cn
If n is even , then
Now words with G and next letter O and IIIrd
Letter D C0 + C1 + C2 + C3 + ...... + Cn = 2n …(i)
and C0 − C1 + C2 − C3 + ...... + Cn = 0 …(ii)
G = 3! = 6 words
D By adding and subtracting Eqs. (i) and (ii), we
O
get
Now words with G and next letter O and third C0 + C2 + C4 + ...... + Cn = 2n −1
Letter I. and C1 + C3 + C5 + ...... + Cn −1 = 2n −1
G = 3! = 6 words
OI 8. The number of different words that can be
formed from the letters of the word
Similarly, “INTERMEDIATE” such that two vowels
never come together, is [18 Sep. 2020, Shift-I]
G O = 3! = 6 words 6! 7! 5! 6! 7! 6! 6!
N (a) × (b) × (c) 6! × (d) ×
2 ! 2 ! 3! 2 ! 3! 2 ! 3! 2 ! 2 ! 3!
Now, we have, Sol. (a)
In the given word “INTERMEDIATE” the
G O
D I N = 1 words vowels and consonants are IEEIAE and
V NTRMDT respectively. Now number of ways to
6!
and arrange consonants first is .
2!
G O Now number of ways to arrange six vowels in
D N I = 1 words
V the seven available and favourable positions are
7
P6 7!
G O =
I D N = 1 words 3! 2! 3! 2!
V
6! 7!
So number of required arrangements = ×
And then, 2! 2! 3!

G O 9. In an apartment there are 30 kids. If each


I N D = 1 words kid plays table tennis with other kid, then
V
the total number of matches played by them
So, total words = rank [18 Sep. 2020, Shift-I]
= 120 + 24 × 3 + 6 × 3 + 4 = 214 th words (a) 30
C2 (b) 30
P2
So, Number of words after GOVIND (c) 30
C2 − 1 (d) 30
P2 − 1
= 720 − 214 = 506 Words
Sol. (a)
7. Choose the correct option regarding the The number of required matches played is same
following statements [18 Sep. 2020, Shift-I] as the number ways to choose two kids among
1. C 0 + C 2 + C 4 + K + C n = 2 n − 1 , if n is even 30 kids and it is equal to 30 C2.

2. C1 + C 3 + C 5 + K + C n − 1 = 2 n − 1 , if n is 10. A polygon has 54 diagonals. The number of


even sides of this polygon is .........
(a) 1 is true, 2 is false (b) 1 is false, 2 is true [18 Sep. 2020, Shift-I]
(c) Both 1 and 2 are false (d) Both 1 and 2 are true (a) 12 (b) 15 (c) 16 (d) 9
72 AP EAMCET Chapterwise Mathematics

Sol. (a) Sol. (b)


Let number of sides of the polygon is n, then Crew side = 4 → 4! ways
number of diagonals is Stroke side = 4 → 4! ways
n(n − 3) Total number of ways = 9 C4 × 8 C4 × 4! × 4!
= 54 (given)
2
Hence, option (b) is correct.
⇒ n(n − 3) =108 ⇒ n = 12
14. All the words that can be formed using
11. Find the total number of rectangles on a alphabets A, H, L, U, R are written as in
normal chessboard. [18 Sep. 2020, Shift-II]
dictionary (no alphabet is repeated). Then
(a) 8 C 2 × 8C 2 (b) 8 C 2 + 8C 2 the rank of the word RAHUL is .......
(c) C 2 × C 2
9 9
(d) 9 P2 × 9 P2 [21 Sep. 2020, Shift-I]

Sol. (c) (a) 70 (b) 71


(c) 73 (d) 74
Total number of Rectangles in n × n chess board
= n + 1 C2 × n + 1 C2 Sol. (d)
∴Total Number of Rectangles in 8×8 chess Given alphabets are A, H, L, U, R, so the number
Boord = 9C2 × 9C2 of words starts with letter A = 4! = 24
The number of words starts with letter
Hence, option (c) is correct. H = 4! = 2. 4
10001 × 100 ! The number of words starts with letter
12. =
2 × 1! + 5 × 2 ! + 10 × 3 ! + K + 10001 × 100 ! L = 4! = 24
[18 Sep. 2020, Shift-II] Now, the next word start with R and order wise
1001 10001 101 100001 words are RAHLU and RAHUL. So in dictionary
(a) (b) (c) (d) order the rank of word RAHUL is
1100 10100 110 101000
24 + 24 + 24 + 1 + 1 = 74
Sol. (b) Hence, option (d) is correct.
Here, general term of denominator is
100 100 15. If n P4 = 1680, then n = [21 Sep. 2020, Shift-I]
∑ (n + 1) n! = ∑ [(n + 1) n!− 2n. n!]
2 2
n =1 n =1 (a) 6 (b) 12 (c) 10 (d) 8
100 100
= ∑ [(n + 1)(n + 1)! − n. n!] − ∑ n ⋅ n! Sol. (d)
n =1 n =1
Since, n P4 = 1680
100 100
= ∑ [(n + 1)(n + 1)!− n. n!] − ∑ (n + 1)!− n! ⇒ n(n − 1) (n − 2)(n − 3) = 1680
n =1 n =1
⇒ n(n −1) (n − 2) (n − 3) = 8 × 7 × 6 × 5 ⇒ n = 8
 2⋅ 2! − 1 ⋅ 1!   2 ! − 1! 
 +3⋅ 3! − Hence, option (d) is correct.
2⋅ 2!   +3! − 2! 
 −  n+1
 M   M 
Cr + 1 n − r +1
16. If = , then m =
 +101 ⋅101! − 100 ⋅100!  +101! − 100! n+1
m
    Cr
[21 Sep. 2020, Shift-I]
= [101 ⋅101!− 1] − [101!− 1]
(a) r (b) r − 1 (c) r + 1 (d) 1 − r
Denominator = 100 ⋅101!
10001 × 100! 10001 Sol. (c)
Consider, = n+1
100 × 101 × 100! 10100 Cr + 1 n− r +1
n+1
=
Hence, option (b) is correct. Cr m
(n + 1)!
13. The crew of an 8-oar boat is to be chosen
(r + 1)! (n − r)! n − r + 1
from 12 men, of whom 3 can row on the ⇒ =
(n + 1)! m
stroke side only. The number of ways in
which the crew can be arranged is r !(n − r + 1)!
[18 Sep. 2020, Shift-II] n− r +1 n− r +1
⇒ = ⇒ m =r + 1
(a) 9 C 4 × 8C 3 × 3! × 4! (b) 9 C 4 × 8C 4 × 4! × 4! r +1 m
(c) 8 C 3 × 8C 3 × 4! × 3! (d) 9 C 4 × 9C 4 × 4! × 4! Hence, option (c) is correct.
Permutations and Combinations 73

17. If the number of rectangles formed on a Sol. (b)


chess board is 1296, then the total number Since, it is given that a lock has 3 rings and each
of squares formed on the chess board is ring has 8 digits, so each ring can be rotated in 8
[21 Sep. 2020, Shift-II] ways, therefore total number of different ways
in which 3 rings can be rotated is 8 × 8 × 8 = 83.
(a) 202 (b) 203 (c) 204 (d) 205
Hence, option (b) is correct.
Sol. (c)
It is given that the number of rectangles formed 20. In how many ways a committee of a
in chess board (Let having size n × n) is 1296 6 members can be formed out of 10 members
n+1
∴ C2 × n + 1 C2 = 1296 but always consisting of a specified member?
2 [21 Sep. 2020, Shift-II]
⇒  n(n + 1)  = 1296 = (36)2 ⇒ n(n + 1) = 36 1 19
  (a) 9 P5 9
(b) P5 9
(c) C 5 (d) C5
 2  2 2 2
⇒ n(n + 1) = 72 ⇒ n = 8
Sol. (c)
So, number of squares of any sizes are
As the committee consists a specified member
n2 + (n − 1)2 + (n − 2)2 + ....... + 12 out of 6 members, then we need to select only 5
= 12 + 22 + 32 + ........ + 82 members out of 9 remaining members and it
8 × 9 × 17 can be done is 9 C5 ways. Hence, option (c) is
= = 4 × 3 × 17 = 12 × 17 = 204 correct.
6
Hence, option (c) is correct. 21. If nC 7 = nC 6 , then nC 2 = [22 Sep. 2020, Shift-I]
18. If ‘n’ is a positive integer, then (a) 858 (b) 13 (c) 1 (d) 78
n
n− 2
Σ r ⋅ C r = (.........)2
2
Sol. (d)
r =1 [21 Sep. 2020, Shift-II]
Given, n C7 = n C6
(a) n(n − 1) (b) n (c) n(n + 1) (d) n + 1
⇒ n = 7 + 6 = 13 {If n Cx = n C y ⇒ either x = y
Sol. (c) or x + y = n}
n
∑ r Cr = 1 C1 + 2 C2 + 3 C3 + ...... + n Cn
2 2 2 2 2
13 × 12
r =1
∴ n C2 = 13C2 = = 13 × 6 = 78
2
Q(1 + x)n = C0 + C1 x + C2 x 2 + C3 x 3 + .....+ Cn x n Hence, option (d) is correct.
On differentiating both sides w.r.t. x, we get
n(1 + x)n −1 = 1. C1 + 2C2 x + 3C3 x 2 + ....... + nCn x n −1 22. How many multiples of 5 are there from 10
to 95 including both 10 and 95?
Now, on multiplying by x both sides, we get
[22 Sep. 2020, Shift-I]
nx(1 + x)n −1 = 1. C1 x + 2C2 x 2 + 3C3 x 3 + ...... + nCn x n
(a) 17 (b) 18 (c) 16 (d) 19
Now again on differentiating both sides w.r.t x,
we get
Sol. (b)
There are 18 multiples of 5 from 10 to 95
n[(1 + x)n −1 + (n − 1) x(1 + x)n − 2]
including both 10 and 95 and, they are 10, 15,
= 12 C1 + 22 C2 x + 32 C3 x 2 + .......+ n2Cn x n −1 20, 25, 30, 35, 40, 45, 50, 55, 60, 65, 70, 75, 80, 85,
Put x = 1, we get 90, 95.
12. C1 + 22 C2 + 32 C3 + ...... + n2Cn 23. Find the number of ways of arrangement
= n[2n −1 + (n − 1)2n − 2] 6 red balls and 6 black balls in a row such
= n2 n−2
[2 + n − 1] = n(n + 1)2 n−2 that no two black ball are together.
[22 Sep. 2020, Shift-I]
Hence, option (c) is correct.
(a) 6! × 6! (b) 7 ! × 6!
19. A number lock has 3 rings and each ring has (c) 2 × 6! × 6! (d) 7 × 6! × 6!
8 digits. Total number of different ways in Sol. (d)
which 3 rings can be rotated is Number of ways to arrange 6 red balls are 6!.
[21 Sep. 2020, Shift-II] Now, there are seven positions to place 6 black
(a) 38 (b) 83 (c) 3 × 8 (d) 8 P3 balls, such that no two black balls are together
74 AP EAMCET Chapterwise Mathematics

Sol. (b)
R1 R2 R3 R4 R5 R6 Given,
So, number of ways to arrange 6 black balls are Number of diagonals in a polygon = 170
7 n(n − 3)
P6. = 170
2
Therefore the required number of arrangements
= 6! × 7P6 = 6! × 7! = 7 × 6! × 6! n(n − 3) = 340
n(n − 3) = 20 × 17
24. If the letters of the word “ASSASSINATION” ∴ n = 20
are arranged at random in a row, then the (n − 2) π 18 π 9 π
∴Measure of interior angle = = =
probability that no two A’s come together is n 20 10
equal to [22 Sep. 2020, Shift-I] Hence, option (b) is correct.
25 23 15 17
(a) (b) (c) (d) 27. If on an average 9 mountain climbers out of
26 26 26 26
10 return safely, what is the probability that
Sol. (c) with 5 climbers out, at least 4 will return
The number of all possible ways to arrange the safely? [23 Sep. 2020, Shift-I]
13!
letters of word ASSASSINATION is = 95 × 7 94 × 7 95 94 × 3
3! 4! 2! 2! (a) (b) (c) (d)
50000 50000 100000 50000
Now, the number of ways to arrange the letters
10! Sol. (b)
S, S, S, S, I, N, T, I, O, N is 9
4! 2! 2! The probability of return safely is , so the
10
and the three A’s we must arrange between the probability that out of 5 climbers, atleast 4 will
gaps or at the end positions and it can be done return safely is
in 11 C3. 4 5
C4     + 5C5  
5 9 1 9
10!
× 11 C3  10   10   10 
4! 2! 2!
So, required probability = 94 (5 + 9) 94 × 14 94 × 7 94 × 7
13! = = = =
3! 4! 2! 2! 105 105 5 × 104 50000
11 × 10 × 9 Hence, option (b) is correct.
3! ×
3! 10 × 9 5 × 3 15
= = = = 28. The number of ways of choosing a
13 × 12 × 11 13 × 12 13 × 2 26
committee from four men and six women so
25. If C 2k −1 = C k +1, then find k
12 12 that the committee includes atleast two men
and exactly twice as many women as men is
[22 Sep. 2020, Shift-II]
[23 Sep. 2020, Shift-I]
(a) 3 (b) 6 (c) 9 (d) 4
(a) 90 (b) 92 (c) 94 (d) 96
Sol. (d)
Sol. (c)
It is given that,
Number of Men = 4
12
C2k −1 =12 Ck + 1
Number of Women = 6
So, either 2k − 1 = k + 1 or Men women Number of ways
2k − 1 + k + 1 = 12 2 4 4
C2 × 6C4 = 90
⇒ k = 2or k = 4.
3 6 4
C3 × 6C6 = 4
{∴If n Cx = n C y then either, x = y or x + y = n}
Total Number of commitees = 90 + 4 = 94
Hence, option (d) is correct.
Hence, option (c) is correct.
26. A regular polygon has 170 diagonals. Then
29. From 5 consonants and 5 vowels, how many
the measure of interior angle of the polygon
words can be formed using 3 consonants and
is [22 Sep. 2020, Shift-II]
5π 9π 7π 17 π
2 vowels? [23 Sep. 2020, Shift-I]
(a) (b) (c) (d) (a) 12000 (b) 2000 (c) 20000 (d) 1200
8 10 10 20
Permutations and Combinations 75

Sol. (a) 32. All the letters of the word ANIMAL are
Consonants are 5 permuted in all possible ways and the
Vowels are 5 permutations thus formed are arranged in
number of words = 5C3 × 5C2 × 5! dictionary order. If the rank of the word
= 10 × 10 × 120 = 12000
ANIMAL is x, then the permutation with
rank x, among the permutations obtained by
Hence, option (a) is correct.
permuting the letters of the word PERSON
30. Find the number of marked points on the and arranging the permutations thus formed
plane, if when connected pairwise by line in dictionary order is [20 April 2019, Shift-I]
segments, the total number of line segments (a) ENOPRS (b) NOSPRE
formed is 15. [23 Sep. 2020, Shift-I] (c) NOEPRS (d) ESORNP
(a) 5 (b) 4 (c) 6 (d) 8 Sol. (d)
Sol. (c) If we arrange the letter of word ANIMAL in
Let number of points be n alphabetic order then, we get AAILMN, now to
According to the question, find the rank of word ANIMAL, x (given)
n(n − 1) AA ……… 4!
n
C2 = 15 ⇒ = 15 AI ……… 4!
2
n(n − 1) = 30 ⇒ n(n − 1) = 6 × 5 AL ……… 4!
∴ n= 6 AM ……… 4!
ANA ……… 3!
Hence, option (c) is correct.
ANIA ……… 2!
31. A bag contains 5 blue and an unknown ANIL ……… 2!
number x of red balls. Two balls are drawn ANIMAL ……… 1
at random. If the probability of both of them 107 = x
5
being blue is , then the value of x is equal Now, similarly for word PERSON, if we arrange
14 the letters in alphabetic order, we get ENOPRS,
to [23 Sep. 2020, Shift-I] now to find the word having 107 ways ,
(a) 8 (b) 5 (c) 3 (d) 6 EN ……… 4!
EO ……… 4!
Sol. (c)
EP ……… 4!
Given,
ER ……… 4!
Number of blue balls = 5 ESN ……… 3!
Number of balls = x ESON ……… 2!
Total number of balls = 5 + x ESOP ……… 2!
Number of ways of drawing 2 blue balls = 5C2 ESORNP ……… 1
( 5 + x) 107 same as x.
Number of ways of drawing 2 balls = C2
So, ESORNP is required word.
Given,
Hence, option (d) is correct.
5
P(drawing 2 blue balls) =
14 33. A student is allowed to choose atmost n
5
C
⇒ ( 5 + x )2 =
5

10
=
5 books from a collection of 2 n + 1 books. If the
C2 14 (5 + x) (4 + x) 14 total number of ways in which he can select
2 ×1 atleast one book is 255, then the value of n is
20 5 [20 April 2019, Shift-I]
⇒ 2 = ⇒ x 2 + 9 x + 20 = 56
x + 9 x + 20 14 (a) 4 (b) 5 (c) 6 (d) 7
⇒ x 2 + 9 x − 36 = 0 ⇒ x 2 + 12x − 3x − 36 = 0 Sol. (a)
⇒ x (x + 12) − 3 (x + 12) = 0 ⇒ x = 3 According to given information,
2n + 1
(Q Number of balls can not be negative.) C1 + 2n + 1 C2 + 2n + 1 C3 + …… + 2n + 1
Cn = x
Hence, option (c) is correct. (Let)
76 AP EAMCET Chapterwise Mathematics

Q2n + 1 C0 + 2n + 1
C1 + 2n + 1
C2 + … + 2n + 1 Cn + … 36. Six persons A, B, C, D, E and F are to be
+ 2n + 1 C2n + 1 = 22n + 1 seated at a circular table facing towards the
⇒ 2x + 2n + 1
C0 + 2n + 1
C2n + 1 = 2 2n + 1 centre. Then the number of ways that can be
done if A must have either E or F on his
[Q n Cn − r = n Cr ] immediate right and E must have either F or
⇒ 2x + 1 + 1 = 22n + 1 [Q n C0 = n Cn = 1] D on his immediate right, is
[21 April 2019, Shift-II]
⇒ x=2 2n
− 1 = 255 (given)
(a) 18 (b) 30
⇒ 22n = 256 = 28 ⇒ n = 4 (c) 12 (d) 24
Hence, option (a) is correct. Sol. (a)
34. The number of five digit numbers that are We have two cases :
divisible by 6 which can be formed by (i) Case I : E is to the immediate right of A.
choosing digits from {0, 1, 2, 3, 4, 5}, when A
repetition is allowed, is [20 April 2019, Shift-II]
E
(a) 648 (b) 540 (c) 1296 (d) 1080
Sol. (d) D or F
If number is divisible by 6, then it should be an
even number and divisible by 3 also, so sum of
the digits should be divisible by 3.
Now last digit can be fill by 0 or 2 or 4. First The seat next to E (on right side) can be
place can be filled by 5 choices (except 0) filled in 2 ways i.e. either D or F
second and third can be filled by 6 choices. Remaining 3 seats can be filled in 3!
Now fourth place is filled such that it must be of ways
form 3k or 3k + 1 or 3k + 2 . So it can be fill by 2 ∴Total number of ways = 2 × 3! = 12
choices (0 or 3) as whole number must be
divisible by 3 so, total numbers (ii) Case II : F is to the immediate right of
= 5 × 6 × 6 × 2 × 3 = 1080 A.
Hence, option (d) is correct. A
F
35. There are 20 straight lines in a plane such
that no two of them are parallel and no three
of them are concurrent. If their points of
intersection are joined, then the number of
new line segments formed are
[21 April 2019, Shift-I] Number of ways E can be filled = 3ways
(a) 3420 (b) 14535 (c) 2907 (d) 17955 [Q E can’t occupy sit on left of A as
there will be no space for D]
Sol. (b)
We know that, if there are n straight lines in a Number of ways for D = 1 ways
plane, no two of which are parallel and no three Remaining seats can be filled in 2 ways
passes through the same point and their point
of intersection are jointed then number of fresh ∴Total number of ways = 3 × 1 × 2 = 6
lines thus introduced is ∴Required total number of ways
1
= n(n − 1) (n − 2) (n − 3) = 12 + 6 = 18 ways
8
1 37. Number of ways of forming a committee of
= × 20 × (20 − 1) × (20 − 2) × (20 − 3) 6 members out of 5 Indians, 5 Americans
8
1
and 5 Australians such that there will be
= × 20 × 19 × 18 × 17 atleast one member from each country in the
8
116280
committee is [21 April 2019, Shift-II]
= = 14535 (a) 3375 (b) 4375 (c) 3875 (d) 4250
8
Permutations and Combinations 77

Sol. (b) Since, students can be arranged in m! ways,


Required cases are as follow: ∴ Total number of ways = m!× nm ways
Indians Americans Australians 39. Consider the following statements
1 1 4 I : The number of non-trivial even divisors of
1 2 3 the number 2 α 1 3 α 2 4 α 3 5 α 4 6 α 5 , is
1 3 2 (α1 + 1) (α 2 + 1)(α 3 + 1)(α 4 + 1)(α 5 + 1) − 2
1 4 1 II : The number of non-trivial odd divisors of
2 1 3
the number 2 α 1 . 3 α 2 . 4 α 3 . 5 α 4 . 6 α 5 , is
α 2 + α 4 +α 5 + α 2α 4 + α 4α 5. Then
2 2 2
[22 April 2019, Shift-I]
2 3 1 (a) I is false and II is false
3 1 2 (b) I is true and II is true
3 2 1 (c) I is false and II is true
(d) I is true and II is false
4 1 1
Sol. (c)
∴ Required number of committee
Given number = 2α 1 3α 2 4α 3 5α 4 6α 5
= 5C1 × 5C1 × 5C4 + 5C1 × 5C2 × 5C3 + 5C1 × 5C3
= 2α1 + 2α 3 + α 5 α 2 + α 5 α 4
3 5
× 5 C2
Number of non-trivial even divisors
+ C1 × C4 × C1 + C2 × C1 × C3 + C2 × C2
5 5 5 5 5 5 5 5
= (α1 + 2α 3 + α 5)(α 2 + α 5 + 1) (α 4 + 1) − 1
× 5C2 + 5C2 × 5C3 × 5C1 + 5C3 × 5C1 × 5C2 + 5C3
And, number of non-trivial odd divisors
× 5C2 × 5C1 + 5C4 × 5C1 × 5C1 = (α 2 + α 5 + 1)(α 4 + 1) − 1
= 5 × 5 × 5 + 5 × 10 × 10 + 5 × 10 × 10 + 5 × 5 × 5 = α 2 + α 4 + α 5 + α 2α 4 + α 4α 5 + 1 − 1
+ 10 × 5 × 10 + 10 × 10 × 10 + 10 × 10 × 5 + 10 × 5 ∴Statement I is false and II is true.
× 10 + 10 × 10 × 5 + 5 × 5 × 5
= 125 + 500 + 500 + 125 + 500 + 1000 + 500 + 500 40. I. The number of all ten digited numbers
+ 500 + 125
that can be formed with all the distinct digits
= 4375 and which are divisible by 4 is15 × 8 ! .
II. The number of positive integers that can
38. In an examination hall there are ‘mn’ chairs be formed by using the digits 0, 1, 2, 3, 4, 5
in m rows and n columns. The number of without any repetition is 1630.
ways in which m students can be seated [22 April 2019, Shift-II]
such that no row is vacant is (a) Only I is true
[22 April 2019, Shift-I] (b) Only II is true
(a) mn n! (b) nm m! (c) mm n! (d) nn m! (c) Both I and II are true
Sol. (b) (d) Both I and II are false
Given that these is ‘mn’ chairs in m rows and n Sol. (b)
columns. I. We have,
m 0, 1, 2, 3, 4, 5, 6, 7, 8, 9
r
c The number formed can be divisible by 4 when
last two digits has to be divisible by 4.
n There are 16 last two digits numbers that are
divisible by 4 without using zero.
So, total number of such 10 digits numbers
= 7 × 7! × 16
Again, there are 6 last two digits numbers that
∴Number of ways in which one student can are divisible by 4 with using zero.
seat in 1st column = n So, total number of such 10 digit numbers
So, similarly m students can seat in nm ways. = 8! × 6
78 AP EAMCET Chapterwise Mathematics

∴ Required numbers Sol. (b)


= 7 × 7! × 16 + 8! × 6 On arranging the letters of world REPEAT in
= 7 × 8! × 2 + 8! × 6 alphabetic order, we get AEEPRT.
= 8!(14 + 6) = 20 × 8! Now, the number of words starts with R.............
5!
So, statements I is wrong. = = 60
II. Six digit numbers = 5 × 5 × 4 × 3 × 2 × 1 = 600 2!
Five digit numbers = 5 × 5 × 4 × 3 × 2 = 600 The number of words starts with E .............
= 5! = 120
Four digit numbers = 5 × 5 × 4 × 3 = 300
The number words starts with P ..............
Three digit numbers = 5 × 5 × 4 = 100
5!
Two digit numbers = 5 × 5 = 25 = = 60
2!
One digit number = 5 = 5
The number of words starts with RA ..............
∴Required number 4!
= = 12
= 600 + 600 + 300 + 100 + 25 + 5 = 1630 2!
So, statement II is correct. The number of words starts with REA ...............
= 3! = 6
41. A man has 5 male and 4 female relatives. His
wife has 4 male and 5 female relatives. The The number of words starts with REE ...............
= 3! = 6
number of ways in which they can invite 5
male and 5 female relatives so that 5 of them The number of words starts with
REPA..................
are man’s relatives and remaining 5 are his
= 2! = 2
wife’s relatives [22 April 2019, Shift-II]
The number of words starts with REPEAT
(a) 5426 (b) 5226 (c) 5526 (d) 5626 = 1! = 1
Sol. (d) Now, the rank of word REPEAT is
= 60 + 120 + 60 + 12 + 6 + 6 + 2 + 1 = 267
Man Wife
Hence, option (b) is correct.
5 male 4 male
43. 15 persons are sitting around a circular table.
4 female 5 female
The number of ways of selecting three
5M 5F persons at a time from them, such that the
4 M, 1 F 1 M, 4 F selected three did not sit together at one
place is [23 April 2019, Shift-I]
3 M, 2 F 2 M, 3F
(a) 455 (b) 15 (c) 45 (d) 440
2 M, 3 F 3 M, 2 F
Sol. (d)
1 M, 4 F 4 M, 1 F The total number of ways selecting 3 persons
from 15 persons seat around a circular table is
Total no of ways 15
C3.
= 5C5 × 5C5 + (5 C4 × 4 C1 × 4 C1 × 5C4) Now, number of ways selecting 3 persons who
+ (5 C3 × 4 C2 × 4 C2 × 5C3)+ (5 C2 × 4 C3 × 4 C3 × 5 C2) sit together at one place is 15.
+ (5 C1 × 4 C4 × 4 C4 × 5C1) So, required number of ways = 15C3 − 15
= 1 × 1 + (5 × 4 × 4 × 5) 15 × 14 × 13
= − 15 = 35 × 13 − 15 = 455 − 15 = 440
+ (10 × 6 × 6 × 10) + (10 × 4 × 4 × 10) 3× 2
+ (5 × 1 × 1 × 5) = 5626 Hence, option (d) is correct.

42. If all the letters of the word REPEAT are 44. If x is the number of ways in which six
permuted in all possible ways and if the six women and six men can be arranged to sit in
letter permutations thus formed are a row such that no two women are together
arranged in the dictionary order, then the and if y is the number of ways they are
rank of the word REPEAT is seated around a table in the same manner,
[23 April 2019, Shift-I] then x : y = [22 April 2018, Shift-I]
(a) 133 (b) 267 (c) 266 (d) 132 (a) 12 : 1 (b) 42 : 1 (c) 16 : 1 (d) 6 : 1
Permutations and Combinations 79

Sol. (b) Now, number of numbers starts with 36, 35 and


6 Boys can be seated in a row in 6P6 ways = 6!. 34 is 6, 6 and 6 respectively.
Now, in the 7 gaps 6 girls can be arranged in 7P6 The 90th number is 34256.
ways. So, 89th number is 34265.
∴ x = 6! × 7P6 = 6! × 7! 47. Three parallel straight lines L1 , L2 and L3 lie
6 Boys can be seated in a circle in
on the the same plane. Consider 5 points on
(6 − 1)! ways = 5! L1 , 7 points on L 2 and 9 points on L 3 . Then
Now, in the 6 gaps 6 girls can be arranged the maximum possible number of triangles
in 6P6 ways.
formed with vertices at these points, is
∴ y = 5! × 6P6 = 5! × 6! [22 April 2018, Shift-II]
Now, x : y = 6! × 7! : 5! × 6!
(a) 1330 (b) 1200
⇒ x : y = 7! : 5! (c) 1201 (d) 129
⇒ x : y = 7 × 6 × 5! : 5!
Sol. (c)
⇒ x : y = 42 : 1
Number of triangles if one vertex at each line is
45. The number of 5-letter words that can be = 5C1 × 7C1 × 9C1 = 315
formed by using the letters of the word Number of triangles if two vertices at L1 and
SARANAM is [22 April 2018, Shift-I] remaining at either L2 or L3 is = 5C2 × 16C1 = 160
(a) 1120 (b) 6720 Number of triangles if two vertices at L2 and
(c) 480 (d) 720 remaining at either L3 or L1 is = 7C2 × 14 C1 = 294
Sol. (c) and number of triangles if two vertices at L3 and
There are 7 letters in the word ‘SARANAM’ of remaining at either L1 or L2 is :
which there are 3 A and all other are distinct.
9
C2 × 12C1 = 432
The five letter words may consist of So, total maximum possible number of triangles
(i) all different letters (using S, A, R, N, M) = 1201
∴ Required words
48. Let S = {0 , 1, 2, 3, ..., 100}. The number of
= 5! = 120
(ii) 2 alike and other different
ways of selecting x , y ∈ S such that x ≠ y and
x + y = 100 is [23 April 2018, Shift-I]
(using 2 A and 3 other)
(a) 51 (b) 40 (c) 50 (d) 100
∴Required words
5!
= 4 C3 × = 4 ×
120
= 240
Sol. (c)
2! 2 Let S = {0, 1, 2, 3, ... , 100} x , y ∈ s.
(iii) 3 alike and other different (using 3A and 2 and x + y = 100
other) If x = 0, y = 100
5! 4 × 3 120 x = 1, y = 99
∴Required words = 4 C2 × = × = 120 x = 49 ⇒ y = 51
3! 2 × 1 6
In this way, the total pairs in 50.
∴Total words = 120 + 240 + 120 = 480 So, the number of required ways = 50
46. If all the digits in the number 53426 are 49. If all the seven letters of the word LEADING
permuted in all possible ways and are are permuted in all possible ways and the
arranged in decreasing order, then the words thus formed are arranged as in the
number having rank 89, is dictionary order, then the word in 2017 th
[22 April 2018, Shift-II] place is [23 April 2018, Shift-I]
(a) 34265 (b) 34256 (a) ELIGDAN (b) ELNADGI
(c) 43526 (d) 43265 (c) ELINADG (d) ELNDAGI
Sol. (a) Sol. (b)
Number of numbers starts with digit 6, 5 and 4 When letters of LEADING are arranged in
is 24, 24 and 24 respectively. ascending order then letters are A, D, E, G, I,
So, total numbers is 72. L, N
80 AP EAMCET Chapterwise Mathematics

Now, words starts with A is 6! = 720, ie. A ........ Case II. If 2 men and 6 women are selected
Similarly, words starts with D = 6! = 720 Number of ways = 10 C2 × 8 C6
Now, words starts with E is also 6! = 720. 10 × 9 8 × 7 × 6 × 5 × 4 × 3
= × = 1260
But total number is 2160 that is more than 2 6 × 5× 4 × 3× 2
2017th place. Hence, word will start with E.
Case III. If 1 man and 7 women are selected
Now, words starts with
number of ways = 10 C1 × 8 C7 = 10 × 8 = 80
EA ........ = 5! = 120
ED ........ = 5! = 120 Case IV. If 0 men and 8 women are selected
EG ........ = 5! = 120 number of ways = 10 C0 × 8 C8 = 1
EI ........ = 5! = 120 total number of required ways
Here, total words till now is 720 + 480 = 1920 = 6720 + 1260 + 80 + 1 = 8061
Now, words start with
51. There are three sections in a question
ELA ........ = 4! = 24
paper, each containing 4 questions. If a
ELD ........ = 4! = 24
candidate has to answer only 5 questions
ELG ........ = 4! = 24 from this paper without leaving any
ELI ........ = 4! = 24 section, then number of ways the candidate
Now, total words is 2016. can make the choice of questions is
Next word is 2017th word and the word is [24 April 2018, Shift-I]
ELNADGI.
(a) 624 (b) 704 (c) 384 (d) 432
50. From a group of 10 men and 8 women, the Sol. (a)
number of ways of forming a committee of 8 Number of ways to select 5 questions from these
members with not more than 5 men and not sections as follows,
less than 5 women is [23 April 2018, Shift-I] A B C

Section 
(a) 8061 (b) 8060 2 2 1
(c) 20997 (d) 20952
2 1 2
Sol. (a) 1 2 2

From group of 10 men and 8 women committee
3 1 1
of 8 members with not more than 5 men and 
not less than 5 women is formed in following 1 3 1
1 1 3
ways. 
Case I. If 3 men and 5 men we selected ⇒ C2 × 4 C2 × 4 C1 × 3 + 4 C3 × 4 C1 × 4 C1 × 3
4

number of ways = 10 C3 × 8 C5 4! 4! 4! 4! 4! 4!
⇒ × × × 3+ × × ×3
10 × 9 × 8 8 × 7 × 6 × 5 × 4 2! 2! 2! 2! 1! 3! 31! ! 1! 3! 1! 3!
= × = 6720
3×2 5× 4 × 3× 2×1 ⇒ 432 + 192 = 624
9
Matrices and
Determinants
1 1  Sol. (b)
 2  is ……… 1 2(x + 3) 3(x 2 + 9 + 3x)
1. The matrix A =  2
−1 −1  f (x) = (x − 3)(x − 5) 1 2(x + 5) 4(x 2 + 25 + 5x)
 
 2 2 1 2 3
[17 Sep. 2020, Shift-I]
⇒ f (3) = f (5) = 0
(a) Unitary (b) Orthogonal
So, f (1) f (3) + f (3) ⋅ f (5) + f (5) f (1) = 0 = f(3)
(c) Nilpotent (d) Involutory
Sol. (c) 3. In P and Q are square matrices such that
 1 1  P 2006 = 0 and PQ = P + Q, then det (Q) will be
 2 2 
A= [17 Sep. 2020, Shift-I]
1 1 
 − −  (a) 0 (b) 1 only (c) −1only (d) ±1
 2 2
Sol. (a)
 1 1  1 1 
 2  2 P 2006 = O and PQ = P + Q
A =
2 2 2 
1 1  1 1  ⇒ P 2006Q = P 2006 + Q ⋅ P 2005
− −  − − 
 2 2  2 2 ⇒ O = O + Q ⋅ P 2005
 1 −1 1 1 
− ⇒ P 2005 ⋅ Q = O ⇒ det. (P 2005 ⋅ Q) = O
 2 2
⇒ A2 =  2 2
1 1 1 1 ⇒ det P 2005(det Q) = O ⇒ det Q = O
− + − + 
 2 2 2 2
4. If P and Q are two non-zero square matrices
 0 0
A2 =   null matrix of the same order such that the product
 0 0 PQ = 0, then ........ [17 Sep. 2020, Shift-II]
So, A is nilpotent matrix (a) Exactly one of them must be singular
x −3 2 x − 18
2
3 x − 81
3 (b) Both P and Q must be singular
2. If f (x) = x − 5 2 x 2 − 50 4 x 3 − 500 , then (c) Both P and Q must be non-singular
(d) None of the options are correct
1 2 3
Sol. (d)
1 f (3) + f (3) f (5) + f (5) f ()
f () 1 is equal to For 2-non zero square matrix; of same order;
[17 Sep. 2020, Shift-I] product may be zero even when matrices are
(a) f(1) (b) f(3) singular or non singular.
(c) f(1) + f(3) (d) f(1) + f(5) So none of the options is correct.
82 AP EAMCET Chapterwise Mathematics

5. Let M and N be two matrices over R of order Sol. (b)


2. Then, MN = NM if .......  3 −4  3 −4  3 −4
M=  ⇒ M2 =   
[17 Sep. 2020, Shift-II] 1 −1 1 −1 1 −1
(a) One of M and N is a diagonal matrix  9 − 4 −12 + 4  5 −8
(b) Both M and N are diagonal matrices =  = 
 3 − 1 −4 + 1   2 −3
(c) Both M and N are invertible matrices
(d) None of these options are true in general  5 −8  3 −4  2 −4
M2 − M =   − =  
Sol. (b)  2 −3 1 −1 1 −2
 a b  p q  2 −4
Let M=  ,N =  So; M n+ 1 − M = M 2 − M =  
 c d r s 1 −2
 ap + br aq + bs 
Then, MN =   8. Which of the following is false?
 pc + dr cq + ds  [18 Sep. 2020, Shift-I]
 ap + qc pb + qd  1. If A is a skew symmetric matrix of order
and NM =  
 ar + cs br + ds  5 × 5, then the rank of A is less than 5.
Clearly, MN = NM 2. If P is a non-zero column matrix and Q is
When, br = 0, qc = 0 a non-zero row matrix, then the rank of
pc = 0, dr = 0 … PQ is 1
or, If b=c=q=r=0 1 2 3 
i.e., both M and N must be diagonal matrices. 3. Rank of 2 3 4  is 2
 
6. Product of all real values of ‘b’ such that 5 6 7 
there is no solution to the system of
equations 2 x + 5 y + z = 19 , 4. If the lines a r x + br y + cr = 0 (r = 1, 2, 3)
− 4 x + by + 6 z = − 42, − 3 y − bz = 81 is ........ are distinct and intersect at a point, then
[17 Sep. 2020, Shift-II]  a1 b1 c1 
(a) − 30 (b) − 48 (c) − 24 (d) − 18 rank of a 2 b2 c2  is 3
 
Sol. (c) a 3 b3 c3 
For no solution of given linear Equations value (a) 1 (b) 2
of given determinant is zero by Cramier's rule, (c) 3 (d) 4
So, Sol. (d)
2 5 1 The skew-symmetric matrix of order 5 × 5, has
D = −4 b 6 =0 rank less than 5 because determinant of odd
0 −3 − b ordered skew symmetric matrix is zero.
If P is a non-zero column matrix and Q is a
⇒ 2(− b + 18) − 5 (4b) + 1 (12) = 0
2
non-zero row matrix, then PQ is a matrix of
⇒ −2b 2 + 36 − 20b + 12 = 0 order 1 × 1, so rank of matrix PQ is one.
⇒ −2b 2 − 20b + 48 = 0 Since, the
⇒ b 2 + 20b − 24 = 0 1 2 3
2 3 4 = 1(21 − 24) − 2(14 − 20) + 3(12 − 15)
Product of roots of above equations is,
c −24 5 6 7
Product = = = − 24
a 1 = −3 + 12 − 9 = 0 and there is no cofactor of any
elements is zero, so rank is 2.
3 − 4 
7. For M =  and for any n ∈N the If the lines a r x + br y + c r = 0, (r = 1, 2,3) are
1  − 1 
distinct and intersect at a point, then matrix
matrix M n + 1 − M n = [17 Sep. 2020, Shift-II]  a1 b1 c1 
2 4  2 − 4 2 − 4 2 4 A =  a 2 b2 c 2  ⇒|A|= 0, so rank of A is not 3.
(a)   (b)  1 − 2  (c)  1 2  (d)  1 2   
 1 − 2         a 3 b3 c 3 
Matrices and Determinants 83

9. Choose the correct option about the matrices  −1 0 0 1 0 0


given below [18 Sep. 2020, Shift-I] ∴ A4 =  0 −1 0 = D 2 ⇒ A8 =  0 1 0
   
π π  0 0 1   0 0 1 
 
 cos 4 sin
4
0
∴ A2020 = (A505)4 = ((A8)63 ⋅ A)4 = (I 63 ⋅ A)4 = A4 ≠ I
 π π 
A = − sin cos 0 Similarly, D 4 = I , so D 2019 ≠ I
 4 4   
 0 0 1 1 0 0 
 π π
  Now, B =  0 cos sin 
 3 3
   0 − sin π cos π 
1 0 0   3 3
 π π  
B = 0 cos sin  1 0 
3 3 0
 π π
 2π 2π 
⇒ B2 =  0 cos sin 
0 − sin cos  3 3 
 3 3 
 0 − sin 2π cos 2π 
 π π  3 3
 cos 6 0 sin 
6  
1 0 
C= 0 1 0  
0
4π 4π 
 π π  ⇒ B4 =  0 cos sin 
− sin cos 0 
 3 3 
6 6
   0 − sin 4 π cos 4 π 
 3 3
 π π 
 
 cos 2 sin 0
1

2
 0 0 
π π  6π 6π 
D = − sin cos 0 ⇒ B6 =  0 cos sin =I
 2 2   3 3 
 0 0 1  0 − sin 6 π cos 6 π 
   3 3
∴ B2020 ≠ I but B2022 = (B6)337 = I 337 = I
(a) A 2020 = I (b) B2020 = I
(c) D 2019
=I (d) B2022 = I  a + ib c + id 
10. If A =   and
Sol. (d) − c + id a − ib
 a + ib − c − id
Given matrix A− 1 =  . Find
 cos π sin π 0 − c + id a − ib 
 

4 4
 (a 2 + b2 + c 2 + d 2). [18 Sep. 2020, Shift-I]
π π
A =  − sin cos 0
4 4 (a) 1 (b) − 1
 
 0 0 1 (c) i (d) − i
 
Sol. (a)
 cos π sin π 0  Given matrix
 2 2   a + ib c + id 
 π π  A= 
⇒ A2 =  − sin cos 0  − c + id a − ib 
 2 2  1  a − ib − c − id 
 0 0 1 So, A−1 =
  |A|  c − id a + ib 

 cos π sin π  1  a − ib − c − id 
0 = 2 2  c − id
 2 2   0 1 0 a +b +c +d 
2 2 a + ib 
 π π   −1 0 0
∴ A2 = D =  − sin cos 0 =  a + ib − c − id 
2 2   =  (given)
   0 0 1   − c + id a − ib 
 0 0 1
  ∴ b = 0 and c = d = 0 and a 2 + b 2 + c 2 + d 2 = 1
84 AP EAMCET Chapterwise Mathematics

11. Find the value of ‘k’, if Consider,


k−2 2k − 3 3k − 4 1 0 1 10 1 0
N ⋅ M10 ⋅ N –1 =     1
k−4 2k − 9 3 k − 16 = 0  0 2  0 1   0 2 
k − 8 2 k − 27 3 k − 64 1 10 1 0 1 5
=   1=  
[18 Sep. 2020, Shift-I] 0 2  0 2  0 1 
(a) 1 (b) 2 (c) 3 (d) 4 Hence, option (a) is correct.
Sol. (d) 13. If A is a Skew-symmetric matrix then
It is given that, (given n ∈N) [18 Sep. 2020, Shift-II]
k − 2 2k − 3 3k − 4 2n
1. A is Skew-symmetric matrix.
k − 4 2 k − 9 3k − 16 = 0 2. A 2 n + 1 is Skew-symmetric matrix.
k − 8 2 k − 27 3k − 64 (a) 1 is true, 2 is false (b) Both 1 and 2 are true
(c) Both 1 and 2 are false (d) 1 is false, 2 is true
On applying, R2 → R2 − R1 and R3 → R3 − R1 ,
we get Sol. (d)
k − 2 2k − 3 3k − 4 Given, A is a skew symmetric matrix
⇒ −2 −6 −12 = 0 ∴ AT = − A ⇒ (A2n)T = (AT)2n = (− A)2n
−6 −24 −60 (A2n)T = A2n
k − 2 2k − 3 3k − 4 ∴A 2n
is Symmetric Matrix
⇒ 1 3 6 =0 (A2n + 1)T = (AT)2n + 1 = (− A)2n + 1
1 4 10 (A2n + 1)T = − A2n + 1
⇒ (k − 2)[30 − 24] − (2k − 3)[10 − 6] A2n + 1 is skew symmetric
Hence, option (d) is correct.
+ (3k − 4) [4 − 3] = 0
⇒ 6k − 12 − 8k + 12 + 3k − 4 = 0 ⇒ k = 4 2 1 1
1 1 1 0 

14. The rank of 0 3 − 1 is .......
12. Let M =   
 and N = 0 2. Then 1 − 1 1  [18 Sep. 2020, Shift-II]
0 1  
NM10 N − 1 = [18 Sep. 2020, Shift-II] (a) 1 (b) 2 (c) 3 (d) 4
 1 5  1 − 5 Sol. (b)
(a)   (b)   2 1 1 
 0 1 0 1 
 1 − 10  1 10 A =  0 3 −1
(c)  (d)   
0 1  0 1 
 1 −1 1 
2 1 1 
Sol. (a)
R3 → 2R3 − R1 =  0 3 −1
1 1 1 1 1 1  
M=  ⇒ M =  0 1  0 1
2
 0 −3 1 
 0 1   
2 1 1 
1 2 1 3 R3 → R3 + R2 =  0 3 −1 
M =2
 ⇒ M3 =    
0 1 0 1  0 0 0 
M ∴Rank of A = 2
1 10 1 0
M10
=  ⇒ N= 
Hence, option (b) is correct.
0 1   0 2
15. If Q is the inverse of A, when
N = 2− 0 = 2
1 − 1 1  4 2 2
 2 0
A = 2 1 − 3 and 10 × Q = − 5 0 x ,
−1 1
Adj N =   ⇒ N = N Adj A
0 1     
1 1 1   1 − 2 3 
1  2 0 1 0
N –1 = ⇒ N –1 =  1  find x = [18 Sep. 2020, Shift-II]
2  0 1  0 2  (a) 2 (b) 3 (c) 4 (d) 5
Matrices and Determinants 85

Sol. (d) 4 (sinθ − cosecθ)2 2020


−1
Given, Q= A ∆ = 4 (cosθ − secθ)2 2020
1 −1 1  4 (tanθ − cot θ)2 2020
A =  2 1 −3
  Q Elements of columns C1 and C2 are
1 1 1  perportional, so ∆ = 0
A = 1(1 + 3) + 1(2 + 3) + 1(2 − 1) = 4 + 5 + 1 Hence, option (c) is correct.
A = 10 17. Let M and N be two invertible square
T
 4 −5 1  matrices over R of order 2 such that N is
Adj A =  2 0 −2  diagonal. Then MNM − 1 is diagonal .........
 
 2 5 3  [21 Sep. 2020, Shift-I]
(a) For all M
 4 2 2
(b) Only when M is a scalar matrix
Adj A =  −5 0 5
  (c) For all diagonal matrices M
 1 −2 3 (d) M must be a null matrix
1
A −1 = Adj A Sol. (c)
A Let the real matrices of order 2
 4 2 2 a b  n1 0
M=  and N =  0 n2 
1 
A −1 = −5 0 5  c d 
10  
 1 −2 3 Q M and N are invertible matrices, so
 4 2 2 1  d −b
M −1 =
10 Q =  −5 0 x  ad − bc  − c a 
 
 1 −2 3 Therefore,
1  a b   n1 0   d − b 
 4 2 2   4 2 2  4 2 2  MNM −1 =
ad − bc  c d   0 n2   − c a 
10 A−1 =  −5 0 x  ⇒  −5 0 5 =  −5 0 x 
      1  a b   n1 d − n1 b 
 1 −2 3  1 −2 3  1 −2 3  =
ad − bc  c d   − n2c n2a 
∴ x=5
1  an1 d – bn2c – an1 b + bn2a 
Hence, option (d) is correct. =
ad − bc  cn1 d – dn2c – cn1 d + dn2a 
 π
16. If θ ∈ 0 ,  , then It is given that MNM −1 is a diagonal matrix,
 2 then
(sin θ + cosec θ)2 (sin θ − cosec θ) 2 2020 ab(n2 − n1) = 0 = cd(n1 − n2)
(cos θ + sec θ) 2 (cos θ − sec θ) 2 2020 = and if M is a diagonal matrix, means b = c = 0,
then the above requirement getting satisfy.
(tan θ + cot θ) 2 (tan θ − cot θ) 2 2020
Hence, option (c) is correct.
[21 Sep. 2020, Shift-I]
18. Let a,b ∈R − {0}, and I 2 be the identity
(a) 1 (b) − 1
(c) 0 (d) 2020 matrix of order 2. Then the rank of the
aI bI 2 
Sol. (c) (block) matrix  2  is
π aI 2 bI 2 
Given determinate , where θ ∈  0,  is
 2 [21 Sep. 2020, Shift-I]
(sinθ + cosecθ)2 (sinθ − cosecθ)2 2020 (a) 2 (b) 1
∆ = (cosθ + secθ)2 (cosθ − secθ)2 2020 (c) 4 (d) 3
(tanθ + cot θ)2 (tanθ − cot θ)2 2020 Sol. (a)
Since a , b ∈ R − {0} and I 2 is a identify matrix of
On applying C1 → C1 − C2 , we get order 2.
86 AP EAMCET Chapterwise Mathematics

 aI bI 2  21. Determinant of skew-symmetric matrix of


Then the block matrix  2  is a
 aI 2 bI 2  order ‘‘three’’ is always [21 Sep. 2020, Shift-II]
non-singular matrix, so the rank of the given (a) 0 (b) 1
block matrix is 2. (c) Depends on elements (d) –1
log 5 729 log 3 5 Sol. (a)
19. The value of
log 5 27 log 9 25 Determinant of odd order skew-symmetric
log 3 5 log 27 5 matrix is always be zero.
× is Hence, option (a) is correct.
log 5 9 log 5 9 [21 Sep. 2020, Shift-I]
(a) 1 (b) 6
22. For any a , b, c ∈ R, the determinant
(c) log 5 9 (d) (log 3 5) × (log 5 81) bc b+ c 1
Sol. (d) ca c + a 1 is equal to
log 5 729 log 3 5 log 3 5 log 27 5 ab a + b 1
× [21 Sep. 2020, Shift-II]
log 5 27 log 9 25 log 5 9 log 5 9 (a) a(b 2 − c 2 ) + b (c 2 − a2 ) + c (a2 − b 2 )
6 log 5 3 log 3 5 1 (b) a(b − c ) + b(c − a) + c(a − b )
log 3 5 log 3 5
= × 3 (c) (a − b )(b − c )(c − a)
3log 5 3 log 3 5 2log 5 3 2log 5 3 (d) abc
2 1 1 1/ 3
= 3log 5 3 log 3 5 × 2log 3 5 log 5 3 Sol. (c)
1 1 1 1 For any a , b , c ∈R, the given determinant
bc b + c 1
= 6[2 − 1] × 1 −  = 4
1
 3 ∆ = ca c + a 1
Q Hence the options ab a + b 1
log 3 5 × log 5 81 = (log 3 5) × (4 log 5 3) = 4
On applying R2 → R2 − R1 and R3 → R3 − R1 ,
Hence, option (d) is correct. we have
1 −1 x  bc b+ c 1
20. If 1 x 1  has no inverse, then the real ∆ = c(a − b) a − b 0
 
b(a − c) a − c
 x −1 1  0

value of x is [21 Sep. 2020, Shift-II] bc b + c 1


(a) 2 (b) 3 (c) 0 (d) 1 = (a − b)(a − c) c 1 0
b 1 0
Sol. (d)
1 − 1 x On applying R3 → R3 − R2, we have
It is given that matrix A = 1 x 1  have no bc b+ c 1
 
 x −1 1  ∆ = (a − b)(a − c) c 1 0
inverse, so b−c 0 0
1 −1 x
|A|= 0 ⇒ 1 x 1 =0 = (a − b)(a − c)[0 − (b − c)] = (a − b)(b − c)(c − a)
x −1 1 Hence option (c) is correct.

⇒ 1(x + 1) + 11
( − x) + x(−1 − x 2) = 0 23. The number of the values of ‘k’ for which the
⇒ x +1+1− x − x − x = 0 3 lines 2 x + y = 1,3 x + 2 y = 2 , kx + 3 y = 3 are
concurrent is ……… [22 Sep. 2020, Shift-I]
⇒ x 3 + x − 2 = 0 ⇒(x − 1)(x 2 + x + 2) = 0
(a) 0 (b) Infinite
Either x = 1 or x 2 + x + 2 = 0 (c) 1 (d) 2
But discriminant of quadratic equation
x 2 + x + 2 = 0 is negative so no real roots.
Sol. (b)
It is given that the lines 2x + y = 1, 3x + 2y = 2
∴|A|= 0 ⇒ x = 1
and kx + 3y = 3 are concurrent, so
Hence, option (d) is correct
Matrices and Determinants 87

2 1 −1 Sol. (d)
3 2 −2 =0 cos x x 1
k 3 −3 Given, f (x) = 2 sin x x2
2x
Q The elements of columns C 2 and C 3 are tan x x 1
proportional to each other, so for any value of k − sin x 1 0 cos x x 1
given lines are concurrent.
∴ f ′(x) = 2 sin x x2
2x + 2 cos x 2x 2
1 2 3 1 1 0  tan x x 1 tan x x 1
24. If A = 1 
1 1 , B = 0 1 3  , cos x x 1
   
1 −1 1 3 0 4  + 2 sin x x 2 2x
2 0 1 sec2 x 1 0
C = 0 1 0 , then ((((ABC)−1)T)−1)T = 0 1 0 1 0 1
 
So, at x = 0, f ′(x) = 0 0 0 + 2 0 2
3 2 1
0 0 1 0 0 1
[22 Sep. 2020, Shift-I]
 64 39 28  63 39 20 1 0 1
(a) 29 16 11 (b) 29 16 11 + 0 0 0 =0
   
 11 2 5  10 2 5  1 1 0
 64 39 27   61 39 28
cos(x + a + b) sin(x + a + b) 10
(c) 28 15 11 (d) 29 16 11
    26. If f (x) = cos(x + b + c) sin(x + b + c) 10 ,
 11 2 5  11 0 5 
cos(x + c + a) sin(x + c + a) 10
Sol. (a) then ( f (2019) f (2020) − f (2020) f (2019) =
1 2 3 1 1 0
[22 Sep. 2020, Shift-II]
Given matrices A = 1 1 1 , B =  0 1 3
    (a) 1 (b) −1
1 − 1 1   3 0 4 (c) 0 (d) 2
2 0 1
Sol. (c)
and C =  0 1 0
  cos (x + a + b) sin (x + a + b) 10
 3 2 1 
f (x) = cos (x + b + c) sin (x + b + c) 10
Q((((ABC)− 1)T)− 1)T = (((C − 1 B− 1 A− 1)T)− 1)T cos (x + a + c) sin (x + a + c) 10
= (((AT)− 1 (BT)− 1 (C T)− 1)− 1)T = (C T BT AT)T = ABC R2 → R2 − R1
1 2 3  1 1 0 2 0 1 R3 → R3 − R1
= 1 1 1  0 1 3  0 1 0 cos (x + a + b)
   
1 − 1 1  3 0 4  3 2 1 f (x) = cos (x + b + c) − cos (x + a + b)
10 3 18 2 0 1  64 39 28 cos (x + a + c) − cos (x + a + b)
=  4 2 7   0 1 0 = 29 16 11 sin (x + a + b) 10
    
 4 0 1   3 2 1  11 2 5  sin (x + b + c) − sin (x + a + b) 0
sin (x + a + c) − sin (x + a + b) 0
cos x x 1
25. If f (x) = 2sin x 2
x 2 x , then the value of
cos (x + a + b)
tan x x 1  a + c c − a
f (x) = −2sin  x + b +  sin  
f ′(x) at x = 0 is equal to [22 Sep. 2020, Shift-I]  2   2 
(a) −1 (b) 1  b + c c − b
−2sin  x + a +  sin  
(c) 2 (d) 0  2   2 
88 AP EAMCET Chapterwise Mathematics

2x − y + 3z = 0
sin (x + a + b) 10 x + y − z =1
 a + c c − a
2cos  x + b +  sin   0 0x + 0y − z = 0
 2   2  2 −1 3
b + c + b
2cos  x + a +  sin c
  0 ∆= 1 1 −1
 2   2 
0 0 −1
cos (x + a + b = 2 (−1 + 0) + 1(−1 + 0) + 3(0) = − 2 − 1
 c − b  c −a  a + c ∆ = − 3≠ 0
= 4sin  sin   − sin  x + b + 
 2   2   2  ∴System of equations have unique solution.
+
− sin  x + a + 
b c Hence, option (b) is correct.

 2 
28. Let A be a square matrix of order 3. Choose
the correct option regarding the following
sin (x + a + b) 10
statements [22 Sep. 2020, Shift-II]
 a + c
cos  x + b +  0 I. There exists a matrix B of order 3 such
 2 
b + c that AB = I 3
cos  x + a +  0
 2  II. There exists a matrix C of order 3 such
c − b  c − a  × 10
that CA = I 3
= 4sin   sin   III. A is invertible
 2   2 
 b + c a+ (a) Only III implies I and II
sin  x + a +  c
  cos  x + b +  − (b) I, II and III are equivalent statements
 2   2 
(c) In I and II, B can be different from C
a + c b + c
sin  x + b + 
 cos  x + a +  (d) None of the above
 2   2  
Sol. (b)
c − b  c − a  sin  a − b 
= 40 sin   sin     Since, given statement (I) AB = I 3
 2   2   2 
⇒ B = I 3 A−1 ⇒ B = A−1 … (i)
= constant
−1
∴ f (x) = constant Statement (II) CA = I 3 ⇒ C = I 3 × A
f (2019) f ( 2020) − f (2020) f ( 2019) = 0 C = A−1 … (ii)
Hence, option (c) is correct. Statement(II) From Eqs. (i) and (ii),
A is invertable matrix
2 −1 3 0 
27. Let A =  1 1 −1 and D = 1. The system
Given Statements (I), (II), (III) are equivalent
    Hence, option (b) is correct.
0 0 1 0 
29. If the mean and standard deviation of a
AX = D has ……… [22 Sep. 2020, Shift-II]
binomial distribution are 20 and 4
(a) No solution respectively, then the number of trials is
(b) A unique solution [23 Sep. 2020, Shift-I]
(c) More than one but finite solutions
(a) 25 (b) 50 (c) 200 (d) 100
(d) Infinitely many solutions
Sol. (b) Sol. (d)
Given, Mean (np) = 20
 2 −1 3   0
SD ( npq) = 4
A = 1 1 −1 , D = 1 
   
 0 0 1   0 Squaring on both sides,
npq = 16
Given, AX = D
20(q) = 16
 2 −1 3   x   0
1 1 −1  y  = 1  4 4 1
q = ⇒ p =1− q =1 − ⇒ p =
     5 5 5
 0 0 −1  z   0 We have, np = 20
Matrices and Determinants 89

n   = 20 ⇒ n = 20 × 5 1 0 
1
 5 31. Evaluate A 2 + 2I , ifA =  
1 2
n = 100
[23 Sep. 2020, Shift-I]
Hence, option (d) is correct.
(a) 2 A (b) 3A (c) 4A (d) 5A
30. If a = 1 + 2 + 4 + … upto n terms, Sol. (b)
b = 1 + 3 + 9 + … upto n terms and 1 0
Given, A = 
c = 1 + 5 + 25 +… upto n terms then 1 2
a 2b 4 c 0 1 0 1 + 0 0 + 0 1 0
1
∆= 2 2 2 = A2 =  = =
1 2 1 2 1 + 2 0 + 4  3 4
2n 3n 5n [23 Sep. 2020, Shift-I] Consider,
(a) (30) n
(b) (10) n 1 0 1 0
A 2 + 2I =  + 2
(c) 0 (d) 2 n + 3n + 5n 3 4 
0 1 
Sol. (c) 1 0  2 0  3 0
= + =
Given, 3 4  0 2  3 6
a = 1 + 2 + 4 + .... upto n terms A2 + 2I = 3A
a (r n − 1) Q a = 1
a=  r=2
Hence, option (b) is correct.
r −1  
1 4 −1
1(2n − 1)
a= ⇒ a = 2n − 1 32. Find the rank of the matrix 2 3 0
2 −1  
b = 1 + 3 + 9 + .... upto n terms 0 1 2 
(3n − 1) 3n − 1 [23 Sep. 2020, Shift-I]
b =1 ⇒ b=
3 −1 2 (a) 1 (b) 2 (c) 3 (d) 4
c = 1 + 5 + 25 + .... upto n terms. Sol. (c)
1(5n − 1) 5n − 1
c= ⇒ c= Given matrix is,
5 −1 4 1 4 −1
A = 2 3 0 
Consider
a 2b 4 c  
 0 1 2 
∆= 2 2 2
1 4 −1
2n 3n 5n
Q|A| = 2 3 0
(3n − 1) (5n − 1)
(2n − 1) 2 4 0 1 2
2 4
∆= 2 2 2 = 1(6 − 0) − 4(4 − 0) − 1(2 − 0)
2n 3n 5n = 6 − 16 − 2 = −12 ≠ 0
∴Rank of non-singular square matrix A having
order 3 × 3 is 3.
(2n − 1) (3n − 1) 5n − 1
Hence, option(c) is correct.
= 2 2 2
2n 3n 5n 1 0 1 1
33. If A =  , P = 0 1 and X = APA , then
T

0 − 1  
2n 3n 5n 1 1 1
A T X 50 A = [20 April 2019, Shift-I]
= 2 2 2 − 2 2 2 = 0− 0
2 n
3n n
5 n
2 3n
5n  0 1 2 1 
(a)   (b)  
 1 0  0 −1
[Q In a determinant if two rows are equal 25 1   1 50
then its value is zero.] (c)   (d)  
=0  1 −25 0 1 
90 AP EAMCET Chapterwise Mathematics

Sol. (d) 2 0 5 
1 0  ∆ 2 =  1 4 −2
Given matrix A =   is orthogonal matrix,  
 0 −1 1 1 1 
because AAT = I. = 2(4 + 2) − 0(1 + 2) + 51
( − 4) = − 3
So, AT X 50 A = AT X 49(APAT) A = AT X 49 AP(AT A)  2 3 0
and ∆ 3 = 1 1 4 = 21( − 4) − 31
( − 4) + 0(1 − 1) = 3
= AT X 49 AP  
 1 1 1 
= AT X 48 (APAT) AP = AT X 48 AP 2……
−3 −3 −3
……= AT AP 50 = I P 50 = P 50 Now, x = = 1,|y|= = 1 and [z] = = −1
−3 −3 3
1 1 1 2 ∴ x = 1,|y| = 1 ⇒ y = ± 1 and [z] = −1
Q P= ⇒ P2 = 
0 1 
0 1 ⇒ z ∈ [−1, 0)
1 3 So, the given system of three equations has
⇒ P3 =  ……
0 1  infinitely many solution.
Hence, option (c) is correct.
1 50
⇒ P 50 =  
0 1  35. Investigate the values of λ and µ for the
1 50 system x + 2 y + 3 z = 6, x + 3 y + 5 z = 9,
So, AT X 50 A = P 50 =   2 x + 5 y + λz = µ and match the values in
0 1 
List - I with the items in List - II.
Hence, option (d) is correct.
[20 April 2019, Shift-I]
34. If [ x ] is the greatest integer less than or
List I List II
equal to x and| x| is the modulus of x, then
the system of three equations (A) λ = 8, µ ≠ 15 1. Infinitely many solutions
2 x + 3| y| + 5[ z] = 0 , x + | y| − 2[ z] = 4,
(B) λ ≠ 8, µ ∈ R 2. No solution
x + | y| + [ z] = 1 has [20 April 2019, Shift-I]
(a) a unique solution (C) λ = 8, µ = 15 3. Unique solution
(b) finitely many solutions
(c) infinitely many solutions A B C A B C
(d) no solution (a) 1 2 3 (b) 2 3 1
(c) 3 1 2 (d) 3 2 1
Sol. (c)
Given system of three equations Sol. (b)
2x + 3|y| + 5[z] = 0 Given system of linear equations is
x + |y| − 2[z] = 4 x + 2y + 3z = 6
and x + |y| + [z] = 1 x + 3y + 5z = 9
According to Cramer’s rule, and 2x + 5y + λz = µ
∆ ∆ ∆ Now, according to Cramer’s rule,
x = 1 , |y | = 2 and [z] = 3
∆ ∆ ∆ 1 2 3 
2 3 5  ∆ =  1 3 5  = 1(3λ − 25) − 2(λ − 10) + 3(5 − 6)
 
where, ∆ = 1 1 −2 2 5 λ
 
1 1 1  =λ−8
= 2 (1 + 2) − 3 (1 + 2) + 5 (1 − 1) = − 3  6 2 3
0 3 5  ∆1 =  9 3 5  = 6(3λ − 25) − 2(9λ − 5µ)
 
∆1 = 4 1 −2
  µ 5 λ  + 3(45 − 3µ) = µ − 15
1 1 1  1 6 3 
= 01( + 2) − 3(4 + 2) + 5(4 − 1) ∆ 2 =  1 9 5  = 1(9λ − 5 µ) − 6(λ − 10) + 3(µ − 18)
= − 18 + 15 = − 3  
2 µ λ
Matrices and Determinants 91

= 3λ − 2 µ + 6 cos α − sin α 0
1 2 6  37. If A = sin α cos α 0, then (Adj A)−1 =
and ∆ 3 =  1 3 9   
   0 0 1
2 5 µ
[20 April 2019, Shift-II]
= 1(3µ − 45) − 2(µ − 18) + 6(5 − 6)
(a) A + I (b) A − I
= µ − 15
(c) A (d) Adj ( A −1 )
Now, if λ = 8 and µ ≠ 15, then system of linear
equations has no solution. Sol. (c)
If λ ≠ 8 and µ ∈ R, then system of linear We have,
equations has unique solution.  cosα – sinα 0
And, if λ = 8 and µ = 15, then system of linear A =  sinα cosα 0
equations has infinite number of solutions,  
 0 0 1 
because ∆ 2 = 3λ − 2µ + 6 is also be zero.
Hence, option (b) is correct. ∴|A|= cos2 α + sin2 α = 1

36. The sum of the values of x so that the matrix Now,|adj A|=|A|n –1 =|A|3–1 =|A|2 = ()
1 2 =1
2 2 1  1 0 0 Also, adj (adj A) =|A|n – 2 A =|A|3– 2⋅ A =|A|A = A
   
 1 3 1 − x 0 1 0  is singular, is Now, (adj A)–1 =
adj(adj A)  adj A 
Q A = |A| 
–1
1 2 2 0 0 1
    |adj A|  
[20 April 2019, Shift-II] = A
(a) 3 (b) 5 (c) 7 (d) 9 Hence, option (c) correct.
Sol. (c) 38. If abc ≠ 0 and the system of equations
Let a matrix x + 7 ay + 2 az = 0, x + 6 by + 2 bz = 0,
2 2 1 1 0 0  2 − x 2 1  x + 5 cy + 2 cz = 0 has a non-trivial solution,
= 1 3 1  − x  0 1 0 =  1 3− x 1  then a , b, c are in [20 April 2019, Shift-II]
     
1 2 2  0 0 1   1 2 2− x  (a) harmonic progression
(b) geometric progression
Q Matrix A is a singular.
(c) arithmetic progression
2 − x 2 1 
(d) arithmetic-geometric progression
∴ |A| = 0 ⇒ 1 3− x 1 = 0
  Sol. (a)
 1 2 2 − x
Given, system of linear equation, abc ≠ 0, is
On applying C1 → C1 + C2 + C3, we get x + 7ay + 2az = 0, x + 6by + 2bz = 0
5 − x 2 1 
and x + 5cy + 2cz = 0 has a non-zero trivial
5 − x 3 − x 1 = 0 solution.
 
5 − x 2 2 − x 1 7a 2a
1 2 1  So, A = 0 ⇒1 6b 2b = 0
 
⇒ (5 − x)1 3 − x 1 = 0 1 5c 2c
 
1 2 2 − x ⇒112
( bc − 10bc) − 7a(2c − 2b) + 2a(5c − 6b) = 0
On applying R2 → R2 − 1 and R3 → R3 − R1 , ⇒ 2bc − 4ac + 2ab = 0
we get ⇒ 2ac = bc + ab
1 2 1 
⇒ b=
2ac
(5 − x)0 1 − x 0 = 0 a+ c
 
0 0 1 − x or
2 1 1
= +
⇒ (5 − x) (1 − x)2 = 0 ⇒ x = 1, 1, 5 b a c
Then, a , b , c are in Harmonic Progression.
So, sum of the required values of x is 7.
Hence, option (a) is correct.
Hence, option (c) is correct.
92 AP EAMCET Chapterwise Mathematics

a + b + 2c a b  90 − 94 8 
1 
39. If c 2a + b + c b = 2, (d) − − 138 46 0 
92  
 2 2 − 8
c a a + 2b + c
then a + b + c − 3 abc = [21 April 2019, Shift-I]
3 3 3 Sol. (b)
x 2 − 25x + 24 = 0 …(i)
(a) 1 − 3ab − 3bc − 3ca (b) 0
(c) 1 − 2 ab − 2 bc − 2ca (d) 1 x − x − 24 x + 24 = 0
2

Sol. (a) x(x − 1) − 24(x − 1) = 0


a + b + 2c a b (x − 1) (x − 24) = 0
⇒ x = 1, 24
c 2a + b + c b =2
Q k is one of the root of the Eq. (i),
c a a + 2b + c
∴ k = 1, 24
1 a b If k = 1,
⇒ 2(a + b + c) 1 b + c + 2a b =2 1 2 1 
1 a c + a + 2b ∴ A =  3 2 3
 
[Applying C1 → C1 + C2 + C3 and 1 1 1 
taking 2(a + b + c) common from C1 ]
| A | = 1 (2 − 3) − 2(3 − 3) + 1(3 − 2)
1 a b
= −1 − 0 + 1 = 0
⇒ 2(a + b + c) 0 b + c + a 0 =2 | A|= 0
0 0 c+ a+ b k = 1, not possible, because given matrix A is
[Applying R2 → R2 − R1 and R3 → R3 − R1 ] singular.
⇒ 2(a + b + c)3 = 2 [expanding along C1 ] Now, k = 24,
1 2 1 
⇒ (a + b + c)3 = 1 ⇒ a + b + c =1
∴ A = 3 2 3 
Now, a 3 + b 3 + c 3 − 3abc  
1 1 24
= (a + b + c) (a 2 + b 2 + c 2 − ab − bc − ca)
| A | = 1(48 − 3) − 2(72 − 3) + 1(3 − 2)
= 1 ⋅ [a 2 + b 2 + c 2 − ab − bc − ca]
= 45 − 138 + 1 = − 92 ≠ 0
= (a + b + c)2 − 2ab − 2bc − 2ca − ab − bc − ca 1
 45 − 69 1 
= 1 − 2ab − 2bc − 2ca − ab − bc − ca
adj A =  − 47 23 1 
= 1 − 3ab − 3bc − 3ca  
 4 0 − 4
40. If k is one of the roots of the equation
 45 − 47 4 
1 2 1
=  − 69 23 0 
x 2 − 25 x + 24 = 0 such that A = 3 2 3 is a  
   1 1 − 4
1 1 k  45 − 47 4 
non-singular matrix, then A − 1 = ∴ A− 1 =
1
⋅ adj A = −
1 
− 69 23 0
| A| 92  
[21 April 2019, Shift-I]  1 1 −4
 90 − 94 8 
1 
(a) − − 138 46 0  41. A value of b for which the rank of the matrix
46  
1 1 − 1 0
 2 2 − 8
4 4 − 3 1
 45 − 47 4 
1  A=  is 3, is
(b) − − 69 23 0 b 2 2 2
92  
9 9
 1 1 −4  b 3 [21 April 2019, Shift-I]
 45 − 47 4 
1  (a) − 2 (b) − 4
(c) − − 69 23 0  (c) − 6 (d) 3
46  
 1 1 − 4 Sol. (c)
Matrices and Determinants 93

Given, 43. Match the items of List-I with the items of


1 1 − 1 0 List-II and choose the correct option.
4 4 − 3 1 
[21 April 2019, Shift-II]
A= 
 b 2 2 2
 9 9 b 3 List I List II
 
A. If A is a non singular matrix of order 3 null
For rank to be 3, there must exist 3 non zero and| A| = a, then|(adj A −1 )−1| = I.
matrix
row.
Now, applying R2 → R2 − 4R1 ; R3 → R3 − 2R1 B. A is a non singular matrix of order 3
and B is any matrix of order 3 such II. a2
 1 1 − 1 0
that AB = O, then B is
 0 0 1 1
=  C. 1 x x2
 b − 2 0 4 2
 9 cos(a − b )y cos ay cos(a + b )y
 9 b 3 III. b
sin(a − b )y sin ay sin(a + b )y
Applying R4 → R4 − 9R1 does not depend on
 1 1 −1 0
 0 D. A is a square matrix of order 3 and
0 1 1 B = A − AT , then|B| is IV. a
= 
b − 2 0 4 2
 0 0 b+ 9 3
V. 0

Again, applying R4 → R4 − 3R2 The correct answer is
 1 1 − 1 0 A B C D A B C D
 0 0 1 1
(a) II IV III I (b) III I IV V
A=  (c) II V III I (d) II I IV V
b − 2 0 4 2
 0 0 b + 6 0
Sol. (d)

(A) Given for a non-singular matrix of order 3,
If rank = 3, then |A| = a.
Last row must have all elements 0. 1  1 
∴ b+ 6=0 Q |(adj A−1)−1| = −1
Q|A | = 
|adj A−1|  | A|
⇒ b=−6
1
=
p q r |A−1|2
42. If A =  r p q and AA T = I then, [Q|adj A| = |A|2, if order of A is 3.]
 
 q r p
=
1
= |A|2 = a 2
2
p +q +
3 3
r3 = [21 April 2019, Shift-II]  1 
 
(a) ±1 (b) pqr (c) 3pqr (d) 3 pqr ± 1  |A|
Sol. (d) (B) It is given that for a non-singular matrix A
Given, of order B, AB = 0
AAT = I ⇒ |AB| = 0

It represents orthogonal matrix. ⇒ |A||B| = 0


Determinant of orthogonal matrix is ±1. ⇒ |B| = 0 (Q|A|≠ 0)
p q r and matrix B should be null matrix.
∴ |A| = r p q = ± 1  1 x x2 
  (C) ∆ = cos(a − b) y cos ay cos(a + b) y

q r p sin(a − b) y sin ay sin(a + b) y
 
⇒ p(p 2 − qr) − q(pr − q 2) + r(r 2 − pq) = ± 1  
= 1[cos ay sin(a + b) y − cos(a + b) y sin ay]
⇒ p 3 − pqr − pqr + q 3 + r 3 − pqr = ± 1
− x[cos(a − b) y sin(a + b) y − cos(a + b) y
⇒ p 3 + q 3 + r 3 − 3pqr = ± 1 sin(a − b) y] + x 2[cos(a − b) y sin ay
⇒ p 3 + q 3 + r 3 = 3pqr ± 1 − sin(a − b) y cos ay]
94 AP EAMCET Chapterwise Mathematics

= sin(by) − x sin(2by) + x 2 sin(by) Sol. (d)


∴ Determinant ∆ does not depend on a. 1 + sin2 x cos2 x 4sin 2x
(D) Q B = A − AT Given, sin x 1 + cos2 x
2
4sin 2x
⇒ BT = (A − AT)T = AT − A = − (A − AT) sin2 x cos2 x 1 + 4sin 2x
⇒ BT = − B Applying R1 → R1 − R3 and R2 → R2 − R3, we get
∴Matrix B is a skew-symmetric matrix. 1 0 −1
∴ |B| = 0. = 0 1 −1
sin2 x cos2 x 1 + 4sin 2x
44. The solution of the linear system of
1 1 −1
equations
Applying C2 → C2 + C1 = 0 1 −1
2 2 3  x  3 y + 11  x   z  sin2 x 1 1 + 4sin 2x
7 1 1  y =  6 z − 1  +  x  + 3 x  is
         Applying R2 → R2  R1 and R3 → R3  R1
0 6 5  z  5 y + 11 4 z 4 y 1 1 −1
[21 April 2019, Shift-II] = −1 0 0 = 2 + 4sin 2x
(a) x = 4, y = − 3, z = 2 (b) x = 2, y = 1, z = 1 −1 + sin2 x 0 2 + 4sin 2x
(c) x = 1, y = − 1, z = 2 (d) x = 2, y = − 4, z = 3 Since, maximum value of sin 2x is 1.
Sol. (a) = 2+ 4 = 6
Given that, 1 0 −2
2 2 3  x   3y + 11  x   z 
7 1 46. If A = −2 −1 2 , then A −1 =
1   y  =  6z − 1  +  x  +  3x   
          3 4 1 
 0 6 5  z   5y + 11  4z   4 y [22 April 2019, Shift-I]

 2x + 2y + 3z   3y + x + z + 11  (a) A − 2 A − 4I
2
(b) A 2 − A − 3I
⇒  7 x + y + z  =  3x + x + 6z − 1 
1
(c) [ A 2 + A + 2 I] (d) A 2 + A − 2 I
    2
 6 y + 5z   5y + 4 y + 4z + 11
Sol. (b)
 2x + 2y + 3z   x + 3y + z + 11
Characteristic equation of any square matrix A
⇒  7 x + y + z  =  4 x + 6z − 1 
    is given by
 6 y + 5z   9 y + 4z + 11  |A − λI| = 0
On comparing, we get  1 0 −2 1 0 0
2x + 2y + 3z = x + 3y + z + 11 ⇒  −2 −1 2 − λ  0 1 0 = 0

   
⇒ x − y + 2z = 11 …(i)  3 4 1   0 0 1 
7 x + y + z = 4 x + 6z − 1
1 0 −2  λ 0 0 
⇒ 3x + y − 5z = − 1 …(ii)  −2 −1 2  −  0 λ 0  = 0

and 6 y + 5z = 9 y + 4z + 11    
 3 4 1   0 0 λ 
⇒ −3y + z = 11 …(iii)
By solving Eqs. (i), (ii) and (iii), we get 1− λ 0 −2
∴ x = 4, y = − 3 and z = 2 ⇒ −2 −1 − λ 2 =0
3 4 1− λ
45. The maximum value of the determinant of
1 + sin 2 x cos 2 x 4 sin 2 x  ⇒ (1 − λ) [−(1 + λ)(1 − λ) − 8] − 2[−8 + 3(λ + 1)] = 0
  ⇒ (1 − λ)[−(1 − λ 2) − 8] − 2(−8 + 3λ + 3) = 0
the matrix  sin x2
1 + cos x
2
4 sin 2 x 
⇒ (1 − λ)(λ 2 − 9) − 2(−5 + 3λ) = 0
 sin 2 x cos 2 x 1 + 4 sin2 x 
  ⇒ λ 2 − 9 − λ 3 + 9λ − 6λ + 10 = 0
is [22 April 2019, Shift-I]
⇒ − λ 3 + λ 2 + 3λ + 1 = 0
(a) 0 (b) 2
⇒ λ3 − λ2 − 3λ − 1 = 0
(c) 4 (d) 6
Matrices and Determinants 95

According to Caley Hamilton, every square 45 81


(a) (b) 36 (c) (d) 9
matrix satisfies its characteristics equations. 2 2
∴ A3 − A2 − 3A − I = 0
−1
Sol. (a)
⇒ A (A3 − A2 − 3A − I) = 0
 7 5  4 3 − 5 3 
⇒ A2 − A − 3I − A−1 = 0 Given, A =  , B =  7 5 and C =  7 − 4
 4 8    
⇒ A−1 = A2 − A − 3I
and find the value of,
∞ 1
47. If the system of simultaneous linear ∑ k Tr {A(BC)k }
equations x + y + z = a, x − y + bz = 2, k =0 3

2 x + 3 y − z =1 has infinitely many solutions, 1 1 1


= 0 Tr {A(BC)0 } + 1 Tr {A(BC)1 } + 2 Tr {A(BC)2}
then b − 5 a = [22 April 2019, Shift-I] 3 3 3
4 1
(a) (b) 3 (c) 7 (d) −3 + 3 Tr {A(BC)3} + ... ∞
5 3
Sol. (b) 1 1
= Tr {A} + Tr {A(BC)} + Tr {A(BC)2} + K ∞ … (i)
Given that system of linear equations 3 9
x+y+ z=a …(i)  4 3  − 5 3 
Now, BC =   
x − y + bz = 2 …(ii)  7 5  7 − 4
2x + 3y − z = 1 …(iii)  − 20 + 21 12 − 12  1 0
has infinitely many solutions, = = =I
 − 35 + 35 21 − 20  0 1 
1 1 1 
1 −1 b  = 0 ∴From Eq. (i), we get

 
Tr {A} + Tr   + Tr   + Tr   + ... ∞ …(ii)
A A A
 2 3 −1  3  9  27 
⇒ 11 ( − 3b) − 1(−1 − 2b) + 1(3 + 2) = 0 and Tr (A) = sum of the diagonals entries of A
⇒ 1 − 3b + 1 + 2b + 5 = 0 = 7 + 8 = 15
⇒ −b = − 7 Hence, from Eq. (ii), we get
⇒ b=7 15 15 15
Adding Eqs. (i) and (ii), we get 15 + + + + K∞
3 9 27
2x + 8z = a + 2
= 151 + + + + K ∞ 
1 1 1
a+ 2
⇒ x + 4z = …(iv)  3 9 27 
2
Multiply Eq. (i) by 3 then subtract Eq. (iii) from  
 1  1 1 1
it, = 15  {Q1 + + + ... ∞
1
1 −  3 9 27
3x + 3y + 3z = 3a
 3 a
− 2x ± 3y m z = − 1 is GP series Sn = }
1−r
x + 4z = 3a − 1 …(v) 3 45
By Eqs. (iv) and (v), we get = 15 × =
2 2
a +2
= 3a −1 3 4 5
2
⇒ a + 2 = 6a − 2 49. If the inverse of the matrix A = 2 − 1 8 is
 
⇒ 6a − a = 4 ⇒ 5a = 4 B, then BT = 5 − 2 7 
∴ b − 5a = 7 − 4 = 3
[22 April 2019, Shift-II]
7 5 4 3 − 5 3   9 1 
48. Let A =  , B = 7 5 and C =  7
26
4 8     − 4  (a)
1 
− 38 − 4 26 
136  
If Tr(S) denotes the trace of a square matrix S  37 − 14 − 11
then  9 − 38 37 
1 

1 (b) 26 − 4 − 14
∑ 3k Tr{ A(BC)k } = 136 
 1

26 − 11
k=0 [22 April 2019, Shift-II]
96 AP EAMCET Chapterwise Mathematics

 9 26 1 
50. If x = α, y = β, z = γ is the solution of the
1 
(c) 37 − 14 − 11
136   system of equations x + y + z = 4,
 − 38 − 4 26  2 x − y + 3 z = 9 , 3 x + y + 2 z = 8 , then
 9 1 26  4 α + 2β + 3 γ = [22 April 2019, Shift-II]
1 
(d) − 38 26 − 4 (a) 0 (b) 1 (c) 12 (d) 19
136  
 37 − 11 − 14 Sol. (c)
Sol. (a) Given, x = α, y = β, z = γ
 3 4 5 and system of equation is
Given, A =  2 − 1 8 , then x+ y+ z=4
  2x − y + 3z = 9
 5 − 2 7
3x + y + 2z = 8
3 4 5 Q We know that AX = B ⇒ X = A−1 B
|A| = 2 − 1 8 −1
 x  1 1 1   4
5 −2 7 ∴  y  =  2 −1 3  9 ...(i)
     
= 3(− 7 + 16) − 4(14 − 40) + 5(− 4 + 5)  z   3 1 2  8
= 27 + 104 + 5 = 136 ≠ 0
1 1 1 
Thus, | A |≠ 0 and therefore A− 1 exists.
Now, A =  2 −1 3
−1 8 2 8  
∴ A11 = = 9, A12 = − = 26,  3 1 2
−2 7 5 7
1
2 −1 A −1 = (adj A)
A13 = =1 |A|
5 −2 1 1 1
A21 = −
4 5
= − 38, A22 =
3 5
= − 4, |A|= 2 −1 3 = 1(−2 − 3) − 1(4 − 9) + 1(2 + 3)
−2 7 5 7 3 1 2

A23 = −
3 4
= 26 = −5 + 5+ 5 = 5
5 −2  −5 5 5   −5 −1 4 
4 5 3 5 adj A =  −1 −1 2  =  5 −1 −1
A31 = = 37, A32 = − = − 14,    
−1 8 2 8  4 −1 −3  5 2 −3
3 4  −5 −1 4 
A33 = = − 11 Therefore, A−1 =  5 −1 −1
1
2 −1
5 
′  5 2 −3
 9 26 1   9 − 38 37 
Now, from Eq. (i),
∴ adj (A) = − 38 − 4 26 =  26 − 4 − 14
 
     x  −5 −1 4   4  3
 37 −14 −11  1 26 − 11  y  = 1  5 −1 −1  9 = 1  3 
  5    5 
Hence,  z   5 2 −3  8 14
 9 − 38 37  3 14
A− 1 =
1
(adj A) =
1 
26 − 4 − 14 = B ⇒ x = = y, z =
| A| 136   5 5
 1 26 − 11
∴4α + 2β + 3γ = 4  + 2  + 3  =
3 3 14 60
= 12
′  5  5  5 5
 9 − 38 37 
∴ BT =
1  26 − 4 − 14 51. The equation obtained by eliminating a , b, c
136  
 1 26 − 11 a b
from the equations x = ,y= ,
 9 26 1  b− c c−a
1 
= − 38 − 4 26  c
136   z= is
 37 − 14 − 11 a−b [23 April 2019, Shift-I]
Matrices and Determinants 97

1 −x x 1 −x x Sol. (c)
(a) 1 − y y = 0 (b) 1 1 −y = 0 According to Cramer’s rule,
1 −z z 1 z 1 ∆ ∆ ∆
x = α = 1 , y = β = 2 and z = γ = 3
1 −x x x y 1 ∆ ∆ ∆
2 −1 8
(c) y 1 −y = 0 (d) y x 1 =0
where ∆ = 3 4 5
− z z −1 1 x y
5 −2 7
Sol. (b) = 2(28 + 10) + 1 (21 − 25) + 8 (− 6 − 20)
Given equations = 76 − 4 − 208 = 76 − 212 = − 136
a 13 −1 8
x= ⇒ a − bx + cx = 0
b−c ∆1 = 18 4 5
b 20 −2 7
y= ⇒ ay + b − cy = 0
c−a
= 13 (28 + 10) + 1 (126 − 100) + 8 (− 36 − 80)
c
and z = ⇒ az − bz − c = 0 = (13 × 38) + 26 − (8 × 116)
a−b
= 494 + 26 − 928 = − 408
Now, on eliminating a , b , c from the above 2 13 8
equations, we get
∆ 2 = 3 18 5
1 −x x
5 20 7
y 1 −y = 0
z − z −1 = 2 (126 − 100) − 13 (21 − 25) + 8 (60 − 90)
= (2 × 26) + (13 × 4) − (8 × 30)
On applying C1 → C1 + C2 + C3, we get
= 52 + 52 − 240 = − 136
1 −x x 1 −x x
2 −1 13
1 1 −y = 0⇒ 1 1 −y = 0
∆ 3 = 3 4 18
−1 −z −1 1 z 1
5 −2 20
Hence, option (b) is correct. = 2(80 + 36) + 1 (60 − 90) + 13 (−6 − 20)
52. If A is a 3 × 3 matrix and| A| = 2, then |Adj = (2 × 116) − 30 − (13 × 26)
(Adj A)|Adj(Adj A) = [23 April 2019, Shift-I] = 232 − 30 − 338 = − 136
So, α = 3, β = 1, γ = 1
(a) 32 A (b) 64 A (c) 16 A (d) 8 A
∴ αβ + βγ + γα = 3 + 1 + 3 = 7
Sol. (a) Hence, option (c) is correct.
As, adj adj (A) = |A |n − 2 A,
3 −3 4 
where n is the order of matrix A.
54. For the matrix A = 2 −3 4 ⋅ A−1 =
and |adj adj (A)| = |A|( n − 1)
2
 
0 −1 1 
2
So, |adj adj A| (adj adj A) = |A |( n − 1) |A |n − 2 A [22 April 2018, Shift-I]
[Q|A| = 2 and n = 3] (a) A (b) A 2
(c) A 3 (d) A 4
= 24 ⋅ 2 A = 25 A = 32 A. Sol. (c)
Hence, option (a) is correct. 3 − 3 4
53. If x = α, y = β, z = γ is the solution, for the Given, A = 2 − 3 4
 
system of equations  0 − 1 1 
3 −3 4
2 x − y + 8 z = 13
Now, | A | = 2 −3 4
3 x + 4 y + 5 z = 18
0 −1 1
5 x − 2 y + 7 z = 20
= 3(− 3 + 4) + 3(2 − 0) + 4(− 2 − 0)
then αβ + βγ + γα = [23 April 2019, Shift-I] = 3+ 6 − 8 =1 ≠ 0
(a) 1 (b) 0 (c) 7 (d) − 3 Here, A11 = 1, A12 = − 2, A13 = − 2
98 AP EAMCET Chapterwise Mathematics

A21 = − 1, A22 = 3, A23 = 3 k 0 0 


A31 = 0, A32 = − 4, A33 = − 3 4  1 0 0
 l   
 1 − 2 − 2
1
 1 −1 0  ⇒  0 0  =  0 1 0
 9  0 0 1 
∴ adj (A) =  − 1 3 3  =  − 2 3 − 4  0 0 m  
   
 0 − 4 − 3  − 2 3 − 3  16 
On comparing, we get
 1 −1 0  k l m
Now, A− 1 = = − 2 3 − 4
adj(A) 
…(i) = 1, = 1 and =1
| A|   4 9 16
 − 2 3 − 3 ⇒ k = 4, l = 9, m = 16
3 − 3 4  3 − 3 4 ∴ k + l + m = 4 + 9 + 16 = 29
A2 = A ⋅ A =  2 − 3 4  2 − 3 4
56. If A and B are the two real values of k for
   
 0 − 1 1   0 − 1 1  which the system of equations x + 2 y + z = 1,
 3 −4 4 x + 3 y + 4 z = k. x + 5 y + 10 z = k 2 is consistent,
A2 =  0 − 1 0 then A + B = [22 April 2018, Shift-I]
 
 − 2 2 − 3 (a) 3 (b) 4 (c) 5 (d) 7

 3 −4 4  3 − 3 4 Sol. (a)
∴ A3 = A2 ⋅ A =  0 − 1 0   2 − 3 4 Given system of equation is,
    x + 2y + z = 1 x + 3y + 4z = k
 − 2 2 − 3  0 − 1 1 
x + 5y + 10z = k2
 1 −1 0  1 2 1
A3 =  − 2 3 − 4 …(ii) ∴ D= 1 3 4
 
 − 2 3 − 3 1 5 10
From Eqs. (i) and (ii), we get A− 1 = A3 = 1(30 − 20) − 210
( − 4) + 1(5 − 3) = 10 − 12 + 2 = 0
k / 2 0 0  Since, D=0
∴Given system of equation is consistent.
55. If A =  0 l/30  and
  Therefore, D1 = 0
 0 0 m / 4  1 2 1
1 / 2 0 0  D1 = k 3 4
A −1 =  0 1 / 3 0  , then k + l + m = k2 5 10
 
 0 0 1 / 4  ⇒ 1(30 − 20) − 210
( k − 4k2) + (5k − 3k2) = 0
[22 April 2018, Shift-I] ⇒ 10 − 20k + 8k2 + 5k − 3k2 = 0
(a) 1 (b) 9 ⇒ 5k2 − 15k + 10 = 0
(c) 14 (d) 29
⇒ k2 − 3k + 2 = 0
Sol. (d)
⇒ (k − 2) (k − 1) = 0
Given,
⇒ k = 2, 1
k / 2 0 0  1 / 2 0 0 
Hence, the real values of k i.e.
A = 0 l / 3 0 and A = 0 1 / 3 0 
  −1 
A = 2 and B = 1
   
 0 0 m / 4  0 0 1 / 4 ∴ A + B = 2+ 1 = 3
Since, AA− 1 = I
57. If A is a square matrix of order 3 and
 k 0 0  1 0 0 A 2 + A + 2 I = 0 , then [22 April 2018, Shift-II]
2  2  1 0 0
 l  1   0 1 0 (a) A can not be a skew-symmetric matrix
⇒  0 0  0 0 =
3 3   (b)| A + I| = 0
    0 0 1 
 0 0 m  0 1 (c) A is non singular and A −1 = ( A + I)−1
0
 4   4  (d)| A|| A + I| = 2
Matrices and Determinants 99

Sol. (a) 5 −2 3
Given matrix equation A + A + 2I = 02 ∆1 = 0 −2 1 = 5(6 − 2) + 2 (0 − 6) + 3(0 + 12)
⇒ A(A + I) = − 2I 6 2 −3
⇒ A(A + I) = −2I = 20 − 12 + 36 ≠ 0
⇒ A A + I = (−2)3 So, no solution.

⇒ A (A + I) = − 8 60. The values of t such that the matrix


⇒ A ≠ 0 and A + I ≠ 0 1 3 2 
and the determinant of skew-symmetric matrix.
 
 2 5 t  has no inverse, are
having odd order is zero. By here A ≠ 0. So, A  4 7 − t − 6
can not be a skew-symmetric matrix.  
[23 April 2018, Shift-I]
58. If A is a square matrix of order 3, then (a) 3, 2 (b) 3, − 2 (c) − 3, 2 (d) − 3, − 2
consider the following statements.
Sol. (c)
I. If| A| = 0, then|Adj A| = 0
1 3 2
II. If| A| ≠ 0 , then| A −1| = | A|−1 Let A =  2 5 t  has no inverse
 
Which of the above statements is/are true?  4 7 − t − 6
[22 April 2018, Shift-II]
So,|A | = 0
(a) Both I and II (b) Neither I nor II
⇒ 1 [− 30 − t (7 − t)] − 3 [−12 − 4t]
(c) I only (d) II only
+ 2 [2(7 − t) −20] = 0
Sol. (a) ⇒ − 30 − 7 t + t 2 + 36 + 12t + 28 − 4t − 40 = 0
For a square matrix A of order 3, ⇒ t 2 + t − 6 = 0 ⇒ t 2 + 3t − 2t − 6 = 0
3−1 2
Adj⋅ A = A = A
⇒ t(t + 3) −2 (t + 3) = 0 ⇒ (t + 3) (t − 2) = 0
If A = 0, then Adj⋅ A = 0 ⇒ t = − 3, + 2
and A ⋅ A−1 = I, if A ≠ 0 So, values of t = − 3 and 2
⇒ A ⋅ A−1 = I ⇒ A A −1 = 1 61. If the point P(α , β , γ) lies on the plane
⇒ A −1
= A
−1 1 9 1
2 x + y + z = 1 and [α β γ ] 8 2 1 = [0 0 0 ],
So, statements I and II, both are correct.  
7 3 1
59. The system of equations x − 2 y + 3z = 5,
then α 2 + β 2 + γ 2 = [23 April 2018, Shift-I]
2 x − 2 y + z = 0 , − x + 2 y − 3 z = 6 has
[22 April 2018, Shift-II] (a) 34 (b) 43 (c) 68 (d) 86
(a) infinitely many solutions Sol. (d)
(b) exactly two solutions Point P(α , β , γ) lies on plane 2x + y + z = 1 …
(c) unique solution 2α + β + γ = 1 ...(i)
(d) no solution 1 9 1
Sol. (d) Now, [α β γ]  8 2 1 = [0 0 0]
 
Given system of equations,  7 3 1
x − 2y + 3z = 5 …(i)
∴[α + 8β + 7γ 9α + 2β + 3γ α + β + γ] = [0 0 0]
2x − 2y + z = 0 …(ii)
⇒ α + 8 β + 7γ = 0 … (ii)
− x + 2y − 3z = 6 …(iii)
9α + 2β + 3γ = 0 … (iii)
1 −2 3
and α+β+ γ=0 … (iv)
Q ∆ = 2 −2 1
Form Eqs. (i) and (iv), we get
−1 2 −3 2α + β + γ = 1 ⇒ α + β + γ = 0 ⇒ α = 1
= 1(6 − 2) + 2(−6 + 1) + 3(4 − 2) put in Eqs. (ii) and (iii), we get
= 4 − 10 + 6 = 0 1 + 8 β + 7γ = 0
100 AP EAMCET Chapterwise Mathematics

⇒ 8 β + 7γ = − 1 … (v) 64. If a ≠ 1, b ≠ − 1, c ≠ − 1 and the system of


and 91() + 2β + 3γ = 0
equations, x = a(y + z), y = b(z + x),
⇒ 2β + 3γ = − 9 … (vi) z = c(x + y) has a non-trivial solution, then.
Eq (v) − 4 × Eq. (vii) [23 April 2018, Shift-II]
8 β + 7γ = − 1 a b c
(a) + + =0
8 β + 12γ = − 36 a+1 b+1 c+1
− − + a b c
− 5γ = 35 (b) + + =1
a+1 b+1 c+1
⇒ γ=−7
abc
Put value of γ in Eqs. (vi), we get (c) =1
2β + 3 (−7) = − 9 ⇒ 2β = 12 ⇒ β = 6 (a + 1) (b + 1) (c + 1)
a+ b+c
Now, α 2 + β 2 + γ 2 (d) =2
(a + 1) (b + 1) (c + 1)
= ()
1 2 + (6)2 + (− 7)2 = 1 + 36 + 49 = 86
Sol. (b)
62. If the system of linear equations given by For system of homogeneous equation, if it has
x + y + z = 3, 2 x + 2 y − z = 3, x + y − z = 1 is non-trivial solution, then ∆ = 0, so
consistent and if (x 0 , y 0 , z 0) is a solution, 1
− 1 1
then 2 x 0 + 2 y 0 + z 0 = [23 April 2018, Shift-I] 1 −a −a a
1
(a) 0 (b) 5 −b 1 −b =0 ⇒ 1 − 1 =0
(c) 7 (d) 6 b
−c −c 1 1
Sol. (b) 1 1 −
c
The system of linear equation given by
R2 → R2 − R1 and R3 → R3 − R1
x+ y+ z=3 … (i) 1
2x + 2y − z = 3 … (ii) − 1 1
x + y − z =1 … (iii) a
1 1
In consistent adding Eqs. (i) and (iii), we get ⇒ 1+ − −1 0 =0
a b
2x + 2y = 4 ⇒ x + y = 2 1 1
1+ 0 − −1
put in Eq. (i) a c
2+ z = 3 ⇒ z =1
⇒−  + 1  + 1 + 1 +  1 + 
1 1   1   1 1
Then, 2x 0 + 2y0 + z0 ab  c   a  c
= 2(x 0 + y0) + z0  
+  + 1 1 +  = 0
1 1
= 2× 2+ 1 = 5 [Q x + y = 2 ⇒ x 0 + y0 = 2] b   a
−1
0 (1 + a) (1 + c) (1 + a) (1 + b) (1 + b) (1 + c)
63. If A =  , then the incorrect option ⇒ + =
 10  ac ab abc
b c 1
among the following is [23 April 2018, Shift-II] ⇒ + = −1 + 1
1+ b 1+ c 1+ a
(a) A 3 − I = A( A − I) (b) ( A 3 + I) = A( A 3 − I) a b c
(c) A − I = A + I
4 2
(d) A 2 + I = A( A 2 − I) ⇒ + + =1
1+ a 1+ b 1+ c
Sol. (d)
65. The rank of the following matrix A is
 0 − 1
For the given matrix A =  , the −2 3 −4 
1 0 
1
2 9 4 5
characteristics equation is A2 + I = 0, so the
A= 
A3 = − A and A4 = I. 4 5 10 −3
1 11 −1 9 
Now, if we analyse, the option  [23 April 2018, Shift-II]
A2 + I = A(A2 − I) is incorrect.
(a) 4 (b) 3 (c) 2 (d) 1
Matrices and Determinants 101

Sol. (b) Sol. (b)


1 − 2 3 − 4 Given,
2 9 4 5 A2 x = cAx → A
Given matrix A =  
 4 5 10 − 3 Ax = c
1 11 − 1 9 
  Ax → c is eigen value of A.
R3 → R3 − (R2 + 2R1) and R4 → R4 − (R2 − R1) ∴ Eigen value of A are 8, 2, 2
1 − 2 3 − 4 Therefore, number of values of c is 2.
2 9 4 5
or A= , 68. A set of values of θ for which the system of
0 0 0 0  equations (sin 3θ) x − y + z = 0 ,
0 0 − 2 0 
  (cos 2θ) x + 4 y + 3 z = 0 , 2 x + 7 y + 7 z = 0 has
So rank (A) = 3. non-trivial solutions, is [24 April 2018, Shift-I]
π π
125 5 25 (a) (n + 1) + (−1)n (here n is any integer)
2 4
66. 343 7 49 = π π
(b) (n − 1) + (−1)n (here n is any integer)
729 9 81 [24 April 2018, Shift-I] 2 3
nπ π
(a) 9! (b) 6! (c) 8! (d) 7! (c) + (−1)n (here n is any integer)
2 6
Sol. (d) π
125 5 25 (d) nπ + (−1)n (here n is any integer)
6
343 7 49
Sol. (d)
729 9 81
For non-trivial solution, ∆ = 0
1 1 1 sin 3θ − 1 1
Applying R1 → R1 , R2 → R2, R3 → R3
5 7 9
cos 2θ 4 3 = 0
25 1 5
2 7 7
= 5⋅ 7 ⋅ 9 49 1 7
81 1 9 On expanding,
⇒ sin 3θ(28 − 21) + 1(7 cos 2 θ − 6)
Applying R2 → R2 − R1 , R3 → R3 − R1
+ 1(7 cos 2 θ − 8) = 0
25 1 5
⇒ 7sin 3θ + 14 cos 2θ − 14 = 0
= 5⋅ 7 ⋅ 9 24 0 2
⇒ sin 3θ + 2cos 2θ − 2 = 0
56 0 4
⇒ 3sinθ − 4sin3 θ − 21
( − cos 2θ) = 0
expanding for a12
⇒ 3sinθ − 4sin3 θ − 4sin2 θ = 0
= − 5⋅ 7 ⋅ 9(96 − 112) = 5.7.9.16 = 7!.
⇒ − sinθ(4sin2 θ + 4sinθ − 3) = 0
 5 −3 0 
67. Let matrix A = −3 5 0, X be a non zero ⇒ sinθ = 0, 4sin2 θ + 4sinθ − 3 = 0
 
⇒ sinθ = 0, (2sinθ − 1) (2sinθ + 3) = 0
 0 0 2
1
matrix of order 3 × 1 and c be a real number. ⇒ sinθ = 0, sinθ = [2sinθ + 3 ≠ 0]
2
If A 2 X = cAX , then the number of distinct
⇒ θ = nπ
values of c is [24 April 2018, Shift-I] π
(a) 3 (b) 2 (c) 1 (d) 0 θ = nπ + (− 1)n .
6
10
Measures of Dispersion
1. If the standard deviation of the numbers 2  n  n
= 1 + 2 + .... +    terms
2, 3, 2x and 11 is 3.5. Find the possible n 2  2
values of x. [17 Sep. 2020, Shift-I]   − 1 + 1 
n n
 
(a) 2 ⋅
7
(b) 3 ⋅
5 2   n
= 2 2 =
2 3 n 2 4
5 7 
(c) 2 ⋅ (d) 3 ⋅  
2 3
Sol. (d) 3. If the mean of 100 observations is 50 and
σ = 35
. their standard deviation is 5, then the sum
2 + 3 + 2x + 11 of squares of all observations is .........
=  4 + 
x
Mean (x) = [17 Sep. 2020, Shift-II]
4  2
1 (a) 50000 (b) 250000
σ = ∑ (x i − x)2
2
(c) 252500 (d) 255000
n
1  Sol. (c)
2 2 2
⇒ (3⋅ 5) =   −2 −  +  −1 − x  +  3x − 4
2 x
    Given; x = 50
4  2  2  2 
n = 100 and S. D (σ) = 5
x 
2
+  7 −   As, x =
Σx i
⇒ Σx i = n. x = 5000.
 2  n
7 Also, S.D. is given by,
⇒ 3x 2 − 16 x + 21 = 0 ⇒ x = ,3
2
3 Σ x i 2  Σx i 
σ= − 
2. The mean deviation about the mean of the n  n 
set of first ‘n’ natural numbers, when ‘n’ is Σx i2 Σx i2
⇒ σ2 = − (x)2 ⇒ 25 = − (50)2
an even number is equal to n 100
[17 Sep. 2020, Shift-II] ⇒ Σx i2 = 252500
n n n
(a) n (b) (c) (d)
2 3 4 4. What is the value of x if the mean of 8, 6, 7,
Sol. (d) 5, x and 4 is 7? [18 Sep. 2020, Shift-II]
1 + 2 + ....n n (n + 1) n + 1 (a) 10 (b) 12 (c) 8 (d) 6
Mean x = = =
n 2n 2 Sol. (b)
Here, n = even number Mean = 7
Now mean deviation is 8 + 6 + 7 + 5+ x + 4
=7
n+1 n+1
M. D (x) =  1 − + 2− 6
 2 2 30 + x = 42
n+1  1 x = 12
+ ....+ n − ×
2  n Hence, option (b) is correct.
Measures of Dispersion 103

5. What is the formula for finding coefficient of The mean of given series
variation, given σ = standard deviation and a , a + d , a + 2d , ......, a + 2nd is
2n + 1
x = mean ≠ 0? [21 Sep. 2020, Shift-I] [2a + (2n)d]
x x 2 = (a + nd) = m (let)
(a) × 100 (b) 2n + 1
σ σ
σ σ 1 2n + 1
(c) (d) × 100 So, the mean deviation = ∑ |x i − m|
x x 2n + 1 i =1
1
Sol. (d) = [|nd|+|nd − d|+|nd − 2d|+ ....+|nd − 2nd|]
2n + 1
Coefficient of variation, having standard 2d
deviation σ and mean x ≠ 0, is = [n + (n − 1) + (n − 2)+ ....+1]
2n + 1
σ
× 100 n(n + 1)d
x =
2n + 1
Hence, option (d) is correct.
Hence, option (b) is correct
6. The mean of 5 observations is 15 and
variance is 9. If two observations having 8. The standard deviation and mean of five
values − 5 and 13 are combined with these observations are 0 and 9 respectively. If one
observations, then what will be the new of the observations is changed such that the
variance? [21 Sep. 2020, Shift-I] mean of the new set of five observations
6259 6259 2659 2659 becomes 10, then their standard deviation is
(a) (b) (c) (d) [22 Sep. 2020, Shift-I]
7 49 7 49
(a) 1 (b) 2
Sol. (d) (c) 3 (d) 0
Let the observations are x1 , x 2 , x 3 , x 4 , x 5, so the
x1 + x 2 + x 3 + x 4 + x 5 Sol. (b)
mean x = = 15 (given)
5 Let the five observations are x1 , x 2 , x 3, x 4 , x 5.
x + x2 + x3 + x4 + x5
⇒ x1 + x 2 + x 3 + x 4 + x 5 = 75 … (i) So, mean = 1 = 9 (given)
5
and variance
⇒ x1 + x 2 + x 3 + x 4 + x 5 = 45 ...(i)
x 2 + x 22 + x 32 + x 42 + x 52
σ2 = 1 − (x)2 = 9 (given) 5 (9 − x i)2
5 and standard deviation = ∑ =0
i =1 5
⇒ x12 + x 22 + x 32 + x 42 + x 52 = 5 [9 + 225] = 1170 …(ii) (given)
5
Now, after combined the − 5 and 13 with the ⇒ ∑ (9 − x i) = 0
2

observations, the new variance i =1

x 2 + x 22 + x 32 + x 42 + x 52 + 25 + 169 ⇒ xi = 9 ∀ i
= 1 Now, let observation x 5 is changed with y, so
7 x + x2 + x3 + x4 + y
75 − 5 + 13
2 mean = 1 = 10 (given)
−   5
 7  ⇒ x1 + x 2 + x 3 + x 4 + y = 50
2
1170 + 25 + 169  83 9548 − 6889 2659 ⇒ 45 − x 5 + y = 50 (from Eq. (i))
= −  = =
7  7 49 49 ⇒ x5 + 5 = y
Hence, option (d) is correct. ⇒ y = 14 {Q x 5 = 9}
Now, the changed standard deviation is
7. The mean deviation from the mean of the
series (a),(a + d),(a + 2 d), ………, (a + 2 nd) is (10 − x1)2 + (10 − x 2)2 + (10 − x 3)2
[21 Sep. 2020, Shift-II] + (10 − x 4)2 + (10 − y)2
n(n − 1)d n(n + 1)d 5
(a) (b)
2n + 1 2n + 1 {Q
n(n + 1)d x1 = x 2 = x 3 = x 4 = 9}
(c) (n(n + 1)d ) (d)
2n 1 + 1 + 1 + 1 + 16 20
= = = 4=2
Sol. (b) 5 5
Hence, option (b) is correct.
104 AP EAMCET Chapterwise Mathematics

9. Calculate variance if Σx i2 = 18000 and 13. If a 0 , a1 , …, a11 are in an arithmetic


Σx i = 960 , for 60 observations progression with common difference d, then
[22 Sep. 2020, Shift-I] their mean deviation from their arithmetic
(a) 44 (b) 22 mean is [20 April 2019, Shift-I]
(c) 32 (d) 6.63 30
(a) |d| (b) 2|d| (c) 3|d| (d) 12|d|
Sol. (a) 11
For 60 observations it’s given that Σx i = 960 Sol. (c)
and Σx i2 = 18000 Since, mean of given data is
a + a1 + a 2 + … + a11 a 0 + a11
Σx i2 Σx
2
x= 0 =
So, the variance = −  i  12 2
60  60  a + a10
2 = 1 =…
18000  960  2
= − 
60  60  Now, deviations from their mean are
|a − a 0| 11 |d|
= 300 − (16)2 = 300 − 256 = 44 |x − a 0| = 11 =
2 2
10. What is the relation between mean and |a10 − a1| 9 |d|
|x − a1| = = , …… and so on
median of a discrete data? [22 Sep. 2020, Shift-II] 2 2
(a) They are always equal So, sum of deviations
= 2
11 |d| 9 |d| 7 |d| 5|d| 3|d| |d|
(b) They are always not equal + + + + +
(c) Sometimes they are equal  2 2 2 2 2 2 
(d) No relation exists between them = 36|d|
Sol. (c) ∴Mean deviation from their arithmetic mean is
36|d|
Mean and median of the data are sometime = 3|d|
equal 12
Hence, option (c) is correct. Hence, option (c) is correct.

11. Find the least positive value of k, if the range 14. The variance of the following continuous
of 15, 14, k, 25, 30, 35 is 23 frequency distribution is [20 April 2019, Shift-I]
[23 Sep. 2020, Shift-I] Class
0-10 10-20 20-30 30-40
(a) 11 (b) 13 Interval
(c) 12 (d) 14
Frequency 2 3 4 1
Sol. (c)
Given data is 15, 14, k, 25, 30, 35 (a) 201 (b) 62 (c) 19 (d) 84
Given Range = 23 Sol. (d)
Max-min = 23 Given,
35 − k = 23 Class Frequency ( x − xi )2
xi xi fi fi ( x − xi )2
[Q Let k be the minimum observation] Interval (fi )
− k = − 12 0-10 2 5 10 196 392
k = 12 10-20 3 15 45 16 48
Hence, option (c) is correct. 20-30 4 25 100 36 144

12. The range of the observations 20, 28, 40, 12, 30-40 1 35 35 256 256

30, 15, 50 is [23 Sep. 2020, Shift-I] Σ xi fi = 190 Σ fi ( x − xi )2


N = Σ fi = 10
= 840
(a) 18 (b) 38
(c) 28 (d) 16 Σ x i fi 190
Q x= = = 19
Sol. (b) N 10
1 1
Given observations are 20, 28, 40, 12, 30, 15, 50 ∴Variance (σ)2 = Σ fi (x − x i)2 = (840) = 84
Range = 50 − 12 = 38 N 10
Hence, option (b) is correct. Hence, option (d) is correct.
Measures of Dispersion 105

15. The variance of the observations 2, 3, 5, 7, After adding 15 to each observation of the data,
11, 13, 17, 22 is [20 April 2019, Shift-II]
the new mean µ ′ = 35 + 15 = 50 but
Σ|µ ′ − x ′i | = Σ|µ − x i|, [where x ′i = x i + 15 ]
(a) 43.75 (b) 48.25
Σ(µ − x i)2
(c) 80 (d) 350 Now, σ = = 21
n
Sol. (a)
⇒ Σ(µ − x i)2 = (21)2 × n
According to the given information,
So, new standard deviation
xi xi − x ( xi − x )2
Σ(µ ′ − x ′i )2 (21)2 × n
σ′ = = = 21
2 –8 64 n n
3 –7 49 ∴ New coefficient of variance
σ′ 21
5 –5 25 = × 100 = × 100 = 21 × 2 = 42
µ′ 50
7 –3 9 Hence, option (b) is correct.
11 1 1
17. For a data consisting of 15 observations x i ,
13 3 9 i = 1, 2 , 3 , K, 15 the following results are
15 15
∑ x i = 170; ∑ x i2 = 2830. If one
17 7 49
obtained :
22 12 144 i =1 i =1

Σ xi = 80 Σ ( xi – x ) =
2 of the observation namely 20 was found
350 wrong and was replaced by its correct value
30, then the corrected variance is
Σ x i 80
x= = = 10 [21 April 2019, Shift-I]
8 8 (a) 80 (b) 78 (c) 76 (d) 75
Q Σ( x i − x)2 = 350
Sol. (b)
Σ(x i − x)2 Given,
∴Variance of the given observations =
8 Σx 2 = 2830 and Σx = 170
350
= = 4375
. Increase in Σx = 10
8
and increase in Σx 2 = (30)2 − (20)2
Hence, option (a) is correct.
= 900 − 400 = 500
16. The coefficient of variation and standard Σx′ = 170 + 10 = 180
deviation of an ungrouped data are 60 and and Σx′2 = 2830 + 500 = 3330
21 respectively. If 15 is added to every 2
observation of the data, then the coefficient Σx 2  Σx 
We know that, variance = − 
of variation of the new data is n  n
2
[20 April 2019, Shift-II] 3330  180 
= −  [here, n = 15]
(a) 30 (b) 42 (c) 40 (d) 20 15  15 
Sol. (b) = 222 − 144 = 78
According to the given information, Coefficient
σ 18. The mean deviation about the mean for the
of variance (CV) = × 100 following data is [21 April 2019, Shift-I]
µ
Where σ is standard deviation and µ is mean of Class 10- 20- 30- 40- 50- 60-
an ungrouped data. 0-10
interval 20 30 40 50 60 70
σ
Q × 100 = 60 and σ = 21 Frequency 4 6 16 28 16 6 4
µ
21 × 10 (a) 35 (b) 10
So, µ= = 35
6 (c) 15 (d) 12
106 AP EAMCET Chapterwise Mathematics

10
Sol. (b) ⇒ ∑ x i yi − 10 x y = c …(i)
i =1
Class | xi − 35| f|i xi − 35| 1
fi xi fi xi Again, σ( x i − yi ) = Σ(x i − yi)2 − (x − y)2
interval 10
0-10 4 5 20 30 120
=  1 Σx 2 + 1 Σy 2 − 1 Σx y − x 2 − y 2 + 2x y 

 10 i 10 i 5
i i

10-20 6 15 90 20 120
  1 Σx 2 − (x)2  +  1 Σy 2 − (y)2  
  10 i   
20-30 16 25 400 10 160
  10 i  
=  
30-40 28 35 980 0 0 1
 − (Σx i yi − 10 x y)
40-50 16 45 720 10 160  5 
c
50-60 6 55 330 20 120 = a2 + b2 − [By Eq. (i)]
5
60-70 4 65 260 30 120
20. For a group of 100 students, the mean x1
Total 80 2800 800
and the standard deviation σ1 of their marks
Σf x 2800 were found to be 40 and 15 respectively.
Q X= i i = = 35
Σfi 80 Later it was observed that the scores 40 and
50 were misread as 30 and 60 respectively. If
1 n
M.D. = ∑ fi | x i − x | the mean and the standard deviation with
N i =1 the corrected observations of the scores, are
=
1 n
∑ fi | x i − 35| [Here, N = 80]
x 2 and σ 2 respectively, then
80 i =1 [21 April 2019, Shift-II]
1 (a) x1 = x2 ; σ1 = σ 2 (b) x1 = x2 ; σ1 < σ 2
= × 800 = 10
80 (c) x1 = x2 ; σ1 > σ 2 (d) x1 > x2 ; σ1 = σ 2

19. The standard deviations of x i (i = 1, 2, …, 10) Sol. (c)


Given, x1 = 40 and σ1 = 15
and y i (i = 1, …, 10) are respectively ‘a’ and ‘b’. Σx i
x , y are the means of these two sets of Now, x1 =
n
observation respectively. If
Σx i
10 ⇒ 40 = ⇒ Σx i = 4000
z i = (x i − x)(y i − y) and ∑ z i = c, then the 100
i =1 Corrected Σx i = Σx i − 30 − 60 + 40 + 50 = 4000
standard deviations of the observation Corrected (Σx i) 4000
(x 2) = = = 40
= (x i − y i ), (i = 1, 2 , …, 10) is n 100
[21 April 2019, Shift-II] Now, σ1 = 15, σ12 = 225
c c 2
(a) a + b +
2 2
(b) a2 + b 2 − Σx i2  Σx i 
5 5 Q σ12 = − 
n  n 
c2 c2
(c) a2 + b 2 − (d) a2 + b 2 + Σx i2  4000 
2
5 5 ⇒ 225 = − 
100  100 
Sol. (b) Σx i2
We have, ⇒ 225 = − 1600
10 10 100
∑ zi = c ⇒ ∑ [(x i − x)(yi − y)] = c ⇒ 22500 = Σx12 − 160000
i =1 i =1
10 ⇒ Σx i2 = 182500
⇒ ∑ (x i yi − x i y − xyi + x ⋅ y) = c
i =1 Corrected Σx i2 = 182500 − (30)2 − (60)2 + (40)2 + (50)2
10 10 10 10
⇒ ∑ (x i yi) − y ∑ x i − x ∑ yi + x y ∑ 1 = c = 182500 − 900 − 3600 + 1600 + 2500 = 182100
i =1 i =1 i =1 i =1 2
corrected Σx i2  corrected Σx i 

10
∑ (x i yi) − 10 x y − 10 x y + 10 x y = c ∴ σ 22 = − 
i =1 n  n 
Measures of Dispersion 107

182100  4000 
2 1
= −  = 1821 − 1600 ∴Mean deviation = Σfi|x i − 21|
100  100  N
⇒ σ 22 = 221 ⇒ σ 2 = 221 = 14.86 288
= = 9.6
30
Hence, x1 = x 2 and σ1 > σ 2
23. The mean deviation of the data 3, 5, 11, 13,
21. The coefficient of variation of 9, 3, 11, 5, 7, is
17, 19, 23, 29 about its arithmetic mean is
[22 April 2019, Shift-I]
[22 April 2019, Shift-II]
100 2 200 2 200 2 100 2
(a) (b) (c) (d) (a) 8.5 (b) 8
7 3 7 3 (c) 7.2 (d) 7
Sol. (c) Sol. (d)
Given numbers are 9, 3, 11, 5, 7. We have,
9 + 3 + 11 + 5 + 7 35
Now, mean (x) = = =7 3 + 5 + 11 + 13 + 17 + 19 + 23 + 29
5 5 X= = 15
8
1
Variance σ 2 = (92 + 32 + 112 + 52 + 72) − (7)2 ∴ Mean deviation about the mean
5
1 | 3 − 15| + | 5 − 15| + |11 − 15| + |13 − 15|
= (81 + 9 + 121 + 25 + 49) − 49
5 + |17 − 15| + |19 − 15| + | 23 − 15| + | 29 − 15|
=
285 8
= − 49 = 57 − 49 = 8
5 12 + 10 + 4 + 2 + 2 + 4 + 8 + 14 56
= = =7
∴Coefficient of variation 8 8
σ2 8 200 2 24. If the weights of 10 persons (in kgs) are
× 100 = × 100 =
x 7 7 observed as : 45, 49, 55, 50, 41, 44, 60, 58, 53,
22. The mean deviation about the mean for the 55, then the variance of their weights is
[22 April 2019, Shift-II]
following data [22 April 2019, Shift-I]
(a) 51 (b) 42.8 (c) 39.4 (d) 35.6
Marks
obtained
0-10 10-20 20-30 30-40 40-50 Sol. (d)
Variance of their weight
Number of 1
Boys
6 8 10 4 2 = (452 + 492 + 552 + 502 + 412 + 442 + 602
10
+ 582 + 532 + 552)
is 2
(a) 9.33 (b) 5.6  45 + 49 + 55 + 50 + 41 + 44 + 60 
 + 58 + 53 + 55
(c) 8.33 (d) 9.6 − 
 10 
Sol. (d)
 
Calculation of mean deviation about mean
1
Marks = [2025 + 2401 + 3025 + 2500 + 1681 + 1936
xi fi fi xi | xi −21| f|i xi −21| 10
obtained
+ 3600 + 3364 + 2809 + 3025] − (51)2
0-10 5 6 30 16 96
1
= [26366] − 2601 = 2636.6 − 2601 = 356
.
10-20 15 8 120 6 48 10
20-30 25 10 250 4 40 25. The variance of the following data is
30-40 35 4 140 14 56 [23 April 2019, Shift-I]

40-50 45 2 90 24 48 xi 1 2 3 4 5 6 7 8 9 10
N = Σfi Σfi xi Σf|i xi − 21| fi 1 2 3 4 5 6 7 8 9 10
= 30 = 630 = 288
Σfi x i 630 (a) 10 (b) 9 (c) 8 (d) 6
Mean x= = = 21
Σfi 30 Sol. (d)
108 AP EAMCET Chapterwise Mathematics

According to given data, 4(9.5)2 + 1(4.5)2 + 10(0.5)2 + 3(55


. )2 + 210
( .5)2
=
xi fi xi fi x − xi fi ( x − xi )2 20
139
=
1 1 1 6 36 4
2 2 4 5 50 σ 139 25 139
∴ CV = × 100 = × 100 =
3 3 9 4 48 x 2 × 12 6
4 4 16 3 36 Hence, option (b) is correct.
5 5 25 2 20 27. The mean and the standard deviation of a
6 6 36 1 6 data of 8 items are 25 and 5 respectively. If
7 7 49 0 0 two items 15 and 25 are added to this data,
8 8 64 1 8 then the variance of the new data is
[22 April 2018, Shift-I]
9 9 81 2 36
(a) 29 (b) 24 (c) 26 (d) 29
10 10 100 3 90
Sol. (a)
Σx i fi 10 × 11 × 21 We have, n = 8, x = 25 and σ = 5
Q N = Σfi = 55, x = = =7
N 6 × 55 Σx i
Q x=
Σfi (x − x i)2 n
∴ σ2 =
N ⇒ Σx i = nx = 8 × 25 = 200
36 + 50 + 48 + 36 + 20 + 6 + 8 + 36 + 90 ⇒ In corrected Σx i = 200 and σ = 5
=
55 1
⇒ σ 2 = 25 ⇒ Σx i2 − (mean)2 = 25
330 n
= =6
55 Σx i2
Hence, option (d) is correct. ⇒ − 625 = 25 ⇒ Σx i2 = 5200
8
26. The coefficient of variation of the following Corrected Σx i2 = 5200 + 225 + 625 = 6050
distribution is [23 April 2019, Shift-I]
and corrected mean = 200 + 15 + 25 = 240
Class 6050  240 
2
0-5 5-10 10-15 15-20 20-25 ∴Corrected variance = −
interval 
10  10 
Frequency 4 1 10 3 2
= 605 − (24)2 = 605 − 576 = 29
25 139 25 139 139 25 × 139
(a) (b) (c) (d) 28. The mean deviation from the median for the
12 6 6 12
following distribution (corrected to two
Sol. (b) decimals) is [22 April 2018, Shift-I]
According to given data,
xi 6 9 3 12 15 13 21 22
Class Frequency xi xi fi | x − xi |
Interval fi fi 4 5 3 2 5 4 4 3
0-5 4 2.5 10 9.5 (a) 13.42 (b) 5.45 (c) 4.97 (d) 11.25
5-10 1 7.5 7.5 4.5 Sol. (*)
10-15 10 12.5 125 0.5
xi fi Cumulative (d i ) = | xi − 15| fi | d i |
15-20 3 17.5 52.5 5.5 frequency
20-25 2 22.5 45 10.5 6 4 4 9 36
N = Σfi Σxi fi 9 5 9 6 30
= 20 = 240
3 3 12 12 36
Σx i fi 240 Σf (x − x i) 2 12 2 14 3 6
∴ x= = = 12 ⇒ σ 2 = i
N 20 N 15 5 19 0 0
Measures of Dispersion 109

xi fi Cumulative (d i ) = | xi − 15| fi | d i |
Sol. (b)
frequency Mid interval
Marks f f ( x) x2 f( x 2 )
13 4 23 2 8 value ( x)
21 4 27 6 24 1-3 2 40 80 4 160
22 3 30 7 21 3-5 4 30 120 16 480
5-7 6 20 120 36 720
N = Σfi Σ|fd
i i|
= 30 = 161 7-9 8 10 80 64 640
2
∑ f⋅x 2
∑ f ⋅ x
 = 20 − 16 = 4
N Q Variance = −
Clearly, N = 30 ⇒ = 15  ∑ f 
2 ∑ f
N
The cumulative frequency just greater than is 31. The standard deviation of the numbers 22,
2
19 and corresponding value of x is 15. 26, 28, 20, 24, 30 is [23 April 2018, Shift-I]
Therefore, median = 15. (a) 2 (b) 2.4 (c) 3.24 (d) 3.42
Clearly, Σfi | x i − 15| = Σfi di = 161 and N = 30 Sol. (d)
161 Given numbers 22, 26, 28, 20, 24, 30
∴ Mean deviation = = 5⋅ 40 (approx)
30 22 + 26 + 28 + 20 + 24 + 30 150
x= = = 25
6 6
29. The arithmetic mean and standard deviation Now, (x i − x)2 is 9, 1, 9, 25, 1, 25
of a data of nine numbers are 13 and 5
respectively. If 3 is included as the 10th item Σ(x i − x)2
Now, SD =
of the data, then the variance of the data of n
ten number is [22 April 2018, Shift-II] 9 + 1 + 9 + 25 + 1 + 25 70
= = = 342
.
(a) 23.5 (b) 21.5 (c) 31.5 (d) 27 6 6
Sol. (c) 32. The marks obtained by students A and B in 3
9
∑ xi 9 examinations are given below
Q i =1 = 13 ⇒ ∑ x i = 117.
9 i =1 Marks of A 30 20 40
9
∑ x i2 Marks of B 70 0 5
and σ 2 = 25 = i =1 − (13)2
9
9
The ratio of the coefficient of variation of
⇒ ∑ x i2 = 9 [25 + 169] marks of A and the coefficient of variation of
i =1
marks of B is [23 April 2018, Shift-I]
Now, after including 10 th item as ‘3’
(a) 3 : 1 (b) 5 : 8 3
9
∑ xi + 3 (c) 1 : 3 (d) 5 : 3 61
i =1 117 + 3
New mean y = = = 12 Sol. (d)
10 10
Marks of A = 30, 20, 40.
 ∑9 x 2 + 9 30 + 20 + 40
 i  xA = = 30
 i =1 
and new variance = − (12)2 3
10
02 + (− 10)2 + (10)2 200
(169 + 25 + 1)9 1755 − 1440 σA = =
= − 144 = = 31.5 3 3
10 10
 (x i − x)2
30. The variance of the following distribution is σ =
 n
[22 April 2018, Shift-II]
2  σ
CV = 
200 1 10 2
(CV) A = × = =
30 30 3 3 3  x 
Marks 1-3 3-5 5-7 7-9
3
Number of students 40 30 20 10 Now, marks of B = 70, 0, 5
70 + 0 + 5
(a) 2 (b) 4 (c) 6 (d) 8 xB = = 25
3
110 AP EAMCET Chapterwise Mathematics

(70 − 25)2 + (0 − 25)2 + (5 − 25)2 So, variance


σB =
3 202 + 122 + 62 + 62 + 122 + 202 + 362
=k
3050 30
σB =
3  10 + 6 + 3 + 32 + 62 + 102 + 182 
2 2 2
= 4 × k 
3050 1 5 122 1 122  30 
(CV)B = × = =
3 25 25 3 5 3 = 4 × 45. 8 = 183. 2
2 1 122 2 1 2 61 35. If the variance of 6, 7, 8, 9, 10, 11 is σ 2, then
(CV) A: (CV)B = : = :
3 3 5 3 3 3 5 3 the variance of 12, 14, 16, 18, 20, 22 is
= 5 : 3 61. [24 April 2018, Shift-I]
(a) 2 σ 2 (b) 4σ 2
33. The standard deviation of the scores 505, 510,
(c) 100 + 2 σ 2 (d) 100 + 4σ 2
515, 520, ……, 595 is [23 April 2018, Shift-II]
(a) 500 + 5 30 (b) 505 + 30 Sol. (b)
(c) 5 30 (d) 5 + 30 Its a fundamental concept if each data is
Sol. (c) doubled then variance is 4 times.
Given, scores 505, 510, 515, 520, ... 595 the So, new variance = (2)2 σ 2 = 4σ 2
mean of these 19 scores = 550 = x
19 36. Consider the frequency distribution
So, ∑ (x i − x)2 = 2(452 + 402 + 352 + K + 52) C.I. 75-175 175-275 275-375 375-475 475-575 575-675 675-775
i =1
= 2 × 52 × [12 + 22 + 32 + K + 92]
fi 3 2 1 0 1 2 3
So, standard derivation
19 9 × 10 × 19 If the variance of this distribution is 60000,
∑( xi − x )2 2 × 52 × then the coefficient of variation of the
= i =1 = 6 = 5 30
n 19 distribution is [24 April 2018, Shift-I]
400 6 400 6
34. If the variance of the distribution (a) 60 (b) (c) (d) 59575
.
17 9
xi 4 8 11 17 20 24 32 Sol. (b)
fi 3 5 9 5 4 3 1
CI fi Mid value xi fi xi
is 45.8, then the variance of the distribution.
[23 April 2018, Shift-II] 75-175 3 125 375
175-275 2 225 450
xi 10 18 24 36 42 50 66
fi 3 5 9 5 4 3 1 275-375 1 325 325

(a) 93.6 (b) 93.9 (c) 183.2 (d) 183.2 375-475 0 425 0

Sol. (c) 475-575 1 525 525


Mean of given observation (x i) is 575-675 2 625 1250
(4 × 3) + (8 × 5) + (11 × 9) + (17 × 5)
675-775 3 725 2175
+ (20 × 4) + (24 × 3) + (32 × 1) 420
= = 14 Σfi = 12 Σfi xi
3+ 5+ 9 + 5+ 4 + 3+ 1 30
= 5100
10 + 6 + 3 + 3 + 6 + 10 + 18
2 2 2 2 2 2 2
So, variance = k Σfi x i 5100
30 X= = = 425
= 45. 8 (given) Σfi 12
Now, mean of given observation (yi) is σ
CV = × 100 [as σ 2 = 60000]
(10 × 3) + (18 × 5) + (24 × 9) + (36 × 5) X
+ (42 × 4) + (50 × 3) + (66 × 1) 900 100 6 400 6
= = 30 = × 100 = .
3+ 5+ 9 + 5+ 4 + 3+ 1 30 425 17
11
Probability
1. Let X be a random variable such that X (x) is Which is probability of a sure event.
the number of heads in X for each x ∈S, As all other values of P (A ∪ B) are less than 1.
So this is always true.
where S is the sample space of random
experiment of tossing three fair coins 4. A die is rolled three times. The probability of
simultaneously. Find the value of ρ(X −1 (2)). getting their sum equal to a prime number of
[17 Sep. 2020, Shift-I] the form 4 n + 1 is ...... [17 Sep. 2020, Shift-II]
3 5 1 3 1 7 5 11
(a) (b) (c) (d) (a) (b) (c) (d)
8 8 8 4 6 36 36 36
Sol. (a) Sol. (c)
Tossing of three coins We obtain following prime sum in three throws
So, ρ(X −1 (2)) means probability of getting two of dice;
tails when three coins are tossed. 3, 5, 7, 11, 13 and 17
3 And sum in from of 4m + 1 are,
⇒ ρ(X −1 (2)) =
8 5, 13 and 17
Following are favourable outcomes for above
2. A bag consists of 3 red balls, 5 blue balls and sums
8 green balls. A ball is selected at random. (1, 2, 2), (2, 1, 2), (2, 2, 1)
What is the probability of not getting a blue (3, 1, 1), (1, 3, 1), (1, 1, 3)
ball? [17 Sep. 2020, Shift-I] (5, 3, 3), (3, 5, 3), (3, 3, 5)
11 3 1 5 (6, 6, 1), (1, 6, 6), (6, 1, 6)
(a) (b) (c) (d)
16 16 2 16 (4, 4, 5), (5, 4, 4), (4, 5, 4)
Sol. (a) (2, 6, 5), (6, 2, 5), (5, 2, 6)
5 11 ………… etc.
P(B) = ⇒ P(B) =
16 16 Total number of out comes of obtaining sum a
prime of from '4m + 1’ = sum 5(6 times) + sum
3. Suppose A, B be two events such that 13 (21 times) + sum 17 (3 times) = 30 cares.
P(A) =0 .9 and P(B) = 0 .8 and P(A ∩ B) ≥ 0 .7. Total number of possible outcomes
Then, we can conclude that such a case is = 6 × 6 × 6 = 216 cares
.......... [17 Sep. 2020, Shift-II]
Hence, required probability =
30 5
= .
(a) Always true 216 36
(b) Always false
5. A box contains 4 defective and 6 good
(c) Not true in some examples
machines. Two machines are selected at
(d) True only in some cases
random without replacement. Find the
Sol. (a) probability that both the machines are good.
Maximum value of probability of A or B is, [17 Sep. 2020, Shift-II]
P (A ∪ B) = P (A) + P (B) − P (A ∩ B) 1 1 1 1
(a) (b) (c) (d)
= 0.9 + 0.8 − 0.7 = 1 2 3 4 5
112 AP EAMCET Chapterwise Mathematics

Sol. (b)  A
P (E 2) ⋅ P  
 E2 
P  2  =
Box contains 4 defective and 6 good machines. E
∴Total number of machines = 10  A  A  A
P(E 2). P   + P(E1).P  
Probability that first machine selected is good  E2   E1 
Number of good m / c 6  A
= = + P(E 3). P  
Total number of m / c 10  E3 
5
Probability that second machine is good = 1 1
×
9 3 2 15
= =
Probability that both machines are good 1 1 1 3 1 1 29
× + × + ×
6 5 1 3 2 3 10 3 6
= × = .
10 9 3
7. There are two dice A and B. Die A has 4 red
6. The content of the 3 boxes are as follows. If and 2 white faces and B has 2 red and
one box is chosen at random and three balls 4 white faces. A coin is tossed once, if it
are drawn from it and they are all of shows head, die A is rolled, if it shows tail,
different colours, find the probability that die B is rolled, if the probability that die A is
they come from box 2. [17 Sep. 2020, Shift-II]  32 
used is   when it is given that red turns
Box 1 contains 1 black, 2 white, 3 red balls,  33 
and up every time in first n throws, then n =
Box 2 contains 1 black, 1 white, 2 red balls, [18 Sep. 2020, Shift-I]
and (a) 5 (b) 6
Box 3 has 5 black, 4 white, 1 red balls. (c) 4 (d) 3
9 15 5 6 Sol. (a)
(a) (b) (c) (d)
29 29 29 29 Let events, E1 : die A is used when head is
Sol. (b) appeared
This is a problem of Baye's theorem, E 2 : die B is used when tail is appeared
R : red face appears.
1B, 2W 1B, 1W 5B, 4w 32
3R 2R 1R Q (P(E1|R) =
33
Box 1 Box 2 Box 3 1 1
∴ P(E1) = , P(E 2) = ,
2 2
Let A = All 3 Ball's are of different colours. n
P(R|E1) = × × ×…n times =  
2 2 2 2
E1 = Box 1 is Choosen 3 3 3  3
E 2 = Box 2 is Choosen n
Similarly, P(R|E 2) =  
1
E 3 = Box 3 is Choosen  3
A
= 3different colour balls comes from a Q By Baye’s theorem
Ei
P(E1)P(R|E1)
particular box. P(E1|R) =
P(E1)P(R|E1) + P(E 2)P(R|E 2)
1
Now, P (E1) = P (E 2) = P (E 3) = 32 2n
3 ⇒ = ⇒n = 5
33 1 + 2n
 A 1
C × C × C1
2 3
3
P  = 1 6 1 =
 E1  C3 10 8. Suppose X has the following probability
 A  1 C × 1 C × 2C1 1 mass function P(X = 0) = 0.2, P(X = 1) = 0.5,
P  = 1 4 1 = P(X = 2) = 0.3. What E[ X 2 ] = ?
 E2  C3 2
[18 Sep. 2020, Shift-I]
 A  5 C × 4 C × 1 C1 1
P   = 1 10 1 = (a) 2.89 (b) 1.70
 E3  C3 6 (c) 1.10 (d) 1.21
Now by Baye's theorem, Sol. (b)
Probability 113

Given probability distribution is Sol. (*)


The perfect cube numbers are in given set
X 0 1 2
{1, 2, 3, 4, …… , 100} are 1, 8, 27, 64, 125, 216,
P( X ) 0.2 0.5 0.3 343, 512, 729, 1000 and the natural numbers
having odd number of division are perfect
2
∑ pi x i ((0.2) × 0 ) + (0.5 × 12) + (0.3 × 22)
2
square numbers and the perfect square numbers
∴E(X 2) = =
∑ pi 0.2 + 0.5 + 0.3 in given set are 1, 4, 9, 16, 25, 36, 49, 64, 81,
0.5 + 1.2 100, 121, 144, 169, 196, 225, 256, 289, 324, 361,
= = 1.7 400, 441, 484, 529, 576, 625, 676, 729, 784, 841,
1
900, 961.
9. In a game, a person wins 5 rupees for getting So required probability is
a number greater than 4 and loses 1 rupee 10 + 31 − 3 38 19
= = .
otherwise, when a fair die is thrown. A man 1000 1000 500
participated in the game, but decided to quit
11. The binomial distribution whose mean is 9
as and when he gets a number greater than 3
4. Then the expected value (mean value) of and whose standard deviation is is equal to
the amount he wins/loses is 2
[18 Sep. 2020, Shift-I]
[18 Sep. 2020, Shift-I]
12 12
(a)  +  (b)  + 
9 8 19 19 1 3 3 1
(a) (b) (c) (d)
19 19 9 8  4 4  4 4
12 12
(c)  +  (d)  + 
Sol. (c) 1 3 3 1
In a game ……die is thrown. A man 2 2 2 2
participated in the game, he decided to throw a Sol. (b)
die thrice but to quit as and when …… the
We know that mean = np = 9 (given)
amount he win/loses is
3
2 and standard deviation = npq =
The probability to win in a throw is and loss a 2
6
4 1 3
throw is . ∴ q=and p = and n = 12
6 4 4
Now following cases are possible So, required binomial distribution (p + q)n
12
(i) W (ii) L W
=  + 
3 1
(iii) L L W (iv) L L L  4 4
So, the expected value (mean value) of the
amount he wins/loses is 12. Three squares of a chessboard are selected at
random. The probability of selecting two
= 5   + 4  ×  + 3  × × 
2 4 2 4 4 2
 6  6 6  6 6 6 squares of one colour and the other of a
different colour is equal to [18 Sep. 2020, Shift-II]
+ (−3)  × × 
4 4 4
10 15
 6 6 6 (a) (b)
17 19
10 32 96 192 17 16
= + + − (c) (d)
6 36 216 216 23 21
360 + 192 + 96 − 192 456 19
= = = Sol. (d)
216 216 9
Total number of ways of selecting 3 square
10. A number is selected at random from the set = 64C
3
{1, 2, 3, 4, ..., 1000}, then the probability of Total number of ways of selecting 2 square of
getting a number which is a perfect cube or a one colour and other square of different colour
natural having odd number of divisors is = (2 white, 1 black) or (1 white, 2 black)
[18 Sep. 2020, Shift-I] = 32C2 ⋅ 32C1 + 32C1 ⋅ 32C2 = 2⋅ 32C 2 ⋅ 32C1
(a)
481
(b)
483
(c)
479
(d)
477 2.32C 2 ⋅ 32C 1
500 500 500 500 Required Probability =
64C3
114 AP EAMCET Chapterwise Mathematics

32 × 31 1 4
2⋅ × 32 P (E1 ) = 1 − =
2 ×1 16 5 5
= =
64 × 63 × 62 21 E 2 be the event of B speaking the truth.
3 × 2 ×1 80
P (E 2) =
Hence, option (d) is correct. 100
4
13. When a coin is tossed 6 times, the P (E 2) =
5
probability of getting more heads than tails 4 1
is ....... [18 Sep. 2020, Shift-II] P (E 2) = 1 − =
5 5
13 15 9 11
(a) (b) (c) (d) Required, Probability = P (E1). P (E 2) + P (E 2). P (E1)
32 32 32 32
1 1 4 4 1 16 17
Sol. (d) = × + × = + =
5 5 5 5 25 25 25
Total outcomes are 26 = 64 (*) No option is correct.
Required Probability
= P(Getting Four Heads) + P(Getting 5 Heads) 16. A and B are two candidates seeking
+ P(Getting 6 Heads) admission in a college. The probability that A
6! 6! 6! is selected is 0.7 and the probability that
4! 2! 51! ! 6! 15 6 1 11 exactly one of them is selected is 0.6. Find
= + + = + + =
64 64 64 64 64 64 32 the probability that B is selected.
Hence, option (d) is correct. [18 Sep. 2020, Shift-II]
(a) 0.15 (b) 0.20 (c) 0.25 (d) 0.30
14. If P and Q each toss three coins. The
Sol. (b)
probability that both gets same number of
Given, P (A) = 0.7
heads, is [18 Sep. 2020, Shift-II]
Probability of exactly one of them is selected
3 1 3 5
(a) (b) (c) (d) = 0.6
8 9 16 16
P[(A ∩ B) ∪(A ∩B)] = 0.6
Sol. (d) P(A) + P(B) − 2P(A ∩ B) = 0.6
Let p and q represents the probability of getting 0.7 + P(B) − 2P(A) ⋅ P(B) = 0.6
head and tail respectively on tossing a coin.
[Q A, B are independent events]
1
So, p =q = P(B) − 2⋅ (0.7) P(B) = 0.6 − 0.7
2 7
2 P(B) − P(B) = −01 .
 1 
3
So, required probability = ∑  3Cr   
3 5
r=0  2 −2 −1
  P(B) =
6 5 10
=   [12 + 32 + 32 + 12]
1 1
 2 P(B) =
4
20 5 ⇒ P(B) = 0.25
= =
64 16 Hence, option (c) is correct.
Hence, option (d) is correct.
17. If the probability for A to fail in an exam is
15. A speaks truth in 20% of the cases and B in 0.2 and that for B is 0.3, then the probability
80% of the cases. Find the probability that that either A or B fails is ≤ ........
their statements about an incident do not [21 Sep. 2020, Shift-I]
match. [18 Sep. 2020, Shift-II] (a) 0.2 (b) 0.4 (c) 0.5 (d) 0.3
3 7 4 8
(a) (b) (c) (d) Sol. (c)
25 25 25 25
Let, P(A) is the probability for A to fail in an
Sol. (*) exam and P(B) is the probability for B to fail in
Let E1 be the event of A speaking the truth an exam.
20 1 Q The probability that either A or B fails is
∴ P (E1 ) = =
100 5 P(A ∪ B) = P(A) + P(B) − P(A ∩ B)
Probability 115

⇒ P(A ∪ B) ≤ P(A) + P(B) 20. In 3 trials of a binomial distribution, the


⇒ P(A ∪ B) ≤ 0. 2 + 0. 3 probability of 2 successes is 9 times the
{Q P(A) = 0. 2 and P(B) = 0. 3 given} probability of 3 successes. Then the
⇒ P(A ∪ B) ≤ 0. 5 probability of success in each trial is
Hence, option (c) is correct. [21 Sep. 2020, Shift-II]
1 1 1 1
18. Bill and George go target shooting together. (a) (b) (c) (d)
2 3 4 5
Both shoot at a target at the same time.
Suppose Bill hits the target with probability Sol. (c)
0.7 where as George, independently, hits the In a 3 trials of a binomial distribution, the
target with probability 0.4. Given that probability of 2 successes = 9 × probability of 3
exactly one shot hit the target, what is the successes
probability that it was George’s shot? ⇒ 3
C2p 2q = 9(3 C3p 3) {where p + q = 1}
[21 Sep. 2020, Shift-I] ⇒ 3p 2q = 9p 3 ⇒ q = 3p
2 2 1 8 1
(a) (b) (c) (d) ∴p= = probability of success in each trial.
3 9 9 9 4
Sol. (b) Hence, option (c) is correct.
Let the probability, Bill hits the target is
21. A random variable X has the probability
P(B) = 0.7 , and similarly, probability that George
hits the target is P(G) = 0.4
distribution as given below. Let E = [ X | X is
prime number] and F = { X | X < 4 }, then
Q Events B and G are independent
P(E ∪ F) = [21 Sep. 2020, Shift-II]
So, P(B ∩ G) = P(B) P(G) = 0.28
∴The probability that George shot when it is X 1 2 3 4 5 6 7 8
given that exactly one shot hit the target is 2 2 2
P( X ) K 2K K 2K 5K K K 2K
012
. 12 2
= = = 38 39 42 17
0.42 + 012
. 54 9 (a) (b) (c) (d)
64 64 64 64
B G
Sol. (a)
Given probability distribution is
0.42 0.12 X 1 2 3 4 5 6 7 8
2 2
2K 5K
P( X ) K 2K K2 K K 2K

0.7 0.28 0.4


Q ΣP(X) = 1
Hence, option (b) is correct. ⇒ 8K 2 + 7K = 1
19. For any two events, A , B if ⇒ 8K 2 + 7K − 1 = 0
P(A ∪ B) = aP(A ∩ B) + bP(A) + cP(B), then ⇒ 8K 2 + 8K − K − 1 = 0
3a + 2b + 5c = ? [21 Sep. 2020, Shift-II] ⇒ 8K (K + 1) − 1(K + 1) = 0
1
(a) 2 (b) 4 ⇒ K = as K > 0.
(c) 3 (d) 1 8
Q The events (given) E = {x|x is a prime}
Sol. (b)
= {2, 3, 5, 7}
As we know for any two events A and B
and F = {X|X < 4} = {1, 2, 3}
P(A ∪ B) = P(A) + P(B) − P(A ∩ B)
∴ P(E ∪ F) = P(E) + P(F) − P(E ∩ F)
Now, on comparing with the given equation
P(A ∪ B) = aP(A ∩ B) + bP(A) + cP(B) = (2K + K 2 + 5K 2 + K) + (K + 2K + K 2) −(2K + K 2)
Then, we get a = −1, b = 1, c = 1 = 6K 2 + 4K = 6
1 4 38
+ =
∴ 3a + 2b + 5c = 3(−1) + 2 + 5 = 4 64 8 64
Hence, option (b) is correct. Hence, option (a) is correct.
116 AP EAMCET Chapterwise Mathematics

22. Given that a throw of three unbiased dice 1 0  0 1   0 0  0 0


 0 0  0 0  0 1  1 0
shows different faces, what is the probability     
that their total is eight? [22 Sep. 2020, Shift-I]  0 1 1 0  0 1 1 0
1 23  0 1 1 0 1 1 1 1 
(a) (b)     
10 256
13 17 ∴ Total possible number of matrices = 16
(c) (d) Number of matrices whose determinant non zero
36 20
=6
Sol. (a) 6 3
∴Required probability = =
Number of ways to get different faces on the a 16 8
throw of three unbiased dice are Hence, option (b) is correct.
6 × 5× 4
6
C3 = = 20 and the number of ways to
3× 2 25. A bag contains books numbered 1 to 20.
get their total is eight (1, 2, 5), (1, 3, 4) only. Three books are drawn from the bag with
2 1 replacement. The probability that largest
So, the required probability = =
20 10 number on the book is 7. [22 Sep. 2020, Shift-II]
Hence, option (a) is correct. 2 1
(a) (b)
23. Five different books are to be distributed 17 20
3 3 3
(c) 1 −   (d)   −  
among four students randomly. The 7 7 6
probability that each child get atleast one  20   20   20 
book is [22 Sep. 2020, Shift-I]
Sol. (d)
21 15 31 51
(a) (b) (c) (d) The total probability of selecting 3 books
64 64 64 64
and whose page number is less than or equal to
3
Sol. (b)
7 is   .
7
Total number of ways to distribute five different  20 
books among four students randomly is
Total probability of selecting 3 books whose
45 = 210 = 1024 and number of ways to distribute 3
page number is less than 7 is   .
6
five different book among four students such  20 
that each child get atleast one book is
3 3
∴ Requried probability =   −  
5! 7 6
× 4! = 240
1! 1! 1! 2! 3!  20   20 
240 24 × 3 × 5 15 Hence, option (d) is correct.
So, required probability is = =
1024 210 64
26. When six coins are tossed simultaneously,
Hence, option (b) is correct.
the probability of getting at least 4 heads is
24. A determinant is chosen at random from the ……… . [22 Sep. 2020, Shift-II]
set of all determinants of order 2 with 11 15 11 15
(a) (b) (c) (d)
elements 0 or 1 only. The probability that the 64 64 32 32
determinant chosen is non-zero is ………
Sol. (c)
[22 Sep. 2020, Shift-II]
4 3 2 5 Total possible chances = 26 = 64
(a) (b) (c) (d)
8 8 8 8 Required probability
= P (Getting 4 Heads) + P (Getting 5 Heads)
Sol. (b)
{TTHHHH} {THHHHH}
Total possible 2 × 2 matrices with only elements
+ P (Getting 6 Heads)
0 and 1 are
{HHHHHH}
1 1  0 0 1 1   0 0
1 1  0 0  0 0 1 1  6! 6! 6!
     15 6 1 22 11
= 2! 4! + 1! 5! + 6! = + + = =
1 0  0 1  1 1   0 1 64 64 64 64 64 64 64 32
 0 1  1 0 1 0 1 1
     Hence, option (c) is correct.
Probability 117

27. The probability of a non-leap year having 30. Suppose the number of accidents occurring
53 Mondays is ……… [22 Sep. 2020, Shift-II] on a highway in each day follows a Piosson
(a)
2
(b)
1
(c)
5
(d)
6 random variable with parameter 3. Then,
7 7 7 7 what is the probability that no accidents
Sol. (b) occur today? [23 Sep. 2020, Shift-I]
1 −1 1 −1
Number of days in non Leap year = 365 (a) (b) (c) (d)
e3 e3 e9 e9
= 52 × 7 + 1 = 52 weeks + 1 day
The one additional day may be any one of the Sol. (a)
day The probability that number of accident occur
∴ n(A) = 1 today if the number of accidents occuring on a
n(s) = 7 highway in each day follows a poisson random
n(A) 1 variable with parameter 3, is
P(Getting 53 sundays) = = 3° e −3 1
n(S) 7 P (X = 0) = = 3
0! e
Hence, option (b) is correct.
Hence, option (a) is correct.
28. A bag contains 10 identical pens, of which
4 are red and 6 are blue. 3 pens are taken out 31. If two sections of strengths 30 and 45 are
at random one after another. Find formed from 75 students who are admitted
probability that all 3 are blue in a school, then the probability that two
[22 Sep. 2020, Shift-II]
particular students are always together in
6 3 1 3 the same section is [20 April 2019, Shift-I]
(a) (b) (c) (d) 66 19 29 18
10 10 6 6 (a) (b) (c) (d)
185 37 185 37
Sol. (c)
Sol. (b)
Total number of balls = 10
According to given informations, the required
Number of Red balls = 4
probability
Number of Blue balls = 6 73! 73!
Total Number of ways of choosing 3 balls = 10
C3 +
73
C28 + 73C43 28! 45! 43! 30!
= =
Number of ways of choosing 3 blue balls = C3 6 75
C30 75!
6
C3 1 30! 45!
∴Required Probability = =
10
C3 6 1 1
+
45 × 44 30 × 29 (30 × 29) + (44 × 45)
Hence, option (c) is correct. = =
75 × 74 (75 × 74)
29. Three dice are thrown. Given that they have (30 × 29) (45 × 44)
a sum of 8, the probability that one of them (2 × 29) + (44 × 3) 29 + (22 × 3)
is a four is [23 Sep. 2020, Shift-I] = =
(5 × 74) 5 × 37
9 3
(a) (b) 29 + 66 95 19
11 7 = = =
4 3 5 × 37 5 × 37 37
(c) (d)
9 8 Hence, option (b) is correct.
Sol. (b) 32. A bag contains 2n coins out of which n − 1
Chances to get sum on throw of 3 dice are are unfair with heads on both sides and the
(1, 1, 6), (1, 2, 5), (1, 3, 4), (2, 2, 4), (2, 3, 3) and
remaining are fair. One coin is picked from
favourable is (1, 3, 4), (2, 2, 4), so required
probability is
the bag at random and tossed. If the
41
3! probability that head falls in the toss is ,
3! + 56
2! 9 3
= =
3! 3! 3! 21 7 then the number of unfair coins in the bag is
+ 3! + 3! + +
2! 2! 2! [20 April 2019, Shift-I]
Hence, option (b) is correct. (a) 18 (b) 15 (c) 13 (d) 14
118 AP EAMCET Chapterwise Mathematics

Sol. (c) Sol. (c)


According to given informations, The given probability distribution for random
1 variable x
(n − 1) × 1 + (n + 1)
2 = 41 x = xi 1 2 3 4 5 6
2n 56
2n − 2 + n + 1 41 P( X = xi ) 0.2 0.3 0.12 0.1 0.2 0.08
⇒ =
4n 56 and two events A = {x i|x i is a prime number}
⇒ (3n − 1) 14 = 41n = {2, 3, 5}
⇒ 42n − 14 = 41n and B = {x i | x i < 4} = {1, 2, 3}
⇒ n = 14 So, P(A ∪ B) = P(X = 1) + P(X = 2) + P(X = 3)
∴Number of unfair coins in the bag is + P(X = 5)
(n − 1) = 13. = 0.2 + 0.3 + 0.12 + 0.2 = 0.82
Hence, option (c) is correct. Hence, option (c) is correct.
33. Bag A contains 6 Green and 8 Red balls and 35. In a poisson distribution with unit mean,

bag B contains 9 Green and 5 Red balls. A
card is drawn at random from a well ∑| x − x|P(X = x) = (x is the mean of the
x =0
shuffled pack of 52 playing cards. If it is a
distribution) [20 April 2019, Shift-I]
spade, two balls are drawn at random from
1 2 2
bag A, otherwise two balls are drawn at (a) e (b) (c) (d)
e e 3e
random from bag B. If the two balls drawn
are found to be of the same colour, then the Sol. (c)
probability that they are drawn from bag A is For the unit mean, x = 1
[20 April 2019, Shift-I] ∞ ∞ e −1
43 1 48 43 So ∑ |x − x| P(X = x) = ∑ |x − 1|
(a) (b) (c) (d) x =0 x =0 x!
181 4 131 138 11 ∞ x − 1 1 ∞ 1 ∞ 1 
= + ∑ = 1 + ∑ − ∑ 
Sol. (a)  
e  0! x =1 x !  e  x =1 (x − 1)! x =1 x ! 

According to given informations, the required 1 1 2


probability = [1 + e − (e − 1)] = (1 + e − e + 1) =
e e e
1  6 C2 + 8 C2  Hence, option (c) is correct.
 
4  14 C2  36. Each of the two boxes A and B contain
=
1  6 C2 + 8 C2  3  9 C + 5C2  10 chits numbered 1 to 10. If one chit is
 14  +  214 
4 C2  4  C2  drawn at random from each of A and B, then
(6 × 5) + (8 × 7) the probability that the number on the chit
= drawn from A is smaller than the number on
[(6 × 5) + (8 × 7)] + 3[(9 × 8) + (5 × 4)]
the chit drawn from B, is [20 April 2019, Shift-II]
30 + 56 86 86 43 9 9 19 17
= = = = (a) (b) (c) (d)
(30 + 56) + 3(72 + 20) 86 + 276 362 181 10 20 20 20
Hence, option (a) is correct. Sol. (b)
34. A random variable X has the probability According to the given information,
distribution If drawn number from A is 1, then the
favourable drawn number from B are 2, 3, 4,
X = xi 1 2 3 4 5 6 …… 10, are total 9 cases.
Similarly, for 2, there are 8 cases and so on.
P( X = xi ) 0.2 0.3 0.12 0.1 0.2 0.08 9 + 8 + 7 + …+ 2+ 1
∴ Required probability =
If A = { x i / x i is a prime number}, 10 × 10
B = { x i / x i < 4} are two events, then 9×5 9
= =
P(A ∪ B) = [20 April 2019, Shift-I] 10 × 10 20
(a) 0.31 (b) 0.62 (c) 0.82 (d) 0.41 Hence, option (b) is correct.
Probability 119

37. An envelope is known to have come from ⇒ 30C = 1


1
either ‘LONDON’ or ‘CLIFTON’. On the ⇒ C=
postal mark only two successive letters ON 30
are legible. The probability that the envelope Put the value of C, in Eq. (i), we get
comes from LONDON is [20 April 2019, Shift-II] 354
So, σ 2 + µ 2 = = 11.8.
12 5 3 2 30
(a) (b) (c) (d)
17 17 17 5 Hence, option (b) is correct.
Sol. (a) 39. In a book of 250 pages, there are
According to the given information, the number 200 typographical errors. Assuming that the
of two successive letters in word LONDON is 5, number of errors per page follow the Poisson
similarly in word CLIFTON is 6. law, then the probability that a random
Let event E1 is selecting word LONDON and sample of 5 pages will contain no
event E 2 is selecting word CLIFTON. typographical error is [20 April 2019, Shift-II]
1 1
So, P(E1) = and P(E 2) = −
4
2 2 (a) e −4 (b) e 5
16
and the event A is getting successive letter ON. −
(c) e 25 (d) e −1
2 1
Then, P(A / E1) = and P(A / E 2) = .
5 6 Sol. (a)
∴Required probability, According to the given information
 A 200 4
P(E1) ⋅ P   λ= =
 E1  250 5
P  1  =
E
 A  A  A e −λ (λx )
Q P(X = x) =
P(E1) ⋅ P   + P(E 2) ⋅ P   x!
 E1   E2  4

1 2 2 ⇒ P(X = 0) = e 5
×
2 × 6 12
= 2 5 = 5 = = ∴Required probability = (P(X = 0))5 = (e −4 / 5)5
 1 × 2 +  1 × 1  2 + 1 12 + 5 17
    = e −4
 2 5  2 6 5 6
Hence, option (a) is correct.
Hence, option (a) is correct.
40. A and B each select one number at random
38. The probability function of a random
from the distinct numbers 1, 2, 3, …, n and
variable X is given by P(X = k) = ck 2 , where c
the probability that the number selected by
is a constant and k ∈ {0, 1, 2, 3, 4}. If σ 2 is A is less than the number selected by B is
the variance of X and µ is the mean of X , 1009
. Now, the probability that the number
then σ 2 + µ 2 = [20 April 2019, Shift-II] 2019
(a) 3.33 (b) 11.8 selected by B is the number immediately
1 next to the number selected by A is
(c) (d) 354
30 [21 April 2019, Shift-I]
2018 2018 2000 2000
Sol. (b) (a) (b) (c) (d)
2019 (2019)2 (2019) (2019)2
Given probability function P(X = k) = Ck2, where
C is a constant and K ∈ {0, 1, 2, 3, 4} Sol. (b)
Q σ 2 = E (x 2) − µ 2 It is given that, A and B each select one number
at random from the distinct numbers 1, 2, 3, …,
⇒ σ + µ = E(x ) =
2 2 2
Σ(x i2) P(x i) n, then the probability that the number selected
= 0(C(0)2) + 1(C()
1 2) + 4(C(2) ) + 9(C(32)) + 16(C(4)2)
2 by A is less than the number selected by B is
n
C2 n(n − 1) 1009
= C + 16C + 81C + 256C = = (given)
= 354 C …(i) n × n 2(n × n) 2019
Σ P(x i) = 1 n − 1 1009
Q ⇒ =
⇒ C + 4C + 9C + 16C = 1 2n 2019
120 AP EAMCET Chapterwise Mathematics

⇒ 2019n − 2019 = 2018n Sol. (a)


⇒ n = 2019 Probability of green ball P(G)
Now, number of ways selecting numbers from 1 2 1 3 1 1 1 1 1
= × + × + × = + +
the distinct numbers 1, 2, 3, …, 2019, by B is the 2 6 3 6 6 6 6 6 36
number immediately next to the number 6 + 6 + 1 13
selected by A is 2018, because there are 2018 = =
36 36
pairs of consecutive numbers.
2018 2018 Let probability of drawn ball is green comes
So, required probability = =
from bag C is P   , then
C
2019 × 2019 (2019)2  G
41. There are 3 bags A , B and C. Bag A contains 2 P   × P(C)
G 1 1
×
 C
P   =
white and 3 black balls, bag B contains 4 C 1 36 1
= 6 6= × =
white and 2 black balls and Bag C contains 3  G P(G) 13 36 13 13
white and 2 black balls. If a ball is drawn at 36
random from a randomly chosen bag, then 43. If a random variable X has the probability
the probability that the ball drawn is black,
distribution given by P(X = 0) = 3C 3 ,
is [21 April 2019, Shift-I]
2 4 5 1 P(X = 2) = 5C − 10 C 2 and P(X = 4) = 4 C − 1,
(a) (b) (c) (d)
3 9 9 9 then the variance of that distribution is
[21 April 2019, Shift-I]
Sol. (b)
68 22 612 128
Given that A has 2 white, 3 black balls, B has (a) (b) (c) (d)
4 white and 2 black balls and C has 3 white and 9 9 81 81
2 black balls. Sol. (d)
Let event of drawing black ball from bags A, B Given,
1 P(X = 0) = 3C 3
and C =
3
⇒ P(X = 2) = 5C − 10C 2
∴Required probability
= Probability of black ball from bag A and P(X = 4) = 4C − 1
+ Probability of black ball from bag B We know that,
+ Probability of black ball from bag C ΣP(X) = 1
1 3 1 2 1 2 3 2 2 ⇒ 3C 3 + (5C − 10C 2) + (4C − 1) = 1
= × + × + × = + +
3 5 3 6 3 5 15 18 15
⇒ 3C 3 − 10C 2 + 9C − 2 = 0
1 1 2 9 + 5 + 6 20 4
= + + = = = ⇒ (C − 1) (3C 2 − 7C + 2) = 0
5 9 15 45 45 9
⇒ (C − 1) (3C − 1) (C − 2) = 0
42. The following table shows the probability of 1
⇒ C = 1, , 2
selecting the boxes A , B and C and the 3
number of balls of different colours ∴ C=
1
contained in them. [21 April 2019, Shift-I] 3
Now,
Box Number of balls Probability X 0 2 4
White Green Red P( X ) 1 5 1
1 9 9 3
A 1 2 3
2 Hence, variance = ΣX P2 − (ΣX P)2
1 2
=  02 × + 4 × + 16 ×  −  + 
B 2 3 1 1 5 1 10 4
3
 9 9 3  9 3
1
C 3 1 2 2
 66  = 60 + 144 − 484
= 
6 20 16 
+  −  
 9 3  27  27 81
1 6 5 7
(a) (b) (c) (d) 204 484 612 − 484 128
13 13 13 13 = − = =
27 81 81 81
Probability 121

44. A box contains 30 toys of same size in which 46. If A and B throw two dice 100 times each
10 toys are white and all the remaining toys simultaneously, then the probability that
are blue. A toy is drawn at random from the both of them will get even number as the
box and it is replaced in the box after noting total at the same time in all the throws is
down its colour. If 5 toys are drawn in this [21 April 2019, Shift-II]
way, then the probability of getting atmost 100 100
(a)   (b)  
1 1
2 white toys is [21 April 2019, Shift-I]
 6  4
2 2 2 5
(a)   (b)   (c)   (d)  
6 8 7 2 100 100
(c)   (d)  
1 3
 9  9  9  3
2  4
Sol. (b) Sol. (b)
Required probability
Let x be the number of die A.
= No white ball + one white ball
and y be the number of die B.
+ two white ball x + y = even (Given condition)
5 4 1 3 2
=   + 5C1     + 5C2    
2 2 1 2 1 So, x = even, y = even or x = odd, y = odd
 3  3  3  3  3 1
5 Since, probability of both even or odd is .
=   [25 + 5⋅ 24 + 10 ⋅ 23]
1 100
2
 3 Therefore, far 100 time  
1
is required
5  4
192 64 × 3
=   [32 + 80 + 80] = 5 =
1 probability.
 3 3 35
2 47. The probabilities of having a defective toy in
8×8
=  
64 8 1 1 2
= = 2 three cartons, A , B, C are , , respectively.
34
3 ×32  9
3 4 5
45. If two unbiased six-faced dice are thrown If a carton is selected at random and a toy
simultaneously until a sum of either 7 or 11 drawn randomly from it is found to be
occurs, then the probability that 7 comes defective, then the probability that it is
before 11 is [21 April 2019, Shift-II] drawn from carton B is [21 April 2019, Shift-II]
1 3 5 5 15 20
(a) (b) (c) (d) (a) (b)
4 4 9 18 47 47
20 15
Sol. (b) (c) (d)
59 59
Let A be the event of obtained sum of 7 and B be
the event of obtained sum of 11.
∴ n(A) = {(2, 5),(5, 2), (3,4), (4, 3), (1, 1), (6, 1)} = 6
Sol. (d)
6 1 Let E1 , E 2 and E 3 denote the events of selecting
Now, P(A) = = diffective toy in cartoons A, B, C respectively
36 6
and A be the event that a toy selected at random
and n(B) = {(5, 6), (6, 5)} = 2
is defective. Then,
2 1
∴ P(B) = = 1
36 18 P(E1) = P(E 2) = P(E 3) =
3
C = Neither a sum of 11 nor a sum of 7 shows of
36 − (6 + 2) 36 − 8 28 7  A 1  A 1  A 2
∴ P(C) = = = = P  = , P  = , P  =
36 36 36 9  E1  3  E 2  4  E 3  5
Required probability (p) By Bay’s rule, we have required probability
= P  2 
6 6
2
28  28   6   28   6 
3 E
= + × +     +    …  A
36 36 36  36   36   36   36   A
P(E 2) ⋅ P  
   E2 
1 1  1 9 3 =
= = × =  A  A  A
6 1 − 7 6 2 4 P(E1) ⋅ P   + P(E 2) ⋅ P   + P(E 3) ⋅ P  
   E1   E2   E3 
 9
122 AP EAMCET Chapterwise Mathematics

1 1
×
1 ⇒ y 2 + 8 y − 48 = 0
= 3 4 = 12 ⇒ y + 12y − 4 y − 48 = 0
2
1 1 1 1 1 2 1 1 2
× + × + × + +
3 3 3 4 3 5 9 12 15 ⇒ y(y + 12) − 4(y + 12) = 0
1 ⇒ (y + 12)(y − 4) + 0
= 12 =
1 180 15
× = ⇒ y=4 (Q y ≠ −12)
59 12 59 59 ⇒ λ2 = 4
180
⇒ λ=2
48. A die is thrown twice. If getting a number ∴Standard deviation = λ = 2
greater than four on the die is considered a 2
success, then the variance of the probability 50. The probability of occurrence of an event is
5
distribution of the number of successes is
and the probability of non-occurrence of
[21 April 2019, Shift-II] 3
2 1 4 8 another event is . If these events are
(a) (b) (c) (d) 10
3 3 9 9
independent, then the probability that only
Sol. (c) one of the two events occur is
On throwing a dice a number greater than four [22 April 2019, Shift-I]
on the die is 5, 6. 27 27 7 14
2 1 (a) (b) (c) (d)
∴Probability of success (p) = = 25 50 25 25
6 3
1 2 Sol. (b)
∴ q =1 − p =1 − = Given,
3 3
2
Now, variance of probability distribution P(A) =
5
= npq (Q n = 2)
2 3
1 2 4 ∴ P(A)′ = 1 − P(A) = 1 − =
= 2× × = 5 5
3 3 9
3
and P(B)′ =
49. If X is a poisson variate such that 2P(X = 1) 10
= 5 P(X = 5) + 2 P(X = 3), then the standard ∴
3 7
P(B) = 1 − P(B)′ = 1 −
=
deviation of X is [21 April 2019, Shift-II] 10 10
(a) 4 (b) 2 Required probability = P(A)P(B)′ + P(A)′ P(B)
2 3 3 7 6 21 27
(c)
1
(d) 2 = × + × = + =
2 5 10 5 10 50 50 50

Sol. (d) 51. Let α be a root of x 2 + x +1= 0 and suppose


Given, 2P(X = 1) = 5P(X = 5) + 2P(X = 3) that a fair die is thrown 3 times. If a , b and c
e −λ λr are the numbers shown on the die, then the
Now, poission’s variate is p(X = r) =
r! probability that α a + α b + α c = 0, is
−λ −λ 5 −λ 3
2e λ 5e λ 2e λ [22 April 2019, Shift-I]
⇒ = +
1! 5! 3! 2 1 1 2
(a) (b) (c) (d)
λ4 λ2 36 27 72 9
⇒ 2= +
24 3 Sol. (d)
3λ4 + 24λ2 Total numbers of ways for ( a, b, c ) = 6 × 6 × 6
⇒ 2=
72 Here, ω, ω2
⇒ 144 = 3λ4 + 24λ2 ω3 = 1, ⇒ ω4 = ω,
⇒ 48 = λ + 8λ
4 2
ω5 = ω2 and ω6 = 1
⇒ λ + 8λ − 48 = 0
4 2
Since, ω2 , ω are roots of x 2 + x + 1 = 0.
Let λ = y
2
Q ωa + ωb + ωc = 0
Probability 123

Now, suitable values for ( a, b, c ) = 6 × 4 × 2 Also, given mean = 0.2


6× 4× 2 8 2 −a + c 2
∴ Required probability = = = ⇒ = …(i)
6 × 6 × 6 36 9 a + b + c 10
b 2
52. Suppose that a bag A contains n red and 2 and = …(ii)
a + b + c 10
black balls and another bag B contains 2 red c−a 2
and n black balls. One of the two bags is Now, 1+ = +1
a + b + c 10
selected at random and two balls are drawn
a + b + c + c − a 12 b + 2 c 12
from it at a time. When it is known that the ⇒ = ⇒ =
two balls drawn are red, if the probability a+ b +c 10 a + b + c 10
that those two balls drawn are from bag A is  c  2 12
⇒ 2  + = [by Eq. (ii)]
6  a + b + c  10 10
, then n =
7 [22 April 2019, Shift-I] 12 2
⇒ 2 P (X = 1) = − =1
(a) 6 (b) 4 (c) 8 (d) 7 10 10
Sol. (b) ⇒ 2 P(X =1) = 1
Let E1 be the event that the ball is drawn from 1
bag A. E 2 be the event that it is drawn from bag ⇒ P(X = 1) = = 0.5
2
(B) and E that ball is red.
54. There are 800 families with four children in
P  1  =
E 6
Given,
 E 7 each family. Assuming equal chance for
By Baye’s, theorem, every child to be a boy or a girl, the number
of families expected to have children of both
1  n C2  sexes is [22 April 2019, Shift-I]
 
2  n + 2 C2 
P  1  ⇒
E 6
= (a) 700 (b) 100
 A 1 n C2 1 2 C2 7 (c) 500 (d) 300
+
2 n + 2 C2 2 n + 2 C2 Sol. (a)
1  C2 n
n(n − 1)
Given that 4 children in each family .
 
2  n + 2 C2  6 2 6 Event of expected to have children of both sexes
⇒ = ⇒ = 1
1  n
C2 2
C  7 2 + n(n − 1) 7 =
 n+2
+ n + 22  2
2 C2 C2  2
∴ Probability of children of both sexes
n (n − 1) 6 = 1 − [P(all boys) + P (all girls)]
⇒ =
n2 − n + 2 7  1 4 1 
4
= 1 −   +   
⇒ 7(n − n) = 6n − 6n + 12
2 2    2
 2 
⇒ 7n − 7n − 6n2 + 6n = 12 ⇒ n2 − n = 12
2
= 1 −  +  = 1 − =
1 1 1 7
⇒ n2 − n − 12 = 0 ⇒ n2 − 4n + 3n − 12 = 0  16 16  8 8
⇒ n(n − 4) + 3 (n − 4) = 0 Hence, the probability for 800 families
⇒ (n − 4)(n + 3) = 0 7
= × 800 = 700
⇒ n =4 [Q n > 0] 8

53. A random variable X has its range {−1, 0, 1}. 55. If two dice are rolled at a time, then the
If its mean is 0.2 and P(X = 0) = 0 .2, then probability of getting an odd number on the
P(X = 1) = [22 April 2019, Shift-I] first die or a total of 7 on both dice is
(a) 0.1 (b) 0.7 (c) 0.4 (d) 0.5 [22 April 2019, Shift-II]
5 2 1 7
Sol. (d) (a) (b) (c) (d)
9 3 12 12
According to given information,
Sol. (d)
X = −1 X =0 X =1 Two dice are rolled, then n(S) = 36
a b c Let A : getting an odd number on the first die
124 AP EAMCET Chapterwise Mathematics

= (1, 1), (1, 2), (1, 3), (1, 4), (1, 5), (1, 6), (3, 1), E 2 be the event that it will not be 4.
(3, 2), (3, 3), (3, 4), (3, 5), (3, 6), (5, 1), (5, 2), 5  A 4  A 1
(5, 3), (5, 4), (5, 5), (5, 6) P(E 2) = ⇒ P   = and P   =
6  E1  5  E2  5
n(A) = 18
1 4
n(A) 18 ×
P(A) = = 6 5
n(S) 36 ∴Required probability =
1 4 5 1
× + ×
B : getting a total of 7 on both dice. 6 5 6 5
= (1, 6), (2, 5), (3, 4), (4, 3), (5, 2), (6, 1) 4
n(B) = 6 30 4
= =
n(B) 6 4 5 9
P(B) = = +
n(S) 36 30 30
A ∩ B = (1, 6), (3, 4), (5, 2) 58. Let S = {1, 2, 3, K, 50} and Ak be the set of
∴ n(A ∩ B) = 3 multiples of k in S for k ∈ N . If x k is a
n(A ∩ B) 3 number chosen from Ak , then match the
P(A ∩ B) = =
n(S) 36 items of List-I with the items of List-II.
P(A ∪ B) = P(A) + P(B) − P(A ∩ B) [22 April 2019, Shift-II]
18 6 3 21 7
= + − = = List I List II
36 36 36 36 12
1
A. P(x 3 < 30) I.
56. If A and B are two events of a random 2
experiment such that P(A) = 0 .3, P(B) = 0 .4 2
and P(A ∩ B) = 0 .5, then B. P(15 < x4 ≤ 36) II.
3
P(A ∪ B) + P(B| A ∪ B) = [22 April 2019, Shift-II] 2
(a) 0.95 (b) 1.15 (c) 1.25 (d) 0.25 C. P( x7 > 35) III.
7
Sol. (b) 1
D. P( x11 > 11) IV.
Given, P(A) = 0.3 4
∴ P(A) = 1 − P(A) = 1 − 0.3 = 0.7 3
and P(B) = 0.4, P(A ∩ B) = 0.5 V.
4
P(A ∩ B) = 0. 5 ⇒ P(A) − P(A ∩ B) = 0.5 9
⇒ P(A ∩ B) = 0. 2 VI.
16
Now, P(A ∪ B) + P(B | A ∪ B)
P(B ∩ (A ∪ B)) The correct match is
= P(A) + P(B) − P(A ∩ B) + A B C D A B C D
P(A ∪ B)
(a) VI I IV V (b) III I VI V
90 20 / 100 90 1
= + = + (c) II V I IV (d) VI I III V
100 50 + 20 + 10 100 4
100 Sol. (d)
90 + 25 115 Given, S = {1, 2, 3, … , 50}
= = = 115
.
100 100 For option A
Let A3 = {3, 6, 9, 12, 15, 18, 21, 24, 27, 30,
57. A speaks truth in 4 out of 5 times. A die is 33, 36, 39, 42, 45, 48}
tossed. If A reports that there is 4 on the die, 9
Now, P(x 3 < 30) =
then the probability that there was 4 on the 16
die, is [22 April 2019, Shift-II] ∴ A → VI
2 4 1 2 For option B
(a) (b) (c) (d)
3 9 3 9 Let
Sol. (b) A4 = {4, 8, 12, 16, 20, 24, 28, 32, 36, 40, 44, 48}
6 1
Let E1 be the event that 4 will be on the die. Now, P(15 < x 4 ≤ 36) = =
1 12 2
P(E1) =
6 ∴ B→ I
Probability 125

For option C, (20 × 19) + (30 × 29) 38 + 87 125 25


= = = =
Let A7 = {7, 14, 21, 28, 35, 42, 49} (50 × 49) 5 × 49 5 × 49 49
2
Now ,P(x 7 > 35) = Hence, option (a) is correct.
7
∴ C → III 61. If E1 and E 2 are two events of a random
For option D, 1 1
experiment such that P(E1) = , P(E1|E 2) = ,
Let A11 = {11, 22, 33, 44} 8 3
3 1
Now, P(x11 > 11) = P(E 2|E1) = , then match the items of List-I
4 4
∴ D→ V with the items of List-II. [23 April 2019, Shift-I]
Hence, A → VI, B, → I, C → III and D → V
List-I List-II
59. If X is a Poisson variate such that P (X = 2)
3
= 9 P (X = 4) + 90 P(X = 6), then the ordered (A) P(E2 ) I.
16
pair (Mean (X), Variance (X ) =
[22 April 2019, Shift-II] 3
(B) P(E1 ∪ E2 ) II.
(a) (1, 2 ) (b) (1, 1) (c) (2, 1) (d) (2, 2) 29

Sol. (b) 3
(C) P(E1|E2 ) III.
Given, 32
P(X = 2) = 9P(X = 4) + 90P(X = 6) 26
(D) P(E1|E2 ) IV.
e − λ λ2 e − λ λ4 e − λ λ6 29
⇒ =9 + 90 ⇒λ =1
2! 4! 6! 13
∴Mean and variance is (1, 1). V.
32
60. From a group of 50 students, two sections The correct match is
comprising of 20 and 30 students are A B C D A B C D
formed. If Ram and Rahim are two particular (a) I III IV II (b) III I IV V
students among the 50 students, then the (c) III I IV II (d) I II V IV
probability that they both belong to the
same section is [23 April 2019, Shift-I]
Sol. (c)
25 12 13 24 For two given events E1 and E 2, the given
(a) (b) (c) (d) 1 1
49 23 23 49 information are P(E1) = , P(E1|E 2) = and
8 3
Sol. (a) 1
P(E 2|E1) =
Total number of students = 50 4
Number of ways of making groups if both the 1 P(E1 ∩ E 2) 1
Q P(E 2|E1) = ⇒ =
students are in first group 4 P(E1) 4
= 48 C18 × 30 C30 = 48 C18 1
⇒ P(E1 ∩ E 2) =
and number of ways of making groups if both 32
the students are in second group 1
= 48 C28 × 20 C20 = 48 C28 P(E1 ∩ E 2) 32 3
∴ P(E 2) = = =
and total number of ways P(E1|E 2) 1 32
= 50 C20 × 30 C30 = 50
C20 3
∴ P (E1 ∪ E 2) = P(E1) + P(E 2) − P(E1 ∩ E 2)
C18 + 48 C28
48
∴Required probability = 50
1
= +
3

1
=
3
C20 8 32 32 16
48! 48! 1 1 7 29
+ + Q P(E1) = and P(E 2) =
18! 30! 28! 20! 30 × 29 20 × 19 8 32
= =
50! 50 × 49 13
and P(E1 ∩ E 2) = P(E1 ∪ E 2) = 1 − P(E1 ∪ E 2) =
20! 30! 20 × 19 × 30 × 29 16
126 AP EAMCET Chapterwise Mathematics
13
 4C   4 C × 5C1 
P(E1 ∩ E 2) 16 26 ∴ Mean = 0 ×  9 3  + 1 ×  29 
∴ P(E1|E 2) = = =  C3   C3 
P(E 2) 29 29
32  4 C × 5C2   4 C × 5C3 
+ 2 ×  19  + 3 ×  09 
26 3  C3   
and P(E1|E 2) = 1 − P(E1|E 2) = 1 − = C3
29 29
(6 × 5) + (2 × 4 × 10) + (3 × 10)
Hence, option (c) is correct. =
9×8×7
62. A Box B1 contains 3 blue balls and 6 red 3× 2
balls. Another Box B2 contains 8 blue balls 30 + 80 + 30 140 5
= = =
and ‘n’ red balls (n ∈ N). A ball selected at 84 84 3
random from a box is found to be red. If p is Hence, option (a) is correct.
the probability that this red ball drawn is
from box B2 , then [23 April 2019, Shift-I] 64. If the mean of a poisson variate X is 1, then
1 3 3 ∞
(a) ≤ p <
7 5
(b) ≤ p < 1
5 ∑|r − 1| P (X = r) =
3 1 r=0 [23 April 2019, Shift-I]
(c) 0 < p ≤ (d) 0 ≤ p ≤
5 7 2 1
(a) 1 (b) 0 (c) (d)
e e
Sol. (a)
According to given information, the required Sol. (c)
1 n It is given that mean of a poisson variate X is
×
2 n+ 8 λ = 1,
probability =
1 n  1 6  e − λ λr e −1
 ×  +  ×  Q P(X = r) = =
 2 n + 8  2 3 + 6 r! r!
∞ ∞ e −1
(According to Baye’s theorem) ∴ ∑ |r − 1| P(X = r) = ∑ |r − 1|
r =0 r =0 r!
n
−1  1
+ ...........
n+ 8 0 1 2
= =
3n
=
3n
= p (given) =e + + +
n 2 3n + 2n + 16 5n + 16  0! 1! 2! 3! 
+
= e −1 1 + + + ......... = e −1 (1 + 1) =
n+ 8 3 1 2 2
 2! 3!  e
For minimum value of p, n must be 1 (as n ∈ N)
3 1 Hence, option (c) correct.
So, p≥ ⇒p ≥
21 7
65. If a die is rolled three times, then the
For maximum value of p , n → ∞ probability of getting a larger number on its
3 1 3
So, p< ⇒ ≤ p< face than the previous number each time, is
5 7 5
[22 April 2018, Shift-I]
Hence, option (a) is correct. 15 5
(a) (b)
63. From a bag containing 4 white and 5 red 216 54
13 1
balls, if 3 balls are drawn at random, then (c) (d)
the mean of the number of red balls among 216 18
the balls drawn, is [23 April 2019, Shift-I] Sol. (b)
5 20 22 25 Here, total number of possible out come
(a) (b) (c) (d)
3 7 7 9 = 63 = 216
Sol. (a) Now, we can choose any three numbers out of 6
Let the random variable is X, then numbers and we can place them in ascending
order and we can place them in ascending order
X 0 1 2 3 in only one way.
4
4
C 2 × C1 5 4
C1 × C 2
5
So, favourable out come = 6C3 = 20
C3
P( X ) 9
C3 9
C3
4
C 0 × 5C 3
9 20 5
C3 9
C3 Now, required probability = =
216 54
Probability 127
2a 3 4 5 6
66. A man is known to speak the truth 2 out of ⇒a+ + a+ a+ a+ a =1
3 times. If he throws a die and reports that it 2 4 8 16 32
3 1 5 3
is six, then the probability that it is actually ⇒ a+ a+ a+ a+ a+ a =1
five, is [22 April 2018, Shift-I] 4 2 16 16
16a + 16a + 12a + 8a + 5a + 3a
(a)
3
(b)
1
(c)
2
(d)
4 ⇒ =1
8 7 7 5 16
⇒ 60a = 16
Sol. (b) 16 4
Let the events ⇒ a= ⇒ a=
60 15
E1 = six occurs ∴ Required probability
E 2 = six does not occurs 3a 1a 3a
= P(x = 2) + P(x = 3) + P(x = 4) = + +
A = the mean reports that it is six. 4 2 16
We have, 12a + 8a + 3a 23a 23 4 23
= = = × = .
1 5 16 16 16 15 60
P(E1) = ⇒ P(E 2) =
6 6
 A 2  A 1
68. If X is a binomial variate with mean 6 and
Now, P   = , P   = variance 2, then the value of P(5 ≤ X ≤ 7) is
 E1  3  E 2  3
[22 April 2018, Shift-I]
 A 4762 4672 5264 5462
P(E1) × P   (a) (b) (c) (d)
 E1  6561 6561 6561 6651
∴ P  1  =
E
 A  A  A Sol. (b)
P(E1) × P   + P(E 2) × P  
 E1   E2  We have,
1 2 np = 6 and npq = 2
×
6 3 2 ⇒ 6×q=2
= =
1 2 5 1 7 1
× + × ⇒ q=
6 3 6 3 3
2 1 2
The probability that it is actually six = ⇒ p =1 − q =1 − =
7 3 3
2
The probability that it is not actually six ⇒ n× = 6 ⇒ n= 9
2 5 3
=1 − =
7 7 ∴P(5 ≤ x ≤ 7) = P(x = 5) + P(x = 6) + P(x = 7)
1 5 1 5 4 6 3 7 2
The probability that is actually five = × = . = 9C5     + 9C6     + 9C7    
2 1 2 1 2 1
5 7 7  3  3  3  3  3  3
67. If the probability function of a random 1 9× 8× 7× 6 9× 8× 7 9× 8 
 k + 1 = 9
× 32+ × 64 + × 128
variable X is defined by P(X = k) = a  k  3  4 × 3× 2×1 3 × 2 ×1 2 ×1 
 2 
1
for k = 0, 1, 2, 3, 4, 5, then the probability = [9 × 8 × 7 × 2 × 4 + 3 × 8 × 7 × 32 + 9 × 4 × 128]
39
that X takes a prime value is 3 × 8 × 7 × 2 × 4 + 8 × 7 × 32 + 3 × 4 × 128
[22 April 2018, Shift-I] =
13 23 11 19 38
(a) (b) (c) (d) 1344 + 1792 + 1536 4672
20 60 20 60 = = .
6561 6561
Sol. (b)
We have, 69. A and B are two events such that P(A) = 0.58,
k+1 P(B) = 0 .32 and P (A ∩ B) = 0 .28. Then the
P(x = k) = a  k 
 2  probability that neither A nor B occurs is
[22 April 2018, Shift-II]
We know that,
ΣP(x = k) = 1 (a) 0.38 (b) 0.62
(c) 0.72 (d) 0.9
⇒ P(x = 0) + P(x = 1)
+ P(x = 2) + P(x = 3) + P(x = 4) + P(x = 5) = 1 Sol. (a)
128 AP EAMCET Chapterwise Mathematics

P(A ∩ B) = P(A ∪ B) = 1 − P(A ∪ B) 72. If the probability distribution of a random


= 1 − {P(A) + P(B) − P(A ∩ B)} variable X is given by [22 April 2018, Shift-II]
= 1 − {0.58 + 0.32 − 0.28} = 1 − 0.62 = 0.38
X = xi 0 1 2 3
70. Two dice are thrown simultaneously. If A is 1 3
P( X = xi ) 3K K
event of getting the sum of the numbers on 8 8
two dice as greater than or equal to 8 and B 9 3 3
is the event of getting a number less than or (a) 3 (b) (c) (d)
4 2 4
equal to 3 on atleast one of the die. Then,
Sol. (d)
P(B / A) = [22 April 2018, Shift-II]
5 6 7 8 Variance = E(X 2) − (E(X))2
(a) (b) (c) (d)
15 15 15 15 Q Sum of P(x = x i) = 1
1 3 1
Sol. (b) ⇒ + + 3k + k = 1 ⇒ k =
8 8 8
Event and the variance
A = (2, 6), (3, 5), (4, 4), (5, 3), (6, 2), (3, 6), (4, 5), (5, 4)  3  2 1  
=  02 ×  + 12 ×  +  22 ×
1 3
 + 3 × 
(6, 3), (4, 6), (5, 5), (6, 4), (5, 6), (6, 5), (6, 6)   8  8  8  8 
and event A ∩ B = (2, 6), (3, 5), (5, 3), (6, 2), 2
 1 
−  0 ×  + 1 ×  +  2 ×  +  3 × 
1 3 3
(3, 6), (6, 3)
  8  8  8  8  
P(B ∩ A) n(B ∩ A) 6 2
So, P(B / A) = = = or . 2 2
= +
P(A) n(A) 15 5 3 12 9   3 6 3 24  12
+ − + + = − 
 8 8 8   8 8 8  8  8
71. A bag contains 6 balls. If 4 balls are drawn at 2
9 12 − 9 3
= 3 −   = 3 − =
3
a time and all of them are found to be red, =
 2 4 4 4
then the probability that exactly 5 of the
balls in the bag are red is [22 April 2018, Shift-II] 73. A manufacturer of locks knows that 2% of
10 5 1 5 his product is defective. If he sells the locks
(a) (b) (c) (d)
19 21 21 7 in boxes each with 100 locks and guarantees
that not more than 2 locks will be defective
Sol. (b) in a box,then the probability that a box will
Let us define the following events fail to meet the guaranteed quality is
E = drawn balls are red. A = 4 red balls in bag. [22 April 2018, Shift-II]
B = 5 red balls in bag. k 100 − K
C k    
100 1 49
−2
C = 6 red balls in bag. (a) 1 − 5e (b) Σ 100
k =2  50   50 
Then, P(A) = P(B) = P(C)
4 (c) 0.02 (d) 1 − 3e −2
P   = 6 4 =
E C 1
and Sol. (a)
 A C4 15
5
The random variable is the number of defective
P   = 6 4 =
E C 5 2
locks with mean m = 100 × =2
 B C4 15 100
6 So, the required probability
P   = 6 4 =
E C 15
and = 1 − [P(r = 0) + P(r = 1) + P(r = 2)]
 C C4 15
 2° 21 22 
By applying Baye’s theorem, = 1 −  e −2 + e −2 + e −2  = 1 − 5e −2
 0! 1! 2! 
P(B) ⋅ P  
E
 B  B
P  = 74. If a number is chosen at random from out of
 A
P(A) P   + P(B) P   + P(C) P  
E E E
the four digited numbers formed by using
 A  B  C
the digits 0, 1, 2, 3, 4, 6 without repetition,
5 then the probability that it is divisible by 4,
15 5
= = is [23 April 2018, Shift-I]
1 5 15 21
+ + (a)
17
(b)
17
(c)
13
(d)
13
15 15 15 100 50 50 25
Probability 129

Sol. (b) 75. Four cards are drawn at random from a pack
If the number divisible by 4 using digits 0, 1, 2, of playing cards. The probability of getting
3, 4, 6 unit place digit is even. exactly two cards from the same suit and the
Case I. If unit digit is 0, then tens place possible remaining two cards from two different suits
digits is 2, 4, 6 is [23 April 2018, Shift-I]
72 × 169 24 × 169
0 (a) (b)
425 × 49 425 × 49
4 3 3 18 × 169 6 × 169
Possible outcome = 4 × 3 × 3 = 36 (c) (d)
425 × 49 425 × 49
Case II. If unit digit is 2, then tens place digits
are 1 and 3 Sol. (a)
Four cards are drawn at random from pack of
2 playing cards.
3 3 2 Then, required probability
(4 C1 × 13C2) (3 C2 × 13C1 × 13C1)
Possible outcomes = 52
= 3 × 3 × 2 = 18 C4
13 × 12
Case III. If unit digit is 4, then tens digits are 0, 4× × 3 × 13 × 13
2 18 × 4 × 13 × 13
2 and 6. = =
52 × 51 × 50 × 49 17 × 25 × 49
(a) If tens digit 0
4 × 3× 2×1
Possible ways
72 × 169
= 4 × 3 = 12 =
425 × 49
0 4
76. A random variable X has the following
4 3
probability distribution
(b) If tens digit not zero, i.e. (2 and 6)
Possible ways X = xi −2 −1 0 1 2
= 3 × 3 × 2 = 18 P( X = xi ) 1 1 1
k k
6 4 6
4
3 3 2 The variance of this random variable is
So, required ways, if unit digit is 4 = 18 + 12 [23 April 2018, Shift-I]
= 30. 5 3 7
(a) 0 (b) (c) (d)
Case IV. If unit digit is 6, then digit are 1 and 3 24 24 4
Possible ways Sol. (d)
We have,
6
x = xi −2 −1 0 1 2
3 3 2
= 3 × 3 × 2 = 18. p( x = x1 ) 1 1 1
k k
Hence, total number of number which is 6 4 6
divisible by 4 = 36 + 18 + 30 + 18 = 102.
We know that sum of probability = 1.
Now, total number of ways of form four digit
Σp(X = x i) = 1
number using 0, 1, 2, 3, 4, 6 is 1 1 1
⇒ + k + + k + =1
6 4 6
7 5
5 5 4 3 ⇒ 2k = 1 − ⇒ k=
12 24
Total number of 4 digit number
Now, E(X) = Σx i pi
= 5 × 5 × 4 × 3 = 300
= − 2  − 1   + 0   + 1  + 2 
1 5 1 5 1
102 17  6  24   4  24   6
∴ Required probability = =
300 50
E(x) = 0
130 AP EAMCET Chapterwise Mathematics

Now, E(X 2) = Σx i2pi Sol. (b)


= (−2)2   + (−1)2   + 0  
1 5 1 The required probability
 6  24   4 = P(A) P(B) P(C) + P(A) P(B) P(C) + P (A)P(B) P(C)
1 2   + (2)2   =  × ×  +  × ×  +  × × 
5 1 1 2 3 1 1 3 1 2 1
+ ()
 24   6  2 3 4  2 3 4  2 3 4
4 5 5 4 16 + 10 + 16 42 7 1 1 1 11
= + + + = = = = + + = .
6 24 24 6 24 24 4 4 8 12 24
var (x) = E (x)2 − [E(x)]2
79. In a certain recruitment test with multiple
7 7
var (x) = − 0 = choice questions, there are four options to
4 4 answer each question, out of which only one
77. For a binomial variate X with parameters is correct. An intelligent student knows 90%
3 1 correct answers while a weak student known
n = 5 and p = , if α = P(X ≥ 3) and only 20% correct answers. If an intelligent
4 9
β = P(X ≤ 2), then 256(β − α) = student gets the correct answer for a
[23 April 2018, Shift-I]
question, then the probability that he was
guessing it, is [23 April 2018, Shift-II]
(a) − 1 (b) 0 (c) 1 (d) 2
1 1 9 1
(a) (b) (c) (d)
Sol. (c) 37 10 37 2
3
For binomial variate n = 5, p = Sol. (a)
4
3 1 Let the events E1 = he guesses the answer
So, q = 1 − = E 2 = he knows the answer
4 4
1 A = he answers correctly
Now, α = p (x ≥ 3) 9 1
9 Then, P(E 2) = , P(E1) =
4 10 10
1  5  3  1   3  1  5  3 
3 2 5
=  C3     + C4     + C5   
5
P(A | E 2) = 1, P(A | E1) =
1
9 4 4 4 4 4  4
= 10. 5 + 5. 5 + 5  =  5 + 5 + 5 
1 27 81 243 1 270 405 243 So,
9  4 4 4  9  4 4 4  P(E1| A) =
P(E1) ⋅ P(A | E1)
1  918  102 102 P(E1) ⋅ P(A | E1) + P(E 2) ⋅ P(A | E 2)
= = = and B = P(x ≤ 2)
9  45  45 1024 1 1
×
3
0 5
3
1 4
3
2 3
10 4 1 1
= 5C0     + 5C1     + 5C2     = = =
1 1 1
 4  4  4  4  4  4  1 × 1  +  9 × 1 1 + 36 37
   
 10 4   10 
1 3 9 1 + 15 + 90 106
= 1 ⋅ 5 + 5⋅ 5 + 10 ⋅ 5 = =
4 4 4 45 1024 80. A random variable X has the following
Now, 256 (β − α) = 256   106

102  distribution [23 April 2018, Shift-II]

 1024 1024 
4 Values of
= 256 × =1 0 1 2 3 4 5 6 7
1024 X(x)
P( X = x) 0 k 2k. 2k 3k k2 2 k2 7 k2 + k
78. A problem is given to 3 students A, B and C
1 1 1
whose chances of solving it are , and Then P(0<X<6)=
2 3 4 2
(b)   (c)  
9 9 3 1
respectively. Then, the probability of the (a) (d)
10  10   10  10
problem being solved by exactly one of them,
if all the three try independently, is Sol. (b)
[23 April 2018, Shift-II] Since, ΣP(X = x i) = 1
3 11 23 1
(a) (b) (c) (d) ⇒ 0 + K + 2K + 2K + 3K + K 2 + 2K 2
4 24 24 4 + 7K 2 + K = 1
Probability 131

⇒ 9K + 10K 2 = 1 Two balls are drawn one after the other without
1 replacement, then following condition are there
⇒ K = , [Q K > 0] to get at least one of them is white
10
(a) First White, second non-white
So, P(0 < K < 6) = K + 2K + 2K + 3K + K 2
7 14
2 So, required probability = ×
8 1 81  9  21 20
= 8K + K 2 = + = = 
10 100 100  10  (b) First non-white, second white
14 7
81. An unbiased coin is tossed n times. If the So, required probability = ×
probability of getting atleast one head is 21 20
greater than 0.8, then the least value of n is 7 6
(c) Both are white = ×
[23 April 2018, Shift-II] 21 20
(a) 2 (b) 3 (c) 4 (d) 5 ∴ Required probability is
7 14 14 7 7 6 17
Sol. (b) = × + × + × = .
21 20 21 20 21 20 30
The probability of getting at least one head
= 1 − (probability of getting no head) 84. In a manufacturing company, three machines
n n A, B and C respectively produce 20%, 30%
= 1 − n Cn   = 1 −   > 0. 8 (given),
1 1
 2  2 and 50% of the total product. The defective
n products from A, B and C are respectively 5%,
⇒   < ⇒ 2n > 5.
1 1
3% and 2%. If an article produced by the
 2 5
company is selected at random and is found
The least value of n is 3. to be defective, then the probability that it is
82. If two numbers a and b are chosen from the produced by machine B is
set of integers 1 to 39, then the probability [24 April 2018, Shift-I]
10 8
that those numbers satisfy the equation (a) (b)
7 a − 9 b = 0 is [24 April 2018, Shift-I] 29 29
9 11
1 4 4 5 (c) (d)
(a) (b) (c) (d) 29 29
742 743 741 741
Sol. (c)
Sol. (c)
Let events,
Sample space for choosing two numbers from Production by machine A = E1
set of integers 1 to 39 = 39C2
Production by machine B = E 2
Favourable elementary events are Production by Machine C = E 3
(a , b) = (9, 7), (18, 14), (27, 21), (36, 28) and let defective production = E
So, probability to satisfy the equation 7a − 9b = 0 Then, probabilities of production by machine
is,
A, B, C respectively is
4 4
= 39 = . 20 30 50
C2 741 P(E1) = , P(E 2) = , P(E 3) =
100 100 100
83. Two balls are drawn from an urn containing Probability of defective production by,
5
7 white, 6 red and 8 black balls one after the Machine A is = P(E / E1) =
other without replacement. Then the 100
probability that atleast one of them is white,  E 3
by machine B is = P   =
is [24 April 2018, Shift-I]  E 2  100
4 13 11 17  E 2
(a) (b) (c) (d) by machine C is = P   =
9 30 30 30  E 3  100
Sol. (d) An article is selected at random and is found to
Given that urn contain 7 white, 6 red, 8 black be defective. Then, the probability that it is
balls. produce by machine B is
132 AP EAMCET Chapterwise Mathematics

P(E 2 / E) 4 1 1 4 10 5
=
+ + 0+ + = =
P(E 2) ⋅ P(E / E 2) 6 6 6 6 6 3
=
P(E1)⋅ P(E / E1) + P(E 2)⋅ P(E / E 2) + P(E 3)⋅ P(E / E 3) 5 5
∴ var(x) = − (0)2 = .
30 3 3 3
×
= 100 100
20 5 30 3 50 2 86. Given that the probability of a man hitting a
× + × + × 1
100 100 100 100 100 100 target with a gun is . If he fires 8 times,
90 / 10000 90 9 3
= = = . then the probability of his hitting the target
290 / 10000 290 29
atleast twice is [24 April 2018, Shift-I]
85. The variance of the random variable X 8 8
(a) 5   (b) 1 − 5  
2 2
having the following distribution  3  3
[24 April 2018, Shift-I] 8 4
(c)   (d)  
2 3
X=k −2 −1 0 1 2  3  8
1 1 1 1 1 Sol. (b)
P( X = k )
6 6 3 6 6 1
Probability to hit the target =
1 2 4 5 3
(a) (b) (c) (d) 1 2
3 3 3 3 Probability to not hit the target = 1 − =
3 3
Sol. (d) Probability to hit the target atleast twice is
Q var(x) = E(x 2) − [E(x)]2 = P(2) + P(3) + P(4) + K + P(8)
E(x) = ΣPi x i = 1 − [P(0) + P()]
1
1 1 1 1 1   2 8 8 2 1
7
= × (− 2) + (− 1) + × 0 + (1) + (2) = 1 −    + C1 ⋅   ⋅ 
6 6 3 6 6    3 3
2 1 1 2  3 
=− − + 0+ + =0 7
= 1 −   ⋅  + 8 ⋅ 
6 6 6 6 2 2 1
E(x 2) = ΣPi x i2  3  3 3
7 8
= 1 −   ⋅ = 1 − 5⋅   .
1 1 1 1 1 2 10 2
= (− 2)2 + (− 1)2+ × (0)2 + (1)2 + (2)2
6 6 3 6 6  3 3  3
12
Trigonometric Functions
and Identities
 π  π 2. cos 48 ° ⋅ cos 12° =
1. cos 2(x) + cos 2  x +  + cos 2  x −  = [17 Sep. 2020, Shift-I]
 3  3 (3 − 5) (3 + 5 )
(a) (b)
[17 Sep. 2020, Shift-I] 8 4
3 1 −3 −1 (3 + 5 ) (3 + 5 )
(a) (b) (c) (d) (c) (d)
2 2 2 2 2 8
Sol. (a) Sol. (d)
π π 1
cos x + cos  x +  + cos2  x − 
2 2
cos 48°⋅ cos12° =( 2cos 48°⋅ cos12°)
 3  3 2
cos 2x + 1 1 1 1 5 + 1  3 + 5
Q cos 2x = 2cos2 x − 1 ⇒ cos2 x = = [cos 60°+ cos 36°] =  + = 
2 2 2 2 4   8 
1 + cos 2x + 2π   3. If sin (θ) + cos ec (θ) = 2, then
 
 1 + cos 2x    3 
⇒ +  sin 2020 (θ) + cos ec 2020 (θ) = ....
 2  2
 
  [17 Sep. 2020, Shift-I]

1 + cos 2x − 2π   (a) 2 2020 . 2019


(b) 20202
   (c) 2 2019
3 
 (d) 2
+ 
 2 
Sol. (d)
  sinθ + cos ec θ = 2
1
2π  ⇒ sinθ + = 2 ⇒ (sinθ = 1)
⇒ 1 + cos 2x + 1 + cos 2x +
1
 sinθ
2   3
2π   So, sin2020 θ + cos2020 θ = 1 + 1 = 2
+1 + cos 2x − 
 3   4. tan 9 ° − tan 27 ° − tan 63° + tan 81° =
2π  2π  
⇒  3 + cos 2x + cos 2x + 
1 [17 Sep. 2020, Shift-II]
 + cos 2x − 
2   3  3   (a) 1 (b) 2 (c) 3 (d) 4

⇒  3 + cos 2x + 2cos 2x ⋅ cos 
1 Sol. (d)
2  3  tan 9°− tan 27°− tan 63°+ tan 81°
⇒  3 + cos 2x + 2cos 2x  −   = tan 81° + tan 9° − (tan 63°+ tan 27°)
1 1
2   2  = cot 9°+ tan 9° − (cot 27°+ tan 27°)
cos2 9°+ sin2 9° cos2 27°+ sin2 27°
[3 + cos 2x − cos 2x] =  
1 3
⇒ = −
2 2 sin 9° .cos 9° sin 27.cos 27
134 AP EAMCET Chapterwise Mathematics

=
1

1 {(1 + sinθ)2 + cos2 θ} / cos2 θ
=
sin 9° .cos 9° sin 27° .cos 27° (1 + sinθ) / cos2 θ
2 2 sin 54 − sin18
= − =2 1 + sin2 θ + 2sinθ + cos2 θ 2(1 + sinθ)
sin18° sin 54° sin 54.sin18 = =
1 + sinθ 1 + sinθ
4 cos 36.sin18
= =4 =2
cos 36.sin18
1 − cos(2 x) + sin(x)
1 π  π  7. =
 + cot    = a + a + a ,
2
5. If  tan  sin(2 x) + cos(x)
2  24   24   [18 Sep. 2020, Shift-I]
(a) sin( x) (b) cos( x)
then a = [17 Sep. 2020, Shift-II]
(c) tan( x) (d) cosec ( x)
(a) 3 (b) 2 (c) 1 (d) 4
Sol. (c)
Sol. (b)
1 − cos 2x + sin x 2sin2 x + sin x
π =
To get cot , we proceed as follows sin 2x + cos x 2sin x cos x + cos x
24
π sin x (2sin x + 1)
2cos2 = tan x.
π 24 cos x (2sin x + 1)
cot =
24 2sin π ⋅ cos π
24 24 8. If 4 cos x + 3 sin x = 5, then find the value of
π 1 + cos π − π tan x = [18 Sep. 2020, Shift-I]
1 + cos  
12 = 4 6 3 4
= (a) (b)
π  π π 4 3
sin 1 + sin −  −3 −4
12 4 6 (c) (d)
4 3
1 3 1 1
1+ × + ×
2 2 2 2 Sol. (a)
=
1 3 1 1 Given, 4 cos x + 3 sin x = 5
× − ×
2 2 2 2 ⇒ 4 + 3 tan x = 5 sec x
= 2+ 2 + 3 + 6 On squaring both sides, we get
π 16 + 9 tan2 x + 24 tan x = 25 sec2 x
and tan = − 2+ 2− 3 + 6
24 ⇒ 16 tan2 x − 24 tan x + 9 = 0
π π
So,  tan + cot  = 2 + 6
1 ⇒ (4 tan x − 3)2 = 0
2 24 24  3
⇒ tan x =
= 22 + 2 + 2= a2 + a + a (given) 4
So, a=2 9. For A, B and C, if A + B + C = 0, then
6. If sec θ = m, tan θ = n, then sin(2 A) + sin(2 B) + sin(2C) is equal to
1  1  [18 Sep. 2020, Shift-I]
m+n+ =
m  n
(a) 4 sin ( A) ⋅ sin (B) ⋅ sin (C )
m+ [17 Sep. 2020, Shift-II] (b) 2 sin ( A) ⋅ sin (B) ⋅ sin(C )
(a) 1 (b) 2 (c) − 1 (d) 3 (c) − 4 sin ( A) ⋅ sin (B) ⋅ sin (C )
Sol. (b) (d) − 2 sin ( A) ⋅ sin (B) ⋅ sin (C )
Given, secθ = m, tanθ = n Sol. (c)
1 1  For A + B + C = 0
∴ m + n + 
m m + n sin 2A + sin 2B + sin 2C
= 2sin (A + B) cos (A − B) + 2sin C cos C
1  1 
=  secθ + tanθ +  = − 2sin C cos (A − B) + 2 sin C cos (A + B)
secθ  secθ + tanθ 
= − 2sin C [cos (A − B) − cos (A + B)]
(secθ + tanθ)2 + 1 = −2 sin C [2sin A sin B]
=
secθ(secθ + tanθ) = −4 sin A sin B sin C.
Trigonometric Functions and Identities 135

π π
10. The value of (sin 210 °) (sin 585°) is = sin  2π −  + sec  4 π + 
 3  3
[18 Sep. 2020, Shift-II]
1 −1 1 −1 π π 3 3
(a) (b) (c) (d) = − sin+ sec = − + 2 = 2−
2 2 2 2 3 3 3 3 2 2
Hence, option (a) is correct.
Sol. (a)
sin 210°⋅ sin 585° = sin(180°+30°).sin(540°+45°) 14. If x ≠ 0, then
−1 −1 1 π   3π 
= − sin 30° (− sin 45°) = × = sin(π + x) cos  + x  tan  − x  cot(2 π − x)
2 2 2 2 2   2 
=
Hence, option (a) is correct.  3π 
sin(2 π − x) cos(2 π + x) cosec(− x) sin  + x 
11. Geometric mean of tan 1°, tan2°, ........,  2 
tan 89 ° is [18 Sep. 2020, Shift-II] [21 Sep. 2020, Shift-I]

(a)
1
(b) 1 (c)
1
(d) 3 (a) 0 (b) − 1 (c) 1 (d) 2
89 3 Sol. (c)
π 3π
Sol. (d) sin(π + x) cos + x  tan − x  cot(2π − x)
tan 1°⋅ tan 2° ... tan 45°⋅ tan 46° ... tan 88°⋅ tan 89° 2   2 
,
tan 1°⋅ tan 2° ... tan 44°⋅ tan 45° tan(90 – 44°) 3π
sin(2π − x) cos(2π + x)cosec(− x)sin + x 
... tan(90° –1°)  2 
tan 1°⋅ tan 2° ... tan 44° tan 45° Cot 44°⋅ Cot 43° (x ≠ 0)
(− sin x) (− sin x) (cot x) (− cot x)
...Cot 1° = =1
(− sin x) (cos x) (− cosec x) (− cos x)
1 1 1
tan 1°⋅ tan 2° ... tan 44°1 ... Hence, option (c) is correct.
tan 44° tan 43° tan 1°
=1 15. If α, β and γ are angles that satisfy the
Hence, option (b) is correct. following conditions, find the value of xyz.
[21 Sep. 2020, Shift-I]
3 sin (θ) + cos(θ)
12. = 1. tan(α) + tan(β) + tan(γ)
 π
sin θ +  = tan(α) ⋅ tan(β) ⋅ tan(γ)
 6 [18 Sep. 2020, Shift-II] 2. x = cos (α) + i sin (α)
(a) − 2 (b) 1 (c) 2 (d) − 1 3. y = cos (β) + i sin (β)
Sol. (c) 4. z = cos (γ) + i sin (γ)
3 sinθ + cosθ 3 sinθ + cosθ (a) 1, but not − 1 (b) − 1, but not 1
=
 π π π (c) ± 1
sin  θ +  sin θ cos + cosθ sin (d) 0
 6 6 6
Sol. (c)
3 sinθ + cosθ
= =2 It is given that,
3 1 tanα + tanβ + tan γ = tanα tanβ tan γ
sinθ + cosθ
2 2 ⇒ tanα + tanβ = − tan γ(1 − tanα tanβ)
Hence, option (c) is correct. tanα + tanβ
⇒ = tan(− γ)
 5π  13 π  1 − tanα tanβ
13. sin   + sec   = ⇒ tan(α + β) = tan(− γ) ⇒α + β = nπ − γ , n ∈ I
 3  3  [21 Sep. 2020, Shift-I]
3 3 ⇒ α + β + γ = nπ, n ∈ I
(a) 2 − (b) 2 + and x = cosα + i sinα , y = cosβ + i sinβ,
2 2
1 1 z = cos γ + i sin γ
(c) 3 + (d) 3 −
2 2 ∴ xyz = e iα . e iβ . e iγ

Sol. (a) = e i( α + β + γ)
= e inπ = cos nπ = 1 or − 1
5π 13π  ∴ xyz = ± 1
sin   + sec  
 3  3  Hence, option (c) is correct.
136 AP EAMCET Chapterwise Mathematics

 23 π   13 π  19. If cos(x) + cos 2(x) = 1, then sin 2(x) + sin 4(x) is


16. tan  −  − cot θ −  =
 3   3  equal to [21 Sep. 2020, Shift-II]
[21 Sep. 2020, Shift-I] (a) 0 (b) 1
π (c) −1
(a) 3 + cot θ (b) 3 − tan  + θ (d) 2
6 
Sol. (b)
π
(c) 3 + tan θ (d) 3 + cot  − θ
 It is given that,
3 
cos x + cos2 x = 1 ⇒ cos x = sin2 x
Sol. (d) ∴ sin2 x + sin4 x = cos x + cos2 x = 1
23π  13π 
tan − 
 − cot  θ −  Hence, option (b) is correct.
 3   3 
−3
2π   π  20. If θ lies in third quadrant and cosθ = find
= − tan 7 π +  + cot  4 π +  − θ  5
 3  3 
the value of tanθ. [21 Sep. 2020, Shift-II]
2π π −2
= − tan + cot  − θ (a)
2
(b)
3 3  3 3
π π −4 4
= − tan π −  + cot  − θ
  (c) (d)
 3 3  3 3
π π π Sol. (d)
= tan + cot  − θ = 3 + cot  − θ
3 3  3  It is given that θ lies in third quadrant and
3
Hence, option (d) is correct. cosθ = −
5
17. The possible value of
sin 6 (θ) + cos 6 (θ) − 3 cos 4 (θ) is
[21 Sep. 2020, Shift-I] 4 5
(a) 2 (b) − 2
(c) − 3 (d) 3 θ
3
Sol. (b) 4
The sin6 θ + cos6 θ − 3cos4 θ So, tanθ =
3
= (sin2 θ + cos2 θ) (sin4 θ + cos4 θ − sin2 θ cos2 θ) Hence, option (d) is correct.
− 3cos4 θ 21. Find the value of cosec 750 °−2(cot 765°)
= 1 × [(sin2 θ + cos2 θ)2 − 3sin2 θ cos2 θ] − 3cos4 θ [21 Sep. 2020, Shift-II]
= 1 − 3 (1 − cos2 θ) cos2 θ − 3cos4 θ (a) 0 (b) 1
= 1 − 3 + 3cos4 θ − 3cos4 θ = − 2 (c) 2 (d) −1

Hence, option (b) is correct. Sol. (a)


cosec750°−2cot 765°
18. Let f (x) = cos(ax) + sin(x) be periodic. Then a = cosec(720°+30°) − 2cot(720°+ 45°)
must be [21 Sep. 2020, Shift-II] = cosec30°−2cot 45° = 2 − (2 × 1) = 2 − 2 = 0
(a) Irrational Hence, option (a) is correct.
(b) Rational
 x m
(c) Positive real number 22. If tan   = , then the value of m
(d) Negative real number correct.  2 n
Sol. (b) sin (x) + ncos(x) is equal to [22 Sep. 2020, Shift-I]
If function f (x) = cos(ax) + sin(x) be periodic, (a) m (b) −m
2π (c) −n (d) n
then LCM of and 2π must exist and it is only
|a| Sol. (d)
possible if a is a rational number. x m
It is given that tan = , so
Hence, option (b) is correct. 2 n
Trigonometric Functions and Identities 137

 2 tan x  x π
  1 − tan2 ⇒ 0< x <
m sin(x) + n cos(x) = m 2 +n 2 4
x x π π
 1 + tan2 

1 + tan2 ∴ 0 < x < as x ∈  0, 
2 2 4  2
m 2
2 
m 1− Hence, option (c) is correct.
 2
 n n 
=m + n 25. The minimum and maximum values of
m2  m2 
1+ 2 1+  2  π  π
n n  cos  x +  + 2 2 sin  x +  are respectively
 3  3
2 m2n n(n2 − m2)
= + [22 Sep. 2020, Shift-II]
m + n
2 2
m2 + n2 (a) −(2 3 − 1) and 2 3 − 1
n(2 m2 + n2 − m2) (b) − (1 + 2 2 ) and 1 + 2 2
= =n
m2 + n2 (c) −3 and 3
(d) −2 and 2
1
23. If sin A + sin B = and cos A + cos B = 1, then Sol. (c)
2
Let
 A − B
sin   equals π π
 2  f (x) = cos  x +  + 2 2 sin  x + 
[22 Sep. 2020, Shift-I]  3  3
13 11 π π
(a) ± (b) ± =  cos x . cos − sin x.sin 
4 4  3 3
7 17 π π
(c) ± (d) ± + 2 2  sin x cos + cos x sin 

4 4  3 3
Sol. (b)  1 3
1 =  cos x. − sin x 
Given sin A + sin B = and cos A + cos B = 1,  2 2
2
 1 3
on square and add the given relations, we get + 2 2 sin x. + cos x. 
1  2 2
2 + 2 (cos A cos B + sin Asin B) = + 1
4 cos x 3sin x
= − + 2sin x + 6 cos x
3 3 2 2
⇒ 2 cos(A − B) = − ⇒ cos(A − B) = −
4 8  3
f (x) = cos x  + 6  + sin x  2 −
1
A − B 
⇒ 1 − 2 sin2 
3 2 
 =−  2
 2  8
A − B  11 A−B 1 3
⇒ 2 sin2 
11 ∴ a= + 6 and b = 2−
 = ⇒ sin =± 2 2
 2  8 2 4
2 2
2 tan(x) 1 +  3
24. For the value of to be positive, a2 + b2 =  6  +  2 −  = 9
1 − tan 2 (x) 2   2 
 π
find values of x, such that x ∈ 0 ,  a2 + b2 = 3
 2
[22 Sep. 2020, Shift-II] ∴ Maximum value of f (x) = a2 + b2 = 3
π π π π
(a)  0,  (b)  0,  (c)  0,  (d)  0,  Minimum value of f (x) = − a2 + b2 = − 3
 3  6  4  8
Hence, option (c) is correct.
Sol. (c)
26. If secθ + tan θ = 2 / 3, then in which quadrant
Given,
2 tan x
>0
does θ lie in? [22 Sep. 2020, Shift-II]
1 − tan2 x (a) I (b) II (c) III (d) IV
tan 2x > 0 Sol. (d)
π 2
Since, 0 < 2x < [Q tan 2x is the in Q1 ] sec θ + tan θ = …(i)
2 3
138 AP EAMCET Chapterwise Mathematics

3
⇒ sec θ − tan θ = …(ii) 29. The difference between the greatest and
2 least values of the function f (x) = − x + sin 2 x
Eqs. (i) + (ii)  π π
13 13 on − ,  is
⇒ 2sec θ = ⇒ secθ =  2 2 [23 Sep. 2020, Shift-I]
6 12
Eqs. (i) −(ii) 3+ 2 3+ 2 π
(a) (b) +
5 5 2 2 6
⇒ 2tanθ = − ⇒ tan θ = − 3 π 3 π
6 12 (c) + (d) −
∴ sec θ is positive and tan θ is negative 2 3 2 6
∴ θ lies in IV quadrant Sol. (*)
π π
Given, f (x) = − x + sin 2x, x ∈  − , 
Hence, option (d) is correct.

27. If cos(θ1) + cos(θ 2) + cos(θ 3) + cos(θ 4) = − 4,  2 2 


π π
then the value of So, f ′(x) = −1 + 2cos 2x, x ∈  − , 
 2 2
θ  θ  θ  θ 
cot  1  + cot  2  + cot  3  + cot  4  = for the maxima and minima f ′(x) = 0
2 2 2 2
1 π π
[23 Sep. 2020, Shift-I] ⇒ cos 2x = ⇒ 2x = − ,
2 3 3
(a) 4 (b) 1 (c) 2 (d) 0
π π
Sol. (d) ⇒ x=− ,
6 6
Given, cos(θ1) + cos(θ2) + cos(θ3) + cos(θ4) = − 4
−π 3 π 3
If θ1 = θ2 = θ3 = θ4 = 180° above Equation is true. ∴ f (x) max =
+ and f (x) min = −
6 2 6 2
θ θ θ θ
cot  1  + cot  2  + cot  3  +  4  π π
 2  2  2  2 and f(− π /2) = , f(π /2) = −
2 2
180°   180°  + cot  180°  + cot  180° 
= cot   + cot       ∴ The difference of greatest and least values, as
 2   2   2   2 
= cot 90° + cot 90° + cot 90° + cot 90° = 0 π π 3 π 3 π

< − <− + < is π
Hence, option (d) is correct. 2 6 2 6 2 2
1 (*) No option is correct.
28. If cosec θ + cot θ = , then θ lies in the
3 30. Let M and m respectively denote the
[23 Sep. 2020, Shift-I] maximum and the minimum values of
(a) 1st quadrant (b) 2nd quadrant [ f (θ)]2 , where f (θ) = a 2 cos 2 θ + b2 sin 2 θ
(c) 3rd quadrant (d) 4th quadrant
+ a 2 sin 2 θ + b2 cos 2 θ. Then M − m =
Sol. (b)
1 [20 April 2019, Shift-I]
Given, cosecθ + cot θ = …(i)
3 (a) a + b
2 2
(b) (a − b )2
(c) a2 b 2 (d) (a + b )2
1 1
⇒ cosecθ − cot θ = = Sol. (b)
cosecθ + cot θ 1/ 3
If
cosecθ − cot θ = 3 …(ii)
10 f (θ) = a 2 cos2 θ + b 2 sin2 θ + a 2 sin2 θ + b 2 cos2 θ
Eqs. (i) + (ii) ⇒ 2cosec θ =
3 ∴[ f (θ)]2 = a 2 cos2 θ + b 2 sin2 θ + a 2 sin2 θ + b 2 cos2 θ
5
cosec θ =
3 + 2 (a 2 cos2 θ + b 2 sin2 θ) (a 2 sin2 θ + b 2 cos2 θ)
1 1
Eqs. (i) − (ii) ⇒ 2cotθ = − 3 Q[ f (θ)]2 will be maximum, if sin2 θ = cos2 θ =
3 2
−8 −4 and will be minimum, if either sin2 θ = 0 or
2cotθ = ⇒ cotθ =
3 3 cos2 θ = 0.
∴Cosec θ is +ve and cotθ is −ve 2
 a2 b2 
∴ θ ∈ θ2 ∴ M = (a 2 + b 2) + 2  +  = 2(a + b )
2 2

Hence, option (b) is correct.  2 2


Trigonometric Functions and Identities 139

and m = (a 2 + b 2) + 2 a 2b 2 = a 2 + b 2 + 2ab cos2 5° − cos215° − sin215° + sin2 35°


∴ M − m = 2(a 2 + b 2) − [a 2 + b 2 + 2ab] + cos15° sin 15° − cos 5° sin 35°
= a 2 + b 2 − 2ab = (a − b)2 = cos 5° (cos 5° − sin 35°) − cos15° (cos15° − sin15°)
Hence, option (b) is correct. + sin2 35° − sin215°
−60 7 = cos 5° (cos 5° − cos 55°) − cos15° (cos15° − cos 75°)
31. If cos A = and tan B = − and neither A + sin 50° sin 20°
61 24
nor B is the second quadrant, then the angle = cos 5° (2sin 30° sin 25°) − cos15° (2sin 45° sin 30°)
B + sin 50° sin 20°
A + lies in the quadrant cos15°
2 [20 April 2019, Shift-I] =− + cos 5° sin 25° + sin 50° sin 20°
2
(a) 1 (b) 2 (c) 3 (d) 4
cos15° 1 1
=− + (2cos 5° cos 65°) + (2sin 50° sin 20°)
Sol. (a) 2 2 2
60 −7 cos15° 1
It is given that cos A = − and tan B = and =− + [cos 70° + cos 60° + cos 30°− cos 70° ]
61 24 2 2
neither A nor B is in the second quadrant, then A cos15° 1
and B will be in third and fourth quadrant =− + [2cos 45° cos15° ]
B 2 2
respectively. So, will be in second quadrant. cos15° cos15°
2 =− + =0
B 2 2
2tan
7 2 =− 7 Hence, option (a) is correct.
Now, tan B = − ⇒
24 B 24
1 − tan2 33. If sinα = p, then the quadratic equation
2
B B α α
⇒ 7 tan2 − 48 tan − 7 = 0 whose roots are tan , cot is
2 2 2 2
⇒  7 tan + 1  tan − 7 = 0
B B [20 April 2019, Shift-Ii]
 2   2  (a) px2 − 2 x + p = 0 (b) px2 + 2 x + p = 0
= − Q ∈ IInd quadrant  (c) px2 + x + p = 0 (d) px2 − x + p = 0
B 1 B
⇒ tan
2 7  2 
B 11 1 Sol. (a)
tan A + tan − α
Q tan  A +  =
B 2 = 60 7 2tan
 2  1 − tan A tan B 1 + 11 Since, sinα = p ⇒ 2 =p …(i)
α
2 420 1 + tan2
60 11 2
[Q cos A = − and A∈IIIrd Quadrant∴tan A = ] α α
61 60 and sum of roots = tan + cot
77 − 60 17 2 2
= = α
420 + 11 431 tan2 +1
2 2
3π  = = [from Eq. (i)]
Q A ∈IIIrd quadrant and ∈ 
B α
, π  and tan p
2  4 
2
tan  A +  is positive.
B α α
 2 and product of roots = tan × cot = 1
2 2
 B
∴  A +  ∈ Ist quadrant. So, equation required quadratic equation
 2 2
x 2 − x + 1 = 0 ⇒ px 2 − 2x + p = 0
Hence, option(a) is correct. p
32. cos 2 5° − cos 215° − sin 215° + sin 2 35° Hence, option (a) is correct.
+ cos15 ° sin15 ° − cos 5 ° sin 35 ° = 34. cos 66 ° + sin 84 ° = [20 April 2019, Shift-II]
[20 April 2019, Shift-I]
1 1
3 (a) ( 3 + 5 ) (b) 5 ( 3 + 1)
(a) 0 (b) 1 (c) (d) 2 4 4
2 1 1
(c) ( 3 + 1) ( 5 + 1) (d) 3 ( 5 + 1)
Sol. (a) 4 4
140 AP EAMCET Chapterwise Mathematics

Sol. (d) π 2π 3π 4π
sin4 + sin4 + sin4 + sin4
cos 66° + sin 84° = cos 66° + cos 6° 8 8 8 8
= 2cos 36° cos 30° 4 5π 6π 7π
+ sin + sin4 + sin4
Qcos C + cos D = 2cos C + D cos C − D  8 8 8
 2 2  = sin4 π + sin4 3π + sin4 5π + sin4 7π 
( 5 + 1)  5 + 1  8 8 8 8 
= 3 cos 36° = 3 Q cos 36° =  4 2π 4 4π 6π 
4  4  + sin

+ sin + sin4
8 8 8 
Hence, option (d) is correct.
π
sin4 + sin4 3π π π
+ cos4  − 
5
= 
35. cosec 48 ° + cosec 96 ° + cosec 192°  8 8 2 8
+ cosec 384 ° = π 7π    4 π π 3π 
+ cos4  −  + sin + sin4 + sin4
[21 April 2019, Shift-I] 2 8    4 2 4 
(a) − 2 (b) − 1
3
= sin4 π + sin4 3π + cos4  − π  + cos4  − 3π  
(c) 0 (d)
    
2 8 8  8  8  
π π π π
Sol. (c) + sin4 + sin4 + sin4  +  
 4 2 2 4  
cosec 48° + cosec 96° + cosec 192° + cosec 384°
= cosec (90° − 42°) + cosec 96° + cosec (270° − 78°) = sin4 π + sin4 3π + cos4 π + cos4 3π 
+ cosec (360° + 24°)  8 8 8 8 
 4 π 4 π π
= sec 42° + cosec 96° + − sec 78° + cosec 24° + sin + sin + cos4 
[Q cosec (90° − θ) = sec θ, cosec (270° − θ)= − secθ]  4 2 4 
π π 3π 3π  
=
1
+
1

1
+
1 =   sin4 + cos4  +  sin4 + cos4 
  8 8  8 8  
cos 42° sin 96° cos 78° sin 24°
 1 4 4

+    + () 1 4 +   
1 1 1 1 1
= − + +
cos 42° cos 78° sin 96° sin 24°   2   2 

(cos 78° − cos 42°) (sin 24° + sin 96°)   π π
2   3π 2 3π  
= + = 1 − 2sin 2
cos  + 1 − 2sin 2
cos 
cos 42° × cos 78° sin 96° × sin 24°   8 8  8 8  
+  +1+ 
1 1
− 2 sin 60° × sin18°
=  4 4 
cos(60° − 18°) cos(60° + 18°)
π π 1 3π 3π 
=  2 − 4 sin 2 cos 2 − 4 sin 2
1
2sin 60° × cos 36° cos 2
+  2 8 8 2 8 8 
sin(60° + 36°) sin(60° − 36°)
+  + 1
1
− 2sin 60° × sin18° 2sin 60° × cos 36°  2 
= +
cos2 (60°) − sin2 (18°) sin2 60 − sin2 36° π 3π
=  2 −  sin2  −  sin2   +
1 1 3
 5 −1  5 + 1  2  4  2  4   2
− 2 × 3/ 2 ×   2× 3/ 2 ×  
 4   4  1 1  1 1  3 1 1 3
= + = 2−   −   + = 2− − + =3
 5 − 1) 
2
 10 − 2 5 
2 
2 2  
2 2  2 4 4 2
(1/2)2 −   ( 3/2)2 −  
 4   
 4  π  π 
37. If x : y : z = tan  + α :tan  + β :
= − 2 3 + 2 3= 0 15  15 
π 2π 3π 4π π  z+x x+y
36. sin 4 + sin 4 + sin 4 + sin 4 tan  + γ  , then sin 2 (γ − α) +
8 8 8 8 15  z−x x−y
4 5π 4 6π 4 7π y+z 2
+ sin + sin + sin = sin 2 (α − β) + sin (β − γ) =
8 8 8 y−z
[21 April 2019, Shift-I] [21 April 2019, Shift-I]
3 5 7
(a) (b) (c) 3 (d) (a) sin2 θ (b) cos 2 θ
2 2 2 (c) 0 (d) 1
Sol. (c) Sol. (c)
Trigonometric Functions and Identities 141

Given (a 2 sin2α θ cos2α + 2 θ + a 2 sin2α + 2cos2α θ)m


=
π π π (a 2 sin2α + 1 θ cos2α + 1)n
x : y : z = tan + α  : tan + β  : tan + γ 
 15   15   15 
a 2m (sin2α θ cos2α θ)m (sin2 θ + cos2 θ)m
π =
∴ x = k tan + α  , a 2n (sin2α + 1 θ cos2α + 1 θ)n
 15 
π a 2m − 2n (sin2α θ cos2α θ)m − n
y = k tan + β  , =
 15  (sinθ cosθ)n
 π  = a 2m − 2n (sinθ cosθ)2α( m − n) − n
z = k tan + γ 
 15 
For independent value of θ
z+ x
Now, sin (γ − α)
2
2α(m − n) − n = 0 ⇒ 2α(m − n) = n
z− x
⇒ 2mα − 2nα = n ⇒ 2mα = n(2α + 1)
tan(12° + γ) + tan(12° + α)
= ⋅ sin2(γ − α)
tan(12° + γ) − tan(12° + α) 39. Assertion (A) : If
sin {24° + (γ + α)}  π θ
= ⋅ sin2(γ − α) 4 sin 4 θ + sin 2 2θ + 4 cos 2  −  = 2, then θ
sin(γ − α)  4 2
= [sin 24° cos(γ + α) + cos 24° sin(γ + α)] × sin(γ − α) lies in 3rd quadrant of 4th quadrant.
= sin 24° [cos(γ + α)sin(γ − α)] Reason : (R) sin 2 θ = sin θ
+ cos 24° [sin(γ + α)sin(γ − α)]
[21 April 2019, Shift-II]
sin 24° cos 24°
= (sin γ − sin2 α) −
2
(cos2 γ − cos2 α) (a) Both (A) and (R) are true and (R) is the correct
2 2
explanation of (A)
…(i)
(b) Both (A) and (R) are true but (R) is not the correct
x+ y 2 sin 24°
Similarly, sin (α − β) = (sin α − sin2 β)
2
explanation of (A)
x−y 2 (c) (A) is true but (R) is false
cos 24°
− (cos2 α − cos2 β) … (ii) (d) (A) is false but (R) is true
2
y+ z sin 24°
Sol. (c)
and sin2(β − γ) = (sin2 β − sin2 γ) Given,
y−z 2 π θ
cos 24° 4sin4 θ + sin2 2θ + 4 cos2  −  = 2
− (cos2 β − cos2 γ) … (iii)  4 2
2 π θ
By adding Eqs. (i), (ii) and (iii), we get = 4sin4 θ + 4sin2 θ cos2 θ + 21 + cos 2 −   = 2
  4 2 
z+ x x+ y
sin2(γ − α) + sin2(α − β) = 4sin2 θ(sin2 θ + cos2 θ) + 21
( + sinθ) = 2
z− x x− y
y+ z 2 = 4sin2 θ + 2 + 2sinθ = 2
+ sin (β − γ) = 0
y−z
⇒|2sinθ| + 2sinθ = 0 ⇒sinθ < 0
38. Let x = a sin α θ cos α + 1 θ, y = a sin α + 1cos α θ, So, θ lies in 3rd or 4th quadrant.

 nπ  (x 2 + y 2)m sin 3 θ cos 3 θ


θ ≠  . If is independent of θ, 40. If x = and y = , where
 2 (xy)n cos 2 θ sin 2 θ
then the relation between α, m and n is 1
sin θ + cos θ = , then x + y =
[21 April 2019, Shift-II] 2
[21 April 2019, Shift-II]
(a) 2 mα = n(2α + 1) (b) m + n = α
48 34 65 79
(c) 2 mα = 2 nα + m (d) 2 m = (2 n + 1)α (a) (b) (c) (d)
9 9 18 18
Sol. (a)
Sol. (d)
Given, x = a sinα θ cosα + 1 θ
sin3 θ cos3 θ
Given, x = and y =
and y = a sinα + 1 θ cosα θ cos θ
2
sin2 θ
(x 2 + y 2)m sin3 θ cos3 θ
Now, Q x+ y= +
(xy)n cos2 θ sin2 θ
142 AP EAMCET Chapterwise Mathematics

sinθ(1 − cos2 θ) cosθ(1 − sin2 θ) ⇒ 2β = α + γ …(ii)


= +
cos2 θ sin2 θ From Eqs. (i) and (ii), we get
γ − α
= sinθ − 1 + cosθ 2 − 1 cos(α + γ) cos + 1 = 0
1 1 1
 cos2 θ   sin θ   2  2  
sinθ cosθ γ − α
cos
= − sinθ + − cosθ 1
⇒  =−
cos2 θ sin2 θ  2  2
sinθ cosθ
= + − (sinθ + cosθ) γ − α 2π
cos2 θ sin2 θ ⇒ =
2 3
sinθ cosθ 1
= + − π
cos2 θ sin2 θ 2 ⇒ γ −α =
3
sin3 θ + cos3 θ 1
= − So, tan (γ − α) = 3
cos2 θsin2 θ 2
(sinθ + cosθ)(sin2 θ + cos2 θ − sinθ cosθ) 1 42. If ABC is not a right angled triangle and
= −
sin2 θ cos2 θ 2 π  π  1 π 
sin  − A sin  − B = − cosec  − C  ,
 1  (1 − sinθ cosθ)
  4  4  2 2  4 
  1
= 2 − then tan A tan B + tan B tan C + tan C tan A =
(sinθ cosθ)2 2 [22 April 2019, Shift-I]
Qsin θ + cos θ = 1 ⇒ sin 2 θ + cos 2 θ + 2sin θ cos θ = 1 (a) cot A + cot B + cot C (b) tan A + tan B + tanC
 2 4 1 1
(c) (d)
1
⇒ 1 + 2sinθ cosθ = ⇒ 2sinθ cosθ = −
3 tan A + tan B + tanC cot A + cot B + cot C
4 4
3 Sol. (b)
⇒ sinθ cosθ = − A+ B+ C= π
8  Given,
We know that,
 1  1 + 3  1   11 
      cos (A + B + C) = cos A cos B cos C (1 − tan A tan B
 2  8  1  2  8  1
= − = − − tan B tan C − tan A tan C)
2
2 9 2
 − 3 and tan A + tan B + tan C = tan A tan B tan C
 
 8 64
π π π
Now, sin  − A sin  − B sin  − C  = −
1
11 64 1 44 1 88 − 9 79 4  4  4 
= × − = − = = 2 2
16 9 2 9 2 18 18 3
⇒   (cos A − sin A) (cos B − sin B)
1
41. Let α ,β and γ be such that 0 < α < β < γ < 2π.  3
For any x ∈ R if (cos C − sin C) = −
1
cos(x + α) + cos(x +β) + cos(x + γ) =0, 2 2
then tan(γ − α) = [22 April 2019, Shift-I] ⇒ cos A cos B cos C (1 − tan A)(1 − tan B)
(a) − 3 (b) 0 (c) 1 (d) 3 (1 − tan C) = − 1
Sol. (d) ⇒ cos A cos B cos C (1 − tan A − tan B − tan C
It is given that for any x ∈ R. + tan A tan B + tan B tan C + tan A tan C
cos(x + α) + cos(x + β) + cos(x + γ) = 0 − tan A tan B tan C)
Now, put x = − α − β − γ, = cos A cos B cos C (1 − tan A tan B
0 < α < β < γ < 2π, we get − tan B tan C − tan A tan C)
cos(β + γ) + cos(α + γ) + cos(α + β) = 0 = cos (A + B + C) (Q A + B + C = π)
α + 2β + γ   γ − α
⇒ 2cos  cos  ⇒ 1 − 2(tan A + tan B + tan C) + tan A tan B
 2   2  + tan B tan C + tan A tan C
+ cos(α + γ) = 0 …(i) = 1 − tan A tan B − tan B tan C − tan A tan C
π ⇒ − 2(tan A + tan B + tan C) = − 2(tan A tan B
Now, put x = − β, we get
2 + tan B tan C + tan A tan C)
sin(β − α) + sin(β − γ) = 0 ⇒ tan A tan B + tan B tan C + tan A tan C
⇒ sin(β − α) = sin(γ − β) = tan A + tan B + tan C
Trigonometric Functions and Identities 143

θ θ = sin 6π + cos π = 0 + (− 1)
43. If tan = cosecθ − sin θ, then tan 2 = [Qsin nπ = 0 and cos nπ = (− 1)n ]
2 2
[22 April 2019, Shift-I] = −1
(a) 2 − 5 (b) −2 + 5 45. The smallest positive value of x (in degrees)
(c) 2 + 5 (d) 2 + 5 for which tan(x + 100 °)
Sol. (b) = tan(x + 50 °) ⋅ tan x ⋅ tan(x − 50 °) is
θ [22 April 2019, Shift-II]
Given, tan = cosecθ − sinθ
2 1°
(a) 25° (b) 82 (c) 55° (d) 30°
1 1 − sin2 θ 2
= − sinθ =
sinθ sinθ Sol. (d)
1 − tan2 θ 
2 tan(x + 100°) = tan(x + 50°) ⋅ tan x ⋅ tan(x − 50°)
  tan(x + 100°)
cos2 θ  2 ⇒ = tan(x + 50°) ⋅ tan x
= = cot θ cosθ = tan(x − 50°)
sinθ θ θ
2tan 1 + tan2 
2 2 sin(x + 100°) cos(x − 50°) sin(x + 50°) sin x
⇒ =
θ θ θ θ sin(x − 50°) cos(x + 100°) cos(x + 50°) cos x
⇒ 2tan2 1 + tan2  = 1 + tan4 − 2tan2
2 2 2 2 sin(x + 100°) cos(x − 50°) + sin(x − 50°)
2θ 4θ 4θ 2θ
cos(x + 100°)
⇒ 2tan + 2 tan = 1 + tan − 2tan ⇒
2 2 2 2 sin(x + 100°) cos(x − 50°) − sin(x − 50°)
cos(x + 100°)
4θ 2θ
⇒ tan + 4 tan − 1 = 0
2 2 sin(x + 50°) sin x + cos(x + 50°) cos x
=
θ −4 ± (4) − (−1)4 −4 ±
2
16 + 4 sin(x + 50°) sin x − cos(x + 50°) cos x
∴ tan2 = = sin(x + 100° + x − 50°) cos(x + 50° − x)
2 2 ×1 2 ⇒ =
sin(x + 100° − x + 50°) − cos(x + 50° + x)
−4 ± 20
= =− 2± 5 sin(2x + 50°) cos(50°)
2 ⇒ =
sin(150°) − cos(2x + 50°)
θ Q tan2 θ >
⇒ tan2 = − 2 + 5 0
2  2  ⇒ − sin(2x + 50°) cos(2x + 50°)
= sin(90° + 60°) cos 50°
44. If the periods of the functions sin(ax + b) and 2
⇒ − sin(2x + 50°) cos(2x + 50°) = cos 60° cos 50°
4 2 2
tan(cx + d) are respectively and , then sin(4 x + 100°) 1
7 5 ⇒ − = cos 50°
sin(| a | + | c |) + cos(| a | − | c |) = 2 2
[22 April 2019, Shift-II]
⇒ sin(4 x + 100°) = − cos 50°
⇒ sin(4 x + 100°) = − sin 40°
(a) − 1 (b) 0
(c) 1 (d) 2 ⇒ sin(4 x + 100°) = sin(− 40°) = sin(180° + 40°)
⇒ (4 x + 100°) = (180° + 40°)
Sol. (a) ⇒ 4 x + 100° = 220°
We know that, ⇒ 4 x = 220° − 100° = 120°

Period of sin(ax + b) = and period of ⇒ x = 30°
|a|
π 46. For α ≠ 0 , if cos(θ + α), cosθ and cos(θ − α) are
tan(ax + b) =
|a| α
2π 4 π 2
in harmonic progression, then sec 2 θ ⋅ cos 2 =
∴ = and = 2
|a| 7 |c| 5 [22 April 2019, Shift-II]
7π 5π 1 1
⇒ |a| = and |c| = (a) 2 (b) 1 (c) (d)
2 2 2 4
∴ sin(| a |+ | c |) + cos(| a | − | c |) = Sol. (c)
7 π 5π   7 π − 5π 
= sin  +  + cos   Given that cos(θ + α), cosθ and cos(θ − α) are in
 2 2  2 2 HP, then
144 AP EAMCET Chapterwise Mathematics

2 1 1 2 5− 2 5 2 5− 2 5 5− 2 5
= + = = =
cosθ cos(θ + α) cos(θ − α) 1 − (5 − 2 5) 2 5− 4 5− 2
2 cos(θ − α) + cos(θ + α)
⇒ = 2π 2⋅ 5 − 2 5
cosθ cos2 θ − sin2 α 2tan  
4π   5
2cosθ cosα Now, tan   = = 5− 2

2
=  5 2π 5− 2 5
1 − tan2 1−
cosθ cos2 θ − sin2 α 5 ( 5 − 2)2
⇒ cos2 θ − sin2 α = cos2 θ cosα 5− 2 5
1−
π 5 − 2)2
⇒ cos θ(1 − cosα) = 1 − cos α cot   =
2 2 4 (
∴ 
 5 2⋅ 5 − 2 5
⇒ cos2 θ(1 − cosα) = (1 + cosα) (1 − cosα)
5− 2
⇒ cos2 θ = 1 + cosα
( 5 − 2)2 − 5 + 2 5 5+ 4 − 4 5 − 5+ 2 5
α = =
⇒ cos2 θ = 2cos2 2( 5 − 2) ( 5 − 2 5) 2( 5 − 2) ( 5 − 2 5)
2
α 1 4− 2 5 (2 − 5)
⇒ cos2 ⋅ sec2 θ = = =
2 2 2( 5 − 2) ( 5 − 2 5) ( 5 − 2) ( 5 − 2 5)
−1
47. If tan A − tan B = x and cot A − cot B = y, =
then cot (A − B) = [23 April 2019, Shift-I]
5− 2 5
π 2π 4π
(a)
xy
(b)
xy Now, tan + 2 tan + 4 cot
x+ y x− y 5 5 5
x− y y− x 5− 2 5 4
(c) (d) = 5 − 2 5 + 2⋅ −
xy xy 5− 2 5− 2 5
Sol. (d) 5− 2 5 5+ 2 4
= 5− 2 5 + 2 × −
Since, tan A − tan B = x 5− 2 5+ 2 5− 2 5
1 1 4
⇒ − = x = 5 − 2 5 + 2 ( 5 + 2) 5 − 2 5 −
cot A cot B 5− 2 5
cot B − cot A
⇒ = x 5 − 2 5 + 2( 5 + 2) (5 − 2 5) −4
cot A cot B =
5− 2 5
Q cot A − cot B = y (given)
y = 5 − 2 5 + (2 5 + 4) (5 − 2 5) −4
So, cot A cot B = −
x 5− 2 5
y
− +1 5 − 2 5 + 10 5 − 20 + 20 − 8 5 − 4
cot A cot B + 1 y− x =
Q cot (A − B) = = x = 5− 2 5
cot B − cot A − y xy
1 π
Hence, option (d) is correct. = = 1 ⋅ 37 = cot = cot 36°
5− 2 5 5
π 2π 4π
48. tan + 2 tan + 4 cot = 3 +1
5 5 5 49. If sin x + sin y = and cos x + cos y =
[23 April 2019, Shift-I] 2
π 2π 3π 4π 3 −1  x − y 2  x + y
(a) cot (b) cot (c) cot (d) cot , then tan 2   + tan   =
5 5 5 5 2  2   2 
Sol. (a) [23 April 2019, Shift-I]
π (a) 8 + 4 3 (b) 6 + 4 3
Given, θ = = 36°
5 (c) 3 + 3 (d) 12 + 6 3
 π
tan   = tan 36° = 5− 2 5 Sol. (a)
 5
It is given that
2π 2 tan 36°
tan 2θ = tan   = sin x + sin y =
3+1
 5  1 − tan2 36°
2
Trigonometric Functions and Identities 145

x+ y x− y 3+1 π 2π
⇒ 2sin cos = ∴ ∠R = π − =
2 2 2 3 3
x+ y x− y 3+ 1  π  π
⇒ sin cos = …(i) 51. If cos  x −  , cos x, cos  x +  are in a
2 2 4  3  3
3 −1
and cos x + cos y = harmonic progression, then cos x =
2 [22 April 2018, Shift-I]
x+ y x− y 3 −1 3 3 3
⇒ 2 cos cos = (a) (b) 1 (c) (d)
2 2 2 2 2 2
x+ y x− y 3 −1
⇒ cos cos = …(ii) Sol. (d)
2 2 4
It is given that,
On dividing relation (i) and (ii), we get
x+ y 3+1 π π
tan = cos x −  , cos x, cos x +  are in H.P.
2 3 −1  3  3
π π
⇒ tan2
x + y
=
3 + 1 + 2 3 2+ 3
= 2cos x −  cos x + 
 3   3
2 3+ 1 − 2 3 2− 3 ⇒ cos x =
 π  π
(x + y) 4 + 3 + 4 3 cos x −  + cos x + 
⇒ tan2 = = 7 + 4 3 ...(iii)  3  3
2 4− 3  2 π
2 cos x − sin 
2
On squaring and adding Eqs. (i) and (ii), we get  3
⇒ cos x =
π
x− y 1 1 2cos x cos
cos2 = × 2 (3 + 1) = 3
2 16 2 π π
⇒ cos2 x cos = cos2 x − sin2
2 x − y 3 3
⇒ sec =2
2 2  π 2 π
⇒ cos x 1 − cos  = sin
x− y x− y  3 3
⇒ tan2 = sec2 −1 = 2−1 =1
2 2 π  π
x− y x+ y ⇒ cos x 1 − cos  = 1 − cos2 
2 

So, tan2 + tan2 = 1 + (7 + 4 3)  3  3


2 2 π π π
=8+ 4 3 ⇒ cos2 x 1 − cos  = 1 − cos  1 + cos 
 3  3  3
Hence, option (a) is correct. π
⇒ cos x = 1 + cos
2
3
50. In ∆PQR, let ∠P > ∠Q . If the radian 2
π  3
measures of ∠P and ∠Q satisfy the equation ⇒ cos2 x = 2cos2 ⇒ cos2 x = 2 ×  
4 sin 3 x − 3 sin x + a = 0 , 0 < a < 1, then the 6  2
3 3
radian measure of ∠R is [23 April 2019, Shift-I] ⇒ cos2 x = 2 × ⇒ cos2 x =
π π 2π 5π 4 2
(a) (b) (c) (d) 3
3 2 3 6 ⇒ cos x = .
2
Sol. (c)
Given, 4sin3 x − 3sin x + a = 0 52. cos 3110 °+ cos 310 °+ cos 3130 ° =
[22 April 2018, Shift-I]
⇒ a = 3sin x − 4sin3 x
3 3
(a) (b)
⇒ a = sin 3x …(i) 4 8
Q sin 3P = a = sin 3Q (c)
3 3
(d)
3 3
[since P and Q satisfy the equation] 8 4
⇒ sin 3P = sin 3Q ⇒ sin 3P = sin(π − 3Q) Sol. (c)
⇒ 3P = π − 3Q ⇒ 3(P + Q) = π cos310 + cos3110 + cos3130°
π
⇒ P+ Q= We know that,
3 3
Q ∠P + ∠Q + ∠R = π cos3 x + cos3(120 − x) + cos3(120 + x) = cos 3x
4
146 AP EAMCET Chapterwise Mathematics

Here, x = 10 1 + cos 504° 1 − cos 252° 


= − 
Now, cos310 + cos3110° + cos3130° 2 2
 
= cos310 + cos3(120° − 10°) cos3(120° + 10°) 1 − cos 252° + 1 − cos 372° + 1 − cos132° 
 
=   cos(3 × 10°) = cos 30°
3 3  2 
 4 4 1 ° °
= (cos 504 + cos 252 )
3 3 3 3 4
= × =
4 2 8 [3 − cos 252° − 2cos 252° cos120° ]
1
 π y  π x = (2cos 378° cos126°) [3 − cos 252° + cos 252° ]
53. If tan  +  = tan 3  +  , 4
 4 2  4 2
3 3 10 + 2 5 10 − 2 5
3 sin x + sin 3 x = cos 18° (− sin 36°) = − × ×
then = 2 2 4 4
1 + 3 sin 2 x [22 April 2018, Shift-II] 3 −3 3
=− 100 − 20 = 80 = − (4 5)
(a) 0 (b) 1 (c) sin2 y (d) sin y 32 32 32
Sol. (d) 3 5
=−
π π 8
Given, tan +  = tan3  + 
y x
4 2 4 2 55. If α , β, γ are any three angles, then
3
1 + tan
y  1 + tan x  cos α + cos β − cos γ − cos(α + β + γ) =
⇒ 2 = 2 [22 April 2018, Shift-II]
y  x
1 − tan  1 − tan  α+β β+ γ γ+α
 (a) 4cos cos cos
2 2 2 2 2
y y  x x
3
α+β β+ γ γ+α
+ sin
cos  cos 2 + sin 2 
(b) 4cos sin sin
⇒ 2 2 = 2 2 2
y y  x x α+β β−γ γ −α
cos − sin  cos − sin  (c) 4cos sin sin
2 2  2 2 2 2 2
On squaring both sides, we are getting α+β β+ γ γ+α
3
(d) 4sin cos cos
1 + sin y  1 + sin x  2 2 2
= 
1 − sin y  1 − sin x  Sol. (b)
On applying componentdo and dividendo rule cosα + cosβ − cos γ − cos(α + β + γ)
2α + β + γ β+ γ β+ γ γ −β 
2 (1 + sin x)3 + (1 − sin x)3 = 2sin sin + 2sin sin 
= 2 2 2  2 
2sin y (1 + sin x)3 − (1 − sin x)3
β + γ  2α + β + γ γ − β
3sin x + sin3 x = 2sin sin + sin
⇒ sin y = 2  2 2 
1 + 3sin2 x
β+ γ α+ γ α+β
= 4sin sin cos
3sin x + sin3 x
So, = sin y 2 2 2
1 + 3sin2 x α+β β+ γ γ+α
= 4 cos sin sin .
2 2 2
54. (cos 252° − sin126 °) (cos 252° +
sin126 °) (sin 2 126 ° + sin 2 186 ° + sin 2 66 °) = 56. The maximum and minimum values of the
function f : [R → [R defined by
[22 April 2018, Shift-II]
 π
3 5 −3 5 −3 5 3 5 f (x) = 5 cos x + 3 cos  x +  + 8 for all x ∈ [R,
(a) (b) (c) (d)  3
8 8 4 4
are respectively. [23 April 2018, Shift-I]
Sol. (b)
(a) 15, 1 (b) 8, − 8
(cos 252° − sin126°) (cos 252° + sin126°) (c) − 7, − 15 (d) 1, − 15
(sin2126° + sin2186° + sin2 66°)
Sol. (a)
= (cos2 252° − sin2126°) π
f (x) = 5 cos x + 3 cos  x +  + 8
(sin2126° + sin2186° + sin2 66°)  3
Trigonometric Functions and Identities 147

π π
= 5 cos x + 3  cos x . cos − sin x sin  + 8 cos 25 ° + sin 25 °
58. If tan θ = and θ is in the third
 3 3  cos 25 ° − sin 25 °
1 3  quadrant, then θ = [23 April 2018, Shift-II]
= 5 cos x + 3  cos x − sin x  + 8
2 2  (a) 200° (b) 205°
13 3 3 (c) 225° (d) 250°
= cos x − sin x + 8
2 2 Sol. (d)
We know that, a cos x − b sin x is lie cos 25° + sin 25° 1 + tan 25°
If tanθ = =
between [− a + b , a + b ]
2 2 2 2 cos 25° − sin 25° 1 − tan 25°

2 2 = tan(45° + 25°)
−  13 +  − 3 3  + 8 ≤ f (x) Qθ is in the third quadrant, so
   
 2  2  tanθ = tan 70° = tan(180° + 70°) = tan 250°
2 2 ⇒ θ = 250°.
≤  13 +  − 3 3  + 8
    7 3π
 2  2  59. If cos A = and < A < 2 π, then
25 2
− 7 + 8 ≤ f (x) ≤ 7 + 8 ⇒ 1 ≤ f (x) ≤ 15
A A
So, maximum value of f (x) is 15 and minimum cos + cos − cos 2 A =
value is 1. 4 2 [23 April 2018, Shift-II]
1 27 3 27
57. If A = { x ∈[0 , 2π] / tan x − tan 2 x > 0} and (a) + (b) −
10 625 10 625
 1 3
+
27 1

27
B =  x ∈[0 , 2 π]/|sin x | <  , then A ∩ B = (c) (d)
 2 10 625 10 625
[23 April 2018, Shift-I] Sol. (a)
π 7π π 7π 3π
(a)  0,  ∪  π, (b)  0,  ∪  π,
7
  Given, cos A = and < A < 2π
 6  6   4  6  25 2
π 5π 7 π  π 7π 98 − 625
(c)  0,  ∪  ,  (d)  ,  Then, cos 2A = 2
49 
 −1= =−
527
 6  6 6  6 6   625 625 625
A 4 A 1
Sol. (a) cos = − and cos =
2 5 4 10
If A = {x ∈ [0, 2π] / tan x − tan2 x > 0}
A A 1 27
∴ tan x − tan2 x > 0 So, cos + cos − cos 2A = + .
4 2 10 625
⇒ tan x (1 − tan x) > 0
 π  2π 
0 < tan x < 1 60. 1 + cos  1 + cos 
π 5π  8  8
So, 0< x < and π < x <
4 4  3π   4π 
π   5π  1 + cos  1 + cos 
⇒ 
x ∈  0,  ∪  π ,   8   8
 4  4  5π   6π   7π 
1 1 + cos  1 + cos  1 + cos  =
Now, B = { x ∈ [0, 2π ] / |sin x |<  8  8  8
2
1 1 [24 April 2018, Shift-I]
⇒ − < sin x < 1 1 1 1
2 2 (a) (b) (c) (d)
1 −1 8 16 32 64
sin x < and sin x >
2 2 Sol. (b)
π 5π 7π 
⇒ 0< x < and  , 2π  and x∈  0,  Given that,
6  6   6  1 + cos π  1 + cos 2π  1 + cos 3π 
     
π 5π  8  8  8
⇒ x ∈  0,  ∪  , 2π 
 6  6 
1 + cos 4π  1 + cos 5π 
   
So, A ∩ B is defined in  8  8
 0, π  ∪  π , 7 π  1 + cos 6π  1 + cos 7π 
       
 6  6   8  8
148 AP EAMCET Chapterwise Mathematics

π π 3π π
= 1 + cos  1 + cos  1 + cos  1 + cos 
A
18 − 16sin2 − 32sin A / 2sin 5A / 2
 8   4   8   2  2
1 + cos π − 3π   1 + cos 3π  1 + cos π − π   = 18 − 8  2sin2  − 16  2sin sin 
A A 5A
    
 8    4    8    2  2 2

=18 − 8(1 − cos A) −16 cos


5A A  5A A  
π π 3π
= 1 + cos  1 + cos  1 + cos  −  − cos + 
 8   4   8   2 2  2 2
3π 3π π [as cos(A − B) − cos(A + B) = 2sin Asin B,
1 1 − cos  1 + cos  1 − cos 
()
 8  4  8 2sin2 A = 1 − cos 2A]
π π 3π  = 18 − 8 + 8 cos A − 16 cos 2A + 16 cos 3A
= 1 + cos  1 − cos  1 + cos 
 8  8  8 = 10 + 8 cos A − 16 [2cos2 A − 1]
1 − cos 3π  1 + 1  1 − 1  + 16 [4 cos3 A − 3cos A]
     
 8  2  2
[as cos 3A = 4 cos A − 3cos A, cos 2A = 2cos2 A − 1]
3

π 3π
= 1 − cos2  1 − cos2  1 − 
1
7
 8  8  2 Now, A lies in third quadrant and tan A =
3
π 3π 1 1  π 3π 1
= sin2 ⋅ sin2 ⋅ =  2sin2   2sin2  ⋅
8 8 2 4 8  8 2

=
1 1 − cos π  1 − cos 3π  ⋅ 1
    √7
4  4  4 2 4

1  1   1  1
= ⋅ 1 −  1 + ⋅
4  2  2 2
1  1 1 1 1 1 1 A 3
= 1 −  ⋅ = ⋅ ⋅ = .
4 2 2 4 2 2 16 cos A =
−3
4
7
61. If A is in the third quadrant and tan A = , = 10 + 8 
− 3   9 
3  − 16  2⋅   − 1
 4    16  
A A 5A
then 18 − 16 sin 2 − 32 sin sin =   − 27  − 3 
+ 16 4 ⋅  − 3⋅ 
2 2 2   64  
 4  
[24 April 2018, Shift-I]
− 27 9 
= 10 − 6 − 16 ⋅ + 16 
1
(a) − 6 (b) 11 + 
(c) 5 (d) 10 8  16 4
9
Sol. (b) = 10 − 6 − 2 + 16 ⋅ = 2 + 9 = 11.
16
13
Trigonometric Equations
1. The general solution of cos (x) − sin (x) = 0 is Sol. (c)
[17 Sep. 2020, Shift-I] 4Sin2 x − 4Sinx + 1 = 0
π π (2 Sinx − 1)2 = 0
(a) nπ − , n∈Z (b) 2 nπ + , n∈Z
4 4
π π 2 Sinx − 1 = 0
(c) nπ + , n ∈ Z (d) 2nπ − , n ∈ Z 1
4 4 Sinx =
2
Sol. (c)
π
cos x − sin x = 0 x = nπ + (−1)n ⋅ ,n ∈ Z
6
⇒ cos x = sin x ⇒ tan x = 1
π Hence, option (c) is correct.
⇒ x = nπ + ; (n ∈ Z)
4 4. If sin α = sin β and cos α = cos β, then
 π α − β = ........ for some integer n.
2. The value of x in 0 ,  satisfying the
 2 [18 Sep. 2020, Shift-II]
1 π
equation (sin x)(cos x) = is (a) nπ (b) 2 nπ +
2
4 π
[18 Sep. 2020, Shift-I] (c) 2 nπ − (d) 2nπ
2
π π π π
(a) (b) (c) (d)
6 3 8 12 Sol. (d)
It is given that, sin α = sin β
Sol. (d)
α −β α + β = 0
Given, sin x cos x =
1
⇒ sin 2x =
1 ⇒ 2sin   cos   …(i)
 2   2 
4 2
and cosα = cosβ
⇒ 2x = 30° , 150° ⇒ x = 15° ,75°
α + β β −α = 0
or x= ,
π 5π  π 
∈  0,  ⇒ 2sin   sin   …(ii)
 2   2 
12 12  2 
from Eqs. (i) and (ii), we get
3. General solution of 4 sin 2(x) − 4 sin(x) + 1 = 0 α −β  2 α + β 2 α + β  
sin2   sin   + cos   =0
is [18 Sep. 2020, Shift-II]  2    2   2  
π
(a) x = 2 nπ ± , n∈Z α −β
3 ⇒ sin2  = 0
π  2 
(b) x = nπ + (− 1)n , n ∈ Z
3 α −β
⇒ = nπ
π 2
(c) x = nπ + (− 1)n , n ∈ Z
6 ⇒ α − β = 2nπ, n ∈Integer
π
(d) x = nπ + ( − 1)n , n ∈ Z Hence, option (d) is correct.
4
150 AP EAMCET Chapterwise Mathematics

5. Find the general solution of ‘sin x + sin 2 x 7. Solve tan(x) + sec (x) = 3 , x ∈[0 ,2π]
+ sin 3 x = cos x + cos 2 x + cos 3 x’ [21 Sep. 2020, Shift-II]
[18 Sep. 2020, Shift-II] π π 13 π 6π
(a) (b) (c) (d)
2 π nπ π 3 6 6 13
(a) 2 nπ + , + , n∈Z
3 2 8 Sol. (b)
2 π nπ π
(b) 2 nπ − , − , n∈Z Given trigonometric equation is
3 2 8
2 π nπ π tan x + sec x = 3, x∈[0, 2π]
(c) 2 nπ + , ± , n∈Z
3 2 8 1
∴ sec x − tan x =
2 π nπ π
(d) 2 nπ ± , + , n∈Z 3
3 2 8 1
So, 2tan x = 3 −
Sol. (d) 3
sin x + sin 2x + sin 3x = cos x + cos 2x + cos 3x 2 1 π 7π
⇒ 2tan x = ⇒ tan x = ⇒ x = or
(sin x + sin 3x) + sin 2x = (cos x + cos 3x) + cos 2x 3 3 6 6
2sin   ⋅ cos   + sin 2x = 2cos   .
4x 2x 4x Hence, option (b) is correct.
 2  2  2
5 −1 n
cos  + cos 2x
2x 8. If sin(2 x) = then x = π + (−1)n (m),
 2 4 2
2sin 2x cos x + sin 2x = 2cos 2x.cos x + cos 2x n ∈ Z, find m. [22 Sep. 2020, Shift-II]
sin 2x(2cos x + 1) = cos 2x(2cos x + 1) π π π π
(a) (b) (c) (d)
(sin 2x − cos 2x) (2cos x + 1) = 0 10 5 20 40
sin 2x − cos 2x = 0 (or) 2cos x + 1 = 0 Sol. (c)
π π 5 −1 π
tan 2x = 1 = tan ⇒ 2x = nπ + sin 2x = ⇒ sin 2x = sin
4 4 4 10
nπ π −1 π nπ π
⇒ x= + or cos x = 2x = nπ + (−1)n ⇒ x= + (−1)n
2 8 2 10 2 20

cos x = cos but given,
3 nπ
2π x= + (−1)n . m
x = 2nπ ± 2
3 π
Hence, option (d) is correct. On comparison, we get m =
20
6. If the roots of the given equation Hence, option (c) is correct.
(cos p − 1) x 2 + (cos p) x + sin p = 0 are real, 9. Find the general solution of
then [18 Sep. 2020, Shift-II] 3 sin 4 (θ) + cos 4 (θ) = 1 [23 Sep. 2020, Shift-I]
− π π
(a) p ∈ (− π, 0) (b) p ∈  ,  (a) nπ only
 2 2
π
(c) p ∈(0, π) (d) p ∈(0, 2 π) (b) nπ + only
4
Sol. (c) π
(c) nπ − only
(cos p − 1) x 2 + (cos p) x + sin p = 0 4
π π
Since, roots are real (d) nπ, nπ + and nπ −
4 4
⇒ ∆ ≥ 0 ⇒ b 2 − 4ac ≥ 0
(cos p) − 4(cos p – 1)(sin p) ≥ 0
2 Sol. (d)
Given equation, is 3 sin4 θ + cos4 θ = 1
cos p ≥ 0 and (cos p – 1) ≤ 0 ∀ P ∈ R
2

2 sin4 x + sin4 x + cos4 x = 1


∴ sin p ≥ 0
∴ p ∈(0, π) ⇒ 2 sin4 x + 1 − 2 sin2 x cos2 x = 1
Hence, option (c) is correct. [Q sin4 x + cos4 x = 1 − 2sin2 x cos2 x]
Trigonometric Equations 151

π π
⇒ 2 sin2 x(sin2 x − cos2 x) = 0 ∴ |cos x| = cos x, ∀ x ∈ , 
 4 3 
2 sin x = 0 (or) sin2 x − cos2 x = 0
2

⇒ sin2 x = 0 (or) sin2 x = cos2 x 2 + {x}


So, 2 sin x cos x =
⇒ sin x = 0 tan2 x = 1 1 − {x}
π Q Maximum value of 2 sin x cos x = sin 2x is ‘1’.
⇒ x = nπ or x = nπ ±
4 2 + {x}
and minimum value of (at {x} = 0) is ‘2’.
1 − {x}
10. If tan θ + tan 2θ + 3 tan θ tan 2θ = 3, then
∴ For the given equation number of solution
the general values of θ are k=0
[20 April 2019, Shift-II] π π
∴ ∀ x ∈  ,  , K tan x = 0.
2
π π  4 3 
(a) (3n + 1) , n ∈ Z (b) (3n + 1) , n∈Z
3 9
π π 12. If 4(sin 2 x sin 4 x + sin 2 x) = 3, then x =
(c) (3n + 1) , n ∈ Z (d) (2 n + 1) , n ∈ Z
6 9 [21 April 2019, Shift-II]
Sol. (b) 2 nπ π nπ π
(a) ± , n∈Z (b) ± , n∈Z
It is given that 3 9 3 9
nπ π nπ 2π
tanθ + tan 2θ + 3 tanθ tan 2θ = 3 (c) + (−1)n , n ∈ Z (d) + (−1)n , n∈Z
3 9 3 9
tanθ + tan 2θ π
⇒ = 3 = tan
1 − tanθ tan 2θ 3 Sol. (b)
π π Given, 4(sin 2x sin 4 x + sin2 x) = 3
⇒ tan(3θ) = tan ⇒ 3θ = nπ + , n ∈ Z
3 3 ⇒ 2(cos 2x − cos 6 x) + 21
( − cos 2x) = 3
π ⇒ 2cos 2x − 2cos 6 x + 2 − 2cos 2x = 3
⇒θ = (3n + 1) , n ∈ Z
9 ⇒ −2cos 6 x + 2 = 3
Hence, option (b) is correct. ⇒ −2cos 6 x = 1
1
11. Let [ x ] denote the largest integer ≤ x. If the ⇒ cos 6 x = −
2
number of solutions of 2π
2 + x − [ x] ∴ 6 x = 2nπ ±
sin x 4 cos 2 x = is k, then for 3
1 − x + [ x] 2nπ 2 π nπ π
⇒ x= ± × = ±
π π 2 6 6 3 3 9
x ∈  , , the value of k tan x
4 3 13. The number of real values of
[21 April 2019, Shift-I]  π 3π 
(a) is equal to 1
x ∈[0 , 2 π] −  ,  satisfying the equation
2 2 
(b) lies in between 21 and 2 3
2
x − 3 sin x + 1
(c) is equal to zero |cos x|2 sin = 1, is [22 April 2019, Shift-I]
1 1
(d) lies in between 3 and (a) 3 (b) 4 (c) 5 (d) 6
2 2
Sol. (c) Sol. (c)
2
x − 3sin x + 1
2 + x − [x] Given that, |cos x|2sin =1
Given, sin x 4 cos2 x =
1 − x + [x] ∴ |cos x| = 1 or 2sin x − 3sin x + 1 = 0
2

2 + {x} cos x = ± 1 or (2sin x −1)(sin x − 1) = 0


⇒ sin x 4 cos2 x = [Q x − [x] = {x}]
1 − {x} 1
x = 0, π , 2π or sin x = or sin x = 1
2 + {x} 2
⇒ 2 sin x |cos x| = = y (let)
1 − {x} π 5π π
x = 0, π , 2π or x = , or x =
2 + {x} 6 6 2
Q y= >0 π 5π  π 3π  
1 − {x} ⇒ x = 0, π , 2π , , Q x ∈ [0, 2π] −  , 
6 6   2 2  
[because {x} ∈ [0, 1), so 2 + {x} and
1 − {x} are positive] ∴ The number of real values of x is 5.
152 AP EAMCET Chapterwise Mathematics

14. If cos 2θ + α sin θ = 2α − 7 has a solution, then 16. The general solution of the equation
[22 April 2019, Shift-II]
3 − 5 sin x + sin 2 x + cos x = 0 is
(a) α ∈ [− 2, 4] (b) α ∈ [− 6, − 2 ]
[23 April 2018 Shift-I]
(c) α ∈[6, 8] (d) α ∈[2, 6]
π π
Sol. (d) (a) nπ + (− 1) , n ∈ Z
n
(b) 2 nπ ± , n∈Z
6 6
Given, cos 2θ + α sinθ = 2α − 7 π 5π
(c) (2 n + 1)π − , n ∈ Z (d) 2 nπ ± , n∈Z
⇒ cos2 θ − sin2 θ + α sinθ − 2α + 7 = 0 6 6
⇒ 1 − 2sin2 θ + α sinθ − 2α + 7 = 0 Sol. (c)
⇒ − 2sin2 θ + α sinθ − 2α + 8 = 0 Given equation,
3 − 5sin x + sin2 x + cos x = 0
− α ± α − 4(− 2α + 8) (− 2)
2
∴ sinθ =
−4 ⇒ 3 − 5sin x + sin2 x = − cos x ,{cos x}
− α ± α 2 + 8(− 2α + 8) On squaring both sides, we get
−1≤ ≤1 ⇒ 3 − 5sin x + sin2 x = cos2 x
4
− 4≤ − α ± (α − 8)2 ≤ 4 ⇒ 2sin2 x − 5sin x + 2 = 0
⇒ 2sin x − 4sin x − sin x + 2 = 0
2
⇒ −4 ≤ − α ± (α − 8) ≤ 4
⇒ − 4 ≤ − 2α + 8 ≤ 4 ⇒ 2sin x (sin x − 2) − 1 (sin x − 2) = 0
1
⇒ 4 ≤ 2α ≤ 12 ⇒ 2 ≤ α ≤ 6 ⇒ sin x = or 2 ⇒ Q sin x ∈ [−1, 1]
2
Now, α ∈[2, 6] 1
∴ sin x = and cos x < 0
15. If the general solution of sin 5 x = cos 2 x is of 2
π 1 − 3
the form a n ⋅ for n = 0 ,±1,±2, ...., then a n = Means, sin x = and cos x =
2 2 2
π
[22 April 2018, Shift-I] Therefore, x = (2n + 1) π − , n ∈ Z.
2n 2 n + (−1)n 6
(a) (b)
5 + 2(−1)n 5 + 2(−1)n 17. The number of solutions of the equation
2n + 1 2n − 1 4 cos 2θ ⋅ cos 3θ = sec θ, when 0 < θ < π, is
(c) (d)
5 + 2(−1)n 5 + 2(−1)n [23 April 2018, Shift-I]
Sol. (b) (a) 2 (b) 4 (c) 6 (d) 8
Given, sin 5x = cos 2x Sol. (c)
π
⇒ sin 5x = sin − 2x  We have,
2  4 cos 2 θ ⋅ cos 3 θ = secθ, (where 0 < θ < π)
π
5x = nπ + (− 1)n  − 2x 
1
⇒ ⇒ 2(2cos 2 θ ⋅ cos 3 θ) =
2  cosθ
π 1
⇒ 5x = nπ + (− 1)n − (− 1)n 2x ⇒ 2[cos 5 θ + cos(− θ)] =
2 cosθ
π [Q 2cos A cos B = cos(A + B) + cos(A − B)]
⇒ 5x + (− 1) (2x) = {2n + (− 1)n }
n
1
2 ⇒ 2[cos 5 θ + cosθ] = [Q cos(−θ) = cosθ]
π cosθ
⇒ x(5 + (− 1) 2) = (2n + (− 1)n)
n
2 ⇒ 2cos 5 θ ⋅ cosθ + 2cos2 θ = 1
π  2n + (− 1) 
n
⇒ (cos 6 θ + cos 4 θ) + (2cos2 θ − 1) = 0
⇒ x=  
2  5 + 2(− 1)n  ⇒ cos 6 θ + cos 4θ + cos 2θ = 0
π [Q 2cos2 θ − 1 = cos 2θ]
⇒ x = an ⋅
2 ⇒ 2cos 4θ ⋅ cos 2θ + cos 4θ = 0
2n + (− 1)n ⇒ cos 4θ (2cos 2θ + 1) = 0
So, an =
5 + 2(− 1)n ⇒ cos 4θ = 0 and 2cos 2θ + 1 = 0
Trigonometric Equations 153

π 1 Clearly smallest positive root lies in the interval


⇒ 4θ = (2n + 1) , cos 2θ = −
2 2  π , 3π  .
 
π 4π  2
⇒ θ = (2n + 1) , 2 θ = 2π / 3 or
8 3
π 3π 5π 7 π π 2π 20. Let P(α, β) and Q(γ, δ) be two points that lie
⇒ θ= , , , , ,
8 8 8 8 3 3 on the curve tan 2 (x + y) + cos 2 (x + y)
So, number of solution of given equation is 6. + y 2 + 2 y = 0 in the XY -plane. If the distance
 π between P and Q is d, then cos d =
18. If θ is in the interval 0 ,  satisfying the
 2 [24 April 2018, Shift-I]
equation cos 2θ ⋅ sec θ + sec 2 θ = 0 , then
4
(a) 0
sin 2 θ = (b) (− 1)n , n ∈ N
[23 April 2018, Shift-II]
(c) ± π
1 3 1 2
(a) (b) (c) (d) (d) ± 2nπ , n ∈ N
3 4 2 3
Sol. (b)
Sol. (d)
We have, tan2(x + y) + cos2(x + y) + y 2 + 2y = 0
Given, cos 2 θ ⋅ sec4 θ + sec2 θ = 0, θ ∈(0, π / 2)
⇒ sec2(x + y) − 1 + cos2(x + y) + y 2 + 2y = 0
⇒ (1 − tan2 θ)sec2 θ + sec2 θ = 0
⇒ sec2 (x + y) + cos2 (x + y) + y 2 + 2y + 1 = 2
⇒ sec2 θ[1 − tan2 θ + 1] = 0
Now, as minimum value of
⇒ tan2 θ = 2 ⇒ sin2 θ = 2 / 3 sec2(x + y) + cos2(x + y) is 2.
19. The smallest positive root of the equation ∴ x + y = 0 and y 2 + 2y + 1 = 0
tan x − x = 0 lies in the interval ⇒ x = −y
[24 April 2018, Shift-I]
and (y + 1)2 = 0
π π
(a)  0,  (b)  , π  ⇒ x = −y
 2 2 
3π  3π and y = −1
(c)  π,  (d)  , 2 π  Thus, x =1
 2   2 
and y = −1
Sol. (c)
Hence, the points P and Q coincides, and so d = 0
Here, tan x − x = 0 ⇒ tan x = x
⇒ cos d = 1 ...(i)
Y y=tan x
Now, again if we take x + Y = π and
y=x y 2 + 2y + 1 = 0, then x = π + 1 and y = −1
Similarly, if x + y = 2π
and y 2 + 2y + 1 = 0
X
π π π Then, x = 2π + 1, y = −1
32
2
∴ d= π
Q cos d = −1 ...(ii)
From Eqs. (i) and (ii), we get cos d = (−1)n , n∈N
14
Properties of Triangles
1. The perimeter of ∆ABC is 36 cm and its tan A tan B tan C
4. In triangle ABC, = = , then
in radius is 8 cm. Then, the area of the 2 3 4
triangle is [17 Sep. 2020, Shift-I] the value of sec 2 A + sec 2 B + sec 2 C =
(a) 144 cm2 (b) 124 cm2 (c) 164 cm2 (d) 104 cm2 [18 Sep. 2020, Shift-I]
Sol. (a) 101 111
(a) (b)
∆ 8 8
In radius (r) = 121 91
S (c) (d)
∆ = (r ⋅ s), ∆ → Area, S → Semi-perimeter 8 8
36 Sol. (b)
⇒ ∆ = 8× = 144 cm2
2 In a ∆ABC, it is given that
2. Let a , b and c be the lengths of the sides of a tan A tan B tan C
= = = k (Let)
triangle with its opposite angles A , B and C 2 3 4
respectively. If ∠C = 60 ° , then the value of ⇒ tan A = 2 k, tan B = 3 k and tan c = 4 k.
c(a + b) + (a 2 + b2) Since in ∆ABC,
is tan A + tan B + tan C = tan A tan B tan C
(b + c) (c + a) [17 Sep. 2020, Shift-I] 3
1 3 ⇒ 9k = 24k3 ⇒ k2 = {Q k ≠ 0}
(a) (b) (c) 1 (d) 3 8
2 2 Now, sec2 A + sec2 B + sec2 C
Sol. (c)
= 3 + tan2 A + tan2 B + tan2 C
a2 + b2 − c 2 a2 + b2 − c 2
cos C = ⇒ cos 60° = 3 24 + 87 111
2ab 2ab = 3 + k2[4 + 9 + 16] = 3 + (29) = = .
8 8 8
⇒ a 2 + b 2 − c 2 = ab
⇒ a 2 + b 2 = ab + c 2 …(i) 5. In a ∆ABC, ∠C = 60 ° and ∠A = 75°. If D is a
c(a + b) + (a 2 + b 2) ca + bc + ab + c 2 point on AC such that the area of ∆BAD is 3
Now, = =1
(b + c)(c + a) bc + ab + c 2 + ac times the area of ∆BCD, then the measure of
3. For a triangle with the sides of lengths 6, 5 ∠ABD is [18 Sep. 2020, Shift-I]
and 9, then the radius of the circle is (a) 30° (b) 45°
[17 Sep. 2020, Shift-I] (c) 60° (d) 90°
3 Sol. (a)
(a) 3 (b) 2 (c) 5 (d)
2 In a ∆ABC, it is given the angles A = 75° and
Sol. (b) C = 60°
∆ s(s − a)(s − b)(s − c) and ∆BAD = 3(∆BCD)
In radius = =
s s ⇒ (AD) = 3(CD)
10(4)(5)(1) AD
= = 2 ⇒ = 3
10 CD
Properties of Triangles 155
A
Sol. (a)
3
75º Given, cos A + cos B + cos C =
2
D π
Above equation is possible only for A = B = C =
3
60º ∴ ∆ABC is equilateral,
B C Hence, option (a) is correct.
As angle B = 45°, Now let ∠ABD = α, then 8. For a right angled triangle having the
∠DBC = 45° − α
lengths of two sides as 2 2 and 5, find the
Now, by sine law
length of the third side. [21 Sep. 2020, Shift-I]
sin α sin 75°
= , (in ∆ABD) (a) 4 2 (b) 15 (c) 17 (d) 13
AD BD
sin (45°−α) sin 60° Sol. (c)
and = , (in ∆DBC)
CD BD Let the third side of right angled triangle is ‘p’,
(CD)sinα sin 75° then either p 2 = (2 2)2 + 52 = 8 + 25 ⇒ p = 33
∴ =
(AD)sin(45°−α) sin 60° or p 2 = 52 − (2 2)2 = 25 − 8 ⇒ p = 17
1 3
+ Hence, option (c) is correct.
1 sinα
⇒ × = 2 2 2 2
3 sin(45°−α) 3 9. If r = in radius, ∆ = Area of ∆ABC, s =
2 semi-perimeter then which of the following
sin α 3+1 is true? [21 Sep. 2020, Shift-I]
⇒ = r
sin (45°−α) 2 (a) ∆ = r + s (b) ∆ = (c) ∆ =(rs )2 (d) ∆ = rs
s
sin (45°−α) 3 −1
⇒ = Sol. (d)
sinα 2
Let the length of sides are a , b and c, so
1 1 3 −1 a+ b+ c
⇒ cotα − = ⇒ cotα = 3 ⇒α = 30° s=
2 2 2 2
6. In a ∆ABC, b : c = 3 : 2 and the angles A, B, Now, from the diagram area of triangle ABC is
C are in AP, then ∠A = [18 Sep. 2020, Shift-I] A
(a) 45° (b) 65° (c) 55° (d) 75° E
Sol. (d) F b
r r
c
It is given that in a ∆ABC, angles A, B, C are in I r
AP, so B = 60°. {Q A + B + C = 180° }
and it is also given that, B D C
a
b 3 sin B 3
= ⇒ =
c 2 sin C 2 ∆ = ∆IBC + ∆ICA + ∆IAB
1 1 1
3 = ar + br + cr
3 1 2 2 2
⇒ 2 = ⇒sin C = ⇒ C = 45°  a + b + c
sin C 2 2 ∆=r  = rs
 2 
So, angle A = 180°− B − C = 180°−105° = 75°
Hence, option (d) is correct.
7. Let A, B and C be three angles of a ∆ ABC
3 10. In a ∆ ABC , a = 1, b = 3 and ∠C = π /6. Then
such that cos A + cos B + cos C = , then the the measure of the third side c =
2
[21 Sep. 2020, Shift-II]
∆ ABC is ......... [18 Sep. 2020, Shift-II]
(a) 4 (b) 3
(a) Equilateral (b) Right angled
(c) 1 (d) 2
(c) Isosceles but not equilateral
(d) Scalene Sol. (c)
156 AP EAMCET Chapterwise Mathematics

In a ∆ABC, it is given that, a = 1 , b = 3 and 13. In a ∆ABC , if a , b, c are its sides and
π
∠C = , then by cosine law, we have a b
6 ∠C = 60 °, find the value of +
π a2 + b2 − c 2 3 1 + 3− c2 b+ c c+ a
cos = ⇒ =
6 2ab 2 2 3 [22 Sep. 2020, Shift-I]
⇒ 3= 4− c ⇒c =1
2 2 3 1
(a) 1 (b) 0 (c) (d)
⇒ c = ±1 ⇒ c = 1 {Q c = length of 2 2
side} Sol. (a)
Hence, option (c) is correct. In a ∆ABC, it is given ∠C = 60°, then
a2 + b2 − c 2
11. If 2 is the length of a side of a triangle with cos 60° =
its opposite angle ‘π /3’, then the 2 ab
circumradius of the triangle is …… ⇒ ab = a 2 + b 2 − c 2 ⇒ c 2 = a 2 + b 2 − ab ...(i)
[22 Sep. 2020, Shift-I]
a b ac + a + b + bc 2 2
2 4 Now, + =
(a) (b) b + c c + a bc + ab + c 2 + ac
3 3
(c) 2 (d) 4 ac + c 2 + ab + bc
= {Q a 2 + b 2 = c 2 + ab}
ac + ab + c 2 + bc
Sol. (a)
According to the question, =1
A 14. The angles A , B, C of a triangle ABC are in
π/3 AP. If AB = 6 , BC = 7 , then AC =
[22 Sep. 2020, Shift-II]
(a) 40 (b) 41 (c) 43 (d) 6
B Sol. (c)
2 C Given In ∆ABC,
∠A, ∠B, ∠C are in AP
π
Q a = 2and∠A = ⇒ 2∠B = ∠A + ∠C
3
In ∆ABC,
So, by sine law, we have A
a 2
= 2R ⇒ = 2R
sin A 3/ 2
2
⇒ R= 6 x
3

12. In ∆PQR, find Σ(q + r) cos P , if p , q, r denote its 60º


7
(p + q + r) B C
sides and s =
2 [22 Sep. 2020, Shift-I] ∠A + ∠B + ∠C = 180°
(a) s (b) s/2 2∠B + ∠B = 180° ⇒ ∠B = 60°
(c) 2s (d) 4s From cosine rule,
Sol. (c) AB2 + BC 2 − AC 2
cos B =
It is given that in a ∆PQR, p, q, r denotes its 2AB × BC
sides, so Σ(q + r) cos P
36 + 49 − x 2
= q cos P + r cos P + r cos Q + p cos Q + p cos R cos 60° =
+ q cos R 2× 6 × 7
= (q cos P + p cos Q) + (r cos P + p cos R) 1 85 − x 2
= ⇒ 41 = 85 − x 2
+ (r cos Q + q cos R) 2 84
=r+ q+ p (by projection law) x 2 = 85 − 42 ⇒ x 2 = 43 ⇒ x = 43
= 2s Q s = p + q + r  Hence, option (c) is correct.
 
 2 
Properties of Triangles 157

15. If the lengths of the sides of a triangle are 17. In ∆ABC , ∠ A = 30 ° +∠C and R − ( 3 + 1)r = 0
15 , 20 , 25 units. Find the circumradius of the where r is the inradius and R is the
triangle. [23 Sep. 2020, Shift-I] circumradius, then [23 Sep. 2020, Shift-I]
(a) 30 units (b) 7.5 units (a) ABC is a right-angled triangle
(c) 12.5 units (d) 20 units (b) ABC is an equilateral triangle
Sol. (c) (c) ABC is acute angled
Given, length of sides are (d) ∠A = 75° ∠B = 60° ∠C = 45°
A Sol. (a)
In a triangle ABC, it is given that A = 30° + C
and R − ( 3 + 1)r = 0 ⇒ R = ( 3 + 1)r
20 25 A B C
⇒ R = ( 3 + 1)4R sin sin sin
2 2 2
B A C
C 15 B ⇒ ( 3 − 1) = 4 sin  2sin sin 
2 2 2
a = 15 ⇒ b = 20 ⇒ c = 25
A−C A + C
⇒ ( 3 − 1) = 4sin  cos
B
The triangle formed by above measurements is − cos
right triangle 2  2 2 
1 1 {Q A = 30° + C}
∴circum radius = × Hypotenuse = × 25 = 12.5
2 2 B B
⇒ ( 3 − 1) = 4sin [cos15°− cos(90°− )]
Hence, option (c) is correct. 2 2
B  1 + 3  B
16. Let a , b and c be the lengths of the sides of a ⇒ ( 3 − 1) = 4 sin    − sin 
2   2 2  2 
triangle with its opposite angles A , B and C
⇒ 4sin2   − 21
 3 B B
( + 3) sin + ( 3 − 1) = 0
respectively. If a = 3, b = 4 and A = sin −1   ,  2 2
 4
then the angle B is [23 Sep. 2020, Shift-I] B ( +
21 3) ± ( +
21 3)2 − 16( 3 − 1)
∴ sin =
(a) 30° (b) 45° (c) 90° (d) 60° 2 8
Sol. (c) ( +
21 3) ± 8 + 4 3 − 16 3 + 16
=
Given a = 3, b = 4 8
A = sin−1  
3 ( +
21 3± 2 12 − 6 3
 4 =
8
A ( +
21 3) ± 2(3 − 3)
=
8
1 3 −1
c b = or = sin 45° or sin 15°
2 2 2
B
= 45° or 15° ⇒ B = 90° or 30°
2
B a C
Hence, option (a) is correct.
3
sin A =
4 18. In triangle ABC, if
We have, a = 2Rsin A b+ c c+ a a + b cos A + cos B
3 = = , then =
3 = 2R 9 10 11 cos C
4 [20 April 2019, Shift-I]
R=2 9 10 11 12
(a) (b) (c) (d)
b = 2Rsin B 10 11 12 13
4 = 2(2)sin B [QR = 2] Sol. (c)
sin B = 1, B = 90° b+ c c+ a a+ b
Let = = =k
Hence, option (c) is correct. 9 10 11
158 AP EAMCET Chapterwise Mathematics

⇒ b + c = 9k, c + a = 10k and a + b = 11k ∆2 C


= tan
and a + b + c = 15k (s − a)(s − b) 2
∴a = 6k, b = 5k and c = 4k ∆2 (s − a)(s − b) ∆2
b2 + c 2 − a2 a2 + c 2 − b2 = = =∆
+ (s − a)(s − b) s(s − c) ∆
cos A + cos B 2bc 2ac
Q = ∆  ∆ ∆ 
cos C a + b2 − c 2
2
 + 
r2(r3 + r1) s − b  s − c s − a
2ab (B) =
25 + 16 − 36 36 + 16 − 25 r1 r2 + r2r3 + r3r1 (s − a) + (s − b) + (s − c)
+ (s − a)(s − b)(s − c)
= 40 48
36 + 25 − 16 ∆ 2S − a − c
×
60 (s − b) (s − a)(s − c)
5 27 1 9 11 =
+ + 3S − a − b − c
11
= 40 48 = 8 16 = 16 = (s − a)(s − b)(s − c)
45 3 3 12
∆ (b)
60 4 4 = =b
Hence, option (c) is correct. s(s − a)(s − b)(s − c)
a sin(A − B) sin A sin(A − B)
19. In a ∆ABC, with usual notation, match the (C) = ⇒ =
c sin(B − C) sin C sin(B − C)
items in List-I with the items in List-II and
choose the correct option. ⇒sin Asin(B − C) = sin(A − B)sin C
[20 April 2019, Shift-I] ⇒sin A(sin B cos C − sin C cos B)
= (sin A cos B − cos A sin B) sin C
List I List II
⇒ 2sin A cos B sin C = sin A sin B cos C
 4R − r1 − r2  + sin B cos A sin C
(A) r1 r2   1. b
 r1 + r2  a a2 + c 2 − b2 c
⇒2 × ×
2R 2ac 2R
r2 (r3 + r1 ) a2 , b 2 , c 2 are  a b a2 + b2 − c 2 
(B) r1r2 + r2 r3 + r3 r1 2.
in AP = × × 
 2R 2R 2ab 
a sin( A − B)
(C) = 3. ∆  b b2 + c 2 − a2 c 
c sin(B − C ) + × × 
 2R 2bc 2R 
A
(D) bc cos 2 4. R r1 r2 r3 ⇒ 2(a 2 + c 2 − b 2) = a 2 + b 2 − c 2 + b 2 + c 2 − a 2
2
⇒ 2a 2 + 2c 2 − 2b 2 = 2b 2 ⇒ 2b 2 = a 2 + c 2
5. s ( s − a)
⇒ a 2 , b 2 , c 2 are in AP.
A B C D A B C D A s(s − a)
(D) bc cos2 = bc × = s(s − a)
(a) 4 3 1 5 (b) 5 4 3 2 2 bc
(c) 3 1 2 5 (d) 4 5 2 1 Hence, option (c) is correct.
Sol. (c) 20. If a , b and c are the sides of ∆ABC for which
4R − r1 − r2  ∆2
(A) r1 r2 = r1 = 8 , r2 = 12 and r3 = 24 , then the ordered
r1 + r2  (s − a)(s − b) triad (a , b, c) = [20 April 2019, Shift-I]
A 
4R − 4R cos  sin cos + sin cos  
C A B B (a) (12, 20, 16) (b) (12, 16, 20)
2 2 2 2 2  (c) (16, 12, 20) (d) 20, 16, 12)
C A B B A  Sol. (b)
4R cos  sin cos + sin cos 
2 2 2 2 2 
 It is given that

4R 1 − cos2 
C r1 = 8 = …(i)
∆2  2 s−a
= ∆
(s − a)(s − b) 4R cos 2C r2 = 12 = …(ii)
2 s−b
Properties of Triangles 159
∆ 3
and r3 = 24 = …(iii) = [(b 2 + c 2 − 3b 2 − c 2) (3b 2 + c 2 + b 2 − c 2)] + 2a 2
s−c 4b 2
3
From Eqs. (i) and (ii), we get = 2 (−2b 2) (4b 2) + 2a 2 = −6b 2 + 2a 2
s−b 2 4b
=
s−a 3 = 2 (a 2 − 3b 2) = 2c 2 [from Eq. (iii)]
⇒ 3s − 3b = 2s − 2a 22. Let ∆ denote the area of a ∆ABC. If α , β , γ are
⇒ 5a + c = 5b …(iv)
the lengths of the altitudes of the ∆ ABC,
From Eqs. (ii) and (iii), we get
s−c 1 then α −2 + β −2 + γ −2 =
=
s−b 2 [20 April 2019, Shift-II]
4
⇒ 2s − 2c = s − b (a) (tan A + tan B + tanC )

⇒ a + 3b = 3c …(v) 1
(b) (cot A + cot B + cot C )
and from Eqs. (i) and (iii), we get ∆
s−c 1 ∆2
= (c) (tan A + tan B + tanC )
s−a 3 2
⇒ 3s − 3c = s − a ∆2
(d) (cot A + cot B + cot C )
⇒ 2a + b = 2c …(vi) 4
On solving Eqs. (iv), (v) and (vi), we get Sol. (b)
(a , b , c) = (12, 16, 20) a2 b2 c2
Since, α −2 + β −2 + γ −2 = + +
Hence, option (b) is correct. 4∆2 4∆2 4∆2
21. In ∆ ABC, if the median AD drawn through A a2 + b2 + c 2
=
4∆2
is perpendicular to the side AC, then
3 ca cos A cos C + 2 a 2 = (a 2 + b 2 − c 2) + (b 2 + c 2 − a 2) + (c 2 + a 2 − b 2)
[20 April 2019, Shift-II] = 2
4 
abc 
(a) c 2 (b) 2c 2 (c) 3c 2 (d) 4c 2 
 4R 
Sol. (b) 2ab cos C + 2bc cos A + 2ac cos B
According to Apollonius Theorem =
a 2b 2c 2
 a2  4R2
c 2 + b 2 = 2  AD 2 +  …(i)
 4 cos C cos A cos B
= + +
A  abc  c  abc  a  abc  b
     
 4R  2R  4R  2R  4R  2R
c
b 1
= (cot A + cot B + cot C)

B a/2 D a/2 C Hence, option (b) is correct.
B + C
and, In right angled 23. In ∆ ABC, (r2 + r3)cot   =
∆CAD,  2 
a2 [20 April 2019, Shift-II]
AD 2 = − b2 …(ii)
4 (a) a + b + c (b) a (c) b (d) c
From Eqs. (i) and (ii), we get Sol. (b)
 a2 a2 
c 2 + b 2 = 2  − b 2 +  ⇒ c 2 + b 2 = a 2 − 2b 2 Given, in ∆ABC
 4 4
B + c  ∆ ∆ 
(r2 + r3) cot 
A
⇒ a 2 = 3b 2 + c 2 = +  tan
…(iii)  2   s − b s − c 2
Now, 3ca cos A cos C + 2a 2
∆ (s − c + s − b) ∆
= = 2s − b − c = a
 b2 + c 2 − a2   a2 + b2 − c 2  (s − b)(s − c) s(s − a)
= 3ca     + 2a
2

 2bc   2ab  Hence, option (b) is correct.


160 AP EAMCET Chapterwise Mathematics

24. 24area (in square units) of ∆ABC if ∠A = 75°, B C s(s − b) s(s − c)


Now, cot cot = ⋅
2 2 (s − a) (s − c) (s − a) (s − b)
∠B = 45 ° and a = 2( 3 + 1) is
[21 April 2019, Shift-I]  A s(s − a) 
Q cot = 
(a) 6 (b) 2 3  2 (s − b) (s − c) 
(c) 6 − 2 3 (d) 6 + 2 3 Put the value of s, we get
Sol. (d) 2a(2a − b) 2a(2a − c)
= ⋅
Given, ∠A = 75°, ∠B = 45° and a = 2( 3 + 1) (2a − a) (2a − c) (2a − a) (2a − b)
x x 2a(2a − b) 2a(2a − c)
In ∆AOC, tan 60° = ⇒ 3= = × = 4=2
y y a(2a − c) a(2a − b)
A
2 r2 r3
26. In a ∆ABC, if = r3 − r1 , then
r2 − r1
45º 30º r1 (r2 + r3)
=
x r1 r2 + r2 r3 + r3 r1 [21 April 2019, Shift-I]
a2 + b 2 + c 2
(a) (b) b − c
∆2
60º 1
B x O y C (c) (d) 2R
2R
2 (√3+1) Sol. (d)
⇒ x= 2r2r3
3y Given, = r3 − r1
r2 − r1
Now, x + y = 2( 3 + 1)
⇒ 3y + y = 2( 3 + 1) ⇒ 2r2r3 = (r2 − r1) (r3 − r1)
∆ ∆  ∆ ∆  ∆ ∆ 
⇒ y( 3 + 1) = 2( 3 + 1) ⇒ 2⋅ ⋅ = −  − 
(s − b) (s − c)  s − b s − a   s − c s − a 
⇒ y=2⇒ x=2 3
Now, area of ∆ABC 2∆2  s −a−s + b s − a − s + c 
⇒ = ∆2   
= area of ∆AOB + area of ∆AOC (s − b)(s − c) (s − b)(s − a)  (s − c) (s − a) 
1 1 1 (b − a) (c − a)
= × x × x + × x × y = x[x + y] ⇒
2
= ×
2 2 2 (s − b) (s − c) (s − b) (s − a) (s − c) (s − a)
1
= × 2 3 × 2( 3 + 1) ⇒ 2(s − a)2 = (b − a) (c − a)
2
= 2 3 ( 3 + 1) = 6 + 2 3 sq units 2(b + c − a)2
⇒ = (b − a) (c − a)
4
B C
25. In a ∆ABC, if 3a = b + c, then cot cot = ⇒ b 2 + c 2 + a 2 + 2bc − 2ca − 2ab = 2 [bc − ba − ac + a 2]
2 2
[21 April 2019, Shift-I] ⇒ b 2 + c 2 + a 2 + 2bc − 2ca − 2ab
1 1 = 2bc − 2ba − 2ac + 2a 2
(a) 1 (b) 2 (c) (d)
3 2 ⇒ b 2 + c 2 + a 2 = 2a 2 ⇒ a 2 = b 2 + c 2
Sol. (b) ∆
× ∆ ⋅ 
1 1 
+ 
Given, 3a = b + c … (i) r1 (r2 + r3) s − a  s − b s − c
Now, =
Let a , b , c are the sides of the ∆ABC and s is the r1 r2 + r2r3 + r3r1 s
semi-perimeter [Q r1 r2 + r2r3 + r3r1 = s 2]
a+ b+ c
s= ∆2(2s − b − c)
2 =
s(s − a) (s − a) (s − c)
Put the value of Eq. (i), we get
a + 3a 4a ∆2(a + b + c − b − c)
s= = = 2a = = a = 2R
2 2 ∆2
Properties of Triangles 161

27. In ∆ABC, if a : b : c = 3 : 5 : 7, then, 29. Let a triangle ABC be inscribed in a circle of


cos A + cos B = [21 April 2019, Shift-II] radius 2 units. If the 3 bisectors of the angles
13 11 12 10 A , B and C are extended to cut the circle at
(a) (b) (c) (d)
7 7 7 7 A1 , B1 and C1 respectively, then the value of
2
 A B C
Sol. (c) AA
 1 cos + BB cos + CC cos
2 =
1 1
Let a = 3x, b = 5x and c = 7 x 2 2
 
b2 + c 2 − a2 a2 + c 2 − b2  sin A + sin B + sin C 
∴cos A + cos B = +  
2bc 2ac
[21 April 2019, Shift-II]
(5x)2 + (7 x)2 − (3x)2 (3x)2 + (7 x)2 − (5x)2
= + (a) 4 (b) 16
2(5x)(7 x) 2(3x)(7x)
(c) 25 (d) 1
25x 2 + 49 x 2 − 9 x 2 9 x 2 + 49 x 2 − 25x 2
= + Sol. (b)
70 x 2 42x 2 A
2 2
65x 33x 65 33
= + = + C1 B1
70 x 2 42x 2 70 42
13 11 24 12
= + = =
14 14 14 7

28. If ABCD is a cyclic quadrilateral with AB = 6, B C


BC = 4, CD = 5, DA = 3 and ∠ABC = θ, then
cosθ = [21 April 2019, Shift-II] A1

(a)
3
(b)
18
(c)
16
(d)
78 Let the ∆ABC is equilateral.
13 76 78 86 ⇒ A = B = C = 60°
Sol. (a) D 5 C and AA1 = BB1 = CC1 = Diameter = 4
2
 AA cos A + BB cos B + CC cos C 
180°–θ  1 2
1 1
∴  2 2
3 4 
 sin A + sin B + sin C 
θ
 
2
A 6 B  (4) cos 30° + 4 cos 30° + 4 cos 30° 
= 
In ∆ABC,  sin 60° + sin 60° + sin 60° 
AC 2 = AB2 + BC 2 − 2(AB)(AC) cosθ 2
  3 3 3   3
2
[By Cosine rule]  4 + +  4×3 
 2 2 2  
⇒ AC 2 = 62 + 42 − 2(6)(4) cosθ = = 2  = (4)2 = 16
 3 3   3 3 
⇒ AC 2 = 36 + 16 − 48 cosθ    2 
 2 
⇒ AC 2 = 52 − 48 cosθ …(i)
Now, In ∆ADC 30. In ∆ABC, if a , b and c are in arithmetic
AC 2 = AD 2 + DC 2 − 2(AD)(DC) cos(180° − θ) progression, then cos A + 2 cos B + cos C =
= 3 + 5 + 2(3)(5) cosθ
2 2
[By Cosine rule] [22 April 2019, Shift-I]
(a) 1 (b) 2
AC = 9 + 25 + 30 cosθ
2
3
(c) (d) 3 + 1
AC 2 = 34 + 30 cosθ …(ii) 2
By Eqs. (i) and (ii), we get Sol. (b)
52 − 48 cosθ = 34 + 30 cosθ It is given, that in ∆ABC, a , b , c are in AP
⇒ 52 − 34 = 48 cosθ + 30 cosθ So, 2b = a + c …(i)
⇒ 18 = 78 cosθ Now, cos A + 2cos B + 2cos C
18 3 b2 + c 2 − a2 a2 + c 2 − b2 a2 + b2 − c 2
⇒ cosθ = = = + 2 +
78 13 2bc 2ac 2ab
162 AP EAMCET Chapterwise Mathematics

ab 2 + ac 2 − a 3 + 2a 2b + 2c 2b − 2b 3 + a 2c + b 2c − c 3 ∆
= Q r1 =
2abc s −a
ab 2 + ac 2 − a 3 + 2b(a 2 + c 2) − 2b 3 + a 2c + b 2c − c 3 1 s−a
= ∴ =
2abc r1 ∆
1 s − b 1 s −c
ab 2 + ac 2 − a 3 + (a + c) (a 2 + c 2) Similarly, = =
r2 ∆ r3 ∆
− 2b 3 + a 2c + b 2c − c 3
= 1 s
2abc and =
r ∆
ab 2 + ac 2 − a 3 + a 3 + ac 2 + ca 2 + c 3
1 1 1 1
− 2b 3 + a 2c + b 2c − c 3 Now, + + +
= r12 r22 r32 r 2
2abc
(s − a)2 (s − b)2 (s − c)2 s 2
ab 2 + 2ac 2 + 2ca 2 + b 2c − 2b 3 = + + + 2
= ∆2 ∆2 ∆2 ∆
2abc
b 2(a + c) + 2ac(a + c) − 2b 3 s 2 + a 2 − 2 as + s 2 + b 2 − 2 sb + s 2 + c 2 − 2sc + s 2
= =
2abc ∆2
b 2(2b) + 2ac(2b) − 2b 3 4s + a + b + c − 2s(a + b + c)
2 2 2 2
= =2 =
2abc ∆2
4s + a + b + c 2 − 4s 2 a 2 + b 2 + c 2
2 2 2
31. If the area of triangle ABC is b2 − (c − a)2, = =
∆2 ∆2
then tan B = [22 April 2019, Shift-I]
13 1 8 33. In a triangle ABC, if the angle A = 60 °, then
(a) 1 (b) (c) (d)
15 4 15 1 1
+ =
Sol. (d) a+b a+c [22 April 2019, Shift-II]
Given, Area of a triangle = b 2 − (c − a)2 3(1 + b − c ) 2
(a) (b)
∆ = b − (c − a)
2 2 a+ b+c a+ b+c
3 a+ b+c
= (b + c − a)(b − c + a) (c) (d)
a+ b+c 3a2
= (2s − 2a) (2s − 2c)
= 4(s − a)(s − c) Sol. (c)
Now s(s − a) (s − b)(s − c) = 4(s − a)(s − c) Since A = 60°, we have,
⇒ s(s − b) = 4 (s − a)(s − c) a 2 = b 2 + c 2 − 2bc cos A = b 2 + c 2 − 2bc cos 60°
(s − a)(s − c) ⇒ a 2 = b 2 + c 2 − bc …(i)
⇒ = tan 
1 1 B
⇒ =
4 s(s − b) 4  2 1 1 3
Let + = [By option (c)]
a+ b a+ c a+ b+ c
2tan 
B 1 1
2× 2a + b + c
 2 4 ⇒ =
3
∴ tan B = = = 2
(a + b) (a + c) (a + b + c)
1 − tan2   1 −
B 1 15
 2 16 16 ⇒ (2a + b + c) (a + b + c) = 3(a + b) (a + c)
1 16 8 ⇒ (b + c)2 + 2a 2 + 3a(b + c) = 3(a + b) (a + c)
= × =
2 15 15
⇒ b 2 + c 2 + 2bc + 2a 2 + 3ab + 3ac
1 1 1 1
32. In ∆ABC, + + + = = 3(a 2 + ac + ab + bc)
r12 r22 r32 r2
[22 April 2019, Shift-I]
⇒ b 2 + c 2 + 2bc + 2a 2 + 3ab + 3ac
a2 + b 2 + c 2 a+b+c = 3a 2 + 3ac + 3ab + 3bc
(a) (b)
∆ 2
∆ 2
⇒ a = b 2 + c 2 − bc
2
2 2
s 4s Which is the same as Eq. (i).
(c) (d)
∆2 ∆2 Hence,
1
+
1
=
3
Sol. (a) a+ b a+ c a+ b+ c
Properties of Triangles 163

34. In a ∆ABC, 8 R 3 Σ sin 3 A cos(B − C) = sin2 A + sin2 B


= sin (A − B)
[22 April 2019, Shift-II] sin (A + B) sin (A − B)
(a) abc (b) 4abc (c) 3R∆ (d) 12 R∆ [Qsin2 A − sin2 B = sin (A + B) sin (A − B)]
Sol. (d) sin2 A + cos2 A
=
Taking equilateral triangle sin (π − C)
A = B = C = 60° [Q A + B + C = π and C = π /2 given]
 3  3  3  3  3  3  1
= =1 [Q C = 90°]
= 8R3 ×    +   +    sin C
 2   2  2 
  Hence, option (b) is correct.
  3 
3
= 8R ×  3 ×   
3 37. a , b, c are the sides of a scalene triangle ABC.
  2  If angles α, β, γ lie between 0 and π such that
 
3× 3 2 a b c
= 8R3 × = R3 × 3 × 3 2 = 12R∆ cos α = , cos β = and cos γ = ,
8 b+ c c+a a+b
α β γ
35. In a ∆ABC, if a : b : c = 4 : 5 : 6, then the ratio then tan 2 + tan 2 + tan 2 =
2 2 2
of the radius of the circumcircle to the radius
[23 April 2019, Shift-I]
of the incircle is [22 April 2019, Shift-II]
1 3
(a) 13 : 7 (b) 15 : 7 (c) 16 : 7 (d) 17 : 9 (a) (b) 2 (c) 1 (d)
3 2
Sol. (c) Sol. (c)
Let a = 4k, b = 5k, c = 6k a
15 7 5 3 It is given cos α =
s = k, s − a = k, s − b = k, s − c = k b+ c
2 2 2 2 α
 K
4 1 − tan2
∆ = 15 × 7 × 5 × 3 ×  
2
⇒ 2 = a
 2 2α b+ c
2 1 + tan
∆ = 15 7  
K 2

 2 On applying componendo and dividendo law,
2 we get
15 7  
K
2 tan2 α /2 b + c − a
∆  2 K =
r= = = 7 2 b+ c+ a
s 15 2
K
b+ c−a
2 ⇒ tan α /2 =
2

abc 4 ⋅ 5⋅ 6K 3 8 a+ b+ c
R= = = K
4∆ K 2
7 β c+ a−b
4 ⋅15⋅ 7 Similarly, tan2 =
4 2 a+ b+ c
R 8 7 16 γ a+ b − c
∴ = : = and tan2 =
r 7 2 7 2 a+ b+ c
α β γ a+b+ c
36. If ABC is a right angled triangle with 90° at C So,tan2 + tan2 + tan2 = =1
2 2 2 a + b +c
a 2 + b2
and a > b, then sin (A − B) = Hence, option (c) is correct.
a 2 − b2
[23 April 2019, Shift-I] 38. In triangle ABC, if r1 = 36 , r2 = 18 and r3 = 12,
(a)
3
(b) 1 (c)
1
(d) 0 then a + b = [23 April 2019, Shift-I]
2 2 (a) 36 (b) 24 (c) 30 (d) 54
Sol. (b) Sol. (d)
On applying sine rule, we get It is given in a ∆ABC
sin2 A + sin2 B ∆ ∆ ∆
sin (A − B) r1 = = 36, r2 = = 18 and r3 = = 12
sin2 A − sin2 B s−a s−b s−c
164 AP EAMCET Chapterwise Mathematics

1 1 1 1 1 1 1 + 2+ 3 1 ⇒ ∠AMC + 90° + ∠C = 180°


⇒ + + = + + = =
r1 r2 r3 36 18 12 36 6 ⇒ ∠AMC = 90° − ∠C …(iii)
s−a+ s−b+ s−c 1 Again at point M.
⇒ =
∆ 6 ∠BMP + ∠PMA + ∠AMC = 180°
s 1 ⇒ ∠BMP + 90° + 90° − ∠C = 180°
⇒ = ⇒ ∆ = 6s
∆ 6 ⇒ ∠BMP = ∠C …(iv)
∆ 6s
So, r1 = = = 36 ⇒ s = 6s − 6a ⇒ 6a = 5s But in ∆BPM,
s−a s−a
∆ 6s ∠BPM + ∠BMP + ∠PBM = 180°
r2 = = = 18 ⇒ s = 3s − 3b ⇒ 3b = 2s
s−b s−b ⇒ ∠BPM + ∠C + ∠B = 180° …(v)
∆ 6s [From Eqs. (i) and (iv)]
and r3 = = = 12 ⇒ s = 2s − 2c ⇒ 2c = s
s−c s−c ∠BPM = ∠A …(vi)
So, ∆2 = 36s 2 Now, it is evident that ABC and MPB are
⇒ s(s − a) (s − b) (s − c) = 36s 2 equivalent triangles.
BM BP PM 1
⇒ = = =
⇒  s −   s −   s −  = 36s
5s 2s s …(vii)
 BC AB AC 2
6  3  2
Also,
3  1  1  1 
⇒ s       = 36s ∠BAC = ∠BAM + ∠MAC
 6   3  2
⇒ ∠BAM = ∠BAC − ∠MAC
⇒ s 2 = (36)2 ⇒ s = 36
⇒ ∠BAM = ∠A − 90°
∴ a = 30 and b = 24 In ∆APM,
So, a + b = 30 + 24 = 54 ∠PAM + ∠APM + ∠AMP = 180°
Hence, option (d) is correct. ⇒ ∠A − 90° + ∠APM + 90° = 180°
39. If the median of a ∆ABC through A is ⇒ ∠APM = 180° − A ⇒ tan(∠APM) =
AM
tan A PM
perpendicular to AC, then = AM AM
tan C ⇒ tan (180° − A) = ⇒ − tan A =
[22 April 2018, Shift-I] PM PM
1 2 ⇒ AM = − PM tan A …(viii)
(a) 1 + 2 (b) − + 1 (c) −2 (d) 1 +
AM
3 3 Now, in ∆ACM, tan C =
AC
Sol. (c)
B ⇒ AM = AC tan C …(ix)
[From Eqs. (viii) and (ix),]
M AM = AC tan C = − PM tan A
P
⇒ PM tan A + AC tan C = 0 … (x)
MP 1
⇒ = ⇒ AC = 2MP
AC 2
A C Hence, PM tan A + 2PM tan C = 0
ABC is the triangle while AM is the median AC ⇒ PM(tan A + 2tan C) = 0
and AM are perpendicular. ⇒ tan A + 2tan C = 0
⇒ ∠MAC = 90° tan A
⇒ tan A = − 2tan C ⇒ =−2
Since, AM is the median, M is the mid-point of tan C
line BC.
⇒ BM = CM = 2BC A B
…(i) 40. In ∆ABC, tan + tan =
Draw a line perpendicular to AM (through M) 2 2
let it intersect the line AB at P. [22 April 2018, Shift-I]
In ∆ABC, C C C C
c cot 2c cot 2c tan c tan
∠A + ∠B + ∠C = 180° …(ii) (a) 2 (b) 2 (c) 2 (d) 2
In ∆AMC, 4s a+ b+c s a+ b+c
∠AMC + ∠CAM + ∠MCA = 180° Sol. (b)
Properties of Triangles 165

 tan A + tan B  = ∆ ∆ II. In ∆ABC , if a = 3, b = 4, c = 6, then ABC is


  +
 2 2  s(s − a) s(s − b) acute angled triangle.
∆  1 1  c∆ Which of the above statements is/are true?
=  + =
s  s − a s − b  s(s − a) (s − b) [22 April 2018, Shift-II]
2c∆ (a) Only I (b) Only II
= (c) Both I and II (d) Neither I nor II
(a + b + c) (s − a) (s − b)
C Sol. (a)
2 c cot
2c s(s − c) 2 Given statements,
= =
(a + b + c) (s − a) (s − b) a + b + c I. In ∆ABC, if c = 6 and cos C = −
11
.
25
41. In a ∆ABC, D, E and F respectively are the
points of contact of the incircle with the 121 625 − 121
sides AB, BC and CA such that AD = α, BE = β Then, sin C = 1 − =
625 625
αβγ
and CF = γ, then = 504 6 14
α +β + γ = =
25 25
[22 April 2018, Shift-I]
c 6 25
(a) R 2 (b) 2R Q = 2R ⇒ R = =
sin C 6 14 2 14
(c) 2r (d) r 2 2×
25
Sol. (d) A So, statement (I) is true.
α α II. In ∆ABC, if a = 3, b = 4, c = 6, then
a 2 + b 2 − c 2 9 + 16 − 36 11
cos C = = =−
2ab 2× 3× 4 24
β r r
γ Q cos C < 0
O
r
∴ ∆ABC is a obtuse angled triangle.
B β γ C So, Statement (II) is false.
From figure
43. In ∆ABC, if a = 3, b = 4 , c = 6, then
ar(∆ABC) = ar(∆AOB) + ar(∆BOC) + ar(∆COA)
A B C

1 1 1
S = cr + ar + br [where ar(∆ABC) = S] cot + cot + cot
2 2 2 2 2 2 =
1 cot A + cot B + cot C [22 April 2018, Shift-II]
⇒ S = r(a + b + c)
2 13 169 61 61
(a) (b) (c) (d)
1 61 61 169 13
⇒ S = r(2s) [where s is semi perimeter]
2 Sol. (b)
S C s(s − a)
⇒ S = rs ⇒ r = cot
A B
+ cot + cot =
s 2 2 2 ∆
S2 s(s − b) s(s − c)
⇒ r = 2
2
+ +
s ∆ ∆
s(s − a) (s − b) (s − c) S
= [3S − (a + b + c)]
⇒ r = 2
s2 ∆
α ⋅β ⋅ γ S S 2 (a + b + c)2
⇒ r2 = [Q 2s = 2α + 2β + 2γ] = (S) = =
α+β+ γ ∆ ∆ 4∆
cos A cos B cos C
42. Consider the following statements. Now, cot A + cot B + cot C = + +
sin A sin B sin C
−11 b 2+ c 2 − a 2 c 2 + a 2 − b 2 a 2 + b 2 − c 2
I. In ∆ABC, if c = 6 and cosC = , then = + +
25 2bc sin A 2ac sin B 2ab sin C
25
R= b2 + c 2 − a2 c 2 + a2 − b2 a2 + b2 − c 2
2 14 = + +
4∆ 4∆ 4∆
166 AP EAMCET Chapterwise Mathematics

a2 + b2 + c 2 b2 + c 2 − a2 6 + 3 + 1 + 2 3 − 4
= Now, cos A = =
4∆ 2bc 2 6( 3 + 1)
A B
cot + cot + cot
C 6+ 2 3 3+ 3 3( 3 + 1)
= = =
2 = (a + b + c)
2
∴ 2 2 2 6( 3 + 1) 6 ( 3 + 1) 6( 3 + 1)
cot A + cot B + cot C a + b + c2
2 2
1
cos A =
(3 + 4 + 6)2 169 2
= = .
9 + 16 + 36 61 2
sin A = 1 −   =
1 1

 2 2
44. If the reciprocals of the lengths of the sides Now, sin2 C − sin2 A
of a ∆ ABC are in harmonic progression, then  1 3 
2
2
−   = +
1 1 3 1 3
= + − =
its ex-radii r1 , r2 , r3 are in [22 April 2018, Shift-II]  2 4   2
 2 4 2 4
(a) Arithmetic progression
(b) Geometric progression 46. In ∆ABC,
(c) Harmonic progression a 3 ⋅ cos(B − C) + b3 ⋅ cos(C − A) + c 3 ⋅ cos(A − B) =
(d) Arithmetico-geometric progression [23 April 2018, Shift-I]
Sol. (c) (a) a b c (b) a + b + c (c) 2 abc (d) 3 abc
Let the sides of a ∆ABC are a , b and c Sol. (d)
1 1 1 In ∆ABC, a 3 cos (B − C)
Q , , are in H.P. ⇒ a , b , c are in A.P.
a b c  2sin (B + C) cos (B − C) 
= a3  
⇒ s − a , s − b , s − c are in A.P.  2 sin (B + C) 
s−a s−b s−c
⇒ , , are in A.P.  sin 2B + sin 2 C) 
∆ ∆ ∆ = a3  
 2 sin (B + C) 
∴r1 , r2 , r3 are in H.P.
 2sin B cos B + 2sin c cos C 
= a3  
45. In ∆ABC if a = 2, b = 6 and c = 3 + 1, then  2 sin (π − A) 
sin 2 C − sin 2 A = [23 April 2018, Shift-I] = a 3 
sin B cos B + sin c cos C 

1+ 3 3 3 3  sin A 
(a) (b) (c) (d) 3  bk cos B + cK cos C 
4 2 4 4 =a  
 ak 
Sol. (c) ∴ a 3 cos (B − C) = a 2 b cos B + a 2 c cos C … (i)
In ∆ABC, a = 2, b = 6, c = 3+1
Similarly,
We know that,
b 3 cos (C − A) = b 2c cos C + b 2a cos A … (ii)
a 2 + b 2 − c 2 (2)2 + ( 6)2 − ( 3 + 1)2
cos C = = and c 3 cos (A − B) = c 2a cos A + c 2b cos B … (iii)
2ab 2⋅ 2⋅ 6
4 + 6 − 3−1 − 2 3 Adding Eqs. (i), (ii) and (iii), we get
= a 3 cos (B − C) + b 3 cos (C − A) + c 3 cos (A − B)
4 6
6 − 2 3 2 3( 3 − 1) = a 2b cos B + a 2c cos C + b 2c cos C
= =
4 6 4 6 + b 2a cos A + c 2a cos A + c 2b cos B
3 −1 = ab (a cos B + b cos A) + ac (a cos C + c cos A)
cos C =
2 2 + bc (b cos C + c cos B)
 3 − 1
2 = abc + abc + abc = 3abc
∴ sin C = 1 −   [Q sinθ = 1 − cos2 θ] r1 − r r2 − r r3 − r
 2 2  47. In ∆ABC, + + =
a b c
8 − 3−1 + 2 3 4+ 2 3 [23 April 2018, Shift-I]
= =
8 8 r + r + r3 r1 + r2 + r3
(a) 1 2 (b)
2+ 3 1 3 s 2s
= = + r1 + r2 + r3 r1 + r2 + r3
4 2 4 (c) (d)
2 3s
Properties of Triangles 167

Sol. (a) 30k2 24k2 20k2


= + +
r − r r2 − r r3 − r 90k 2
99k 2
110k2
In ∆ABC, 1 + +
a b c 1 8 2 11 + 8 + 6 25
= + + = = .
∆ ∆ ∆ ∆ ∆ ∆ 3 33 11 33 33
− − −
s−a s s−b s s−c s
= + +
a b c 50. If I is the incentre of ∆ABC and P1 , P2 , P3 are
 ∆ ∆ ∆ ∆ respectively the radii of the circumcircles of
∴  r1 = r2 = r3 = and r = 
 s−a s−b s−c s the ∆ IBC, ∆ICA and ∆IAB, then P1 P2 P3 =
∆(s − s + a) ∆(s − s + b) ∆(s − s + c) [23 April 2018, Shift-II]
= + +
a s(s − a) b s(s − b) c s(s − a) (a) 2Rr (b) 2 Rr 2
4R
∆ 1 ∆ 1 ∆ 1 (c) 2 R 2 r (d)
= . + . + . r
s−a s s−b s s−c s
r r r r + r2 + r3 Sol. (c)
= 1 + 2 + 3 = 1 π A
s s s s In ∆IBC, ∠BIC = − , let circumcentre of ∆IBC
2 2
C A 3b π
48. In ∆ABC, if a cos 2 + c cos 2 = , then is C1 , then ∠BC1 C = +
A
2 2 2 2 2
[23 April 2018, Shift-II] π
= sin +  = cos
a/2 A A
(a) 2b = a + c (b) b 2 = ac So,
P1 2 2 2
1 1 1
(c) = + (d) a = c a
b a c ⇒ P1 =
A
Sol. (a) 2cos
2
In a ∆ABC, if b c
C A 3b Similarly, P2 = and P3 =
a cos2 + c cos2 = 2cos
B
2cos
C
2 2 2 2 2
s(s − c) s(s − a) 3b
⇒ a + c = abc
ab bc 2 So, P1 P2P3 =
A B C
s 3b s 3b 8 cos cos cos
⇒ (2s − a − c) = ⇒ (b) = 2 2 2
b 2 b 2
A B C
a+ b+ c abc × sin sin sin
⇒ =3 ⇒ 2b = a + c. 2 2 2
b =
sin Asin Bsin C
49. If s is the semi-perimeter of ∆ABC and if r
s−a s−b s−c  A = (2R) (2R) (2R) = 2R2r.
= = , then ∑ sin 2   = 4R
4 5 6  2
s −a s − b s − c
[23 April 2018, Shift-II] 51. In ∆ABC , if = = , then
11 12 13
74 25 74 25
(a) (b) (c) (d)  A C 
25 74 33 33 tan 2   + tan 2   =
 2  2 [24 April 2018, Shift-I]
Sol. (d) 290 290
s−a s−b s−c (a) (b)
Let = = =k 429 143
4 5 6 143 113
(c) (d)
⇒ b + c − a = 8k, a + c − b = 10k 33 33
and a + b − c = 12k
Sol. (a)
So, a + b + c = 30k ⇒ s = 15k, a = 11k, b = 10k and s−a s−b s−c
c = 9k Given that, = = =k
11 12 13
(s − b) (s − c) (s − c) (s − a)
Now, Σsin2   =
A
+ s − a = 11k
 2 bc ca
s − b = 12 k
(s − a) (s − b)
+ s − c = 13k
ab
168 AP EAMCET Chapterwise Mathematics

= a 2 + b 2 − 2ab  cos2 − sin2 


On adding, C C
3s − (a + b + c) = 36k  2 2
3s − 2s = 36k [as a + b + c = 2s] = a 2 + b 2 − 2ab cos C
s = 36 k
 a2 + b2 − c 2 
A C = a 2 + b 2 − 2ab  
Now, tan + tan2
2
 2ab 
2 2
(s − b) (s − c) (s − a) (s − b) ⇒ a2 + b2 − a2 − b2 + c 2 = c 2
= +
s(s − a) s(s − c)
(12k) (13k) (11k) (12k) 12 13 11  53. In a triangle, if the ex-radii r1 , r2 , r3 are in the
= + = +
(36k) (11k) (36k) (13k) 36 11 13 ratio 1 : 2 : 3, then its sides are in the ratio
[24 April 2018, Shift-I]
1 169 + 121  290
= ⋅ = (a) 5 : 8 : 9 (b) 5 : 4 : 3
3  11 × 13  429 (c) 7 : 9 : 11 (d) 1 : 2 : 3
52. In ∆ABC, Sol. (a)
 A + B 2 C  Given that, r1 , r2, r3 are ex-radii of triangle and
(a − b) sin 2
 + (a + b) sin   =
2 2
 2   2 r1 : r2 : r3 = 1 : 2 : 3
[24 April 2018, Shift-I] r1 = x, r2 = 2x, r3 = 3x
∆ ∆
(a) b 2 (b) a2 s − a = ⇒s − a = … (i)
(c) c 2 (d) a2 + b 2 − c 2 r1 x
∆ ∆
Sol. (c) s − b = ⇒s − b = … (ii)
r2 2x
In ∆ABC, A + B + C = π
∆ ∆
A+ B= π−C s − c = ⇒s − c = … (iii)
r3 3x
A + B
= (a − b)2 sin2   + (a + b) sin
2 2C
 2  2 On adding Eqs. (i), (ii) and (iii), we get
∆ ∆ ∆
π − 3s − (a + b + c) = + +
= [a 2 + b 2 − 2ab]sin2  
C
 x 2x 3x
 2 
11∆
C s=
+ [a 2 + b 2 + 2ab]sin2 6x
2
C C From Eqs. (i), (ii), (iii), we get
= (a 2 + b 2) cos2 − 2ab cos2 5∆ 8∆ 9∆
2 2 a= ,b= ,c=
2C C 6x 6x 6x
+ (a + b )sin
2 2
+ 2ab sin2
2 2 So, a : b : c = 5 : 8 : 9.
15
Inverse Trigonometric
Functions
1. The domain of is, x ∈ (−1, 1) both values −1 and 1 are excluded
 x − 3 Now, cosπ = −1
f (x) = cos −1   − log 10 (4 − x) is …………
 2  and cos 2π = 1
[17 Sep. 2020, Shift-I] Also, cos− 2π = 1
(a) (1, 4) (b) [1, 4) (c) (1, 4] (d) [1, 4] Hence, sin−1 (cos x) is non differentiable at π, 2π
and −2π.
Sol. (b)
x − 3 1 + cos x 
Let f (x) = cos−1   − log10 (4 − x) 4. Let f (x) = tan − 1  ;
 2   sin x 
For domain of log10 (4 − x) ⇒ 4 − x > 0 ⇒ x < 4  sin x 
x − 3 x−3 g(x) = tan − 1   , then
For domain of cos−1   ⇒ −1 ≤ ≤1 1 − cos x 
 2  2
⇒ 1≤ x ≤ 5 ∫ ( f (x) + g(x))dx = [18 Sep. 2020, Shift-II]
Now domain of f (x) will be x ∈[1, 4) πx x2 x2 x2 x2
(a) − (b) πx − (c) πx + (d) πx +
 π π 2 4 2 4 2
2. If θ ∈ − ,  , then cos −1 (sin θ) is equal to
 2 2 Sol. (b)
[17 Sep. 2020, Shift-I]  2cos2 x 
π π π θ f (x) = tan −1  1 + cos x  = tan−1  2 
  
(a) −θ (b) θ − (c) +θ (d) π +  sin x  x x
2 2 2 2  2sin cos 
 2 2
Sol. (a) −1   π x π x
= tan−1  cot
x
π π  = tan  tan −   = −
θ ∈  − ,  , cos−1 (sinθ) = ?  2    2 2  2 2
 2 2 
−1  sin x 
π π and g(x) = tan  
⇒ cos−1  cos − θ  =  − θ  1 − cos x 
  2    2 
 2sin x / 2 cos x / 2 −1  x
= tan−1   = tan  cot 
3. The function y = sin − 1(cos x) is not  2sin2 x / 2   2
π x π x
differentiable at ......... [17 Sep. 2020, Shift-II] = tan−1  tan −   = −
  2 2   2 2
(a) π only (b) 2 π only
(c) − 2 π only (d) All options are correct x2
Sol. (d)
∴ ∫ ( f (x) + g(x)) dx = ∫ (π − x) dx = π x − 2
+c

Domain for sin−1 x Hence, option (b) is correct.


170 AP EAMCET Chapterwise Mathematics

5. If θ = cot −1(7) + cot −1(8) + cot −1(18), then cotθ ⇒ 625 − 625x 2 = 0 ⇒ x = ± 1
is equal to [22 Sep. 2020, Shift-I] So, sum of the values of ‘x’
(a) 2 (b) 3 = 0 + 1 −1 = 0
(c) 4 (d) 1 Hence, option (a) is correct.
Sol. (b) 7. If α and β are the least and the greatest
It is given that, values of f (x) = (sin − 1 x)2 + (cos − 1 x)2 for all
θ = cot − 1 (7) + cot − 1 (8) + cot − 1 (18)
x ∈ R respectively, then 8(α + β) =
= tan− 1   + tan− 1   + tan− 1  
1 1 1 [21 April 2019, Shift-I]
 7  8  18  (a) π 2 (b) 11 π 2
 1 1  (c) 9 π 2 (d) 25 π 2
 +  1
= tan− 1  7 8  + tan− 1 Sol. (b)
1 − 1  18
  f (x) = (sin− 1 x)2 + (cos− 1 x)2
 7 × 8
−1
 3 1  Let sin x = a and cos− 1 x = b
 + 
− 1  15  −1 1  −1 Then, f (x) = a 2 + b 2 = (a + b)2 − 2ab
= tan   + tan   = tan  11 18 
 55  18  1 − 3  Put the value of a and b
 
 11 × 18  f (x) = (sin− 1 x + cos− 1 x)2 − 2sin− 1 x cos− 1 x
54 + 11 
= tan− 1  − 1 1 
 = tan   =
π2
− 2sin− 1 x cos− 1 x
 195   3 4
1
⇒ tan θ = ⇒ cot θ = 3 [Q sin− 1 x + cos− 1 x = π /2]
3 π2 π
Hence, option (b) is correct. = − 2sin− 1 x  − sin− 1 x 
4 2 
6. The sum of the values of x satisfying the π2
= − π sin− 1 x + 2(sin− 1 x)2
3x   4 x 4
equation sin −1   + sin −1   = sin −1 (x), is
 5  5  For minimum and maximum value,
1 1
[20 April 2019, Shift-II] f ′(x) = 0 − π ⋅ + 4sin− 1 x =0
1 1 1 1− x 2
1 − x2
(a) 0 (b) (c) (d)
2 3 4 1
= [4sin− 1 x − π] = 0 = sin− 1 x = π / 4
Sol. (a) 1 − x2
Given, equation 1
x = sin π / 4 =
sin−1   + sin−1
3x 4x
= sin−1 x , x ∈ [−1, 1] 2
 5 5  1 
Therefore, f ′ ′   = + ve
 3x 16 x 2 4x 9x2   2
∴sin−1  1− + 1−  = sin−1 x
 5 25 5 25  f (x)min = when x = 1 / 2
 
π2
3x 16 x 2 4x 9x2 ∴ f (x)min =− 2sin− 1 x cos− 1 x
⇒ 1− + 1− = x 4
5 25 5 25 π2 π π π2
Either x = 0 ∴ f (x)min = − 2⋅ ⋅ ⇒ α =
4 4 4 8
3 25 − 16 x 2 4 25 − 9 x 2 π2
or + =1 f (x)max = −1
− 2sin x cos x −1
25 25 4
⇒ 9(25 − 16 x 2) + 16(25 − 9 x 2) + 24 25 − 16 x 2 We can see that, f (x) is maximum, when x = − 1
π2 π
25 − 9 x 2 = 625 f (x)max = − 2 −  (π)
4  2
⇒ 24 25 − 16 x 2 25 − 9 x 2 = 288 x 2 π2 5π 2 5π 2
β= + π2 = ⇒β=
⇒(25 − 16 x ) (25 − 9 x ) = 144 x
2 2 4
4 4 4
Inverse Trigonometric Functions 171

 π 2 5π 2  We know that,
Hence, 8(α + β) = 8 + π π 3π
 8 4  0 ≤ cos−1 x ≤ π ⇒ − ≤ cos−1 x − ≤
4 4 4
 π 2 + 10 π 2  2
=8  = 11 π π 9π2
0 ≤  cos−1
2
8 x−  ≤
   4 16
2
  π 9π2
0 ≤ 2 cos−1 x −  ≤
n
1
8. If ∑ tan−1  k 2 + k + 1 = tan−1(θ), then θ =  4 8
k=1 2
π2 π π2 π2
[21 April 2019, Shift-II] ≤ 2 cos−1 x −  + ≤5
8  4  8 4
n n n
(a) (b) (c) 1 (d)
n+2 n+1 n−1 π 2 5π 2 11 π 2
∴Required sum = + =
8 4 8
Sol. (a) There is no option match.
n  1 
Given, ∑ tan−1  2 −1
 = tan θ 10. If x = a is a solution of the equation
k =1  k + k + 1
x 2x
n  1  n
−1 [(k + 1) − k]
sin − 1 + sin − 1 = sin − 1 x, then the roots of
Now, ∑ tan−1  2  = ∑ tan 3 3
k =1  k + k + 1  k =1 1 + k(k + 1)
the equation x 2 − ax − 1 = 0 are
n
= ∑ [tan−1 (k + 1) − tan−1 k] [22 April 2019, Shift-II]
k =1 1
(a) ± 1 (b) , 1
= tan−1 2 − tan−1 1 + tan−1 3 − tan−1 2 + tan−1 4 2
− tan−1 3 + … + tan−1 (n + 1) − tan−1 n (c) ±
1 1
(d) − , 1
2 2
 n + 1 − 1
= tan−1 (n + 1) − tan−1 1 = tan−1   Sol. (a)
1 + n + 1 
x 2x
 n  Given, sin− 1
+ sin− 1 = sin− 1 x
= tan−1  
3 3
 2 + n x 4x2 2x x2 
⇒sin− 1  1− + 1−  = sin− 1 x
 n  n 3 9 
∴tan−1  −1
 = tan θ ⇒θ =  9 3
 2 + n n+ 2
x 9 − x 2 − 1 = 0
1 2
⇒ 9 − 4x2 +
9. The sum of the maximum and the minimum  9 9 
π2 ⇒ 54 = 18 9 − x 2 ⇒ 3 = 9 − x2
values of 2(cos −1 x)2 − π cos −1 x + , is
4 ⇒ 9 = 9 − x2 ⇒ x2 = 0
[22 April 2019, Shift-I]
⇒ x=0 ⇒ a=0
π2 3π2 3π2
(a) (b) (c) (d) 4 π 2 ∴ Equation becomes x 2 − 1 = 0 ⇒ x = ± 1
8 8 2
Hence, roots are ± 1.
Sol. (*)
π2 11. The solution of the equation
Given, 2(cos −1 x)2 − π cos −1 x + π
4 sin −1 x + sin −1 2 x = is
Let cos−1 x = y 3 [23 April 2019, Shift-I]
π2 (a) −
1 3 1 3
∴ 2y 2 − πy + (b)
4 2 7 2 7
π π2
= 2  y 2 − y +
1 2 1 2
(c) (d) −
 2  4 2 7 3 7
 π
2
π2  π2 π
2
π2
= 2  y −  −  + = 2  y −  + Sol. (b)
  4 16  4  4 8 Given equation sin−1 x + sin−1 2x =
π
π
2
π2 3
= 2  cos−1 x −  + Let x = sinθ
 4 8
172 AP EAMCET Chapterwise Mathematics

π
Then, θ + sin−1 (2 sin θ) = 13. Consider the following statements.
3 I. sin −1 (y 2 − 4 y + 6) + cos −1 (y 2 − 4 y + 6)
−1 π π
⇒ sin (2 sin θ) = − θ
3 = ,∀ y ∈R
2
 π  3 1
⇒ 2sin θ = sin  − θ = cos θ − sin θ II. sec −1 (y 2 − 4 y + 6) + cosec −1(y 2 − 4 y + 6)
3  2 2 π
5 3 = ,∀ y ∈R
⇒ sin θ = cos θ 2
2 2
Which of the above statement(s) is/are true?
3
⇒ tanθ = [22 April 2018, Shift-II]
5
(a) Only I (b) Only II
3 1 3
⇒ sinθ = = = x (c) Both I and II (d) Neither I nor II
28 2 7
Hence, option (b) is correct.
Sol. (b)
Given statements,
12. Let x, y be real numbers such that x ≠ y and sin−1 (y 2 − 4 y + 6) + cos−1 (y 2 − 4 y + 6) =
π
,
−1
xy ≠ 1. If ax + bsec(tan x) = c and 2
x+y ∀ y∈R …(i)
−1
ay + bsec(tan y) = c, then = ⇒ sec−1 (y 2 − 4 y+ 6) + cosec−1
1 − xy
π
[22 April 2018, Shift-I] (y 2 − 4 y + 6) = , ∀ y ∈ R …(ii)
2 ab 2 ac 2 ab 2 ac 2
(a) (b) (c) (d) y 2 − 4 y + 6 = (y − 2)2 + 2 ≥ 2 , ∀ y ∈ R.
a2 − b 2 a2 + c 2 a2 + b 2 a2 − c 2 Q
So, statement (i) is false and statement (ii) is
Sol. (d)
true.
Let x = tanθ, then we have
 x  y
a tanθ + b secθ = c 14. If cos − 1   + cos − 1   = θ, then
 2  2
⇒ a sinθ + b = c cosθ
⇒ c cosθ − a sin b = b 9 x 2 − 12 xy cosθ + 4 y 2 = [23 April 2018, Shift-I]
Let a + c =r
2 2 2 (a) 36sin2 θ (b) 37 sin2 θ (c) 39sin2 θ (d) 36cos 2 θ
So, for some α, Sol. (a)
c = r cosα, We have,
a = r sinα
cos−1   + cos−1   = θ
x y
a  2  3
Also, tanα =
c x y  x   y 
2 2

cosα cosθ − sinα sinθ =


b ⇒ cos−1  . − 1 −    1 −     = θ
Thus, 2 3   2    3  
r  
b b
⇒ cos(α + θ) = ⇒ α + θ = ± cos− 1  x  
2 2

1 −    1 −  y
x y
r r ⇒ . −    = cos θ
2 3   2    3 
Let α + θ be the positive solution and α + φ the
negative solution, where  x  
2
y 
2
1 −    1 −   
xy
y = tan φ = − cos θ =
6   2    3 
∴ α + φ = − (α + θ)
⇒ − 2α = θ + φ squaring both sides, we get
2
 x2   y2 
⇒ tan(− 2α) = tan(θ + φ) ⇒  − cos θ = 1 −
xy
 1 − 
− 2tanα  6   4  9
⇒ = tan(θ + φ)
1 − tan2 α x 2 y 2 −12xy cos θ + 36 cos2 x (4 − x 2) (9 − y 2)
⇒ =
− 2a / c x+ y 2ac x+ y 36 36
⇒ = ⇒ 2 = .
1 − a 2 / c 2 1 − xy a − c 2 1 − xy ⇒ x 2 y 2 − 12 xy cos θ + 36 cos2 x
Inverse Trigonometric Functions 173

= 36 − 9 x 2 − 4 y 2 + x 2 y 2 1  1
16. tan −1 2 + cot −1 (− 3) + cot −1 + tan −1  −  =
2
⇒ 9 x − 12xy cos θ + 4 y = 36 (1 − cos2 θ)
2
3  2
⇒ 9 x 2 − 12xy cos θ + 4 y 2 = 36 sin2 θ [24 April 2018, Shift-I]
3π π π 3π
(a) − (b) (c) − (d)
n  2r  2 2 2 2
15. lim ∑ tan –1  4  =
n→ ∞
r =1  r + r + 2
2 Sol. (d)
tan− 1 2 + cot − 1 (− 3) + cot − 1   + tan− 1  − 
1 1
[23 April 2018, Shift-II]
π π −π −π  3  2
(a) (b) (c) (d)
= tan− 1 (2) − tan− 1   + π − cot − 1 (3) + cot − 1  
4 2 4 2 1 1
 2  3
Sol. (a)
= tan− 1 (2) − tan− 1   + π − tan− 1   + tan− 1 (3)
1 1
Since,
 2  3
r 4 + r 2 + 1 = (r 2 − r + 1) (r 2 + r + 1)
 2− 1 
  −1
 2  + π + tan− 1 (3) − tan− 1  1 
2r = tan 
So, tan− 1   1  3
 1 + (r + r + 1)   1 + 2⋅ 
4 2
 2
= tan− 1 (r 2 + r + 1) − tan− 1 (r 2 − r + 1)    3− 1 
n  2r  − 1  3 −1 3 
So, ∑ tan− 1  4 = tan   + π + tan 
 2 1
 r + r + 2    1 + 3⋅ 
r =1 2
 2  3
n
= ∑ {tan− 1 (r 2 + r + 1) − tan− 1 (r 2 − r + 1)}  −1 −1 −1 x − y 
r =1  as tan x − tan y = tan  
  1 + xy  
= tan− 1 (n2 + n + 1) − tan− 1 ()
1
= tan− 1   + π + tan− 1  
3 8
n2 + n n2 + n  4  6
= tan− 1 = tan− 1 2
1+ n + n+1
2
n + n+ 2
= π + tan− 1   + tan− 1  
3 4
−1   4  3
n 2r
So, lim ∑ tan  4 
 r + r + 2
n → ∞ r =1 2
= π + tan− 1   + cot − 1   = π + π /2
3 3
 4  4
 n + n  π 2
= lim tan− 1  2  = .
n→ ∞  n + n + 2 4 = 3π /2 Q tan− 1 x + cot − 1 x = π 
 2 
16
Hyperbolic Functions
1 Sol. (b)
1. If tanh (x) = , then tanh (3 x) is
θ cos h(θ) − 1
3 Since, tan h2   =
[17 Sep. 2020, Shift-I]  2 cos h(θ) + 1
8 7 2 sec x − 1
(a) (b) (c) 1 (d) = (Q cos h θ = sec x given)
9 9 3 sec x + 1
Sol. (b) x
2sin2
1 1 − cos x 2 = tan2 x
tanh(x) = , tanh(3x) = ? = =
3 1 + cos x 2cos2 x 2
−x
e −e
x
1 2
⇒ x = ⇒ 2e x = 4 e − x ⇒ (e 2 x = 2)
e + e−x 3 1
e 3 x − e −3 x (e 2 x )3 − 1 8 − 1 7
4. If tan h − 1 (x + iy) =
and tanh(3 x) = 3 x = = = 2
e + e −3 x (e 2 x )3 + 1 8 + 1 9 −1  2x  i −1  2y 
tan h   + tan  ,
2. The function f (x) =sech(x) on R has the range 1 + x 2 + y 2  2 1 − x 2 − y 2 
....... [17 Sep. 2020, Shift-II] where x, y ∈R, then tan h − 1 (iy) =
(a) (0, ∞ ) (b) (0, 1] [18 Sep. 2020, Shift-II]
(c) [1, ∞ ) (d) (1, ∞ ) (a) i tan h− 1 ( y) (b) − i tan h− 1 ( y)
Sol. (b) (c) i tan− 1 ( y) (d) − i tan− 1 ( y)
f (x) = sec h x Sol. (c)
1 2 We have,
= = x
cos h x e + e−x 1
tan h−1 (x) = tan−1 (ix)
2 2 i
At x = 0, f (x) = = =1
e 0 + e −0 2 Put, x = iy , we get
1 −1
At x ≠ 0, f (x) is less than 1 or e x + e − x is greater tan h−1 (iy) = tan−1 (− y) = tan−1 (y) = i tan−1 (y)
than 2. i i
Also f (x) ≠ 0 at any value of x. Hence, option (c) is correct.
Hence, range sech x 5. Which of the following is true about f (x) = 3
= (0, 1] sinh (x) − 2 cosh (x), ∀ x ∈ R
 θ (a) f is an odd function [22 Sep. 2020, Shift-I]
3. If sec(x) = cos h(θ), then find tanh   2
 2 (b) f is a periodic function
[18 Sep. 2020, Shift-I] (c) f is a strictly increasing function on R
(d) f is a strictly decreasing function on R
(a) sec   (b) tan2  
x
2 x
2 2 Sol. (c)
Given, function, f (x) = 3 sin h(x) − 2 cos h(x),
(c) tanh   (d) sec h  
2x x
2

2 2 ∀ x ∈R
Hyperbolic Functions 175

 ex − e− x   ex + e− x  π  π 
= 3  − 2  8. If θ = and x = log cot  + θ , then
 2   2   
6  4
1 x 5 −x 1 x
= e − e = (e − 5e − x ) sin h(x) = [20 April 2019, Shift-II]
2 2 2 1 1
1 x −x
(a) 3 (b) (c) − 3 (d) −
Q f ′(x) = (e + 5e ) > 0 ∀ x ∈ R 3 3
2
∴The, function f is a strictly increasing function Sol. (c)
π  π 
, x = log cot  + θ
on R.
At θ =
Hence, option (c) is correct. 6   4  
1 + tanh (x / 2)  π π 
6. = ⇒ x = log cot  + 
1 − tanh (x / 2)   4 6  
[23 Sep. 2020, Shift-I]
(a) e − x (b) e x (c) 2e x / 2 (d) 2e − x / 2 cot π / 4 cot π / 6 – 1
⇒ ex =
cot π / 6 + cot π / 4
Sol. (b)
3 −1
1 + tan h  cos h   + sin h   ⇒ ex =
x x x
 2  2  2 3+1
=
1 − tan h  cos h   − sin h   3+1
x x x
∴ e– x =
 2  2  2 3 –1
ex /2 + e−x /2 ex /2 − e−x /2  ex − e−x  3 −1 3+1
+  Q sin hx =  −
= x /2 2 −x /2 2

2
 ex − e−x 3+1 3 −1
e + e ex /2 − e−x /2  ex + e−x Q sin h(x) = =
− Q cos hx =  2 2
2 2  2 
(3 + 1 − 2 3) − (3 + 1 + 2 3)
2. e x / 2 =
x /2 4
= 2 = e = ex /2 + x /2 = ex
−x /2 4 3
2. e e −x /2
=− =− 3
4
2
Hence, option (c) is correct.
Hence, option (b) is correct.
 − π π
7. If sin x cos h y = cos θ , cos x sin h y = sinθ and 9. If x ∈  ,  , then log sec x =
 2 2 [21 April 2019, Shift-I]
4 tan x = 3. Then, sinh2 y = [20 April 2019, Shift-I]
(a) 2 cosech− 1  cot 2 − 1
x
4 9
(a) (b)  2 
5 16
−1
9 16 (b) 2 cosech  cot 2 x
+ 1
(c) (d)  2 
25 25
−1 2 x 
(c) 2 coth  cosec − 1
Sol. (d)  2 
−1 2 x
+ 1
3
Since, tan x = (d) 2 coth  cosec
4  2 
9 16
⇒ sin2 x = and cos2 x = Sol. (c)
25 25
π π
and sin x cosh y = cosθ, cos x sin h y = sinθ For x ∈  − ,  ,
 2 2
Q cos2 θ + sin2 θ = 1
log sec x = y (let)
⇒ (sin x cos h y) 2 + (cos x sin h y) 2 = 1
9 16 ⇒ sec x = ey
⇒ (1 + sin h2 y) + sin h2 y = 1 ⇒ cos x = e− y
25 25
⇒ 9 + 9sin h2 y + 16sin h2 y = 25 sin hy e y − e− y
Q =
16 cos hy e y + e− y
⇒ sin h2 y =
25 cos hy − sin hy e − y
⇒ = y = cos2 x
Hence, option (d) is correct. cos hy + sin hy e
176 AP EAMCET Chapterwise Mathematics

2
 sec h −1 + tan h −1 + sin h −1 
1 1 1
 cos h y − sin h y 
  Now, e  2 2 2

⇒  2 2  = cos2 x
( 5 + 1)
 cos h y + sin h y  = (2 + 3) ⋅ ( 3)
 2 2 2
y y (2 3 + 3)( 5 + 1)
cos h − sin h =
⇒ 2 2 = cos x 2
y y 2 15 + 2 3 + 3 5 + 3
cos h + sin h =
2 2 2
y
2cos h 3 + 3 5 + 2 3 + 2 15
⇒ 2 = 1 + cos x = cot 2 x =
y 1 − cos x 2
2sin h 2
2  π x  y
11. If y = log e tan  +  , then tan h   =

y x
cot h = cot 2 = cosec2 − 1
x  4 2  2
1 2 2 [22 April 2019, Shift-I]
= cot h−1  cosec2 − 1
y x
⇒ (a) cot  
x
(d) tan 
x
2  2  2
(b) tan x (c) cot hx
2
−1 
⇒ y = 2cot h  cosec 2 x
− 1 Sol. (d)
 2 
 −1 1 1 1 Given,
+ tan h−1 + sin h−1 
 sec h π
y = log e tan + 
 2 x
10. e 2 2
=
4 2
[21 April 2019, Shift-II]
  π x   1 + tan x 
2+ 3 3+ 3 5 + 3 15  tan  + tan   
(a)   4 2  2
2 = log e = log e 
 1 − tan π  tan x   x
 1 − tan 
3+2 3+ 3 5 + 2 15      
(b)   4  2  2
2 x
2+ 3 3+ 4 5 + 5 15 1 + tan
2 ⇒ e +1 =
y
(c) 2
2 ⇒ ey =
1 − tan
x e y
−1 2tan
x
2+ 3 3−4 5 + 5 15 2 2
(d)
e y −1
⇒ tan h  = tan 
2 x y x
⇒ y = tan
Sol. (b) e +1 2  2  2
1 + 1 − x 2 
Q sec h−1 x = log e   12. The set of all real values of x for which
 x 
  1+ x 1 + 1 − x 2 
1 + x  f (x) = log e + log e   +

tan h−1 x =
1
log   1− x  x 
2 1 − x 
1 + 1 + x 2 
and sin h−1 x = log(x + x 2 + 1) cot h− 1 x + log e   + sin h− 1 x is

1 + 1 − 1 / 4 
 x 
sec h −1
1 log e  
 
∴ =e
1 /2
e 2 defined is [22 April 2019, Shift-II]
(1 + 3/ 2 ) (a) φ (b) (0, 1)
e
log e 1/2
= elog e ( 2 + 3)
= 2+ 3 (c) (− 1, 1) (d) (0, 1]

1
1 + 1 
 2
Sol. (b)
log  
−1 1 2 1 − 1  We have,
tan h
 2
Similarly, e 2
=e = 3
1+ x 1 + 1 − x 2 
log  +
1 1
+ 1  f (x) = log + log  
sin h −1 1
1− x  x 
and e 2
=e 2 4 
 
log e  +
1 5 1 + 1 + x 2 
2

2  5+1 + cot h−1 x + log   + sin h−1 x
e =  
2  x 
Hyperbolic Functions 177

1+ x θ ex /2 − e−x /2
= tanh  
x
Now, log will be defined when ⇒ tan = x / 2
1− x 2 e + e−x /2  2
θ
= tanh−1  tan 
x
1+ x 1+ x ⇒
> 0⇒ >0 2  2
1− x 1− x
θ
⇒ −1 
x = 2 tanh  tan 
– + –  2
–1 1 Hence, option (b) is correct.
∴ x ∈ (−1, 1) 1
14. tanh −1 + coth −1 3 =
1 + 1 − x  2 2 [22 April 2018, Shift-I]
log   will be defined when (a) log 6 (b) log 6
 x 
  (c) −log 6 (d) −log 6
1 + 1 − x2 Sol. (a)
> 0 and 1 − x 2 > 0 and x ≠ 0
tan h − 1   + cot h − 1 (3)
x 1
⇒ 1 − x > 0, x ≠ 0, x > 0
2
[Q 1 − x > 0]
2  2
1 + 1   3
⇒ (1 − x)(1 + x) > 0, x ≠ 0, x > 0
1  2
 1  3 + 1 1   1  4
= ln  + ln  = log  2  + log  
– + – 2 1 −  2  3 − 1  2
1 1 2  2
 2  2
–1 1
1 1
⇒ x ∈(0, 1) = log 3 + log 2 = log 3 + log 2
2 2
1 + 1 + x 2 
 will be defined when 1 + 1 + x > 0
2
log  = log( 3 ⋅ 2) = log 6.
 x  x
 
15. If sec θcosh y = cosec x and
⇒ x>0 [Q 1 + x 2 > 0] cosec θ sinh y = sec x , then sinh 2 y =
∴ x ∈ (0, ∞) [22 April 2018, Shift-II]
Domain of cot h−1 x is R − [−1, 1] and domain of (a) cos 2 x (b) cos x (c) sin2 x (d) sin x
sin h−1 x is R.
Sol. (a)
∴ f (x) will be defined on (0, 1).
Since, secθ cosh y = cosec x
π π   π θ  and cosec θ sinh y = secx
13. If − < θ < , then log  tan  +   =
2 2   4 2  So, cosθ = sin x cosh y …(i)
[23 April 2019, Shift-I] and sinθ = cos x sinh y …(ii)
−1  tan θ  θ
(b) 2 tanh −1  tan 
If we square and add Eqs. (i) and (ii),
(a) tanh  
 2  2 ⇒ 1 = sin2 x cosh2 y + cos2 x sinh2 y
(c) coth −1  tan θ  θ
(d) 2coth −1  tan  ⇒ sin2 x(1 + sinh2 y) + cos2 x sinh2 y = 1
 
 2  2 ⇒ sinh2 y + sin2 x = 1
Sol. (b) ⇒ sinh2 y = 1 − sin2 x = cos2 x
 π θ   π   − π π
Let log  tan  +   = x 16. If x = log e cot  + θ  and θ ∈  ,  , then
  4 2   4   4 4
θ consider the following statements
1 + tan
π θ
⇒ tan  +  = e x 2 = ex
⇒ [23 April 2018, Shift-I]
 4 2 θ I. cosh x = sec 2θ
1 − tan
2 II. sinh x = − tan 2θ
On applying componendo and dividendo rule, we
(a) I is true and II is false
get
θ (b) I is false and II is true
2 tan
2 = e −1
x
(c) Both I and II are true
2 e +1
x
(d) Both I and II are false
178 AP EAMCET Chapterwise Mathematics

Sol. (c) 1 1
= = = − tan 2θ
 π   π  − cot 2θ
We have, x = log cot  + θ tan  + 2θ
2 
 4  
ex + e−x ∴ Statement II is also true.
cos h x =
2
 1
log  cot 
π
+ θ   − log  cot 
π
+ θ   17. sec h 2  tanh −1  + cosech 2 (cot h −1 3) =
e  4  
+ e  4    2 [23 April 2018, Shift-II]
=
2 35 3 25 35
(a) (b) (c) (d)
  9 2 4 4
  π  1 
 cot  + θ +  Sol. (d)
    π 
4 cot  + θ 
sech2  tanh− 1  + cosech2 (cot h− 1 (3))
1
 4  
=   2
2
π π   3 
cot 2  + θ + 1 cosec2  + θ = sech2  sech− 1    + cosech2 (cosech− 1 8)
4  4    2 
= =
 π   π 
2 cot  + θ 2 cot  + θ 3
= + 8=
35
4  4  4 4
 π 
sin  + θ 1
1 4  18. If x = − , sin h −1 x + cosech −1 x =
= . 2
2  π π
2 sin  + θ cos  + θ
 
[24 April 2018, Shift-I]
4  4 
7 − 3 5  3 + 5
1 (a) loge   (b) loge  
=  2   2 
π π
2 sin  + θ cos  + θ  ( 5 − 1) (2 + 3 )
4  4  (c) loge 
2 
1 1  
= =
 π  π 
sin  2  + θ  sin  + 2θ  ( 5 + 1) (2 +
(d) loge 
3 )
 4  2  
 2 
1
= = sec 2θ Sol. (a)
cos 2θ
1
Hence, statement I is true. Given, x=−
2
ex − e−x
Again, sinh x = Q sinh−1 x = ln(x + x 2 + 1)
2
π
log  cot  + θ  
π
− log  cot  + θ   1 1 
e  4 
−e  4  and cos ech−1 x = ln + + 1
= x 
2
x
2
π ∴ sinh−1 x + cosech−1 x = ln(x + x 2 + 1)
cot  + θ −
1
4  π

cot  + θ 1
+ ln +
1 
+1
4 
= x x2 
2
π π  1 1 
cot  + θ − 1 1 − tan2  + θ
 2 = ln − + + 1  + ln(−2 + 4 + 1)
4  4   2 4 
= =
 π   π   1 5
2cot  + θ 2tan + θ = ln − +  + ln(−2 + 5)
4  4 
 2 2
1 1
= =  5 −1  ( 5 − 1) 
  π + θ  tan 2 π + θ  = ln  + ln( 5 − 2) = ln × ( 5 − 2) 
 2 tan        2   2 
 4    4 
 1 − tan2  π + θ   5 − 2 5 − 5 + 2  7 − 3 5
   = ln  = ln 
 4   2   2 
17
Rectangular Cartesian
Coordinates
1. Find the transformed equation of and 3y = a sin t − b cos t …(ii)
x cos θ + y sin θ = p, when the axes are Squaring and adding Eqs. (i) and (ii), we get
rotated through an angle θ. 9 x 2 − 6 x + 1 + 9 y2 = a 2 + b 2
[17 Sep. 2020, Shift-I] ⇒ 9 x 2 + 9 y2 − 6 x = a 2 + b 2 − 1
(a) x = p (b) y = p ∴ k = a2 + b2 −1
(c) x + y = p (d) x − y = p
3. If the axes are rotated through an angle 45°,
Sol. (a) then the co-ordinates of the point
Given x cosθ + y sinθ = P …(i) (4 2 , − 6 2) in the new system are ……… .
The axis are rotated through angle (θ), then [17 Sep. 2020, Shift-II]
x y (a) (− 10, − 2 ) (b) (− 2, − 10)
x cos θ –sin θ (c) (10, 10) (d) (− 2, 10)
y sin θ cos θ Sol. (b)
Let (x , y) are coordinates related to old axes and
⇒ x = X cosθ − Y sinθ and y = X sinθ + Y cosθ
(X , Y) are related to rotated (new) axes then,
Put in Eq. (i)
x = X cosθ − Y sinθ
⇒ X cos 2 θ − Y sin θ ⋅ cos θ + X sin 2 θ + Y sin θ ⋅ cos θ = P
⇒ X=P y = X sinθ + Y cosθ
Here, θ = 45°
2. Locus of the centroid of a triangle whose
x = 4 2, y = − 6 2
vertices are (1, 0). (a cos t, a sin t),
X Y X Y
(b sin t, − b cos t) is 9 x 2 + 9 y 2 − 6 x = k. Then, So, 4 2= − and −6 2 = +
2 2 2 2
the value of k is equal to [17 Sep. 2020, Shift-II]
Adding we have,
(a) a2 + b 2 (b) a2 + b 2 − 1 X
(c) a2 + b 2 + 1 (d) 0 − 2 2 = 2⋅ ⇒X = −2
2
Sol. (b) and subtracting,
Centroid is given by 2Y
10 2 = − ⇒ Y = −10.
x + x2 + x3 y + y2 + y3 2
x= 1 and y = 1
3 3 4. To which point the origin is to be shifted in
So, order to eliminate first powers of x and y
1 + a cos t + b sin t
x= (x 1 and y1 terms) from the equation
3
0 + a sin t + (− b cos t) 4 x 2 + 9 y 2 − 8 x + 36 y + 4 = 0 ?
and y=
3 [18 Sep. 2020, Shift-I]
⇒ (3x − 1) = a cos t + b sin t …(i) (a) (1, 2 ) (b) (− 1, 2 ) (c) (1, − 2 ) (d) (− 1, − 3)
180 AP EAMCET Chapterwise Mathematics

Sol. (c) 7. If A(2 , − 3) and B(− 2 , 1) are two vertices of a


Given equation is ∆ ABC and if the centroid of ∆ABC lies on the
4 x 2 + 9 y 2 − 8 x + 36 y + 4 = 0 line 2 x + 3 y = 1, then the locus of vertex C of
⇒ 4(x − 1)2 + 9(y + 2)2 = 36 ∆ABC is equal to [18 Sep. 2020, Shift-II]
(x − 1)2 (y + 2)2 (a) 2 x + 3 y = 5 (b) 2 x + 3 y = 9
⇒ + =1
9 4 (c) 3 x + 2 y = 5 (d) 3 x + 2 y = 9
If we shift the origin to (1, − 2), the equation reduce
x 2 y2 Sol. (d)
to + = 1 and it is free from x and y terms. Let third vertex be C = (h, k)
9 4
A ≡ (2, − 3)
5. The ratio in which the point P, whose B ≡ (−2, 1)
abscissa is 3, divides the join of A(6 , 5) and h −2 + k 

Centroid (G) =  ,
B(− 1, 4) is equal to ......... [18 Sep. 2020, Shift-II] 3

3 
(a) 3 : 4 (b) 4 : 3
(c) 3 : 2 (d) 2 : 3 Since, given G lies on 2x + 3y = 1
−2 + k 
2  + 3 
h
Sol. (a)  =1
 3  3 
Let the point P where abscissa is 3, divides the
join of points A(6, 5) and B(−1, 4) is λ:1, then 2h − 6 + 3k = 3
−λ + 6 2h + 3k = 9
3=
λ +1 ∴Required Locus is 2x + 3y = 9
⇒ 3λ + 3 = − λ + 6 Hence, option (d) is correct.
⇒ 4λ = 3 8. The point to which the origin should be
⇒ λ:1 = 3: 4 shifted so that the equation
Hence, option (a) is correct. y 2 − 6 y − 4 x + 13 = 0 is transformed to the
6. When the origin is shifted to (2 , 3) the form y 2 + Ax = 0 is [21 Sep. 2020, Shift-I]
transformed equation (a) (3, 1) (b) (− 1, − 1) (c) (1, 3) (d) (− 1, 3)
x 2 + 3 xy − 2 y 2 + 17 x − 7 y − 11 = 0 , then the Sol. (d)
original equation of curve is ......... Given equation is
[18 Sep. 2020, Shift-II] y 2 − 6 y − 4 x + 13 = 0 ⇒ (y − 3)2 = 4(x − 1)
(a) x2 − 2 y2 − 3 xy + 4 x − y + 20 = 0 ⇒ (y − 3)2 − 4(x − 1) = 0
(b) x2 − 2 y2 + 3 xy + 4 x − y − 20 = 0 By analysis of above form, it we shift the origin to
(c) x2 − 2 y2 − 3 xy − 4 x − y + 20 = 0 (–1, 3), then it transformed to the y 2 + 4 x = 0.
(d) x2 − 2 y2 − 3 xy + 4 x − y − 20 = 0 Hence, option (d) is correct.
Sol. (b) 9. Let O denote the origin. If M(1, 2), N (0, 1)
It is given that the origin is shifted to point (2, 3) and A(x , y) are points such that xy > 0 and
and due to that the transformed equation of the
curve is,
x + y < 1, then choose the correct statement.
[21 Sep. 2020, Shift-I]
x 2 + 3xy − 2y 2 + 17 x − 7 y − 11 = 0, to get the original
equation of curve, replace (x , y) by (x − 2, y − 3), so (a) A cannot lie inside ∆OMN
by doing this. (b) A lies inside ∆OMN
(c) A lies only in the first quadrant
we get, (x − 2)2 + 3(x − 2)(y − 3) − 2(y − 3)2
(d) A lies inside ∆OMN or in the third quadrant
+17(x − 2) − 7(y − 3) − 11 = 0
Sol. (*)
⇒ x 2 − 4 x + 4 + 3(xy − 3x − 2y + 6) − 2(y 2 − 6 y + 9)
It is given, for a point A(x , y)
+17 x − 34 − 7 y + 21 − 11 = 0
xy > 0 and x + y < 1
⇒ x + 3xy − 2y + 4 x − y − 20 = 0
2 2
The shaded region in the diagram represent the
Hence, option (b) is correct. position of point A.
Rectangular Cartesian Coordinates 181

y
12. The equation of the locus of a point P(x , y , z)
2 M (1, 2) such that it’s distance from the X -axis is equal
0, 1) to its distance from the plane x + z = 1 is
N(
[21 Sep. 2020, Shift-II]
1 (a) x2 − 2 y2 − z2 + 2 xz − 2 x − 2 z + 1 = 0
(b) x2 − 2 y2 − z2 + 2 xz − 2 x − 2 z − 1 = 0
O (c) x2 + 2 y2 + z2 + 2 xz − 2 x − 2 z + 1 = 0
x
0 1 2 (d) x2 − 2 y2 − z2 + 2 xz − 2 x + 2 z + 1 = 0

x+y=1 Sol. (a)


As it is given that the distance of a point P(x , y, z)
(*) No option is correct. from the X-axis is equal to its distance from the
10. The locus of a point P such that PA + PB = 4 plane x + z = 1, so
where A(2, 3, 4), B(− 2 , 3 , 4) is |x + z − 1|
(x − x)2 + (y − 0)2 + (z − 0)2 =
[21 Sep. 2020, Shift-I] 12 + 12
(a) y2
+ z2
+ 6 y + 8 z + 25 = 0 ⇒ 2(y + z ) = x + z + 1 + 2x z − 2x − 2z
2 2 2 2

(b) y2 − z2 + 6 y + 8 z − 25 = 0 ⇒ x 2 − 2y 2 − z 2 + 2xz − 2x − 2z + 1 = 0
(c) y2 + z2 − 6 y − 8 z + 25 = 0 Hence, option (a) is correct.
(d) y2 + z2 − 6 y − 8 z − 25 = 0 13. A point P(−3, − 2) is such that the sum of
Sol. (c) squares of its distances from the co-ordinate
Given points are A(2, 3, 4) and B (− 2 , 3, 4) and axes is equal to the square of its distance
AB = 4. from the line x − y = 1. Then the equation of
Now let point P(x , y, z), such that the locus of P is [22 Sep. 2020, Shift-I]
PA + PB = 4 = AB means point P is collinear with (a) x2 + y2 − 2 xy − 2 x − 2 y − 1 = 0
points A and B and lies between them, so (b) x2 + y2 + 2 xy + 2 x + 2 y + 1 = 0
x−2 y−3 z−4 (c) x2 + y2 + 2 xy + 2 x − 2 y − 1 = 0
= =
2+ 2 3− 3 4 − 4 (d) x2 + y2 − 2 xy + 2 x − 2 y + 1 = 0
⇒ y − 3= 0 = z − 4 Sol. (c)
or (y − 3)2 + (z − 4)2 = 0 It is given that the sum of squares of distance of
⇒ y + z 2 − 6 y − 8z + 25 = 0
2 point P(x , y) is equal to the square of its distance
Hence, option (c) is correct. from the line x − y = 1, so
(x − y − 1)2
x 2 + y2 =
11. The locus of the point whose ratio of distance 2
from the origin to its distance from (− 2 , − 3) ⇒ 2x 2 + 2y 2 = x 2 + y 2 + 1 − 2xy − 2x + 2y
is 5 : 7 , is given by ......... [21 Sep. 2020, Shift-I]
⇒ x 2 + y 2 + 2xy + 2x − 2y − 1 = 0
(a) 24( x2 + y2 ) − 100 x − 150 y − 325 = 0
14. If M is a point on the line y = x and points
(b) 24( x2 + y2 ) + 100 x + 150 y − 325 = 0
(c) 24( x2 + y2 ) − 100 x + 150 y + 325 = 0
P(0 , 1), Q(2 , 0) are such that PM + QM is
(d) 2 x2 + 2 y2 = 325 minimum, the coordinates of M are
[22 Sep. 2020, Shift-II]
Sol. (a)
(b)  , 
13 13
Let the point P(x , y), such that OP : AP = 5 : 7, (a) (0, 0)
 17 17 
where O is the origin and A (− 2, − 3), so
(c)  ,  (d)  , 
1 1 31 31
x 2 + y2 5 7 7 7 7
=
(x + 2)2 + (y + 3)2 7
Sol. (*)
⇒ 49(x 2 + y 2) = 25[x 2 + y 2 + 4 x + 6 y + 13] Given, P = (0, 1), Q =(2, 0)
⇒ 24 (x + y ) − 100 x − 150 y − 375 = 0
2 2
Let M = (a , a) [Q M lies on the line y = x]
Hence, option (a) is correct. In order to Make PM + QM is minimum
182 AP EAMCET Chapterwise Mathematics

⇒ P , M , Q Must be collinear 16. Find the coordinates of M in the original


∴ Slope of PM = Slope of QM system if the point M changes to (4 , − 3) when
a −1 a − 0
= the axes are rotated through an angle of 135°.
a−0 a−2 [22 Sep. 2020, Shift-II]
(a − 1) (a − 2) = a 2 −1 7
(a)  ,  (b)  , 
1 7
a 2 − 3a + 2 = a 2  2 2 2 2
−1 7 
(c)  (d) 
2 1 7 
− 3a = − 2 ⇒ a = ,  , 
3  2 2  2 2
∴ M is  ,
2 2
 Sol. (*)
3 3
Let M ′ = (4, − 3) = (x ′ , y′)
(*) No option is correct.
θ =135°
15. AB is a line segment moving between the Let coordinates of M be (x , y)
axes such that ‘A’ lies on X -axis and ‘B’ lies we have,
on Y -axis. If P is a point on AB such that x = x ′ cos θ − y′sinθ and y = x ′sinθ + y′ cosθ
PA = b and PB = a, then the equation of locus
⇒ x = 4 ⋅ cos 135 + 3sin 135° and
of P is [22 Sep. 2020, Shift-II]
y = 4 ⋅ sin 135° + (−3) cos 135°
x2 y2 x2 y2
+ 2 =1 + 2 =1 −1
x = 4 ⋅   + 3   and
(a) (b) 1
b 2
a a 2
b ⇒
 2  2
x2 y2 x2 y2
(c) 2 + =1 (d) 2 + 2 = 1 1 1
2a 2 b2 2b a y = 4⋅ + (−3)
2 2
Sol. (b) 1 1
x=− and y =
Let P(h, k) be any point in the locus cm 2 2
 −1 1 
∴ M = (x , y) =  , 
B  2 2
a (*) No option is correct.
θ P (h, k)
N 17. If the sum of the distances of a point from
h
b two perpendicular lines in a plane is 1, then
k k
θ its locus is [22 Sep. 2020, Shift-II]
h M A (a) Two intersecting lines (b) Square
(c) A straight line (d) Circle

Let A = (x , 0), B = (0, y) Sol. (b)


Given, PA = b, PB = a Let P = (h, k) be any point in the locus
In ∆PMA, let x , y -axes be two perpendicular lines
k ∴Distance from P(h, k) to X-axis = 1 (given)
sin θ =
b |k| = 1
In ∆BNP, Distance from P(h, k) to Y-axis = 1 ⇒ |h| = 1
h
cos θ = According to the question,|h| + |k| = 1
a
∴The locus of point (h, k) is|x| + |y| =1
We have,
sin2 θ + cos2 θ = 1 ∴It is a square
k2 h2 Hence, option (b) is correct
+ =1
b2 a2 18. The point to which the origin should be shifted
2 2

h k
+ 2 =1 so that the equation y 2 − 6 y − 4 x + 13 = 0 will
a2 b not contain term in y and the constant term, is
x2 y2
Required locus of point P(h, k) is 2
+ 2 =1 [23 Sep. 2020, Shift-I]
a b (a) (1, 1) (b) (1, 2 )
Hence, option (b) is correct. (c) (2, 1) (d) (1, 3)
Rectangular Cartesian Coordinates 183

Sol. (d) Sol. (d)


Given equation, is y 2 − 6 y − 4 x + 13 = 0 …(i) According to given information, if we draw the
Let origin to be shifted to the point (h, k) figure.
So, put x → x + h Y
y → y + k in Eq. (i)
From (y + k)2 − 6(y + k) − 4(x + h) + 13 = 0
y 2 + k2 + 2yk − 6 y − 6k − 4 x − 4h + 13 = 0
2b
To eliminate y and constant terms
put coefficient of y = 0 and constant = 0 2a
2k − 6 = 0 and k2 − 6k − 4h + 13 = 0 X
O
k=3 9 − 6(3) − 4h + 13 = 0
Let the equation of circle is
−9 − 4h + 13 = 0
x 2 + y 2 + 2gx + 2 fy + c = 0
h =1
Q 2 g2 − c = 2a
∴ Required point = (h, k) = (1 ,3)
Hence, option (d) is correct. and 2 f 2 − c = 2b
19. Two points A and B with co-ordinates (1, 1) then g2 − a 2 = 0 and f 2 − b 2 = 0
and (−2 , 3) respectively are given. Then the so, g2 − a 2 = f 2 − b 2
locus of a point P so that the area of ∆PAB is ⇒ g2 − f 2 = a 2 − b 2
9 sq. units is given by [23 Sep. 2020, Shift-I] On taking locus of the centre (− g, − f), we get
(a) 2 x + 3 y + 13 = 0 and 2 x + 3 y − 23 = 0 x 2 − y2 = a 2 − b 2
(b) 2 x + 3 y − 23 = 0 and 2 x + 3 y − 13 = 0 Hence, option (d) is correct.
(c) 2 x + 3 y − 13 = 0 and 2 x − 3 y + 23 = 0 21. When the coordinate axes are rotated about
(d) 2 x − 3 y + 23 = 0 and 2 x + 3 y + 13 = 0 the origin in the positive direction through
Sol. (a) π
an angle , if the equation 25 x 2 + 9 y 2 = 225
Let P (x , y) be any point on the locus 4
A = (1, 1) is transformed to αx 2 + βxy + γy 2 = δ, then
⇒ B = (−2, 3) (α + β + γ − δ)2 = [20 April 2019, Shift-I]
Given, Ar (∆PAB) = 9 cm2
1 (a) 3 (b) 9
|x1 (y2 − y3) + x 2 (y3 − y1 ) + x 3 (y1 − y2)| = 9 (c) 4 (d) 16
2
|x (1 − 3) + 1 (3 − y) + (−2) (y − 1)| = 18 Sol. (b)
|−2x + 3 − y − 2y + 2| = 18 After rotation of coordinate axes about the origin
π
|−2x − 3y + 5|= 18 in the positive direction through on angle , the
4
−2x − 3y + 5 = ± 18 (or) −2x − 3y + 5 = 18
new coordinates are (X , Y) have relation with
or −2x − 3y + 5 = −18 older coordinates (x , y) is
2x + 3y + 13 = 0 (or) 2x + 3y − 23 = 0
(x , y) = [(X cosθ − Y sinθ), (Y cosθ + X sinθ)), where
Hence, option (a) is correct. π
θ=
20. Two straight rods of lengths 2a and 2b move 4
 X X 
=  
along the coordinate axes in such a way that Y   Y
− ,  + 
their extremities are always concyclic. Then  2 2  2 2 
the locus of the centres of such circles is so, 25x 2 + 9 y 2 = 225 becomes
[20 April 2019, Shift-I] 2 2
X − Y  X + Y  = 225
(a) 2( x2 + y2 ) = a2 + b 2 25  + 9 
 2   2 
(b) 2( x2 − y2 ) = a2 + b 2
(c) x2 + y2 = a2 + b 2 ⇒ 34 X 2 + 34Y 2 − 32XY = 450
(d) x2 − y2 = a2 − b 2 ⇒ 17 X 2 + 17Y 2 − 16 XY = 225
184 AP EAMCET Chapterwise Mathematics

On comparing, we get and x sinθ − (1 + cosθ) y = − a sinθ … (ii)


α = γ = 17, β = −16 and δ = 225 Subtracting Eq. (ii) from Eq. (i), we get
∴ (α + β + γ − δ)2 = (34 − 16 − 15)2 = 32 = 9 (1 − cosθ) y + (1 + cosθ) y = 2a sinθ
Hence, option (b) is correct. ⇒ y[1 − cosθ + 1 + cosθ] = 2a sinθ
22. The locus of a point P(x , y) satisfying the ⇒ y = a sinθ
Putting the value of y in Eq. (i), we get
equation (x − 2)2 + y 2 + (x + 2)2 + y 2 = 4 , is
x sinθ + (1 − cosθ) a sinθ = a sinθ
[20 April 2019, Shift-II]
⇒ sinθ[x + (1 − cosθ)a] = a sinθ
(a) an ellipse (b) a parabola
⇒ x + a − a cosθ = a
(c) a line segment (d) a circle
⇒ x − a cosθ = 0
Sol. (c)
⇒ x = a cosθ
Given, equation
Now, x 2 + y 2 = (a cosθ)2 + (a sinθ)2
(x − 2)2 + y 2 + (x + 2)2 + y 2 = 4
⇒ x 2 + y 2 = a 2 cos2 θ + a 2 sin2 θ
⇒ Sum of distance of a point P(x , y) w.r.t. points ⇒ x 2 + y 2 = a 2 (cos2 θ + sin2 θ)
A(2, 0) and B(−2, 0) = Distance between points
[Q sin2 θ + cos2 θ = 1]
A(2, 0) and B((−2, 0)
⇒ x + y =a ,
2 2 2
⇒ AP + BP = AB
whose represent a circle.
∴Locus of point P(x , y) is a line segment.
Hence, option (c) is correct. 25. A line L has intercepts a and b on the
coordinate axes. When the axes are rotated
23. If the origin is shifted to (2, 3) and the axes through a given angle θ keeping the origin
are rotated through an angle 45° about that fixed, this line L has the intercepts p and q.
point, then the transformed equation of Then [21 April 2019, Shift-I]
2 x 2 + 2 y 2 − 8 x − 12 y + 18 = 0 is
(a) a2 + b 2 = p2 + q 2 (b) a2 + p2 = b 2 + q 2
[20 April, 2019 Shift-II] 1 1 1 1 1 1 1 1
(c) 2 + 2 = 2 + 2 (d) 2 + 2 = 2 + 2
(a) x2 − 7 y2 − 14 xy − 2 = 0 (b) x2 + y2 = 4 a p b q a b p q
(c) x2 − y2 = 4 (d) 8 x2 − 2 y2 = 9
Sol. (d)
Sol. (b)
Equation of a line
Given, equation is x y
2x 2 + 2y 2 − 8 x − 12y + 18 = 0 L: + =1 …(i)
a b
⇒ 2(x − 2)2 + 2(y − 3)2 = 8 rotate the axis with respect to angle α
⇒ (x − 2)2 + (y − 3)2 = 4 x = x cosα − y sinα 
 …(ii)
After the shifting of origin to (2, 3), the y = x sinα + y cosα 
transformed equation becomes x 2 + y 2 = 4 and
after the rotation of axes through an angle 45° Put the values of Eq. (ii) in Eq. (i), we get
about the point, the transformed equation is x cosα − y sinα x sinα + y cosα
+ =1
x 2 + y 2 = 4. Hence, option (b) is correct. a b
cosα sinα   cosα − sinα  = 1
24. For any value of θ, if the straight lines x  +  + y 
 a b   b α 
x sin θ + (1 − cos θ) y = a sin θ and
x sin θ − (1 + cos θ) y + a sin θ = 0 intersect at x
rotated axis L’ : + = 1
y
P(θ), then the locus of P(θ) is a p q
[21 April 2019, Shift-I] cosα sinα 1
∴ + =
(a) straight line (b) circle a b p
(c) parabola (d) hyperbola cosα sinα 1
and − =
Sol. (b) b a q
Given, equations of straight lines are cosα sinα 
2
 cosα − sin α 
2

= 
1 1
x sinθ + (1 − cosθ) y = a sinθ ⇒ + +  + 
…(i) p2 q 2  a b   b a 
Rectangular Cartesian Coordinates 185

cos2 α sin2 α 2sinα cosα cos2 α sin2 α Sol. (c)


= + + + +
a2 b2 ab b2 a2 Let the original and new coordinates of the curve
−2sinα cosα are as following form
ab
x y
cos2 α + sin2 α cos2 α + sin2 α
= + x π π
a2 b2 cos sin
4 4
1 1 1 1
⇒ + 2 = 2 + 2 y π π
p2 q a b − sin cos
4 4
26. If the sum of the distances from a variable x y
point P to the given points A(1, 0) and B(0 , 1) is Now, x= − …(i)
2 2
2, then the locus of P is [21 April 2019, Shift-II]
x y
(a) 3 x2 + 3 y2 − 4 x − 4 y = 0 and y= + …(ii)
2 2
(b) 16 x2 + 7 y2 − 64 x − 48 y = 0
(c) 3 x2 + 2 xy + 3 y2 − 4 x − 4 y = 0 Substituting Eqs. (i) and (ii) in curve
(d) 16 x2 + 38 xy + 7 y2 − 64 x − 48 y = 0 9 x 2 + 25y 2 = 225
2 2
x − y  x + y  = 225
Sol. (c) ⇒ 9  + 25 
 2   2 
Let P is (x , y)
Given that, AP + BP = 2  x 2 + y 2 − 2xy   x 2 + y 2 + 2xy 
⇒ 9  + 25  = 225
⇒ (x − 1) + y +
2 2
x + (y − 1) = 2
2 2  2   2 

⇒ (x − 1) + y = 2 −
2 2
x 2 + (y − 1)2 ⇒ 9 x 2 + 9 y 2 + 18 xy + 25x 2 + 25y 2 − 50 xy = 450
⇒ 34 x 2 + 34 y 2 − 32xy = 450
⇒(x − 1)2 + y 2 = 4 + x 2 + (y − 1)2 − 4 x 2 + (y − 1)2
⇒ 17 x 2 + 17 y 2 − 16 xy = 225
(Squaring on both sides)
⇒x 2 + y 2 + 1 − 2x = 4 + x 2 + (y 2 + 1 − 2y) 28. A straight line meets the X and Y axes at the
points A, B respectively. If AB =6 units, then
− 4 x 2 + (y − 1)2
the locus of the point P which divides the
⇒ x 2 + y 2 + 1 − 2x = 4 + x 2 + y 2 + 1 − 2y line segment AB such that AP : PB = 2 :1, is
− 4 x 2 + (y − 1)2 [22 April 2019, Shift-I]
(a) 3 x2 + y2 = 36 (b) 4 x2 + y2 = 36
⇒ 2y − 2x = 4 − 4 x 2 + (y − 1)2
(c) 3 x2 + y2 = 16 (d) 4 x2 + y2 = 16
⇒ y − x − 2 = − 2 x 2 + (y − 1)2 Sol. (d)
⇒ x 2 + y 2 + 4 − 2xy + 4 x − 4 y B (0, b)
= 4(x 2 + y 2 + 1 − 2y)
⇒ x + y + 4 − 2xy + 4 x − 4 y = 4 x 2 + 4 y 2 + 4 − 8 y
2 2
1
⇒ 3x 2 + 3y 2 − 4 x − 4 y + 2xy = 0
P(h, k)
27. If the equation of a curve C is transformed to
9 x 2 + 25 y 2 = 225 by the rotation of the 2

coordinate axes about the origin through an O


π A (a,0)
angle in the positive direction then the
4 Let A(a , 0) and B(0, b) and point P(h, k) divides the
equation of the curve C, before the line AB.
transformation is [21 April 2019, Shift-II]  a × 1 + 0 0 × 2b 
(a) 17 x2 + 16 xy + 17 y2 = 225 Now, P(h, k) =  , 
 1+2 1+ 2
(b) 17 x2 + 23 y2 = 391
P(h, k) =  , 
a 2b
(c) 17 x2 − 16 xy + 17 y2 = 225 ⇒
3 3
(d) 23 x2 + 17 y2 = 391
186 AP EAMCET Chapterwise Mathematics

a
∴ h= B(0, q)
3
2b
and k=
3 P(h, k)
⇒ a = 3h
3k
and b=
2 A(p, 0)
Also, given that AB = 6
(a − 0)2 + (0 − b)2 = 6 Given , AB = a
⇒ a + b = 36
2 2 (h, k) = mid-point of AB

(h, k) =  , 
9k2 p q
⇒ 9h2 + = 36  2 2
4
k2 ∴ p = 2h, q = 2k
⇒ h2 + =4
4 ⇒ A =(p , 0) =(2h, 0)
⇒ 4h2 + k2 = 16 B = (0, q) = (0, 2k)
Since, length of AB = a
∴ Required locus of P is
(2h)2 + (2k)2 = a
4 x 2 + y 2 = 16.
⇒ 4h2 + 4k2 = a
29. Let P be the point (4, 1) and Q be its image in
4h2 + 4k2 = a 2
the line y = x . If Q is translated through a a2
distance 2 units along the negative Y-axis to ⇒ h2 + k2 =
4
reach the point R, then the co-ordinates of R
a2
are [22 April 2019, Shift-II] ∴ Required locus is x 2 + y 2 =
4
(a) (–1, 2) (b) (1, –2)
∴ Hence, solution is (c).
(c) (–1, –2) (d) (1, 2)
Sol. (d) 31. If (h, k) be the point to which the origin has to
be shifted in order to get the transformed
The image of P(4, 1) on the line y = x is Q.
equation of y 2 − 4 x + 6 y + 17 = 0 as y 2 = 4 ax ,
So, the coordinates of Q are (1, 4).
then h2 + k 2 = [23 April 2019, Shift-I]
The image of Q(1, 4) in the mirror y = − 2 , i.e.
X-axis is (1, 2). So, the coordinates of R are (1, 2). (a) 11 (b) 1 (c) 25 (d) 13
Sol. (d)
30. A line moves such that the portion of it Given equation is y 2 − 4 x + 6 y + 17 = 0
intercepted between the coordinate axes is of
⇒ (y + 3)2 = 4 (x − 2)
constant length a, then the locus of the mid
If origin is shifted to the point (2, − 3), then the
point of that line segment is equation y 2 − 4 x + 6 y + 17 = 0 get transformed as
[23 April 2019, Shift-I] y 2 = 4ax.
x2 y2
(a) + = a2 (b) x2 + y2 = a2 So, (h, k) = (2, − 3) ⇒ h2 + k2 = 4 + 9 = 13
4 4
Hence, option (d) is correct.
a2 a2
(c) x2 + y2 = (d) x2 + y2 =
4 2 32. Let A (2, 3), B (3, −6), C (5, −7) be three
points. If P is a point satisfying the condition
Sol. (c)
PA 2 + PB2 = 2 PC 2, then a point that lies on the
Let A = (p , 0), B = (0, q) locus of P is [22 April 2018, Shift-I]
Let P =(h, k) be the mid-point of AB. (a) (2,−5) (b) (−2,5) (c) (1310
, ) (d) (−13,−10)
Rectangular Cartesian Coordinates 187

Sol. (d) 34. The equation of the locus of a point


Given, points are A(2, 3, ) B(3, − 6), C(5, − 7). (2 cos θ − 3 , 3 sin θ − 4) is [22 April 2018, Shift-II]
Let point P be (x , y), then according to condition (a) 9 x2 + 4 y2 + 54 x + 32 y + 181 = 0
PA2 + PB2 = 2PC 2 (b) 4 x2 + 9 y2 + 54 x + 32 y + 109 = 0
⇒(x − 2)2 + (y − 3)2 + (x − 3)2 + (y + 6)2 (c) 9 x2 + 4 y2 − 54 x + 32 y + 109 = 0
= 2[(x − 5)2 + (y + 7)2 ] (d) 9 x2 + 4 y2 + 54 x + 32 y + 109 = 0
⇒ x 2 + 4 − 4 x + y 2 + 9 − 6 y + x 2 + 9 −2 6 x Sol. (d)
+ y + 36 + 12y
= 2[x 2 + 25 − 10 x + y 2 + 14 y + 49] Given point (2cosθ − 3, 3sinθ − 4) = (x , y) Let
x+ 3
⇒ 2x 2 + 2y 2 − 10 x + 6 y + 58 ⇒ cosθ =
= 2x 2 + 2y 2 − 20 x + 28 y + 148 2
⇒ 10 x − 22y = 90 y+ 4
and sinθ =
By checking options, 3
1. 10(2) − 22(− 5) = 20 + 110 = 130 ⇒ cos2 θ + sin2 θ = 1
2. 10(− 2) − 22(5) = − 20 − 110 = − 130
2 2

⇒  x + 3 +  y + 4  = 1
   
3. 10(13) − 2210
( ) = 130 − 220 = − 90  2   3 
4. 10(− 13) − 22(− 10) = − 130 + 220 = 90 ⇒ 9(x 2 + 6 x + 9) + 4(y 2 + 8 y + 16) = 36
So, point (− 13, − 10) lies on the locus of P.
⇒ 9 x 2 + 4 y 2 + 54 x + 32y + 145 = 36
33. If the coordinates of a point P changes to ⇒ 9 x 2 + 4 y 2 + 54 x + 32y + 109 = 0
(2 ,−6 ) when the coordinate axes are rotated
through an angle of 135°, then the 35. When the origin is shifted to the point (2, 3)
coordinates of P in the original system are and then the coordinate axes are rotated
[22 April 2018, Shift-I] through an angle π / 3 in the counter
(a) (−2,6) (b) (−62
, ) clockwise sense, then the transformed
(c) (2 2 ,4 2 ) (d) ( 2 , − 2 ) equation of 3 x 2 + 2 xy + 3 y 2 −
Sol. (c) 18 x − 22 y + 50 = 0 is [22 April 2018, Shift-II]
If (x , y) is old coordinates and (X , Y) are new (a) 3 x2 + 3 y2 − 1 = 0
coordinates, when axes are rotated through an (b) (6 + 3 )x2 − 2 xy + (6 − 3 )y2 − 2 = 0
angle of θ, then (c) 4 x2 + 2 y2 − 1 = 0
x = X cosθ − Y sinθ and y = X sinθ + Y cosθ (d) (6 − 3 )x2 + (6 + 3 )y2 + 2 xy = 0
∴ x = 2cos135° − (− 6)sin135° Sol. (b)
and y = 2sin135° + (− 6) cos135° We first change the origin, by putting x′ + 2 and
[Qθ = 135° and P(x , y) = (2, − 6)] y′ + 2 for x and y respectively.
 − 1  1  The new equation will be
⇒ x = 2  + 6 
 2  2 ⇒ 3(x ′ + 2)2 + 2(x ′ + 2) (y′ + 3) + 3(y′ + 3)2 −
− 1 18(x ′ + 2) −22(y′ + 3) + 50 = 0
y = 2  − 6 
1
and 
 2  2 It reduces to
⇒ x=
4 3x ′2 + 2x ′y ′ + 3y ′2 − 1 = 0 …(i)
2 Dropping the suffices, we get the equation
8
and y= 3x 2 + 2xy + 3y 2 − 1 = 0
2
To turn the axes through an angle of π/ 3, in the
⇒ x=2 2 counter clockwise sense, we must write
and y = 4 2. x′ − 3y′ 3x ′ + y′
for x and for y. The Eq. (i) will
∴Coordinates of P in the original system are 2 2
(2 2, 4 2). then be
188 AP EAMCET Chapterwise Mathematics

 x ′ − 3y′ 
2
 x ′− 3y′   3x ′ + y′  3
⇒ tan θ =
3  + 2    + 4
 2   2   2 
4
⇒ cos θ =
2
 3x ′ + y′ 
3  −1 = 0 5
 2 
3
By dropping the suffices and sin θ =
5
3(x − 3y)2 + 2(x − 3y) ( 3x + y) + 4 3
So, x = x1 − y1
3 ( 3x + y)2 − 4 = 0 5 5
⇒ 3(x 2 + 3y 2 − 2 3xy) + 2 ( 3x 2 − 2xy − 3y 2) 3 4
and y = x1 + y1
+ 3 (3x + y + 2 3xy) − 4 = 0
2 2 5 5
x and y satisfy the equation
⇒ (12 + 2 3) x − 4 xy + (12 − 2 3) y 2 − 4 = 0
2
x 2 + y2 = 9
⇒ (6 + 3) x 2 − 2xy + (6 − 3) y 2 − 2 = 0 2 2

⇒ 4 x − 3y  + 3x + 4 y  = 9
 1 1  1 1
36. Let A , B and C be three points in a plane. The 5 5  5 5 
locus of a point P moving such that 16 2 9 2 24
⇒ x1 + y1 − x1 y1
PA 2 + PB2 = 2 PC 2 is a [23 April 2018, Shift-I] 25 25 25
9 2 16 2 24
(a) Straight line (b) Pair of straight lines + x1 + y1 + x1 y1 = 9
(c) Circle (d) Parabola 25 25 25

Sol. (a) ⇒ x12 + y12 = 9


Let A (x1 , y1 ), B (x 2 , y2) and (x 3 , y3) and P (x , y) be So, transformed equations is x 2 + y 2 = 9
points on xy plane. 38. If a point P moves such that the sum of the
Now, PA2 + PB2 = 2PC 2 distances from P to the point A (1, − 1) and
⇒(x − x1 )2 + (y − y1 )2 + (x − x 2)2 + (y − y2)2 B (−1, 1) is always 4, then the equation for the
= 2 [(x − x 3)2 + (y − y3)2 ] locus of P is [23 April 2018, Shift-II]
(a) 16 x2 − 64 x + 7 y2 = 48
⇒ x + x − 2xx1 + y 2 + y12 − 2yy1
2 2
1
(b) 3 x2 + 2 xy + 3 y2 = 8
+ x 2 + x 22 − 2xx 2 + y 2 + y22 − 2yy2
(c) 6 x + 4 y = 3
= 2[x 2 + x 32 − 2xx 3 + y 2 + y32 − 2yy3 ]
(d) x2 + y2 − 8 x + 6 y = 0
⇒ x(−2x1 − 2x 2 + 4 x 3) + y (−2y1 − 2y2 + 4 y3)
Sol. (b)
+ x12 + x 22 + y12 + y 2 − 2x 32 − 2y32 = 0
Let point P(x , y), so
this equation is in the form of ax + by + c = 0
Hence, locus of p is a straight line. (x − 1)2 + (y + 1)2 + (x + 1)2 + (y − 1)2 = 4

37. When the coordinate axes are rotated by an ⇒ (x − 1)2 + (y + 1)2 + (x + 1)2 + (y − 1)2
 3 + 2 (x − 1)2 + (y + 1)2 (x + 1)2 + (y − 1)2 = 16
angle tan − 1   about the origin, then the
 4
⇒ 2(x 2 + y 2 + 2) + 2 (x − 1)2 + (y + 1)2
equation x 2 + y 2 = 9 is transformed to the
equation. [23 April 2018, Shift-I] (x + 1)2 + (y − 1)2 = 16
(a) x2 − y2 = 9 (b) x2 + y2 + 2 xy = 4 ⇒ (x 2 + y 2 + 2) + (x − 1)2 + (y + 1)2
(c) x2 + y2 = 9 (d) x2 − y2 + 9 = 0
(x + 1)2 + (y − 1)2 = 8
Sol. (c)
When the coordinate axis rotated by are angle θ ⇒ [(x − 1)2 + (y + 1)2 ][(x + 1)2 + (y − 1)2 ]
about origin, then = (6 − x 2 − y 2)2
x = cos θ x1 − sin θ y1 ⇒ [x 2 + y 2 − 2x + 2y + 2] [x 2 + y 2 + 2x − 2y + 2]
and y = sin θ x1 + cos θ y1 = (x 2 + y 2 − 6)2
3 ⇒ (x 2 + y 2 + 2)2 − 4(x − y)2 = (x 2 + y 2 − 6)2
Here, θ = tan−1
4
Rectangular Cartesian Coordinates 189

⇒ [(x 2 + y 2 + 2) − (x 2 + y 2 − 6)] y = mx
[(x 2 + y 2 + 2) + (x 2 + y 2 − 6)] and x − y + 10 = 0
= 4(x − y)2
x − mx = − 10
⇒ 8(2x 2 + 2y 2 − 4) = 4(x 2 + y 2 − 2xy)
− 10 10
⇒ 4 x 2 + 4 y 2 − 8 = x 2 + y 2 − 2xy x= =
1 − m m −1
⇒ 3x 2 + 2xy + 3y 2 = 8 10m
and y=
39. The transformed equation of m −1
3 x 2 − 6 xy + 8 y 2 = 8 when the axes are rotated  10 10m 
π ∴ A , 
about the origin through an angle in the  m − 1 m − 1
4
100 100m2
positive direction, is [23 April 2018, Shift-II] So, OA = +
(a) 5 x2 + 10 xy + 17 y2 + 16 = 0 (m − 1) 2
(m − 1)2
(b) 5 x2 + 10 xy + 17 y2 − 16 = 0 ( + m2)
1001
=
(c) 5 x2 − 10 xy + 17 y2 − 16 = 0 (m − 1)2
(d) 5 x2 − 10 xy + 17 y2 + 16 = 0 10 (1 + m2)
=
Sol. (b) m −1
Given transformed equation is Similarly, coordinate of point B is
3x 2 − 6 xy + 8 y 2 = 8 …(i)
π π  20 20m 
Now, x ′ = x cos − y sin ∴ B , 
4 4  m − 1 m − 1
x− y
= Then, OB =
400
+
400m2
2 (m − 1) 2
(m − 1)2
π π
and y′ = x sin + y cos
4 4 20 1 + m2
=
x+ y (m − 1)
=
2 Let point P(h, k) lies on the line y = mx
Before transformation the equation is So, k = mh
2 2
x − y  x − y  x + y + 8 x + y = 8
3  − 6     ⇒ m=
k
 2   2  2   2  h
⇒ 3(x 2 + y 2 − 2xy) − 6(x 2 − y 2) and OP = h2 + k2
+ 8(x 2 + y 2 + 2xy) = 16
⇒ 5 x2 + 17 y2 + 10 xy − 16 = 0. Now, OA, OP, OB in H.P.
1 1 1
40. A variable line ‘L’ passing through the origin So, , , in AP.
OA OP OB
cuts two parallel lines x − y + 10 = 0 and 1 1 1
x − y + 20 = 0 at two points A and B Hence, = +
OP OA OB
respectively. If P is a point on line ‘L’ such
2 m −1 m −1
that OA, OP, OB are in harmonic progression, = +
h2 + k2 10 1 + m2 20 1 + m2
then the locus of P is [24 April 2018, Shift-I]
m −1 
(a) 3 x + 3 y + 40 = 0 (b) 3 x + 3 y + 20 = 0 1+ 
2 1
=
(c) 3 x − 3 y + 40 = 0 (d) 3 x − 3 y + 20 = 0 h2 + k2 10 1 + m2  2
Sol. (c) Putting the value of m, we get
Let the equation of line passing through origin is  k − 1
 
y = mx which cut the parallel lines x − y + 10 = 0 2 h  3
= ⋅
and x − y + 20 = 0 at points A and B respectively. h + k
2 2
k2 2
10 ⋅ 1 + 2
Then, co-ordinates of points A and B are, h
190 AP EAMCET Chapterwise Mathematics

2 3(k − h) / h Sol. (a)


⇒ =
h2 + k2 h2 + k2 When the axes are rotated through 45° about the
20
h origin in the positive direction then replace (x , y)
y − x
by 
3(k − h) x y 

2
= + , + 
 2 2 2 2
h2 + k2 20 h2 + k2
So, original equation of the curve,
⇒ 3(k − h) = 40 2

⇒ 17
x
+
y   x + y  − x + y 
⇒ 3h − 3k + 40 = 0  − 16   
 2 2   2 2  2 2
Hence, locus of the point is − x
2

+ 17 
y 
+  = 225
3x − 3y + 40 = 0.  2 2

41. When the coordinate axes are rotated  x2 y2   y2 x2 


⇒ 17 + + xy − 16 − 
through an angle of 45° about the origin in  2 2   2 2
the positive direction, if the transformed  x2 y2 
+ 17 + − xy = 225
equation of a curve is  2 2 
17 x 2 − 16 xy + 17 y 2 = 225, then the original x2 y2
⇒ 17 + 17 + 17 xy − 8 y 2 + 8 x 2
equation of that curve is [24 April 2018, Shift-I] 2 2
(a) 25 x2 + 9 y2 = 225 x2 y2
+ 17 + 17 − 17 xy = 225
(b) 9 x2 − 25 y2 = 225 2 2
(c) 25 x2 − 16 xy + 9 y2 = 225 ⇒ 17 x 2 + 17 y 2 − 8 y 2 + 8 x 2 = 225
(d) 9 x2 + 25 y2 = 225 ⇒ 25x 2 + 9 y 2 = 225
18
Straight Line and Pair of
Straight Lines
1. Find the value of k, if the angle between the 3. If p and q are lengths of the perpendiculars
straight lines represented by from origin to the lines
2 x 2 + 5 xy + 3 y 2 + 6 x + 7 y + 4 = 0 is tan −1 (k). x sec (θ) + y cosec (θ) = k and
[17 Sep. 2020, Shift-I] x cos (θ) − y sin (θ) = k cos (2θ) respectively,
−1
(a)   only (b)   only
1 then [17 Sep. 2020, Shift-I]
 5  5
(a) p2 + 4 q 2 = k 2 (b) 4 p2 + q 2 = k 2
1 (c) p2 + q 2 = 4k 2 (d) p2 + q 2 = k 2
(c) ± (d) 0
5
Sol. (b)
Sol. (c) |0 + 0 − k| |0 − 0 − k cos2 θ|
Straight lines 2x 2 + 5xy + 3y 2 + 6 x + 7 y + 4 = 0 p= ,q=
sec θ + cos ec θ
2 2
cos2 θ + sin2 θ
Angle between straight lines represented by 2
k
ax 2 + by 2 + 2gx + 2 fy + c + 2hxy = 0 is ⇒ p2 = , q 2 = k2 cos2 2θ
sec2 θ + cos ec2θ
2 h2 − ab
tanθ = k2
a+ b ⇒ p2 = , q 2 = k2 cos2 2θ
1 1
+
2 (5/ 2)2 − 6 1 cos2 θ sin2 θ
tanθ = =±
5 5 ⇒ p 2 = k2 sin2 θ ⋅ cos2 θ, q 2 = k2 cos2 2θ
θ = tan K = tan  ±  ⇒  K = ± 
−1 
−1 1 1 k2
⇒ ⇒ p 2 = sin2 2 θ, q 2 = k2 cos2 2θ
 5  5 4
2. A square is formed by the lines x = 0 , y = 0 , So, p2 +
q 2 k2
=
x = 1, y = 1. Then, the equations of its 4 4
diagonals will be [17 Sep. 2020, Shift-I] ⇒ 4p 2 + q 2 = k2
1
(a) y = x, x + y = 2 (b) 2 y = x, x + y = 4. If x 2 + y 2 + 6 x + 2ky + 25 = 0 to touch Y -axis,
2
1 then k = [17 Sep. 2020, Shift-I]
(c) y = x, x + y = 1 (d) y = 2 x, x + y =
4 (a) ± 20 (b) − 1, − 5 (c) ± 5 (d) 4
Sol. (c) Sol. (c)
Equation of OB Given circle
1 − 0 x 2 + y 2 + 6 x + 2ky + 25 = 0 touches Y −axis
y =  x ⇒ (y = x)
1 − 0  and here centre is (−3, − k) so radius = 3 units
Equation of AC Also radius = 9 + k2 − 25
y − 0 = −1(x − 1) ⇒ y = − x + 1 ⇒ x + y = 1 ⇒ k2 − 16 = 3 ⇒ k2 = 25 ⇒ k = ± 5
192 AP EAMCET Chapterwise Mathematics

5. If pairs of straight lines x 2 − 2pxy − y 2 = 0 and 8. If the pair of straight lines 6 x 2 − 5 xy + y 2 = 0


x − 2 qxy − y = 0 be such that each pair
2 2
makes angles α and β with the X -axis, then
bisects the angle between the other pair, then tan (α − β) = [17 Sep. 2020, Shift-I]
[17 Sep. 2020, Shift-I] 1
(a) 2 (b) (c) 3 (d) 7
(a) pq =1 (b) pq = 2 7
(c) pq = − 2 (d) pq = − 1 Sol. (b)
Sol. (d) 6 x 2 − 5xy + y 2 = 0
2
Equation of angle bisector for ⇒  y − 5 y + 6 = 0
   
ax 2 + 2hxy + by 2 = 0  x  x
x 2 − y2 xy  y − 3  y − 2 = 0
⇒ = ⇒   
a−b h x  x 
and for x 2 − 2pxy − y 2 = 0
⇒ y = 3x and y = 2x are straight lines
x 2 − y2 xy 2xy
⇒ = ⇒ x 2 − y2 + =0 tanα = 3, tanβ = 2
1 − (−1) − p p 3− 2 1
tan(α − β) = =
Given equation of angle bisector is 1+ 6 7
x 2 − 2qxy + y 2 = 0
9. The image of the point P(3, 8) with respect to
On comparing
2 the line x + 3 y = 7, assuming the line to be a
⇒ = −2q ⇒ pq = −1 plane mirror, is equal to .........
P
[17 Sep. 2020, Shift-II]
6. The angle between the lines 2 x + 11y − 7 = 0 (a) (1, 4) (b) (− 1, 4)
and x + 3 y + 5 = 0 is ……… [17 Sep. 2020, Shift-I] (c) (1, − 4) (d) (− 1, − 4)
(a) tan−1   (b) tan−1  
17 4 Sol. (d)
 13   35 
Let image is P ′(h, k)
(c) tan−1  
1
(d) tan−1 (7 ) P (3, 8)
7

Sol. (c)
Given lines 2x + 11 y − 7 = 0 and x + 3y + 5 = 0
2 1 mirror
m1 = − , m2 = − x+3y=7
11 3
m1 − m2 1
tanθ = = P′ (h, k)
1 + m1 m2 7
Then PP′ is perpendicular to given line and mid
θ = tan−1  
1
 7 point of PP′ lies over line.
So, we have
7. If the lines 3 x + 4 y − 5 = 0 , 2 x + 3 y − 4 = 0  8 − k  × −1 = − 1
and px + 4 y − 6 = 0 all meet at the same  
 3− h  3
point, then p is equal to [17 Sep. 2020, Shift-I]
⇒ 8 − k = 3 (3 − h) …(i)
(a) −2 (b) 0 (c) 1 (d) 2
3+ h 8 + k
Sol. (d) And, + 3   =7
2  2 
3x + 4 y − 5 = 0 …(i)
2x + 3y − 4 = 0 …(ii) 3 + h + 24 + 3k = 14
and px + 4 y − 6 = 0 …(iii) ⇒ h + 3k + 13 = 0 …(ii)
From Eqs. (i) and (ii), we get From Eqs. (i) and (ii), we have
x = −1, y = 2 h = −1 and k = −4
So, −p + 8 − 6 = 0 ⇒ p = 2 So image is, (−1, −4).
Straight Line and Pair of Straight Lines 193

10. The line x + y = 1 meets the lines represented Sol. (a)


by the equation y 3 − 6 xy 2 + 11 x 2 y − 6 x 3 = 0 at For perpendicular pairs, a + b = 0
Now for pairs,
the points P, Q, R. If O is the origin, then
(OP)2 + (OQ)2 + (OR)2 = 2x 2 = y(x + 2y)
[17 Sep. 2020, Shift-II]
⇒ 2x 2 − 2y 2 − xy = 0
85 121 212 217
(a) (b) (c) (d) Coefficient of x 2 + coefficient of y 2 = 2 + − 2 = 0
72 72 72 72
So, there are perpendicular lines.
Sol. (b)
We are given with a cubic equation which 12. The polar of (1, − 2) with respect to
represents 3-lines. x 2 + y 2 − 10 x − 10 y + 25 = 0 is
Let lines have, [17 Sep. 2020, Shift-II]
y − m1 x = 0; y − m2 x = 0 (a) 4 x + 7 y + 30 = 0 (b) 4 x + 7 y − 30 = 0
and y − m3 x = 0 (c) 4 x − 7 y + 30 = 0 (d) x + y = 0
Then, (y − m1 x) (y − m2 x)(y − m3 x) = 0 Sol. (b)
is the combined equations Given circle is,
Which in this case is, x 2 + y 2 − 10 x − 10 y + 25 = 0
y 3 − 6 xy 2 + 11 x 2 y − 6 x 3 = 0 Given point ‘P’ is (1, −2)
By comparison we are able to put above equation Equation of polar of point P with respect to circle
are, is,
(y − x)(y − 2x)(y − 3x) = 0 x + 1  y − 2 + 25 = 0
x ⋅1 + y (−2) − 10   −10  
So, lines are,  2   2 
L1 ⇒ y − x = 0 ⇒ x − 2y − 5(x + 1) − 5(y − 2) + 25 = 0
L2 ⇒ y − 2x = 0 ⇒ x − 2y − 5x − 5 − 5y + 10 + 25 = 0
and L3 ⇒ y = 3x = 0 ⇒ −4 x − 7 y + 30 = 0
Their intersection points with ⇒ 4 x + 7 y − 30 = 0
x + y = 1 are,
13. The lines represented by ax 2 + 2hxy + by 2 = 0
P ≡ x + y = 1 and y − x = 0
are perpendicular to each other, if ........
P ≡  , 
1 1
⇒ [17 Sep. 2020, Shift-II]
 2 2
(a) h2 = a + b (b) a + b = 0
Q ≡ x + y = 1 and y − 2x = 0
(c) h2 = ab (d) h = 0
Q ≡  , 
1 2
⇒ Sol. (b)
 3 3
Pair of straight lines through origin are given by
R ≡ x + y = 1 and y − 3x = 0
ax 2 + 2hxy + by 2 = 0
R ≡  , 
1 3

 4 4 Angle ‘θ’ between lines is given by,
So, QP 2 + OQ2 + OR2 2 h2 − ab
tanθ =
a+ b
=  +  +  +  +  + 
1 1 1 4 1 9
 4 4   9 9   16 16  When lines are perpendicular θ = 90°
2 5 5 36 + 40 + 45 121 ⇒ tanθ = ∞ ⇒ a + b = 0
= + + = =
4 9 8 72 72 14. A light ray emerging from a point source at
11. Which of the following pair of straight lines A(2 , 3) is reflected on the Y -axis at point ‘B’
intersect at right angles? [17 Sep. 2020, Shift-II] and passes through point C(5 , 10), then the
coordinates of ‘B’ are [17 Sep. 2020, Shift-II]
(a) 2 x2 = y( x + 2 y) (b) ( x + y)2 = x( y + 3 x)
(c) 2 y( x + y) = xy (d) y = ± 2 x (a) (5, 0) (b) (0, 5) (c) (0, 2 ) (d) (2, 0)
194 AP EAMCET Chapterwise Mathematics

Sol. (b) y
Sol. (a)
Let the angle of inclination of line θ, and as passed
(5, 10) B
+θ through point A(1,2), so equation of line is
P (0, y) Normal
–θ x −1 y − 2 6
= =±
(2, 3) A
cosθ sinθ 3
∴ General point on the line is
6 6
P(1 ± cosθ, 2 ± sinθ)
Let point of reflection is (0, y) then, by law of 3 3
reflection, Let the point P on the straight line x + y = 4, so
Angle of incidence = Angle of reflection 6
⇒ tanθ = tan(−θ) 3± (sin θ + cosθ) = 4
3
{As angles are measured in opposite directions.}
6
y − 3 y − 10 ⇒ ± (sinθ + cosθ) = 1
⇒ m1 = − m2 ⇒ = 3
−2 5
On squaring both sides, we get
⇒ 5y − 15 = − 2y + 20 ⇒ 7 y = 35, y = 5
2 (1 + sin 2θ) = 3
So, point is (0, 5).
1
⇒ sin 2θ = ⇒ 2θ = 30° and 150° ⇒θ = 15° and 75°
15. If a diagonal of a square is along the line 2
8 x − 15 y = 0 and one of its vertices is (1, 2),
then the equations of the sides of the square 17. The lines ax 2 + 2hxy + by 2 = 0 are at right
passing through this vertex are angles if [18 Sep. 2020, Shift-I]
[18 Sep. 2020, Shift-I] (a) a + b = 0 (b) a + b = 1
(a) 23 x − 7 y + 9 = 0, 7 x + 23 y + 53 = 0 (c) h2 − ab = 0 (d) a = b
(b) 23 x − 7 y − 9 = 0, 7 x + 23 y − 53 = 0
Sol. (a)
(c) 23 x + 7 y − 9 = 0, 7 x + 23 y − 53 = 0
The acute angle (θ) between lines represented by
(d) 23 x + 7 y − 9 = 0, 7 x + 23 y + 53 = 0
equation ax 2 + 2h xy + by 2 = 0, is
Sol. (b)  2 h2 − ab 
Equation of line passes through point (1, 2) and θ = tan−1  
inclined with an angle 45° with respect to line  |a + b| 
8 x − 15y = 0, because sides of a square inclined If lines are perpendicular then a + b = 0.
with diagonal at 45°, is
8 / 15 ± 1 18. The angle between the straight lines
y− 2= (x − 1) x 2 + 4 xy + y 2 = 0 is .......
1 m  × 1
8 [18 Sep. 2020, Shift-I]
 15  (a) 30° (b) 45°
8 ± 15 (c) 60° (d) 90°
⇒ y− 2= (x − 1)
15 m 8 Sol. (c)
23 −7 Let the angle (θ) between the straight lines
⇒ y − 2 = (x − 1) and y − 2 = (x − 1)
7 23 x 2 + 4 xy + y 2 = 0
⇒ 23x − 7 y − 9 = 0 and 7 x + 23y − 53 = 0 2 (2)2 − 1  2 h2 − ab 
Then, tan θ = Q tanθ = 
16. A straight line is drawn through the point 1+1  (a + b) 
A(1, 2) such that its point of intersection with 2 3
= = 3
6 2
the straight line x + y = 4 is at a distance
3 ⇒ θ = 60°
from the given point ‘A’. Find the angle which 19. The combined equation of the lines passing
the line makes with the positive direction of through the origin and having slopes 2/3 and
X -axis. [18 Sep. 2020, Shift-I] −2 / 3 is [18 Sep. 2020, Shift-I]
(a) θ = 15° and 75° (b) θ = 75° and 45° (a) 2 x2 − 9 y2 = 0 (b) 4 x2 − xy − 9 y2 = 0
(c) θ = 45° and 60° (d) θ = 60° and 30° (c) 4 x2 − 9 y2 = 0 (d) 4 x2 + xy − 9 y2 = 0
Straight Line and Pair of Straight Lines 195

Sol. (c) ⇒ [(1 + ma) − (1 + mb)] (1 + mc)


The equation of lines passes through origin having = [(1 + mb) − (1 + mc)](1 + ma)
2 2 ⇒ 2 (1 + ma) (1 + mc) = (1 + mb) (1 + mc + 1 + ma)
slopes and − are respectively 2x + 3y = 0 and
3 3 ⇒ 2 (1 + ma) (1 + mc) = (1 + mb) (2 + ma + mc)
2x − 3y = 0, so combined equation is
(2x − 3y)(2x + 3y) = 0 ⇒ 4 x 2 − 9 y 2 = 0. 22. If (a , 8) is a point on the join of (2, 5) and
(4 , − 1) then [18 Sep. 2020, Shift-I]
20. A homogeneous equation of second degree in 8 3
x and y represents which of the following? (a) a = (b) a = (c) a = 1 (d) a = − 1
3 8
[18 Sep. 2020, Shift-I]
(a) Two lines Sol. (c)
(b) A pair of straight lines through the origin Since points A(a , 8), B(2, 5) and C(4,−1) are
(c) Only one line through origin collinear, so
(d) A circle whose centre isn’t the origin Slope of line AB and slope of line BC are same
3 6
Sol. (b) = ⇒ a − 2 = −1 ⇒ a = 1
a − 2 −2
A homogeneous equation of second degree in
x and y, ax 2 + 2 hxy + by 2 = 0, represents a pair of 23. The sine of the angle between the pair of
straight lines through origin, provided h2 ≥ ab. lines represented by the equation
21. Given that lines L1 : y = ma x , L2 : y = mb x and x 2 − 7 xy + 12 y 2 = 0 is [18 Sep. 2020, Shift-II]
L 3 : y = mc x make equal intercepts on the line 1 1 1 1
(a) (b) (c) (d)
x + y = 1, then [18 Sep. 2020, Shift-I] 12 13 170 11
(a) 2(1 + ma ) (1 + mc ) = (1 + mb ) (1 + mc ) Sol. (c)
(b) 2(1 + ma ) (1 + mc ) = (1 + mb ) (2 + ma + mc ) x 2 − 7 xy + 12y 2 = 0
(c) (1 + ma ) (1 + mb ) = (2 + mc ) (1 + ma + mc ) a = 1(2h = −7) b = 12
(d) (1 + ma )(1 + mb ) = (1 + mb ) (2 + ma + mc )
49 1
2 − 1 ⋅12 2.
Sol. (b) 2 h2 − ab 4 2
tan θ = = =
Since, the point of intersecting of given lines a+ b 13 13
L1 : y = ma x, L2 : y = mb x and L3 : y = mc x
with line x + y = 1 are 1
tan θ =
 1 ma   1 mb  13
A , , B  ,  and
 1 + ma 1 + ma   1 + mb 1 + mb 
 1 mc 
C ,  respectively.
 1 + mc 1 + mc 
√170 1
It is given that,
AB = BC ⇒ AB2 = BC 2
2 2 θ
 1 1   ma mb 
⇒ −  +  −  13
 1 + ma 1 + mb   1 + ma 1 + mb  1
2 2 sin θ =
 1 1   mb mc  170
= −  +  − 
 1 + mb 1 + mc   1 + mb 1 + mc  Hence, opiton (c) is correct.
(mb − ma)2 (ma − mb)2 24. The value of ‘k’ for which the equation
⇒ +
(1 + ma) (1 + mb)
2 2
(1 + ma)2 (1 + mb)2 x 2 − 4 xy − y 2 + 6 x + 2 y + k = 0 represents a
(mc − mb)2 (mb − mc)2 pair of straight lines is equal to ........
= + [18 Sep. 2020, Shift-II]
(1 + mb) (1 + mc)
2 2
(1 + mb)2 (1 + mc)2
4 −3
(ma − mb)2 (mb − mc)2 (a) (b)
⇒ 2 =2 5 5
(1 + ma) (1 + mb)
2 2
(1 + mb)2 (1 + mc)2 −4 3
(c) (d)
⇒ (ma − mb) (1 + mc) = (mb − mc) (1 + ma) 5 5
196 AP EAMCET Chapterwise Mathematics

Sol. (c) 26. The equation of lines passing through (5, 3)


x 2 − 4 xy − y 2 + 6 x + 2y + k = 0 …(i) and perpendicular to 2 x + y − 7 = 0 is
a = 1, 2h = −4, b = −1, 2g = 6, 2 f = 2, c = k [18 Sep. 2020, Shift-II]
Eq. (i) Represents a pair of straight line (a) 2 y − x − 2 = 0 (b) 2 y − x + 2 = 0
∆=0 (c) x + y − 8 = 0 (d) 2 y − x − 1 = 0
⇒ abc + 2 fgh − af 2 − bg2 − ch2 = 0 Sol. (d)
1 −1)(k) + 2(1)(3)(−2) − 1(1)2 + 1(3)2 − k(−2)2 = 0
()( 2x + y + 7 = 0 ⇒ P = (5, 3)
− k − 12 − 1 + 9 − 4k = 0 Required line is b (x − x1 ) − a (y − y1 ) = 0
−5k − 4 = 0 1(x − 5) − 2(y − 3) = 0
−4
k= x − 5− 2y + 6 = 0
5 x − 2y + 1 = 0
Hence, option (c) is correct. ⇒ 2y − x − 1 = 0
25. A straight line L1 passing through A(3, 1) Hence, option (d) is correct.
meets the coordinate axes at P and Q such 27. If the sum of the slopes of the lines given by
that its distance from the origin O is 4 x 2 + 2 λxy − 7 y 2 = 0 is equal to the product
maximum. Then area of ∆OPQ is ....... sq.
of the slopes, then λ is equal to
units [18 Sep. 2020, Shift-II]
[18 Sep. 2020, Shift-II]
100 25 50 200
(a) (b) (c) (d) (a) − 4 (b) 4 (c) − 2 (d) 2
3 3 3 3
Sol. (c) Sol. (c)
4 x 2 + 2λxy − 7 y 2 = 0
A = (31
,)
a = 4, 2h = 2λ , b = −7
Let slope of line be m
Given, sum of slopers = product of slopers
∴ y − y1 = m(x − x1 ) [be the required line]
m1 + m2 = m1 m2
y − 1 = m(x − 3)
−2h a − 2λ 4
y − 1 = mx − 3m = ⇒ = ⇒ λ = −2
b b −7 −7
mx − y + (1 − 3m) = 0 …(i)
Hence, option (c) is correct.
A (–2, 3)
28. The pair of straight lines represented by the
equation 3 dx 2 − 5 xy + (d 2 − 2) y 2 = 0 . If the
F
E lines are perpendicular to each other, for how
many values of d this condition will be
satisfied? [18 Sep. 2020, Shift-II]
B (1, –2) D C (2, 1) (a) 0 (b) 2 (c) 1 (d) 3

The greatest distance of line from origin passes Sol. (b)


through A (3, 1) is perpendicular to the given line. Given, pair of straight lines is
∴ OA ⊥ PQ 3dx 2 − 5xy + (d 2 − 2) y 2 = 0
Slope of OA × Slope of PQ = –1 Since, lines are perpendicular
1
× m = −1 ⇒ m = −3 ⇒ x 2 coefficient + y 2 coefficient = 0
3 3d + d 2 − 2 = 0
Put, m = −3in Eq. (i), d 2 + 3d − 2 = 0
−3x − y + (1 − 3)(−3) = 0 −3 ± 9 − 4 ⋅(−2) −3 ± 17
d= =
− 3x − y + (10) = 0 2⋅ 1 2
3x + y + 10 = 0 −3 + 17 −3 − 17
= (or) =
1 c2 1 100 50 2 2
∴Area of ∆ POQ = = = sq units
2 ab 2 3 ×1 3 Number of possible value of d are 2
Hence, option (c) is correct. Hence, option (b) is correct.
Straight Line and Pair of Straight Lines 197

29. If the slope of the line ax + (3 − a) y + 7 = 0 is Q The obtain lines makes equal intercepts with
the axes, so
7 then the value of integral part of ‘a’ is
λ −1 λ −1
[18 Sep. 2020, Shift-II] = ⇒ |λ − 4| = |5λ + 3| {Q λ ≠ 1}
5λ + 3 λ−4
(a) 3 (b) 7 (c) 0.5 (d) 3.5
7 1
Sol. (a) ⇒ λ=− ,
4 6
Given line ax + (3 − a) y + 7 = 0
∴Equation of required lines are
Given slope = 7
− 23x − 23y + 11 = 0 and 23x − 23y + 5 = 0
−a a
⇒ =7 ⇒ = 7 ⇒ a = 7a − 21 or 23x + 23y − 11 = 0 and 23x − 23y + 5 = 0
3− a 3− a
7 Hence, option (a) is correct.
6a = 21 ⇒ a = ⇒ a = 3.5
2 32. Find the equation of pair of straight lines
a = 3 + 0.5 ⇒ [a] = 3 that bisect the angles between the lines
Hence, option (a) is correct. represented by ax 2 + 2 hxy + by 2 = 0 .
30. Find the equation of the right bisector of the [21 Sep. 2020, Shift-I]
line segment joining the points (3 , 4) and x2 + y2 xy x 2 + y2 xy
(a) = (b) =
(− 1, 2). [21 Sep. 2020, Shift-I] a+ b h a−b h
(a) 2 x + y − 5 = 0 (b) 2 x− y + 5 = 0 x2 + y2 h x 2 − y2 xy
(c) = (d) =
(c) 2 x + y + 5 = 0 (d) 2 x − y − 5 = 0 a− b xy a−b h
Sol. (a) Sol. (d)
The right angle bisector of the line segment The equation of pair of straight lines that bisect
joining points A(3, 4) and B(−1, 2) passes through the angles between the lines represented by
the mid-point of A and B perpendicular to line x 2 − y2 xy
segment AB. ax 2 + 2hxy + by 2 = 0 is =
a−b h
Now, the mid-point of A(3, 4) and B(−1, 2) is
3 − 1 4 + 2 Hence, option (d) is correct.
M  ,  = M(1, 3)
 2 2  33. If the lines 3 x + y − 2 = 0, px + 2 y − 3 = 0 and
3 − (−1) 2 x − y − 3 = 0 are concurrent, then p =
and slope of right angle bisector is = − = −2
4− 2 [21 Sep. 2020, Shift-I]
∴ Equation of required right angle bisector is (a) − 5 (b) 5 (c) 3 (d) − 3
y − 3 = −2 (x − 1)
Sol. (b)
⇒ 2x + y = 5 or 2x + y − 5 = 0
It is given that lines
Hence, option (a) is correct.
3x + y − 2 = 0, px + 2y − 3 = 0 and 2x − y − 3 = 0
31. The equation of the line through the are concurrent, so
intersection of 3 x − 4 y + 1 and 5 x + y − 1 = 0 3 1 −2
which cuts off equal intercepts on the axes is p 2 −3 = 0
given by [21 Sep. 2020, Shift-I] 2 −1 −3
(a) 23 x + 23 y − 11 = 0 (b) 23 x + 23 y + 11 = 0
⇒ 3(− 6 − 3) − 1(− 3p + 6) − 2(− p − 4) = 0
(c) 23 x − 23 y − 11 = 0 (d) 23 x − 23 y + 11 = 0
⇒ − 27 + 3p − 6 + 2p + 8 = 0
Sol. (a) ⇒ 5p − 25 = 0 ⇒ p = 5
Equation of family of lines passes through the Hence, option (b) is correct.
intersection of lines 3x − 4 y + 1 = 0 and
5x + y − 1 = 0 is 34. If the pair of straight line given by
(3x − 4 y + 1) + λ (5x + y − 1) = 0 Ax 2 + 2 Hxy + By 2 = 0 , where (H 2 > AB), forms
⇒ (3 + 5λ) x + (λ − 4) y + (1 − λ) = 0 an equilateral triangle with the line
x y ax + by + c = 0 , then (A + 3 B) (3 A + B) =
⇒ + = 1, {λ ≠ 1}
λ −1 λ −1 [21 Sep. 2020, Shift-I]
5λ + 3 λ − 4 (a) 4H2 (b) 2 H2 (c) − 2 H2 (d) − 4H2
198 AP EAMCET Chapterwise Mathematics

Sol. (a) ⇒ pq + 1 = 0
According to the given information in the Hence, option (c) is correct.
question the angle between lines represented by
π 37. If one of the lines 2 x 2 − xy + by 2 = 0 passes
the equation Ax 2 + 2Hxy + By 2 = 0 must be .
3 through the point (−4 , − 2), then b2 =
π 2 H − AB
2 [21 Sep. 2020, Shift-II]
So, tan = (a) − 6 (b) 36 (c) 4 (d) 16
3 |A + B|
⇒ 3(A + B) = 4 (H − AB)
2 2 Sol. (b)
⇒ 3A2 + 10 AB + 3B2 = 4H 2 It is given that one of the lines 2x 2 − xy + by 2 = 0
⇒ 3A + 9 AB + AB + 3B2 =
2
4H 2 passes through the point (−4, −2), so
⇒ 3A(A + 3B) + B(A + 3B) = 4H 2 2(−4)2 − (− 4)(−2) + b (−2)2 = 0
⇒ (A + 3B) (3A + B) = 4H 2 ⇒ 32 − 8 + 4b = 0 ⇒ b = − 6
Hence, option (a) is correct. ∴ b 2 = 36
35. If the equation, Hence, option (b) is correct.

x 2 + 2 2 xy + 2 y 2 + 4 x + 4 2 y + 1 = 0 38. The equation of the line through the point


represents a pair of straight lines which are (2, 3) such that its x-intercept is twice its
parallel to each other. Find the distance y-intecept is ……… . [21 Sep. 2020, Shift-II]
between them is [21 Sep. 2020, Shift-I] (a) x + 2 y − 8 = 0 (b) 4 x + y + 2 = 0
(c) 2 x + 33 y − 46 = 0 (d) 4 x + 3 y − 11 = 0
(a) 4 units (b) 2 units
(c) 2 3 units (d) 4 3 units Sol. (a)
Sol. (b) It is given that line makes x-intercept twice as
Given equation of pair of parallel straight lines compare to y-intercept, so let the equation of line
x 2 + 2 2 xy + 2y 2 + 4 x + 4 2 y + 1 = 0 is
x y
(x + 2y + 2 + 3) (x + 2y + 2 − 3) = 0 + =1 …(i)
2a a
Now, distance between parallel lines Q Line (i) passes through (2, 3) so
x + 2y + 2 + 3 = 0 and x + 2y + 2 − 3 = 0 is 2 3 4
+ =1 ⇒ =1⇒a = 4
|2 + 3 − 2+ 3| 2 3 2a a a
= = 2 units.
1+ 2 3 ∴Equation of required line is
Hence, option (b) is correct. x + 2y − 8 = 0
Hence, option (a) is correct.
36. If the pairs of straight lines x 2 − 2qxy − y 2 = 0
and x 2 − 2 pxy − y 2 = 0 bisect the angles 39. The point of intersection of the pair of lines
between each other, then which of the x 2 + xy + 2 y 2 − 3 x + 2 y + 4 = 0 is
following is correct? [21 Sep. 2020, Shift-II] [21 Sep. 2020, Shift-II]
(a) 1 − pq = 0 (b) pq − 1 = 0 (a) (1, 2) (b) (−12, )
(c) pq + 1 = 0 (d) pq = 0 (c) (−2,1) (d) (2, − 1)
Sol. (c) Sol. (d)
It is given that the pair of straight lines The point of intersection of the pair of lines
x 2 − 2pxy − y 2 = 0 and x 2 − 2qxy − y 2 = 0 bisect, the f (x , y) ≡ x 2 + xy + 2y 2 − 3x + 2y + 4 = 0 is same as
angles between each other, then equations the intersection of curve obtaining after partial
x 2 − y2 xy differentiating of the curve w.r.t. x and y
= and x 2 − 2qxy − y 2 = 0 represents respectively.
1 − (−1) − p
∂f
2 Q = 2x + y − 3 = 0 …(i)
same line, so x 2 + xy − y 2 = 0 represents ∂x
p
∂f
2
x 2 − 2qxy − y 2 = 0 on comparing, we get = −2q and = x + 4y + 2 = 0 …(ii)
p ∂y
Straight Line and Pair of Straight Lines 199

By cross-multiplication method, we get 4


Q Sum of slopes = m + 3m = − = −4 ⇒ m = −1 and
x −y 1 1
= = a a
2 + 12 4 + 3 8 − 1 product of the slopes = m(3m) = ⇒ m2 =
⇒ (x , y) = (2, − 1) 1 3
a
Hence, option (d) is correct. ∴ (−1) = ⇒ a = 3
2

3
40. The distance between the lines 3 x + 4 y = 9
Hence, option (c) is correct.
and 6 x + 8 y = 15 is equal to ……… units
[21 Sep. 2020, Shift-II] 43. If 3a + 5b + 6 c = 0 then the family of lines
1 3 5 7 ax + by + c = 0 pass through the fixed point
(a) (b) (c) (d) [21 Sep. 2020, Shift-II]
10 10 10 10
(a)  ,  (b)  ,  (c)  ,  (d)  , 
5 1 1 1 1 1 1 5
Sol. (b)  6 2  2 3  3 2  2 6
The distance between given parallel lines
3x + 4 y − 9 = 0 Sol. (d)
6 x + 8 y − 15 = 0 Given equation of family of lines
18 − 15 3 ax + by + c = 0
is = unit
36 + 64 10 3a + 5b
Q 3a + 5b + 6c = 0, so ax + by − =0
Hence, option (b) is correct. 6
⇒ 3 a(2x − 1) + b(6 y − 5) = 0
41. If the length of the intercept made on the So, the family of lines pses through the fixed
line y = ax by the lines y = 2 and y = 6 is less
point  ,  .
1 5
than 5 [21 Sep. 2020, Shift-II]  2 6
−4 4
(a) a ∈ (−∞, ∞ ) (b) a ∈  ,  Hence, option (d) is correct.
 3 3
−3 4
(c) a ∈  ,  (d) a <
−4
or a >
4 44. If the equation 4 x 2 + hxy + y 2 = 0 represent
 4 3 3 3 coincident lines, then ‘h’ is equal to
Sol. (d) [22 Sep. 2020, Shift-I]
The point of intersection of lines y = ax and y = 2 (a) 1 (b) 2 (c) 3 (d) 4

is A , 2 and of the lines y = ax and y = 6 is


2 Sol. (d)
a 
Since the equation 4 x 2 + hxy + y 2 = 0 represents
B , 6 , so
6
the coincident lines, then
a  2
 h  − 4 × 1 = 0 ⇒ h = − 4, 4
 
 2
2

AB = 6− 2
  + (6 − 2) < 5
2
(given)
a a Hence, option (d) is correct.
16
⇒ + 16 < 25 45. If the equation
a2
ax 2 + 2 hxy + by 2 + 2 gx + 2 fy + c = 0 represents
⇒ a ∈  −∞ , −
16 4  4 
⇒ a2 >  ∪  ,∞ two straight lines equidistant from the
9  3  3 
origin, then f 4 − g 4 = [22 Sep. 2020, Shift-I]
Hence, option (d) is correct.
(a) bf 2 − ag 2 (b) ag 2 − bf 2
42. If slope of one line of ax + 4 xy + y = 0 is 3
2 2
(c) c(bf 2 − ag 2 ) (d) c(af 2 − bg 2 )
times the other, then the value of ‘a’ is Sol. (c)
[21 Sep. 2020, Shift-II]
Let the lines represented by equation
(a) −3 (b) −1 (c) 3 (d) 1 ax 2 + 2hxy + by 2 + 2gx + 2 fy + c = 0
Sol. (c) are y = m1 x + c1 and y = m2 x + c 2
Let the slope of lines are m and 3m respectively So, ax 2 + 2hxy + by 2 + 2gx + 2 fy + c
which are represented by ax 2 + 4 xy + y 2 = 0 = (m1 x − y + c1 )(m1 x − y + c 2)
200 AP EAMCET Chapterwise Mathematics

m1 m2 −(m1 + m2) 1 24 − 3a 
⇒ = = ⇒ a   = 24
a 2h b  2 
m1 C2 + m2 C1 −(C1 + C2) C1 C2 ⇒ 3 a 2 − 24 a + 48 = 0
= = = …(i)
2g 2f c Q Discriminant = 242 − 4 × 3 × 48 = 0
Now, according to the information given in ∴ a = 4 and b = 6
question, as lines are equidistance from origin, so If a is positive and b is negative then ab = − 24 and
|C1 | |C2| 3 a + 2 b = 24.
=
1 + m1 2
1 + m22 24 − 3a 
⇒ 2  = − 24 ⇒ 3 a − 24a − 48 = 0
2
 2 
⇒ C12 (1 + m22) = C22 (1 + m12)
⇒ C12 − C22 = C22 m12 − C12 m22 Q Discriminant = 242 + 4 × 3 × 48 > 0
⇒ (C1 + C2)(C1 − C2) = (C2 m1 + C1 m2) 24 ± 2 × 242
∴ a= =4+ 4 2 Qa > 0
(C2 m1 − C1 m2) …(ii) 6
From Eq. (i), we have and b=6−6 2
2f 2g If a is negative and b is positive, then ab = − 24
C1 + C2 = − and m1 C2 + m2 C1 =
b b and 3 a + 2 b = 24
c 9 ⇒ a = 4 − 4 2 and b = 6 + 6 2
and C1 C2 = and m1 m2 =
b b
And at the last it is not possible that a and b both
4 f 2 4c f 2 − bc 2 are negative.
∴ |C1 − C2|= 2
− =2 = f 2 − bc
b b b2 |b| So, 3 triangles are possible.

4 g2 ac 2 47. The acute angle between the lines x − y = 0 ,


and |C2 m1 − C1 m2|= −4 2 = g2 − ac
b 2
b |b| and y = 0 is [22 Sep. 2020, Shift-I]
π π π 5π
Therefore from Eq. (ii), we have (a) (b) (c) (d)
6 4 3 6
2f 2 2g 2
− × f 2 − bc = × g2 − ac
b |b| b |b| Sol. (b)
Equation of given lines x − y = 0 and y = 0.
⇒ f ( f − bc) = g (g − ac) (on squaring both sides)
2 2 2 2
 |1 × 0 − 1 × 1 + 0 × 0 |
⇒ f 4 − g4 = c(bf 2 − ag2) θ = cos− 1  
 1+1 1 
46. The number of possible straight lines passing
π
= cos− 1   =
1
through the point (2 , 3), while forming a
 2 4
triangle with coordinate axes enclosing an
area 12 sq. units is [22 Sep. 2020, Shift-I] 48. The point of intersection of lines denoted by
(a) 1 (b) 2 (c) 3 (d) 4 3 x 2 − 11 xy + 10 y 2 − 7 x + 13 y + 4 = 0 is
Sol. (c) [22 Sep. 2020, Shift-I]
Let the equation of line forming a triangle having (a) (3, 1) (b) (1, 3)
area 12 sq. units is (c) (0, 0) (d) (−3, − 1)
x y Sol. (a)
+ =1 ...(i)
a b Equation of given lines
So, | ab | = 24 ...(ii) f (x , y) ≡ 3x 2 − 11 xy + 10 y 2 − 7 x + 13y + 4 = 0
Q Line (i) passes through point (2, 3), so The point of intersection of line is same as the
2 3
+ =1 ∂f ∂f
...(iii) point of intersection of lines = 0 and =0
a b ∂x ∂y
from Eqs. (ii) and (iii), we get ∂f
∴ = 6 x − 11 y − 7 = 0
| 2 b + 3 a | = 24 ...(iv) ∂x
If a and b are positive, then ∂f
and = − 11 x + 20 y + 13 = 0
ab = 24 and 3 a + 2 b = 24 ∂y
Straight Line and Pair of Straight Lines 201

by cross multiplication method, we have Sol. (d)


x − y 1 Given pair of lines
= =
− 143 + 140 78 − 77 120 − 121 xy + x + y + 1 = 0
x y 1 x(y + 1) + (y + 1) = 0
⇒ = = (y + 1) (x + 1) = 0
3 1 1
⇒ x = − 1 or y = −1
∴Point of intersection is (3, 1).
and second pair of line
49. If the lines joining the origin to the points of xy + 3x + 3y + 9 = 0
intersection of y = mx + 1 and x 2 + y 2 = 1 are
x(y + 3) + 3(y + 3) = 0
perpendicular, then ……… (x + 3) (y + 3) = 0
[22 Sep. 2020, Shift-I]
x = − 3 or y = − 3
(a) m = −1only (b) m = 1only ∴ Quadrilateral so formed by above lines is a square
(c) m = ±1 (d) m = 0
Hence, option (d) is correct.
Sol. (c)
52. The equation of pair of straight lines parallel
Equation of line joining origin to the points of to X -axis and touching the circle
intersection of y = mx + 1 and x 2 + y 2 = 1 is x 2 + y 2 − 6 x − 4 y − 12 = 0 [22 Sep. 2020, Shift-II]
x 2 + y 2 = (y − mx)2
(a) y2 − 4 y − 21 = 0 (b) y2 + 4 y − 21 = 0
⇒ x 2 + y 2 = y 2 − 2mxy + m2 x 2 (c) y2 − 4 y + 21 = 0 (d) y2 + 4 y + 21 = 0
⇒ (1 − m ) x + 2mxy = 0
2 2
Sol. (a)
The line are perpendicular if coefficient of
Given circle is x 2 + y 2 − 6 x − 4 y − 12 = 0
x 2 + coefficient of y 2 = 0
⇒ 1 − m2 + 0 = 0 ⇒ m = ± 1 Centre = (3, 2)
Radius = (3)2 + (2)2 + 12 = 9 + 4 + 12 = 25
50. The equation of the line through the point
(−1, 3) in symmetrical form, when the angle r=5
made by the line with the positive direction Since, tangents are paralle to X-axis
of X -axis is 120°, is given by ∴ Let y = k be the tangent to the given circle
[22 Sep. 2020, Shift-I] ⇒ y − k = 0 touches the circle
( x + 1) ( y − 3) ( x + 1) ( y + 3) ∴Perpendicular distance
(a) = =r (b) = =r
−1 / 2 3 /2 1/ 2 3 /2 from C(3, 2) to y − k = 0 = radius
( x + 1) ( y + 3) ( x + 1) ( y − 3) |2 − k|
(c) = =r (d) = =r = 5 ⇒ |2 − k| = 5
−1 / 2 3 /2 1/ 2 3 /2 1
Sol. (a) 2− k = ± 5
The equation of line through the point (x1 , y1 ) in 2 − k = 5 (or) 2 − k = − 5
symmetrical form, if it’s inclination with positive k = − 3 (or) k = 7
direction of X-axis is θ is
∴ Required equation of tangents are
x − x1 y − y1
= =r y = − 3 and y = 7
cos θ sin θ
⇒ (y + 3) = 0 and (y − 7) = 0
So, the equation of the line in symmetric form ∴Equation of pair of tangents is (y + 3)(y − 7) = 0
where (x1 , y1 ) = (− 1, 3) and θ= 120° is
x +1 y−3 y 2 − 4 y − 21 = 0
= =r Hence, option (a) is correct.
cos 120° sin 120°

x +1 y− 3
= =r
53. If the lines y = 3 x + 1 and 2 y = x + 3 are
− 1/ 2 3/ 2 equally inclined to the line y = mx + 4, then
the value of ‘m’ is equal to [22 Sep. 2020, Shift-II]
51. The quadrilateral formed by the pairs of line
xy + x + y + 1 = 0, xy + 3 x + 3 y + 9 = 0 is 1± 3 2 − 1± 5 2
(a) (b)
[22 Sep. 2020, Shift-II] 7 7
1± 5 2
(a) Parallelogram (b) Rhombus (c) 0 (d)
(c) Rectangle (d) Square 7
202 AP EAMCET Chapterwise Mathematics

Sol. (d) Since, reflection coincides to each other, so


Given Lines are, π
3θ = π ⇒ θ =
y = 3x + 1 ⇒ m1 = 3 ⇒ 2y = x + 3 3
1 3 1 Hence, option (a) is correct.
y = x + ⇒ m2 =
2 2 2 55. The equation of the line passing through the
⇒ y = mx + 4 point of intersection of lines 2 x − y + 2 = 0
According to the question, and x + y + 4 = 0 and the point (5 , − 2) is
m1 − m m2 − m [23 Sep. 2020, Shift-I]
=
1 + m1 m 1 + m2 m (a) y + 2 = 0 (b) y − 2 = 0
(c) 4 x − 3 y − 6 = 0 (d) x − y − 7 = 0
m1 − m2 
{Q Angle between lines is tanθ = 
1 + m1 m2  Sol. (a)
1 Given, 2x − y + 2 = 0 …(i)
−m x+ y+ 4= 0
3− m …(ii)
= 2
1 + 3m 1 Eqs. (i)+(ii) ⇒ 3x + 6 = 0 ⇒ x = − 2 ⇒ y = − 2
1+ m
2 ∴Point of intersection P = (−2, −2)
3− m 1 − 2m Given Q = (5, −2)
= ←→ y − y1
1 + 3m 2+ m Equation of line PQ is y − y1 = 2 (x − x1 )
x 2 − x1
3− m 1 − 2m
⇒ =± −2 + 2
1 + 3m 2+ m y + 2= (x + 2) ⇒ y + 2 = 0
5+ 2
Case (i) Hence, option (a) is correct.
3− m  1 − 2m 
=−  56. The length of the perpendicular from (1, − 2)
1 + 3m  2 + m
(3 − m)(2 + m) = (2m −1) (1 + 3m) to the line 12 x + 5 y + 63 = 0 is ……
6 + m − m2 = 6m2 − m − 1 [23 Sep. 2020, Shift-I]
7m2 − 2m − 7 = 0 85
(a) 4 (b) 5 (c) 6 (d)
2± (−2) − 4 ⋅ 7(−7)
2 13
m=
2⋅ 7 Sol. (b)
2± 4 + 196 2 ± 10 2 1 ± 5 2 Given, P = (1, −2)
= = =
14 14 7 L : 12x + 5y + 63 = 0
|ax1 + by1 + c1 |
1± 5 2 Length of perpendicular =
∴ m= a2 + b2
7
Hence, option (d) is correct. |12 (1) + 5 (−2) + 63|
=
(12)2 + (5)2
54. If S1 and S2 are two straight lines such that
|12 − 10 + 63| 65
the reflection of S1 in S2 and the reflection of = = = 5 units
169 13
S2 in S1 coincide, the angle between S1 and S2
Hence, option (b) is correct.
is equal to [23 Sep. 2020, Shift-I]
(a)
π
(b)
π 57. If the line y = mx is one of the bisectors of
3 6 x 2 + 4 xy − y 2 = 0 , then value of 2m =
π
(c) (d) Data insufficient [23 Sep. 2020, Shift-I]
4
(a) −1 + 5 (b) 1 + 5 (c) −1 − 51 (d) 1 − 51
Sol. (a)
Sol. (a)
Let angle between two lines S1 and S2 be θ, then
angle between S1 and reflection of S2 with respect Given Equation of pair of straight line is
to S1 is θ and similarly the angle between S2 and x 2 + 4 xy − y 2 = 0
reflection of s1 with respect to S2 is θ, so angle a = 1 |2h = 4| b = − 1
between reflections is 3θ. h= 2
Straight Line and Pair of Straight Lines 203

We know that equation of angle bisector of line Sol. (c)


ax 2 + 2hxy + by 2 = 0 Given equation of pair of line is
x 2 − y2 xy lx 2 + 3xy − 2y 2 − 5x + 5y + k = 0 …(i)
is =
a−b h ∴ a = l| 2h = 3| b = − 2| 2g = − 5| 2 f = 5| c = k
Hence, a = 1, b = −1, h = 2 If Eq. (i) represents, pair of perpendicular lines
x 2 − y2 xy a+ b=0
∴ = = x 2 − y 2 − xy = 0
2 2 l + (−2) = 0
Put, y = mx , we get l = 2 ⇒a = 2 ⇒∆ = 0
x 2 − m2 x 2 − mx 2 = 0 abc + 2 fhg − af 2 − bg2 − ch2 = 0
5 −5 3
⇒ m2 + m − 1 = 0 ⇒ m =
−1 ± 5 (2) (−2) (k) + 2      
 2  2   2
2 2 2 2
−5
− l   + 2  − k   = 0
⇒ 2m = − 1 ± 5 ⇒ 2m = − 1 + 5 5 3
 2  2  2

− k   = 0
58. Statement I Two lines which pass through − 4k −
75 25 25
− +
9
a given fixed point and are equally inclined 4 2 2  4
to two other lines passing through the same 9k 75
−4k − =
point, are always perpendicular to each other. 4 4
Statement II Angle bisectors of two 25 k 75
intersecting lines are always perpendicular to − = k = −3
4 4
each other [23 Sep. 2020, Shift-I]
(a) Both the statements are true and statement II is 60. The equation of the line through the point of
the correct explanation of the statement I. intersection of the lines 3 x − 4 y + 1 = 0 and
(b) Both the statements are true but statement II is not 5 x + y −1 = 0 and making equal non-zero
the correct explanation of the statement I. intercepts on the coordinate axes is
(c) Statement I is true and statement II is false. [20 April 2019, Shift-I]
(d) Statement I is false and statement II is true. (a) 2 x + 2 y = 3 (b) 23 x + 23 y = 6
Sol. (c) (c) 23 x + 23 y = 11 (d) 2 x + 2 y = 7
The angle bisectors are pair of lines which are Sol. (c)
equally inclined to the other two given lines and We have, equation of the line through the point of
passes through the same point of intersection of intersection of the lines 3x − 4 y + 1 = 0 and
lines and the angle bisectors are perpendicular to 5x + y − 1 = 0 is
each other. (3x − 4 y + 1) + λ(5x + y − 1) = 0
B2 ⇒ (3 + 5λ) x + (λ − 4) y = (λ − 1)
x y
⇒ + =1
λ −1 λ −1
3 + 5λ λ−4
B1 According to given information
λ −1 λ −1
= and λ ≠ 1
3 + 5λ λ − 4
⇒ λ − 4 = 5λ + 3
7
L2 ⇒ 4λ = − 7 ⇒ λ = −
L1 4
So, equation of required line is
59. If lx 2 + 3 xy − 2 y 2 − 5 x + 5 y + k = 0 represents  3 − 35 x +  − 7 − 4 y =  − 7 − 1
     
a pair of perpendicular lines, then  4  4   4 
[23 Sep. 2020, Shift-I] 23 23 11
⇒ − x− y=− ⇒ 23x + 23y = 11
(a) k = ± 3, l = ± 2 (b) k = − 22, l = − 12 4 4 4
(c) k = − 3, l = 2 (d) k = − 16, l = 9 Hence, option (c) is correct.
204 AP EAMCET Chapterwise Mathematics

61. The line through P(a , 2), where a ≠ 0, making Sol. (a)
an angle 45° with the positive direction of the The point B(x1 , y1 ) is the reflection of point A w.r.t.
line x − y + 5 = 0, so
x 2 y2
X -axis meets the curve + = 1 at A and D ⇒
x1 − 1 y1 + 2
= =−2
1 + 2+ 5
=−8
9 4 1 −1 2
and the coordinate axes at B and C. If ⇒ x1 = − 7, y1 = 6
PA , PB, PC and PD are in a geometric
so, B(−7, 6)
progression, then 2a = [20 April 2019, Shift-I]
Similarly point C(x 2 , y2) is the reflection of point A
(a) 13 (b) 7 (c) 1 (d) −13 w.r.t., line x + 2y = 0, so
Sol. (a) x 2 − 1 y2 + 2 1−4 6
⇒ = =−2 =
Since equation of line passes through P (a , 2), a ≠ 0 1 2 5 5
making an angle 45° with positive direction of the 11 2
⇒ x2 = and y2 =
X-axis is : 5 5
x−a y−2 So, C  ,  .
r r 11 2
= =r⇒ x =a+ and y = 2 +  5 5
1 1 2 2
2 2 Now, equation of line BC is
So for point B (as it is on X-axis), so, r = − 2 2 2
6−
y−6= 5 (x + 7)
So B(a − 2, 0) for point C (as it is on Y-axis) 11
r = −a 2 −7 −
5
So, C(0, 2 − a) 28 −14
∴ PB = 4 + 4 = 2 2 ⇒ y−6= (x + 7) ⇒ y − 6 = (x + 7)
−46 23
PC = a2 + a2 = a 2 ⇒ 14 x + 23y = 138 − 98 ⇒ 14 x + 23y − 40 = 0
For points A and D Hence, option (a) is correct.
2 2
a + r  2 + r  63. If each line of a pair of lines passing through
   
 2  2 = 1 origin is at a perpendicular distance of 4
+
9 4 units from the point (3, 4), then the equation
2 2
of the pair of lines is
4 a +
r   r  [20 April 2019, Shift-I]
⇒  + 9 2 +  = 36
 2  2 (a) 7 x2 + 24 xy = 0 (b) 7 y2 + 24 xy = 0
 r2  (c) 7 y2 − 24 xy = 0 (d) 7 x2 − 24 xy = 0
⇒13  + 
8a 36 
+  r + 4a = 0, let having roots
2

 2  2 2 Sol. (b)
r1 = PA and r2 = PD, so Let equation of line passes through origin having
4a 2 slope mis y − mx = 0, according to given information
r1 r2 =
13 |4 − 3m|
= 4 ⇒ 16 + 9m2 − 24m = 16 + 16m2
2 1 + m2
8a 2 24
= = (PA)(PD) ⇒ 7m2 + 24m = 0 ⇒ m = 0 or m = −
13 7
Q PA, PB, PC and PD are in GP. so so combined equation of required lines
8a 2
y y +
(PA)(PD) = (PB) (PC) ⇒ = 4a ⇒ 2a = 13 24 
x  = 0 ⇒ 7 y 2 + 24 xy = 0
13  7 
Hence, option (a) is correct. Hence, option (b) is correct.
62. The equation of the perpendicular bisectors 64. Variable straight lines y = mx + c make
of the sides AB and AC of a ∆ABC are
x − y + 5 = 0 and x + 2 y + 5 = 0, respectively. intercepts on the curve y 2 − 4 ax = 0 which
If A is (1, − 2), then the equation of the subtend a right angle at the origin. Then the
straight line BC is [20 April 2019, Shift-I] point of concurrence of these lines y = mx + c
(a) 14 x + 23 y − 40 = 0 (b) 12 x + 17 y − 28 = 0 is [20 April 2019, Shift-I]
(c) 14 x − 29 y − 30 = 0 (d) 7 x − 12 y + 15 = 0 (a) (4a, 0) (b) (2 a, 0) (c) (−4a, 0) (d) (−2 a, 0)
Straight Line and Pair of Straight Lines 205

Sol. (a) Sol. (a)


On homogenisation of the curve y 2 − 4ax = 0 by Point of intersection of lines x + y − 1 = 0 and
line y = mx + c, we are getting combined equation x − y − 1 = 0 is A(1, 0).
of straight lines which subtend a right angle at Similarly, point of intersection of lines
the origin, so x − y − 1 = 0 and x − 3y + 3 = 0 is B(3, 2),
A and point of intersection of lines x − 3y + 3 = 0
and x + y − 1 = 0 is C(0, 1).
C1 B1
1 + 3 + 0 0 + 2 + 1
Now, centroid of ∆ABC is  , 
 3 3 
 4 
=  , 1
3 
B C
A (1, 0)
A1
y − mx 
y 2 − 4ax   = 0 ⇒ c + 4am = 0 …(i) x+y–1=0 x–y–1=0
 c 
On putting the value of ‘c’ in the line, we get
C B
y = m(x − 4a), represent family of line passes (0, 1) x–y–1=0 (3, 2)
through (4a , 0). Hence, option (a) is correct.
Hence, option (a) is correct.
65. A line passing through P(4 , 2) cuts the
coordinate axes at A and B respectively. If O is 67. The quadratic equation whose roots are the
the origin, then the locus of the centre of the coordinates of the circumcentre of the triangle
circum-circle of ∆OAB is [20 April 2019, Shift-II] formed by the points (−2 , − 1), (6 , − 1)(2 , 5) is
(a) x−1 + y−1 = 2 (b) 2 x−1 + y−1 = 1 [20 April 2019, Shift-II]
(c) x−1 + 2 y−1 = 1 (d) 2 x−1 + 3 y−1 = 1 (a) x2 − 5 x + 6 = 0 (b) 2 x2 − 9 x + 9 = 0
Sol. (b) (c) 3 x2 − 8 x + 4 = 0 (d) 6 x2 − 13 x + 6 = 0
Let a line cuts the coordinate axes at A and B Sol. (c)
respectively is Equation of perpendicular bisector of line joining
x y
+ =1 …(i) points A(−2, − 1) and B(6, − 1) is
a b
[Q mid-point of AB is (2, − 1)]
So, A(a , 0) and B(0, b) and line (i) passes through
) so
P(4, 2, x=2 …(i)
4 2 and similarly equation of perpendicular bisector of
+ =1 …(ii)
a b line joining points B(6, − 1) and C(2, 5) is
Now, coordinate of centre of the circumcircle of [Q mid-point of BC is (4, 2)]
∆OAB is mid-point of hypotenuse of right angle 4
∆OAB and it is mid-point of AB. So, centre of the y − 2 = (x − 4)
6
circumcircle of ∆OAB is  ,  .
a b
 2 2 ⇒ 2x − 3y = 2 …(ii)
Now, from Eq. (ii), on taking locus of point Now, point of intersection of perpendicular
 a , b  , we get bisector is the circumcentre of ∆ABC, so
 
 2 2 On solving Eqs. (i) and (ii), we get C  2,  .
2
2 1  3
+ = 1 ⇒ 2x −1 + y −1 = 1
x y It is given that quadratic equation has roots are
Hence, option (b) is correct. 2
2 and , so equation of required quadratic
3
66. The centroid of the triangle formed by the lines
equation is x 2 −  2 +  x + 2  = 0
2 2
x + y − 1 = 0, x − y − 1 = 0, x − 3 y + 3 = 0 is  3  3
[20 April 2019, Shift-II]
−4 −8 ⇒ 3x 2 − 8 x + 4 = 0
(a)  , 1 (b)  , 1 (c)  , 3 (d)  , 3
4 8
3   3  3   3  Hence, option (c) is correct.
206 AP EAMCET Chapterwise Mathematics

68. The straight line x + y + 1 = 0 bisects an angle and the equation of diagonal of square not passes
through origin is 2x − 4 y + c = 0 …(iv)
between a pair of lines, of which one is
The diagonals given by the Eqs. (iii) and (iv) are
2 x − 3 y + 4 = 0 . Then the equation of the perpendicular if
other line in that pair is [20 April 2019, Shift-II]
1 h+ 6
(a) 2 x + 3 y + 4 = 0 (b) x − y + 1 = 0   = −1
2  3 − 2h 
(c) 5 x − 5 y + 9 = 0 (d) 3 x − 2 y + 5 = 0
Sol. (d) ⇒ h + 6 = 4h − 6 ⇒ 3h = 12 ⇒ h= 4
Point of intersection of lines x + y + 1 = 0 and  8−3 10  1 , − 2
So, point A =  ,−  =  
 9 + 16 9 + 16  5 5
2x − 3y + 4 = 0 is A − ,  .
7 2
 5 5
And the line (iv) passes through the mid-point
Now, Let a point P(−2, 0) on the line ‘M’of line joining points A , −  and O(0, 0).
1 2
2x − 3y + 4 = 0 and the image of point P(−2, 0)  5 5
w.r.t. line x + y + 1 = 0 lying on the other line in
So, M =  , −  .
1 1
the bisector pair. If the coordinate of image of  10 5
point P(−2, 0) w.r.t. line x + y + 1 = 0 is (h, k), then
h+ 2 k− 0 −2 + 1
= = − 2× =1 A (1/5, –2/5)
1 1 2
⇒ h = − 1 and k = 1 M
⇒ (h, k) = (−1, 1)
Now, equation of required bisector of angle is line
L1
joining the points A − ,  and (−1, 1) is
7 2 O L2
 5 5 (0, 0)
3/ 5 2 4
y −1 = (x + 1) ⇒ 2y − 2 = 3x + 3 ∴ + + c = 0⇒c = −1 [By Eq. (iv)]
2/ 5 10 5
⇒ 3x − 2y + 5 = 0 So, (h, c) = (4, − 1)
Hence, option (d) is correct. Hence, option (a) is correct.
69. If the pairs of straight lines represented by 70. The equation of the bisectors of the angles
3 x 2 + 2 hxy − 3 y 2 = 0 and between the lines joining the origin to the
3 x 2 + 2 hxy − 3 y 2 + 2 x − 4 y + c = 0 form a points of intersection of the curve
square, then (h, c) = [20 April 2019, Shift-II] x 2 + xy + y 2 + x + 3 y + 1 = 0 and the line
(a) (4, − 1) (b) (−1, 4) (c) (−4, 1) (d) (1, − 4) x + y + 2 = 0 is [20 April 2019, Shift-II]
(a) x2 + 4 xy − y2 = 0 (b) 2 x2 + 5 xy − y2 = 0
Sol. (a)
(c) x2 + 6 xy − 2 y2 = 0 (d) 2 x2 − 4 xy + 2 y2 = 0
Given, equations of pair of straight lines
3x 2 + 2hxy − 3y 2 = 0 …(i)
Sol. (a)
Homogenise the equation of the given equation of
and 3x 2 + 2hxy − 3y 2 + 2x − 4 y + c = 0 …(ii)
curve
both represents pair of perpendicular lines.
x 2 + xy + y 2 + x + 3y + 1 = 0 …(i)
Now, the point of intersection the lines given by
with the help of given line
ax 2 + 2hxy + by 2 + 2gx + 2 fy + c = 0 is
 hf − bg gh − af  x + y + 2= 0 …(ii)
 ,  , So for the pair of lines (ii), the To get the equation of the lines joining the origin
 ab − h2 ab − h2 
to points of intersection of curve (i) and line (ii).
point of intersection is 2
x + y  x + y
1 h − 3(−2) 
 h(−2) − (−3) ()  2h − 3 h+ 6  x 2 + y 2 + xy + (x + 3y)  +   =0
A ,  = A ,−   −2   −2 
 − 9 − h2 − 9 − h2   9 + h2 9 + h2 
1
The equation of diagonal of square passes through ⇒ x 2 + y 2 + xy − (x 2 + 4 xy + 3y 2)
2
h+ 6
origin is y= x …(iii) 1
+ (x 2 + 2xy + y 2) = 0
3 − 2h 4
Straight Line and Pair of Straight Lines 207

⇒ 3x 2 − 2xy − y 2 = 0 …(iii) ⇒
4k
= − 1 ⇒ 4k − 7h = 0 …(i)
Now, equation of the bisectors of the angle − 7h
between the pair of straight line (iii) is Slope of AB × slope of CF = − 1
x 2 − y2 xy Q M CF 
= ⇒ x 2 + 4 xy − y 2 = 0
3+ 1 −1  
⇒ M AB × M CF = − 1  3 − 0 = − 3
Hence, option (a) is correct. − 2 − 0 2 

71. If P(α , β) be a point on the line 3 x + y = 0 −3 2
⇒ M AB × = − 1 ⇒ M AB =
such that the point P and the point Q(1, 1) lie 2 3
on either side of the line 3 x = 4 y + 8 , then Equation of AB
2
[21 April 2019, Shift-I] y + 1 = (x − 5)
8 −8 8 −8 3
(a) α > ,β < (b) α < ,β <
15 5 15 5 ⇒ 3y + 3 = 2x − 10 ⇒ 2x − 3y = 13
8 −8 8 −8 A(h, k) lie on line AB
(c) α > ,β > (d) α < ,β >
15 5 15 5 2h − 3k = 13 …(ii)
Sol. (a) From Eqs. (i) and (ii), we get
Line L : 3x = 4 y + 8 = 3x − 4 y − 8 h = − 4, k = − 7
L(1 ,1 ) = 3 − 4 − 8 = − 9 < 0 Hence, third vertex of the triangle = (− 4, − 7)
Now, L( α , β) > 0 73. The distance from the origin to the
3x − 4 y − 8 > 0 orthocentre of the triangle formed by the
⇒ 3x − 4(− 3x) − 8 > 0 [Q y = − 3x] lines x + y − 1 = 0 and 6 x 2 − 13 xy + 5 y 2 = 0 is
8 8
15x − 8 > 0 ⇒ x > ⇒α> [21 April 2019, Shift-I]
15 15 11 2 11 2
3x − 4 y − 8 > 0 (a) (b) 13 (c) 11 (d)
2 24
⇒ − 5y − 8 > 0 [Q 3x = − y]
8 −8 Sol. (d)
⇒ 5y + 8 < 0 ⇒ y < − ⇒ β <
5 5 Given lines are x + y − 1 = 0
and 6 x 2 − 13xy + 5y 2 = 0
72. Two vertices of a triangle are (5, − 1) and
(− 2 , 3). If the origin is the orthocentre of this – + –
triangle, then the coordinates of the third –1 1
vertex of that triangle are [21 April 2019, Shift-I]
− 7 ⇒ 6 x − 10 xy − 3xy + 5y 2 = 0
2

(a) (4, 7) (b)  − 2, 


 2  ⇒ 2x(3x − 5y) − y(3x − 5y) = 0
(c) (− 4, − 7 ) (d) (− 2, 3) ⇒ (2x − y) (3x − 5y) = 0
Sol. (c) ⇒ 2x − y = 0 or 3x − 5y = 0
Slope of BC × slope of AD = − 1 Let orthocentre be (h, k).
Slope of OP × slope of AB = − 1
A (h, k)
k
× −1 = −1 ⇒ h = k …(i)
h
Now, slope of OB × slope of AD = − 1
F E
O  3 − k
  3 5
⇒ 2×  8  = − 1 ⇒ − 2h = h −
5 4 8
 − h
8 
B (5, –1) D C (–2, 3) 3 5 6+ 5
⇒ + = 3h ⇒ = 3h
4 8 8
3+ 1 k k 4
⇒ × = −1⇒ × = −1 11 11
− 2− 5 h h −7 ⇒ = 3h ⇒ h =
8 24
208 AP EAMCET Chapterwise Mathematics

11 Sol. (b)
∴ k=
24 Given, equation
2 2
Now, OP = h + k =  11  +  11  = 11
   
2 2
2 A (2, –7)
 24   24  24
74. The combined equation of two lines L and L1

=0
is 2 x 2 + axy + 3 y 2 = 0 and the combined

y–1
4x+
equation of two lines L and L 2 is
2 x 2 + bxy − 3 y 2 = 0 . If L1 and L 2 are
perpendicular, then a 2 + b2 =
[21 April 2019, Shift-I] B 3x–4y+1=0 C
(a) 26 (b) 29 (c) 13 (d) 85
4x + y − 1 = 0 …(i)
Sol. (a) and 3x − 4 y + 1 = 0 …(ii)
Let L ⇒ y = mx, L1 ⇒ y = kx, L2 Slope of line (i) (m1 ) = − 4
1 3
⇒ y=− x and slope of line (ii) m2 =
k 4
Now, (y − mx) (y − kx) = 0 Here, AB = AC
y 2 − ykx − mxy + mkx 2 = 0 ∴∆ABC is an isosceles.
⇒ mkx 2 − (k + m) xy + y 2 = 0 AB and AC both passes through points (2, −7).
Given that, 2x 2 + axy + 3y 2 = 0  3   19
−4 − −
2 2 a  4   4 19 19
or x + xy + y 2 = 0 Now, (m3) tanθ = = = − =
3 3 
1 + (−4) 3
  −2  −8 8
2 a   4   
On comparing, mk = , − (k + m) = … (i)
3 3 Now, required equation of line AC is
Now,  x
(y − mx)  y +  = 0
3 19

 k (y + 7) = 4 8 ( x − 2)
1 +    
3 19
xy mx 2
⇒ y2 + − mxy − =0  4  8 
k k
 24 − 76 
− x 2 +  − m xy + y 2 = 0
m 1  
⇒  32 
k k  ⇒ (y + 7) = ( x − 2)
 32 + 57 
which is 2x 2 + bxy − 3y 2 = 0  
 32 
2 b
or − x 2 − xy + y 2 = 0 52
3 3 ⇒ y + 7 = − ( x − 2)
−m −2 b 1 89
On comparing, = ,− = −m
k 3 3 k ⇒ 89 y + 623 = − 52x + 104
2k b 1 − km ⇒ 52x + 89 y + 519 = 0
So, m= ,− = …(ii)
3 3 k 76. In a ∆ABC, 2 x + 3 y + 1 = 0, x + 2 y − 2 = 0 are
By solving Eqs. (i) and (ii), we get
2
the perpendicular bisectors of its sides AB
m = , k = 1 and a = − 5, b = − 1 and AC respectively and if A = (3 , 2), then the
3
equation of the side BC is [21 April 2019, Shift-II]
∴ a 2 + b 2 = 25 + 1 = 26
(a) x + y − 3 = 0 (b) x − y − 3 = 0
75. A straight line 4 x + y − 1 = 0 through the (c) 2 x − y − 2 = 0 (d) 2 x + y − 2 = 0
point A(2 , − 7) meets the line BC whose Sol. (b)
equation is 3 x − 4 y + 1 = 0 at the point B. Given that, equation of AB is 3x − 2y + c = 0
Then the equation of the line AC such that passes through A(3, 2) ⇒ C = − 5
AB = AC , is [21 April 2019, Shift-II]
– + –
(a) 89 x − 52 y − 162 = 0 (b) 52 x + 89 y + 519 = 0
(c) 4 x − y − 15 = 0 (d) 4 x + 3 y + 13 = 0 –1 1
Straight Line and Pair of Straight Lines 209

∴ Equation becomes 3x − 2y − 5 = 0 α − 2  2 − α 
m = − (gradient of AB) = −  =  
Here, D = (1, − 1)  4   4 
Since, D is mid-point of AB. Put value of m in Eq. (ii), we get
Let coordinates of B are (α , β). 2 − α  2 − α = 0
α   + 2 
3+ α 2+ β  4   4 
∴ = 1 and = −1
2 2 2α − α 2 4 − 2α
⇒ 3 + α = 2 and 2 + β = − 2 + =0
4 4
⇒ α = −1 and β = − 4
⇒ − α2 + 4 = 0
∴ B is (−1, − 4).
⇒ α2 = 4 ⇒ α = ± 2
Similarly, equation of AC is 2x − y + c = 0 is passes
through (3, 2) ⇒ C = − 4 ⇒ 2x − y − 4 = 0 78. The number of values of a for which the pair
Here, E is a point of intersection of x + 2y − 2 = 0 of lines represented by
and 2x − y − 4 = 0. 3 ax 2 + 5 xy + (a 2 − 2) y 2 = 0 are at right angles
∴ E = (2, 0) to each other, is [21 April 2019, Shift-II]
Since, E is mid-point of AC. (a) 2 (b) 1
Let coordinates of C are (α1 , β1 ) (c) infinitely many (d) 0
3 + α1 2 + β1
∴ = 2 and =0 Sol. (a)
2 2 The pair of lines represented by the given
⇒ α1 = 4 − 3 and β1 = − 2 equation will be perpendicular to each other, if
⇒ α1 = 1 and β1 = − 2 Coefficient of x 2 + Coefficient of y 2 = 0
∴ C is (1, − 2). ⇒ 3a + a 2 − 2 = 0
∴Required equation of BC is −3 ±
9 + 8 −3 ± 17
−2 + 4 2 ⇒ a= =
(y + 2) = (x − 1) ⇒ y + 2 = (x − 1) 2 2
1+1 2 Thus, there are two values.
⇒ y + 2 = (x − 1) ⇒ x − y − 3 = 0 79. The set of values that β can assume so that
77. If the perpendicular bisector of the line the point (0 , β) should lie on or inside the
segment joining A(α , 3) and B(2 , − 1) has triangle having sides 3 x + y + 2 = 0 ,
y-intercept 1, then α = [21 April 2019, Shift-II] 2 x − 3 y + 5 = 0 and x + 4 y − 14 = 0, is
(a) 0 (b) ± 1 (c) ± 2 (d) ± 3 [22 April 2019, Shift-I]
(a)  ,  (b)  ,  (c)  − ,  (d)  , 
5 7 2 5 1 2 1 5
Sol. (c)
 3 2   3 2   3 3   2 2 
Let the equation of perpendicular bisector is
y = mx + c Sol. (a)
Here, c = 1 According to given information, the diagram will
∴ y = mx + 1 …(i) be as following
Mid-point of points A(α , 3) and B(2, − 1) is Y
 α + 2 , 1 .
 
 2 
α+ 2 
Since, Eq. (i) passes through 
7
, 1 . 0, 2
 2  5
0, 3
α + 2
So, 1 = m  +1
 2  –5 ,0
m(α + 2) 2 (0, 0)
⇒ 1= +1 X′ X
2 (0, –2) (14, 0)
⇒ 2 = m(α + 2) + 2 2x–3y+5=0 3x+y+2=0 x+4y–14=0
⇒ m(α + 2) = 0 ⇒ mα + 2m = 0 …(ii) Y′
m = gradient of the perpendicular line Let L1 ≡ 3x + y + 2 = 0
210 AP EAMCET Chapterwise Mathematics

x y
⇒ + =1 81. If the mid-points of the sides BC , CA and AB
2 −2
− of a triangle ABC, are respectively (2, 1),
3
(−1, − 2) and (3, 3), then the equation of the
L2 ≡ 2x − 3y + 5 = 0 side BC is [22 April 2019, Shift-I]
x y
⇒ + =1 and L3 ≡ x + 4 y − 14 = 0 (a) x − 2 y = 0 (b) 5 x − 4 y = 6
5 5
− (c) 2 x + 3 y = 8 (d) 3 x − 2 y = 6
2 3
x y
Sol. (b)
⇒ + =1
14 7 / 2 A

It is clear from the diagram that set of values of


β ∈  , .
5 7
 3 2 R(3,3) Q (–1,–2)
80. If (λ , λ + 1), λ ∈ Z belongs to the region
2

between the lines x + 2 y − 5 = 0 and


3 x − y + 1 = 0 which includes the origin, then
B C
the possible number of such points is P(2, 1)
[22 April 2019, Shift-I] Let the coordinate of B is (a , b).
(a) 4 (b) 3 (c) 2 (d) Infinite Mid-point of BQ = Mid-point of PR
Sol. (c) ⇒  a −1 , b − 2 =  3 + 2 , 3 + 1 
   
Given equation of lines are  2 2   2 2 
x + 2y − 5 = 0 a −1 5 b −2 4
…(i) ⇒ = and =
and 3x − y + 1 = 0 …(ii) 2 2 2 2
from Eq . (i), origin O and P are on the same side. ⇒ a = 5 + 1 and b = 4 + 2
⇒ a = 6 and b = 6
Y
∴ B(6, 6) and P(2, 1)
Now, equation of BC is
1 −6 −5
y−6= (x − 6) ⇒ y − 6 = (x − 6)
2− 6 −4
⇒ 4 y − 24 = 5x − 30
–1 , 0 ⇒ 5x − 4 y − 30 + 24 = 0 ⇒ 5x − 4 y = 6
3 O(0,0)
X′ X
(5,0) 82. The distance between the pair of lines
3x– y+1=0 x + 2y–5=0 x 2 + 2 2 xy + 2 y 2 + 4 x + 4 2 y +1 = 0 , is
Y′
[22 April 2019, Shift-I]
⇒ (−5)(λ 2 + 2λ − 3) > 0
(a) 4 2 (b) 2 2 (c) 2 (d) 6 2
⇒ λ 2 + 2λ − 3 < 0
⇒ λ + 3λ − λ − 3 < 0
2 Sol. (c)
⇒ λ(λ + 3) − 1(λ + 3) < 0 Given, equation of pair of lines is
⇒ (λ + 3)(λ −1) < 0] x 2 + 2 2xy + 2y 2 + 4 x + 4 2y + 1 = 0
⇒ λ ∈ (−3, 1) The distance between the pair of lines
For Eq. (ii), ax 2 + 2hxy + by 2 + 2gx + 2 fy + c = 0 is given by
(λ) λ −  > 0
1
 g2 − ac
3 d=2
a(a + b)
λ ∈ (− ∞ , 0) ∪  , ∞ 
1

3  Here, a = 1, b = 2, c =1, g = 2
∴ λ = {−2, − 1} 4 −1 3
∴ Required distance = 2 =2 =2
Hence, there are 2 points. 1 (1 + 2) 3
Straight Line and Pair of Straight Lines 211

83. A pair of lines S = 0 together with the lines 85. The vertices of a triangle OBC are O(0 , 0),
given by the equation B(− 3 , − 1) and C(− 1, − 3). If the line joining
8 x 2 −14 xy + 3 y 2 + 10 x + 10 y − 25 = 0 form a the point D on OC and E on OB is parallel to
BC and the perpendicular distance of O from
parallelogram. If its diagonals intersect at the
1
point (3, 2), then the equation S =0, is DE is , then the equation of DE is
[22 April 2019, Shift-I] 2
[22 April 2019, Shift-II]
(a) 6 x2 − 9 xy + y2 − 25 x + 30 y + 25 = 0
(b) 8 x2 − 14 xy + 3 y2 − 25 x + 30 y + 50 = 0 (a) x + y + 2 = 0 (b) 2 x + 2 y − 2 = 0
(c) 2 x + 2 y + 2 = 0 (d) 2 x − 2 y + 2 = 0
(c) 8 x2 − 14 xy + 3 y2 − 50 x + 50 y + 75 = 0
(d) 6 x2 + 14 xy − 3 y2 − 30 x + 40 y − 75 = 0 Sol. (b)
Sol. (c)
Equation of given pair of straight lines is
O (0, 0)
8 x 2 − 14 xy + 3y 2 + 10 x + 10 y − 25 = 0
E
⇒ (4 x − y − 5)(2x − 3y + 5) = 0
B(–3, –1) D
Now point of intersection of lines
4x − y − 5 = 0
C(–1, –3)
2x − 3y + 5 = 0 is (2, 3)
So equation of S = 0 is Equation of line BC
(4 x − y + c1 )(2x − 3y + c 2) = 0 − 3+ 1
y+1= (x + 3)
and S = 0 passes through a point P(x1 , y1 ) such −1 + 3
that mid-point of P(x1 , y1 ) and (2, 3) is (3, 2).
⇒ y+1= − x − 3 ⇒ y= − x − 4
so, x1 = 4 and y1 = 1 y=− x+ c …(i)
∴ c1 = − 15 and c 2 = − 5 y+ x−c=0
So, required equation is −c 1 c 1 1
⇒ = ⇒ = ⇒ c=
(4 x − y − 15)(2x − 3y − 5) = 0 2 2 2 2 2
⇒ 8 x 2 − 14 xy + 3y 2 − 50 x + 50 y + 75 = 0 1
Now, y = − x + [Since, DE is parallel to BC]
2
84. The normal form of the line x + y + 1 = 0 is ⇒ 2y = − 2x + 1 ⇒ 2y + 2x − 1 = 0
[22 April 2019, Shift-II]
⇒ 2x + 2y − 1 = 0 ⇒ 2x + 2y − 2=0
1
(a) x cos (45° ) + y sin (135° ) =
2 86. A variable line passing through the fixed
(b) x cos (45° ) + ysin (45° ) =
1 point (α , β) intersects the co-ordinate axes at
2 A and B. If O is the origin, then the locus of
1 the centroid of the triangle OAB is
(c) xcos(225° ) + ysin(225° ) =
2 [22 April 2019, Shift-II]
1 (a) βx + αy = 3 xy (b) αx + βy = 3 xy
(d) xcos(45° ) + ysin(45° ) = −
2 (c) αx − βy = 3 xy (d) βx − αy = 3 xy
Sol. (c) Sol. (a)
Given, x + y+1= 0 Let equation of line is
x y
+ =1
−x − y 1 a b
⇒ =
2 2
B (0, b)
x +  −
1 1  1
⇒ −  y=
2  2 2 (α, β)
1
⇒ cos 225° x + sin 225° y =
2
(Q cosθ and sinθ negative in 3rd quadrant) O(0, 0) A(a, 0)
212 AP EAMCET Chapterwise Mathematics

Since, it passes through (α , β) 88. The distance between the pair of lines
α β
∴ + =1 …(i) represented by
a b x 2 + 2 2 xy + 2 y 2 + 4 x + 4 2 y + 1 = 0 , is
0 + a + 0 0 + b + 0
Q Centroid  ,  = (h, k) [22 April 2019, Shift-II]
 3 3 
(a) 1 (b) 2 (c) 2 (d) 4
⇒  a , b  = (h, k)
  Sol. (b)
 3 3
Given, x 2 + 2 2xy + 2y 2 + 4 x + 4 2y + 1 = 0
Now, bα + aβ = ab
bα aβ ab 3 We know that,
+ = ×
3 3 3 3 The distance ‘d’ between the pair of straight lines
represented by
⇒ kα + hβ = 3hk
ax 2 + 2hxy + by 2 + 2gx + 2 fy + c = 0 is given by
Now, locus of centroid is
g2 − ac
⇒ αy + βx = 3xy ⇒ βx + αy = 3xy d=2
a(a + b)
87. If A is the orthocentre of the triangle formed
Here, a = 1, b = 2, c = 1, g = 2
by 2 x 2 − y 2 = 0 , x + y − 1 = 0 and B is the
4 −1 3
centroid of the triangle formed by ∴ d=2 =2 =2
( + 2)
11 3
2 x 2 − 5 xy + 2 y 2 = 0 , 7 x − 2 y − 12 = 0 , then the
distance between A and B is 89. The intercept form of the equation of the
[22 April 2019, Shift-II] straight line passing through the point (4 , − 3)
(a) 5 (b) 1 (c) 5 (d) 2 and perpendicular to the line passing
through the points (1, 1) and (2 , 3) is
Sol. (a) [23 April 2019, Shift-I]
Given, x−4 y+ 3 x y
2x 2 − y 2 = 0 ⇒ y 2 = 2x 2
(a) = (b) + =1
1 1 −2 −1
⇒ y = ± 2x and x + y = 1 x y
(c) − = 1
x y
(d) + = 1
8 6 1 1
(4, 8)
Sol. (b)
Let A = (4, − 3)
B = (1, 1)
7x–2y–12=0
y=2x C = (2, 3)
3−1
Slope of BC = =2
2−1
1
∴Slope of required line = −
(0, 0) x=2y (2, 1) 2
[Q Lines are perpendicular]
Q Point of intersection of perpendicular is ∴Equation of line passing through A(4, − 3) and
orthocentre. −1
having slope is
The two perpendiculars are y = x and 2
x − 2y + 2 − 1 = 0 −1
y − y1 = m(x − x1 ) ⇒ y + 3 = (x − 4)
∴Point of intersection of y = x and 2
x − 2y + 2 − 1 = 0 ⇒ 2y + 6 = − x + 4 ⇒ x + 2y + 2 = 0
∴ A = (1, 1) ⇒ x + 2y = − 2
x y
Now, 2x 2 − 5xy + 2y 2 = 0 + = 1 [Q divided by − 2 on both sides]
−2 −1
⇒ (2x − y) (y − 2x) = 0
∴ Hence, option (b) is correct.
⇒ y = 2x or x = 2y and 7 x − 2y − 12 = 0
Straight Line and Pair of Straight Lines 213

7x − y + 3
90. If x + 2 y − 3 = 0, 3 x + 4 y − 7 = 0, 2 x + 3 y − 4 or = − (x + y − 3)
5
= 0 and 4 x + 5 y − 6 = 0 are the equations of
7 x − y + 3 = 5x + 5y − 15
four lines, then [23 April 2019, Shift-I]
or 7 x − y + 3 = − 5x − 5y + 15
(a) they are the sides of a square ⇒ 2x − 6 y + 12 = 0
(b) they are all concurrent lines
or 12x + 4 y − 12 = 0
(c) they are the sides of a parallelogram
⇒ x − 3y + 6 = 0 or 3x + y − 3 = 0
(d) not all of them are concurrent
1
Sol. (d) Slope = , Slope = − 3 (integer)
3
Given lines are Here, third side is parallel to one angle bisector.
x + 2y − 3 = 0 …(i) ∴Required slope of third side is − 3.
3x + 4 y − 7 = 0 …(ii)
92. The pair of lines lx 2 + 2(l + m) xy + my 2 = 0
2x + 3y − 4 = 0 …(iii)
4 x + 5y − 6 = 0 …(iv)
lies along two diameters of a circle and
On solving Eqs. (i) and (ii), we get point of divides the circle into 4 sectors. If the area of
intersection bigger sector is 5 times the area of smaller
lm
i.e. P = (1, 1) sector, then =
Here, P(1, 1) is not satisfy Eqs. (iii) and (iv)
(l + m)2 [23 April 2019, Shift-I]
∴All the given lines are not concurrent. 1 2 11 13
(a) (b) (c) (d)
∴Hence, answer is (d). 2 3 12 12

91. Two equal sides of an isosceles triangle are Sol. (c)


given by 7 x − y + 3 = 0 and x + y − 3 = 0. If Let angle between given pair of lines is ‘θ’.
the slope m of the third side is an integer, According to given information,
then m = [23 April 2019, Shift-I]
A2 = 5A1 ⇒ (π − θ) r 2 =  θr 2  × 5
1 1
(a) 3 (b) 1 2 2 
(c) − 1 (d) − 3
Sol. (d) A2
Given equation of sides are r
A1
θ
A
π–θ

π
⇒ π − θ = 5θ ⇒ θ =
6
π l+m
cos =
6 (l − m)2 + 4 (l + m)2
B C
3 l+m
7x − y + 3 = 0 …(i) ⇒ =
2 5l + 5m2 + 6lm
2
x + y − 3= 0 …(ii)
3 (l + m)2
Equation of angle bisectors of Eqs. (i) and (ii) are ⇒ = 2
7x − y + 3 x+ y−3 4 5l + 5m2 + 6lm

49 + 1 1+1 ⇒ 15 l 2 + 15 m2 + 18 lm = 4l 2 + 4m2 + 8 lm
7x − y + 3 x+ y−3 ⇒ 11 l 2 + 11m2 + 10 lm = 0

50 2 ⇒ 11 (l 2 + m2 + 2lm) − 22 lm + 10 lm = 0
7x − y + 3 x+ y−3 ⇒ 11 (l + m)2 − 12 lm = 0
=± lm 11
5 1 ⇒ =
7x − y + 3 (l + m)2 12
∴ = (x + y − 3)
5 Hence, option (c) is correct.
214 AP EAMCET Chapterwise Mathematics

93. The line 3 x + 4 y − 5 = 0 cuts the curve 3× 0 + 2× a 3× b + 2× 0  2a 3b 


 ,  ⇒  , 
 3+ 2 3+ 2   5 5
2 x + 3 y = 5 at A and B. If ‘O’ is the origin,
2 2

then ∠AOB = [23 April 2019, Shift-I] If is given that, the point (2, − 1) divides the AB in
π π the ratio 3 : 2.
(a) (b)
6 3 2a 3b −5
π π ∴ = 2 and = − 1 ⇒ a = 5 and b =
(c) (d) 5 5 3
2 8 Hence, the required equation of the required line is
Sol. (c) x y x 3y
+ =1 ⇒ − =1
Given equation of line is 3x + 4 y − 5 = 0 5 −5/ 3 5 5
3x + 4 y
=1 …(i) ⇒ x − 3y = 5 ⇒ x − 3y − 5 = 0
5
Equation of curve is 2x + 3y 2 = 5
2
…(ii) 95. The equation of the line passing through the
point of intersection of the lines
2 x + y − 4 = 0 , x − 3 y + 5 = 0 and lying at a
A distance of 5 units from the origin, is
[22 April 2018, Shift-I]
B
(a) x − 2 y − 5 = 0 (b) x + 2 y − 5 = 0
(c) x + 2 y + 5 = 0 (d) x − 2 y + 5 = 0
O Sol. (b)
The equation of a line passing through the
Homogenising Eq. (ii) using Eq. (i) intersection of 2x + y − 4 = 0 and x − 3y + 5 = 0 is
2x 2 + 3y 2 = 51
()2 (2x + y − 4) + λ(x − 3y + 5) = 0 …(i)
3x + 4 y 
2 ⇒ x(2 + λ) + y(1 − 3λ) + 5λ − 4 = 0
⇒ 2x 2 + 3y 2 = 5  
 5  This is at a distance of 5 units from the origin.
 9 x 2 + 16 y 2 + 24 xy  5λ − 4
⇒ 2x 2 + 3y 2 = 5   ∴ = 5
 25  (2 + λ)2 + (1 − 3λ)2
⇒ 10 x 2 + 15y 2 = 9 x 2 + 16 y 2 + 24 xy (5λ − 4)2
⇒ =5
⇒ x − y 2 + 24 xy = 0
2
4 + λ + 4λ + 1 + 9λ2 − 6λ
2

Here, (coefficient of x 2 ) + (coefficient of y 2 ) (5λ − 4)2


= 1 + (− 1) = 0 ⇒ =5
10λ2 − 2λ + 5
π
∴ ∠AOB! = 90° (or)
2 ⇒ 25λ2 + 16 − 40λ = 50λ2 − 10λ + 25
∴Hence, answer is (c). ⇒ 25λ2 + 30λ + 9 = 0
94. If the portion of a line intercepted between 3
By solving, we get λ = −
the coordinates axes is divided by the point 5
(2, −1) in the ratio of 3:2, then the equation 3
Putting the value of λ = − in Eq. (i), we get
of that line is [22 April 2018, Shift-I] 5
(a) 5 x − 2 y − 20 = 0 (b) 2 x − y − 5 = 0 3
⇒ (2x + y − 4) − (x − 3y + 5) = 0
(c) 3 x − y − 7 = 0 (d) x − 3 y − 5 = 0 5
⇒ 10 x + 5y − 20 − 3x + 9 y − 15 = 0
Sol. (d) ⇒ 7 x + 14 y − 35 = 0 ⇒ x + 2y − 5 = 0
Let the equation of the line be
x y 96. The equation of the line joining the centroid
+ =1 with the orthocentre of the triangle formed
a b
The line meets the coordinate axes at A(a , 0) and by the points (−2 ,3) , (2 ,−1), (4 ,0) is
[22 April 2018, Shift-I]
B(0, b) respectively. The coordinate of the point
which divides the line joining A(a , 0) and B(0, b) in (a) x + y − 2 = 0 (b) 11x − y − 14 = 0
(c) x − 11y + 6 = 0 (d) 2 x − y − 2 = 0
the ratio 3 : 2 are
Straight Line and Pair of Straight Lines 215

Sol. (b) 23
Here, M1 + M 2 = 16 and M1 M 2 =
Let AD and BE are altitudes of the triangle. 3
∴Equation of AD is given by ∴ m1 − m2 = (m1 + m2)2 − 4(m1 m2)
−1 23 92 768 − 92 26
y − 3= (x + 2) = (16)2 − 4 × = 256 − = =
Slope of BC 3 3 3 3
−1 ∴ m1 = 8 +
13
and m2 = 8 −
13
⇒ y − 3= (x + 2)
 0 + 1 3 3
 
 4 − 2 26
m1 − m2 3
y − 3 = − 2(x + 2) tanθ = = = 3
1 + m1 m2 1+
23
y − 3 = − 2x − 4 3
⇒ 2x + y + 1 = 0 … (i) We know that, than 60°
∴Equation of BE is given by So, θ = 60°
2
−1 Slope of line 2x + 3y + 4 = 0 is −
y+1= ( x − 2) 3
Slope of AC 13 2
Angle between line of slope 8 + and − is
−1 3 3
⇒ y+1= ( x − 2)
 0 − 3 13 2
  8+ +
 4 + 2 tanα = 3 3 = 3
1 +  8 +
13   2
y + 1 = 2(x − 2) ⇒ y + 1 = 2x − 4   
 3   3
⇒ y = 2x − 5 … (ii)
∴ θ = 60° ⇒ α = 60°
Since, orthocentre is the intersecting point of
altitudes. Hence, line from an equilateral triangle.

∴On solving Eqs. (i) and (ii), we get orthocentre 98. If the line x + 2 y = k intersects the curve
(1, − 3). x 2 − xy + y 2 + 3 x + 3 y − 2 = 0 at two points A
− 2 + 2 + 4 3 − 1 + 0
Also, centroid of ∆ABC =  ,  and B and if O is the origin, then the
 3 3  condition for ∠AOB = 90 ° is
=  , 
4 2 [22 April 2018, Shift-I]
 3 3
(a) k 2 + k + 1 = 0 (b) k 2 − 2 k + 10 = 0
(c) 2 k 2 + 9k − 10 = 0 (d) 3k 2 + 8k − 1 = 0
∴ Equation of line joining (0, − 1) and  ,  is
4 2
 3 3
2 Sol. (c)
+ 3 We have,
y + 3= 3 (x − 1)
4 x 2 − xy + y 2 + 3x + 3y − 2 = 0 …(i)
−1
3 and
x + 2y
⇒ y + 3 = 11(x − 1) x + 2y = k ⇒ =1
k
⇒ y + 3 = 11 x − 11
By homogeneous of Eq. (i), we get
⇒ 11 x − y − 14 = 0.
x + 2y 
x 2 − xy + y 2 + 3x  
97. The lines represented by the equations  k 
23 x 2 − 48 xy + 3 y 2 = 0 and 2 x + 3 y + 4 = 0 form x + 2y 
2

+ 3y   x + 2y  = 0
 − 2 
[22 April 2018, Shift-I]  k   k 
(a) an isosceles triangle (b) a right angled triangle ⇒ k2 x 2 − k2 xy + k2 y 2 + 3kx 2 + 6kxy + 3kxy + 6ky 2
(c) an equilateral triangle (d) a scalene triangle − 2x 2 − 8 xy − 8 y 2 = 0
Sol. (c) ⇒ x 2 (k2 + 3k − 2) − (k2 − 9k + 8)
We have, xy + (k2 + 6k − 8) y 2 = 0
23x 2 − 48 xy + 3y 2 = 0 Since, ∠AOB = 90°
⇒ 3y 2 − 48 xy + 23x 2 = 0 ∴k2 + 3k − 2 + k2 + 6k − 8 = 0 ⇒ 2k2 + 9k − 10 = 0
216 AP EAMCET Chapterwise Mathematics

99. If 2 x 2 + 3 xy − 2 y 2 = 0 represents two sides of a Sol. (c)


parallelogram and 3 x + y + 1 = 0 is one of its Point of intersection of given line L = 0 and line
segment PQ is
diagonals, then the other diagonal is
(4 × 1) + (1 × 2) (4 × ())
1 + (1 × (−1)) 
[22 April 2018, Shift-I] R ,  6 3
=R , 
 5 5   5 5
(a) x − 3 y + 1 = 0 (b) x − 3 y + 2 = 0
(c) x − 3 y = 0 (d) 3 x − y = 0 Now, point R on the Line L = 0,
18 12
Sol. (c) So, + −k=0 ⇒ k=6
5 5
As we know that,
Now, equation of Line concurrent with the lines
If the lines ax 2 + 2hxy + by 2 = 0 be two sides of a L = 0 and PQ is
parallelogram and the line lx + my = 1 be one of its
diagonals, then other diagonal is (3x + 4 y − 6) + λ(2x + y − 3) = 0 …(i)
y(bl − hm) = x(am − hl) .....(i) ∴ Line Eq. (i) is parallel to the Line y = x , so
3 + 2λ 7
Here, a = 2, b = − 2, h =
3 − =1 ⇒ λ = − .
2 4+ λ 3
l = − 3, m = − 1 ∴ Equation of required line is
Putting all values in Eq. (i), we get 5x − 5y − 3 = 0

∴ y  6 +  = x  − 2 +  ⇒ y   = x  
3 9 15 5 102. The orthocentre and the centroid of ∆ABC are
 2  2  2  2  14 
15y = 5x ⇒ 3y = x ⇒ x − 3y = 0 (5, 8) and 3 ,  respectively. The equation
 3
100. A straight line L with negative slope passes of the side BC is x − y = 0. Given that the
through the point (1, 1) and cuts the positive image of the orthocentre of a triangle with
coordinate axes at the points A and B. If O is respect to any side lies on the circumcircle of
the origin, then the minimum value of that triangle, then the diameter of the
OA + OB as L varies, is [22 April 2018, Shift-II] circumcircle of ∆ABC is [22 April 2018, Shift-II]
(a) 1 (b) 2 (a) 10 (b) 2 10 (c) 4 10 (d) 8 10
(c) 3 (d) 4
Sol. (c)
Sol. (d) The centroid on a non-equilateral triangle divides
Equation of line having slope ‘m’ passes through the line joining orthocentre and circumcentre in
the point (1, 1) is 2 : 1, so let the coordinates of circumcentre is (h, k),
y − 1 = m(x − 1) …(i) then
m −1 
So, A , 0 and B(0, 1 − m)  2h + 5 , 2k + 8  =  3, 14 
 m     
 3 3   3 
Now, OA + OB = 1 −  + (1 − m) = 2 −  m + 
1 1
 ⇒ h = 2, k = 3
m  m
And image of orthocentre (5, 8) with respect to side
Q m is negative, so minimum value of
BC, x − y = 0 is (8, 5)
−  m +  = 2
1
 m So, the radius of circumcircle of ∆ABC is
So, minimum value of OA + OB = 2 + 2 = 4 (8 − 2)2 + (5 − 3)2 = 40

101. If the straight line L ≡ 3 x + 4 y − k = 0 cuts Then diameter = 4 10.

the line segment joining the points P (2 , − 1) 103. If a pair of perpendicular lines through the
and Q(1, 1) in the ratio 4 : 1, then the origin together with the straight line
equation of the line parallel to the line y = x 2 x + 3 y = 6 form an isosceles triangle, then
and concurrent with the lines PQ and L = 0 is the area of that triangle (in sq units) is
[22 April 2018, Shift-II] [22 April 2018, Shift-II]
(a) 2 x − 2 y + 7 = 0 (b) x − y + 1 = 0 6 6 36 27
(a) (b) (c) (d)
(c) 5 x − 5 y − 3 = 0 (d) y = x + 3 13 13 13 13
Straight Line and Pair of Straight Lines 217

2 2
Sol. (c)  14 f − 8 g  +  14 g − 12 f  = 2
   
Since, ∆AOB is a right angled triangle, so  25   25  5
π ⇒ 196 f 2 + 64 g2 − 224 fg + 196 g2 + 144 f 2
∠A =∠B =
4 − 336 gf = 250
O (0, 0) ⇒ 260 g2 + 340 f 2 − 560 gf = 250
⇒ 26 g2 + 34 f 2 − 56 gf = 25 …(v)
From Eqs. (ii) and (v)
26 g2 + 34 f 2 − 56 gf = 25
16 g2 + 24 f 2 − 56 gf = − 100
A B
P − − + +
10 g2 + 10 f 2 = 125
2x+3y=6 25
g2 + f 2 =
So, AB = 2AP = 2OP 2
6 6 105. Let a , b and c be distinct and none of them is
Now, OP = =
4+ 9 13 equal to 1. If the lines x + ay + a = 0,
1 36 bx + y + b = 0 and cx + cy + 1 = 0 are
So, Area of ∆AOB = × AB × OP = OP 2 = concurrent, then the value of
2 13
a b c
104. If the equation + + is
a −1 b −1 c −1 [23 April 2018, Shift-I]
3 x 2 + 7 xy + 2 y 2 + 2 gx + 2 fy + 2 = 0 represents
a pair of intersecting lines and the square of (a) 1 (b) − 1 (c) 2 (d) 0
the distance of their point of intersection Sol. (a)
2
from the origin is , then f 2 + g 2 = Given equations are
5 x + ay + a = 0
[22 April 2018, Shift-II]
bx + y + b = 0 and cx + cy + 1 = 0
25 25
(a) (b) 25 (c) 50 (d) The above three lines are concurrent, so
4 2 1 a a
Sol. (d) b 1 b =0
The equation, c c 1
3x 2 + 7 xy + 2y 2 + 2gx + 2 fy + 2 = 0 …(i)
⇒11( − bc) − a(b − bc) + a(bc − c) = 0
represents a pair of intersecting lines, so ⇒1 − bc − ab + abc + abc − ac = 0
2
⇒1 + 2abc = ab + bc + ac
(3) (2) (2) + 7(g) ( f) − 3 f 2 − 2g2 − 2   = 0
7
 2 a b c
Now, + +
⇒ 24 + 14 gf − 6 f 2 − 4 g2 − 49 = 0 a −1 b −1 c −1
⇒ 4 g2 + 6 f 2 − 14 gf + 25 = 0 …(ii) a(b − 1) (c − 1) + b(a − 1) (c − 1) + c(a − 1) (b − 1)
=
To get the point of intersection of pair of straight (a − 1) (b − 1) (c − 1)
lines, we have to differentiate partially the Eq. (i) abc − ab − ac + a + abc − ab − bc + b + abc
with respect to x and y respectively. − ac − bc + c
=
then we are getting abc + a + b + c − ab − bc − ac − 1
6 x + 7 y + 2g = 0 …(iii) 3abc − 2(ab + bc + ac) + a + b + c
=
and 7x + 4y + 2f = 0 …(iv) abc + a + b + c − (ab + bc + ac) − 1
Now, point of intersection of eqs. (iii) and (iv) is 3abc − 21
( + 2abc) + a + b + c
=
the point of intersection of pair of straight line abc + a + b + c − (1 + 2abc) − 1
Eq. (i). 3abc − 2 − 4abc + a + b + c
=
 14 f − 8 g , 14 g − 12 f  abc + a + b + c − 1 − 2abc − 1
 
 25 25  − abc + a + b + c − 2
= =1
Now, according to the question, − abc + a + b + c − 2
218 AP EAMCET Chapterwise Mathematics

A (–2, 3)
106. If ad − bc ≠ 0, then the area (in sq. units) of
the parallelogram formed by the lines
ax + by + 2 = 0, ax + by + 5 = 0, cx + dy + 3 = 0 F
E
and cx + dy + 7 = 0 is [23 April 2018, Shift-I]
1 5
(a) (b)
| ad − bc | | ad − bc | B (1, –2) D C (2, 1)
7 12
(c) (d) ∴Circumcentre of triangle is a point where
| ad − bc | | ad − bc | perpendicular bisector of sides meet.
Sol. (d) So, DE is perpendicular bisector of BC.
We have, ax + by + 2 = 0 and ax + by + 5 = 0 are D is mid-point of BC, so coordinate of
1 + 2 − 2 + 1  3 − 1
parallel lines. ∴ D  , =D , 
ax + by + 2 = 0  2 2  2 2 
−a 2 1+ 2
⇒ y= x− Slope of BC = =3
b b 2−1
and ax + by + 5 = 0
− a −5 1
⇒ y= x Since DE ⊥ BC, slope of DE = −
b b 3
−a −2 5 Now, equation of DE is
So, m1 = , c1 = , c2 = −
b b b  y + 1  = − 1  x − 3
   
Now, again  2 3  2
cx + dy + 3 = 0 1 1 1
−c 3 ⇒ y+ =− x+
⇒ y= x− 2 3 2
d d ⇒ 3y = − x
and cx + dy + 7 = 0
⇒ x + 3y = 0 … (i)
−c 7
⇒ y= x− Now, FE is perpendicular bisector of AB
d d
− 1 1
−c −3 −7 ∴Coordinate of F  , 
So, m2 = , d1 = , d2 =  2 2
d d d
3 − (− 2) − 5
(c − c 2) (d1 − d2) Slope of AB = =
Then, area of parallelogram = 1 − 2 − ()
1 3
m1 − m2
3
So, slope of EF =
 − 2 + 5  − 3 + 7  5
   
 b b  d d 1 3
= ∴Equation of FE, y − = (x + 1 / 2)
a c 2 5
− +
b d ⇒ 3x − 5y = − 4 … (ii)
3 4 DE and EF intersect at circumcentre, so
×
b d − 12 12 Multiplying by 3 in Eq. (i) and subtract then, we get
= = = .
− (ad − bc ) ad − bc | ad − bc | 3x + 9 y = 0
bd 3x − 5y = − 4
− + +
107. The circumcentre of the triangle with vertices
at (− 2 , 3), (1, − 2) and (2, 1) is 14 y = 4
2
[23 April 2018, Shift-I] y=
7
(a)  ,  (b)  − , 
6 2 6 2
7 7  7 7 Put in Eq. (i), we get
2
x = − 3×
(c)  , −  (d)  − , − 
6 2 6 2
7
7 7  7 7 −6
⇒ x=
Sol. (b) 7
− 6 2
Let vertices of ∆ABC is A(− 2, 3), B(1, − 2) and C (2, 1) So, coordinate of circumcentre is  , .
 7 7
Straight Line and Pair of Straight Lines 219

108. If the straight line 2 x + 3 y + 1 = 0 bisects the Put in Eq. (i), we get
−9 14
angle between a pair of lines, one of which in y= x+
this pair is 3 x + 2 y + 4 = 0 , then the equation 46 23
of the other line in that pair of lines is ⇒ 46 y = − 9 x + 28 ⇒ 9 x + 46 y − 28 = 0
[23 April 2018, Shift-I] 109. If the straight lines 2 x + 3 y − 1 = 0,
(a) 3 x + 4 y − 9 = 0 (b) 6 x − 7 y − 14 = 0 x + 2 y − 1 = 0 and ax + by − 1 = 0 form a
(c) 9 x + 46 y − 28 = 0 (d) 9 x − 23 y − 12 = 0 triangle with orthocentre at the origin, then
Sol. (c) (a , b) = [23 April 2018, Shift-II]
Let the angle bisector of 3x + 2y + 4 and (a) (−8, 8) (b) (0, 7 )
(c) (6, 4) (d) (−3, 3)
θ θ Sol. (a)
Equation of line perpendicular to line
ax + by − 1 = 0 and passes through the origin is
bx − ay = 0 … (i)
Now, altitude (i) passes through the intersection
y=mx+C
3x+2y+4=0 of lines 2x + 3y − 1 = 0, and x + 2y − 1 = 0
2x+3y+1=0
So, −b−a=0
y = mx + c is 2x + 3y + 1 = 0 ⇒ a+ b=0 … (ii)
−3 Due to relation (ii), the line ax + by − 1 = 0,
Slope of line 3x + 2y + 4 is m1 =
2 1
becomes x− y− =0 … (iii)
−2 a
Slope of line 2x + 3y + 1 = 0 is m2 =
3 Now, the altitude perpendicular to the line
Angle between lines and bisector are equal, 2x + 3y − 1 = 0, and passes through origin is
m2 − m1 m − m2 3x = 2y … (iv)
so = The point of intersection line (iii) and
1 + m1 m2 1 + m1 m2
1 1 
x + 2y − 1 = 0 is  1 +  , 1 −   satisfy the
2 1
− 2/ 3 + 3/ 2 m + 2/ 3 5 m + 2/ 3
⇒ = ⇒ =  3 
a 3  a
1+1 1−
2m 12 1 − 2m / 3 line …(iv)
3
So, a = − 8 and b = 8
10m
⇒ 5− = 12m + 8 ∴ (a, b ) = (− 8, 8).
3
 10 + 12 m = − 3 110. The product of the perpendicular distances
⇒  
 3  from (1, − 1) to the pair of lines
46 9 x 2 − 4 xy + y 2= 0 , is [23 April 2018, Shift-II]
⇒ m= − 3 ⇒ m= − 2 3
3 46 (a) 1 (b) (c) (d) 2
So, equation of required line is 3 2
9 Sol. (c)
y=− x+ c … (i)
46 The given pair of straight lines,
Now, for intersection point, equation x 2 − 4 xy + y 2 = 0
3x + 2y + 4 = 0 and 2x + 3y + 1 = 0, is given by
⇒ x 2 − 4 xy + 4 y 2 = 3y 2
6x + 4y + 8 = 0
⇒ (x − 2y)2 − ( 3y)2 = 0
6x + 9y + 3 = 0
− − − ⇒ x − (2 + 3) y = 0
5y = 5
or x − (2 − 3) y = 0 … (i)
y = 1 and x = − 2
intersection point is (− 2, 1). So, perpendicular distances from point (1, − 1) to
lines (i)
Required line is also passes through (− 2, 1)
−9 |1 + 2 + 3 | |1 + 2 − 3 |
So, 1 = (− 2) + c ⇒ c = 1 −
9
⇒ c=
14 d1 = , d2 =
46 23 23 1 + (2 + 3)2 1 + (2 − 3)2
220 AP EAMCET Chapterwise Mathematics

(3 + 3) (3 − 3) Sol. (d)
So, d1 d2 = ×
8+ 4 3 8−4 3 Let L1 ≡ 2x + 3y + 6 = 0
9−3 6 3 L2 ≡ 3x − y − 13 = 0
= = =
64 − 48 4 2
Equation of line passing through the point of
111. The vertex A of a triangle lies on the lines intersection.
x + y = 1 and 2 x + 3 y = 6 . If the orthocentre of L1 + λL2 = 0
 3 22  (2x + 3y + 6) + λ(3x − y − 13) = 0
the triangle is O  ,  , then the equation of
7 7  (2 + 3λ) x + (3 − λ) y + 6 − 13λ = 0
OA in the normal form is [24 April 2018, Shift-I] Which is parallel to the line
1 3x − 4 y + 5 = 0
(a) xcos α + ysinα = 7; α = tan−1
7 On comparing the coefficient, we get
2 + 3λ 3 − λ 6 − 13λ
; α = tan−1  
13 1
(b) xcos α + ysinα = = =
17  4 3 −4 5
(c) xcos α + ysin α = ; α = tan−1 
13 13  − 8 − 12λ = 9 − 3λ

4  17  17
13 λ=− putting this value in required equation,
(d) xcos α + ysinα = ; α = tan−1 (4) 9
17 we get
Sol. (d)  2 − 51  x +  3 + 17  y + 6 + 221 = 0
   
 9  9 9
Vertex A is point of intersection of the lines
x + y = 1 and 2x + 3y = 6 ⇒ − 33x + 44 y + 275 = 0
Point of intersection = (− 3, 4) ⇒ 3x − 4 y − 25 = 0.
So, A(− 3, 4) and O , 
3 22
113. If O, G, S are respectively the orthocentre,
7 7 
centroid and circumcentre of a triangle
Equation of line passing through two points whose vertices are A(2 , 3), B(2 , 4) and C(4 , 3),
 22 − 4
  then AO2 + 9 BG 2 + 4 CS2 = [24 April 2018, Shift-I]
 
y−4= 7 (x + 3) (a)
77
(b) 13 (c)
8
(d)
5
3
+ 3 36 9 4
7
6 Sol. (b)
y−4=− (x + 3) Coordinate of vertices of triangle
24
x + 4 y = 13 ) B(2, 4), C(4, 3)
A(2, 3,
Normal form of line x cosα + y sinα = P ∴ AB = 1, BC = 5, CA = 2
13 1 4 So, ∆ABC is right angle triangle where right angle
Where P = , cosα = , sinα =
17 17 17 at A that is orthocentre also.
4 Coordinate of orthocentre is O(2, 3.
)
⇒ tanα =
1 Coordinates of centroid
x + x 2 + x 3 y1 + y2 + y3 
Hence, required equation is =  1 , 
13  3 3 
x cosα + y sinα = ; α = tan− 1 (4).
G =  , 
17 8 10
3 3
112. The equation of the line passing through the
point of intersection of the lines G divide the line joining O and S in the ratio 2 : 1
2 x + 3 y + 6 = 0 , 3 x − y − 13 = 0 and parallel to – + –
the line 3 x − 4 y + 5 = 0 is
–1 1
[24 April 2018, Shift-I]
(a) 3 x − 4 y + 75 = 0 (b) 3 x − 4 y + 15 = 0 8 2x + 2
= ⇒x = 3
(c) 3 x − 4 y + 25 = 0 (d) 3 x − 4 y − 25 = 0 3 3
Straight Line and Pair of Straight Lines 221

10 2y + 3 ⇒ 4 + 3+ k = 0 ⇒ k = − 7
⇒ y = ⇒ S  3, 
7 7
=
3 3 2  2 ⇒ 2x + 3y − 7 = 0 …(iii)
So, AO2 + 9BG2 + 4 CS 2 Point of intersection of Eqs. (ii) and (iii)
x = , y = ⇒  , 
AO2 = (2 − 2)2 + (3 − 3)2 = 0 ⇒ AO2 = 0 7 7 7 7
2 2 5 5  5 5
BG2 =  2 −  +  4 −  = ⇒ 9BG2 = 8
8 10 8
 3  3 9 Perpendicular line to the Eq. (2) is
2 x + y + k = 0 which pass through (2, 1)
CS 2 = (4 − 3)2 +  3 −  = ⇒ 4CS 2 = 5
7 5
 2 4 ⇒ 2+ 1 + k = 0 ⇒ k = − 3
∴ AO2 + 9BG2 + 4CS 2 = 13. Hence, ⊥ line is x + y − 3 = 0 …(iv)
Point of intersection of Eqs. (iv) and (i)
114. If two sides of a triangle are given by
x + y − 3= 0
3 x 2 − 5 xy + 2 y 2 = 0 and its orthocentre is (2 , 1) ,
3x − 2y = 0 ⇒ x = , y = ⇒  , 
6 9 6 9
then the equation of the third side of the
triangle is [24 April 2018, Shift-I]
5 5  5 5
(a) 5 x − 10 y + 1 = 0 (b) 10 x + 5 y − 1 = 0 So, third side of triangle pass through these point
 7 , 7  and  6 , 9  is
(c) 5 x − 10 y = 21 (d) 10 x + 5 y = 21    
 5 5  5 5
Sol. (d) 9 7
Given pair equation of two sides of triangle, −
7 5 5 7
⇒ y− = x − 
3x 2 − 5xy + 2y 2 = 0 5 6− 7  5
⇒ (3x − 2y) (x − y) = 0 5 5
So, equation of sides are 7 2  7
⇒ y− = x − 
3x − 2y = 0 … (i) 5 −1  5
x− y=0 … (ii) 5y − 7 − 10 x + 14
⇒ =
Perpendicular line to the Eq. (i) 5 5
2x + 3y + k = 0 which pass through the point (2, 1). ⇒ 10 x + 5y = 21.
19
Circle and System
of Circles
1. If one end of diameter of the circle Sol. (c)
x + y − 4 x − 6 y + 11 = 0 is (3 , 4), then the
2 2 Let the point be (h, k)
|4h + 3k − 12| 4
other end of the diameter is [17 Sep. 2020, Shift-I] So, = ⇒|4h + 3k − 12|= 4
5 5
(a) (0, 1) (b) (1, 1) (c) (1, 2 ) (d) (1, 0)
⇒ (4h + 3k = 16) or (4h + 3k = 8)
Sol. (c)
(h, k) lies on circle so
Given, circle x 2 + y 2 − 4 x − 6 y + 11 = 0
h2 + k2 = 4
⇒ Centre = (2, 3)
 2  16 − 4h  2   2  8 − 4h  2 
One end of diameter = (3, 4) ⇒  h +   = 4 or  h +   = 4
 3     3  
Let other end be (h, k)
h+ 3 k+ 4 ⇒ (25h2 − 128h + 220 = 0) or (25h2 − 64h + 28 = 0)
So, = 2, = 3 ⇒ h = 1, k = 2
2 2
⇒ 25h2 − 128h + 220 = 0 ⇒ Imaginary point
2. The equation of a circle with centre (5, 4) So, 25h2 − 64h + 28 = 0 will be considered and
and touch the Y -axis is [17 Sep. 2020, Shift-I]  h = 2, 14 
(a) x2 + y2 − 10 x − 8 y − 16 = 0  
 25
(b) x2 + y2 − 10 x − 8 y − 61 = 0 At h = 2, k = 0
(c) x2 + y2 + 10 x + 8 y + 16 = 0 14 48
At h = ,k=
(d) x2 + y2 − 10 x − 8 y + 16 = 0 25 25
So, (h, k) = (2, 0) or  , 
Sol. (d) 14 48
 25 25
Centre (5, 4) circle touches Y-axis
So, radius = 5units
4. The equation of the circle passing through
Equation of circle
(0, 0) and which makes intercepts a and b on
⇒ (x − 5)2 + (y − 4)2 = 52 the coordinate axes is [17 Sep. 2020, Shift-I]
⇒ x 2 + y 2 − 10 x − 8 y + 16 = 0 (a) x2 + y2 + ax + by = 0
3. The point on the circle x 2 + y 2 = 4 whose (b) x2 + y2 + ax − by = 0
distance from the line 4 x + 3 y − 12 = 0 is 4/5 (c) x2 + y2 − ax + by = 0
units is equal to [17 Sep. 2020, Shift-I] (d) x2 + y2 − ax − bx = 0
(a)  , 
12 36
 25 25 
(b) (4, 0) Sol. (d)
Centre =  ,
a b
−14 48  
(c) (2, 0) (d)  ,  2 2
 25 25 
Circle and System of Circles 223

a2 + b2 So, slope of normal is, mN = − 1 / mT = − 1


Radius = Equation of normal in one point forms is,
2
y − y1 = m (x − x1)
⇒ y − 1 = − 1 (x − 1)
(0, b) ⇒ x + y − 2= 0

O 7. The length of the diameter of the circle


b
x 2 + y 2 − 6 x − 8 y = 0 is .......... units
[17 Sep. 2020, Shift-II]
a (a, 0) (a) 5 (b) 10
(c) 15 (d) 20
Equation of circle Sol. (b)
2 2
a2 + b2
⇒  x −
a  b Cirle is x 2 + y 2 − 6 x − 8 y = 0
 + y−  =
 2  2  4
Radius of circle is,
⇒ x 2 + a 2 − ax + y 2 + b 2 − bx = a 2 + b 2
r= g2 + f 2 − c = 32 + 42 − 0 = 5 units
⇒ x 2 + y 2 − ax − bx = 0
Diameter = 2r = 10 units
5. Find the value of m + n, if the circumference
8. The length of the tangent drawn from any
of the circle x 2 + y 2 + 8 x + 8 y − m = 0 is
point on the circle x 2 + y 2 + 2 gx + 2 fy + c1 = 0
bisected by the circle x 2 + y 2 − 2 x + 4 y + n = 0 .
to the circle x 2 + y 2 + 2 gx + 2 fy + c2 = 0 is
[17 Sep. 2020, Shift-I]
[17 Sep. 2020, Shift-II]
(a) − 56 (b) 56 (c) 50 (d) −34
(a) c 2 − c1 (b) c12 + c 22
Sol. (a)
(c) c1 + c 2 (d) c1 − c 2
Let S1 ≡ x 2 + y 2 + 8 x + 8 y − m = 0
Sol. (a)
S2 = x 2 + y 2 − 2x + 4 y + n = 0
Circles are C1 ≡ x 2 + y 2 + 2gx + 2 fy + c1 = 0
Equation of common chord
⇒ S1 − S2 = 0 and C2 ≡ x 2 + y 2 + 2gx + 2 fy + c 2 = 0
⇒ 10 x + 4 y − (m + n) = 0 Clearly circles are concentric,
⇒ 10 x + 4 y = (m + n) A
Centre of bisected circle is (−4, −4) which will lie
on common chord r1
C1
So, {(m + n) = −56} T
6. The equation of normal at (1, 1) to the circle r2 O
C2
x + y − x − 3 y − 4 = 0 is [17 Sep. 2020, Shift-II]
2 2

(a) x + y − 2 = 0 (b) 2 x − y − 1 = 0
(c) x − y + 2 = 0 (d) x − y − 2 = 0
Clearly, length of tangent is
Sol. (a)
Circle is, x 2 + y 2 − x − 3y − 4 = 0 AT = AO2 − OT 2 = r12 − r22
Slope of tangent at (1, 1) is obtained by = (g2 + f 2 − c1) − (g2 + f 2 − c 2)
differentiating above equation,
= c 2 − c1
2x + 2yy′ − 1 − 3y ′ = 0
1 − 2x 9. If 3 x + y + k = 0 is a tangent to the circle
⇒ y′ =
2y − 3 (1 , 1) x 2 + y 2 = 10 , then k = ........ [17 Sep. 2020, Shift-II]
1− 2 (a) ± 7 (b) ± 5
⇒ y′ = =1
2− 3 (c) ± 9 (d) ± 10
224 AP EAMCET Chapterwise Mathematics

Sol. (a) Sol. (d)


As circles intersects in two distinct points
3x+y+k=0
(x –1)2+(y–3)2=r2

C1 C2
t=3x+y+k=0

x2+y2=10
(x2 + y2–8 x+2 y+8=0
Length of perpendicular from (0, 0) on
So, |r1 − r2|<|C1 C2|< r1 + r2 …(i)
3x + y + k = 0 = radius of circle.
Here, C1 = (1, 3), C2 = (4, − 1)
3× 0 + 0 + k ∴ C1 C2 = (4 − 1)2 + (−1 − 3)2 = 9 + 16 = 5
⇒ = 10
32+ 12
Also, r2 = g + f −C=
2 2
42 + 1 − 8 = 3
2

k
⇒ = ± 10 Hence, from Eq. (i), we have 2 < r < 8.
10
⇒ k = ± ( 10)2
12. The equation of the smallest circle passing
through the intersection of the line x + y = 1
⇒ k = ± 10
and the circle x 2 + y 2 = 9 is .......
10. The equations of tangents to the circle [18 Sep. 2020, Shift-I]
x 2 + y 2 = 10 from the point (4 , − 2) are (a) x2 + y2 − 9 − ( x + y + 1) = 0
[17 Sep. 2020, Shift-II] (b) x2 + y2 − 9 − ( x + y − 1) = 0
(a) x + y = 2, 3 x + 2 y = 16 (c) x2 + y2 − 9 − x + y − 1= 0
(b) 5 x + y = 18, 3 x − y = 4 (d) x2 + y2 − 9 + x + y − 1= 0
(c) 3 x + y = 10, x − 3 y = 10
(d) 5 x − y = 4, x + y = 0 Sol. (b)
Sol. (c) The family of circles passes through the intersection
of circles x 2 + y 2 = 9 and line x + y = 1 is
For a tangent, length of perpendicular from origin
= radius. (x 2 + y 2 − 9) + λ(x + y − 1) = 0
So we check distance from (0, 0) and for any ⇒ x + y 2 + λx + λy − (λ + 9) = 0 having centre
2

tangent it must be 10 .  − λ ,− λ  .
 
For 3x + y = 10 and x − 3y = 10 ,  2 2
Distance of (0, 0) from line 3x + y = 10 is, For the smallest circle, the line x + y = 1 must be
diameter of the circle, so
3 × 0 + 0 − 10 10
d= = = 10 λ λ
− − = 1 ⇒ λ = −1
32 + 12 10
2 2
Also from x − 3y = 10, So, equation of the required circle is
0 − 3 × 0 − 10 x 2 + y2 − x − y − 8 = 0
d= = 10 or, (x 2 + y 2 − 9) − (x + y − 1) = 0
32 + 12
So, option (c) is correct. 13. A circle is drawn touching the X -axis, with
its centre at the point of reflection of (m , n) on
11. If the two circles (x − 1)2 + (y − 3)2 = r 2 and the line y − x = 0. Then the equation of the
x 2 + y 2 − 8 x + 2 y + 8 = 0 intersect in two circle is [18 Sep. 2020, Shift-I]
different points, then what can we conclude (a) x2 + y2 − 2 mx − 2 ny + m2 = 0
about r? [17 Sep. 2020, Shift-II] (b) x2 + y2 − 2 mx + 2 ny + m2 = 0
(a) r < 2 (b) r = 2 (c) x2 + y2 + 2 nx − 2 my − n2 = 0
(c) r > 2 (d) 2 < r < 8 (d) x2 + y2 − 2 nx − 2 my + n2 = 0
Circle and System of Circles 225

Sol. (d) Sol. (a)


The point of reflection of (m, n) on the line y − x = 0 According to the question, from the diagram, in
is (n, m), so equation of circle having centre (n, m) ∆PQR
and let radius r is PQ
tanθ = ⇒ PR = PQ cotθ …(i)
(x − n)2 + (y − m)2 = r 2 …(i) PR
Q Circle (i) touches the X − axis, so r = m
So, equation of required circle is
(x − n)2 + (y − m)2 = m2
r r
P R
or x 2 + y 2 − 2nx − 2my + n2 = 0 90º–θ θ

14. The radical centre of the circles


x 2 + y 2 − 4 x − 6 y + 5 = 0,
x 2 + y 2 − 2 x − 4 y − 1 = 0 and χ

x + y − 6 x − 2 y = 0 is equal to
2 2

[18 Sep. 2020, Shift-I]


Q S
(a)  ,  (b)  , 
33 20 33 10
 4 3  4 3 and in ∆PRS,
33 − 20  33 − 10  RS
(c)  ,  (d)  ,  tan (90° − θ) = ⇒ PR = RS tan θ …(ii)
 4 3   4 3  PR
∴ PQ cot θ = RS tanθ
Sol. (*) PQ
⇒ tanθ = …(iii)
Equation of given circles RS
S1 : x 2 + y 2 − 4 x − 6 y + 5 = 0 From Eqs. (ii) and (iii), we have
S2: x 2 + y 2 − 2x − 4 y − 1 = 0 PQ
PR = RS = PQ. RS
and S3: x 2 + y 2 − 6 x − 2y = 0 RS
∴ Radical axis of circles S1 and S2 is ⇒ 2r = PQ. RS Q PR = 2r
2x + 2y − 6 = 0 16. The area of an equilateral triangle inscribed
⇒ x+ y=3 …(i) in the circle x 2 + y 2 − 6 x + 2 y − 28 = 0 is
Similarly, the radical axis of circles S2 and S3 is
sq. units [18 Sep. 2020, Shift-I]
− 4 x + 2y + 1 = 0 …(ii)
27 3 37 3 31 3 57 3
Q Radical centre is point of concurrency of radical (a) (b) (c) (d)
2 2 2 2
axes, from radical axes Eqs. (i) and (ii), we get
11 7 Sol. (d)
y= and x =
6 6 Equation of given circle is
 7 11  x 2 + y 2 − 6 x + 2y − 28 = 0
∴Radical centre is  , 
6 6 ⇒ (x − 3)2 + (y + 1)2 = 38
(*) No option is correct. Now, area of equilateral triangle
15. Let PQ and RS be tangents at the extremities A
of a diameter PR of a circle of radius r such
that PS and RQ intersect at a point X on the r1 C1
circumference of the circle, then 2r equals T
[18 Sep. 2020, Shift-I] r2 C2
PQ + RS O
(a) PQ ⋅ RS (b)
2
2 PQ ⋅ RS (PQ )2 + (RS )2
(c) (d)
PQ + RS 2
226 AP EAMCET Chapterwise Mathematics

ABC = 3(Area of ∆GBC) 19. The length of the chord intercepted by the
= 3 r 2 sin 120°  = (38) circle x 2 + y 2 − 6 x + 8 y − 5 = 0 on the line
1 3 3 57 3
=
2  2 2 2 2 x − y = 5 is equal to ....... units
17. The equation of the circle with centre (2, 3) [18 Sep. 2020, Shift-II]
and touching the line 3 x − 4 y + 1 = 0 is (a) 10 (b) 12 (c) 7 (d) 8
[18 Sep. 2020, Shift-II] Sol. (a)
(a) x2 + y2 + 4 x + 4 y + 12 = 0 Centre = (3, – 4)
(b) x2 + y2 − 4 x − 6 y − 14 = 0 Radius = 9 + 16 + 5 = 30
(c) x2 + y2 − 4 x − 6 y + 14 = 0 d= Perpendicular distance from (3, – 4) to the
(d) x2 + y2 − 4 x − 6 y + 12 = 0 chord 2x − y − 5 = 0
2(3) − (−4) − 5 6+ 4− 5
Sol. (d) d= ⇒ d= ⇒ d= 5
4+1 5
Centre c = (2, 3)
radius = Perpendicular distance from centre (2, 3) length of chord = 2 r 2 − d 2 = 2 30 − 5
to the line 3x − 4 y + 1 = 0 = 2 × 5 = 10 units
Hence, option (a) is correct.

(2, 3) 20. If one of the two circle


r x 2 + y 2 + α1 (x − y) + c = 0 and
x 2 + y 2 + α 2 (x − y) + c = 0, lies within the
P 3x–4y+1=0
other, then ......... (where α1 , α 2 ∈R, α1 ≠ α 2 )
3(2) − 4(3) + 1 7 − 12 5
r= = ⇒ r= =1 [18 Sep. 2020, Shift-II]
3 + (−4)
2 2 25 5 (a) c< 0 (b) c = 0 (c) c > 0 (d) c ≥ 0
Equation of circle is Sol. (c)
(x − 2)2 + (y − 3)2 = (1)2
Equation of given circles are
x 2 + 4 − 4 x + y2 + 9 − 6 y = 1 x 2 + y 2 + α1 (x − y) + C = 0
x + y − 4 x − 6 y + 13 − 1 = 0
2 2
and x 2 + y 2 + α 2(x − y) + C = 0
x 2 + y 2 − 4 x − 6 y + 12 = 0
Since, if one of the two given circles, lies within
Hence, option (d) is correct. the other, then
18. Consider the family of circles C1 C2 < |r1 − r2|
x 2 + y 2 − 2 x − 2 λy − 8 = 0 which passes  − α1 − α1   −α2 , −α2 
Where, C1 =  ,  , C2 =  
 2 2   2 2 
through two fixed points A and B distance
between them is [18 Sep. 2020, Shift-II] α12 α 22
r1 = − C and r2 = −C
(a) 4 (b) 4 2 (c) 6 (d) 8 2 2
Sol. (c) (α1 − α 2)2 α12 α 22
∴ < −C− −C
x + y − 2x − 2λy − 8 = 0
2 2
2 2 2
(x 2 + y 2 − 2x − 8) − λ (2y) = 0
α12 + α 22 − 2α1 a 2  α12   α2 
x 2 + y 2 − 2x − 8 = 0 and 2y = 0 ⇒ y = 0 ⇒ < − C +  2 − C
2  2   2 
x 2 − 2x − 8 = 0
Distance between points A, B = x1 − x 2 α12 α2
−2 −C 2 −C
2 2
D b 2 − 4ac 4 − 4 ⋅1(−8)
= = = ⇒ −α1α 2 < −2C − α12 − 2C α 22 − 2C
a a 1
= 36 = 6 ⇒ (α12 − 2C)(α 22 − 2C) < (α1α 2 − 2C)2
Hence, option (c) is correct. ⇒ α12α 22 − 2(α12 + α 22)C + 4C 2 < α12α 22 − 4Cα1α 2 + 4C 2
Circle and System of Circles 227

⇒ 2C[α12 + α 22 − 2α1α 2] > 0 ⇒ x−y=0


⇒ C(α1 − α 2) > 0 ⇒ C > 0 as α1 ≠ α 2
2 Hence, option (c) is correct.

Hence, option (c) is correct. 23. Find the minimum radius of the circle which
is orthogonal to both the circles
21. The circle x 2 + y 2 − 6 x − 10 y + p = 0 neither
x 2 + y 2 + 4 x + 3 = 0 and
intersects nor touch the coordinate axes and
the point (1, 4) lies inside the circle. Then the x 2 + y 2 − 12 x + 35 = 0 . [18 Sep. 2020, Shift-II]
range of possible values of ‘p’ is (a) 1 (b) 4 (c) 17 (d) 15
[18 Sep. 2020, Shift-II] Sol. (d)
(a) 23< p < 25 (b) 25 < p < 29 Centre of circle cuts the circles
(c) 21 < p < 23 (d) 12 < p < 21
S1 : x 2 + y 2 + 4 x + 3 = 0
Sol. (b) S2: x 2 + y 2 − 12x + 35 = 0
The equation of given circle is Radical Axis is S1 − S2 = 0 ⇒ x = 2
x 2 + y 2 − 6 x − 10 y + p = 0 and (2, 0) lies on the line segment joining centres
⇒ (x − 3)2 + (y − 5)2 = 34 − p of circles S1 and S2
∴ The point (1, 4) lies inside the circle, so Minimum radius = length of tangent from (2, 0)
to circle S1 = 0 (or) S2 = 0
1 + 16 − 6 − 40 + p < 0
= 4 + 8 + 3 = 15
⇒ p < 29 …(i)
Hence, option (d) is correct.
∴The circle neither intersects nor touches the
coordinate axis, then 24. Find the maximum distance of the point
r = 34 − p < 3 ⇒ 34 − p < 9 K(10, 7) from the circle
⇒ p > 25 … (ii) x 2 + y 2 − 4 x − 2 y − 20 = 0 [21 Sep. 2020, Shift-I]
and r = 34 − p < 5 (a) 25 (b) 10 (c) 15 (d) 5
⇒ 34 − p < 25 Sol. (c)
⇒ p> 9 … (iii) Equation of given circle is
from in equalities Eqs. (i), (ii) and (iii), we get x 2 + y 2 − 4 x − 2y − 20 = 0
25 < p < 29
Hence, option (b) is correct. ⇒ (x − 2)2 + (y −1)2 = 25
Having centre C(2, 1) and radius r = 5
22. The equation of the normal to the circle
∴ The maximum distance of the point K(10, 7)
 1 1 
x 2 + y 2 = 16 at the point  ,  is from the given circle is CK + r
 3 3
= (10 − 2)2 + (7 − 1)2 + 5
[18 Sep. 2020, Shift-II]
3 = 64 + 36 + 5 = 10 + 5 = 15 unit
(a) x + y = 0 (b) x − y =
4 Hence, option (c) is correct.
3
(c) x − y = 0 (d) x + y = 25. The centre of a circle is (2, − 3) and the
4
circumference is 10 π. Then its equation is
Sol. (c) [21 Sep. 2020, Shift-I]
Centre C = (0,0)
(a) x2 + y2 + 4 x + 6 y + 12 = 0
P = 
1 1 
,  (b) x2 + y2 − 4 x + 6 y + 12 = 0
 3 3
(c) x2 + y2 − 4 x + 6 y − 12 = 0
Since, every normal passes through centre
(d) x2 + y2 − 4 x − 6 y − 12 = 0
∴Equation of Normal = Equation of CP
1
−0 Sol. (c)
⇒ y−0= 3 (x − 0) ⇒ y = 1 ⋅ x Let the radius of required circle is ‘r’, so the
1
−0 circumference = 2πr = 10 π (given)
3 ⇒ r=5
228 AP EAMCET Chapterwise Mathematics

and centre of required circle is (2, − 3) Sol. (a)


So, the equation of required circle is The centre of the circle x 2 + y 2 − 3x − 4 y − 1 = 0 is
(x − 2)2 + (y + 3)2 = 25 C(3/ 2, 2), now as it is given that the required circle
⇒ x 2 + y 2 − 4 x + 6 y − 12 = 0 passes through the point C(3/ 2, 2) and having
centre (5, 2), so radius of required circle is
Hence, option (c) is correct.
2
r=  5 − 3 + (2 − 2)2 = 7
26. If y = 3 x + k1 and y = 3 x + k2 are two 


2 2
parallel tangents of a circle of radius 2 units,
∴ Equation of required circle is
then| k1 − k2 | is equal to [21 Sep. 2020, Shift-I] 2
(x − 5)2 + (y − 2)2 =  
7
(a) 1 (b) 8 (c) 4 (d) 2
 2
Sol. (b) ⇒ 4 x 2 + 4 y 2 − 40 x − 16 y + 116 = 49
The distance between parallel tangents to a circle
⇒ 4 x 2 + 4 y 2 − 40 x − 16 y + 67 = 0
is equals to the diameter of the circle, so
Hence, option (a) is correct.
y=√3x+k2
29. If the chord of contact of tangents from a
point A to a given circle passes through B,
C
2 then the circle with AB as a diameter will
......... [21 Sep. 2020, Shift-I]
(a) Touch the given circle internally
y=√3x+k1 (b) Cut the given circle orthogonally
|k1 − k2| (c) Touch the given circle externally
=4 ⇒|k1 − k2| = 8 (d) Neither intersect nor touch the given circle
1+ 3
Hence, option (b) is correct. Sol. (b)
Let an equation of circle
27. Find the equation of circle having normals
x 2 + y 2 + 2gx + 2 fy + c = 0 and point A(x1 , y1)
(x − 1)(y − 2) = 0 and a tangent 3 x + 4 y = 6 ?
[21 Sep. 2020, Shift-I] Then equation of chord of contact is
(a) ( x − 1) + ( y − 2 ) = 1 (b) ( x − 2 ) + ( y − 1) = 1
2 2 2 2 xx1 + yy1 + g(x + x1) + f (y + y1) + c = 0
(c) ( x + 1)2 + ( y + 2 )2 = 1 (d) ( x + 2 )2 + ( y + 1)2 = 1 ⇒ (x1 + g) x + (y1 + f) y + (gx1 + fy1 + c) = 0
Let another point B (x 2 , y2) through which chord
Sol. (a) (i) passes, so
The equation of normals to the circle are x − 1 = 0 x1 x 2 + gx 2 + y1 y2 + fy2 + gx1 + fy1 + c = 0 … (ii)
and y − 2 = 0, so centre of the circle is Now equation of circle having AB as diameter is
(1, 2) and since 3x + 4 y = 6 is the tangent to the (x − x1)(x − x 2) + (y − y1) (y − y2) = 0
3+ 8 − 6
circle so radius r = =1 ⇒ x 2 + y 2 − (x1 + x 2) x − (y1 + y2) y
32 + 42
+ x1 x 2 + y1 y2 = 0 …(iii)
∴ Equation of required circle is By the circles
(x − 1)2 + (y − 2)2 = 1 x 2 + y 2 + 2gx + 2 fy + c = 0 and circle (iii)
Hence, option (a) is correct. the 2g1 g2 + 2 f1 f2
x + x2  + y2 
= − 2g  1 y
28. A circle passes through the centre of another
 − 2f  1 
circle x 2 + y 2 − 3 x − 4 y − 1 = 0 and whose  2   2 
centre is (5, 2). Then the equation of this = − gx1 − gx 2 − fy1 − fy2
circle is ......... [21 Sep. 2020, Shift-I] = x1 x 2 + y1 y2 + c {from Eq. (ii)}
(a) 4 x2 + 4 y2 − 40 x − 16 y + 67 = 0 = c1 + c 2
Q 2g1 g2 + 2 f1 f2 = c1 + c 2
(b) 3 x2 + 3 y2 − 40 x − 16 y + 67 = 0
∴ Circles x 2 + y 2 + 2gx + 2 fy + c = 0 and Eq. (iii)
(c) 2 x2 + 2 y2 − 40 x − 16 y + 67 = 0 cuts orthogonally.
(d) x2 + y2 − 10 x − 4 y + 67 = 0 Hence, option (b) is correct.
Circle and System of Circles 229

30. If the point (1, 4) lies inside the circle Sol. (b)
x 2 + y 2 − 6 x − 10 y + p = 0 and the circle does The radical axis of any two circles is perpendicular
to the line joining their centres.
not touch or intersect the coordinates axes,
Let, the equation of two circles are
then [21 Sep. 2020, Shift-II]
(a) 0 < p < 34 (b) 25 < p < 29 x 2 + y 2 + 2g1 x + 2 f1 y + c1 = 0
(c) 9 < p < 25 (d) 7 < p < 29 and x 2 + y 2 + 2g2 x + 2 f2 y + c 2 = 0
Sol. (b) The equation of the radical axis is
Equation of given circle 2(g1 − g2) x + 2( f1 − f2) y + (c1 − c 2) = 0 …(i)
x 2 + y 2 − 6 x − 10 y + p = 0 g1 − g2
Q Slope of line (i) is − = m1 (let)…(ii)
⇒ (x − 3)2 + (y − 5)2 = 34 − p f1 − f2
∴ 34 − p > 0 ⇒ p < 34 …(i) and slope line joining centres of the circles is
Q Circle doesn’t touch or intersect the coordinate f1 − f2
= m2 (let)
axis, so g1 − g2
34 − p < 3, (for X-axis) g1 − g2 f1 − f2
Q m1 m2 = − × = −1
⇒ 34 − p < 9 ⇒ p > 25 …(ii) f1 − f2 g1 − g2
and 34 − P < 5, (for Y-axis) Hence, option (b) is correct.
⇒ 34 − p < 25
33. In ∆ ABC, the circle that touches the sides BC
⇒ p> 9 …(iii)
internally and other two sides AB and AC
Q Point (1, 4) lies inside the circle, so
externally, is called ……… .
1 + 16 − 6 − 40 + p < 0 ⇒ p < 29 …(iv)
[21 Sep. 2020, Shift-II]
From inequalities Eqs. (i), (ii), (iii) and (iv), we get
25 < p < 29 (a) Ex circle opposite to angle A
(b) Inscribed circle opposite to angle A
Hence, option (b) is correct.
(c) Circumcircle of the triangle
31. The radical axis of the co-axial system of (d) No such circle exists
circles with limiting points (1, 2) and (−2 ,1) is
Sol. (a)
[21 Sep. 2020, Shift-II]
In ∆ABC, the circle that touches the side BC
(a) x + 3 y = 0 (b) 2 x + 3 y = 0 internally and other two sides AB and AC
(c) 3 x + 2 y = 0 (d) 3 x + y = 0 externally is called Ex-circle opposite to angle A.
Sol. (d) Hence, option (a) is correct.
The radical axis of the co-axial system of circles
with limiting points A(1, 2) and B(−2, 1) is the 34. The length of the tangent drawn from the
perpendicular bisector of line joining limiting mid-point of the line joining the origin and
points A and B. the point (4 , − 4), to the circle 2 x 2 + 2 y 2 − y = 0
Q Mid-point of A and B is  − ,  and slope of
1 3
 2 2 is ……… units [22 Sep. 2020, Shift-I]

perpendicular to AB is −3 (a) 3 2 (b) 2


(c) 10 (d) 3
∴Equation of required radical axis is
y − = −3 x +  ⇒ y − = −3x − Sol. (d)
3 1 3 3
2  2 2 2 The mid-point of line joining origin and the point
⇒ 3x + y = 0 (4, − 4) is P(2, − 2), so the length of tangent drawn
Hence, option (d) is correct. from point P(2, − 2) to the given circle
2x 2 + 2y 2 − y = 0
32. The radical axis of any two circles is ………
y
to the line joining their centres or x 2 + y 2 − = 0 is
2
(a) Parallel [21 Sep. 2020, Shift-II]
(− 2)
(b) Perpendicular (2)2 + (− 2)2 −
(c) Intersecting but not perpendicular 2
(d) Can’t be determined = 4 + 4 + 1 = 3 units.
230 AP EAMCET Chapterwise Mathematics

35. If the circles x 2 + y 2 − 2 x − 2 y − 7 = 0 and Sol. (a)


x + y + 4 x + 2 y + k = 0 cut orthogonally,
2 2 Equation of given circle
x 2 + y 2 − 2x − 4 y − 20 = 0
then the length of their common chord is
……… units [22 Sep. 2020, Shift-I] ⇒ ( x − 1)2 + ( y − 2)2 = 25 ...(i)
6 12 Having centre C1 (1, 2) and radius 5 units.
(a) 2 (b) 5 (c) (d)
13 13 Q Another circle touches the circle (i) at point (5, 5)
Sol. (d) having radius 5 units, so centre ‘C2’ of the second
circle is collinear with C1 and point (5, 5) is the
Since the given circles
mid-point of C1 C2, so C2(9, 8).
x 2 + y 2 − 2x − 2y − 7 = 0 and
So, equation of the second circle is
x 2 + y 2 + 4 x + 2y + k = 0 cut orthogonally, then
(x − 9)2 + (y − 8)2 = 25
(− 2 ) ( 2 ) + (− 2 ) ( 1 ) = k − 7
⇒ x 2 + y 2 − 18 x − 16 y + 120 = 0
⇒ − 4 − 2= k − 7⇒k =1
So, equation of common chord is 38. If the circle x 2 + y 2 + 6 x − 2 y + k = 0 bisects
6 x + 4 y + 8 = 0 ⇒ 3x + 2y + 4 = 0
the circumference of the circle
∴ Length of the chord, AB = 2(AM)
x 2 + y 2 + 2 x − 6 y − 15 = 0 , then ‘k’ is equal to
x 2 + y 2 − 2x − 2y − 7 = 0
[22 Sep. 2020, Shift-II]
(a) 21 (b) −21 (c) 23 (d) −23
Sol. (d)
C (1, 1) Given, equation of circles are
3 S : x 2 + y 2 + 6 x − 2y + k = 0
3x+2y+4=0 S ′ : x 2 + y 2 + 2x − 6 y − 15 = 0
A M B
Centre = (−1, 3)
Equation of common chaod is S − S ′ = 0
(3 + 2 + 4)2 4 x + 4 y + (k + 15) = 0
= 2 AC 2 − CM 2 = 2 9 − … (i)
9+ 4 Centre of S ′ lies on the above line
117 − 81 36 12 ∴(−1, 3) lies on Eq. (i)
=2 = 2× = units
13 13 13 4(−1) + 4(3) + k + 15 = 0 ⇒ k = − 23
Hence, option (d) is correct.
36. For the circle x 2 + y 2 − 9 = 0, find the
equation of the chord having (1, 2) as its 39. The length of the tangent from (6 , 8) to the
mid-point. [22 Sep. 2020, Shift-I] circle x 2 + y 2 = 4 is [22 Sep. 2020, Shift-II]
(a) x + 2 y + 5 = 0 (b) x − 3 y − 5 = 0 (a) 6 (b) 2 6 (c) 4 6 (d) 5 6
(c) x − 3 y + 5 = 0 (d) x + 2 y − 5 = 0
Sol. (c)
Sol. (d) Let P = (6, 8)
Equation of chord having mid-point (1, 2) w.r.t. S : x 2 + y2 − 4 = 0
circle x 2 + y 2 − 9 = 0 is
Length of tangent = S11 = (6)2 + (8)2 − 4
1 x + (2) y − 9 = 12 + 22 − 9 ⇒ x + 2y = 5
()
= 36 + 64 − 4 = 96 = 4 6
37. Find the equation of a circle with radius Hence, option (c) is correct.
5 units and touching the circle
40. If the length of the tangent from ( f , g) to the
x 2 + y 2 − 2 x − 4 y − 20 = 0 at the point (5, 5). circle x 2 + y 2 = 6 be twice the length of the
[22 Sep. 2020, Shift-I] tangent from the same point to the circle
(a) x2 + y2 − 18 x − 16 y + 120 = 0 x 2 + y 2 + 3 x + 3 y = 0 , then
(b) x2 + y2 + 18 x + 16 y − 120 = 0 f 2 + g 2 + 4 f + 4 g + 2 is equal to
(c) x2 + y2 − 18 x + 16 y − 120 = 0 [22 Sep. 2020, Shift-II]
(d) x2 + y2 + 18 x + 16 y + 120 = 0 (a) −1 (b) 1 (c) 0 (d) −2
Circle and System of Circles 231

Sol. (c) 9 17
∴ r = g2 + f 2 − c = +1+1 ⇒ r=
Given, P = ( f , g) 4 4
S : x 2 + y2 − 6 = 0 17 π
∴Area = πr 2 =
4
S ′ : x 2 + y 2 + 3x + 3y = 0
Hence, option (a) is correct.
According to the question,

S11 = 2 S11 43. The equation of a circle which touches the
X -axis and whose centre is (1, 2) is
Squaring on both sides [23 Sep. 2020, Shift-I]
′ )2
( S11 )2 = (2 S11 (a) ( x − 2 )2 + ( y − 1)2 = 4
S11 = 4S11′ (b) ( x − 1)2 + ( y − 2 )2 = 4
(g + f − 6) = 4 (g2 + f 2 + 3g + 3 f)
2 2
(c) ( x − 1)2 + ( y + 2 )2 = 4
3g2 + 3 f 2 + 12g + 12 f + 6 = 0 (d) ( x + 2 )2 + ( y − 1)2 = 4
Dividing by 3 on both sides, Sol. (b)
g2 + f 2 + 4 g + 4 f + 2 = 0 Given, centre = (1, 2)
Hence, option (c) is correct. If a circle touches the X-axis, then radius
= y-coordinate of centre
41. The radius of any circle touching the lines
∴ r=2
3 x − 4 y + 5 = 0 , 6 x − 8 y − 9 = 0 is
[22 Sep. 2020, Shift-II]
23 20 19
(a) 1 (b) (c) (d)
15 19 20 (1, 2)
2
Sol. (d) r
Given lines are 3x − 4 y + 5 = 0 … (i)
1
6x − 8y − 9 = 0
9
⇒ 3x − 4 y − = 0 … (ii) ∴ Required equation of circle is
2
(x − h)2 + (y − k)2 = r 2
Given lines (i) and (ii) are parallel to each other
and they are touching the required circle (x − 1)2 + (y − 2)2 = 22
∴Distance between lines (i) and (ii) = diameter of (x − 1)2 + (y − 2)2 = 4
circle
Hence, option (b) is correct.
9
5+
|C1 − C2| 2 44. If a line drawn from a fixed point M(a , b) cuts
∴ = 2r ⇒ = 2r
a + b 2 2
3 + (−4) 2 2
the circle x 2 + y 2 = k 2 at C and D, then
∴ 2r =
19 / 2
⇒ 2r =
19
⇒ r=
19 MC × MD is equal to [23 Sep. 2020, Shift-I]
5 10 20 (a) a2 + b 2 + k 2 (b) a2 + b 2 − k 2
Hence, option (d) is correct. (c) a2 − b 2 − k 2 (d) k 2
42. Find the area of the circle Sol. (b)
(x + 1) (x + 2) + (y − 1) (y + 3) = 0
Given, M = (a , b)
[22 Sep. 2020, Shift-II]
17 π 17 π 2π π Equation of circle is x 2 + y 2 = k2
(a) (b) (c) (d)
4 2 17 3
Sol. (a) T
Given circle is,
(x + 1) (x + 2) + (y − 1)(y + 3) = 0 M
C D
x 2 + y 2 + 3x + 2y − 1 = 0
3 We have, MT 2 = MC × MD
2g = 3 ⇒ g = ⇒ 2f = 2 ⇒ f = 1 ⇒ c = −1 …(i)
2
232 AP EAMCET Chapterwise Mathematics

MT = length of tangent from M (a , b) to the circle θ 4


⇒ tan =
= S1 2 1 2 + (3)2 − 2 ()
() 1 + 4(3) − 11
MT = a 2 + b 2 − k2 θ 4 θ 4
⇒ tan = ⇒ tan =
2 9 2 3
∴ From Eq. (i),
2  
4
MC × MD = ( a 2 + b 2 − k2)2 2tanθ/2  3
sinθ = =
MC × MD = a 2 + b 2 − k2 1 + tan2 θ/2 2
1 +  
4
Hence, option (b) is correct.  3
8 / 3 24
45. If one end of the diameter of sinθ = =
25 / 9 25
x + y − 2 x − 6 y − 15 = 0 is (4 , 1), then the
2 2

⇒ θ = sin−1  
24 24
co-ordinates of the other end is ∴ sinθ =
25  25
[23 Sep. 2020, Shift-I]
(a) (5, − 2 ) (b) (−2, 5) (c) (1, 3) (d) (−2, − 5) Hence, option (a) is correct.

Sol. (b) 47. If the angle between a pair of tangents drawn


Given equation of circle is, from a point P to the circle
x 2 + y 2 − 2x − 6 y − 15 = 0 x 2 + y 2 + 4 x − 6 y + 9 sin 2 α + 13 cos 2 α = 0 is
2α, then the equation of the locus of P is
Centre = (− g, − f ), c = (1, 3)
[20 April 2019, Shift-I]
Given one end of diameter P = (4, 1).
(a) x2 + y2 + 4 x − 6 y + 4 = 0
other end of diameter Q = (a , b)
(b) x2 + y2 + 4 x − 6 y − 9 = 0
∴Mid-point of diameter = centre of circle
 4 + a , 1 + b  = (1, 3) (c) x2 + y2 − 4 x + 6 y − 4 = 0
  (d) x2 + y2 + 4 x − 6 y + 9 = 0
 2 2 
Sol. (d)
4 +a 1+b
=1 =3 According to given information, on drawing the
2 2 figure.
4 +a =2 1+b =6
a = −2 b =5
∴Other end is Q = (−2, 5) (–2, 3)
α
C α
Hence, option (b) is correct. P(x1,y1)
r
46. The angle between the pair of tangents
drawn from (1, 3) to the circle A
x 2 + y 2 − 2 x + 4 y − 11 = 0 [23 Sep. 2020, Shift-I] Q tanα =
AC
PA
(a) sin−1   (b) sin−1  
24 7
 25   25  ⇒
4 + 9 − 9sin2 α − 13cos2 α
(c) cos −1   (d) tan−  
24 7 tanα =
 25   24  x12 + y12 + 4 x1 − 6 y1 + 9sin2 α + 13cos2 α
Sol. (a) 13sin2 α − 9sin2 α
Given Circle is, =
x12 + y12 + 4 x1 − 6 y1 + 9 + 4 cos2 α
x 2 + y 2 − 2x + 4 y − 11 = 0
P = (1, 3) 4sin2 α
=
r = 1 + 4 + 11 ⇒ r = 16 ⇒ r=4 x12 + y12 + 4 x1 − 6 y1 + 9 + 4 cos2 α

If θ is the angle, between the tangents, then sin2 α 4sin2 α


⇒ = 2
θ r cos α x1 + y1 + 4 x1 − 6 y1 + 9 + 4 cos2 α
2 2
tan =
2 S11 ⇒ x12 + y12 + 4 x1 − 6 y1 + 9 + 4 cos2 α = 4 cos2 α
Circle and System of Circles 233

⇒ x12 + y12 + 4 x1 − 6 y1 + 9 = 0 x 2 + y 2 − 10 x − 4 y + 21 = 0 orthogonally.


On taking locus of point P(x1 , y1), we get Then 5 g − 10 f + 3 c = [20 April 2019, Shift-I]
x 2 + y2 + 4 x − 6 y + 9 = 0 (a) 0 (b) 1 (c) 3 (d) 9
Hence, option (d) is correct. Sol. (d)
The equation of given circle
48. The equation of the circle whose radius is 3
and which touches internally the circle x 2 + y 2 + 2gx + 2 fy + c = 0 …(i)
x 2 + y 2 − 4 x − 6 y − 12 = 0 at the point (−1, − 1) having centre (− g, − f) lying on the line
2x + 3y − 7 = 0
is [20 April 2019, Shift-I]
so 2g + 3 f + 7 = 0 …(ii)
(a) 5 x2 + 5 y2 + 9 x − 6 y − 7 = 0
Since circle (i) cuts the given circles
(b) 5 x2 + 5 y2 − 8 x − 14 y − 32 = 0
x 2 + y 2 − 4 x − 6 y + 11 = 0
(c) 5 x2 + 5 y2 − 6 x + 8 y − 8 = 0
and x 2 + y 2 − 10 x − 4 y + 21 = 0
(d) 5 x2 + 5 y2 + 6 x − 8 y − 12 = 0 orthogonally
Sol. (b) so, 2g(−2) + 2 f (−3) = c + 11
Equation of given circle is ⇒ 4 g + 6 f + c + 11 = 0 …(iii)
x 2 + y 2 − 4 x − 6 y − 12 = 0 having centre C1 (2, 3) and 2g(−5) + 2 f (−2) = c + 21
and radius r1 = 4 + 9 + 12 = 5. ⇒ 10 g + 4 f + c + 21 = 0 …(iv)
From Eqs. (iii) and (iv), we get
Let the required circle having centre C2(h, k) and
radius is given as 3 touches the given circle at 6 g − 2 f + 10 = 0 ...(v)
A(−1, − 1). From Eq. (ii) and (v), we get
The point A(−1, − 1) divides the line joining the 11 f + 11 = 0 ⇒ f = − 1
centres C1 (2, 3) and C2(h, k) externally in 5 : 3 so so, g = − 2 ⇒c = 3
∴5 g − 10 f + 3c = 5(−2) − 10(−1) + 3(3)
 5h − 3(2) 5k − 3(3) 
(−1, − 1) =  ,  = − 10 + 10 + 9 = 9
 5− 3 5− 3 
Hence, option (d) is correct.
5h − 6 5k − 9 
⇒ (−1, − 1) =  ,  50. If the radical axis of the circles
 2 2 
x 2 + y 2 + 2 gx + 2 fy + c = 0 and
⇒ 5h − 6 = − 2 and 5k − 9 = − 2
2 x 2 + 2 y 2 + 3 x + 8 y + 2 c = 0 touches the circle
4 7
⇒ h = and k = x 2 + y 2 + 2 x + 2 y + 1 = 0 , then
5 5
so equation of required circle is (4 g − 3)( f − 2) = [20 April 2019, Shift-I]
2 2 (a) 0 (b) −1 (c) 1 (d) 2
 x − 4  +  y − 7  = (3)2
   
 5  5 Sol. (a)
8 x 16 14 y 49 The radical axis of the circles
⇒ x2 − + + y2 − + =9
5 25 5 25 x 2 + y 2 + 2gx + 2 fy + c = 0

⇒ x 2 + y2 −
8 x 14 y 65
− + =9 and 2x 2 + 2y 2 + 3x + 8 y + 2c = 0
5 5 25 is (4 g − 3) x + (4 f − 8) y = 0 …(i)
8 x 14 y 13
⇒ x 2 + y2 − − + =9 Since, the radical axis (i) touches the circle
5 5 5
x 2 + y 2 + 2x + 2y + 1 = 0, so
⇒ 5x 2 + 5y 2 − 8 x − 14 y − 32 = 0 − (4 g − 3) − (4 f − 8)
= 1 + 1 −1
Hence, option (b) is correct.
(4 g − 3)2 + (4 f − 8)2
49. Suppose that the circle ⇒(4 g − 3)2 + (4 f − 8)2 + 2(4 g − 3)(4 f − 8)
x 2 + y 2 + 2 gx + 2 fy + c = 0 has its centre on = (4 g − 3)2 + (4 f − 8)2
2 x + 3 y − 7 = 0 and cuts the circles ⇒ 8 (4 g − 3)( f − 2) = 0 ⇒ (4 g − 3)( f − 2) = 0
x 2 + y 2 − 4 x − 6 y + 11 = 0 and Hence, option (a) is correct.
234 AP EAMCET Chapterwise Mathematics

51. A is the centre of the circle Let required pole is P(x1 , y1), so equation of polar of
point (P(x1 , y1) with respect to given circle is T = 0.
x + y − 2 x − 4 y − 20 = 0 . If the tangents
2 2
⇒ xx1 + yy1 + 3(x + x1) + 4(y + y1) − 96 = 0
drawn at the points B(1, 7) and D(4 , − 2) on the
⇒(x1 + 3) x + (y1 + 4) y + (3x1 + 4 y1 − 96) = 0 …(i)
circle meet at the point C, then area of the
The line (i) represent the line 5x + 7 y − 78 = 0
quadrilateral ABCD (in square units) is
only.
[20 April 2019, Shift-II] x + 3 y1 + 4 3x1 + 4 y1 − 96
(a) 75 (b) 64 (c) 56 (d) 45 So, 1 = = = k (Let)
5 7 −78
Sol. (a) ⇒ x1 = 5k − 3, y1 = 7k − 4 and 3x1 + 4 y1 = 96 − 78k
Equation of the given circle is So, 3(5k − 3) + 4(7k − 4) = 96 − 78k
x 2 + y 2 − 2x − 4 y − 20 = 0 …(i) ⇒ 15k + 28k + 78k = 96 + 9 + 16
⇒ 121k = 121 ⇒ k = 1
Now, equation of tangent at the point B(1, 7) on
So, x1 = 2 and y1 = 3
the circle is
So, required point of concurrence of all conjugate
x + 7 y − (x + 1) − 2(y + 7) − 20 = 0 ⇒ 5y = 35
lines of the given line w.r.t. given circle is (2, 3).
⇒ y=7 …(ii)
Hence, option (d) is correct.
Similarly, equation of tangent at the point D(4, − 2)
on the circle is 53. The number of common tangents to the
4 x − 2y − (x + 4) − 2(y − 2) − 20 = 0 circles x 2 + y 2 + 4 x − 6 y − 12 = 0 and
⇒ 3x − 4 y = 20 …(iii) x 2 + y 2 − 8 x + 10 y + 5 = 0 is
Now, point of intersection of tangents (ii) and [20 April 2019, Shift-II]
(iii) is C(16, 7).
(a) 4 (b) 3 (c) 2 (d) 1
Now, area of required quadrilateral ABCD
Sol. (c)
B Equation of given circles
r √S1 S1 : x 2 + y 2 + 4 x − 6 y − 12 = 0

A
C and S2 : x 2 + y 2 − 8 x + 10 y + 5 = 0
(16, 7) Q Centre of S1 is C1 (−2, 3) and radius of S1 is r1 = 5
and centre of S2 is C2(4, − 5) and radius of S2 is
r2 = 6.
D
1 QC1 C2 = (4 + 2)2 + (−5 − 3)2 = 36 + 64 = 10
= 2 × Area of ∆ABC = 2 × r S1
2 and r1 + r2 = 5 + 6 = 11
[where r = radius of circle (i) Q r1 + r2 > c1 c 2
= 1 + 4 + 20 = 5 ∴There are only two common tangent to circle S1
and S2.
and S1 = 256 + 49 − 32 − 28 − 20
Hence, option (c) is correct.
= 225 = 15] = 5 × 15 = 75
54. If the two circles (x − 1)2 + (y − 3)2 = r 2 and
Hence, option (a) is correct.
x 2 + y 2 − 8 x + 2 y + 8 = 0 intersect at two
52. The point of concurrence of all conjugate
distinct points, then [20 April 2019, Shift-II]
lines of the line 5 x + 7 y − 78 = 0 with respect
(a) 2 < r < 8 (b) 1 < r < 9
to the circle x 2 + y 2 + 6 x + 8 y − 96 = 0 is
(c) r = 2 (d) r = 8
[20 April 2019, Shift-II]
(a) (−2, 3) (b) (3, − 2 ) (c) (3, 2 ) (d) (2, 3)
Sol. (a)
Equation of given circles
Sol. (d)
S1 : (x − 1)2 + (y − 3)2 = r 2
The all conjugate lines of the line 5x + 7 y − 78 = 0
with respect to circle x 2 + y 2 + 6 x + 8 y − 96 = 0 and S2 : (x − 4)2 + (y + 1)2 = 32
passes through the pole of the given line with If circles S1 and S2 intersect at two distinct points,
respect to given circle. then
Circle and System of Circles 235

|r1 − r2|< C1 C2 < (r1 + r2) So, coordinate of radical centre is (2, 1) and radius
So, |r − 3|< (4 − 1) + (−1 − 3) < r + 3
2 2 of required circle is equals to length of tangent
drawn from radical centre (2, 1) to any one of the
⇒ (r − 3) < 5 < r + 3 circle
⇒ r + 3 > 5and |r − 3|< 5 = 4+1+ 8− 7 = 6
⇒ r > 2 and r − 3 ∈ (−5, 5) So, equation of required circle is :
⇒ r ∈ (−2, 8) ⇒ 2 < r < 8 (x − 2)2 + (y − 1)2 = 6
Hence, option (a) is correct.
⇒ x 2 + y 2 − 4 x − 2y − 1 = 0.
55. If the circle x + y + 6 x − 2 y + k = 0 bisects
2 2
Hence, options (a) is correct.
the circumference of the circle
57. The power of the point B(− 1, 1) with respect to
x 2 + y 2 + 2 x − 6 y − 15 = 0 , then k =
the circle S ≡ x 2 + y 2 − 2 x − 4 y + 3 = 0 is p. If
[20 April 2019, Shift-II]
(a) 21 (b) −21 (c) −23 (d) 23 the length of the tangent drawn from B to
the circles S = 0 is t, then the point (2, 3) with
Sol. (c) respect to the circle S′ = 0 having centre at
Equation of given circles (p , t2) and passing through the origin.
S1 : x 2 + y 2 + 6 x − 2y + k = 0 [21 April 2019, Shift-I]
and S2 : x 2 + y 2 + 2x − 6 y − 15 = 0 (a) lies inside the circle S ′ = 0
Since, the circle S1 bisects the circumference of the (b) lies outside the circle S ′ = 0
circle S2, then the common of circles S1 and S2 will (c) lies on the circle S ′ = 0
passing through the centre of circle S2 (−1, 3). (d) is the centre of the circle S ′ = 0
Q Equation of common chord of circles S1 and S2 is Sol. (a)
4 x + 4 y + (k + 15) = 0 …(i) Given equation of circle
Q Chord (i) is passes through point (−1, 3,) so S ≡ x 2 + y 2 − 2x − 4 y + 3 = 0
− 4 + 12 + k + 15 = 0 ⇒ k = − 23
∴ p = (− 1)2 + (1)2 − 2(− 1) − 4(1) + 3
Hence, option (c) is correct.
=1 + 1 + 2− 4 + 3= 3
56. The equation of the circle which cuts the Q t= p ⇒ t= 3
circles x 2 + y 2 + 4 x − 7 = 0 ,
Now, circle whose centre is (p , t 2), i.e. (3, 3)
2 x 2 + 2 y 2 + 3 x + 5 y − 9 = 0, x 2 + y 2 + y = 0
(x − 3)2 + (y − 3)2 = r 2
orthogonally is [20 April 2019, Shift-II]
Since, this circle passes through (0, 0)
(a) x2 + y2 − 4 x − 2 y − 1 = 0
∴ (0 − 3)2 + (0 − 3)2 = r 2
(b) x2 + y2 − 4 x − 6 y − 3 = 0
⇒ r 2 = 9 + 9 = 18
(c) x2 + y2 − 4 x − 2 y − 3 = 0
(d) x2 + y2 − 2 x − 4 y − 1 = 0 So, circle S′ will be
(x − 3)2 + (y − 3)2 = 18
Sol. (a)
Now, point (2, 3) w.r.t. to circle
The centre of the circle which cuts the given
(x − 3)2 + (y − 3)2 = 18 is
circles having equation
S1 : x 2 + y 2 + 4 y − 7 = 0 = (2 − 3)2 + (3 − 3)2 − 18

S2 : 2x 2 + 2y 2 + 3x + 5y − 9 = 0 = 1 − 18 = − 17 < 0
So, point (2, 3) lies inside the circle S′ = 0
and S3 : x 2 + y 2 + y = 0
is radical centre of circles S1 , S2 and S3.
58. If tangents are drawn to the circle
Q Equation of radical axis of circles S1 and S2 is
x 2 + y 2 = 12 at the points where it intersects
x − y − 1 = 0. …(i) the circle x 2 + y 2 − 5 x + 3 y − 2 = 0 , then the
and equation of radical axis of circles S2 and S3 is coordinates of the point of intersection of
x + y − 3 = 0. …(ii) those tangents are [21 April 2019, Shift-I]
236 AP EAMCET Chapterwise Mathematics

(a)  − 6,  (b)  6,  and radius = 225 + 1 − 1 = 225 = 15


18 18
 5  5
and C2 : x + y − 14 x + 6 y + 33 = 0
2 2
 − 18  − 18 
(c)  − 6,  (d)  6,  ∴Centre (O′) = (7, − 3)
 5   5 
and radius = 49 + 9 − 33 = 25 = 5
Sol. (d)
Since, point T divides OO′ in 15 : 5 i.e., 3 : 1
Let (h, k) be the point of intersection of the
internally.
tangents. Then, the chord of contact of tangents is
21 − 15 − 9 + 1   3
the common chord of the circles x 2 + y 2 = 12 and ∴ T =  ,  =  , − 2

 4 4  2 
x 2 + y 2 − 5x + 3y − 2 = 0.
Also, point D divides OO′ in 15 : 5 i.e., 3 : 1
x2+y2=12 x2+y2–5 x+3 y–2=0 externally.
21 + 15 − 9 − 1 
D =  ,  = (18, − 5)
 2 2 
Now, centre of circle with TD as diameters of
mid-point of TD
18 + 3 / 2 − 2 − 5  39 − 7 
5x–3y–10=0 =  , = , 
 2 2   4 2 

The equation of the common chord is 60. If the circles x 2 + y 2 + 2λx + 2 = 0 and
5x − 3y − 10 = 0 …(i) x 2 + y 2 + 4 y + 2 = 0 touch each other, then
Also, the equation of the chord of contact is λ= [21 April 2019, Shift-I]
hx + ky − 12 = 0 … (ii) (a) ± 1 (b) ± 2 (c) ± 3 (d) ± 4
Eqs. (i) and (ii) represents the same line.
Therefore, Sol. (b)
h k − 12 − 18 The centres of the two circles are C1 (− λ, 0) and
= = ⇒ h = 6, k =
5 − 3 − 10 5 C2(0, − 2) and their radii are λ2 − 2 and 2. So,
 18 
Hence, the required point is  6, −  . the two circles will touch each other, if
 5 C1 C2 = sum of radii
59. If the point of intersection of the pair of the ⇒ λ2 + 4 = λ2 − 2 + 2
transverse common tangents and that of the
pair of direct common tangents drawn to the ⇒ λ + 4 = λ − 2 + 2 + 2 2 λ2 − 2
2 2

circles x 2 + y 2 − 14 x + 6 y + 33 = 0 and ⇒ 4 = 2 2 λ2 − 2 ⇒ 2 = λ2 − 2
x 2 + y 2 + 30 x − 2 y + 1 = 0 are T and D ⇒ 2 = λ2 − 2 ⇒ λ2 = 4 ⇒ λ = ± 2
respectively, then the centre of the circle
having TD as diameter is [21 April 2019, Shift-I] 61. The equation of the circle whose diameter is
− 7
(a)  , (b)  , 
39 39 7 the common chord of the circles

2 4   4 2 x 2 + y 2 + 2 x + 3 y + 1 = 0 and
− 7 39 − 7 
(c)  , (d)  ,
39 x 2 + y 2 + 4 x + 3 y + 2 = 0 is
 
 4 2  2 2 
[21 April 2019, Shift-I]
Sol. (c) (a) 2 x2 + 2 y2 + x + 3 y + 2 = 0
C1 (b) 2 x2 + 2 y2 + 2 x + 6 y + 1 = 0
C2
(c) 2 x2 + 2 y2 + 4 x − 3 y − 1 = 0
15
O O′ 5
t (d) x2 + y2 + 2 x + 6 y − 2 = 0
D
(–15, 1) (7, –3)
T Sol. (b)
The equation of the common chord of the circles
x 2 + y 2 + 2x + 3y + 1 = 0 and
Given, C1 : x 2 + y 2 + 30 x − 2y + 1 = 0 x 2 + y 2 + 4 x + 3y + 2 = 0 is given by
∴ centre (O) = (− 15, 1) 2x + 1 = 0 [using : S1 − S2 = 0]
Circle and System of Circles 237

The equation of a circle passing through the Since, given that pair of lines joining the origin
intersection of the given circles is and the points of intersection of the line and
(x 2 + y 2 + 2x + 3y + 1) curve are at right angles.
+ λ (x 2 + y 2 + 4 x + 3y + 2) = 0 ∴ θ=0
⇒ Coefficient of x 2 + Coefficient of y 2 = 0
⇒ x 2(1 + λ) + y 2(1 + λ) + (1 + 2λ)
2x + 3y(1 + λ) + 1 + 2λ = 0 ⇒ 1 − a − a2 + 1 − b − b2 = 0
 1 + 2λ  1 + 2λ ⇒ a2 + b2 + a + b − 2 = 0
⇒ x + y + 
2 2
 2x + 3y + = 0 …(i)
1+ λ  λ +1 ∴Locus of (a , b) is circle
Since, 2x + 1 = 0 is a diameter of this circle. x 2 + y2 + x + y − 2 = 0 …(i)
 2λ + 1 3 By comparing in Eq. (i) with
Therefore, its centre  − , −  lies on it
 λ+1 2 x 2 + y 2 + 2gx + 2 fy + c = 0, we get
 2λ + 1  1 1
⇒ − 2  +1= 0 g = , f = and c = − 2
 λ+1  2 2
2 2
⇒ − 4λ − 2 + λ + 1 = 0 ⇒ − 3λ − 1 = 0 ∴ Radius = g2 + f 2 − c = 1  + 1  + 2
   
1  2  2
λ=−
3 1 1 1 5
= + + 2= + 2=
1 4 4 2 2
On putting λ = − in Eq. (i), we get
3
63. If the lines x + 2 y − 5 = 0 and 2 x − 3 y + 4 = 0
1 − 2  1−
2
  lie along diameters of a circle of area is 9 π,
⇒ x + y + 
2 2 3  2x + 3y + 3 =0
1 − 1  1 then the equation of the circle is
− +1
 3 3 [21 April 2019, Shift-II]
 1/ 3 1/ 3 (a) x2 + y2 − 2 x − 4 y − 4 = 0
⇒ x + y + 
2 2
 2x + 3y + =0
 2/ 3 2/ 3 (b) x2 + y2 + 2 x − 4 y − 4 = 0
1 (c) x2 + y2 + 2 x + 4 y − 4 = 0
⇒ x 2 + y 2 + x + 3y + =0
2 (d) x2 + y2 − 2 x + 4 y − 4 = 0
⇒ 2x 2 + 2y 2 + 2x + 6 y + 1 = 0 Sol. (a)
62. If the pair of lines joining the origin and the Given equation of lines are
points of intersection of the line ax + by = 1 x + 2y − 5 = 0 …(i)
and the curve x 2 + y 2 − x − y − 1 = 0 are at and 2x − 3y + 4 = 0 …(ii)
right angles, then the locus of the point (a , b) Multiply Eq. (i) by 2 and subtract it from Eq. (ii),
we get
is a circle of radius [21 April 2019, Shift-II]
2x − 3y + 4 = 0
3 5 5
(a) 2 (b) (c) (d) 2x + 4 y − 10 = 0
2 2 2
− − +
Sol. (c) −7 y + 14 = 0
Given equation of line is ax + by = 1 and equation 14
of curve is x 2 + y 2 − x − y − 1 = 0. y=− =2
−7
By homogenising in both equations, we get Put the value of y = 2 in Eq. (i), we get
⇒ x 2 + y 2 − x(ax + by) − y(ax + by) − (ax + by)2 = 0 x + 2(2) − 5 = 0 ⇒ x + 4 − 5 = 0 ⇒ x = 1
⇒ x 2 + y 2 − ax 2 − bxy − axy − by 2 − (a 2 x 2 Hence, centre is (1, 2).
Given that, area of circle = 9π
+ b 2 y 2 + 2abxy) = 0
⇒ πR 2 = 9 π ⇒ R 2 = 9
⇒ x 2 + y 2 − ax 2 − bxy − axy − by 2 − a 2 x 2 − b 2 y 2
So, required equation of circle is
− 2abxy = 0
(x − 1)2 + (y − 2)2 = 9
⇒ x 2(1 − a − a 2) + xy(− a − b − 2ab) + y 2(1 − b − b 2) = 0
238 AP EAMCET Chapterwise Mathematics

⇒ x 2 + 1 + y 2 + 4 − 2x − 4 y = 9
⇒ x 2 + y 2 − 2x − 4 y − 4 = 0
5 5
64. Given that a > 2b > 0 and that the line C1 C2

y = mx − b 1 + m is a common tangent to
2

the circles x 2 + y 2 = b2 and (x − a)2 + y 2 = b2 .


4 x + 3y = 10
Then the positive value of m is
4
[21 April 2019, Shift-II] ∴Slope of tangent= −
2b b 3
(a) (b) 3
a − 2b a − 2b Slope of line perpendicular to tangent =
4
a2 − 4b 2 2b 3
(c) (d) ∴ m = tanθ =
2b a − 4b
2 2 4
3 4
Sol. (d) sinθ =
and cosθ =
5 5
Given equation of circles are x −1 y − 2
Now, = =±5
x 2 + y2 = b 2 …(i) 4 3
and (x − a) + y = b
2 2 2
…(ii) 5 5
 4  3
⇒ x =  ±5 × + 1 and y = ± 5 × + 2
Equation of common tangent
 5  5
y = mx − b 1 + m2 …(i) (given)
⇒ x = (±4 + 1) and y = (±3 + 2)
Common tangent equation for second circle ⇒ x = 5, −3 and y = 5, −1
(x − a)2 + y 2 = b 2 is y = m(x − a) + b 1 + m2 …(ii) So, C1 (5, 5) and C2(−3, −1).
By Eqs. (i) and (ii), we get ∴Equation of required circles are
mx − b 1 + m2 = mx − am + b 1 + m2 (x − 5)2 + (y − 5)2 = (5)2 (Q r = 5)
or (x + 3) + (y + 1) = (5)
2 2 2
⇒ − b 1 + m2 = − am + b 1 + m2
⇒ x + 25 − 10 x + y 2 + 25 − 10 y = 25
2

⇒ am = 2b 1 + m2 ⇒ a 2m2 = 4b 2(1 + m2)


or x 2 + 9 + 6 x + y 2 + 1 + 2y = 25
⇒ a 2m2 = 4b 2 + 4b 2m2
⇒ x 2 + y 2 − 10 x − 10 y + 25 = 0
⇒ a 2m2 − 4b 2m2 = 4b 2 ⇒ m2(a 2 − 4b 2) = 4b 2
or x 2 + y 2 + 6 x + 2y − 15 = 0
4b 2 2b
⇒ m = 22
⇒m = Hence, option (c) is correct.
(a − 4b 2) a 2 − 4b 2
66. If the angle between the circles
65. Two circles each of radius 5 units touch each x 2 + y 2 + 4 x − 5 = 0 and
other at (1, 2) and 4 x + 3 y = 10 is their π
x 2 + y 2 + 2 λy − 4 = 0 is , then λ =
common tangent. The equation of that circle 3
[21 April 2019, Shift-II]
among the two given circles, such that some
(a) ± 5 (b) ±2 (c) ± 3 (d) ± 6
portion of it lies in every quadrant is
[21 April 2019, Shift-II] Sol. (a)
(a) x2 + y2 + 6 x + 2 y + 15 = 0 Given circles are
(b) x + y + 2 x + 6 y − 15 = 0
2 2 x 2 + y2 + 4 x − 5 = 0 …(i)
(c) x + y + 6 x + 2 y − 15 = 0
2 2
and x + y + 2λy − 4 = 0
2 2
…(ii)
(d) x2 + y2 − 6 x + 2 y − 15 = 0 Here, g = 2, f = 0, c = − 5
Sol. (c) g′ = 0, f ′ = λ, c′ = − 4
2gg′ + 2 ff ′ − c − c ′
Given, equation of common tangent is Q cosθ =
4 x + 3y = 10 2 g + f 2 − c g′2 + f ′2 − c ′
2
Circle and System of Circles 239

π 2(2)(0) + 2(0) + 5 + 4 Q θ = π  ⇒ 3λ (2λ − 3) − 2(2λ − 3) = 0


⇒ cos =  
3 2 4 + 0 + 5 0 + λ2 + 4  3 3 2
⇒ (2λ − 3)(3λ − 2) = 0 ⇒ λ = ,
1 9 2 3
⇒ = 3
2 2 9 λ2 + 4 Hence, required equation, when λ =
2
1 3
⇒ = ⇒ x 1 +  + y 1 +  + x  6 × − 4
2 3 2 3  3
2 2 λ2 + 4  2  2  2 
⇒ λ2 + 4 = 9 ⇒ λ2 = 5 ⇒ λ = ± + y 4 × − 6 − 12 × − 12 = 0
5 3 3
 2  2
67. The equation of a circle passing through the 5 2 5 2
points of intersection of the circles ⇒ x + y + x(5) + y(0) − 30 = 0
2 2
x 2 + y 2 − 4 x − 6 y − 12 = 0 ,
⇒ 5x 2 + 5y 2 + 10 x − 60 = 0
x 2 + y 2 + 6 x + 4 y − 12 = 0 and having radius
⇒ x 2 + y 2 + 2x − 12 = 0
13 is [21 April 2019, Shift-II]
2
(a) x2 + y2 − 2 x − 12 = 0 and required equation when λ =
3
(b) x2 + y2 + 2 y − 12 = 0
⇒ x 1 +  + y 1 +  + x  6 × − 4
2 2 2 2  2
(c) x2 + y2 − 2 y − 13 = 0  3  3  3 
(d) x2 + y2 + 2 x − 12 = 0
+ y 4 × − 6 − 12 × − 12 = 0
2 2
 3  3
Sol. (d)
2  5 2  5  8 
The required circle is ⇒ x   + y   + x(0) + y − 6 − 20 = 0
 3  3 3 
(x 2 + y 2 − 4 x − 6 y − 12) + λ(x 2 + y 2 + 6 x
+ 4 y − 12) = 0 5x 2 5y 2 10 y
⇒ + − − 20 = 0
[using S1 + λS2 = 0] 3 3 3
⇒ x 2(1 + λ) + y 2(1 + λ) + x(6λ − 4) + y(4λ − 6) ⇒ 5x 2 + 5y 2 − 10 y − 60 = 0

− 12λ − 12 = 0 ⇒ x 2 + y 2 − 2y − 12 = 0
x(6λ − 4) y(4λ − 6) 12(λ + 1) Hence, option (d) is correct.
⇒ x 2 + y2 + + − =0
1+ λ 1+ λ (λ + 1) 68. If the equation of the circle having its centre
x(6λ − 4) y(4λ − 6) in the second quadrant touches the
⇒ x 2 + y2 + + − 12 = 0
(1 + λ) (1 + λ) x y
coordinate axes and also the line + = 1 is
Given that, r = 13 5 12
3λ − 2 2λ − 3 x 2 + y 2 + 2 λx − 2 λy + λ 2 = 0 , then λ =
Here, g = , f = and c = −12
λ +1 λ +1 [22 April 2019, Shift-I]
Therefore, 13 = g + f − c
2 2 (a) 3 (b) 10 (c) 15 (d) −2

(3λ − 2)2 (2λ − 3)2 Sol. (b)


⇒ 13 = + + 12 Given, equation of circle is
(λ + 1)2 (λ + 1)2
x 2 + y 2 + 2λx − 2λy + λ 2 = 0 …(i)
⇒ 13(λ + 1)2 = (3λ − 2)2 + (2λ − 3)2 + 12(λ + 1)2
Here, centre is (−λ , λ) and radius = λ
⇒ 13(λ2 + 2λ + 1) Since, circle (i) touches12x + 5y − 60 = 0
= 9λ2 + 4 − 12λ + 4λ2 + 9 − 12λ + 12λ2 + 12 + 24λ ∴ r =d
⇒ 13λ2 + 13 + 26λ = 25λ2 + 25 |−7λ − 60|
⇒ λ=
144 + 25
⇒ 12λ2 + 12 − 26λ = 0
⇒ 13λ = | − 7λ − 60|
⇒ 12λ2 − 26λ + 12 = 0
⇒ 7λ + 60 = 13λ or −7λ − 60 = 13λ
⇒ 6λ2 − 13λ + 6 = 0 ⇒ 6λ = 60 or 20λ = − 60
⇒ 6λ2 − 9λ − 4λ + 6 = 0 ⇒ λ =10 or λ = − 3 ⇒ λ = 10 (Q λ ≠ −3)
240 AP EAMCET Chapterwise Mathematics

69. The equation of a circle passing through the Here, C1 = (2, 1), C2 = (3, 2)
point (2, 8), touching the lines and r1 = 4 + 1 − 1 = 4 = 2
4 x − 3 y − 24 = 0 and 4 x + 3 y − 42 = 0 and and r2 = 9 + 4 − 4 = 9 =3
having the x coordinate of its centre less than
or equal to 8 is [22 April 2019, Shift-I] Now, C1 C2 = (3 − 2)2 + (2 − 1)2 = 1 + 1 = 2
(a) x + y + 2 x − 8 y − 8 = 0
2 2
and r1 + r2 = 2 + 3 = 5
(b) x2 + y2 − 4 x − 6 y − 12 = 0 ∴ C1 C2 < r1 + r2
(c) x2 + y2 + 4 x − 10 y + 4 = 0 So, circles intersect at two distinct point.
(d) x2 + y2 − 6 x − 4 y − 24 = 0 let P(x , y) be the point of intersection of tangents
6 − 6 4 − 3
Sol. (b) ∴ P(x , y) =  , 
 −1 −1 
(2,8)
Hence, P(x , y) = (0, − 1)
4x–3y–24=0 4x+3y–42=0
71. The circle S =0 cuts the circle
x 2 + y 2 − 4 x + 2 y − 7 = 0 orthogonally. If
(h, k) (2, 3) is the centre of the circle S = 0, then its
radius is [22 April, 2019 Shift-I]
(a) 2 (b) 1 (c) 3 (d) 4
By the property of distance, Sol. (a)
4h − 3k − 24 = 4h + 3k − 42 Given that, S = 0 circle cuts the circle
 5   5  x 2 + y 2 − 4 x + 2y − 7 = 0 orthogonally and centre
= (h − 2)2 + (k − 8)2 of circle S = 0 is (2, 3).
⇒ 4h − 3k − 24 = ± (4h + 3k − 42) As we know that, if two circles intersect orthogonally,
either 4h − 3k − 24 = 4h + 3k − 42 Then, 2gg′ + 2 ff ′ = c + c ′
⇒ 6k = 18 ⇒ k = 3 Here,(g, f) = (2, 3) and (g′ , f ′) = (2, − 1)
or 4h − 3k − 24 = − 4h − 3k + 42 c′ = − 7
66 33 ∴ 2(− 2) (2) + 2(3)( − 1) = c − 7
⇒ 8h = 66 ⇒ h = = >8
8 4 ⇒ 8 − 6 = c − 7 ⇒ 2= c − 7 ⇒ c = 9
Now, (4h − 3k − 24)2 = 25[(h − 2)2 + (k − 8)2] Now, required radius = (2)2 + (3)2 − 9
By solving this, we get, h = 2 = 4+ 9−9= 4=2
∴ Centre is (2, 3).
Now, required circle is 72. The equation of the circle which cuts the
(x − 2)2 + (y − 3)2 = 25 circles
⇒ x 2 + 4 − 4 x + y 2 + 9 − 6 y = 25 S1 ≡≡ x 2 + y 2 − 4 = 0
⇒ x 2 + y 2 − 4 x − 6 x = 12 S2 ≡≡ x 2 + y 2 − 6 x − 8 y +10 = 0
70. The point of intersection of the common S3 ≡≡ x 2 + y 2 + 2 x − 4 y − 2 = 0
tangents drawn to the circles
at the extremities of diameters of these
x 2 + y 2 − 4 x − 2 y +1 = 0 and
circles is [22 April 2019, Shift-I]
x 2 + y 2 − 6 x − 4 y + 4 = 0 , is [22 April 2019, Shift-I] (a) x2 + y2 − 4 x − 6 y − 4 = 0
(a)  ,  (b)  ,  (d)  ,  (b) x2 + y2 + 4 x − 4 = 0
5 3 6 1 12 7
(c) (0, − 1)
2 2  5 5  5 5 (c) x2 + y2 = 25
Sol. (c) (d) x2 + y2 + x + y + 1 = 0
Given equation of circles are Sol. (a)
x 2 + y 2 − 4 x − 2y + 1 = 0 …(i) Let the equation of required circle is
and x 2 + y2 − 6 x − 4 y + 4 = 0 …(ii) S ≡ x 2 + y 2 + 2gx + 2 fy + c = 0
Circle and System of Circles 241

and equation of given circles α y


tan = …(i)
S1 ≡ x 2 + y 2 − 4 = 0 2 x+ 5
S2 ≡ x 2 + y 2 − 6 x − 8 y + 10 = 0 Equation of line BQ
5sinβ
S3 = x + y + 2x − 4 y − 2 = 0
2 2
y−0= (x − 35)
5 (cosβ − 1)
Q Circle S = 0 cuts the circles S1 = 0, S2 = 0 and β β 5− x
S3 = 0 at the extermities of the diameters, so y = − cot (x − 5) ⇒ tan =
common chord of S = 0 and S1 = 0 passes through 2 2 y
the centre of the circle S1 = 0, so c = − 4 π
Now, α −β = …(ii)
Similarly (2g + 6) x + (2 f + 8) y − 14 = 0 passes 2
through (3, 4), so 6 g + 8 f + 36 = 0 α β π  α β π
⇒ − = ⇒ tan −  = tan
⇒ 3g + 4 f + 18 = 0 …(i) 2 2 4  2 2 4
and (2g − 2) x + (2 f + 4) y − 2 = 0, passes through α β
tan − tan
(−1, 2), so ⇒ 2 2 = tan π
−2g + 4 f + 8 = 0 α β 4
…(ii) 1 + tan ⋅ tan
From Eqs. (i) and (ii), we get (g, f) = (−2, − 3) 2 2
y 5− x y 2 − [25 − x 2]
So equation of required circle is −
x 2 + y2 − 4 x − 6 y − 4 = 0 x+ 5 y y(x + 5)
⇒ =1 ⇒ =1
y 5− x 5− x
1+ ⋅ 1+
73. A circle of radius 5 units passes through x+ 5 y 5+ x
A(− 5 , 0) and B(5 , 0). If P(5 cos α , 5 sin α), x 2 + y 2 − 25 5 + x + 5 − x
Q(5 cos β , 5 sin β) are two points on this circle ⇒ =
y(x + 5) (5 + x)
π
such that α − β = , then the locus of the ⇒ x 2 + y 2 − 25 = 10 y ⇒ x 2 + y 2 − 10 y − 25 = 0
2
point of intersection of the lines AP and BQ is 74. Two circles which cut each other
[22 April 2019, Shift-II] orthogonally, pass through the points (a , 0)
(a) x2 + y2 − 10 x − 25 = 0 and (− a , 0). If both of them touch the line
(b) x2 + y2 + 10 x − 25 = 0 y = mx + c, then c 2 = [22 April 2019, Shift-II]
(c) x2 + y2 + 10 y − 25 = 0 (a) a2 + m2 (b) a2 (1 + m)2
(d) x + y − 10 y − 25 = 0
2 2
(c) a (1 + m )
2 2
(d) a2 (1 + 2 m2 )
Sol. (d) Sol. (b)
Given point A(− 5, 0) and B(5, 0). Equation of family of circles passes through points
And P(5cosα , 5sinα) and Q(5cosβ , 5sinβ) (a , 0) and (−a , 0) is
(x − a)(x + a) + (y − 0)(y − 0) + λ(y − 0) = 0
B(0, q) ⇒ x 2 + y 2 + λy − a 2 = 0 …(i)
Q The two circles cut each other orthogonally then,
λ λ
P(h, k) 2 (0)(0) + 2    = a 2 + a 2
 2  2
⇒ λ2 = 4a 2 ⇒ λ = ± 2a.
So, equation of circles are
A(p, 0)
x 2 + y 2 + 2ay − a 2 = 0
and x 2 + y 2 − 2ay − a 2 = 0
Equation of line AP
5sinα It is given, that above both the circles touches line
y−0= (x + 5) y = mx + c,
5(cosα + 1) |− a − c| |a − c|
2sinα /2 cosα /2 So, = 2a =
⇒ y= (x + 5) 1 + m2 1 + m2
2cos2 α /2
α a 2 + c 2 + 2ac a 2 + c 2 − 2ac
⇒ y = tan (x + 5) ⇒ = 2a 2
=
2 1 + m2 1 + m2
242 AP EAMCET Chapterwise Mathematics

⇒ a 2 + c 2 + 2ac = 2a 2(1 + m2) = a 2 + c 2 − 2ac 76. The centre of the circle which passes through
On adding, we get the vertices of the triangle formed by the lines
2a 2 + 2c 2 = 4a 2(1 + m2) y = 0 , y = x and 2 x + 3 y = 10 , is
[23 April 2019, Shift-I]
⇒ a 2 + c 2 = 2a 2 + 2a 2m2
(a)  − , −  (b)  , − 
5 1 5 1
⇒ c 2 = a 2 + 2a 2m2  2 2 2 2
⇒ c 2 = a 2(1 + 2m2).
(c)  − , −  (d)  , 
1 1 5 1
 2 2 2 2
75. For the system of circles given by
(x 2 + y 2 + 2 gx) + λ (x 2 + y 2 + 2 fy + k) = 0 , Sol. (b)
where g ≠ 0, f ≠ 0 and λ is a parameter, if the Given equation of sides are
line joining the point circles of the system y=0 …(i)
k y= x …(ii)
subtends a right angle at the origin, then 2 = 2x + 3y = 10 …(iii)
f
A
[22 April 2019, Shift-II]
1
(a) − 1 (b) 1 (c) 2 (d)
2 1 3
Sol. (c)
Given, system of family of circles is B C
2
(x 2 + y 2 + 2gx) + λ(x 2 + y 2 + 2 fy + k) = 0
Solving Eqs. (i) and (iii) to get vertex ‘A‘
⇒ (1 + λ) x 2 + (1 + λ) y 2 + 2gx + 2 fλy + λk = 0
∴ A = (5, 0)
g fλ λk Solving (i) and (ii) to get vertex ‘B’
⇒ x 2 + y2 + 2 +2 y+ =0
1+ λ 1+ λ 1+ λ B =(0, 0)
For point circle, radius should be zero. Solving Eqs. (ii) and (iii) to get vertex ‘C’
g2 f 2λ2 λk C = (2, 2)
∴ + = Let equation of circle be x 2 + y 2 + 2gx + 2 fy + c = 0
(1 + λ) 2
(1 + λ)2 1 + λ
…(iv)
⇒ g2 + f 2λ2 = k(λ + λ2) Since, Eq. (iv) passes through B(0, 0)
⇒ ( f 2 − k)λ2 − kλ + g2 = 0 …(i) ⇒ C=0
Let the quadratic equation (i) has root λ1 and λ 2, Since, Eq. (iv) passes through A(5, 0)
then the coordinates of point circles are 25 + 0 + 10 g + 0 + 0 = 0 ⇒ g = − 5 / 2
 Eq. (iv) passes through C(2, 2)
g f λ1   g f λ2 
c1  − ,−  and c1  − ,− . ∴ 4 + 4 + 4g + 4 f = 0 ⇒ g + f + 2 = 0
 1 + λ1 1 + λ1   1 + λ2 1 + λ2 
5 1
Q Lines OC1 and OC2 are perpendicular to each so, − + f + 2= 0 ⇒ f =
2 2
f λ2 f λ1  5 −1 
∴Centre of circle = (− g, − f) =  , 
1 + λ 2 1 + λ1 2 2 
× = −1 ⇒ f 2(λ1 λ 2) = − g2 …(ii)
g g
∴ Hence, answer is (b).
1 + λ2 1 + λ2
77. The distance between the polar of P(2, 3) with
Q product of roots of equation (i), we get
respect to the circle x 2 + y 2 − 2 x − 2 y + 1 = 0
g2
λ1 λ 2 = 2 …(iii) and the polar of the inverse point of P with
f −k respect to the same circle is [23 April 2019, Shift-I]
From (ii) and (iii), we get 4 12
(a) 0 (b) (c) (d) 1
 g2  5 5
f 2 2  = −g ⇒ f = − f + k
2 2 2
 f − k Sol. (b)
k Equation of polar of point P(2, 3) with respect to
⇒ 2f = k ⇒ 2 = 2
2
f the circle x 2 + y 2 − 2x − 2y + 1 = 0 is
Circle and System of Circles 243

T = 0 ⇒ 2x + 3y − (x + 2) − (y + 3) + 1 = 0 5= r + 2
⇒ x + 2y − 4 = 0 …(i) ∴ r=3
End equation of line joining of points P(2, 3) and ∴ Required equation of circle having centre at
centre of the given circle C(1, 1) is (6, 5) and r = 3 is
2 (x − 6)2 + (y − 5)2 = 32
y − 1 = (x − 1) ⇒ 2x − y − 1 = 0 …(ii)
1 x 2 + y 2 − 12x − 10 y + 52 = 0
QInverse point of P with respect to the given circle is ∴ Hence, answer is (d).
point of intersection of polar Eq. (i) and line CP, Eq.
79. If the circles (x + a)2 + (y + b)2 = a 2 and
(ii), so coordinate of the inverse point is Q  ,  .
6 7
 5 5 (x + c)2 + (y + d)2 = d 2 cut orthogonally, then
Now, equation of polar of point Q  ,  with b(b − 2 d) =
6 7 [23 April 2019, Shift-I]
 5 5
(a) c(c − 2 a) (b) c(2 a − c )
respect to given circle is (c) d (2c − a) (d) a(a − 2c )
x + y −  x +  −  y +  + 1 = 0
6 7 6 7
 Sol. (b)
5 5 5  5
Given equation of circles are
⇒ x + 2y − 8 = 0 …(iii)
(x + a)2 + (y + b)2 = a 2
∴Required distance between polars (i) and (iii) is
4 4 ⇒ x 2 + y 2 + 2ax + 2by + b 2 = 0 …(i)
=
1+ 4 5 (x + c) + (y + d) = d
2 2 2

Hence, option (b) is correct. ⇒ x 2 + y 2 + 2cx + 2dy + c 2 = 0 …(ii)


78. A circle S of radius 2 units lies in the first From Eq. (i) g1 = a ; f1 = b, c1 = b 2

quadrant and touches both the coordinate


From Eq. (ii) g2 = c ; f2 = d ; c 2 = c 2
axes. The equation of the circle with centre at
(6, 5) and touching the circle S externally is If circles (i) and (ii) are orthogonal,
[23 April 2019, Shift-I] ⇒ 2[g1 g2 + f1 f2] = c1 + c 2
(a) x2 + y2 − 12 x − 10 y + 12 = 0 ⇒ 2(ac + bd) = b 2 + c 2
(b) x2 + y2 − 12 x − 10 y − 20 = 0 ⇒ 2ac + 2bd = b 2 + c 2
(c) x + y − 12 x − 10 y + 25 = 0
2 2
⇒ 2ac − c 2 = b 2 − 2bd
(d) x2 + y2 − 12 x − 10 y + 52 = 0
⇒ c(2a − c) = b(b − 2d)
Sol. (d) ∴ b(b − 2d) = c(2a − c)
From figure ∴Hence, answer is (b).

Y 80. The equation of the circle having the


common chord of the circles x 2 + y 2 − 8 x = 0
and x 2 + y 2 − 9 = 0 as its diameter is
r [23 April 2019, Shift-I]
C2 (6,5) (a) x + y − 72 x − 207 = 0
2 2
2
(b) x2 + y2 + 72 x + 207 = 0
C1 (2,2) (c) 32 x2 + 32 y2 − 72 x − 207 = 0
(d) 32 x2 + 32 y2 + 72 x − 207 = 0

X Sol. (c)
O
The equation of circle passes through the
intersection of circles x 2 + y 2 − 8 x = 0 and
Centre of given circle (C1) = (2, 2)
x 2 + y 2 − 9 = 0 is
radius = 2units
Centre of required circle (C2) = (6, 5) (x 2 + y 2 − 8 x) + λ (x 2 + y 2 − 9) = 0
C1 C2 = r + 2 (Q from figure) ⇒ (1 + λ) x 2 + (1 + λ) y 2 − 8 x − 9λ = 0
244 AP EAMCET Chapterwise Mathematics

8 λ Sol. (c)
⇒ x 2 + y2 − x−9 = 0 having centre
1+ λ 1+ λ Since, circle touches both coordinates axes, then
 4  centre will be (h, h) and radius = h
C , 0
1 + λ  3h − 4h − 12 − h − 12
∴ =h ⇒ =h
Now, equation of common chord of given circles is
(3) + (− 4)
2 2 5
8 x = 9, and the common chord is the diameter of
 4  ⇒ − h − 12 = ± 5h ⇒ − 12 = ± 5h + h
the circle having centre C  , 0 , so
1 + λ  ⇒ − 12 = 6h or − 12 = − 4h
32 = 9 + 9λ ⇒ h = − 2 or 3 ⇒ h = 3 [Q h > 0]
23 32 ∴Equation of circle will be
⇒ λ= and (1 + λ) =
9 9 (x − 3)2 + (y − 3)2 = 32
∴ Equation of required circle is ⇒ x 2 − 6 x + 9 + y2 − 6 y + 9 = 9
32 2 32 2
x + y − 8 x − 23 = 0 ⇒ x 2 + y2 − 6 x − 6 y + 9 = 0
9 9
⇒ 32x 2 + 32y 2 − 72x − 207 = 0 83. The pole of the straight line 9 x + y − 28 = 0
Hence, option (c) is correct. with respect to the circle
81. If the lengths of the tangents drawn from P 2 x 2 + 2 y 2 − 3 x + 5 y − 7 = 0 is
to the circles x 2 + y 2 − 2 x + 4 y − 20 = 0 and [22 April 2018, Shift-I]
x + y − 2 x − 8 y + 1 = 0 are in the ratio 2 : 1,
2 2 (a) (3, 1) (b) (3,−1) (c) (−31)
, (d) (4,−8)
then the locus P is [22 April 2018, Shift-I] Sol. (b)
(a) x2 + y2 + 2 x + 12 y + 8 = 0 Let (h, k) be the pole of the line 9 x + y − 28 = 0
(b) x2 + y2 − 2 x + 12 y + 8 = 0 with respect to the circle
(c) x2 + y2 + 2 x − 12 y + 8 = 0 3 5 7
x 2 + y 2 − x + y − = 0. Then, the equation of
(d) x2 + y2 − 2 x − 12 y + 8 = 0 2 2 2
polar is
Sol. (d) 3 5 7
hx + ky − (x + h) + (y + k) − = 0
We know that, length of tangent drawn from 4 4 2
(x1 , y1) to the circle x 2 + y 2 + 2gx + 2 fy + c = 0 is  3  5 3
⇒ x h −  + yk +  − h + k − = 0
5 7
 4  4 4 4 2
x12 + y12 + 2gx1 + 2 fy1 + c
Let P(h, k) ⇒ x(4h − 3) + y(4k + 5) − 3h + 5k − 14 = 0
∴According to the question, This equation and 9 x + y − 28 = 0 represent the
h2 + k2 − 2h + 4k − 20 2 same line.
=
4h − 3 4k + 5 − 3h + 5k − 14
h2 + k2 − 2h − 8k + 1 1 ∴ = = = λ (say)
9 1 − 28
⇒ h2 + k2 − 2h + 4k − 20 = 4(h2 + k2 − 2h − 8k + 1)
3 + 9λ λ−5
⇒ h2 + k2 − 2h + 4k − 20 = 4h2 + 4k2 − 8h − 32k + 4 ⇒h = ,k= , − 3h + 5k − 14 = − 28λ
4 4
⇒ 3h2 + 3k2 − 6h − 36k + 24 = 0
3 + 9λ   λ − 5 − 14 = − 28λ
⇒ h2 + k2 − 2h − 12k + 8 = 0 ⇒ − 3  + 5 
 4   4 
∴Locus of point P is x + y − 2x − 12y + 8 = 0.
2 2
⇒ − 9 − 27λ + 5λ − 25 − 56 = − 112λ
82. The equation of a circle touching the ⇒ − 22λ − 90 = − 112λ ⇒ 90λ = 90 ⇒ λ = 1
coordinate axes and the line 3 x − 4 y = 12 is Hence, the pole of the given line is (3, − 1).
[22 April 2018, Shift-I] 84. The point of intersection of the direct
(a) x2 + y2 + 6 x + 6 y + 9 = 0 common tangents drawn to the circles
(b) x2 + y2 + 6 x + 6 y − 9 = 0 (x + 11)2 + (y − 2)2 = 225 and
(c) x2 + y2 − 6 x − 6 y + 9 = 0 (x − 11)2 + (y + 2)2 = 25 is [22 April 2018, Shift-I]
(d) x2 + y2 − 6 x − 6 y − 9 = 0
Circle and System of Circles 245

−11  9 + 17 − 26
(a)  ,1 (b) (−22,4) = =0
 2  6 17
(c)  ,−1 θ = 90°
11
(d) (22,−4)
2 
(5)2 + (3)2 − [ (− 2 + 2)2 + (7 − 0)2 ]2
(C) cosθ =
Sol. (d) 2× 5× 3
The direct common tangents to two circles meet [Q r1 = 5, r2 = 3, c1 = (− 2, 7), c 2 = (− 2, 0)]
on the line of centres and divide it externally in 25 + 9 − 49 − 15 − 1
the ratio of the radii centres of the two circles are = = =
(− 11, 2) and (11, − 2) and their radii are 15 and 5. 30 30 2
∴Point of intersection So, θ = 120° or 60°.
 11 × 15 − (− 11) × 5 − 2 × 15 − 2 × 5
= ,  86. If the radical axis of the circles
 15 − 5 15 − 5  x 2 + y 2 + 2 gx + 2 fy + c = 0 and
165 + 55 − 30 − 10 
=  ,  = (22, − 4). 2 x 2 + 2 y 2 + 3 x + 8 y + 2 c = 0 touches the circle
 10 10 
x 2 + y 2 + 2 x + 2 y + 1 = 0 , then
85. In List-I, a pair of circles is given in A, B, C [22 April 2018, Shift-I]
and in List-II, angle between those pair of 3 3
circles is given. Match the items from List-I (a) g = or f = 2 (b) g ≠ , f = 2
4 4
to List-II. [22 April 2018, Shift-I] 3 2
(c) g = or f =/ 2 (d) g = or f = 1
List-I List-II 4 5

A ( x − 2 ) + y = 2 ( x − 2 ) + ( y − 1) = 1 I 90°
2 2 2 2 Sol. (a)
Let point P (a , b)
B x 2 + y2 − 6 x − 6 y + 9 = 0 II 135°
S1 (a , b) = S2(a , b)
x 2 + y2 − 4 x + 4 y − 9 = 0
3
C x 2 + y 2 + 4 x − 14 y + 28 = 0 III 60° ⇒ a + b + 2ga + 2 fb + c = a 2 + b 2 +
2 2
a + 4b + c = 0
2
x 2 + y2 + 4 x − 5 = 0
a  2g −  + b(2 f − 4) = 0
3
IV 30°  2
this is the locus of radical axis.
The correct matching is
So, x  2g −  + y(2 f − 4) = 0 is radical axis of
3
A B C A B C
 2
(a) I II III (b) II I III
(c) III I IV (d) IV III I given circles.
This touched the x 2 + y 2 + 2x + 2y + 1 = 0
Sol. (b)
We know that, angle between two circles is given So, radius = 12 + 12 − 1 = 1 and centre = (− 1, − 1)
by
So, radius of circle = distance between centre and
r 2 + r22 − d 2 touching point.
cosθ = 1 , where r1 and r2 are radius and
2r1 r2
 3 − 2g + (2 f − 4)
d is distance between centres.  
2 
1=
( 2)2 + (1)2 − [ (2 − 2)2 + (1 − 0)2 ]2 2
(A) cosθ =  3 − 2g + (2 f − 4)2
2× 2 ×1  
2 
[∴r1 = 2, r2 = 1, c1 = (2, 0), c 2 = (2, 1)]
Taking square both sides,
2+ 1 −1 1
= =
2  − 2g (2 f − 4) = 0
3
2 2 2 2 
∴ θ = 45° or 135°
3
(3)2 + ( 17)2 − [ (3 − 2)2 + (3 + 2)2 ]2 So, − 2g = 0 or 2 f − 4 = 0
(B) cosθ = 2
2 × 3 × 17 3 3
2g = or 2 f = 4 ⇒ g = or f = 2
[Q r1 = 3, r2 = 17, c1 = (3, 3), c 2 = (2, − 2)] 2 4
246 AP EAMCET Chapterwise Mathematics

P (x1, y1)
87. From a point P (0 , b) two tangents are drawn to
the circle x 2 + y 2 = 16 and these two tangents 30º 30º
intersect X -axis is two points A and B. If the
5y=5x+K
area of ∆PAB is minimum, then the equation
of its circumcircle is [22 April 2018, Shift-II] A
B
(a) x2 + y2 = 16 2 (b) x2 + y2 = 64 10
(c) x2 + y2 = 32 (d) x2 + y2 = 4 2 C (6, 8)

Sol. (c)
Equation of pair of tangents from point (0, b)
drawn to the circle x 2 + y 2 = 16 is ⇒ x12 + y12 − 12x1 − 16 y1 = 300
(x 2 + y 2 − 16) (b 2 − 16) = (by − 16)2 …(i) ⇒ (x1 − 6)2 + (y1 − 8)2 = 400 …(i)
For point A and B , put y = 0, then we are getting Since, line AB, 5y = 5x + k, is a chord of contact of
(x 2 − 16) (b 2 − 16) = 162 circle with respect to point P(x1 , y1), so,
xx1 + yy1 − 6(x + x1) − 8(y + y1) = 0
16b 2
⇒ x2 = x (x1 − 6) + y(y1 − 8) − (6 x1 + 8 y1) = 0
b 2 − 16 x1 − 6 y1 − 8 6 x1 + 8 y1
∴ = = = λ(let)
4b −5 5 K
⇒ x=±
b − 16
2 x1 = 6 − 5λ and y1 = 8 + 5λ …(ii)
4b From Eqs. (i) and (ii), we are getting
So, x-coordinate of A and B is ± 25λ2 + 25λ2 = 400
b 2− 16
6 x + 8 y1
⇒ λ = ± 2 2⇒λ = 1
1  8b  4b 2 k
Now, area of ∆PAB = ∆ = b = 6 x + 8 y1 36 − 30λ + 64 + 40λ
2  b 2 − 16  b 2 − 16 ⇒ K = 1 =
λ λ
d∆ 100
Now, for minimum area =0 ⇒ k= + 10 ⇒ k = 10 ± 25 2 ⇒ k = 5(2 ± 5 2)
db λ
b
⇒ b 2 − 16 (8b) = 4b 2 89. If a circle S with radius 5 touches the circle
b 2 − 16
x 2 + y 2 − 6 x − 4 y − 12 = 0 at (− 1, − 1), then
⇒ 8(b 2 − 16) = 4b 2 ⇒ b 2 = 32 ⇒ b = ± 4 2 the length of the tangent from the centre of
16 2 the circle S to the given circle is
So, x − coordinate of A and B is ± =±4 2
4 [22 April 2018, Shift-II]
So, P(0, ± 4 2), A(4 2, 0) and B(−4 2, 0) (a) 5 3 (b) 65 (c) 10 (d) 3 11
Q ∆PAB is a right angled triangle, so equation of Sol. (a)
circumcircle of ∆PAB isx 2 + y 2 = 32. Given circle is
x 2 + y 2 − 6 x − 4 y − 12 = 0
88. If the angle between the tangents drawn to
the circle x 2 + y 2 − 12 x − 16 y = 0 at the ⇒ (x − 3)2 + (y − 2)2 = 25
points where the line 5 y = 5 x + k cut the According to the question,
circle is 60°, then the value of k is A
[22 April 2018, Shift-II] 5
(a) 5 + 2 (b) 5(2 ± 5 2 )
B
(c) 2 ± 5 2 (d) 5 ± 5 2 5 5 C (3, 2)
Sol. (b)
Let the point P(x1 , y1), then
Q ∆CAB is right angle triangle, so
10 1
= ∴ AB2 = BC 2 − AC 2 = 100 − 25 = 75
x 12 + y 12 − 12x 1 − 16y 1 3 ⇒ AB = 5 3
Circle and System of Circles 247

90. If a circle S passing through the point (3, 4) 92. If P(x1 , y1) is a point such that the length of
cuts the circle x 2 + y 2 = 36 orthogonally, the tangents from it to the circles
then the locus of the centre of S is
x 2 + y 2 − 4 x − 6 y − 12 = 0 and
[22 April 2018, Shift-II]
(a) x2 + y2 − 6 x − 8 y + 11 = 0 x 2 + y 2 + 6 x + 18 y + 26 = 0 are in the ratio
(b)6 x + 8 y − 61 = 0 2 : 3, then the locus of P is [23 April 2018, Shift-I]
(c) x2 + y2 − 8 x − 6 y + 11 = 0 (a) x2 + y2 + 24 x − 36 y + 62 = 0
(d) 6 x + 8 y + 11 = 0 (b) x2 + y2 − 24 x + 36 y + 62 = 0
(c) x2 + y2 − 24 x − 54 y − 88 = 0
Sol. (b)
(d) x2 + y2 + 24 x + 36 y + 62 = 0
Let the circle is x 2+ y 2 + 2gx + 2 fy + c = 0, having
centre (− g, − f), since it passes through the point Sol. (*)
(3, 4) Given equations of circle are
So, 9 + 16 + 6 g + 8 f + c = 0 …(i) x 2 + y 2 − 4 x − 6 y − 12 = 0
And circle is intersecting the other circle and x 2 + y 2 + 6 x + 18 y + 26 = 0
x 2 + y 2 = 36 orthogonally, so
Tangents from P(x1 , y1) to the circles are in the
2g (0) + 2 f (0) = c − 36 ratio of 2 : 3.
⇒ C = 36 …(ii) x12 + y12 − 4 x1 − 6 y1 − 12 2
So, =
From Eqs. (i) and (ii) x12 + y12 + 6 x1 + 18 y1 + 26 3
−6 g − 8 f = 61,
x12 + y12 − 4 x1 − 6 y1 − 12 4
Now, on taking locus of point (− g, − f), we are ⇒ =
getting 6 x + 8 y − 61 = 0. x12 + y12 + 6 x1 + 18 y1 + 26 9

91. The line x − 2 = 0 cuts the circle ⇒ 5x12 + 5y12 − 60 x1 − 126 y1 − 212 = 0

x 2 + y 2 − 8 x − 2 y + 8 = 0 at A and B. The Then, the locus of P(x1 , y1) is


5x 2 + 5y 2 − 60 x − 126 y − 212 = 0
equation of the circle passing through the
points A and B and having least radius is (*) No option is correct.
[22 April 2018, Shift-II] 93. If the lines 2 x + y + 12 = 0, kx − 3 y − 10 = 0
(a) x2 + y2 − 4 x + 2 y − 1 = 0 are conjugate with respect to the circle
(b) x2 + y2 − 4 x − 2 y = 0 x 2 + y 2 − 4 x + 3 y − 1 = 0 , then k =
(c) x2 + y2 − 4 x − 2 y + 1 = 0 [23 April 2018, Shift-I]
(d) x2 + y2 − 4 x + 4 y = 0 (a) 4 (b) − 9 (c) − 3 (d) − 5
Sol. (b) Sol. (a)
Equation of circles passes through the point of Lines 2x + y + 12 = 0, kx − 3y − 10 = 0 are
intersection of line x − 2 = 0 and the circle conjugate with respect to given circle.
x 2+ y 2 − 8 x − 2y + 8 = 0, is ∴Pole of line 2x + y + 12 = 0 lie on the line
(x 2 + y 2 − 8 x − 2y + 8) + λ(x − 2) = 0 kx − 3y − 10 = 0
Equation of polar of pole (x , y) to circle
⇒ x 2 + y 2 + (λ − 8) x − 2y + (8 − 2λ) = 0 …(i)
x 2 + y 2 − 4 x + 3y − 1 = 0 is
For minimum radius, it is necessary that centre of
3
circle Eq. (i) lies on the line x − 2 = 0 , so xx1 + yy1 − 2 (x + x1) + (y + y1) − 1 = 0
λ − 8 2
−   − 2= 0
⇒ x(x1 − 2) + y  y1 +  − 2x1 + y1 − 1 = 0
 2  3 3
 2 2
⇒ λ − 8= −4
it is same as 2x + y + 12 = 0
⇒ λ=4
3 3
y1 + − 2x1 + y1 − 1
So, equation of required circle is x1 − 2 2 2
So, = =
x 2 + y 2− 4 x − 2y = 0 2 1 12
248 AP EAMCET Chapterwise Mathematics

⇒ x1 − 2 = 2y1 + 3 and 12 y1 +18 = − 2x1 +


3
y1 − 1 and x 2 + y 2 + kx + 6 y − 59 = 0
2 Centre of first circle is (6, 3) and radius
⇒ x1 − 2y1 = 5 …(i) r1 = 36 − 9 − 4 = 2
21
⇒ 2x1 + y1 + 19 = 0 …(ii)
Centre of second circle is  , − 3 and radius
k
2
[From Eqs. (i) and (ii)] 2 

2x1 +
21
y1 + 19 = 0 k2 k2
r= + 9 + 59 = + 68
2 4 4
2x1 − 4 y1 − 10 = 0 Now, distance between centres is
− + +
2
d=  k − 6 + 62
 21 + 4 y = − 29  
  1 2 
 2 
k2
  y = − 29 ⇒ y = − 2
29 d= − 6k + 72
  1 4
 2 1

r12 + r22 − d 2
put in Eq. (i) Now, cosθ =
2r1 r2
x1 + 4 = 5 ⇒ x1 = 1
Point (x1 , y1) lie of kx − 3y −10 = 0 r12 + r22 − d 2
⇒ cos 45° =
⇒ 1 − 3(− 2) − 10 = 0
k() 2r1 r2
⇒ k+ 6−4=0 ⇒ k=4 k2 k2 k2
⇒ 2⋅ 2 + 68 = 4 + + 68 − + 6k − 72
94. The length of the transverse common tangent 4 4 4
of the circles x 2 + y 2 − 2 x + 4 y + 4 = 0 and k2
⇒ 2 2 + 68 = 6k
x + y + 4 x − 2 y + 1 = 0 is [23 April 2018, Shift-I]
2 2 4
k2 36k2 17k2
(a) 3 (b) 17 (c) 15 (d) 3 ⇒ + 68 = ⇒ = 68
4 8 4
Sol. (d)
⇒ k2 = 16 ⇒ k2 = ± 4
S1 : x 2 + y 2 − 2x + 4 y + 4 = 0
Hence, k=−4
Centre, C1 (1, − 2) and r1 = 1
and S2 : x 2 + y 2 + 4 x − 2y + 1 = 0 96. If the lengths of the tangents drawn from a
Centre C2(− 2, 1) and r2 = 2
point P to the three circles x 2 + y 2 − 4 = 0 ,
Distance between centres, d is x 2 + y 2 − 2 x + 3 y = 0 and
d = (1 + 2)2 + (− 2 − 1)2 x 2 + y 2 + 7 y − 18 = 0 are equal, then the
coordinates of P are [23 April 2018, Shift-I]
d = 18 = 3 2
(a) (2, 5) (b) (3, 4)
Q d > r1 + r2 (c) (4, 3) (d) (5, 2)
∴S1 and S2 are not intersecting each other.
Sol. (d)
The length of transversal common tangent is
Radical centre is the locus of point P from which
L= d 2 − (r1 + r2)2 = (3 2)2 − 9 = 9 equal length of tangent can be drawn to circle.
L = 3units So, S1 − S2 = 0
95. If the angle between the circles ⇒(x 2 + y 2 − 4) − (x 2 + y 2 − 2x + 3y) = 0
x 2 + y 2 − 12 x − 6 y + 41 = 0 and ⇒ 2x −3y − 4 = 0 … (i)
x + y + kx + 6 y − 59 = 0 is 45°, then a value
2 2 and S1 − S3 = 0
of k is [23 April 2018, Shift-I]
⇒(x 2 + y 2 − 4) − (x 2 + y 2 + 7 x − 18) = 0
(a) 0 (b) − 4 (c) − 3 (d) − 1 ⇒ − 7 y + 14 = 0
Sol. (b) ⇒ y=2 … (ii)
Given circles, [From Eqs. (i) and (ii)]
2x − 3(2) − 4 = 0
x 2 + y 2 − 12x − 6 y + 41 = 0
Circle and System of Circles 249

2x − 6 − 4 = 0 99. The equation of the circle which passes


⇒ 2x = 10 ⇒ x = 5 through the point (3 , 2) bisects the
So, radical centre P is (5, 2). circumference of the circle x 2 + y 2 = 15 and
97. If a circle touches the lines 3 x − 4 y − 10 = 0 cuts the circle x 2 + y 2 + 4 x + 6 y + 3 = 0
and 3 x − 4 y + 30 = 0 and its centre lies on orthogonally is [23 April 2018, Shift-II]
the line x + 2 y = 0, then the equation of the (a) x2 + y2 + 6 x + 8 y − 43 = 0
circle is [23 April 2018, Shift-II]
(b) x2 + y2 + 6 x − 8 y − 15 = 0
(a) x2 + y2 + 4 x − 2 y − 11 = 0
(c) x2 + y2 − 6 x + 8 y − 11 = 0
(b) x2 + y2 + 2 x − 4 y − 11 = 0
(d) x2 + y2 − 6 x − 8 y + 21 = 0
(c) x2 + y2 − 4 x + 2 y − 11 = 0
(d) x2 + y2 + 2 x − y − 11 = 0 Sol. (b)
Let the equation of required circle is
Sol. (a) x 2 + y 2 + 2gx + 2 fy + c = 0 … (i)
Distance between parallel lines 3x − 4 y − 10 = 0
Since, circle (i) passes through point (3, 2), so
and 3x − 4 y + 30 = 0 is the length of diametre of
1 40 9 + 4 + 6g + 4 f + c = 0
required circle, so radius = =4
2 9 + 16 ⇒ 6 g + 4 f + c + 13 = 0 … (ii)
Since, circle (i) bisects the circumference of the
and mid-point of intersection of lines
circle x 2 + y 2 = 15, so the common chord passes
3x − 4 y − 10 = 0, x + 2y = 0 and 3x − 4 y + 30 = 0,
x + 2y = 0, is the centre of required circle, so through the centre of the circle x 2 + y 2 = 15 .
2 − 6 − 1 + 3 So, c + 15 = 0 ⇒ c = − 15 … (iii)
centre is  ,  = (− 2, 1), then equation
 2 2  Since, circle Eq. (i) cuts the circle
of required circle is, x 2 + y 2 + 4 x − 2y − 11 = 0. x 2 + y2 + 4 x + 6 y + 3 = 0
Orthogonally, so
98. If the line 4 x + 4 y − 11 = 0 intersects the 4g + 6 f = c + 3 … (iv)
circle x + y − 4 x − 6 y + 4 = 0 at A and B,
2 2 From Eqs. (ii), (iii) and (iv)
then the point of intersection of the tangents g = 3, f = − 4, c = − 15
drawn at A, B is [23 April 2018, Shift-II] So, required equation of circle is
(a) (−1, 2 ) (b) (−1, − 2 ) (c) (2, 1) (d) (−2, − 1) x 2 + y 2 + 6 x − 8 y − 15 = 0.
Sol. (d) 100. x 2 + y 2 + 2 x + 4 y − 20 = 0 and
Let required point of intersection is (x1 , y1), the x 2 + y 2 + 6 x − 8 y + 10 = 0 are the given
equation of chord of contact with respect to given circles. Which one of the following is correct?
circle x 2 + y 2 − 4 x − 6 y + 4 = 0 is [23 April 2018, Shift-II]
xx1 + yy1 − 2(x + x1) − 3(y + y1) + 4 = 0 (a) They intersect orthogonally and will have two
⇒(x1 − 2) x + (y1 − 3) y + (4 − 3y1 − 2x1) = 0 … (i) common tangents. The length of their common
Let the Eq. (i) represent the line 5 3
chord is
AB, 4 x + 4 y − 11 = 0 itself, then 2
x1 − 2 y1 − 3 4 − 3y1 − 2x1 (b) They intersect at right angles and will have two
= = = K (let) common tangents. The length of their common
4 4 − 11 chord is 2
Then, x1 = 4K + 2 (c) They do not intersect orthogonally and will have
three common tangents. The length of their direct
y1 = 4K + 3
common tangent is 5
And 2x1 + 3y1 − 4 = 11K … (ii) (d) They touch each other internally and will have only
From Eq. (ii) one common tangent
8K + 4 + 12K + 9 − 4 = 11K Sol. (a)
9K + 9 = 0 ⇒ K = − 1, The equation of given two circles are
So, (x1 , y1) = (− 2, − 1). x 2 + y 2 + 2x + 4 y − 20 = 0 … (i)
250 AP EAMCET Chapterwise Mathematics

and x 2 + y 2 + 6 x − 8 y + 10 = 0 …(ii) ⇒ 5x 2 + 5y 2 + 30 x + 40 y − 60 = 0
Q 2g1 g2 + 2 f1 f2 = 6 − 16 = − 10 = c1 + c 2 ⇒ x 2 + y 2 + 6 x + 8 y − 12 = 0
So, circles intersects each other orthogonally and
will have two common tangents. 102. The equation of the pair of lines joining the
Now, equation of common chord is
origin to the points of intersection of two
2x − 6 y + 15 = 0 … (iii)
circles x 2 + y 2 − 4 x + 8 y + 5 = 0 and
So, length of common chord is x 2 + y 2 + 2 x + 4 y − 3 = 0 is
(− 2 + 12 + 15)2 [24 April 2018, Shift-I]
2 25 −
4 + 36 (a) 13 x2 + 6 xy − 28 y2 = 0
(b) xy − 28 y2 = 0
625
= 2 25 − (c) ( x + 4) ( x − 5) = 0
40
(d) 13 x2 + 68 xy − 28 y2 = 0
1000 − 625 3
=2 =5 .
40 2 Sol. (d)
Let S1 : x 2 + y2 − 4 x + 8 y + 5 = 0 ...(i)
101. The equation of the circle passing through
the points of intersection of the circles and S2 : x 2 + y 2 + 2x + 4 y − 3 = 0 ...(ii)
x 2 + y 2 + 4 x + 6 y − 12 = 0 and Then, equation of common chord in given by
x + y − 6 x − 4 y − 12 = 0 and cutting the
2 2 S1 − S2 = 0

circle x 2 + y 2 − 4 x + 4 y + 8 = 0 orthogonally ⇒ (x 2 + y 2 − 4 x + 8 y + 5)
is [23 April 2018, Shift-II] −(x 2 + y 2 + 2x + 4 y − 3) = 0
(a) x2 + y2 + 6 x + 8 y + 12 = 0 ⇒ −6 x + 4 y + 8 = 0
(b) x2 + y2 + 8 x + 6 y − 12 = 0 ⇒ −3x + 2y + 4 = 0
(c) x2 + y2 + 6 x + 8 y − 12 = 0 ⇒ 3x − 2y = 4
(d) x2 + y2 − 6 x − 8 y − 12 = 0 x y
⇒ + =1 ...(iii)
4 / 3 −2
Sol. (c)
Now, the required equation of pair of lines is
Equation of the circle passing through the points
given by homogenization of Eqs. (i) and (iii) or
of intersection of the circles
Eqs. (ii) and (iii)
x 2 + y 2 + 4 x + 6 y − 12 = 0 … (i)
On homogenization of Eqs. (i) and (iii), we get
and x 2 + y 2 − 6 x − 4 y − 12 = 0 … (ii)  x y  x y
x 2 + y2 − 4 x  +  + 8 y + 
is (x 2 + y 2 + 4 x + 6 y − 12)  4 / 3 −2  4 / 3 −2
2
+ λ(x 2 + y 2 − 6 x − 4 y − 12) = 0  x y
+5  +  =0
4 − 6λ 6 − 4λ  4 / 3 − 2
⇒ x 2 + y2 + x+ − 12 = 0 …(iii)
1+ λ 1+ λ
⇒ x 2 + y 2 − 4 x  x −  + 8 y x + 
3 y 3 y
Since, the another circle 4 2 4 −2
2
x 2 + y 2 − 4 x + 2y + 8 = 0
+5 
… (iv) 3x y 
−  =0
cuts the circle (iii) orthogonally, then  4 2
⇒ x 2 + y 2 − 3x 2 + 2xy + 6 xy − 4 y 2
 4 − 6λ   6 − 4λ 
− 2  +   = 8 − 12 = − 4 9
1+ λ  1+ λ  y 2 3xy 
+5  x 2 + −  =0
 16 4 4 
⇒ − 8 + 12λ + 6 − 4λ = − 4 − 4λ
1 45 2 5 2 15xy
⇒ 12λ = − 2 ⇒ λ = − ⇒ −2x 2 − 3y 2 + 8 xy + x + y − =0
6 16 4 4
So, required equation of circle is ⇒ −32x 2 − 48 y 2 + 128 xy + 45x 2 + 20 y 2 − 60 xy = 0
6(x 2 + y 2 + 4 x + 6 y − 12) − (x 2 + y 2 − 6 x − 4 y − 12) = 0 ⇒ 13x 2 − 28 y 2 + 68 xy = 0
Circle and System of Circles 251

103. If the circle x 2 + y 2 + 2 gx + 2 fy + c = 0 (c > 0)


touches both the coordinate axes and lies in
the third quadrant, then the length of the O A P (2, –14)
chord intercepted by the circle on the line
x + y + c = 0 is [24 April 2018, Shift-I]
(a) 2c (b) c Shortest distance of the point P(2, − 14) to the
c circle is AP = d
(c) c (d)
2 AP = OP − OA
Sol. (a) (OA = R)
Given equation of circle OP = (− 3 − 2)2 + (− 2 + 14)2
x 2 + y 2 + 2gx + 2 fy + c = 0 (c > 0) = 25 + 144 = 169 = 13
Coordinate of centre = (− g, − f) ⇒ AP = 13 − 5 = 8 = d
radius = g2 + f 2 − c Length of tangent from the point P(2, − 14) is
= (2)2 + (− 14)2 + 6 ⋅ 2 + 4(− 14) − 12
A (–√c, 0) = 4 + 196 + 12 − 56 − 12 = 144 = 12 = l
So, required d + l = 8 + 12 = 20 = 2 5.

B (0, –√c) 105. The number of common tangents to the


(–√c, –√c)
circles x 2 + y 2 − 4 x − 2 y + k = 0 and
x+y+√c=0
x 2 + y 2 − 6 x − 4 y + l = 0 , having radii 2 and 3
respectively, is [24 April 2018, Shift-I]
Circle touch both the axes, so (a) 4 (b) 2 (c) 3 (d) 1
g2 = f 2 = c ⇒ g = ± c ⇒ f = ± c
Sol. (b)
So, coordinate of centre lies in third quadrant so, Given circle are,
centre is (− c , − c)
x 2 + y 2 − 4 x − 2y + k = 0
equation of given line x + y + c =0
∴ Centre c1 (2, 1) and r1 = 2
So, line pass through the point of contact to axes
x 2 + y2 − 6 x − 4 y + l = 0
of the circle.
Centre c 2(3, 2) and r2 = 3
Hence, intercept AB = (− c − 0)2 + (0 + c)2
= c + c = 2c c1 c 2 = (2 − 3)2 + (1 − 2)2 = 1 + 1 = 2

104. If the shortest distance from (2, − 14) to the c1 c 2 < r1 + r2


So, circles have two common tangents.
circle x 2 + y 2 + 6 x + 4 y −12 = 0 is d and the
length of the tangent drawn from the same 106. The centre of the circle which intersects the
point to the circle is l, then d + l = circle x 2 + y 2 − 2 x − 2 y − 2 = 0 orthogonally
[24 April 2018, Shift-I] and passes through the point (2, 0) and
touches the X -axis is [24 April 2018, Shift-I]
(a) 13 (b) 2 5
(c) 12 (d) 5 (a) (4, 1) (b) (−1, 2 ) (c) (1, 4) (d) (2, − 1)

Sol. (b) Sol. (d)


Given circle, x + y + 6 x + 4 y − 12 = 0
2 2 Let equation of circle,
Here, centre is O(− 3, − 2) x 2 + y 2 + 2gx + 2 fy + c = 0 … (i)
and radius = 9 + 4 + 12 = 25 = 5 Whose centre is = (− g, − f)
252 AP EAMCET Chapterwise Mathematics

Circle passes through the point (2, 0), so this Sol. (b)
point satisfy the circle. Given equations of circles are
4 + 0 + 4g + 0 + c = 0
c1 ≡ x 2 + y 2 + 2gx + 2 fy + c = 0
⇒ 4 + 4g + c = 0 … (ii)
Centre O(− g, − f)
Now, circle Eq. (i) touches the X-axis
radius r1 = g2 + f 2 − c
So, g2 = c … (iii)
From Eqs. (ii) and (iii), we get c 2 ≡ x 2 + y 2 + 2gx + 2 fy + c sin2 α + (g2 + f 2) cos2 α
4 + 4 g + g2 = 0
Centre O(− g, − f)
(g + 2)2 = 0 ⇒ g = − 2 ⇒ c = 4
radius
Now, circle Eq. (i) intersect the circle r2 = g2 + f 2 − c sin2 α − g2 cos2 α − f 2 cos2 α
x 2 + y 2 − 2x − 2y − 2 = 0 orthogonally
r2 = g2(1 − cos2 α) + f 2(1 − cos2 α) − c sin2 α
So, condition of orthogonality
2g1 g2 + 2 f1 f2 = c1 + c 2 = g2 + f 2 − c ⋅ sinα
2(− 2) (− 1) + 2 f ⋅ (− 1) = 4 − 2 as 0 < α < π /2 ⇒ 0 < sinα < 1 ⇒ r2 < r1
4 − 2f = 2 Circles c1 and c 2 are concentric circles
4 − 2 = 2f ⇒ f = 1
Now, r2 = g2 + f 2 − c sinα ⇒ r2 = r1 sinα
Hence, coordinate of centre is (2, − 1).
107. If from any point on the circle P
x 2 + y 2 + 2 gx +2 fy + c = 0 , tangents are A α
drawn to the circle r2 r1
x 2 + y 2 + 2 gx + 2 fy + c sin 2 α + (g 2 + f 2) O
B
 π
cos 2 α = 0 , 0 < α <  , then the angle
 2
between those tangents is
r2
[24 April 2018, Shift-I] = sinα ⇒ ∠APO = α
π π r1
(a) (b) (c) 2α (d) α
4 3 ⇒ ∠APB = 2∠APO = 2α.
20
Conic Sections
1. The length of the latus rectum of the parabola (x − 1)2 (y − 2)2
⇒ 2
+ 2
=1
169 {(x − 1) + (y − 3) } = (5 x − 12 y + 17) is
2 2 2
 1   1 
   
[17 Sep. 2020, Shift-I]  2  3
14 12 28 56 h k
(a) (b) (c) (d) ⇒ Centre (1 , 2)
13 13 13 13
b2 1/ 3 1
Sol. (c) e = 1− = 1− =
a2 1/ 2 3
Given parabola,
1 1 1
169[(x − 1)2 + (y − 3)2] = (5x − 12y + 17)2 a= ,b= ⇒ ae =
2
2 3 6
5x − 12y + 17 
⇒ (x − 1)2 + (y − 3)2 =   Foci (h ± ae , k) ≡ (1 ±
1
, 2)
 13  6
⇒ (SP = PM)
3. The value of ‘k’ so that the line y = 2 x + k
Here, focus is S(1, 3)
and directrix (5x − 12y + 17) = 0 may touch the ellipse 3 x 2 + 5 y 2 = 15 is
∴ Distance of focus from directrix [17 Sep. 2020, Shift-I]
5 − 36 + 17 14 (a) ± 23 (b) ± 13 (c) ± 33 (d) ± 32
⇒ 2a = ⇒ 2a =
25 + 144 13 Sol. (a)
∴ Latusrectum = 4a =
28 x 2 y2
For + = 1 and y = mx + c is tangent
13 a2 b2
2. Foci of the ellipse Condition of tangency is (c 2 = a 2m2 + b 2)
2 x 2 + 3 y 2 − 4 x − 12 y +13 = 0 are ………… Here,
x 2 y2
+ = 1 and y = 2x + k is a tangent
[17 Sep. 2020, Shift-I] 5 3
⇒ k2 = 5 × 4 + 3 ⇒ k2 = 23 ⇒ k = ± 23
(a)  1 + , 2  and  1 − , 2 
1 1
 6   6  4. Tangents are drawn to the hyperbola
(b)  + 1, 2  and  − 1, 2  x 2 − 9 y 2 = 9 from point (3, 2). Then, the area
1 1
 6   6 
of the triangle formed by the tangents and

(c)  2, 1 +
1   1  the chord of contact is ……… sq units.
 and  2, 1 − 
 6  6 [17 Sep. 2020, Shift-I]
(d)  2, + 1 and  2, − 1
1 1 (a) 10 (b) 6 (c) 12 (d) 8
 6   6 
Sol. (d)
Sol. (a) x 2 y2
Given hyperbola − =1
Given ellipse 9 1
2x 2 + 3y 2 − 4 x − 12y + 13 = 0
So, y = mx ± 9m2 − 1 represent tangents which
⇒ 2(x − 1)2 + 3(y − 2)2 = 1 passes through (3, 2)
254 AP EAMCET Chapterwise Mathematics

⇒ m=
5 b1 / 3 y + a1 / 3 x + (a 2b 2)1 / 3 = 0
12 Here, 4a = 32 ⇒ a = 8
∴ Tangents are
4b = 256, b = 64
12y = 5x + 9 and x = 3
xx yy So tangent is,
Chord of contact will be 21 − 21 = 1 (64)1 / 3 y + (8)1 / 3 x + (82 × 642)1 / 3 = 0
a b

3x 2y
− = 1 ⇒ x − 6y = 3 ⇒ 4 y + 2x + 64 = 0
9 1
7. If the latus rectum of a hyperbola through
The vertices of the triangle formed with tangents
one focus subtends an angle 60° at the other
and chord will be (3, 2), (3, 0) and  −5, − 
4
 3
focus, then its eccentricity is ........
[17 Sep. 2020, Shift-II]
3 2 1
1 (a) 2 (b) 6 (c) 3 (d) 5
So, area of ∆ = 3 0 1 = 8 unit 2
2 Sol. (c)
− 5 − 4 /3 1 Given, latus rectum of one hyperbola subtends an
angle of 60° at other follows,
5. If x + 5 = 0 is the directrix and (− 3, 0) the
A
vertex of a parabola, then equation of this
parabola is ........ [17 Sep. 2020, Shift-II]
(a) y2 = 8( x + 3) (b) y2 = 8( x − 3) b 2 /a
30º
(c) x2 = 8( y + 3) (d) x2 = 8( y − 3) 60º C
B F2 (ae, 0)
Sol. (a)
Given parabola is,

–x=–5 F1 (–ae, 0)
From ∆ABC,
b2 / a 1 b2
2 units F (–1, 0) tan 30° = ⇒ = 2
2ae 3 2a e
(–3, 0) 2 units ∴ Required equation of directrix isx = a
25
x= ⇒ 8 x − 25 = 0
8
8. The equation of the tangent to the parabola
So, its equation is, y 2 = 12 x , which makes an angle 30 ° with the
y 2 = 4a(x − (−3) positive direction of X -axis is given by
Where a = 2units. x − 3 y + 9 = 0, then its points of contact is
∴ y 2 = 8 (x + 3) [18 Sep. 2020, Shift-I]
(a) (− 9, − 6 3 ) (b) (9, − 6 3 )
6. The common tangent to the parabola (c) (− 9, 6 3 ) (d) (9, 6 3 )
y 2 = 32 x and x 2 = 256 y will be Sol. (d)
[17 Sep. 2020, Shift-II] Let the point of tangency of tangent
(a) 2 x + 4 y + 64 = 0 (b) x + 2 y − 32 = 0 x − 3 y + 9 = 0 to the parabola y 2 = 12x is (x1 y1).
(c) 2 x + 4 y + 32 = 0 (d) 4 x + 2 y + 64 = 0 Since equation of tangent to the parabola y 2 = 12x
Sol. (a) at point (x1 , y1) is yy1 = 6(x + x1)
Given parabolas are, 6 x − y1 y + 6 x1 = 0, which represent the tangent
y 2 = 32x and x 2 = 256 y x − 3y + 9 = 0, so on comparing, we get
6 − y1 6x
We use a standard result to find equation of = = 1 ⇒ (x1 , y1) = (9,6 3)
common tangent. Equation of tangent common to 1 − 3 9
y 2 = 4ax and x 2 = 4by is, Therefore, point of contact is (9,6 3).
Conic Sections 255

So, slope of tangent is  − 


9. In an ellipse, two vertices are (5, 0) and 4
(0 , − 4). Then the equation of the ellipse is  3
[18 Sep. 2020, Shift-I] Therefore equation of tangent is
2 2 2 2 4 4 4
(a)
x
+
y
=1 (b)
x
+
y
=1 y=− x+ ⇒ y= − x−3
16 25 25 16 3 −4 / 3 3
x 2
y 2
⇒ 4 x + 3y + 9 = 0.
(c) + =1 (d) x2 + y2 = 41
5 4 12. The equation of the ellipse with its focus at
Sol. (b) (6 , 2) centre at (1, 2) and which passes through
Equation of ellipse having vertices (5, 0) and the point (4 , 6) is [18 Sep. 2020, Shift-II]
x 2 y2 ( x − 1)2 ( y − 2 )2
(0, − 4) is + =1 (a) + =1
25 16 25 16
( x − 1)2 ( y − 2 )2
10. The lengths of the sides of the rectangle of (b) + =1
greatest area that can be inscribed in the 25 20
( x − 1)2 ( y − 1)2
ellipse x 2 + 4 y 2 = 64 are [18 Sep. 2020, Shift-I] (c) + =1
45 16
(a) 6 2, 4 2 (b) 8 2, 4 2 ( x − 1)2 ( y − 2 )2
(c) 8 2, 8 2 (d) 16 2, 4 2 (d) + =1
45 20
Sol. (b) Sol. (d)
Equation of given ellipse is
Given, Focus S = (6, 2)
x2 y2
x 2 + 4 y 2 = 64 ⇒ + =1 Centre C = (1, 2) = (h, k) (say)
64 16
Point P = (4, 6)
Let the vertex P (8 cosθ, 4sin θ) of the rectangle
Required Equation of ellipse is
y
(x − 1)2 (y − 2)2
+ =1 … (i)
P (8 cos θ, 4 sin θ) a2 b2
Q
Since, Eq. (i) passes through P (4, 6)
(4 − 1)2 (6 − 2)2
x + =1
O a2 b2
x +4y2=64
2 9 16
+ =1 … (ii)
R S a2 b2
Since, Focus = (6, 2)
PQRS having greatest area. (h + ae , k) = (6, 2)
∴ Area = A = 4 (8 cosθ) (4sinθ) = 64 sin 2θ ∴ h + ae = 6, k = 2
For greatest area sin 2θ =1 , so A = 64 sq. units and 1 + ae = 6 ⇒ ae = 5 ⇒ a 2e 2 = 25
π
θ= . b 2 = a 2 (1 − e 2) ⇒ b 2 = a 2 − a 2e 2
4
Therefore point P(4 2, 2 2) b 2 = a 2 − 25 ⇒ a 2 = b 2 + 25 … (iii)
2
So, length of the sides are 8 2 and 4 2. Put a value in Eq. (ii),
9 16
+ 2 =1
11. The equation of the tangent to the parabola b + 25 b
2

y 2 = 16 x , which is perpendicular to the line 9b 2 + 16(b 2 + 25) = b 2 (b 2 + 25)


3 x − 4 y + 5 = 0 is given by [18 Sep. 2020, Shift-I]
9b 2 + 16 b 2 + 400 = b 4 + 25b 2
(a) 4 x − 3 y + 9 = 0 (b) 4 x + 3 y − 9 = 0
(c) 4 x + 3 y + 9 = 0 (d) 4 x − 3 y − 9 = 0 b 4 = 400 ⇒ b 2 = 20
Sol. (c) From Eq. (iii),
Equation of tangent to the parabola y = 16 x 2 a 2 = 20 + 25
which is perpendicular to the line 3x − 4 y + 5 = 0. ⇒ a 2 = 45
2 2
Put a , b Value in Eq. (i),
256 AP EAMCET Chapterwise Mathematics

(x − 1)2 (y − 2)2 13 + k = 0 ⇒ k = −13


+ =1
45 20 ∴Required Line is 3x − 4 y − 13 = 0
[∴ Answer written in the paper was 3x − 4 y = 13
wrong in RHS it should be 1] Hence, option (b) is correct.
Hence, option (d) is correct.
15. The co-ordinates of focus of the parabola
13. The eccentricity of an ellipse, with its centre 5 x 2 = − 12 y are [18 Sep. 2020, Shift-II]
as origin, is 1/2. If one of the directrices is −3  − 3
(a)  , 0 (b)  , 0 (c)  0, (d)  0,
3 3
x = 4, then the equation of the ellipse is  
5   5   5  5 
given by [18 Sep. 2020, Shift-II]
(a) 4 x2 + y2 = 12 (b) x2 + 3 y2 = 12 Sol. (d)
(c) 4 x2 + 3 y2 = 12 (d) 3 x2 + 4 y2 = 12 Given Parabola,
−12
5x 2 = −12y ⇒ x2 = y
Sol. (d) 5
Centre = (0, 0) −12 −3
4a = ⇒ a=
1
Eccentricity (e) = 5 5
2 −3
Focus = (0, a) =  0, 
Equation of directrix is x = 4  5
a
⇒ = 4 ⇒ a = 4e ⇒ a = 2 Hence, option (d) is correct.
e
b 2 = a 2 (1 − e 2) = 4 1 − 
1 16. The co-ordinates of the focus of the parabola
 4 (x + 3)2 = 2(y − 5) is [18 Sep. 2020, Shift-II]
b =32
−5 
(a)  (b)  − 3, 
11
, 5
x 2 y2  2   2
∴Required Equation of Ellipse is + =1
4 3  − 11  1
(c)  3,  (d)  0, 
3x 2 + 4 y 2 = 12  2   2
Hence, option (d) is correct. Sol. (b)
14. The equation of the transverse axis of (x + 3)2 = 2(y − 5)
hyperbola (x − 3)2 + (y + 1)2 = (4 x + 3 y)2 is ∴ Vertex = (−3, 5), a =
1
[18 Sep. 2020, Shift-II] 2
∴Focus (s) =  −3, 5 +  =  −3, 
(a) 3 x + 4 y = 13 (b) 3 x − 4 y = 13 1 11
(c) 4 x − 3 y = 13 (d) 3 x − 4 y = 9  2  2

Sol. (b) Hence, option (b) is correct.

(x − 3) + (y + 1) = (4 x + 3y)
2 2 2
17. Find the eccentricity of an ellipse, if the
2
4 x + 3y  length of its latus rectum is 4 units and
(x − 3) + (y + 1) = 25 
2 2

 5  distance between its vertex and the nearest
2 focus is 3/2 units. [21 Sep. 2020, Shift-I]
4 x + 3y 
(x − 3)2 + (y + 1)2 = 25   1 2 1 3
 25  (a) (b) (c) (d)
2
3 3 9 4
4 x + 3y 
(x − 3)2 + (y + 1)2 = 5  Sol. (a)
 5 
x 2 y2
∴ It is of the form SP = e PM Let the equation of an ellipse + = 1, (a > b)
a2 b2
∴ Focus = (3, –1) Then according to the question,
Equation of directrix = 4 x + 3y 2b 2
Length of latus rectum = =4 …(i)
Since, Transverse axis perpendicular to directrix a
and passing through focus. 3 3
and a − ae = ⇒ a (1 − e) = …(ii)
∴ 3x − 4 y + k = 0 2 2
3(3) − 4(−1) + k = 0 From Eqs. (i) and (ii), we get
Conic Sections 257

2b 2 And the tangent 4 x + 3y = − 24 cuts the major and


a 4 2b 2 8 minor axes at point P(0, − 8) and Q(− 6, 0).
= ⇒ 2 = (1 − e)
a(1 − e) 3 a 3 (*) No option is correct.
2 20. The line x = m 2 meets an ellipse 9 x 2 + y 2 = 9
4  b2 2
(1 − e 2) = (1 − e) Q 2 = 1 − e  in the real and distinct points if and only if
3  a  [21 Sep. 2020, Shift-II]
4 4 1 (a)|m|> 1 (b)|m|< 1 (c)|m|> 2 (d)|m|< 2
⇒ 1+ e = ⇒ e = −1 ⇒ e =
3 3 3
Hence, option (a) is correct. Sol. (b)
Since, the line x = m2 meets the ellipse
18. The parabola with directrix x + 2 y − 1 = 0 and 9 x 2 + y 2 = 9 in the real and distinct points, so on
focus (1, 0) is ....... [21 Sep. 2020, Shift-I] solving line and ellipse, we get
(a) 4 x2 − 4 xy + y2 − 8 x + 4 y + 4 = 0 9m4 + y 2 = 9 ⇒ y 2 = 91
( − m4) > 0
(b) 4 x2 + 4 xy + y2 − 8 x + 4 y + 4 = 0
⇒ m4 < 1 ⇒|m|< 1
(c) 4 x2 + 4 xy + y2 + 8 x − 4 y + 4 = 0
Hence, option (b) is correct.
(d) 4 x2 − 4 xy + y2 − 8 x − 4 y + 4 = 0
21. Find the condition for the line ax + by + c = 0
Sol. (a)
x 2 y2
Equation of parabola having directrix to be a normal to an ellipse + =1
x + 2y − 1 = 0 and focus (1, 0) is 4 36
|x + 2y − 1| [21 Sep. 2020, Shift-II]
= (x −1)2 + (y − 0)2 1 1 144 1 1 128
1+ 4 (a) 2
+ 2
= 2
(b) 2
+ 2
=
a b c a b c2
⇒ (x + 2y − 1)2 = 5 [(x − 1)2 + y 2] 1 9 256 1 9 32
(c) + = (d) + =
⇒ x 2 + 4 y 2 + 1 + 4 xy − 2x − 4 y = 5(x 2 + y 2 − 2x + 1) a2 b2 c2 a2 b2 c2
⇒ 4 x − 4 xy + y − 8 x + 4 y + 4 = 0
2 2
Sol. (c)
Hence, option (a) is correct. Let a point P(2cosθ, 6sinθ) on the ellipse
2 2 x 2 y2
x y + = 1, so equation of normal to the ellipse at
19. For the ellipse + = 1, if a tangent with 4 36
18 32 point P is
−4
slope intersects the major and minor x − 2cosθ y − 6sinθ
3 =
cosθ sinθ
axes at P and Q respectively. Find P and Q. 2 6
[21 Sep. 2020, Shift-I]
⇒ 2x secθ − 4 = 6 ycosecθ − 36
(a) P(6, 0), Q(0, 8) (b) P(0, 6), Q(8, 0)
⇒ 2x secθ − 6 ycosecθ + 32 = 0 …(i)
(c) P(3 2 , 0), Q(0, 4 2 ) (d) P(0, 3 2 ), Q(4 2 , 0)
Let normal (i) represent the line ax + by + c = 0
Sol. (*) a b c
x2 y2 So = =
Equation of tangent to the ellipse + =1 2secθ −6cosecθ 32
18 32
c 3c
having slope −
4
is ⇒ cosθ = , sinθ = −
3 16a 16b
cos2 θ + sin2 θ = 1
x ± 18  + 32
4 16 Q
y =−
3  9 c2 9c 2
⇒ 2
+ =1
4 256a 256b 2
⇒ y =− x ± 8
3 1 9 256
⇒ + = 2
⇒ 4 x + 3y = ± 24 a2 b2 c
The tangent 4 x + 3y = 24 cuts the major and Hence, option (c) is correct.
minor axes at point P(0, 8) and Q (6, 0).
258 AP EAMCET Chapterwise Mathematics

22. Intersection of two perpendicular tangents to 26. The vertices of the hyperbola
x 2 y2 7 x 2 − 49 y 2 = 343 having eccentricity ‘4/3’ is
the hyperbola − = 1 lies on the circle
4 2 [22 Sep. 2020, Shift-I]
x + y = ………
2 2
[21 Sep. 2020, Shift-II] (a) (0, 0) (b) (±3, 0) (c) (0, ± 5) (d) (± 7, 0)
(a) 2 (b) 12 (c) 2 (d) 2 3 Sol. (d)
Sol. (a) Equation of given hyperbola is
Locus of point of intersection of perpendicular x 2 y2
7 x 2 − 49 y 2 = 343 ⇒ − =1
x 2 y2 49 7
tangents to hyperbola − = 1 is the director
4 2 So, the coordinate of vertices are (± 7, 0).
circle x + y = 4 − 2 = 2
2 2
Hence, option (d) is correct.
Hence, option (a) is correct. 27. The number of values of ‘c’ for which the line
23. The equation of the tangent to the parabola x 2 y2
y = 4 x + c touches the ellipse + = 1 is
y = 12 x at (3 , − 6) is
2
[21 Sep. 2020, Shift-II] 4 1
(a) x − y + 9 = 0 (b) x + y + 3 = 0 [22 Sep. 2020, Shift-I]
(c) x + y − 3 = 0 (d) x = 3 (a) 0 (b) 1
(c) 2 (d) Infinite
Sol. (b)
The equation of tangent to the parabola y 2 = 12x at Sol. (c)
(3, −6) is The possible value of ‘c’ for which the line
y(−6) = 6(x + 3) ⇒ x + y + 3 = 0 x2 y2
y = 4 x + c touches the ellipse + = 1 is
Hence, option (b) is correct. 4 1
c = ± 4× 4+1 = ± 3
24. The equation of the tangent to the parabola
y 2 = 8 x inclined at 30° to the X -axis is 28. The equation of the tangent to the ellipse
[21 Sep. 2020, Shift-II] x 2 + 16 y 2 = 16 which makes an angle 60 °
(a) 3 x − 3 y + 14 = 0 (b) 2 x − 3 y + 14 = 0 with the X -axis is …………[22 Sep. 2020, Shift-I]
(c) 2 x − 3 y + 7 = 0 (d) x − 3 y + 6 = 0 (a) 3 x − y + 7 = 0 (b) 3 x − y − 7 = 0
Sol. (d) (c) 3 x + y − 7 = 0 (d) 3 x − y = 0
The equation of tangent to the parabola y 2 = 8 x Sol. (a,b)
a
having slope m = tanθ is y = mx + . x2 y2
Equation of tangent to the ellipse + = 1,
m 16 1
So, the equation of required tangent is have slope m = tan 60° = 3 is y = 3x ± 48 + 1
1 2
y= x+ ⇒ x − 3y + 6 = 0 ⇒ 3x − y + 7 = 0 or 3x − y − 7 = 0
3 1/ 3
Hence, option (d) is correct. 29. The length of the latus rectum of a parabola
25. If the line y = 2 x + k is normal to the whose focal chord PSQ is such that PS = 3 and
QS = 2 is [22 Sep. 2020, Shift-I]
parabola y = 4 x , then k = [22 Sep. 2020, Shift-I]
2
24 12 6 12
(a) (b) (c) (d)
(a) −10 (b) 10 (c) 12 (d) −12 5 5 5 10
Sol. (d) Sol. (a)
Since, equation of normal to the parabola y 2 = 4 ax We know that the length of the semi latus rectum
having slope m is y = mx − 2am − am3. is the harmonic mean of focal radii, so length of
So, equation of normal to the parabola y 2 = 4 x is the latus rectum
y = 2x + k.  2(PS) (QS) 
= 2 
Therefore, k = − 2 (1) (2) − (1) (2)3  PS + QS 
= − 4 − 8 = − 12 2 × 3 × 2 24
= 2× =
Hence, option (d) is correct. 3+ 2 5
Conic Sections 259

30. The equation of the hyperbola with focus 32. The directrix of the parabola 2 y 2 + 25 x = 0 is
(1, 2), e = 3 and directrix 2 x + y = 1 is given by [23 Sep. 2020, Shift-I]
[22 Sep. 2020, Shift-II] (a) 8 x − 25 = 0 (b) 8 y − 25 = 0
(a) 2 y2 − 12 xy − 7 x2 + 2 x − 14 y + 22 = 0 (c) 25 x − 28 = 0 (d) 25 y − 8 = 0
(b) 2 y2 + 12 xy + 7 x2 − 2 x + 14 y − 22 = 0 Sol. (a)
(c) 2 y2 − 12 xy − 7 x2 − 2 x − 14 y − 22 = 0 Given Equation of parabola, is 2y 2 + 25x = 0
(d) 2 y + 12 xy + 7 x + 2 x + 14 y + 22 = 0
2 2
25
⇒ 2y 2 = − 25x ⇒ y 2 = − x
2
Sol. (a)
25 25
Given, Focus (S) = (1, 2) ∴ 4a = + ⇒ a=+
2 8
Eccentricity (e) = 3
Direction
Equation of Directrix is 2x + y = 1
Required equation of hyperbola is SP = ePM
|2x + y − 1|
(x − 1)2 + (y − 2)2 = 3
22 + 12 O M
Squaring on both sides,
3
(x − 1)2 + (y −2)2 = (2x + y − 1)2
5 x=a
⇒ x 2 + 1 − 2x + y 2 + 4 − 4 y ∴ Required equation of directrix isx = a
3 25
= (4 x 2 + y 2 + 1 + 4 xy − 2y − 4 x) x= ⇒ 8 x − 25 = 0
5 8
⇒ 5(x 2 + y 2 − 2x − 4 y + 5) 33. The equation of the asymptotes of the
= 3(4 x 2 + y 2 + 4 xy − 4 x − 2y + 1) hyperbola 2 x 2 + 5 xy + 2 y 2 − 11 x − 7 y − 4 = 0
5x + 5y − 10 x − 20 y + 25
2 2
is [23 Sep. 2020, Shift-I]
= 12x 2 + 3y 2 + 12xy − 12x − 6 y + 3 (a) 2 x2 + 5 xy + 2 y2 − 11x − 7 y − 9 = 0
⇒ 5x 2 + 5y 2 − 10 x − 20 y + 25 − 12x 2 (b) 2 x2 + 5 xy + 2 y2 − 11x − 7 y + 5 = 0
−3y 2 − 12xy + 12x + 6 y − 3 = 0 (c) 2 x2 + 5 xy + 2 y2 − 11x − 7 y + 4 = 0
(d) 2 x2 + 5 xy + 2 y2 − 11x − 7 y + 9 = 0
⇒ 2y 2 − 7 x 2 − 12xy + 2x −14 y + 22 = 0
∴ 2y 2 − 12xy − 7 x 2 + 2x −14 y + 22 = 0 Sol. (b)
Given equation of hyperbola, is
Hence, Option (a) is correct.
2x 2 + 5xy + 2y 2 − 11 x − 7 y − 4 = 0
31. The eccentricity of the ellipse
The equation of pair of asymptotes, is
4 x 2 + 25 y 2 = 100 is [22 Sep. 2020, Shift-II]
2x 2 + 5xy + 2y 2 − 11 x − 7 y + λ = 0 …(i)
21 21 21 21
(a) (b) (c) (d) ∴ abc + 2 fgh − af 2 − bg2 − ch2 = 0
5 2 4 25
2
−7 −11   5  −7 
Sol. (a) (2)(2) (λ) + 2      − 2 
 2   2   2  2
4 x 2 + 25y 2 = 100
2 2
−11 
x 2 y2 − 2   5
 − λ  = 0
+ =1  2   2
25 4
4λ +
385  49  
− 2  − 2
121   25
∴ a 2 = 25 and b 2 = 4  − λ  = 0
4  4  4   4
a2 − b2 25 − 4 21 16λ + 385 − 98 − 242 − 25λ = 0
Ecentricity = = =
a2 25 5 45 − 9λ = 0
Hence, option (a) is correct. ⇒ λ=5
260 AP EAMCET Chapterwise Mathematics

From Eq. (i) 4 a(1 + m2)(c + am2)


2x 2 + 5xy + 2y 2 − 11 x − 7 y + 5 = 0
So, 40 = 4 a(1 + 4)(1 + a(4))
Hence, option (b) is correct.
⇒ 40 = 16(5a(1 + 4a))
34. If the line y = 2 x + c touches the curve ⇒ 1 = 2a(1 + 4a)
x 2 + 4 y 2 = 4 , then c 2 = [23 Sep. 2020, Shift-I] ⇒ 8a 2 + 2a − 1 = 0
(a) 65 (b) 17 (c) 63 (d) 8 ⇒ 8a 2 + 4a − 2a − 1 = 0
Sol. (b) ⇒ 4a(2a + 1) − 1(2a + 1) = 0
1 1
Given equation of ellipse is, ⇒ a = or − ⇒ 4a = 1 or − 2
4 2
x2 y2
x 2 + 4 y2 = 4 ⇒ + =1 Hence, option (b) is correct.
4 1
If y = 2x + c touches the above ellipse, then 37. The locus of the points of intersection of
c 2 = a 2m2 + b 2 [a 2 = 4; b 2 = 1] perpendicular normals to the parabola
y 2 = 4 ax is [20 April 2019, Shift-I]
c 2 = 4 (2)2 + 1 = 17
(a) y2 − 2 ax + a2 = 0 (b) y2 + ax + 2 a2 = 0
Hence, option (b) is correct.
(c) y − ax + 2 a = 0
2 2
(d) y2 − ax + 3a2 = 0
35. The angle between the tangents drawn from
the point (1, 4) to the parabola y 2 = 4 x is Sol. (d)
Let the equation of normal having slope ‘m’ to the
[23 Sep. 2020, Shift-I]
parabola y 2 = 4ax is
π π π π
(a) (b) (c) (d)
6 4 3 2 y = mx − 2am − am3 and passes through (h, k), then
Sol. (c) k = mh − 2am − am3 is cubic equation in ‘m’. Let
Given, equation of parabola is y 2 = 4 x having roots m1 , m2 and m3.
k
⇒ 4a = 4 ⇒ a = 1 and point P = (1, 4) so, m1 m2m3 = −
a a
∴Equation of tangent is y = mx + and m1 m2 = − 1 due to perpendicular normals.
m
3
h − 2a − a  
1 k k k k
4 = m.()
1 + ⇒ m2 − 4m + 1 = 0 so m3 = ⇒ k=
m a a a  a
∴ m1 + m2 = 4 ⇒ m1 m2 = 1 h k2
⇒ 1= − 2− 2
∴ (m1 − m2)2 = (m1 + m2)2 − 4m1 m2 = 16 − 4 ⋅()
1 a a
(m1 − m2)2 = 12 ⇒ (m1 − m2) = 12 = 2 3 k2 h
⇒ = − 3 ⇒ k2 = a(h − 3a)
Let θ be the angle between tangents. a2 a
On taking locus (h, k), we get
m1 − m2 2 3 2 3
∴ tanθ = = = y 2 − ax + 3a 2 = 0
1 + m1 m2 1 + 1 2
Hence, option (d) is correct.
π
tanθ = 3 ⇒ θ=
3 38. P is a variable point on the ellipse
Hence, option (c) is correct. x2 y2
+ = 1 with foci F1 and F2 . If A is the area
36. The parabola x 2 = 4 ay makes an intercept of a 2 b2
of the triangle PF1 F2 , then the maximum value
length 40 units on the line y = 1 + 2 x , then of A is [20 April 2019, Shift-I]
a value of 4 a is [20 April 2019, Shift-I] e ae ab
(a) (b) (c) aeb (d)
(a) 2 (b) −2 ab b e
(c) −1 (d) 4
Sol. (c)
Sol. (b) Let point P(a cosθ, b sinθ) on the ellipse
Since length of the chord intercepted from the x2 y2
line y = mx + c by the parabola x 2 = 4ay is + = 1.
a2 b2
Conic Sections 261

∴ Area of ∆PF1 F2 =
1
(2ae) b|sinθ| = aeb |sinθ| = A ⇒ y − 2= x − 3+ 3
2 y − 2 = x − 3 + 3 or y − 2 = x − 3 − 3
π 3π x− y−4= 0
For maximum value of A, θ = or so or
2 2 ⇒ x − y + 2= 0
Amax = aeb.
Hence, option (a) is correct.
Hence, option (c) correct.
41. The vertex and the focus of the parabola
39. If the line joining the points A(α) and B(β) on 2 y 2 + 5 x − 6 y + 1 = 0 are respectively
2 2
x y
the ellipse + = 1 is a focal chord, then [20 April 2019, Shift-II]
25 9 −7 3
(a)  ,  ,  ,  (b)  ,  ,  , 
7 3 3 3 53 3
α β  10 2   40 2   10 2   40 2 
one possible value of cot ⋅ cot is
7 − 3  7 7  −7 −3
(c)  , (d)  ,  ,  , 
2 2 7 17
,  , 
[20 April, 2019 Shift-I]  10 2   10 8   10 2   10 8 
(a) −3 (b) 3 (c) −9 (d) 9 Sol. (a)
Sol. (c) Equation of given parabola is
Since equation of chord joining the points A(α) 2y 2 + 5x − 6 y + 1 = 0
x2 y2
and B(β) on the ellipse + = 1 is ⇒ 2(y 2 − 3y) + 5x + 1 = 0
25 9
α+β y α+β α −β  2
3 
2
⇒ 2 y 2 − 3y +   −    + 5x + 1 = 0
x 3
cos + sin = cos …(i)

5 2 3 2 2   2  2 

Q Chord (i) is the focal chord so, it will pass 2
⇒ 2 y −  − + 5x + 1 = 0
3 9
through focus (4, 0)
4 α+β α −β  2 2
cos = cos 2
2 y −  = − 5 x − 
5 2 2 3 7

 α β α β  2  10 
⇒ 4  cos cos − sin sin 
 2 2 2 2 2
⇒ y − 3  5  7
α β α β   = 4 −   x −  …(i)
= 5 cos cos + sin sin   2  8  10 
 2 2 2 2
The coordinate of vertex of parabola (i) is  , 
7 3
 α β   α β   10 2
⇒ 4  cot cot − 1 = 5 cot cot + 1
 2 2   2 2  and coordinate of focus of parabola (i) is
α β  − 5 + 7 , 3 =  −50 + 56 , 3 =  6 , 3 or  3 , 3
⇒ cot cot = − 9        
2 2  8 10 2  80 2  80 2  40 2
Hence, option (c) is correct. Hence, option (a) correct.
40. The equation of a tangent to the hyperbola 42. If a normal chord at a point t(≠ 0) on the
16 x 2 − 25 y 2 − 96 x + 100 y − 356 = 0 which
parabola y 2 = 9 x subtends a right angle at its
makes an angle 45° with its transverse axis is
vertex, then t = [20 April 2019, Shift-II]
[20 April 2019, Shift-I]
(a) 3 (b) 5 (c) ± 3 (d) ± 2
(a) x − y + 2 = 0 (b) x − y + 4 = 0
(c) x + y + 2 = 0 (d) x + y + 4 = 0 Sol. (d)
Equation of given parabola is
Sol. (a)
Given equation of hyperbola y2 = 9 x …(i)
16 x 2 − 25y 2 − 96 x + 100 y − 356 = 0 Equation of normal chord at a point t(≠ 0) on the
parabola (i) is
(x − 3) 2
(y − 2) 2
⇒ − =1 …(i) tx + y = 2  t + t 3
9 9
…(ii)
25 16  4 4
Now equation of tangent to the hyperbola (i)
Q The chord (ii) subtends a right angle at vertex
having slope ‘1’ is
of parabola V(0, 0), so first homogenise the
y − 2 = 1(x − 3) + 251 () − 16 parabola (i) with the help of line (ii), we get
262 AP EAMCET Chapterwise Mathematics

  (a) x2 − (1 + 17 )x + 17 = 0
 tx + y 
y − 9x
2
 =0 …(iii) (b) x − (1 − 17 )x − 17 = 0
2

 9 t + 9 t3  (c) x3 − x2 − 17 x + 17 = 0
2 4 
(d) x3 − x2 + 17 x − 17 = 0
The sum of the coefficients of x 2 and y 2 terms
should be zero. Sol. (c)
So, 1 −
9t
= 0 ⇒1−
1
=0 If line 4 x − y + c = 0 touches the ellipse
9
t+ t
9 3 1 t2 x2 y2
+ + = 1, then
2 4 2 4 4 1
1 t2 c 2 = 4(4) + 1 ⇒ c 2 = 17 ⇒ c = ± 17
⇒ + = 1 ⇒ t2 = 2 ⇒ t = ± 2
2 4 Now, from the options, the equation
Hence, option (d) is correct. x 3 − x 2 − 17 x + 17 = 0
43. The major and minor axes of an ellipse are ⇒ x 2(x − 1) − 17(x − 1) = 0
along the X -axis and Y -axis respectively. If ⇒ (x − 1)(x 2 − 17) = 0 ⇒ x = 1, ± 17
its latusrectum is of length 4 and the Q The cubic equation x 3 − x 2 − 17 x + 17 = 0 have
distance between the foci is 4 2, then the roots ± 17.
equation of that ellipse is [20 April, 2019 Shift-II]
Hence, option (c) is correct.
(a) 2 x2 + y2 = 16 (b) x2 + 2 y2 = 16
x2 y2 x2 y2 45. If e1 , e2 are respectively the eccentricities of
(c) + =1 (d) + =1
2 3 3 2 the curves 9 x 2 − 16 y 2 − 144 = 0 and
Sol. (b) e12 e22
9 x 2 − 16 y 2 + 144 = 0 , then =
Let the equation of ellipse e12 + e22
x2 y2
2
+ 2 = 1, (a > b) …(i) [20 April 2019, Shift-II]
a b (a) 2 (b) 1 (c) 3 (d) 2
Q Length of latusrectum of ellipse (i) is
Sol. (b)
2b 2
= 4 (given) …(ii) Given, e1 is the eccentricity of hyperbola
a
9 x 2 − 16 y 2 = 144 …(i)
and distance between the foci is
e 2 is the eccentricity of hyperbola
2ae = 4 2 (given)
9 x 2 − 16 y 2 = − 144 …(ii)
b2  b2 
⇒ a 1− 2 =2 2 Q e = 1 − 2 , (b > a) (i) is a hyperbola and (ii) is a conjugate
a  a  hyperbola, We have,
⇒ a2 − b2 = 8 1 1
∴ + =1
e12 e 22
⇒ a 2 − 2a − 8 = 0 [from Eq. (ii), b 2 = 2a]
e12 + e 22
⇒ (a − 4) (a + 2) = 0 ∴ =1
⇒ a = 4, − 2 ⇒ a = 4 {Q a > 0} e12 e 22
So, b2 = 8 e12 e 22
⇒ =1
∴ Equation of required ellipse is e12 + e 22
x2 y2 Hence, option (b) is correct.
+ = 1 ⇒ x 2 + 2y 2 = 16
16 8 46. If the focus of a parabola divides a focal
Hence, option (b) is correct. chord of the parabola into segments of
44. If c ∈ R be such that the line 4 x − y + c = 0 lengths 5, 3 units, then the length of the
latusrectum of that parabola is
touches the ellipse x 2 + 4 y 2 = 4 , then an
[21 April 2019, Shift-I]
equation having all such values of c among 15 25 15
its roots is [20 April 2019, Shift-II] (a) (b) 20 (c) (d)
4 2 2
Conic Sections 263

Sol. (d) x2 y2
or + = 1 at ( 5 cosθ, 2sinθ)
Since, semi latusrectum is the harmonic mean of 5 4
segments. 5
2 × 5 × 3 30 15 ⇒ Slope of normal = 5y / 4 x = tanθ
So, semi laturectum length = = = 2
5+ 3 8 4 ∴ Equation of line is
15 15
∴ Length of latusrectum = 2 × = y − 2sinθ =
5
tanθ (x − 5 cosθ)
4 2 2
47. The angle between the tangents drawn to the 5 sinθ
⇒ y= tanθ × x − … (ii)
parabola y 2 = 4 x from the point (1, 4) is 2 2
[21 April 2019, Shift-I] By comparing Eqs. (i) and (ii), we get
π π 2π π t=−
sinθ
⇒ sinθ = − 2t … (iii)
(a) (b) (c) (d)
4 3 5 6 2
1 5 2
Sol. (b) = tanθ ⇒ tanθ =
t 2 5t
Equation of a tangent to a parabola y 2 = 4 x is
2
1 ∴ sinθ = … (iv)
y = mx +
m 4 + 5t 2
Since, it passes through point (1, 4). So, From Eqs. (iii) and (iv), we get
1 2
4= m+ − 2t =
m 4 + 5t 2
or 4m = m2 + 1 ⇒ m2 − 4m + 1 On squaring both sides, we get
4 ± 16 − 4 4 ± 12 t 2(4 + 5t 2) = 1 ⇒ 5t 4 + 4t 2 = 1
∴ m= =
2 2
49. If the tangents drawn from a point P to the
m= 2± 3
ellipse 4 x 2 + 9 y 2 − 24 x + 36 y = 0 are
∴ m1 = 2 + 3 and m2 = 2 − 3 perpendicular, then the locus of P is
Hence, angle between two slopes is [21 April 2019, Shift-I]
m1 − m2 2 3 (a) x2 + y2 − 6 x + 4 y + 13 = 0
tanθ = =
1 + m1 m2 1+1 (b) x2 + y2 − 6 x + 4 y − 13 = 0
π π (c) x2 + y2 = 26
⇒ tanθ = 3 ⇒ tanθ = tan ⇒θ=
3 3 (d) x2 + y2 + 6 x − 4 y − 13 = 0
48. If the tangent drawn to the parabola y = 4 x 2
Sol. (b)
at (t2 , 2 t) is the normal to the ellipse Given, equation of ellipse is
4 x 2 + 5 y 2 = 20 at ( 5 cos θ , 2 sin θ), then 4 x 2 + 9 y 2 − 24 x + 36 y = 0
[21 April 2019, Shift-I] ⇒ 4(x 2 − 6 x + 9) + 9(y 2 + 4 y + 4) − 36 − 36 = 0
5 100 ⇒ 4(x − 3)2 + 9(y + 2)2 = 72
(a) 5 t 4 + 4 t 2 = 1 (b) + 2 =1
t4 t
(x − 3)2 (y + 2)2
(c) t = sinθ (d) cosθ = t + 1 ⇒ + =1
18 8
Sol. (a) Here, required locus is director circle.
Given, tangent to parabola y 2 = 4 x at (t 2 , 2t) is
∴ (x − 3)2 + (y + 2)2 = 26
y ⋅ 2t = 2(x + t 2) ⇒ yt = x + t 2
⇒ x 2 − 6 x + 9 + y 2 + 4 + 4 y = 26
x
⇒ y= + t … (i) ⇒ x 2 + y 2 − 6 x + 4 y = 13
t
Normal to the ellipse 4 x 2 + 5y 2 = 20 ⇒ x 2 + y 2 − 6 x + 4 y − 13 = 0
264 AP EAMCET Chapterwise Mathematics

50. The normal at a point on the parabola 52. Let P be any point on the ellipse
y 2 = 4 x passes through (5 , 0). If two more 7 x 2 + 16 y 2 = 112, S be a focus, L be the
normals to this parabola also pass through corresponding directrix and PM be the
(5, 0), then the centroid of the triangle perpendicular distance from P to directrix L.
formed by the feetof these three normals is SP
Then =
[21 April 2019, Shift-II] PM [21 April 2019, Shift-II]
(a)  , 
1 1 1 1 3 1
(b) (2, 0) (c) (5, 0) (d) (0, 2 ) (a) (b) (c) (d)
2 2 4 2 4 2
Sol. (b) Sol. (c)
Given equation of parabola y 2 = 4 x Given equation of ellipse is
Here, a = 1 x2 y2
7 x 2 + 16 y 2 = 112 ⇒ + =1 …(i)
Equation of normal at point P(5, 0) 16 7
y = − tx + 2t + t 3 ⇒ 0 = − 5t + 2t + t 3 Z′
⇒ 3t = t 3 ⇒ t 2 = 3 ⇒ t = ± 3, 0

Co Normal Point P Q R
M P
( at 12, 2 at 1 ) ( at 22, 2 at 2 ) ( at 32, 2 at 3 )

Directrix
= ( 3, 2 3 ) ( 3, − 2 3 ) ( 0, 0) S (focus)
 3 + 3 + 0 2 3 − 2 3 + 0 Z′
Hence, Centroid =  , 
 3 3  x2 y2
By comparing the Eq. (i) with 2
+ 2 = 1, we get
a b
=  , 0 = (2, 0)
6
3  a 2 = 16, b 2 = 7

51. The equation of the normal to the parabola 7 9 3


∴ e = 1− = =
y 2 = 4 x which is perpendicular to 16 16 4
x + 3 y + 1 = 0 is [21 April 2019, Shift-II] As we know that,
(a) 3 x − y = 33 (b) 3 x − y + 33 = 0 SP SP 3
=e ∴ =
(c) 3 x + y = 33 (d) 3 x + y + 33 = 0 PM PM 4
Sol. (a) 53. If tangents are drawn to the ellipse
Given equation of parabola is y 2 = 4 x. x 2 y2
+ = 1 at the ends of latusrectum, then
The equation of the normal to the parabola 9 5
y 2 = 4ax at (am2 , − 2am) is y = mx − 2am − am3, the area of the quadrilateral thus formed is
where m is the slope of the normal. [21 April 2019, Shift-II]
Here, a = 1, so the equation of the normal is 15 13
(a) 27 (b) (c) (d) 45
y = mx − 2m − m3 4 2

Since, the normal is perpendicular to the line Sol. (a)


x + 3y + 1 = 0. x2 y2
We have, + =1
Then, m = slope of normal to line x + 3y + 1 = 0 is 9 5
m ×  −  = − 1 ⇒ m = 3
1 Let e be the eccentricity of the ellipse. Then,
 3 5 2
e2 = 1 − ⇒ e =
∴ y = 3x − 2(3) − (3)3 9 3
⇒ y = 3x − 6 − 27 The coordinates of the end-points of
latusrectum are
⇒ y = 3x − 33
−5
L  2,  , M  −2,  , M′  −2,  and L′  2, − 
5 5 5
⇒ 3x − y = 33  3  3  3  3
Conic Sections 265

The equation of tangents at these points are 36 6


⇒ b2 = ⇒ b=
2x + 3y − 9 = 0 …(i) 7 7
−2x + 3y − 9 = 0 …(ii) x 2 7 y2
∴ Eq. (i) becomes − =1
2x + 3y + 9 = 0 …(iii) 36 36
−2x + 3y + 9 = 0 …(iv) As we know that,
Y b 2 = a 2(e 2 − 1)
36 1
A ⇒ = 36(e 2 − 1) ⇒ = e 2 − 1
2, 5 
 7 7
 5  3
M –2,  1 8 8 2 2
3 ⇒ e =1 + = ⇒e =
2
=
7 7 7 7
X′ X 2 2
B ∴ Eccentricity of hyperbola be .
D 7

M′ –2,  L′ 2, 
–5  –5  55. The length of the latusrectum of the parabola
3 3
C 20 (x 2 + y 2 − 6 x − 2 y +10) = (4 x − 2 y − 5)2 , is
Y′ [22 April, 2019 Shift-I]
5
By solving above equations, we get (a) (b) 2 5 (c) 5 (d) 4 5
2
A(0, 3), B − , 0 , C(0, − 3) and D  , 0
9 9
 2  2  Sol. (c)
Given, equation of parabola is
Now, AC = (3 + 3)2 = 62 = 6
20(x 2 + y 2 − 6 x − 2y + 10) = (4 x − 2y − 5)2
2
and BD =  9 + 9 = 92 = 9 4 x − 2y − 5
2

 2 2
 ⇒ (x 2 + y 2 − 6 x − 2y + 10) =  
 20 
1
Now, area of quadrilateral = × 6 × 9 = 27 sq. units 2
4 x − 2y − 5
2 ⇒ (x − 3)2 + (y − 1)2 =   …(i)
 20 
54. A hyperbola with centre at (0, 0) has its
transverse axis along X -axis whose length is In Eq. (i), focus is (3, 1) and equation of directrix
is 4 x − 2y − 5 = 0.
12. If (8, 2) is a point on the hyperbola, then
its eccentricity is [21 April 2019, Shift-II] |12 − 2 − 5|
So, distance from focus to directrix is
8 2 2 3 9 20
(a) (b) (c) (d) 5
7 7 7 7 = = 2a
2
Sol. (b) Now, length of latursrectum = 4a
Given, centre of hyperbola is (0, 0) and its
5
transverse axis is along X-axis and its length is 12. = 2.(2a) = 2. = 5
2
x 2 y2
Let the equation of hyperbola be − = 1 …(i) 56. y = 3 x − 2 is a straight line touching the
a2 b2
Here, 2a = 12 ⇒ a = 6 [Given] parabola (y − 3)2 = 12(x − 2). If a line drawn
Put the value of a in Eq. (i), we get perpendicular to this line at P on it, touches
x 2 y2 the given parabola, then the point P is
− =1 …(ii) [22 April 2019, Shift-I]
36 b 2
(a) (−1, − 5) (b) (−1, 5)
Eq. (ii) passes through point (8, 2).
(c) (−2, − 8) (d) (2, 4)
64 4 16 4
∴ − =1⇒ − 2 =1
36 b 2 9 b Sol. (a)
16 4 7 4 According to the given information,
⇒ −1 = 2 ⇒ = 2
9 b 9 b Angle between the tangents is 90°.
266 AP EAMCET Chapterwise Mathematics

⇒ The point on the directrix Sol. (a)


Here, x − 2=− 3 We know that,
x = −1 is directrix If two lines l1 x + m1 y + n1 = 0 and
Given tangent is y = 3x − 2
l 2 x + m2 y + n2 = 0 are conjugate lines with respect
∴ y = 3 (−1) − 2 = − 5 to the hyperbola
∴ Point P is (−1, − 5.
)
x 2 y2
− =1, then a 2l1 l 2 − b 2m1 m2 = n1 n2
57. If (l , m) is the circumcentre of an equilateral a2 b2
x2 y2 For hyperbola,
triangle inscribed in the ellipse =1 +
a b2 2 x2 y2
5x 2 − 6 y 2 =15 ⇒ − =1
having vertices at points with eccentric 3 5/ 2
angles θ1 , θ 2 and θ 3 , then 5
Here,a 2 = 3, b 2 = , l1 = 2, l 2 = 3,
2
[cos(θ1 − θ 2) + cos(θ 2 − θ 3) + cos(θ 3 − θ1)] = 2
3 m1 = − k, m2 = − 1 and n1 = 3, n2 =1
[22 April, 2019 Shift-I] 5
∴ 3 × 2 × 3 − (− k) (−1) = 3 × 1
9l 2 9m2 l2 m2 2
(a) + −1 (b) + −3
2 a2 b2 a2 b2 ⇒
5 5
18 − k = 3 ⇒ 15 = k ⇒ k = 6
3l 2 3m2 3l 2 3m 2
3 2 2
(c) + −1 (d) + 2 −
a2 b2 a2 b 2 59. For the parabola y 2 + 2 x + 2 y − 3 = 0, match
Sol. (c) the items in List-I with those from List-II.
As we know coordinates of circumcentre of an [22 April 2019, Shift-II]
equilateral triangle is same as centroid, so
a cosθ1 + a cosθ2 + a cosθ3 List I List II
(l , m) =  ,
 3 A. Vertex I. 2x − 5 = 0
b sinθ1 + b sinθ2 + b sinθ3  3 
 B. Focus II.
 , − 1
3  2 
3l
⇒ = cosθ1 + cosθ2 + cosθ3 …(i) C. Equation of the Directrix III. x−2 = 0
a
3m D. Equation of the Axis IV. y + 1= 0
and = sinθ1 + sinθ2 + sinθ3 …(ii)
b
V. (2, − 1)
Now, after squaring and adding the Eqs. (i) and
(ii), we get VI.  3
 2, 
9l 2 9m2  2
+ 2 = Σ cos2 θ1 + 2Σ cosθ1 cosθ2 + Σsin2 θ1
a2 b
+ 2Σsinθ1 sinθ2 The correct match is
A B C D A B C D
9l 2 9m2
⇒ + 2 = 3 + 2Σ cos(θ1 − θ2) (a) V VI I III (b) V II I IV
a2 b (c) VI V IV I (d) II VI III IV
2 3l 2 3m2
⇒ Σ cos(θ1 − θ2) = 2 + 2 − 1 Sol. (b)
3 a b
Given equation of parabola is
2
⇒ [cos(θ1 − θ2) + cos(θ2 − θ3) + cos(θ3 − θ1)] y 2 + 2x + 2y − 3 = 0
3
⇒ y 2 + 2y + 1 = − 2x + 3 + 1
3l 2 3m2
= 2 + 2 −1
a b ⇒ (y + 1)2 = − 2(x − 2)

(y + 1)2 = − 4  (x − 2)
1
58. If 2 x − ky + 3 = 0, 3 x − y + 1 = 0 are conjugate ⇒
 2
lines with respect to 5 x 2 − 6 y 2 =15, then k =
Q (y − k)2 = 4a(x − h)
[22 April 2019, Shift-I]
Here, vertex = (h, k) = (2, − 1)
(a) 6 (b) 4 (c) 3 (d) 2
Conic Sections 267

focus = (h − a , k) =  2 − , − 1 =  , − 1


1 3 Sol. (d)
 2  2  2a 2 2 a 2
1 Given, = (2a) ⇒ =
Directrix ⇒ x = h + a = 2 + b 3 b 3
2
5 a2
x = ⇒ 2x − 5 = 0 ∴ Eccentricity, e = 1 − 2
2 b
Equation of axis a2 4 5 5
⇒ e2 = 1 − = 1 − ⇒ e2 = ⇒ e =
⇒ y = k ⇒ y = −1 ⇒ y + 1 = 0 b2 9 9 3
Here, A → V, B → II, C → I, D → IV
62. If y = x + c is a normal to the ellipse
Hence, option (b) is correct.
x 2 y2
60. The area (in sq units) of the triangle formed + = 1, then c 2 =
25 9 [22 April 2019, Shift-II]
by the normal to the parabola y 2 = 16 x whose 128 17
1 (a) (b) (c) 34 (d) 225
slope is with the co-ordinates axes is 17 128
2
Sol. (a)
[22 April, 2019 Shift-II]
x 2 y2
(a)
9
(b)
27
(c)
54
(d)
81 Given that, y = x + c and + =1
4 4 4 4 25 9
x2 y2
Sol. (d) We know that, normal to ellipse 2 + 2 = 1 is
a b
Given equation of parabola is y 2 = 16 x
m2(a 2 − b 2)2
1 c = 2
2
Here, a = 4 and m = a + b 2m2
2
Normal equation to parabola y 2 = 4ax is Here, m = 1, a 2 = 25, b 2 = 9
y = mx − 2am − am3 1(25 − 9)2 (16)2 256 128
∴ c2 = = = =
x 1 1 25 + 9 × 1 34 34 17
∴ y = − 2(4) × − (4)
2 2 8
x 1 x − 8 −1 63. The area (in sq units) of the quadrilateral
⇒ y= − 4− ⇒y= formed by the four common tangents drawn
2 2 2
⇒ 2y = x − 9 ⇒ x − 2y = 9 x 2 y2
to the two hyperbolas 2 − 2 = 1 and
a b
y2 x 2
− = 1(a > b) is
(0, 0) (9, 0) a 2 b2 [22 April 2019, Shift-II]
(a) a2 − b 2 (b) 2(a2 − b 2 )
(0, 9/2) a2 − b 2 a2 − b 2
(c) (d)
2 2
Required area Sol. (b)
0 0 Equation of tangent having slope m to the
1 1  81  81
= 9 0 = −  = sq. units x 2 y2
2 2  2 4 hyperbola, 2 − 2 = 1 is y = mx ± a 2m2 − b 2 …(i)
0 − 9/ 2 a b
Q Tangent (i) is also tangent to hyperbola,
61. If the major axis of an ellipse lies on the y2 x 2
Y -axis, its minor axis lies on the X -axis and − =1 …(ii)
a2 b2
2
the length of its latusrectum is equal to of From Eq. (i) and (ii), we get
3
its minor axis, then the eccentricity of that (mx + a 2m2 − b 2)2 x 2
− 2 =1
ellipse is [22 April 2019, Shift-II] a2 b

(a)
3
(b)
1
(c)
2
(d)
5 ⇒ b 2(m2 x 2 + a 2m2 − b 2 + 2mx a 2m2 − b 2)
2 2 3 3 − a 2 x 2 − a 2b 2 = 0
268 AP EAMCET Chapterwise Mathematics

⇒ (b 2m2 − a 2) x 2 + 2mb 2( a 2m2 − b 2 x) ⇒ m2 + mx + (10 − y) = 0


+ (a 2b 2m2 − b 4 − a 2b 2) = 0 Since, above line is tangent, then D = 0
b 2 − 4ac = 0
Q Line (ii) is tangent to hyperbola (ii), so
D=0 x 2 − 4 ⋅ 1 ⋅ (10 − y) = 0
⇒ 4m2b 4 (a 2m2 − b 2) − 4(b 2m2 − a 2) x 2 = 410
( − y) ⇒ x 2 = − 4(y − 10)
Hence, answer is (c).
(a 2b 2m2 − b 4 − a 2b 2) = 0
⇒ a 2b 4 m4 − b 6m2 = a 2b 4 m4 − b 6m2 − a 2b 4 m2
x2 y2
66. The ellipse += 1 (b > a) and the
− a 4 b 2m2 + a 2b 4 + a 4 b 2 a 2 b2
⇒ a b m + a b m = a b + a4b2
2 4 2 4 2 2 2 4 parabola y 2 = 4 ax cut at right angles. If e is
⇒ m2(a 2b 4 + a 4 b 2) = a 2b 4 + a 4 b 2 the eccentricity of the ellipse, then 2 e 2 =
⇒ m2 = 1 ⇒ m = ± 1 [23 April 2019, Shift-I]
1 1 1
So, the equation of common tangents are (a) 1 (b) (c) (d)
2 8 3
y=± x± a2 − b2
Sol. (a)
Now, required area (in Sq. units) Given equation of parabola is y 2 = 4ax, then slope
1
= 4 × ( a 2 − b 2)2 = 2(a 2 − b 2) of tangent to be given parabola at point of
2 1
intersection (at 2 , 2at) is m1 = ,
64. The parametric equations of the parabola t
y 2 − 8 x − 4 y − 12 = 0 are [23 April 2019, Shift-I] and slope of tangent to the given ellipse
x2 y2
(a) x = 2 + 2 t 2 , y = − 2 + 4t 2
+ 2 = 1 at point of intersection (at 2 , 2at) is
a b
(b) x = 2 + 4t , y = − 2 + 2 t 2 b2
m2 = − 2 ×  
t
(c) x = − 2 + 2 t 2 , y = 2 + 4t a  2
(d) x = − 2 + 4t , y = 2 + 2 t 2 Q m1 m2 = − 1
Sol. (c)  1  b2 t  b2
⇒   × − 2 ×  = −1 ⇒ 2 = 2
Given equation of parabola is  t  a 2 a
y 2 − 8 x − 4 y − 12 = 0 a2 1
Q e2 = 1 − =1 − (Q b > a)
y − 4 y = 8 x + 12
2 b2 2
(y − 2)2 = 8(x + 2) ⇒ 2e 2 = 1

∴ 4a = 8 ⇒ a = 2 Hence, option (a) is correct.


∴Parametric equations are 67. Let a tangent drawn at any point on the
x + 2 = at 2, y − 2 = 2at x 2 y2
ellipse + = 1 cut the X -axis at Q. Let R
x = − 2 + 2t 2, y = 2 + 4t 25 16
∴Hence answer is (c). be the image of Q with respect to y = x . If S is
a circle with QR as its diameter, then the
65. For any non-zero real value of m, the fixed point through which the circle S
equation of the parabola to which the line passes is [23 April 2019, Shift-I]
mx − y + 10 + m 2 = 0 is a tangent, is (a) (5, 4) (b) (4, 5) (c) (0, 0) (d) (0, 5)
[23 April 2019, Shift-I] Sol. (c)
(a) x2 = y − 10 (b) y2 = 4 ( x − 2 ) x2 y2
(c) x2 = − 4 ( y − 10) (d) x2 = − 4 y The equation of given ellipse is + = 1.
25 16
Sol. (c) Let a point P(5cos θ, 4sinθ) on the ellipse, then
equation of tangent to the ellipse at point P is
Given, equation of tangent to the parabola is
cos θ   sinθ  = 1
mx − y + (10 + m2) = 0 x   + y  …(i)
 5   4 
Conic Sections 269

 5  Sol. (c)
So, point Q  , 0 ⇒ Point R (0, 5sec θ)
 cos θ  Given, equation of line is y = 6 x + 1 and equation
Now, equation of circle with QR as its diameter is of parabola is y 2 = 24 x.
x(x − 5 sec θ) + y (y − 5sec θ) = 0 The locus of the point of intersection of
⇒ x 2 + y 2 − (5 sec θ) x − (5 sec θ) y = 0 perpendicular tangent to a parabola is its directrix.
So, required point is the point of intersection of
The above circle passes through the origin. y = 6 x + 1 and directrix x = − 6.
Hence, option (c) is correct. Hence, its coordinates are (− 6, − 35).
68. If the normals drawn to the hyperbola xy = 4 70. A point on the parabola whose focus is S(1,−1)
at (α i , β i ) (i = 1, 2 , 3 , 4) are concurrent at the and whose vertex is A(1,1) is
point (a , b), then [22 April 2018, Shift-I]
(α1 + α 2 + α 3 + α 4)
(α1α 2α 3α 4) = (a)  3, 
1
(b) (1, 2 ) (c)  2, 
1
(d) (2, 2 )
(β1 + β 2 + β 3 + β 4)  2  2
[23 April 2019, Shift-I]
−16b −16a 4b 4a
Sol. (a)
(a) (b) (c) (d) The gradient of the line joining the focus S(1, − 1)
a b a b
and vertex A(1, 1) is
Sol. (b) −1 −1
m= =0
The equation of normal to the given hyperbola 1 −1
xy = 4 at point  2t ,  is
2
Let Q(h, k) be the point of intersection of the axis
 t
AS with the directrix. The A(1, 1) will be the
2t 4 − xt 3 + yt −2 = 0 …(i) mid-point of QS.
h+1 k −1
Q Normal (i) passes through point (a, b), so ∴ = 1 and = 1 ⇒ h = 1 and k = 3
2t 4 − at 3 + bt − 2 = 0 , the equation having roots 2 2
α α α α ∴Q is the point (1, 3)
are 1 , 2 , 3 and 4 , so
2 2 2 2 So, the directrix passes through the point (1, 3)
1 a and has the gradient 0.
(α1 + α 2 + α 3 + α 4) = ⇒α1 + α 2 + α 3 + α 4 = a
2 2 The equation of the directrix is
 α1α 2  y − 3= 0
∑  =0
 4  Let P(x , y) be any point on the parabola and M be
ααα −b αααα the foot of the perpendicular drawn from P on the
∑ 1 2 3 = and 1 2 3 4 = − 1 directrix.
8 2 16
∴ PS = PM
4 4 4 4
Q β1 + β 2 + β 3 + β 4 = + + + As, PS 2 = PM 2
α1 α 2 α 3 α 4
Σα1α 2α 3 −b / 2 × 8 y − 3
2
=4 =4 =b As, (x − 1)2 + (y + 1)2 =  
α1α 2α 3α 4 −16  1 
(α + α 2 + α 3 + α 4) ⇒ (x − 1)2 + (y + 1)2 = (y − 3)2
∴ 1 (α1α 2α 3α 4)
β1 + β 2 + β 3 + β 4
⇒ (x − 1)2 = 81
( − y)
a(−16) a
= = − 16 By checking option (a),
b b
(3 − 1)2 = 8 1 − 
1
Hence, option (b) is correct. ⇒
 2
69. The line y = 6 x + 1 touches the parabola 1
⇒ (2) = 8 ×
2
y 2 = 24 x . The coordinates of a point P on this 2
line, from which the tangent to y 2 = 24 x is ⇒ 4=4
Hence, point  3,  lies on the parabola
1
perpendicular to the line y = 6 x + 1, is
 2
[22 April 2018, Shift-I]
(a) (−1,−5) (b) (−2,−11) (c) (−6,−35) (d) (−7,−41) (x − 1)2 = 81
( − y).
270 AP EAMCET Chapterwise Mathematics

71. An ellipse having the coordinate axes as its 3 × 3 3 cosθ − 18 3 3 × 3 3 cosθ + 18 3


P=
axes and its major axis along Y -axis, passes 27 cos θ + 36sin θ
2 2
27 cos2 θ + 36sin2 θ
through the point (−3 , 1) and has eccentricity
2 36 × 27 − 9 × 27 cos2 θ
=
. Then its equation is 36sin2 θ + 27 cos2 θ
5 [22 April 2018, Shift-I]
(a) 3 x2 + 5 y2 − 15 = 0 (b) 5 x2 + 3 y2 − 32 = 0 9 × 27(4 − cos2 θ)
=
(c) 3 x2 + 5 y2 − 32 = 0 (d) 5 x2 + 3 y2 − 48 = 0 ( − cos2 θ) + 27 cos2 θ
361
9 × 27(4 − cos2 θ)
Sol. (c) =
Let the equation of ellipse is 36 − 9 cos2 θ
x2 y2 9 × 27(4 − cos2 θ)
+ 2 =1 … (i) = = 27.
a 2
b 9(4 − cos2 θ)
2 a2 − b2 2 73. The equation of the hyperbola whose
Given, e= ⇒ =
5 a2 5 asymptotes are the lines 3 x + 4 y − 2 = 0 ,
a −b 2 2
2 2 x + y + 1 = 0 and which passes through the
⇒ = ⇒ 5a 2 − 5b 2 = 2a 2 point (11, ) is [22 April 2018, Shift-I]
a2 5
(a) 6 x2 + 11xy + 4 y2 − 30 x + 2 y + 7 = 0
5b 2
⇒ 3a 2 = 5b 2 ⇒ a 2 = (b) 6 x2 + 11xy + 4 y2 − x + 2 y − 22 = 0
3
(c) 6 x2 + 11xy + 4 y2 − x + 2 y + 22 = 0
Now, ellipse passes through (− 3, 1)
(d) 6 x2 + 11xy + 4 y2 − 3 x − 7 y − 11 = 0
(− 3)2 ()
12
⇒ 2
+ 2 =1 ⇒ 9b 2 + a 2 = a 2b 2 Sol. (b)
a b
Equation to the asymptotes are given as
5b 2 5b 4 32b 2 5b 4 32 3x + 4 y − 2 = 0
⇒ 9b 2 + = ⇒ = ⇒ b2 = … (i)
3 3 3 3 5 and 2x + y + 1 = 0 … (ii)
From Eq. (i), we get Eqs.(i) and (ii) may be given by
x2 y2 (3x + 4 y − 2) (2x + y + 1) = 0
⇒ + =1 … (iii)
32 32
As, the equation to the hyperbola will differ from
3 5 Eq. (iii) only by a constant, it may be given by
⇒ 3x 2 + 5y 2 = 32 (3x + 4 y − 2) (2x + y + 1) = λ … (iv)
⇒ 3x 2 + 5y 2 − 32 = 0. (where λ is a constant)
(1, 1) lies on the curve given by
72. The product of the perpendicular distances Eq. (iv), we have
drawn from the points (3, 0) and (−3 , 0) to the (3 + 4 − 2) (2 + 1 + 1) = λ
x 2 y2  9 ⇒ (5) (4) = λ ⇒ λ = 20
tangent of the ellipse + = 1 at 3 ,  is
36 27  2 Hence, the equation of the hyperbola will be
[22 April 2018, Shift-I] (3x + 4 y − 2) (2x + y + 1) = 20
(a) 36 (b) 27 ⇒ 6 x 2 + 3xy + 3x + 8 xy + 4 y 2
(c) 9 (d) 63 + 4 y − 4 x − 2y − 2 = 20
Sol. (b) ⇒ 6 x 2 + 4 y 2 + 11 xy − x + 2y − 22 = 0
Let P(6 cosθ, 3 3 sinθ) be any point on the ellipse ⇒ 6 x 2 + 11 xy + 4 y 2 − x + 2y − 22 = 0.
x2 y2
+ = 1. The equation of the tangent at
36 27 74. If a perpendicular drawn through the vertex
x
P(6 cosθ, 3 3sinθ) is cosθ +
y
sinθ = 1. O of the parabola y 2 = 4 ax to any of its
6 3 3 tangent meets the tangent at N and the
⇒ 3 3x cosθ + 6 y sinθ − 18 3 = 0 …(i) parabola at M, then ON⋅OM =
The product of the lengths of the perpendiculars [22 April 2018, Shift-II]
from (3, 0) and (− 3, 0) an Eq. (i) is given by (a) 4a2 (b) 3a2 (c) 2 a2 (d) a2
Conic Sections 271

Sol. (a) Sol. (b)


Let a point P(at 2 , 2at) over the parabola y 2 = 4ax. Equation of given ellipse
So, equation of tangent at point P is 9 x 2 + 4 y 2 − 18 x − 8 y − 23 = 0
Y ⇒ 9(x − 1)2 + 4(y − 1)2 = 36
2
y =4ax (x − 1)2 (y − 1)2
P ⇒ + =1
4 9
N Equation of latus rectum is
X 5
O ⇒ y −1 = ± 3 ⇒ y =1 ± 5.
3
77. The equation of the tangent of the ellipse
M
4 x 2 + 9 y 2 = 36 at the end of the latusrectum
lying in the second quadrant, is
yt = x + at 2 …(i)
[22 April 2018, Shift-II]
Q NM is normal to the tangent Eq. (i) and passes
(a) 5 x − 3 y + 1 = 0 (b) x − 3 y + 5 = 0
through origin ‘O’ so equation of line NM is
(c) 5 x − 3 y + 3 = 0 (d) 5 x − 3 y + 9 = 0
y = − tx …(ii)
 at 2
Sol. (d)
at 3 
So, points N ≡  − ,  Equation of given ellipse is
 1+ t 1+ t 
2 2
x2 y2
4 x 2 + 9 y 2 = 36 ⇒ + =1
M ≡  2 , − 
4a 4a 9 4
and
t t  Now, coordinate of end of the latus rectum lying
in the second quadrant is P  − 5,  .
a 2t 4 a 2t 6 16a 2 16a 2 4
So, ON ⋅ OM = + × + 2  3
(1 + t )2 2
(1 + t )2 2
t4 t
So, the equation of tangent at point P is
4a 2t 2 1
= 1 + t2 + 1 = 4a 2. − 4 5x + 12y = 36
t(1 + t 2) t2
⇒ 5x − 3y + 9 = 0
75. Let α1 and α 2 be the ordinates of two points
78. If the product of the lengths of the
A and B on a parabola y 2 = 4 ax and let α 3 be
perpendiculars from any point on the
the ordinate of the point of intersection of its hyperbola 16 x 2 − 25 y 2 = 400 to its
tangents at A and B. Then, α 3 − α 2 =
[22 April 2018, Shift-II]
asymptotes is p and the angle between the
θ
(a) α 3 − α1 (b) α 3 + α1 two asymptotes is θ, then p tan =
(c) α1 (d) α1 − α 3 2
[22 April 2018, Shift-II]
Sol. (d) 400 320 4 25
Ordinate of point of intersection of tangents at A (a) (b) (c) (d)
41 41 5 16
α + α2
and B whose ordinates are α1 and α 2 is 1 ,
2 Sol. (b)
α + α2 Equation of given hyperbola is
So, α3 = 1
2 16 x 2 − 25y 2 = 400
2α 3 = α1 + α 2 x 2 y2
⇒ − =1 …(i)
⇒ α 3 − α 2 = α1 − α 3. 25 16
4
76. The equations of the latusrectum of the and equation of asmptotes is y = ± x
ellipse 9 x + 4 y − 18 x − 8 y − 23 = 0 are
2 2 5
⇒ 4 x + 5y = 0 and 4 x − 5y = 0 …(ii)
[22 April 2018, Shift-II]
Let a point on hyperbola (i), A(5secα , 4 tanα), so,
(a) x = − 1 ± 5 (b) y = 1 ± 5
lengths of the perpendiculars from point A to the
2 5
(c) x = 1± (d) y = 2 ± 5 asymptotes is
3
272 AP EAMCET Chapterwise Mathematics

20secα + 20 tanα 20secα − 20 tanα Equation of directrix,


and
16 + 25 16 + 25 X=−a
400 θ 1 −1
So, P= and tan =
4 ⇒ x + 2= − ⇒ x = −2
41 2 5 2 2
θ −5
So, P tan =
320 ⇒ x= ⇒ 2x + 5 = 0
2 41 2
Equation of axis,
79. For the parabola y 2 + 6 y − 2 x + 5 = 0, match Y =0
the items in List-I with the suitable item in ⇒ y + 3= 0
List-II given below :
80. If 5 x − 2 y + k = 0 is a tangent to the parabola
List-I List-II
y 2 = 6 x , then their point of contact is
(I) Vertex (A)  − 3 , − 
 3
 2  [23 April 2018, Shift-I]
(a)  ,  (b)  ,  (c)  ,  (d)  , 
6 6 6 6 6 6 6 6
(II) Focus (B)  3 , − 
 3  5 5  5 25   25 5   25 25 
2 
(III) Equation of the directrix (C) 2 x + 5 = 0 Sol. (c)
(IV) Equation of the axis (D) 2 x + y + 3 = 0 Given curve
y2 = 6 x
(E) y + 3 = 0 difference w.r.t ‘x’
(F) ( − 2, − 3) dy dy 3
⇒ 2y =6 ⇒ = … (i)
dx dx y
The correct matching is [23 April 2018, Shift-I]
Given equation of tangent is 5x −2y + k = 0
I II III IV I II III IV 5 3 5 6
(a) F A E C (b) F A C E Slope = ⇒ = ⇒ y =
2 y 2 5
(c) A B C D (d) F A C D
Substitute value of y in y 2 = 6 x
Sol. (b) 2
Parabola, y 2 + 6 y − 2x + 5 = 0  6  = 6 x ⇒ 36 = 6 x ⇒ x = 6
 
 5 25 25
⇒ y 2 + 6 y = 2x − 5
Hence, point of contact is  , 
6 6
⇒ y + 6 y + 9 = 2x − 5 + 9
2
 25 5
⇒ (y + 3)2 = 2(x + 2)
⇒ Y 2 = 2X 81. If S and S′ are the foci of an ellipse, B is one
end of the minor axis and ∠SBS′ = 90 °, then
(where, Y = y + 3 and X = x + 2) the eccentricity of that ellipse is
This parabola is in the form of y 2 = 4ax by comparing [23 April 2018, Shift-I]
1 3 7 1 1
4a = 2 ⇒ a = (a) (b) (c) (d)
2 2 4 2 2
Vertex = (0, 0)
Sol. (c)
⇒ x + 2 = 0 and y + 3 = 0
S and S1 are the foci of an ellipse
⇒ x = − 2 and y = − 3
∴Coordinate of foci = (± ae , 0)
So, vertex (− 2, − 3)
Y
Focus = (a , 0) B (0, b)

(x + 2, y + 3) =  , 0
1
2 
⇒ x + 2 = 1 / 2 and y + 3 = 0 X′
O
X
3 S(–ae,0) S′(ae, 0)
⇒ x=− ,y=−3
2
 3 
So, focus  − , − 3 .
 2  Y′
Conic Sections 273

b −b 1 16 25 5
Slope of SB = Slope of S1 B = ⇒ = ⇒ e2 = ⇒ e=
ae ae e 2 25 16 4
Now, ∠SBS ′ = 90 Hence, eccentricity of conjugate hyperbola =
5
So, slope of SB × slope of S ′ B = − 1 4
b  b b2 84. If a parabola passess through the points
⇒ ×  −  = −1 ⇒ 2 2 = 1 ⇒ b 2 = a 2e 2
ae  ae  a e (−2 , 1), (1, 2) and (−1, 3) having horizontal axis,
We know that, then the length of the latus rectum of that
b 2 = a 2(1 − e 2) parabola is [23 April 2018, Shift-II]
5 2 1
⇒ a 2e 2 = a 2(1 − e 2) ⇒ e 2 = 1 − e 2 (a) 5 (b) (c) (d)
2 5 5
1
⇒ 2e 2 = 1 ⇒ e = Sol. (c)
2
Let the equation of parabola having horizontal
82. The points of intersection of the axis, vertex at (h, k) is
perpendicular tangents drawn to the ellipse (y − k)2 = 4a(x − h) … (i)
4 x 2 + 9 y 2 = 36 lie on the curve.
Q Parabola (i) passes through the points (− 2, 1),
[23 April 2018, Shift-I] (1, 2) and (− 1, 3,
) so
(a) x2 + y2 = 13 (b) x2 − y2 = 5 (k − 1)2 = 4a(− 2 − h)
x2 y2 ⇒ k2 − 2k + 1 = − 8a − 4ah
(c) x + y = 5 (d) + =1 …(ii)
9 4
⇒ (k − 2) = 4a(1 − h)
2

Sol. (a) ⇒ k2 − 4k + 4 = 4a − 4ah … (iii)


We have, ellipse
and (3 − k) = 4a(− 1 − h)
2
4 x 2 + 9 y 2 = 36
⇒ k2 − 6k + 9 = − 4a − 4ah … (iv)
x2 y2 x2 y2
⇒ + =1 ⇒ 2 + 2 =1 From Eqs. (ii), (iii) and (iv), we get
9 4 3 2
2
So, a 2 = 9 and b 2 = 4. 4a = .
5
The points of intersection of the perpendicular
tangents lie on director circle. 85. The equation of one of the common tangents
Equation of director circle is of the circle x 2 + y 2 − 6 y + 4 = 0 and the
x 2 + y2 = a 2 + b 2 ⇒ x 2 + y2 = 9 + 4 parabola y 2 = x is [23 April 2018, Shift-II]
⇒ x + y = 13
2 2
(a) 2 x − y + 1 = 0 (b) 2 x − y = 1
(c) 4 x − y + 1 = 0 (d) x − 2 y + 1 = 0
5
83. If the eccentricity of a hyperbola is , then Sol. (d)
3
the eccentricity of its conjugate hyperbola is Let the equation of tangent to the parabola y 2 = x,
[23 April 2018, Shift-I] having slope m is,
5 5 5 8 1
(a) (b) (c) (d) y = mx + … (i)
3 4 2 5 4m
Sol. (b) For common tangent to the circle
5 x 2 + y 2 − 6 y + 4 = 0 … (ii)
Eccentricity of given hyperbola =
3 1
3−
Let, eccentricity of its conjugate hyperbola = e 4m
∴ Radius 9 − 4 =
Q relationship between  1 + m2
1 1  
So, + = 1 1 1
 5
2
e2  eccentricity is 2 + 2 = 1 ⇒ ( + m2) = 9 +
51
1

3
⇒ m=
1
   e1 e2  16m2 2m 2
 3
So, equation of common tangent is
9 1 1 9
⇒ + =1 ⇒ 2 =1 − 1 1
25 e 2 e 25 y = x + ⇒ x − 2y + 1 = 0.
2 2
274 AP EAMCET Chapterwise Mathematics

π π π
= cos or cos −  ⇒ θ = ±
86. The equation of the ellipse having a vertex at 1
⇒ cos 2θ =
3 2 3  3 6
(6 , 1), a focus at (4, 1) and the eccentricity is
5
[23 April 2018, Shift-II]
88. A tangent to the curve
( x − 1)
2
( y − 1) 2
( x − 1) ( y − 1)
2 2 9 b2 x 2 − 4 a 2 y 2 = 36 a 2 b2 makes intercepts of
(a) + = 1 (b) + =1
16 25 25 16 unit length on each of the coordinate axes,
( x + 1)2 ( y + 1)2 ( x + 1)2 ( y + 1)2 then the point (a , b) lies on
(c) + = 1 (d) + =1 [23 April 2018, Shift-II]
25 16 16 25
(a) x2 − y2 = 1 (b) x2 + y2 = 1
Sol. (b)
(c) 4 x2 − 9 y2 = 1 (d) 4 x2 + 9 y2 = 1
Let the equation of ellipse is,
(x − h)2 (y − k)2 Sol. (c)
+ =1 … (i)
a2 b2 Equation of given curve is,
Now, x2 y2
a − ae = 2 2
− 2 =1 … (i)
4a 9b

a 1 −  = 2 ⇒ a = 5
3
⇒ Let at point (2a secθ, 3b tanθ) on the curve (i).
 5
So, equation of tangent at point is
So, b=4
x y
Now, vertex comparing the vertex, we are getting + =1
2a 3b

6 − h = 5⇒ h = 1 secθ tanθ
and 1 − k = 0⇒k =1 According to the question,
So, equation of required ellipse is 2a = secθ and 3b = − tanθ
(x − 1)2 (y − 1)2
+ = 1. So, 4a 2 − 9b 2 = 1
25 16
On taking locus of point (a , b), we are getting
87. If the tangent at the point
 16  4 x 2 − 9 y 2 = 1.
 4 cos 2θ , sin 2θ on the ellipse
 11  89. If the double ordinate of the parabola y 2 = 8 x
16 x + 11 y = 256 touches the circle
2 2
is of length 16, then the angle subtended by
x 2 + y 2 − 2 x = 15, then θ = it at the vertex of the parabola is
[23 April 2018, Shift-II] [24 April 2018, Shift-I]
π π π π π π 3π π
(a) ± (b) ± (c) ± (d) ± (a) (b) (c) (d)
3 6 4 8 2 3 4 4

Sol. (b) Sol. (a)


Equation of tangent at point  4 cos 2θ, sin 2θ on
16 Let A(x1 , y1) and B(x1 , – y1) are the co-ordinate of
 11  end points of double ordinate length of
the ellipse16 x 2 + 11 y 2 = 256 is AB = 2y1 = 16
y1 = 8
16 x(4 cos 2θ) + 11 y sin 2θ = 256
16
 11 
A y2=8x
y
⇒ 4 x cos 2θ + 11 y sin 2θ = 16 …(i)
Since, tangent (i) is tangent to the circle
x 2 + y 2 − 2x − 15 = 0, so α
x
(0, 0) α
| 4 cos 2θ − 16 |
=4
16 cos2 2θ + 11sin2 2θ
⇒ cos2 2θ − 8 cos 2θ + 16 = 11 + 5cos2 2θ B

⇒ 4 cos 2θ + 8 cos 2θ − 5 = 0.
2
A(x1 , 8) and B(x1 , − 8)
Conic Sections 275

A and B lies on parabola. So, 8 2



64 = 8 x1 ⇒ x1 = 8 m2 − m1
tanθ = = 3 3
So, coordinate of A and B are (8, 8), (8, − 8) 1 + m1 m2 1 + 8 ⋅ 2
8 3 3
⇒ tanα = = 1 6 / 3 18
8 tanθ = =
25 25
π
α= (Angle with X-axis made by OA) 9
4
18
Angle subtended by double ordinate AB at vertex So, 5 tanθ = 5⋅
= 2α = 2⋅ π / 4 = π / 2. 25
18
90. If P and the origin are the points of = 5⋅ = 3 2.
5
intersection of the parabolas y = 32 x and
2

2 x 2 = 27 y ; and if θ is the acute angle 91. If the latusrectum of an ellipse subtends a


right angle at the centre of that ellipse, then
between these curves at P, then 5 tanθ = the eccentricity of that ellipse is
[24 April 2018, Shift-I] [24 April 2018, Shift-I]
(a) 2 (b) 2 3 5+1 5 −1
(c) 3 2 (d) 3 (a) (b)
4 2
Sol. (c) 10 − 2 5 10 + 2 5
Points of intersection of curves (c) (d)
5 5
y 2 = 32x … (i)
Sol. (b)
and 2x 2 = 27 y … (ii)
x2 y2
Let equation of ellipse + 2 =1
From Eqs. (i), and (ii), we get a 2
b
2
 y2 
2⋅   = 27 y L
 32
2⋅ y 4 = 27 ⋅ 32⋅ 32⋅ y θ
C S
y = 0, y 3 = 512⋅ 27
⇒ y = 24 L′
From Eq. (i), we get
Let LL′ is latusrectum, then co-ordinate of
x=0
 b2 
⇒ x = 18 L  ae , 
 a
So, coordinate of P(18, 24).
LL′ subtend π /2 Angle at the centre, so angle
Equation of tangent through point P(18, 24) on the LCS = π / 4
curve y 2 = 32x
b2
⇒ y ⋅ 24 = 16(x + 18) π
⇒ tan = a
⇒ 3y = 2x + 36 4 ae
∴ Slope m1 = 2 / 3 b2
⇒ 1= 2
Again equation of tangent through point P(18, 24) a e
on the 2x 2 = 27 y ⇒ a 2e = b 2 = a 2(1 − e 2)
(y + 24)
⇒ 2x ⋅18 = 27 ⇒ e = 1 − e2
2
⇒ 8 x = 3y + 72 ⇒ e + e −1 = 0
2

⇒ 3y = 8 x − 72 −1 ± 5
⇒ e=
Slope, m2 = 8 / 3 2
Angle between these curve θ = Angle between 5 −1
tangents drawn from points P ⇒ e= .
2
276 AP EAMCET Chapterwise Mathematics

92. If the tangent at the point (1, 2) on the 93. If the equation of one asymptote of the
ellipse 3 x 2 + 4 y 2 = 19 is also a tangent to the hyperbola 14 x 2 + 38 xy + 20 y 2 + x − 7 y − 91 = 0
parabola y 2 − kx = 0 , then k = is 7 x + 5 y − 3 = 0 , then the other asymptote is
[24 April 2018, Shift-I] [24 April 2018, Shift-I]
57 − 57 57 − 57 (a) 2 x − 4 y + 1 = 0 (b) 2 x + 4 y + 1 = 0
(a) (b) (c) (d)
16 64 64 16 (c) 2 x − 4 y − 1 = 0 (d) 2 x + 4 y − 1 = 0
Sol. (d) Sol. (b)
Equation of tangent at the point (1, 2) on the Given hyperbola,
ellipse 3x 2 + 4 y 2 = 19 is 14 x 2 + 38 xy + 20 y 2 + x − 7 y − 91 = 0 …(i)
3⋅ x ⋅1 + 4 ⋅ y ⋅ 2 = 19
On factorising 14 x + 38 xy + 20 y , we get
2 2
3x + 8 y = 19 …(i)
This is also tangent to the parabola = (7 x + 5y) (2x + 4 y)
y 2 − kx = 0 …(ii) One of the asymptote is 7 x + 5y − 3 = 0
From Eqs. (i) and (ii), we get Then, let other asymptote is 2x + 4 y + k = 0
19 − 8 y  So, on combining
y 2 − k  =0
 3  (7 x + 5y − 3) (2x + 4 y + k) = 0 …(ii)
⇒ 3y 2 + 8ky − 19k = 0 has equal roots, so On equating the coefficient of x from Eqs. (i) and
(ii), we get
∴ D = (8k)2 − 4(3) (− 19k) = 0
7k − 6 = 1
⇒ 64k2 = − 4 ⋅ 3⋅19k
⇒ k =1
− 57
⇒ k= . So, other asymptote is, 2x + 4 y + 1 = 0.
16
21
Vector Algebra
π vectors of B and C are respectively 3 $i − 2 $j + k$
1. Find|a × b|2, if| a | = 2,| b | = 3 and (a , b) =
6 and 5 $i + $j − 3 k$ , then
[17 Sep. 2020, Shift-I]
AB ⋅ AC + BA ⋅ BC + CA ⋅ CB =
(a) −9 (b) 9 (c) 3 (d) − 3 [17 Sep. 2020, Shift-I]
Sol. (b) (a) 28 (b) 29 (c) 27 (d) 25
2
|a × b|2 = [|a||b|sinθ]2 =  2 × 3 ×  = 9
1 Sol. (b)
 2 $
Given, B = 3$i − 2$j + k
2. If a = α i$ + 3$j − 6 k$ and b = 2$i − $j + βk$ , then $
C = 5$i + $j − 3k
the values of α , β so that a and b may be $
BC = 2$i + 3$j − 4k
collinear are [17 Sep. 2020, Shift-I]
max {AB, BC , AC} = BC
(a) (5, 3) (b) (6, 2) (c) (2, − 6) (d) (−6, 2) ∴BC is hypotenuse of ∆ABC
Sol. (d) ∠A = 90°
$ , b=2$i − $j + βk
a=α$i + 3$j − 6k $ ∴ AB ⋅ AC = 0
For (a) and (b) may be collinear (a × b) = 0 BA ⋅ BC = |BA||BC|cos B
$i $j k $ CA ⋅ CB = |CA||CB| cos C
⇒ α 3 −6 = 0 ∴AB − AC + BA − BC + CA − CB
2 −1 β = 0 + |BC|(|BA| cos B + |CB| cos C )
= 0 +|BC ||BC | [Q By projection formula]
$ (−α − 6) = 0
⇒ $i(3β − 6) − $j(αβ + 12) + k =|BC|2− ( (2)2 + 32 + 42)2 = 4 + 9 + 16 = 29
⇒ (3β − 6) = 0, (αβ + 12) = 0 and (α + 6) = 0
⇒ ( β = 2), (α = −6)
5. If a + b + c = 0 and| a | = 3,| b | = 5,| c | = 7,
then the angle between a and b is ........
3. Let a , b , c be three vectors [17 Sep. 2020, Shift-I] [17 Sep. 2020, Shift-II]
(i) (a × b) × c = (a ⋅ c) b − (b ⋅ c) a (a) 30° (b) 120° (c) 90° (d) 60°
(ii) (a) × (b × c) = (a ⋅ b)c − (a ⋅ c) b Sol. (d)
(a) (i) is incorrect, (ii) is correct
Given, a + b + c = 0 ⇒ (a + b) = − c
(b) (i) is correct, (ii) is incorrect
(c) Both (i) and (ii) are correct ⇒ (a + b)2 = c 2 ⇒ a 2 + b2 + 2a ⋅ b = c 2
(d) Both (i) and (ii) are incorrect ⇒ 2a ⋅ b = c 2 − a 2 − b2
Sol. (b) As square of a vector = square of its magnitude,
(i) is correct and (ii) is incorrect. ⇒ 2a ⋅ b = 49 − 9 − 25 ⇒ 2a ⋅ b cosθ = 15
⇒ θ = cos−1   = 60°
15 1
4. ABC is a right-angled triangle in which ⇒ cosθ =
2× 3× 5  2
max { AB, BC , AC } = BC . If the position
278 AP EAMCET Chapterwise Mathematics

6. If a, b, c and r are vectors such that a is not Following triangle came,


perpendicular to b ⋅ r × b = c × b and r ⋅ a = 0 AB + BC = AC
then r = [17 Sep. 2020, Shift-II] ⇒ AB + BC + CA = 0 …(i)
(c ⋅ a ) (c ⋅ a ) and AB + BC − AC = 0 …(ii)
(a) c + b (b) b − c
(b ⋅ a ) (b ⋅ a ) Also, CA + AB = CB
(c ⋅ a ) (c ⋅ a ) ⇒ CA + AB − CB = 0
(c) c − b (d) b + c
(b ⋅ a ) (b ⋅ a ) or AB − CB + CA = 0 …(iii)
Sol. (c) 8. If the vectors ai$ + $j + k$ , i$ + b$j + k$ and
Given, r × b = c × b ⇒ (r − c) × b = 0 $i + $j + ck$ are coplanar, where (a , b, c ≠ 1), then
⇒ r − c is parallel to b. 1 1 1
⇒ (r − c) = λ b or r = c + λ b …(i) the value of + + =
1− a 1− b 1− c
Also,
[17 Sep. 2020, Shift-II]
r ⋅ a = 0 ⇒ (c + λb) ⋅ a = 0
(a) 2 (b) 0 (c) − 1 (d) 1
⇒ c ⋅ a + λb ⋅ a = 0
c⋅a  Sol. (d)
or λ = −   …(ii)
b⋅a  For coplanar vectors,
So, from Eq. (i) we substitute ‘λ’ from Eq. (ii), we a ⋅|b × c| = 0
get ax a y az
c⋅a 
r = c −  b
⇒ bx by bz = 0
b⋅a  cx cy cz
7. In a ∆ABC (shown in the figure below). State a 1 1
whether the following are true or false Substituting values we have, 1 b 1 = 0
[17 Sep. 2020, Shift-II]
1 1 c
C R2 → R2 − R1 and R3→ R3 → R1
a 1 1
⇒ 1 − a b −1 0 =0
1− a 0 c −1
⇒ a (b − 1) (c − 1) − (1 − a) (c − 1) − (1 − a) (b − 1) = 0
A B ⇒ Dividing by (1 − a) (1 − b) (1 − c), we get
a 1 1
(i) AB + BC + CA = 0 + + =0
1 − a 1− b 1− c
(ii) AB + BC − AC = 0
1 1 1 1 a
(iii) AB − CB + CA = 0 ⇒ + + = − =1
1− a 1− b 1− c 1− a 1− a
(iv) AB + BC − CA = 0
(a )(i) True (ii) True (iii) True (iv) False 9. Let u, v and w be three vectors in R 3. Then,
(b) (i) True (ii) False (iii) True, (iv) False any vector Z ∈R 3 can be written as
(c) (i) False (ii) False (iii) True (iv) True
z = au + bv + cw for some scalars a, b and c if
(d) (i) False (ii) True (iii) False (iv) False
and only if [17 Sep. 2020, Shift-II]
Sol. (a) (a) Each pair of u, v and w are not parallel
C (b) Each of u, v and w can be written as a linear
combination of the other two
(c) All have different magnitude and directions
(d) None of the options are correct
Sol. (d)
As given vector u, v and w given may be not
A B parallel but they may be antiparallel
Vector Algebra 279

So, z ≠ a u + b v + c w Sol. (c)


So first is incorrect. If in a ∆ABC, AB = u and AC = v and D is
Also, if u = v + w mid-point of B and C, then
v=u+ w AB + AC u + v
AD = = .
w=u+ v 2 2
{mid-point formula}
Then, u + v + w = 2(u + v + w)
⇒ u + v + w = 0≠ z 13. Let u, v v and w be three vectors such that
So, option (b) is incorrect. u + v + w = 0,| u | = 3,| v | = 5 and| w | = 7.
Similarly, option (c) is incorrect. Then the angle between u and v is
[18 Sep. 2020, Shift-I]
10. If PO + OQ = QO + OR , then .......
(a) 60° (b) 70° (c) 80° (d) 90°
[17 Sep. 2020, Shift-II]
Sol. (a)
(a) Q is the mid-point of PR
Given that,|u|= 3,|v|= 5 and |w|= 7
(b) Q divides PR in 2 : 1
(c) Q divides PR in 1 : 2 and u + v + w = 0 ⇒ u + v = − w
(d) Q divides PR in − 1 : 2 ⇒ |u|2 + |v|2 + 2 u. v = |w|2
Sol. (a) ⇒ 9 + 25 + 2 × 3 × 5 cosθ = 49,
From diagram below, {if angle between u and v is θ}
1
P Q R ⇒ cos θ = ⇒ θ = 60°
2
14. If α is the angle between two vectors
p = 3 i$ + 4 $j − k$ and q = 2 $i − $j + k$ , then
sin(α) = [18 Sep. 2020, Shift-I]
145 135
(a) (b)
156 156
O 155 165
(c) (d)
Using triangle law, 156 156
PO + OQ = QP and QO + OR = QR Sol. (c)
Given; PO + OQ = QO + OR ⇒ QP = QR If α is an acute angle between vectors p and q, then
⇒ |QP| = |QR| ⇒ QP length = QR length |p × q| (4 − 1)2 + (−3 − 2)2 + (−3 − 8)2
∴ Q is mid point of PR Sin α = =
|p||q| 9 + 16 + 1 4 + 1 + 1
11. Let u and v be two non-zero vectors. Then 9 + 25 + 121 155
the magnitude of the cross product u × v is = =
always [18 Sep. 2020, Shift-I] 156 156
(a) <| u || v | (b) =| u || v | 15. Equation of the perpendicular bisector of the
(c) >| u || v | (d) = 0 line joining the points whose position vectors
Sol. (a) are a and b respectively is [18 Sep. 2020, Shift-I]
The cross product of u and v non-zero vectors is (a) (2r − a − b) ⋅(a − b) = 0
u × v =|u||v|sinθ n $ , if angle between them is θ. (b) (2r − a − b) ⋅(a + b) = 0
So, |u × v|=|u||v||sinθ| (c) (2r + a + b) ⋅(a − b) = 0
(d) (2r − a + b) ⋅ (a + b) = 0
⇒ |u × v|≤ |u||v| ∴|sin θ|≤ 1
Sol. (a)
12. Let ABC be a triangle. Let u = AB and v = AC.
The mid-point of line joining points whose
If D is a middle point of BC, then AD = a + b
position vectors are a and b is M   and the
[18 Sep. 2020, Shift-I]  2 
u−v v −u u+ v
(a) (b) (c) (d) u + v direction ratio vector of line joining of given
2 2 2 points is (a − b).
280 AP EAMCET Chapterwise Mathematics

P (r ) $
(2 + λ) $i +6j$ – 2k
=
(2 + λ)2 + 36 + 4
$
(2 + λ) $i +6j$ – 2k
A (a) M a+b B (b ) d =
2 λ2 + 4λ + 44
a$ × d$
$i $j $
k
Let a variable point p(r) on the perpendicular
bisector of AB, so = 1 1 1
MP ⊥ BA (2 + λ) 6 −2

⇒  r − a + b  ⋅ (a − b) = 0 λ2 + 4λ + 44 λ2 + 4λ + 44 λ2 + 4λ + 44
 
 2  $i $j k$
⇒ (2r − a − b) ⋅ (a − b) = 0 1
= 1 1 1
16. If a = 2i$ + $j − 3k$ and b = 3$i − $j + 2k$ , then λ + 4λ + 44 2 + λ 6 −2
2

find the angle between the vectors 2a + b 1 $ (6 − 2 − λ)


and a + 2 b. [18 Sep. 2020, Shift-I] = $i(−8) − $j(−2 − 2 − λ) + k
λ2 + 4λ + 44
 36   72 
(a) cos − 1   (b) cos − 1   a$ × d$ =
1 $
− 8$i + (4 + λ) j + (4 − λ) k
 42 × 35   24 × 32 
λ + 4λ + 44
2
 52   24 
(c) cos − 1   (d) cos − 1   $
a ×d =
$ 1
64 + (4 + λ)2 + (4 − λ)2
 74 × 65   18 × 32  λ + 4λ + 44
2

Sol. (*) 64 + 16 + λ2 + 16 + λ2
$ and b = 3$i − $j + 2k
$ 2=
Given vectors a = 2$i + $j − 3k λ2 + 4λ + 44
So, 2a + b = 7$i + $j − 4k $ Squaring on both sides,
a + 2b = 8$i − $j + k $ 2λ2 + 96
and 2= 2
λ + 4λ + 44
Let angle between vectors (2a + b) and (a + 2 b), so
$ ) . (8$i – $j + k
$ 2λ2 + 8λ + 88 = 2λ2 + 96 ⇒ 8λ = 8 ⇒ λ = 1
(7$i + $j − 4k
θ = cos−1 Hence, option (b) is correct.
49+1+16 64 + 1 + 1
18. Find ‘λ’ if a, b, c are three non-coplanar
56 − 1 − 4
= cos−1  
51
= cos−1 vectors such that
66 66  66 
[4a + 3b − c 4a + 3b + 2 c a − 4b − c ]
(*) No option is correct. = (λ2 + λ + 1) [a b c ] [18 Sep. 2020, Shift-II]
17. If the magnitude of the vector product of the (a) − 7, 8 (b) − 7, − 6 (c) 7, − 8 (d) − 7, − 8
vector $i + $j + k$ with a unit vector along the Sol. (c)
sum of the vectors 2 $i + 4 $j − 5 k$ and 4 3 −1
4 3 2 [a b c] = (λ2 + λ + 1)[a b c]
λ$i + 2 $j + 3 k$ is equal to 2, then the value of
1 −4 −1
‘λ’ is [18 Sep. 2020, Shift-II]
4(5) − 3(−6) − 1(−19) = λ2 + λ + 1
(a) − 1 (b) 1 (c) 0 (d) 2
57 = λ2 + λ + 1
Sol. (b)
$ ⇒ λ + λ − 56 = 0 2
Let a = $i + $j+ k
⇒ (λ − 7)(λ + 8) = 0 ⇒ λ = 7 (or) −8
$
b = 2$i + 4$j – 5k Hence, option (c) is correct.
c = λ$i + 2$j+3k $
19. In a regular hexagon ABCDEF,
$
b + c = (2 + λ)$i + 6$j – 2k AD + EB + FC = (3 λ − 8)AB. Then λ =
b+ c [18 Sep. 2020, Shift-II]
d$ = unit vector along (b + c) =
|b + c| (a) 3 (b) 4 (c) 5 (d) 6
Vector Algebra 281

Sol. (b) Sol. (b)


Since, AD||BC and AD = 2BC A vector which is coplanar to the vectors
2$i + $j + k$ and $i + $j + k $ is
EB||FA and EB = 2FA
FC||AB and FC = 2AB $ ) + b($i − $j + k
a (2$i + $j + k $)
AD + EB = 2(BC + FA) = 2(AO + FA) … (i) (2a + b) $i + (a − b)$j + (a + b) k$ … (i)
In ∆ AOF, Eq. (i) is orthogonal to 3$i + 2$j + 6k $
FA + AO + OF = 0
FA + AO = − OF … (ii) 3(2a + b) + 2(a − b) + 6(a + b) = 0
Put, Eqs. (ii) in (i) 14 a + 7b = 0
AD + EB = 2(− OF) = 2FO 2a + b = 0
Required unit vector,
AD + EB = 2AB
$
(2a + b) $i + (a − b) $j + (a + b) k
Now consider, =±
AD + EB + FC = 2AB + 2AB [∴ FC = 2AB] (2a + b) + (a − b) + (a + b)2
2 2

(3λ − 8)AB = 4AB [∴ given] $


3a$j − ak 3$j − k $
=± =±
3λ − 8 = 4 ⇒ 3λ = 12 ⇒ λ = 4 (3a) + a
2 2 10
Hence, option (b) is correct.
Hence, option (b) is correct.
20. Two u and v are parallel if and only if
[18 Sep. 2020, Shift-II]
23. If a = 2i$ − 3$j − 4 k$ , b = $i + 4 $j − 2k$ ,
(a) u and v have the same direction c = 3 $i − $j + 4 k$ then, [a × b b × c c × a ] =
(b) u and v have the opposite direction [21 Sep. 2020, Shift-I]
(c) One of them is a scalar multiple of the other (a) 4900 (b) 6400 (c) 8100 (d) 12100
(d) The dot product of u and v are zero
Sol. (d)
Sol. (c)
Given vector
If two vectors are parallel then one is scalar $
a = 2$i − 3$j − 4k
multiple of others.
b = $i + 4$j − 2k$ and c = 3$i − $j + 4k
$
Hence, option (c) is correct.
and as we know that
21. Let u and v be two non zero vectors, if
[a × b b × c c × a ] = [a b c]2
| u + v | = | u − v |, then [18 Sep. 2020, Shift-II]
2 −3 −4
(a) u and v have the same direction
(b) u and v are perpendicular and [a b c] = 1 4 −2
(c) u and v have the opposite direction 3 −1 4
(d) Data Insufficient = 2(16 − 2) + 3 (4 + 6) − 4 (− 1 − 12)
Sol. (b) = 28 + 30 + 52 = 110
u+v = u – v ∴[a × b b × c c × a ] = [a b c]2
Squaring on both sides, = (110)2 = 12100
2 2
u+v = u – v Hence, option (d) is correct.
2 2 2 2
u + v + 2uv = u + v + 2(uv) 24. Let u and v be two vectors in a plane. Then
4(u. v) = 0 ⇒ u. v = 0 ⇒ u ⊥ v any vector w in the plane can be written as
Hence, option (b) is correct.
w = au + bv for some scalars ‘a’ and ‘b’ if and
only if [21 Sep. 2020, Shift-I]
22. The unit vectors orthogonal to 3$i + 2 $j + 6 k$ (a) None of u and v is a scalar multiple of the other
and coplanar with 2 $i + $j + k$ and i$ − $j + k$ are (b) None of| u| and| v | is a scalar multiple of the other
(c) u and v have different direction
[18 Sep. 2020, Shift-II]
(d) u and v are perpendicular to each other
1 $ $ 1
(a) ± (2 i − k ) (b) ± (3$j − k$ )
5 10 Sol. (b)
1 1 Any vector w in the plane containing vector u and
(c) ± (2 i$ − 3$j) (d) ± (2 $i + 3$j − 2 k$ )
13 17 v can be written as
282 AP EAMCET Chapterwise Mathematics

w = au + bv for some scalars ‘a’ and ‘b’ if and only Now, equation of line segment passes through the
if u and v vectors are not parallel means none of u points u and v is
and v is a scalar multiple of the other. x −1 y + 2
= … (i)
Hence, option (b) correct. 4 −7
Now, the point vectors having position vectors
25. If a and b are unit vectors such that a + b is 1 1
OP = − $i, OQ = − $j and OR = − 2$i + 3$j are given.
also a unit vector, then the angle between a 7 4
and b is ........ [21 Sep. 2020, Shift-I]  1 
The point P  − , 0 , by putting in line (i)
(a) 60° (b) 90° (c) 30° (d) 120°  7 
Sol. (d) 1
− −1
7 2 −8 2 −2 2
For two vectors a and b, it is given a + b is also = ⇒ = ⇒ =
unit vector, so 4 −7 4 × 7 −7 7 −7
|a + b| = 1 ⇒ (a + b)2 = 1 Q Point P  − , 0 satisfied the line, so it is on the
1
 7 
⇒ |a|2 + |b|2 + 2a .b =1
line segment (i)
⇒ 1 + 1 + 2a.b = 1
The point Q 0, −  , by putting in line (i)
1
1 
⇒ a .b = − 4
2 1
− + 2
Let angle between a and b is ‘θ’ 0 −1 1 1
1 = 4 ⇒ − =−
Then cosθ = − ⇒ θ = 120° 4 7 4 4
2  1
Q Point Q  0, −  also satisfied the line (i), so it
Hence, option (d) is correct.  4
26. Let u = − 2i$ + 2$j + k$ and v = $i − 2$j + 2k$ . lies on the line segment.
Then angle between u and v is ....... Now, the point R(− 2, 3), on putting in the
−2 − 1 3 + 2
[21 Sep. 2020, Shift-I] equation of line (i) ≠
− 4 4 −7
(a) cos −1  4
  (b) cos −1   Q Point Q(− 2, 3) not satisfied the line (i), so it
 9  3 
doesn’t line on the line segment.
− 4
(c) cos −1  4
  (d) sin−1   Hence, option (a) is correct.
 3  9 

Sol. (a) 28. Let ABCDEF be a regular hexagon with the


$ and vertices A , B,C , D , E , F counterclock-wise. Then
Let angle between vector u = − 2$i + 2$j + k
the vector AB + AF + CD + EF is equal to
$ $ $
v = i − 2 j + 2k is θ, then [21 Sep. 2020, Shift-II]
|u. v| |− 2 − 4 + 2| 4 (a) DE + FA (b) CB + ED (c) BC + FA (d) BC + DE
cosθ = = =
|u||v| 4+ 4+1 1+ 4+ 4 9 Sol. (d)
θ = cos−1  
4 In a regular hexagon ABCDEF,

 9 We know that,
Hence, option (a) is correct. AB + BC + CD = AD = 2 BC
27. Let u = i$ − 2$j and v = − 3$i + 5$j. Consider A
three points P, Q and R having position B F
− 1$ − 1 $
vectors i, j and − 2 i$ + 3 $j respectively.
7 4 C E
Among these, the points in the line segment
passing through u and v are[21 Sep. 2020, Shift-I] D
(a) Only P and Q (b) Only P and R ∴ AB + CD = BC …(i)
(c) Only Q and R (d) All P, Q and R Similarly, as AF = CD …(ii)
Sol. (a) and as we know that
Given vectors u = $i − 2$j and v = − 3$i + 5$j. CD + DE + EF = CF = 2DF⇒CD + EF = DE …(iii)
Vector Algebra 283

From Eqs. (i) and (iii), we get 1


∴Area of triangle = (P × Q)
AB + CD + CD + EF = BC + DE 2
⇒ AB + AF + CD + EF = BC + DE $i $j k$
{from Eq. (ii)} 1
= | 3 5 −1 |
2
29. In quadrilateral ABCD, AB = a , BC = b , 1 2 3
AD = b − a if M is the midpoint of BC and N 1 $ (6 − 5)|
= |i$(15 + 2) − $j(9 + 1) + k
 4
is a point on DM such that DN =   DM, 2
 5 1 $ | = 1 289 + 100 + 1
then 5 AN = = |17$i − 10$j + k
[21 Sep. 2020, Shift-II] 2 2
(a) AC (b) 2 AC 1
= 390 sq. units
(c) 3AC (d) 4AC 2
Sol. (c) Hence, option (c) is correct.
It is given that for a quadrilateral 32. Given, ∆ABC such that A is 2$i − $j + k$ , B is
ABCD, i$ − 3 $j − 5 k$ and C is 3 i$ − 4 $j − 4 k$ , then ∆ABC is
AB = a , BC = b, AD = b − a [21 Sep. 2020, Shift-II]
b
∴According to the question, BM = and (a) An equilateral triangle (b) A right-angled triangle
2
  (c) An isosceles triangle (d) A scalene triangle
DN 4
=  4
QDN =   DM given
NM 1  5  Sol. (b)
4 Vertices of ∆ABC are given as
∴ AN − AD = (AM − AN) $
1 A is 2$i − $j + k
5AN = 4AM + AD = 4 a +  + (b − a)
b B is i − 3 j − 5k
$ $ $

 2 $
and C is 3$i − 4$j − 4k
= 4a + 2b + b − a = 3(a + b) = 3AC $ , BC = 2$i − $j + k
$
∴ AB = − $i − 2$j − 6k
Hence, option (c) is correct. $
and CA = $i − 3$j − 5k
30. Let u = − 2i$ + 2$j + k$ and v = $i − 2$j + 2k$ . Then
Q |AB|= 1 + 4 + 36 = 41
the component of v on u is
[21 Sep. 2020, Shift-II] |BC|= 4+1+1 = 6
4 −4 −2 2
(a) (b) (c) (d) and |CA|= 1 + 9 + 25 = 35
3 3 3 3
Q |AB|2 =|BC|2 +|CA|2
Sol. (b)
$ ∴∆ABC is an right-angled triangle.
Given vectors are u = −2$i + 2$j + k
Hence, option (b) is correct.
$
and v = $i − 2$j + 2k
33. If P divides the line segment joining the
v. u −2 − 4 + 2 −4
∴Component of v on u is = = = pointsA and B in the ratio 2 :1 and the
|u| 4+ 4+1 3
position vectors of A and B are i$ − 2 $j and
Hence, option (b) is correct.
−3 $i + 5 $j respectively, then the position vector
31. If P = 3i$ + 5$j − k$ and Q = $i + 2$j + 3 k$ are two of p is [22 Sep. 2020, Shift-I]
sides of a triangle, then its area is equal to 5$i − 8$j −5$i + 8$j
………… sq. units. [21 Sep. 2020, Shift-II] (a) (b)
3 3
390 390 390 5$i + 8$j −5$i − 8$j
(a) (b) 390 (c) (d) (c) (d)
4 2 8 3 3
Sol. (c) Sol. (b)
Sides of a triangle given as It is given that the position vectors of points A
$ and Q = $i + 2$j + 3k
P = 3$i + 5$j − k $
and B are OA = $i − 2$j and OB = − 3$i + 5$j.
284 AP EAMCET Chapterwise Mathematics

So, position vector of point P. Which divides the Let P , Q, R and S are the mid-point of sides of a
line segment joining A and B in the ratio 2 : 1 is quadrilateral ABCD respectively.
1($i − 2$j) + 2(− 3$i + 5$j) − 5$i + 8$j a +b
OP = = P = mid-point of AB =
1+ 2 3 2
b+c
Q = mid-point of BC =
34. If a and b are two unit vectors and θ is the 2
angle between them, then the unit vector c+d
along the angular bisector of a and b is given R = mid-point of CD =
2
by ……… [22 Sep. 2020, Shift-I] a +d
a +b a +b S = mid-point of AD =
(a) (b) 2
2 sin (θ /2 ) 2 cos(θ / 2 ) a +b c+d
a −b a +b +
(c) (d) Mid-point of PR = 2 2 =a +b+c+d
(2 cos θ/ 2 ) cos (θ / 2 ) 2 4
Sol. (b) a +b c+d
+
Since a and b are unit vectors and angle between Mid-point of SQ = 2 2
them is θ, so unit vector along the angle bisector of 2
a and b is p = λ(a + b) where | p | = 1. a +b+c+d
=
θ 4
Now, since p and a are inclined at angle , so ∴E = mid-point of PR = mid-point of SQ
2
θ a +b+c+d
p ⋅ a = | p ||a | cos ∴ OE =
2 4
θ 4OE = a + b + c + d
⇒ λ(a + b) ⋅ a = 1 × 1 cos
2 ∴OA + OB + OC + OD = 4 OE
θ x OE = 4OE ⇒ x = 4
⇒ λ(1 + cos θ) = cos {Qa ⋅ b = 1 × 1 cos θ = cos θ}
2 Hence, option (a) is correct.
2 θ θ 1 a +b
⇒ λ 2 cos = cos ⇒ λ =
θ
⇒p =
θ
37. Let u and v be two vectors in R 2. If
2 2 2 cos 2 cos
2 2 |u + v|2 = 2(| u|2 + |v|2 ) , then ……
[22 Sep. 2020, Shift-II]
35. If a , b and c are three vectors such that
(a) u = v
a × b = c , b × c = a and a , b , c are mutually (b) u and v need not be same but they have same
perpendicular to each other, then|b| is equal direction
to [22 Sep. 2020, Shift-I] (c) u and v need not be same but they have the
(a) −1only (b) 0 only (c) 1 only (d) ± 1 opposite direction
Sol. (c) (d) u = 2 v
It is given that a, b and c are mutually Sol. (a)
perpendicular vectors, such that a × b = c and |u + v|2 = 2(|u|2 + |v|2)
b × c = a.
|u|2 + |v|2 + 2 uv = 2|u|2 + 2|v|2
So, (b × c) × b = c
⇒ (b ⋅ b) c − (b ⋅ c) b = c |u|2 + |v|2 = 2 u. v
⇒ b ⋅ b = 1 ⇒|b |2 = 1 ⇒|b | = 1 |u|2 + |v|2 − 2u. v = 0 ⇒ (u − v)2 = 0

36. If O is any point OA + OB + OC + OD = xOE, u−v=0 ⇒ u=v


Hence, option (a) is correct.
then find x , given that ABCD is quadrilateral,
E is the point of intersection of the line 38. If a , b and c are position vectors of the
joining the mid-points of opposite sides. (a − c) × (b − a)
[22 Sep. 2020, Shift-II]
vertices of ∆ABC , then =
(b − a) ⋅ (c − a)
(a) 4 (b) 3 (c) 5 (d) 9
[22 Sep. 2020, Shift-II]
Sol. (a) (a) cotC (b) tan A (c) tanC (d) − tan A
Vector Algebra 285

Sol. (b) 3 3 3 3
=
AB + BC = (AB + BC) = AC
(a − c) × (b − a) CA × AB 2 2 2 2
=
(b − a).(c − a) AB ⋅ AC Hence, option (b) is correct.

A (a) 40. If p × q = p × r and p ⋅ q = p ⋅ r, then ………


[22 Sep. 2020, Shift-II]
(a) p = r (b) q = r (c) p = q (d) p + q = 0
Sol. (b)
p ×q = p × r
⇒ p × (q − r) = 0 … (i)
B (b) C (c ) p.q = p. r
|CA||AB|.sin A ⇒ p.(q − r) = 0 … (ii)
= = tan A From Eqs. (i) and (ii) we can say that
|AB||AC|cos A
p is neither parallel nor Perpendicular to (q − r)
Hence, option (b) is correct.
⇒ q −r=0 [Q p ≠ 0]
39. For a parallelogram ABCD, if L and M are ⇒ q=r
mid-points of BC and CD, then AL + AM = Hence, option (b) is correct.
[22 Sep. 2020, Shift-II]
 π π  π π
2 3 5 41. cos + i sin  × cos + i sin 
(a) AC (b) AC (c) AC (d) 3 AC  2 2  4 4
3 2 2
 π π
Sol. (b) × cos + i sin  ×………∞ =
 8 8 [22 Sep. 2020, Shift-II]
Given, ABCD is a parallelogram
L , M are mid points of sides BC , CD respectively (a) 1 (b) 0 (c) −1 (d) 2
Sol. (c)
D M C
 cos π + i sin π   cos π + i sin π   cos π + i sin π 
   
 2 2 4 4  8 8
L ....∞
π π π
i i i
e 2 . e 4 . e 8 ....∞
A B e i( π / 2 + π / 4 + π / 8 + ...... ∞)

1 1  π 
⇒ BL = BC ⇒ DM = DC 
i 2 

Q π + π + π + ... ∞ in GP 
2 2 1 − 1   2 
4 8
In ∆ABC, = e  2  
AB + BL = AL ∴ a = π , r = 1 ⇒ S∞ = a 
 4 2 1 − r 
1
AB + BC = AL … (i) = e i( π ) = cos π + i sin π = − 1 + i.0 = −1
2
In ∆ADM, Hence, option (c) is correct.
AD + DM = AM 42. Find the angle between the diagonals of
1
AD + DC = AM … (ii) parallelogram PQRS, if PQ = 3 $i − 2 $j + 2 k$ and
2 PS = i$ − 2 k$ [22 Sep. 2020, Shift-II]
Eqs. (i) + (ii)
3 3
1 1 (a) Only cos θ = − (b) Both cosθ = ±
⇒ AL + AM = AB + BC + AD + DC 10 10
2 2
3 11
1 1 (c) tanθ = − (d) tanθ = −
= AB + BC + BC + AB 10 10
2 2
Q AD = BC Sol. (a)
Q DC = AB $
PQ = 3$i − 2$j + 2k … (i)
 
286 AP EAMCET Chapterwise Mathematics

$
PS = $i − 2k Sol. (c)
$
Let a = 2$i − $j + k
S R
$
b = $i + 3$j − k
1
Area of parallelogram = |a × b|
2
$i $j $
k
1
= 2 −1 1
P Q 2
1 3 −1
$
Eqs. (i) + (ii) ⇒ PQ + PS = 4$i − 2$j + 0k
1 $ (6 + 1)|
PR = 4 i − 2 j
$ $ = |$i(1 − 3) − $j(−2 − 1) + k
2
$
Eqs. (i) − (ii) ⇒ PQ − PS = 2$i − 2$j + 4k 1
= |−2$i + 3$j + 7k $ | = 1 4 + 9 + 49
– QS = 2i − 2 j + 4k
$ $ $ 2 2
1 62
⇒ $
QS = − 2$i + 2$j − 4k = 62 =
2 2
Let θ be the angle between diagonal PR and QS Hence, option (c) is correct.
$)
PR . QS (4$i − 2$j )(−2$i + 2$j − 4k
∴ cos θ = = 45. Number of unit vectors of the form
|PR||QS| 16 + 4 4 + 4 + 16
a $i + b $j + c k$ , where a , b, c ∈ W is
−8 − 4 −12
= = [23 Sep. 2020, Shift-I]
20 ⋅ 24 2 5 ⋅ 2 6
(a) 2 (b) 4 (c) 3 (d) 6
−3 9 3
= =− =− Sol. (c)
30 30 10 $ is an unit vector,
It is given that a$i + b$j + ck
3
∴ cos θ = − where a , b , c ∈ W, so only one of the quantity in
10
a , b and c can be 1 and other must be zero. so
Hence, option (a) is correct. $
possible unit vectors in the form of a$i + b$j + ck
43. Let u and v be two vectors. Then $ , 0$i + $j + 0k
are $i + 0$j + 0k $ and 0$i + 0$j + k
$.
|u − v| = ||u| − |v|| if and only if
Hence, option (c) is correct.
[23 Sep. 2020, Shift-I]
(a)|u| = |v| 46. The scalar product of the vector a = i$ + $j + k$
(b) u and v have the opposite direction with a unit vector along the sum of the
(c) u and v have the same direction
vectors b = 2 $i + 4 $j − 5 k$ and c = λi$ + 2 $j + 3 k$
(d) u and v are perpendicular to each other
is equal to one. Then, λ = [23 Sep. 2020, Shift-I]
Sol. (c)
(a) −1 (b) 1 (c) −2 (d) 2
Given|u − v| =||u| − |v||
Squaring on both sides, Sol. (b)
$
Given, a = $i + $j + k
|u|2 + |v|2 − 2u. v = |u|2 + |v|2 − 2|u||v|
$
b = 2$i + 4$j − 5 k
−2|u|.|v|cosθ = −2|u||v|
cosθ = 1 c = λ$i + 2$j + 3k$
θ=0 $
b + c = (2 + λ) $i + 6 $j − 2k
∴u , v have the same direction.
b+c
Hence, option (c) is correct d=unit vector along (b + c) =
|b + c|
44. The area of the parallelogram, whose $
(2 + λ) $i + 6$j − 2k
=
diagonals are 2 $i − $j + k$ and $i + 3 $j − k$ , is
(2 + λ)2 + 36 + 4
equal to ……… sq. units [23 Sep. 2020, Shift-I] $
(2 + λ) $i + 6$j − 2k
26 2 62 2 d =
(a) (b) (c) (d)
2 26 2 62 (2 + λ)2 + 40
Vector Algebra 287

According to the question, $ ) + 3 (2$i + 3$j + 4k


6(2$i + 5$j + 7 k $)
d ⋅a = 1 6+ 3
$ ] [$i + $j + k
[(2 + λ) $i + 6$j − 2k $] $
=1 18$i + 39$j + 54k 13 $
= = 2$i + $j + 6k
(2 + λ)2 + 40 9 3
Hence, option (d) is correct.
(2 + λ) + 6 − 2 = (2 + λ)2 + 40
(2 + λ) + 4 = (2 + λ)2 + 40 49. If the position vectors of the vertices A , B and
Squaring on both sides, C of ∆ABC are $i + 2 $j − 5 k$ , −2 $i + 2 $j + k$ and
(2 + λ)2 + 42 + 2⋅ (2 + λ)4 = (2 + λ)2 + 40) 2 $i + $j − k$ respectively, then ∠ B =
16 + 8(2 + λ) = 40 [20 April 2019, Shift-I]
8(2 + λ) = 24 −1  7 
(b) cos −1 
8 
(a) cos   
2+ λ = 3 ⇒ λ =1  3 10   105 
(c) cos −1 
1 
(d) cos −1  −
Hence, option (b) is correct. 7 
 
 42   3 10 
47. If PQRST is a pentagon, then the resultant of
forces, PQ, PT , QR , SR , TS and PS is Sol. (b)
[23 Sep. 2020, Shift-I] $ and BC = 4$i − $j − 2k
Q BA = + 3$i − 6k $
(a) 3 PT (b) 3 PQ (c) 3 PS (d) 0 BA ⋅ BC
∴ cos(∠B) =
Sol. (c) |BA||BC|
Consider, 12 + 12 24
S R = =
9 + 36 16 + 1 + 4 45 × 21
24 8
= =
3 105 105
T −1  8 
⇒ ∠B = cos  
Q  105 
P Hence, option (b) is correct.
PQ + PT + QR + SR + TS + PS
(PQ + QR) + (PT + TS) + (SR + PS) 50. If the position vectors of the vertices of a
PR + PS + SR + PS ∆ABC are OA = 3 $i + $j + 2 k,
$ OB = $i + 2 $j + 3 k$
(PR + SR) + 2PS and OC = 2 $i + 3 $j + k,
$ then the length of the
PS + 2 PS = 3PS altitude of ∆ABC drawn from A is
48. If 4 i$ + 7 $j + 8 k$ , 2$i + 3 $j + 4 k,
$ 2 i$ + 5 $j + 7 k$ are [20 April 2019, Shift-I]
respectively the position vectors of the 3 3 3 3
(a) (b) (c) (d)
vertices A , B, C of ∆ABC, then the position 2 2 2 2
vector of the point where the bisector of Sol. (b)
angle A meet BC is [20 April 2019, Shift-I]
Since, length of altitude of ∆ABC drawn from A is
13 $ 13 $
(a) 2 $i + j + 2 k$ (b) 2 $i − j + 6 k$ 1
| AB × AC |
3 3 (Area of ∆ ABC) 2
13 $ h= =
(c) 2 i$ + 13$j + 6k$ (d) 2 i$ + j + 6k$ 1
|BC|
1
| BC |
3 2 2
Sol. (d) Q AB = − 2$i + $j + k $
As we know bisector of angle A divides the BC in AC = − $i + 2$j − k $
ratio c : b. $
where c is length of side AB = 4 + 16 + 16 = 6 and BC = $i + $j − 2k
$i $j $
and b is length of side AC = 4+ 4+1 = 3 k
∴ Position vector of the point where the bisector So, AB × AC = −2 1 1
of angle A meet BC is −1 2 −1
288 AP EAMCET Chapterwise Mathematics

4 + 4 + 1 |C|  = |C|
$ (−4 + 1) = − 3$i − 3$j − 3k
= $i (−1 − 2) − j(2 + 1) + k $ 1 3
= ...(i)
 2 2
∴ |AB × AC| = 3 3
and |BC| = 6 Q |C − A| = 2 2 (Given)
3 3 3 ⇒|C| + |A| − 2C ⋅ A = 8
2 2
∴ h=
=
2 3 2 ⇒ |C|2 + 9 − 2|C| = 8 [as A ⋅ C = |C|]
Hence, option (b) is correct. ⇒ (|C| −1)2 = 0 ⇒|C| = 1
51. A new tetrahedron is formed by joining the So, (A × B) × C =
3
[from Eq. (i)]
centroids of the faces of a given tetrahedron 2
OABC. Then the ratio of the volume of the Hence, option (b) is correct.
new tetrahedron to that of the given
53. If the vectors AB = $i + 3$j + 4 k$ ,
tetrahedron is [20 April 2019, Shift-I]
3 1 5 1 AC = 5 $i + $j + 2 k$ are two sides of a triangle
(a) (b) (c) (d)
25 27 62 162 ABC, whose centroid is G, then|AG| =
[20 April 2019, Shift-II]
Sol. (b)
2 1
Let the position vectors of the vertices of (a) 22 (b) 22 (c) 22 (d) 18
tetrahedron OABC are OA = a , OB = b and OC = c 3 3
1 Sol. (a)
So, volume of tetrahedron OABC = [a b c]
6 It the vectors AB = $i + 3$j + 4k $
Now, position vectors of vertices of new and AC = 5$i + $j + 2k, then
a +b b+c c+a a +b+c $ ) + (5$i + $j + 2k
$)
tetrahedron are , , and , ($i + 3$j + 4k
3 3 3 3 AG = , where G is the
3
so, coterminous edge vectors of the new centroid of ∆ABC.
a b c
tetrahedron are , and . 4 $
3 3 3 AG = 2$i + $j + 2k
3
∴Volume of new tetrahedron is 
1 a b c
16 16
6  3 3 3 ∴ |AG| = 4 + + 4= 8+
9 9
1
= [a b c] 72 + 16 1 2
6 × 27 = = 88 = 22
9 3 3
1
So, the required ratio = Hence, option (a) is correct.
27
Hence, option (b) is correct. 54. If A , B, C and D are points whose position
vectors are $i + $j + k$ , 4 $i − $j + 2 k$ , 5i$ + $j ,
52. Let A = 2$i + $j − 2k$ and B = $i + $j. If C is a
7 i$ + 2 $j + 3 k$ respectively, then the projection
vector such that A ⋅ C = |C|,|C − A| = 2 2 and
of AB on CD is [20 April 2019, Shift-II]
the angle between A × B and C is 30°, then 4 2
the value of|(A × B) × C| is [20 April 2019, Shift-I] (a) (b)
3 7
2 3
(a) (b) (c) 3 (d) 2 3 7
3 2 (c) (d)
4 2
Sol. (b)
$ and B = $i + $j, so
Sol. (d)
Since, A = 2$i + $j − 2k Given, position vectors of points A, B, C and D are
$i $j k $ $ , 4$i − $j + 2k
$i + $j + k $ , 5$i + $j and 7$i + 2$j + 3k
$.
A × B = 2 1 −2 = $i (2) − $j(2) + k $ (2 − 1) $ and CD = 2$i + $j + 3k $
So, AB = 3$i − 2$j + k
1 1 0
∴ Projection of AB on CD is
$
= 2$i − 2$j + k |AB ⋅ CD| |6 − 2 + 3| 7 7
= = = =
Q |(A × B) × C| = |A × B||C|sin 30° |CD| 4+1+ 9 14 2
[as angle between A × B and C = 30° (given)] Hence, option (d) is correct.
Vector Algebra 289

55. Let a and b be unit vectors with θ as the Sol. (d)


acute angle between them. If If is given that a × (a × c) + b = 0
1 ⇒ (a ⋅ c)a − (a ⋅ a) c = − b
|a − b| = sin λθ, then 4 λ 2 =
2 Let a ⋅c = x
[20 April 2019, Shift-II] ⇒ xa − c = −[Q|
b a| = 1 ⇒ a ⋅ a = 1]
(a) 4 (b) 1 (c) 3 (d) 2 On squaring both sides, we get
Sol. (b) x 2|a|2 + |c|2 − 2x(a ⋅ c) = |b|2
1 ⇒ x 2 + 4 − 2x 2 = 1[as|b| = 1 and |c| = 2]
It is given, that |a − b| = sin(λθ)
2
⇒ x = 3 ⇒(a ⋅ c)2 = 3
2
On squaring both sides, we get
Hence, option (d) is correct.
⇒ |a |2 + |b|2 − 2|a||b|cosθ = 4sin2(λθ)
⇒ 1 + 1 − 2 cosθ = 4sin2(λθ) 58. If 3$i − 2$j − k$ , 2$i + 3$j − 4 k$ , − $i + $j + 2k$ and
θ 4 i$ + 5 $j + λk$ are respectively the position
⇒ 2(1 − cosθ) = 4sin2(λθ) ⇒ 4sin2   = 4sin2(λθ)
 2 vectors of four coplanar points P , Q, R and S,
1 Q (1 − cosθ) = 2sin2 θ 
then λ = [21 April 2019, Shift-I]
⇒ λ= 46 46 146 146
2  2 (a) (b) − (c) (d) −
2
17 17 17 17
4λ2 = 4   = 1
1
∴ Sol. (d)
 2
Given, P = 3$i − 2$j − $
k
Hence, option (b) is correct.
Q = 2$i + 3$j − 4k$
56. Let a = 2i$ − 3$j + 4 k$ , b = i$ + 2$j − 2k$ and
$
R = − $i + $j + 2k
c = 3 $i − $j + k$ . The volume (in cubic units) of
and $
S = 4$i + 5$j + λk
the parallelopiped having a + b + c, a − b + c
and a + b − c as coterminus edges is Here, $
PS = (4 − 3)$i + (5 + 2)$j + (λ + 1)k
[20 April 2019, Shift-II]
$
= $i + 7$j + (λ + 1)k
(a) 6 (b) 7 (c) 28 (d) 36
$
PQ = − $i + 5$j − 3k
Sol. (c)
It is given that, and $
PR = − 4$i + 3$j + 3k
a = 2$i − 3$j + 4k$ , b = $i + 2$j − 2k
$
$i $j $
k
and c = 3i − j + k
$ $ $
Now, PQ × PR = − 1 5 −3
So, vectors −4 3 3
a + b + c = 6 $i − 2$j + 3k$
$ (− 3 + 20)
= $i(15 + 9) − $j(− 3 − 12) + k
a − b + c = 4$i − 6$j + 7k $
= 24$i + 15$j + 17k $
and a + b − c = 0$i − 0$j + k$
Since, P , Q, R and S are coplanar, PS ⋅ (PQ × PR) = 0
So, the required volume of the parallelopiped
having a + b + c, a − b + c and a + b − c as $ ]⋅ [24$i + 15$j + 17k
⇒[$i + 7$j + (λ + 1)k $] = 0
coterminus edges is
⇒ 24 + 105 + 17(λ + 1) = 0 ⇒ 129 + 17λ + 17 = 0
6 −2 3
146
v = 4 −6 7 = |1 (−36 + 8)| = |−28| = 28. ⇒ 17λ = − 146 ⇒ λ = −
17
0 0 1
59. If OA = 2$i + 2$j + k$ , OB = 2i$ + 4 $j + 4 k$ and
Hence, option (c) is correct. the length of the internal bisector of ∠BOA of
57. If|a| = 1,|b| = 1,|c| = 2 and a × (a × c) + b = 0, triangle AOB is k, then 9 k 2 =
then (a ⋅ c)2 = 0 [20 April 2019, Shift-II] [21 April 2019, Shift-I]

(a) 1 (b) 2 (c) 4 (d) 3 (a) 225 (b) 136 (c) 712 (d) 20
290 AP EAMCET Chapterwise Mathematics

Sol. (b) 61. Let A = (α ,1, 2α), B = (3, 1, 2) and


Given vectors
C = 4 $i − $j + 3 k$ . If AB × C = 6 i$ + 9 $j − 5 k$ ,
OA = 2$i + 2$j + k$
then α 2 + α + 5 = [21 April 2019, Shift-I]
and OB = 2$i + 4$j + 4k$
(a) 11 (b) 7 (c) 9 (d) 5
Q |OA| = 4 + 4 + 1 = 3 = m (let) and
Sol. (b)
|OB| = 4 + 16 + 16 = 6 = n (let) Given, A = (α , 1, 2α)
Q The angle bisector of ∠BOA intersect the side AB B = (3, 1, 2)
at point P in the ratio 3 : 6 = 1 : 2, so and C = 4$i − $j + 3k$
2(OA) + 1(OB) Now, AB = (3 − α)$i + (2 − 2α)k$
OP =
3 $i $j $
k
6$i + 8$j + 6k$ 8
= = 2$i + $j + 2k$ ∴ AB × C = 3 − α 0 2 − 2α
3 3
2 4 −1 3
22 +   + 22
8
∴ |OP| =
 3 $ (− 3 + α)
= $i(2 − 2α) − $j(9 − 3α − 8 + 8α) + k
64 $
= (2 − 2α)$i + (5α − 1)$j + (α − 3)k
= 4+ + 4
9 $ , then
Comparing it with 6$i + 9$j − 5k
72 + 64 136
= = =k 2 − 2α = 6, 5α − 1 = 9 and α − 3 = − 5
9 9
2α = − 4 ⇒ α = − 2
9k2 = 9
136 
∴  = 136 ∴ α 2 + α + 5 = (− 2)2 − 2 + 5 = 4 − 2 + 5 = 7
 9 
62. If a makes an acute angle with b, r ⋅ a = 0 and
60. If a + xb + yc = 0,
r × b = c × b, then r = [21 April 2019, Shift-I]
a × b + b × c + c × a = 6(b × c), then the locus
of the point (x , y) is [21 April 2019, Shift-I]
(a) a × c − b (b) c × a
c ⋅a  c ⋅a 
(a) x + y = 1
2 2
(b) x + y − 5 = 0 (c) c −  b (d) c +  b
b⋅a  b⋅a 
(c) 2 x + 6 y = 5 (d) x + y + 6 = 0
Sol. (c)
Sol. (b) Given, r⋅a = 0
Given, a + xb + yc = 0
and r×b=c×b
As we know that, if a + b + c = 0
So, a × (r × b) = a × (c × b)
Then, a × b = b × c = c × a ≠ 0
⇒ r(a ⋅ b) − b(a ⋅ r) = c(a ⋅ b) − b(a ⋅ c)
Now, if a + xb + yc = 0
⇒ r(a ⋅ b) = c(a ⋅ b) − b(a ⋅ c)
Let x(a × b) = xy(b × c) = y(c × a) = p
c⋅a 
p
⇒ a × b = ,b × c =
p
and c × a =
p ⇒ r = c − b 
 a ⋅b
x xy y
p p p 63. If a , b , c are non-zero, non-collinear vectors
∴a × b + b × c + c × a = + +
x xy y
and a × b = b × c = c × a , then a + b + c =
 x + y + 1 [21 April 2019, Shift-II]
=p 
 xy  (a) 3 a (b) 0 (c) 3 (a × b) (d) 3 (b × c )
 x + y + 1 Sol. (b)
=  × xy × (b × c)
 xy  Let a + b + c = k
= (x + y + 1) (b × c) ∴ a × (a + b + c) = k × a
Comparing it with 6(b × c), ⇒ a ×a + a ×b+ a ×c= k×a
x + y+1= 6 ⇒ 0+ a ×b−c×a = k×a
∴ x+ y=5 ⇒ a ×b−a ×b= k×a
⇒ x + y − 5= 0 ⇒ 0= k×a [Qa × b = c × a ]
Hence, option (b) satisfies it. ⇒ k=0 ⇒a +b+c=0
Vector Algebra 291

64. If V = 2$i + $j − k$ , W = $i + 3k$ and U is a  a ⋅ b


∴ AB = OB − OA = b −  2  a
unit vector, then the maximum value of  |a| 
[U V W] is [21 April 2019, Shift-II] (a ⋅ a)b − (a ⋅ b)a a × (b × a)
= =
(a) 57 (b) 59 (c) 60 (d) 10 + 6 |a|2 |a|2
Sol. (b) ∴ Both (A) and (R) are true and (R) is the correct
$ and W = $i + 3k
Given, V = 2$i + $j − k $ explanation of (A).
$i $j k $ 66. For a non-zero real number x, if the points

Now, V × W = 2 1 −1 with position vectors

 


1 0 3
 (x − u)i$ + x$j + xk$ , xi$ + (x − v)$j + xk,
$
$ (0 − 1)
= i(3 − 0) − $j(6 + 1) + k
xi + xj + (x − w)k and
$ $ $

= 3i − 7 j − k
$ $ $ (x − 1)i$ + (x −1)$j + (x −1)k$ are coplanar, then
∴ [U V W] = U ⋅ (V × W) [22 April 2019, Shift-I]
= |U||V × W|cosθ = 1( 59) cosθ (a) u + v + w =1 (b) uvw =1
1 1 1
[Q|U| = 1 and |V × W| = 9 + 49 + 1 = 59] (c) + + = 1 (d) uv + vw + uw = 1
u v w
= 59 [Q Maximum value of cosθ is (1)] Sol. (c)
Hence, maximum value of [U V W] is 59. Let OA = (x − u)$i + x$j + xk$
65. Assertion (A) : If a , b are two non collinear OB = x$i + (x − v)$j + xk$
vectors, then the vector component of b along $
OC = x$i + x$j + (x − w)k
a × (b × a)
the line perpendicular to a is $
and OD = (x − 1)$i + (x − 1)$j + (x −1)k
|a|2
Here, DA = (1 − u)$i + $j + k $
Reason (R) : a × (b × c) = (a ⋅ c)b − (a ⋅ b)c
$
DB = $i + (1 − v)$j + k
 c c
and vector component of b on c is  b ⋅  $
 |c| |c| and DC = $i + $j + (1 − w)k
[21 April 2019, Shift-II] Since, points are collinear
(a) Both (A) and (R) are true and (R) is the correct [DA DB DC] = 0
explanation of (A) 1 − u 1 1 
(b) Both (A) and (R) are true but (R) is not the correct ⇒  1 1− v 1 =0
explanation of (A)  
 1 1 1 − w
(c) (A) is true but (R) is false
(d) (A) is false but (R) is true On solving this, we get
1 1 1
Sol. (a) + + =1
u v w
The vector component of b along the line parallel
 a ⋅ b a 67. If P is a point lying on the line passing
to a is  
 |a|  |a| through the point A(i$ − $j +3 k$ ) and parallel to
B the vector 2 $i + $j − 2 k$ such that|AP| =18,
then a position vector of P is
[22 April 2019, Shift-I]
b c
(a) −13$i − 5$j + 9k $ (b) 11$i + 7 j − 15 k$
$
(c) 13 $i − 5$j + 9k (d) 13$i + 5$j − 9k$

O Sol. (d)
a A
According to the given information, the diagram
 a ⋅ b
∴ OA =  2  a is shown as below.
 |a| 
292 AP EAMCET Chapterwise Mathematics

P Sol. (d)
Given,
m= $ )]
3 × unit vectors of [($i + $j) × ($j − k
A
 $i $j k$ 
O $) = 1 1 0 
Q ($i + $j) × ($j − k

 
 0 1 −1
 
Given, AP = 18 $ ()
= $i(−1 − 0) − $j(−1 − 0) + k 1
$
OA = $i − $j + 3k $
= −i + j+k
$ $
$
2$i + $j − 2k $
Now, AP =18 × − $i + $j + k
∴ m = 3×
(2) + ()
2
1 + (−2)
2 2
1 +1 + 1
$ $ $
2i + j − 2k $
2$i + $j − 2k $
=18 × =18 × − $i + $j + k $
= 3× = − $i + $j + k
4 +1 + 4 3 3
$ $
= 6(2i + j − 2k) = 12i + 6 j −12k
$ $ $ $ and n = 2 6 × unit vectors of [(2$i − $j) × ($j + 2k $ )]
By triangle law,  $i $j k $
$ 12$i + 6$j −12k
OP = OA + AP = $i − $j + 3k+ $ $  
∴ (2i − j) × ( j + 2k) =  2 −1 0 
$ $ $
$
= 13$i + 5$j − 9k 0 1 2
 
68. a , b , c are three vectors such that $
= i(−2 − 0) − j(4 − 0) + k(2 + 0) = − 2$i − 4$j + 2k
$ $ $
|a| =1,|b| = 2 ,|c| = 3 and b. c =0. If the $ $
−2$i − 4$j + 2k −2$i − 4$j + 2k
projection of b along a is equal to the n=2 6× =2 6×
4 + 16 + 4 2 6
projection of c along a, then|2a + 3 b − 3c|=
[22 April 2019, Shift-I] $
= − 2$i − 4$j + 2k
(a) 3 (b) 22 (c) 9 (d) 11 1
Now, required area = (m × n)
Sol. (d) 2
1 $ ) × (−2$i − 4$j + 2k
$ )|
Given, = |(− $i + $j + k
|a| = 1,|b| = 2,|c| = 3 and b ⋅ c = 0 2
$i $j k $
Now, | 2a + 3b − 3c|
1
= 4|a|2 + 9|b|2 + 9|c|2 + 12a ⋅ b − 18b ⋅ c − 12a ⋅ c = −1 1 1
2
−2 −4 2
= 4 + 9(4) + 9(9) + 12a ⋅ b − 0 − 12a ⋅ c
1 $ $ (4 + 2)|
= 4 + 36 + 81 + 12a ⋅ b − 12a ⋅ c = ×|i(2 + 4) − $j(−2 + 2) + k
2
Since, projection of b along a is equal to projection 1 $ | = 1 62 + 62
= × |6$i + 6k
of c along 2 2
a ⇒a ⋅ b = a ⋅ c 1
= × 6× 2= 3 2
= 121 + 6 a ⋅ b − 6 a ⋅ b = 121 = 11 2
70. a = i$ − $j + k$ , b = $i − 2$j + k$ , c = pi$ + 2$j + qk$
69. Let m be a vector of magnitude 3 and
and d = pi$ + q$j + 2 k$ are given vectors. If the
perpendicular to the vectors i$ + $j and $j − k.
$
projection of c on a is 5 3 units and if
Let n be another vector of magnitude 2 6 a, b and c form a parallelopiped of volume
and perpendicular to the vectors 2 i$ − $j and 5 cubic units, then tan −1 (b. d) =
$j + 2 k$ . The area (in sq. units) of the triangle [22 April 2019, Shift-I]
formed with m and n as sides is π π
(a) (b)
[22 April 2019, Shift-I] 2 3
π π
(a) 2 (b) 6 (c) 2 3 (d) 3 2 (c) (d)
4 6
Vector Algebra 293

Sol. (c) ⇒ x =y+z … (ii)


Given, By solving Eqs. (i) and (ii), we get
$ , b = $i − 2$j +
a = $i − $j + k $
k x = 0, y = − c
1
c = p$i + 2$j + qk $ , d = p$i + $
q$j + 2k Q x + y + z =1 ⇒ y = ±
2 2 2
2
and projection of c on a = 5 3 ∴ c=±
1 $
(− $j + k)
a ⋅c p+ q−2 2
=5 3 ⇒ =5 3
|a| 3 72. a , b , c are three mutually perpendicular unit
⇒ p + q − 2 = 15 ⇒ p + q =17 …(i) vectors in the right handed system. If the
Also, given that [a b c] = 5 points P , Q , R with position vectors
1 −1 1  2a + 5 b − 4 c, a + 4 b − 3c and ka + 7 b − 6 c
⇒ 1 −2 1  = 5 respectively lie on a line, then, the ratio in
  which the point P divides QR is
 p 2 q 
[22 April 2019, Shift-II]
⇒ 1(−2q − 2) + 1(q − p) + 1(2 + 2p) = 5
(a) 1 : 2 (b) − 1 : 3 (c) 3 : 1 (d) − 1 : 2
⇒ −2q − 2 + q − p + 2 + 2p = 5
⇒ p − q =5 …(ii)
Sol. (a)
Given, P = 2a + 5b − 4c
By solving Eqs. (i) and (ii), we get
Q = a + 4b − 3c
p = 11, q = 6 and R = ka + 7b − 6c
Now, b ⋅ d = ($i − 2$j + k$ ). (p$i + q$j + 2k
$)
Let point P divides QR in λ :1.
= p − 2q + 2 λ 1
= 11 − 12 + 2 =1 Q P R
π π λOR + OQ
∴ 1 = tan−1  tan  =
tan−1 (b ⋅ d) = tan−1 () ∴ = OP
 4 4 λ +1
λ(ka + 7 b − 6 c) + (a + 4b − 3c)
71. Given, a = 2i$ + $j + k$ , b = $i + 2$j − k$ and a unit ⇒ = 2a + 5b − 4 c
λ +1
vector c are coplanar. If c is perpendicular to 7λ + 4
a, then c = [22 April 2019, Shift-I] ∴ =5
λ +1
1 $ $ $ 1 $
(a) ± ( i − j − k) (b) ( i − 2 $j) ⇒ 7λ + 4 = 5λ + 5
3 5
−1 $ $ $ 1
(c) ( i + j + k) (d) ±
1 $)
( − $j + k ⇒ 2λ = 1 ⇒ λ =
3 2 2
∴ Required ratio = 1 : 2.
Sol. (d)
Given, $
a = 2$i + $j + k 73. If a = i$ − $j − k$ , b = 2$i − 3$j + k$ and p1, p 2are
$
b = $i + 2$j − k the orthogonal projection vectors of a on b
$ and b on a respectively, then
Let c = x$i + y$j + zk
(p1 + p 2) ⋅ (p1 − p 2) = [22 April 2019, Shift-II]
∴ c⋅a = 0 46 25 44 88
$ ).(2$i + $j + k
$) = 0 (a) − (b) (c) (d) −
⇒ ( x i + y j + zk
$ $
21 7 7 21
⇒ 2x + y + z = 0 …(i) Sol. (d)
Since, a, b and c are coplanar. Given, a = $i − $j − k$ and b = 2$i − 3$j + k$
∴ [a b c] = 0
a ⋅ b ($i − $j − k$) ⋅ (2$i − 3$j + k$)
x y z  ∴ P1 = =
2 1 1 =0 |b| 4+ 9+1

  2+ 3−1 4
1 2 −1 = =
14 14
⇒ x(−1 − 2) − y(−2 − 1) + z(4 − 1) = 0 a ⋅ b ($i − $j − k$) ⋅ (2$i − 3$j + k$)
and P2 = =
⇒ −3 x + 3 y + 3 z = 0 |a| 1+1+1
⇒ −x + y + z =0
294 AP EAMCET Chapterwise Mathematics

2+ 3−1 4
= = 76. If a = i$ − 2$j + k$ , b = $i + 3$j − 2k$ , c = 2$i + $j − k$
3 3
Now, (P1 + P2) ⋅ (P1 − P2) and d = i$ + $j + k$ , then the volume (in cubic
units) of the tetrahedron having
= 
4 4  4 4 
+  − 
 14 3   14 3 (a × b) × c , b , d as its coterminous edges is
2 2 [22 April 2019, Shift-II]
= 
4   4  16 − 16
 −  = (a)
15
(b) 90
 14   3  14 3 2
48 − 224 −176 88 21 66
= = =− (c) (d)
42 42 21 2 5
74. Let a , b , c be three non-coplanar vectors and Sol. (c)
b×c c×a a×b Given,
a′= , b′ = , c′ = . The a = $i − 2$j + $
k
[a b c] [a b c] [a b c]
b = $i + 3$j − 2k$
length of the altitude of the parallelopiped
formed by a ′, b′, c′ as coterminous edges, $ and d = $i + $j + k
c = 2$i + $j − k $ Here,
with respect to the base having a ′ and c′ as $
$i $j k
its adjacent sides is [22 April 2019, Shift-II]
1 1 a ×b= 1 − 2 1
(a)| a | (b) (c)| c | (d)
| b| | a × c| 1 3 −2

Sol. (b) $ (3 + 2)
= $i (4 − 3) − $j (− 2 − 1) + k
Let the length of altitude is h. = i + 3 j + 5k
$ $ $
[b × c c × a a × b]
Now, |a ′ × c′ | × h = [a ′ b′ c′] = $i $j $
k
[a b c]3
∴ (a × b) × c = 1 3 5
[a b c]2 1
= = 2 1 −1
[a b c]3 [a b c]
$ (1 − 6)
= $i(− 3 − 5) − $j(− 1 − 10) + k
Now, |a ′ × c′ | = {(b × c) × (a × b)}
|(a b c) b | 1 = − 8 i + 11 j − 5k
$ $ $
h= 2
=
[a b c] |a b c | 1
Now, volume = |(a × b) × c b d |
1 6
⇒ |b | h = 1 ⇒ h =
|b | − 8 11 − 5
1
75. Let a , b , c be three non-coplanar vectors. Let = 1 3 −2
6
1 1 1
Si (i = 1, 2 , 3 , 4 , 5 , 6) denote the six scalar triple
products formed by all possible permutations 1
= |[−8(3 + 2) −11(1 +2) − 5(1 − 3)]
of a, b, c. If i , j , k , l are randomly chosen 6
distinct numbers from 1 to 6 and if 1
= |[− 40 − 33 + 10]|
S S S S 6
x = i + k , y = i − k , then x 2 + y 2 =
Sj Sl Sj Sl [22 April 2019, Shift-II] 63 21
= = sq unit
(a) 1 (b) 4 6 2
(c) 8 (d) 2 77. If α, β, γ are distinct real numbers and
Sol. (b) α + β + γ ≠ 0, then the points with position
As we know that, vectors α$i + β$j + γk$ , β$i + γ$j + αk$ and
|S1|=|S2|=|S3|= ...... =|S6|=|a b c| γi$ + α$j + βk$ are
2
[23 April 2019, Shift-I]
2
S  S  (a) collinear
∴ x 2 + y2 = 2  i  + 2 k 
S   Sl  (b) vertices of a scalene triangle
 j 
(c) vertices of an isosceles triangle
= 2(1)2 + 2(1)2 = 4 (d) vertices of an equilateral triangle
Vector Algebra 295

Sol. (d) 79. a = 3i$ + $j − k$ , b = $i − 4 $j + 5k$ , c = 4 i$ + 5$j − k$


Let position vectors are are three vectors and a vector r is
$
OA = α$i + β$j + γk perpendicular to both the vectors b and c. If
$ and
OB = β$i + γ$j + αk, r ⋅ a = 9, then r = [23 April 2019, Shift-I]
$ (a) 3 ($i − $j − k$ ) (b) 3 ($i − $j + k$ )
OC = γ$i + α$j + βk
$ (c) 9 ($i − $j − k$ ) (d) 9($i − $j + k$ )
So, AB = (β − α)$i + (γ − β)$j + (α − γ)k
$ and Sol. (a)
BC = (γ − β)$i + (α − γ)$j + (β − α)k $ , b = $i − 4$j + 5k
$ and
Given vectors a = 3$i + $j − k
$
CA = (α − γ)$i + (β − α)$j + (γ − β)k $ $ $
c = 4i + 5j − k
Q | AB | = (α − β)2 + (β − γ)2 + (γ − α)2 $i $j $
k
| BC| = (β − γ) + (γ − α) + (α − β)
2 2 2
Qb × c = 1 −4 5
and | CA | = (γ − α)2 + (α − β)2 + (β − γ)2 4 5 –1
$ (5 + 16)
= $i (4 − 25) − $j (−1 − 20) + k
Q | AB| = | BC| = | CA |
= (α − β)2 + (β − γ)2 + (γ − α)2 ≠ 0 = − 21$i + 21 $j + 21k $ = 21(– $i + $j + k
$)

[Qα,β , γ are distinct real numbers] ⇒ |b × c| = 21 3


So, given position vectors are vertices of an Now, vector r, which is perpendicular to b and c,
equilateral triangle. so
Hence, option (d) is correct. b×c
r = ± |r |
78. Given three vectors a = 2i$ − $j + k$ , b = i$ +2$j− k$ |b × c |

and c = $i + $j − 2 k$ , a vector in the plane of b Q r.a = 9


|r| $ ) ⋅ (3$i + $j − k
$ )] = 9
and c whose projection on a is of magnitude ⇒ ± [21(− $i + $j + k
|b × c|
2
is |r |
3 [23 April 2019, Shift-I] ⇒ ± [21 (− 3 + 1 − 1)] = 9
21 3
(a) − 2 $i − $j + 5k$ (b) 2 $i + 3$j + 3k$ ⇒ |r | = 3 3 (Q|r |> 0)
(c) 2 $i + $j + 5k$ (d) 2 $i − 3$j + 3k$ $)
21(− $i + $j + k $)
∴ r=± 3 3 = ± 3(− $i + $j + k
Sol. (a) 21 3
Given three vectors are $)
= − 3($i + $j + k) or 3($i − $j − k
$ b = $i + 2$j − k
a = 2$i − $j + k, $
Hence,option (a) is correct.
and c = $i + $j − 2k $
80. AB = a and AC = b are the sides of a ∆ABC. P
Now, let a vector in plane of b and c is is a point on AB and Q is a point on BC such
d = b + λc AP 1 BQ 1
$ that = and = . If the point of
= (1 + λ) $i + (2 + λ) $j − (1 + 2λ) k PB 2 QC 2
2 intersection of AQ and CP is D and the area
Q Projection of vector d on a is =
3 of ∆BCD is 7 square units, then the area of
d ⋅a 2
the ∆ABC (in the same sq units) is
⇒ = [23 April 2019, Shift-I]
|a| 3
49 49 7 7
(a) (b) (c) (d)
( + λ) − (2 + λ) − (1 + 2λ)
21 2 4 2 2 4
⇒ =
4+1+1 3
Sol. (a)
⇒ 2 + 2λ −2 − λ − 1 − 2λ = 2 ⇒ λ = − 3 According to given informations,
$
So, required vector d = − 2$i − $j + 5k a 2a + b
AP = and AQ =
Hence, option (a) is correct. 3 3
296 AP EAMCET Chapterwise Mathematics

Let D divides the line AQ in ratio λ :1 and CP in  $i − $j  (2$i + 3$j + 4k


$) 2− 3 1
µ :1. = ±   . = =
 2  4 + 9 + 16 2 29 58
a
µ +b
λ(2a + b)
= 3
Hence, option (c) is correct.
So, =
3(λ + 1) µ+1
82. If a , b , c are three unit vectors such that
On comparing
µ 2λ λ 3 1
= and
1
= a × (b × c) = b + c, then the angles
3(µ + 1) 3(λ + 1) µ + 1 3(λ + 1) 2 2
µ 2 between a , b and a , c respectively are
⇒ = ⇒µ = 6 [23 April 2019, Shift-I]
3(µ + 1) µ + 1
1 λ 3 (a) 60°, 30° (b) 120°, 30°
So, = ⇒ 3λ + 3 = 7λ ⇒ λ = (c) 60°, 150° (d) 120°, 150°
7 3(λ + 1) 4
2a + b Sol. (b)
⇒ AD =
7 3 1
Given that a × (b × c) = b+ c
1 2 2
Now area of ∆BCD = | BC × BD |
2 3 1
⇒ (a . c) b − (a . b) c = b+ c
b − 5a 
(b − a) × 
1 2 2
= 
2  7  Let angle between unit vectors a and c is ‘α’ and
1 angle between unit vectors a and b is ‘β’, then
= |(−5b × a) − (a × b)|
14 3 1
(cos α) b − (cos β) c = b+ c
4
= |a × b| = 7 (given) 2 2
14 3 1
1 49 ⇒ cos α = and cosβ = −
⇒ |a × b| = 2 2
2 4
⇒ α = 30° and β = 120°
49
⇒ Area of ∆ABC = Hence, option (b) is correct.
4
Hence, option (a) is correct. 83. If a = i$ − 2$j − 3k$ , b = 2$i + $j − k$ , c = $i + 3$j − 2k$ ,
81. If a = i$ + $j + k$ , b = i$ + $j + 2k$ and then
[(a × b) × (b × c)(b × c) × (c × a)(c × a) × (a × b)] =
c = 2 $i + 3 $j + 4 k$ then the magnitude of the
[22 April 2018, Shift-I]
projection on c of a unit vector that is (a) 160000 (b) −8000
perpendicular to both a and b is (c) 400 (d) −40
[23 April 2019, Shift-I]
1 1 1 3 Sol. (a)
(a) (b) (c) (d)
29 3 6 58 29 We have, a = i$ − 2$j − 3k$

Sol. (c) b = 2$i + $j − k$


Given vectors are c = $i + 3$j − 2k$
$ , b = $i + $j + 2k
a = $i + $j + k $ and c = 2$i + 3$j + 4k
$ Now,
$i $j k $ (a × b) × (b × c) = [a b c] b − [ab b] c = [a b c] b
(b × c) × (c × a) = [b c a] c − [b c c] a = [a b c] c
Then, a × b = 1 1 1
1 1 2 (c × a) × (a × b) = [c a b] a − [c a a] b = [a b c] a
$ (1 − 1) = $i − $j 1 −2 −3
= $i (2 − 1) − $j (2 − 1) + k
Now, [a b c] = 2 1 −1
So, unit vector perpendicular to both a and b is
1 3 −2
$i − $j
= ± = 1(−2 +3) + 2(−4 +1) −3(6 − 1) = 1 − 6 − 15 = − 20
2
$i − $j ∴ [(a × b) × (b × c) (b × c) × (c × a)
Now the magnitude of the projection of ± on (c × a) × (a × b)]
2
c is = [− 20 b − 20 c − 20 a]
Vector Algebra 297

− 40 − 20 20 Now, let θ be angle between a and a + b + c


= − 20 − 60 40 a ⋅ (a + b + c) | a |2
∴cosθ = =
− 20 40 60 | a || a + b + c | | a || a + b + c |
= − 40[− 3600 − 1600] + 20[− 1200 + 800] λ2 1
= = .
+ 20[− 800 − 1200] λ + 3λ 3
= 208000 − 8000 − 40000 = 160000.
86. It a, b and c are non-coplanar vectors and the
84. If a = i$ + 2$j + 3k$ , b = − $i + 2$j + k$ , four points with position vectors 2a + 3 b − c,
c = $i + 2 $j − 2 k$ , n is perpendicular to both a a − 2 b + 3c, 3a + 4 b+2c and ka − 6 b + 6 c are
and b and θ is the angle between c and n coplanar, then k = [22 April 2018, Shift-I]
then sinθ = [22 April 2018, Shift-I] (a) 0 (b) 1 (c) 2 (d) 3
2 2 2 3 Sol. (b)
(a) (b) (c) (d)
3 3 3 3 2 Let A(2a + 3b − c), B(a − 2b + 3c)
Sol. (b) C(3a + 4b − 2c) and D(ka − 6b + 6c)
We have, n ⊥ a and n ⊥ b ∴ n =a×b ∴ AB = − a − 5b + 4c
i j k AC = a + b − c
= 1 2 3 = − 4i − 4 j + 4k AD = (k − 2) a − 9b + 7c
−1 2 1 Since, A, B, C , D are coplanar
| n × c| −1 −5 4
Again, sinθ =
| n || c | ∴ 1 1 −1 = 0
k−2 −9 7
i j k
Now, n × c = − 4 − 4 4 = − 4$j − 4k$ ⇒ (k − 2) (5 − 4) + 91( − 4) + 7(− 1 + 5) = 0
1 2 −2 ⇒ k − 2 − 27 + 28 = 0
⇒ k − 1 = 0 ⇒ k = 1.
| − 4$j − 4k$ |
∴ sinθ = 87. If P and Q are two points on the curve
| − 4i$ − 4$j + 4k$ || i$ + 2$j − 2k$ |
y = 2 x + 2 such that OP ⋅ i$ = − 1 and OQ ⋅ i$ = 2,
(− 4)2 + (− 4)2 then the magnitude of (QR − 4 OP) is
=
(− 4)2 + (− 4)2 + (4)2 1 2 + (2)2 + (− 2)2
() [22 April 2018, Shift-II]
16 + 16 4 2 2 (a) 10 (b) 1 (c) 5 (d) 100
= = =
16 + 16 + 16 1 + 4 + 4 (4 3 ) × 3 3 3 Sol. (a)
Let point P(p , 2p + 2) and Q (q , 2q + 2)
85. If a, b and c are mutually perpendicular
vectors of the same magnitude, then the Now, OP = Pi + 2p + 2 j
cosine of the angle between a and a + b + c is and OQ = qi + 2q + 2 j
[22 April 2018, Shift-I] According to the question,
1 1 1 3 OP ⋅ i = − 1
(a) (b) (c) (d)
2 3 2 2 ⇒ p = −1 ⇒ OP = − $i + 2$j
Sol. (b) and OQ ⋅ i = 2 ⇒ q = 2
Let| a | = | b | = | c | = λ ⇒ OQ = 2$i + 16$j
Now, | a + b + c |2 = (a + b + c) ⋅ (a + b + c) So, OQ − 4OP = 6$i + 8$j
= | a |2 + | b |2 + | c |2 + 2(a ⋅ b + b ⋅ c + c ⋅ a) ∴Magnitude of (OQ − 4OP) = 36 + 64 = 10.
= λ2 + λ2 + λ2 + 2(0 + 0 + 0) [Q a ⊥ b , b ⊥ c, c ⊥ a] 88. Let m be the unit vector orthogonal to the
= 3λ2 vector $i − $j + k$ and coplanar with the vectors
∴ | a + b + c| = 3λ 2 $i + $j and $j − k$ . If a = i$ − k$ , then the length
298 AP EAMCET Chapterwise Mathematics

of the perpendicular from the origin to the 90. If a and b are two unit vectors such that
plane r ⋅ m = a ⋅ m is [22 April 2018, Shift-II] c = (a × c) + b, then the maximum value of
1 1 [a bc] is [22 April 2018, Shift-II]
(a) (b)
26 5 1 3
(a) 1 (b) (c) (d) 2
5 2 2
(c) (d) 1
26
Sol. (b)
Sol. (c) c = (a × c) + b
The vector m is coplane with (2i$ + $j) and ($j − k$ ) ⇒ c
2
=|a + c|2 +|b|2 +2 (a × c) ⋅ b
So, $)
m = x(2$i + $j) + y($j − k ⇒ 2[abc ] =|a × c|2 + 1−|c|2
m = 2xi$ + ( x + y)$j − yk$ Let angle between vectors a and c is ‘θ’.
$ , so
∴ m is orthogonal to the vector $i − $j + k 1 2 2
So, [a b c] = [ c sin2 θ − c + 1]
2
2x − (x + y) − y = 0 1 2
⇒ x = 2y …(i) = [1 − c cos2 θ] …(i)
2
∴ m = 1 ⇒ 4 x 2 + (x + y)2 + y 2 = 1 Now, c ⋅ (a ⋅ c) = (a × c)2 + b ⋅ (a × c)
⇒ 16 y 2 + 9 y 2 + y 2 = 1 ⇒ [a b c] = c sin2 θ
2
…(ii)
1
⇒ 26 y = 1 ⇒ y = ±
2
. From Eq. (i) and (ii)
26 1 2 2 2 1
2 {1 − c cos2 θ} = c sin2 θ ⇒ c =
So, x=± 2 cos2 θ + 2sin2 θ
26
2 sin2 θ sin2 θ
→ Q [a b c] = c sin2 θ = =
m = ± 
4 $ 3 $ 1 $ 2
cos θ + 2sin θ 2
1 + sin2 θ
∴ i+ j− k
 26 26 26  1
=
The length of the perpendicular from the origin to cosec2θ + 1
the plane r. m = a. m is
For maximum value of [a b c] , cosec2 θ = 1, so
So, a. m = ($i − k$ ) . 
4 $ 3 $ 1 $
i+ j− k [a b c]max =
1
 26 26 26  2
4 1 5
= + = units. 91. If α, β and γ are non-zero vectors such that
26 26 26
|β| = |γ| = 1 and|α| = 10, then
89. If a , b , c are non-coplanar unit vectors such (α × (β + γ ) × (β × γ ) ⋅ (β − γ ) = [22 April 2018, Shift-II]
b+c (a) 10 (b)1 (c) 0 (d) 12
that a × (b × c) = , then the angle
2 Sol. (c)
between a and b is [22 April 2018, Shift-II] (α × (β + γ)) × (β × γ) ⋅ (β − γ)
π π = (α × (β + γ)). [(β × γ) × (β − γ)]
(a) (b)
6 4 = (α × (β + γ)). [(β ⋅ (β − γ)γ − (γ ⋅ (β − γ)β]
π 3π
(c) (d) 2
= (α × (β + γ)) ⋅ [(β − β ⋅ γ)γ − (γ ⋅ β − γ )β]
2
2 4
Sol. (d) = (α × (β + γ)) ⋅ [(1 − β ⋅ γ)] (γ + β]
Since, a × (b × c) = (1 − β ⋅ γ)[α × β) ⋅ γ + (α × γ) ⋅ β]
b+c ⇒ (1 − β ⋅ γ) [(α × β) ⋅ γ − (α × β) ⋅ γ] = 0
= (a ⋅ c) ⋅ b − (a ⋅ b) c = (given)
2 92. a , b , c are three vectors such that| a | = 3,
−1 1
So, a ⋅b = ⇒ cosθ = − |b | = 5,| c | = 7. If a , b , c are perpendicular to
2 2
the vectors b + c, c + a , a + b respectively,
{Q a and b are unit vector where θ
is angle between vectors a and b} then (a + b + c)2 − 2 = [23 April 2018, Shift-I]
3π (a) 15 (b) 9
⇒ θ= .
4 (c) 22 (d) 25
Vector Algebra 299

Sol. (b) 3 2 3
(a) (b) (c) 2 (d)
We have, | a | = 3,|b| = 5,|c| = 7 2 3 2
a is perpendicular to (b + c) Sol. (a)
So, a ⋅ (b + c) ⇒ a ⋅ b + a ⋅ c = 0 …(i) Let a = 2i$ + $j − 2k$
and b is perpendicular to c + a
|a| = 22 + 12 + (−2)2 = 3 and b = i$ + $j
So, b ⋅ (c + a ) = 0
⇒ b ⋅ c + a⋅ b = 0 …(ii) |b| = 1 + 1 = 2
and c is perpendicular to (a + b) $i $j k$
∴ c ⋅ (a + b) = 0 ⇒ a ⋅ c + b ⋅ c = 0 …(iii)
a × b = 2 1 −2 = i$(2) − $j(2) + k$ ()
1 = 2$i − 2$j + k$
Adding Eqs. (i), (ii) and (iii), we get
1 1 0
2(a ⋅ b + b ⋅ c + c ⋅ a) = 0 …(iv)
Now, |a + b + c|2 |a × b| = 22 + (−2)2 + ()
12 =3
= |a|2 + |b|2 + |c|2 + 2(a ⋅ b + b ⋅ c + c ⋅ a)
Now, a ⋅ c = |c | and |c − a |= 2 2
= (3)2 + (5)2 + (7)2 + 0 = 83
Squaring both side
Now, (a + b + c)2 − 2 = 83 − 2 = 81 = 9 | c − a |2 = (2 2)2
⇒ | c | + |a | − 2c ⋅ a = 8
2 2

93. If a , b , c are unit vectors and the maximum


⇒| c |2 + 9 − 2| c | = 8 ⇒ | c |2 − 2| c | + 1 = 0
value of| a − b |2 + | b − c |2 + | c − a |2 is k,
Let | c| = x
then k(2a 2 + 3 b 2 − 4 c 2) = [23 April 2018, Shift-I]
⇒ x 2 − 2x + 1 = 0 ⇒ (x − 1)2 = 0
(a) 6 (b) 8 (c) 9 (d) 12
⇒ x = 1 ⇒ | c| = 1
Sol. (c) Now,|(a × b) × c | = | a × b|| c |sin 30°
a, b and c are unit vectors. 1 3
= 3×1 × =
| a| = | b | = | c| = 1 2 2
Now, | a − b |2 + | b − c |2 + | c − a |2
95. If a = i + j + k, b = 2i − j + 3k and c = i − j
= 2(a2 + b 2 + c2) − 2(a ⋅ b + b ⋅ c + c ⋅ a)
and if 6 i + 2 j + 3 k = λ 1 (a × b)
= 21
( + 1 + 1) − 2(a ⋅ b + b ⋅ c + c ⋅ a)
+ λ 2 (b × c) + λ 3 (c × a), then (λ 1 , λ 2 , λ 3) =
= 6 − 2(a ⋅ b + b ⋅ c + c ⋅ a) …(i) [23 April 2018, Shift-I]
| a − b |2 + |b − c|2 + | c − a |2
(a)  , ,  (b)  , , 
11 4 19 4 11 19
is maximum when  5 5 5 5 5 5 
2(a ⋅ b + b ⋅ c + c ⋅ a) is minimum using. (c)  , ,  (d)  , , 
4 19 11 19 11 4
|a + b + c|2 = a2 + b 2 + c2 + 2(a ⋅ b + b ⋅ c + c ⋅ a) 5 5 5  5 5 5

We know that, Sol. (b)


a + b + c = 0, then a ⋅ b + b ⋅ c + c ⋅ a is minimum Let a = i$ + $j + k , b = 2i$ − $j + 3k$ and c = i$ − $j
0 = 1 + 1 + 1 + 2 (a ⋅ b + b ⋅ c + c ⋅ a) i$ $j k$
2 (a ⋅ b + b ⋅ c + c ⋅ a) = − 3
Now, a × b = 1 1 1
Substitute in Eq. (i)
2 −1 3
= 6 − ( − 3) = 9
Hence, k =9 = i$(3 + 1) − $j (3 − 2) + k$ (−1 − 2)
Now, k (2a2 + 3b 2 − 4c2) = 9 (2 + 3 − 4) = 9 ()
1 =9 = 4$i − $j − 3k$
94. Let a = 2i + j − 2k and b = i + j be two $i $j k$
vectors. c is a vector such that a ⋅ c =| c |and and b × c = 2 − 1 3
| c − a | = 2 2. If the angle between a × b and 1 −1 0
c is 30°, then|(a × b) × c | is equal to
[23 April 2018, Shift-I] = i$ (0 + 3) − $j (0 − 3) + k$ (−2 + 1)
300 AP EAMCET Chapterwise Mathematics

= 3$i + 3$j − k$ $i $j k$
$i $j k$ Now, a × b = 2 1 −3
and c × a = 1 − 1 0 1 −2 1
1 1 1 = i(1 − 6) − $j(2 + 3) + k$ ( − 4 − 1)
= $i (− 1 − 0) − $j (1 − 0) + k$ (1 + 1) = $i − $j + 2k$ a × b = − 5i$ − 5$j − 5k$
Now, 6i$ + 2$j + 3k$ = λ1 (a × b) + λ 2 (b × c) $i $j k$
+ λ 3 (c × a) Now, c × d = − 1 1 − 4
⇒ 6i$ + 2$j + 3k$ = λ1 (4i$ − $j − 3k$ )
1 1 1
λ (3$i + 3$j − k$ ) + λ (− $i − $j + 2k$ )
2 3

⇒ 6$i + 2$j + 3k$ = $i (4λ1 + 3λ 2 − λ 3) = i$(1 + 4) − $j(− 1 + 4) + k$ ( − 1 − 1)


+ $j (− λ1 + 3λ 2 − λ 3) + k$ (−3λ1 − λ 2 + 2λ 3) = 5$i − 3$j − 2k$
by comparing $i $j k$
4λ1 + 3λ 2 − λ 3 = 6 …(i) Now, (a × b) × (c × d) = − 5 − 5 − 5
− λ1 + 3λ 2 − λ 3 = 2 …(ii) 5 −3 −2
and −3λ1 − λ 2 − 2λ 3 = 3 …(iii)
Eqs. (i) and (ii), we get = i$(10 − 15) − $j(10 + 25) + k$ ( + 15 + 25)
4
5 λ1 = 4 ⇒ λ1 = = − 5$i − 35$j + 40k$ = 5(− i$ − 7$j + 8k$ )
5
put in Eqs. (ii) and (iii), we get |(a × b) × (c × d) | = 5 (− 1)2 + (−7)2 + (8)2
4
3λ 2 − λ 3 = 2 + = 5 1 + 49 + 64 = 5 114
5
14
⇒ 3λ 2 − λ 3 = …(iv) 97. If a and b are two unit vectors such that
5 2
4 27 a + b is also a unit vector, then a − b =
and − λ 2 + 2λ 3 = 3 + 3 . ⇒ − λ 2 + 2λ 3 =
5 5 [23 April 2018, Shift-II]
multiply by 3.
81 (a) 1 (b) 2 (c) 3 (d) 0
− 3λ 2 + 6λ 3 = …(v)
5 Sol. (c)
Add Eqs. (iv) and (v)
95 If resultant of two unit vectors is unit vector, then
5λ 3 = ⇒ 5λ 3 = 19 2π
5 angle between them is , so
3
19
λ3 = 2π
5 | a − b |2 = | a |2 + | b |2 − 2| a || b |cos
put in Eq. (iv) 3
3λ 2 =
14 19
+ ⇒ λ2 =
33 = 1 + 1 − 21() ()1 (− 1 / 2) = 2 + 1 = 3.
5 5 15 98. If A, B, C and D are four points in the plane
= (λ1 , λ 2 , λ 3) =  , , 
11 4 11 19 2 2 2 2
λ2 = such that AB + CD = BC + DA = 100 ,
5 5 5 5
then AC ⋅ BD = [23 April 2018, Shift-II]
96. If a = 2i + j − 3k, b = i − 2 j + k, 1
(a) 10 (b) 0 (c) (d) 1
c = − i + j − 4 k and d = i + j + k, then 10
|(a × b) × (c × d)| = [23 April 2018, Shift-I]
Sol. (b)
(a) 5 114 (b) 5 94 (c) 5 124 (d) 5 104
Let A, B, C and D are vertices of a quadrilateral
Sol. (a) So,
Given that, a = 2i$ + $j − 3k$ | AC |2 + | BD |2 = | AB |2 + | CD |2 + 2 BC ⋅ AD …(i)
b = $i − 2$j + k$ and
c = − $i + $j − 4 k$ and d = $i + $j + k$ | AC |2 + | BD |2 = | AD |2 + | BC |2 + 2AB ⋅ DC …(ii)
Vector Algebra 301

C Sol. (b)
D Since, (b × c) × (c × a) = [a b c] c
(c × a) × (a × b) = [a b c] a
(a × b) × (b × c) = [a b c] b
O
and [b + c c + a a + b] = 2[a ⋅ b ⋅ c]
A and [b × c c × a a × b] = [a b c]2
B [(b × c) × (c × a) (c × a) × (a × b)
From Eqs. (i) and (ii), (a × b) × (b × c)]
So,
BC ⋅ AD = AB ⋅ DC [b + c c + a a + b] [b × c c × a a × b]
⇒(OC − OB) ⋅ (OD − OA) = (OB − OA) ⋅ (OC − OD) [[a b c] c [a b c] a [a b c] b]
=
⇒ OC ⋅ OD − OC ⋅ OA − OB ⋅ OD + OB ⋅ OA 2[a b c] [a b c]2
= OB ⋅ OC − OB ⋅ OD
− OA ⋅ OC + OA ⋅ OD [a b c]3 [a b c] [a b c]
= = =1
⇒ OC ⋅ OD + OB ⋅ OA = OB ⋅ OC + OA ⋅ OD 2[a b c]3 2
⇒ OC ⋅ (OD − OB) + OA ⋅ (OB − OD) = 0  1 1 0 
⇒ OC ⋅ BD + OA ⋅ DB = 0 ⇒ BD ⋅ (OC − OA) = 0 Q [a b c] = 0 1 1 = 2
 
⇒ BD ⋅ AC = 0.  1 0 1 
99. If a = $i + 2 $j + 3k$ , b = 2$i + 3 $j + 2k$ and c is a
102. In ∆ABC, if D and E are the mid-points of the
vector perpendicular to b , then sides BC and CA respectively, then
 a ⋅ (b × c)  a ⋅ b   a ⋅ c 2(AD + EB) = [24 April 2018, Shift-I]
 2 
(b × c) +  2  b +  2 = 3
 b × c   b   c 
(a) 3AB (b) AB (c) 2AB (d) 3BC
2
[23 April 2018, Shift-II]
Sol. (a)
(a) 14 (b) 14 (c) 13 (d) 17
A
Sol. (a)
Any vector r can be written in linear combination E
of two non-parallel vector b and c, as
 r⋅ b   r⋅ c   r ⋅ (b × c) 
r=  b+   c+   (b × c)
 | b |2   | c |2   | b × c |2  B D C
So, r = a ⇒ | r | = | a| = 1 + 4 + 9 = 14. 1
AD = AC + CD = AC + CB … (i)
100. If a, b , c are three vectors such that 2
[D is mid-point of CB]
a = 1, b = 2 , c = 3 and
1
2 a ⋅ b = b ⋅ c = c ⋅ a = 2, Then, [ a b c]2 = EB = EC + CB = AC + CB … (ii)
2
[23 April 2018, Shift-II] [E is mid-point of AC]
(a) 15 (b) 14 (c) 12 (d) 8 On adding Eqs. (i) and (ii), we get
3 3
Sol. (a) AD + EB = AC + CB
2 2
a ⋅b a ⋅b a ⋅c 1 1 2
3 3
[a b c]2 = b ⋅ a b ⋅ b b ⋅ c = 1 4 2 = 15 = (AC + CB) = AB
2 2
c⋅a c⋅b c⋅c 2 2 9 ⇒ 2(AD + EB) = 3AB
103. If a , b ,c are vectors of equal magnitude such
101. If a = $i + $j , b = $j + k$ , c = $i + k$ , then that (a , b) = α, (b , c) = β, (c , a) = γ, then the
[(b × c) × (c × a) (c × a) × (a × b)(a × b) × (b × c)] minimum value of cos α + cos β + cos γ is
[b + c c + a a + b] [b × c c × a a × b] [24 April 2018, Shift-I]
[23 April 2018, Shift-II] 3 3 1 1
(a) (b) − (c) (d) −
(a) 0 (b) 1 (c) 3 (d) 2 2 2 2 2
302 AP EAMCET Chapterwise Mathematics

Sol. (b) 1
= (BC) ⋅ (AM)
a ⋅b b⋅c 2
We have, cos α = , cosβ =
|a||b| |b||c| 1
= | BC || AM |
c⋅a 2
and cos γ =
|c||a| Area of ∆ABC
1
Also, it is given that|a| = |b| = |c| = λ (say), = |α × β + β + γ + γ × α |
2
where λ > 0.
a ⋅b b⋅c c⋅a 1 1
Then, cos α = 2 ; cos β = 2 and cos γ = 2 ⇒ | BC || AM | = |α × β + β × γ + γ × α |
λ λ λ 2 2
1 | α × β + β × γ + γ × α|
⇒ cos α + cos β + cos γ = 2 ⇒ | AM | =
λ |γ − β |
(a ⋅ b + b ⋅ c + c ⋅ a) …(i)
105. If a , b , c are three non-coplanar vectors, then
We know that,
match the items of List-I with those of
|a + b + c| = (a + b + c) ⋅ (a + b + c)
2
List-II. [24 April 2018, Shift-I]
= [|a|2 + |b|2 + |c|2 + 2(a ⋅ b + b ⋅ c + c ⋅ a)]
List-I List-II
⇒[λ2 + λ2 + λ2 + 2(a ⋅ b + b ⋅ c + c ⋅ a)] ≥ 0
A. [b × c c × a a × b ] = I. [a b c ]2
[Q |a + b + c|2 ≥ 0]
⇒ 2(a ⋅ b + b ⋅ c + c ⋅ a) ≥ − 3λ2 B. [a × b a × c b ] = II. 2 [a b c ]
3
⇒ a ⋅ b + b ⋅ c + c ⋅ a ≥ − λ2 …(ii) C. [a + b b + c c + a ] = III. [ a b c ]
2
Now, from Eqs. (i) and (ii), we get D. For three mutually IV. [a b c ] [a ⋅ b ]
perpendicular unit
cos α + cos β + cos γ ≥ 2  − λ2  = −
1 3 3 V. 0
vectors a , b, c :
λ  2  2
Thus, the minimum value of cos α + cos β + cos γ The correct answer is
3
is − . A B C D
2 (a) III IV V II
104. In ∆ABC if A(α), B(β) and C(γ) are the position (b) IV V II III
vectors of the vertices, then the length of the (c) I IV V III
perpendicular from A to BC is (d) I IV II III
[24 April 2018, Shift-I] Sol. (d)
(a)|α × β | + |β × γ | + |γ × α | (A) [b × c c × a a × b]
(b)| α × β + β × γ + γ × α |
| α × β + β × γ + γ × α| = (b × c) × (c × a) ⋅ (a × b)
(c)
| α − β| = {d × (c × a)} ⋅ (a × b)
| α × β + β × γ + γ × α| [let d = b × c]
(d)
| γ − β| = {(d ⋅ a)c − (d ⋅ c)a } ⋅ (a × b)
= {[a b c]c − [b c c]a } ⋅ (a × b)
Sol. (d)
= {[a b c]c − 0} ⋅ (a × b)
Let ABC be a triangle and α, β, γ be the position
vector of the vertices A, B, C respectively. Let AM = [a b c] {c ⋅ (a × b)}
be the perpendicular from A to BC = [a b c] [c a b] = [a b c]2
Then, Area of ∆ABC (B) [a × b a × c ⋅ b]
A (α) = {(a × b) × (a × c)} ⋅ b
= {d × (a × c)} ⋅ b
[let d = a × b]
= {(d ⋅ c)a − (d ⋅ a)c} ⋅ b
B (β) M C (γ ) = {(a × b) ⋅ c} a ⋅ b − {(a × b) ⋅ a } c ⋅ b
Vector Algebra 303

= [a b c] [a ⋅ b] − [a b a ] [c ⋅ b] 106. a, b and c are three unit vectors such that no


= [a b c] [a ⋅ b] − 0 two of them are collinear. If
= [a b c] [a ⋅ b] b = 2 {a × (b × c)} and α is the angle between
(C) [a + b b + c c + a ] a , c and β is the angle between a , b, then
= (a + b) ⋅ {(b + c) × (c + a)} cos (α + β) = [24 April 2018, Shift-I]
3 3 1 1
= (a + b) ⋅ (b × c + b × a + c × c + c × a) (a) (b) − (c) (d) −
2 2 2 2
= (a + b) ⋅ (b × c + b × a + c × a) [c × c = 0]
= a ⋅ (b × c) + a ⋅ (b × a) + a ⋅ (c × a) Sol. (b)
+ b ⋅ (b × c) + b ⋅ (b × a) + b ⋅ (c × a) We have,
b = 2{a × (b × c)}
= [a b c] + 0 + 0 + 0 + 0 + [b c a ]
b = 2{(a ⋅ c)b − (a ⋅ b)c}
= [a b c] + [a b c]
= 2[a b c] b = 2(a ⋅ c)b − 2(a ⋅ b)c
(D) a , b, c are three mutually perpendicular On comparing b and c, we get
units vector ∴ 2(a ⋅ c) = 1 and a ⋅ b = 0
a ⋅b = b⋅c = c⋅a = 0 α is a angle between a, c and β is the angle
= [(a + b + c) b × c c × a ] between a, b.
= (a + b + c) ⋅ {(b × c) × (c × a) 1
|a ||c | cosα = , a ⋅ b = 0
= (a + b + c) ⋅ (a × b) 2
[Q b × c = a and c × a=b] π
cos α = cos , and cosβ = cos
π
=a ⋅ (a × b) + b ⋅ (a × b) + c ⋅ (a × b) 3 2
= 0 + 0 + [c a b] π
∴ α = π / 3 and β =
= [a b c] 2
Hence, A → I, B → IV, C → II, D → III. ⇒ cos(α + β) = − sin π / 3 = − 3 / 2.
22
Three Dimensional
Geometry
1. If a line makes angles α ,β , γ with the positive 3. If a line makes angles 90°, 135° and 45° with
directions of X , Y and Z-axes respectively, the positive directions of X , Y , Z-axes
then the value of sin 2 α + sin 2 β + sin 2 γ = respectively. Then, its direction cosines are
......... [17 Sep. 2020, Shift-II]
[17 Sep. 2020, Shift-I]
−1
(a)  0,
1 
(b)  0,
1 1 
(a) 1 (b) 2 (c) 3 (d) −1 ,  , 
 2 2  2 2
Sol. (b) −1 − 1 − 1
(c)  0, (d)  0,
1
,  , 
Direction cosines will be (cosα , cosβ , cos γ)  2 2  2 2
So, cos2 α + cos2 β + cos2 γ = 1
Sol. (a)
⇒ sin2 α + sin2 β + sin2 γ = 2 Given angles made by line with axes are
2. The direction cosines of the line which is α = 90°, β = 135°, γ = 45°
perpendicular to the lines with direction So, direction cosines are,
cosines proportional to 1, − 2 , − 2 and 0 , 2 , 1 cosα = cos 90° = 0
1
is given by [17 Sep. 2020, Shift-I] cosβ = cos135° = −
2 1 2 −2 −1 −2 2
(a) , , (b) , ,
1
3 3 3 3 3 3 cos γ = cos 45° =
2 1 −2 2 −1 2 2
(c) , , (d) , ,
3 3 3 3 3 3
4. The distance of the plane 2 x − y − 2z − 9 = 0
Sol. (d) from the origin is ........ units.
Let l , m, n be the direction cosines of the required [17 Sep. 2020, Shift-II]
line.
(a) 3 (b) 3 (c) 1 (d) 9
Since, it is perpendicular to the lines whose
direction cosines are proportional to1, − 2, − 2 and Sol. (a)
0, 2, 1 respectively Distance of plane Ax + By + Cz + d = 0 from (x1 y1 z1)
Thus, l − 2m − 2n = 0 and 2m + n = 0 is given by
On solving Ax1 + By1 + Cz1 + D
d=
l m n
= = A 2 + B2 + C 2
2 −1 2
By comparision, substituting values
Thus, DR’s of required line are proportional to
(2, −1, 2) 2 × 0 + (−1)(0) + (−2)(0) − 9 −9
d= = = 3 units.
Hence, direction cosines are =  , − , 
2 1 2
22 + (−1)2 + (−2)2 9
 3 3 3
Three Dimensional Geometry 305

5. If origin is the centroid of the triangle PQR direction ratios of line joining points (–1,–2,1) and
(1, 2,l) is –2,–4,1–l.
with vertices P(2 a , 2 , 6), Q(− 4 , 3 b, − 10) and
∴ Both line segments are parallel, so
R(8 , 14 , 2 c), then the values of a, b, c
k − 4 −4 −4
respectively are [17 Sep. 2020, Shift-II] = = ⇒ k = 2 and l = 5
16 16 −2 −4 1 − l
(a) 2, ,−2 (b) −2, − ,−2 ∴ k+ l = 7
3 3
16 16
(c) − 2 , − , 2 (d) − 2 , , − 2 8. Equation of the line passing through the
3 3 intersection of the plane x + 2 y + 3 z = 4 and
Sol. (c) x −1 y +1 z −1
the line = = and parallel to
Centroid is given by, 2 1 −1
x + x2 + x3 the vector (2 $i − 3 $j) × (i$ + 2 $j − k$ ) is
x= 1
3
[18 Sep. 2020, Shift-I]
y + y2 + y3
y= 1 x− 5 y−1 z+1 x− 5 y−1 z−1
3 (a) = = (b) = =
z1 + z2 + z3 3 2 −7 −3 −2 7
z= x− 5 y−1 z+1 x− 5 y−1 z+ 1
3 (c) = = (d) = =
−3 −2 −7 −3 2 7
Using values given,
2a − 4 + 8 Sol. (c)
0= ⇒a = − 2
3 The general point p on the line
2 + 3b + 14 −16 x −1 y + 1 z −1
Also, 0= ,b= = = =r (Let) …(i)
3 3 2 1 −1
6 − 10 + 2C is P(2r + 1, r − 1, 1 − r).
and 0= ,c=2
3 Let the point P is the intersection of line (i) and
6. Which of the following is false? the plane x + 2y + 3z = 4, so
[18 Sep. 2020, Shift-I] 2r + 1 + 2r − 2 + 3 − 3r = 4 ⇒ r=2
1. If (a , b, c) are direction ratios of a line, So, point P(5, ⋅1,−1)
then a 2 + b2 + c 2 ≠ 1. $i $j $
k
2. The direction cosines of a line can be its $ $ $ $ $
Now, (2i − 3 j) × (i + 2 j − k) = 2 −3 0
direction ratios but not vice-versa. 1 2 −1
3. If (l , m , n) is one set of direction cosines,
then (− l , − m , − n) is also a valid set. $ (4+ 3) = 3$i + 2$j + 7k
= $i (3) − $j(–2) + k $
4. If (l1 , m1 , n1) and (l2 , m2 , n2) are direction ∴ Equation of required line is
cosines of perpendicular lines, then x − 5 y −1 z + 1
= =
l1 l2 + m1 m2 + n1 n2 = 1. 3 2 7
(a) 1 (b) 2 (c) 3 (d) 4 x − 5 y −1 z + 1
or = =
Sol. (d) −3 −2 −7
If the direction cosines of two perpendicular lines 9. The equation of the plane through the
are l1 , m1 , n1 and l 2 , m2 , n2, then intersection of the planes x + 2 y + 3 z − 4 = 0
l1 l 2 + m1 m2 + n1 n2 = 0. and 4 x + 3 y + 2 z + 1 = 0 and passing through
7. If the line joining the points (k ,3, 4), (4 , 7 , 8) is the origin is ......... [18 Sep. 2020, Shift-II]
parallel to the line joining the points (a) 17 x + 14 y + 11z = 0 (b) 7 x + 4 y + z = 0
(− 1, − 2 , 1), (1, 2 , l) , then k + l = (c) x + 14 y + 11z = 0 (d) 17 x + y + z = 0
[18 Sep. 2020, Shift-I] Sol. (a)
(a) 2 (b) 5 (c) 7 (d) − 3 Required Equation of plane is given by,
Sol. (c) (x + 2y + 3z − 4) + λ (4 x + 3y + 2z + 1) = 0 …(i)
The direction ratios of line joining points (k,3,4) Since above plane passing through (0, 0, 0)
and (4,7,8) is k–4, –4, –4 and similarly the − 4 + λ ()
1 =0 ⇒ λ=4
306 AP EAMCET Chapterwise Mathematics

Put λ = 4 in Eq. (i) (a) 9, 6,− 2 (b) −9, − 6,2


(x + 2y + 3z − 4) + 4 (4 x + 3y + 2z + 1) = 0 (c) −9, 6, − 2 (d) 9, − 6, − 2
17 x + 14 y + 11 z = 0 Sol. (c)
Hence, option (a) is correct. Direction cosines of a line are given as
9 6 2
10. The feet of perpendicular from the point − , ,− respectively, so the direction ratios
A(1, 0, 3) to the join of the points B(4 , 7 , 1) 11 11 11
and C (3, 5, 3) is [21 Sep. 2020, Shift-I] are −9, 6, −2 respectively.
Hence, option (c) is correct.
(a)  , ,  (b)  , , 
5 7 17 10 17 6
3 3 3   3 3 3 13. The line segment joining the points A(2,3, 4)
(c)  0, ,  (d)  , , 
1 3 1 3 7 and B(− 3 ,5 , − 4) intersects yz-plane at the
 2 2  5 5 5 point [21 Sep. 2020, Shift-II]

(a)  0, , 
Sol. (a) 19 4
(b) (0, 4, 5)
Since equation of line joining points B(4, 7, 1) and  5 5
(c)  9, ,4
C(3, 5, 3) is 14
(d) (0, 0, 0)
x − 4 y − 7 z −1  5 
= =
4− 3 7− 5 1− 3 Sol. (a)
x − 4 y − 7 3 −1
⇒ = = =λ (let) …(i) Let yz-plane intersects the line joining points
1 2 −2 A(2, 3, 4) and B(−3, 5,−4) in the ratio λ :1 at point
Now the general point over the above line (i) is M, then
P(λ + 4, 2λ + 7, 1 − 2λ) −3λ + 2 5λ + 3 −4λ + 4 
M  , , 
Let point P is the foot of perpendicular of point  λ +1 λ +1 λ +1 
A(1, 0, 3) over the line (i), so Q On yz-plane, x-coordinate = 0
AP ⊥ line ⇒ λ = 2/ 3
⇒ (λ + 4 − 1)(1) + (2λ + 7 − 0)(2)
So, point M have coordinates  0, , 
19 4
+ (1 − 2λ − 3)(−2) = 0  5 5
⇒ λ + 3 + 4λ + 14 + 4λ + 4 = 0 ⇒ 9λ + 21 = 0
Hence, option (a) is correct.
7
⇒ λ=−
3 14. If two lines are parallel to each other, then
∴ The coordinate of point P is which of the following is true? (if (l1 , m1 , n1)
and (l2 , m2 , n2) are direction cosines of the
 4 − 7 , 7 − 14 , 1 + 14  =  5 , 7 , 17 
    two lines). [21 Sep. 2020, Shift-II]
 3 3 3 3 3 3
(a) l1 l2 + m1m2 + n1n2 = 0 (b) Σ(l1 l2 − m2 m1 )2 = 0
Hence, option (a) is correct.
l m n
(c) 1 = 1 = 1 (d) l1l2 + m1m2 + n1n2 = 1
11. The equation of the plane whose intercepts l2 m2 n2
on x , y , z axes are 1, 2, 4 respectively is
Sol. (c)
[21 Sep. 2020, Shift-II]
The two lines having direction cosines l1 , m1 , n1
(a) 4 x + 2 y + z = 4 (b) 4 x + 2 y + z = 2
and l 2 , m2 , n2 respectively are parallel to each other,
(c) 4 x + 2 y + z = 1 (d) x + 2 y + 4 z = 0 then
Sol. (a) l1 m1 n1
= =
Equation of plane having intercepts with l 2 m2 n2
co-ordinate axes as 1,2, 4 respectively is Hence, option (c) is correct.
x y z
+ + = 1 ⇒ 4 x + 2y + z = 4 15. Equation of the plane passing through the
1 2 4
x −1 y − 2 z − 5
Hence, option (a) is correct. intersection of the lines= =
1 2 −3
12. The direction cosines of a line are x +5 y− 4 z+3
−9 6 −2 and = = and parallel to the
, , respectively. Then its direction 3 −1 4
11 11 11 xy-plane is [22 Sep. 2020, Shift-I]
ratios are [21 Sep. 2020, Shift-II] (a) z = 4 (b) z = 2 (c) z = 5 (d) z = −5
Three Dimensional Geometry 307

Sol. (c) 2x − 3y + 6z + 21 = 0
Equation of given lines and 2x − 3y + 6z − 14 = 0
x −1 y − 2 z − 5 | 21 − d | | d + 14 |
= = = r1 (let) ...(i) So, = ⇒ 21 − d = d + 14
1 2 −3 4 + 9 + 36 4 + 9 + 36
x+ 5 y−4 z+ 3 7
and = = = r2 (let) ...(ii) ⇒ 2d = 7 ⇒ d =
3 −1 4 2
are intersecting lines, then on equating the ∴Equation of required plane is
parametric points on lines, we get 4 x − 6 y + 12 z + 7 = 0
r1 + 1 = 3 r2 − 5 18. The points (5, − 4 , 5), (−3, − 3, 2) and (−1, − 6 , 8)
2r1 + 2 = − r2 + 4
form …… [22 Sep. 2020, Shift-I]
and − 3 r1 + 5 = 4 r2 + 3 (a) an isosceles triangle
So on solving, r1 = 0 and r2 = 2 (b) an equilateral triangle
∴Point of intersection is A(1, 2, 5). (c) a right-angled isosceles triangle
Now the equation of plane parallel to xy-plane and (d) a right-angled triangle
passes through point A is z = 5.
Sol. (a)
Hence, option (c) is correct.
Given points A(5, − 4, 5), B(− 3, − 3, 2) and
16. Find the angle between the planes C(− 1, − 6, 8)
x + 2 y + 2 z − 5 = 0 and 3 x + 3 y + 2 z − 8 = 0 ∴ AB = 64 + 1 + 9 = 74
[22 Sep. 2020, Shift-I]
BC = 4 + 9 + 36 = 7
(a) cos 
3 
(b) cos −1 
−1 13 
  CA = 36 + 4 + 9 = 7
 22   3 22  and

(c) cos −1 
1 
(d) cos −1  
13 Q Length of sides BC and CA are equal, but

 3 22   31 BC 2 + CA2 ≠ AB2
∴∆ABC is an isosceles triangle.
Sol. (b)
Equation of given planes are 19. Two lines whose direction cosines are given
x + 2y + 2 z − 5 = 0 by al + bm + cn = 0 and fmn + gnl + hlm = 0
and 3x + 3y + 2 z − 8 = 0 are perpendicular to each other if ………
[22 Sep. 2020, Shift-I]
So, angle between them is
f g h f g h
 (3 × 1) + (3 × 2 ) + (2 × 2 )  (a) + + =0 (b) − − = 0
θ= cos− 1   a b c a b c
 32 + 32 + 22 12 + 22 + 22  f
(c) +
g h
− =0
f g h
(d) − + = 0
a b c a b c
 13 
= cos− 1  
 22 (3)  Sol. (a)
Let the direction cosines of lines are l1 , m1 , n1 and
θ = cos − 1 
13 
⇒  l 2 , m2 , n2.
 3 22 
Since, al + bm + cn = 0 and, fmn + gnl + hlm = 0
17. The equation of the plane mid-parallel to the So, on eliminating ‘l’, we get
planes 2 x − 3 y + 6 z + 21 = 0 and bm + cn 
2 x − 3 y + 6 z − 14 = 0 is given by ……… fmn + (gn + hm) −  =0
 a 
[22 Sep. 2020, Shift-I]
⇒ afmn = bhm2 + cgn2 + (bg + ch) mn = 0
(a) 4 x + 6 y − 12 z + 7 = 0
2
(b) 4 x − 6 y − 12 z − 7 = 0 ⇒ bh  + (bg + ch − af)  + cg = 0 having
m m
(c) 4 x − 6 y + 12 z + 7 = 0  n  n
(d) 4 x + 6 y + 12 z − 7 = 0 m1 m
roots and 2 , so
Sol. (c) n1 n2
Let equation of plane 2x − 3y + 6z + d = 0 is mm
product of roots = 1 2 =
cg
equidistance from two parallel planes n1 n2 bh
308 AP EAMCET Chapterwise Mathematics



Similarly on eliminating ‘m’, we get Dr’s of AB = (9 − 12, 13 − 21, 15 − 10)
al + cn 
gnl + (hl + fn) −  =0 = (−3, − 8, 5) = (3, 8, − 5)
 b  ←

⇒ bgnl = ahl 2 + cfn2 + (ch + af) ln = 0 Equation of line AB is
x − 9 y −13 z −15
2 = = =λ
⇒ ah  + (ch + af − bg)  + cf = 0, having roots
l l
3 8 −5
 n  n
x−9 y − 13 z − 15
l1 l ll cf = λ, = λ, =λ
and 2 , so product of roots 1 2 = . 3 8 −5
n1 n2 n1 n2 ah
x = 3λ + 9, y = 8λ + 13, z = − 5λ + 15
Q The lines are perpendicular.
∴Q = (3λ + 9, 8λ + 13, − 5λ + 15)
∴ l1 l 2 + m1 m2 + n1 n2 = 0
Dr’s of PQ = (3λ + 9 − 5, 8λ + 13 − 7, − 5λ + 15 − 3)
⇒  cg + cf + 1 n n = 0 ⇒ f + g + h = 0
  Dr’s of PQ = (3λ + 4, 8λ + 6, − 5λ + 12)
 bh ah  1 2 a b c ←

Since, AB ⊥ PQ
20. The cartesian equation of a line
∴ a1 a 2 + b1 b2 + c1 c 2 = 0
2 x − 3 = 3 y + 1 = 5 − 6 z. The vector equation
3(3λ + 4) + 8(8λ + 6) − 5(−5λ + 12) = 0
of the line passing through the point
9λ + 12 + 64λ + 48 + 25λ − 60 = 0 ⇒ λ = 0
(7, − 5, 0) and parallel to the given line is
Put, λ = 0 in Q
[22 Sep. 2020, Shift-I]
∴ Q = (9, 13, 15)
(a) r = (5$i − 7 $j ) + λ(3$i + 2 $j − k$ ) ∴ Foot of Perpendicular Q = (9, 13, 15)
(b) r = (7 $i + 5$j ) + λ (3$i − 2 $j + k$ ) Hence, option (d) is correct.
(c) r = (7 $i − 5$j ) + λ(3$i + 2 $j − k$ )
22. The Cartesian equation of the line passing
(d) r = (− 5$i + 7 $j ) + λ (−3$i − 2 $j − k$ ) through the point (−1, 3 , − 2) and
Sol. (c) x y z
perpendicular to the lines = = and
Equation of given line is 1 2 3
2x − 3 = 3y + 1 = 5 − 6z
x +2 y −1 z +1
= = is
3 1 5 −3 2 5 [22 Sep. 2020, Shift-II]
x− y+ z− x−1 y + 3 z−2 x−1 y+ 3 z − 2
2= 3= 6 (a) = = (b) = =
3 2 −1 2 7 4 −2 −7 −4
x+ 1 y + 3 z+ 2 x+1 y − 3 z+ 2
∴Equation of line passes through point (7, − 5, 0) (c) = = (d) = =
$ is 2 7 4 2 −7 4
and having parallel vector 3$i + 2$j − k
$ $ $ $)
r = ( 7 i − 5 j ) + λ( 3 i + 2 $j − k
Sol. (d)
P = (−1, 3, − 2)
21. The foot of perpendicular from (5, 7 , 3) to the Let dr’s or required line be (a , b , c)
x y z
join of (9 , 13 , 15) and (12 , 21, 10) is ……… Dr’s of line = = are (1, 2, 3)
[22 Sep. 2020, Shift-II] 1 2 3
(a) (−2, − 19, 7 ) (b) (2, 19, 7 ) Given that required line perpendicular to above line
(c) (2, 2, 3) (d) (9, 13, 15) ⇒ a1 a 2 + b1 b2 + c1 c 2 = 0
Sol. (d) ⇒ a + 2b + 3c = 0 … (i)
Let A = (9, 13, 15) Similarly required line perpendicular to
x + 2 y −1 z + 1
P = =
−3 2 5
⇒ −3 a + 2 b + 5 c = 0 … (ii)
On solving Eqs. (i) and (ii), we get
A B a b c
Q = =
(x, y, z) 10 − 6 −9 − 5 2 + 6
B = (12, 21, 10) ⇒
a
=
b
=
c
P = (5, 7, 3) and Q = (x , y, z) 2 −7 4
Three Dimensional Geometry 309

∴ Required line is passing through (−1, 3, − 2) and 6m + 3n


= 2 ⇒ 6m + 3n = 2m + 2n
having dr’s (2, −7,4) m+ n
x +1 y − 3 z + 2
∴ = = 4m = − n ⇒
m
=−
1
2 −7 4 n 4
23. The direction cosines of two lines are ∴ m: n = −1: 4
3 1 3 − 3 1 3 We know that,
, , and , , . Then the If point P divides a line segment in the ratio of
2 4 4 2 4 4 m : n, then its harmonic conjugate will divide same
angle between the lines is equal to segment in the ratio of − m : n
[22 Sep. 2020, Shift-II] ∴ Required ratio = − m : n = 1 : 4
∴ Required harmonic conjugate
(a) 30° (b) 60° (c) 45° (d) 90°
 mx 2 + nx1 my2 + ny1 mz2 + nz1 
Sol. (b) = , , 
 m+ n m+ n m+ n 
Given,
1(6) + 4(3) 1(−17) + 4(−2) 1(−4) + 4(2) 
 3 1 3 − 3 1 3 =  , , 
Dr’s are  , ,  and  , ,   1+ 4 1+ 4 1+ 4 
 2 4 4   2 4 4 
 18 −25 4   18 4
= , ,  =  , − 5, 
∴ Angle between lines is  5 5 5  5 5
cos θ = |l1 l 2 + m1 m2 + n1 n2| Hence option (d) is correct.
3 3 1  1  3  3
−  +   +   25. If P(−3, − 2, 4), Q(−9 , − 8 , 10) and R(−5, − 4 , 6)
2  2  4  4 4  4 
are collinear, then the ratio in which R
−3 1 3 −3 1
= + + = + divides PQ is [23 Sep. 2020, Shift-I]
4 16 16 4 4
(a) 1 : 2 (b) 2 : 1
1
cos θ = ⇒ θ = 60° (c) 3 : 1 (d) 1 : 3
2
Hence, option (b) is correct. Sol. (a)
Given, P = (−3, −2, 4)
24. The harmonic conjugate of (2, 3, 4) with Q = (−9, −8, 10)
respect to the points (3 , − 2 , 2) and (6 , − 17 , − 4) R = (−5, −4, 6)
is [22 Sep. 2020, Shift-II]
Let R divide PQ in the ratio of K :1
(b)  , , 
1 1 1
(a) (11, − 16, 2 ) kx + 1 x1 ky2 + 1 ⋅ y1 kz2 + 1 ⋅ z1 
 2 3 4 ∴ R =  2 , , 
 k +1 k +1 k +1 
18 −5 4 
(c) (0, 0, 0) (d)  , ,  k (−9) + (−3) k (−8) + 1(−2) k (10) + 1.4 
 5 1 5 (−5, − 4, 6) =  , , 
 k+1 k+1 k+1 
Sol. (d) −9k − 3
∴ −5 =
Let A = (3, − 2, 2) k +1
B = (6, −17, −4) ⇒ −5k − 5 = − 9k − 3
P = (2, 3, 4) 1
4k = 2⇒ k =
Let P divides AB in the ratio of m : n. 2
 mx + nx1 my2 + ny1 mz2 + nz1  ∴ Required ratio is 1 : 2
P= 2 , ,
 m+ n m+ n m + n  Hence, option (a) is correct.
 m(6) + n(3) m(−17 ) + n(−2) 26. The angle between the lines with direction
(2, 3, 4) =  , ,
 m+ n m+ n ratios (2 , − 2 , 1) and (1, − 2 , 2) is
m(−4 ) + n(2)  [23 Sep. 2020, Shift-I]

(a) cos  
−1 
(b) cos −1  
m+ n 4 8

 9  9
 6m + 3n −17m − 2 n −4m + 2 n  π π
(2, 3, 4) =  , ,
m + n 
(c) (d)
 m+ n m+ n 6 2
310 AP EAMCET Chapterwise Mathematics

Sol. (b) 29. The plane 3 x + 4 y + 6 z + 7 = 0 is rotated


(2, − 2, 1) (1, − 2, 2)
Given, Dr's are and about the line r = (i$ + 2 $j − 3 k$ ) + t(2 i$ − 3 $j + k$ )
a1 b1 c1 a 2 b2 c 2
Angle between line is until the plane passes through origin. The
a1 a 2 + b1 b2 + c1 c 2 equation of the plane in the new position is
cosθ = [20 April 2019, Shift-I]
a1 + b12 + c12 a 22 + b22 + c 22
2

(2) (1) + (−2)(−2) + (1)(2) 8 (a) x + y + z = 0 (b) 6 x + 3 y − 4 z = 0


= = (c) 4 x − 5 y − 2 z = 0 (d) x + 2 y + 4 z = 0
4+ 4+ 1 1+ 4+ 4 3⋅ 3
⇒ θ = cos−1  
8 8 Sol. (a)
cosθ =
9  9 Equation of plane passes through the origin and
Hence, option (b) is correct. containing the line r = ($i + 2$j − 3k $ ) + t(2$i − 3$j + k
$)

27. If (a1 , b1 , c1), (a 2 , b2 , c2) are direction cosines of is


$ ) × (2$i − 3$j + k
(r − 0) ⋅ [($i + 2$j − 3k $ )] = 0
two lines making an angle θ with each other,
then cosθ = [23 Sep. 2020, Shift-I] $ (−3 − 4)] = 0
⇒ r ⋅[$i(2 − 9) − $j (1 + 6) + k
(a) a1 a2 + b1 b2 + c1 c 2 $) = 0
⇒ r ⋅ (−7$i − 7$j − 7k
(b)|a1 a2 + b1 b2 + c1 c 2|
$ ) = 0 or x + y + z = 0
⇒ r ⋅ ($i + $j + k
(c) (a1a2 + b1b2 + c1c 2 ) ( a12 a22 + b12 b22 + c12c 22 )
4 Hence, option (a) is correct.
(d)
3
30. If A(2, 4 , − 1), B(3, 6 , − 1) and C(4 , 5, 1) are three
Sol. (b)
Given, DC's are (a1 , b1 , c1) and (a 2 , b2 , c 2). consecutive vertices of a parallelogram, then
∴ cosθ = |a1 a 2 + b1 b2 + c1 c 2| [Q standard result] its fourth vertex is [20 April 2019, Shift-II]
Hence, option (b) is correct. (a) (1, 3, 3) (b) (1, 3, − 3)
(c) (3, 3, − 1) (d) (3, 3, 1)
28. A(2, 3, 5), B(α , 3, 3) and C(7 , 5, β) are the
Sol. (d)
vertices of a triangle. If the median through
A = (2, 4, − 1), B = (3, 6, − 1), C = (4, 5, 1)
A is equally inclined with the co-ordinate
α Let D = (x , y, z)
axes, then cos −1   = Since, diagonals of a parallelogram are bisect each
β [20 April 2019, Shift-I] other.
−1 π
(a) cos −1   (b) ⇒ Mid-point of AC = Mid-point of BD
 9 2  2 + 4 , 4 + 5 , −1 + 1  =  3 + x , 6 + y , −1 + z 
π    
(d) cos −1  
2  2
(c) 2 2   2 2 2 
3 5
 3, 9 , 0 =  3 + x , 6 + y , −1 + z 
   
Sol. (a)  2   2 2 2 
) B(α , 3, 3) and C(7, 5, β)
Given, points A(2, 3, 5, 3+ x 6 + y 9 −1 + z
∴ = 3; = ; =0
α+ 7 3 + β
∴ Mid-point of BC is D  , 4,  2 2 2 2
 2 2  3 + x = 6; 6 + y = 9; −1 + z = 0
Q Direction ratios of line joining points x = 3 ; y = 3; z = 1
α+ 7 3+ β  α + 3 β − 7
A(2, 3, 5) and D  , 4,  is  ,1, . ∴Fourth vertex D = (3, 3, 1).
 2 2   2 2  Hence, option (d) is correct.
Q The line segment AD is equally inclined with
the co-ordinate axes, so 31. If the line joining the points A(2, 3, −1) and
α+ 3 β−7
=1 = ⇒ α = − 1 and β = 9 B(3 , 5 , − 3) is perpendicular to the line joining
2 2 C(1, 2 , 3) and D(3 , y , 7), then y =
α
∴ cos−1   = cos−1  − 
1 [20 April 2019, Shift-II]
β  9
(a) 1 (b) 3
Hence, option (a) is correct. (c) 5 (d) 7
Three Dimensional Geometry 311

Sol. (c) |− 18 + 4 + 3| ( 11)2 11


= = =
Given, A = (2, 3 − 1) 61 11 61 11 61
B = (3, 5 − 3) 11 11
cosθ = ⇒ θ = cos−1
DR’s of AB = (3 − 2, 5 − 3, − 3 + 1) = (1, 2, − 2) 61 61
and C = (1, 2, 3) ∴Hence, option (c) is correct.
D = (3, y, 7)
33. A(3, 2, − 1), B(4 , 1, 1), C(6 , 2, 5) are three points.
∴ DR’s of CD = (3 − 1, y − 2, 7 − 3) = (2, y − 2, 4)
Given, AB ⊥ CD If D , E , F are three points which divide BC,
CA, AB respectively in the same ratio 2 : 1,
⇒ a1 a 2 + b1 b2 + c1 c 2 = 0
then the centroid of ∆DEF is
⇒ (1)(2) + 2(y − 2) + (−2) (4) = 0
[21 April 2019, Shift-I]
⇒ 2y − 10 = 0 ⇒ y = 5
(a)  , , 
13 5 5
Hence, option (c) is correct. (b) (13, 5, 5)
 3 3 3
(d)  , , 
32. A plane is making intercepts 2, 3, 4 on X , Y 11 4 1
(c) (4, 2, 1)
and Z-axes respectively. Another plane is  3 3 3
passing through the point (−1, 6 , 2) and is Sol. (a)
perpendicular to the line joining the points A(3, 2, –1)
(1, 2 , 3) and (−2 , 3 , 4). Then angle between the
two planes is [20 April 2019, Shift-II]
1
12
(a) 90° (b) cos −1
61 F E
−1 11 −1 5 O
(c) cos (d) cos 2
61 6
Sol. (c) B (4, 1, 1) 2 D 1 C(6, 2, 5)
Given, X- intercept (a) = 2
Y- intercept (b) = 3 2 × 6 + 4 × 1 2 × 2 + 1 × 1 2 × 5 + 1 × 1
Here, D =  , , 
Z - intercept (c) = 4  3 3 3 
x y z
=  , , 
∴Equation of the plane is + + =1 16 5 11
2 3 4  3 3 3
6 x + 4 y + 3z = 12 …(i)
Similarly, E =  , ,  ⇒ F =  , , 
12 6 3 11 4 1
Given, points are A = (−1, 6, 2)  3 3 3  3 3 3
B = (1, 2, 3) Let centroid of ∆DEF is O(x , y, z).
C = (−2, 3, 4) 16 12 11
+ +
DR’s of BC = (−2, − 1, 3 − 2, 4 − 3) = (−3, 1, 1) 3 = 13
∴x = 3 3
∴Equation of plane passing through A(−1, 6, 2) 3 3
and having DR’s (−3, 1, 1) is given by 5 6 4 11 1
+ + +1+
a(x − x1) + b(y − y1) + c(z − z1) = 0 5 3= 5
y = 3 3 3 = and z = 3
−3(x + 1) + 1(y − 6) + 1(z − 2) = 0 3 3 3 3
−3x − 3 + y − 6 + z − 2 = 0  13 5 5
Hence, coordinates are  , ,  .
 3 3 3
−3x + y + z − 11 = 0
− 3x + y + z = 11 …(ii) 34. If A = (1, 8, 4), B = (2, − 3, 1), then the
Angle between the planes (i) and (ii) is direction cosines of a normal to the plane
|a1 a 2 + b1 b2 + c1 c 2| AOB is [21 April 2019, Shift-I]
cosθ =
a12 + b12 + c12 a 22 + b22 + c 22 2 5 −7 2 10 7 10 − 19 10
(a) , , (b) , ,
78 78 78 9 90 90
|(6) (−3) + (4) (1) + (3) (1)| − 11 2 6 −9
= (c)
4
,
9
, (d) , ,
36 + 16 + 9 9 +1 + 1 218 218 218 11 11 11
312 AP EAMCET Chapterwise Mathematics

Sol. (b) 9
− −1 − k
2 − 3− 2
Given, A = (1, 8, 4) and B = (2, − 3, 1) ⇒ =
$ 2 1
∴ OA = $i + 8$j + 4k 11
$ ⇒ − − k = − 10
and OB = 2$i − 3$j + k 2
$ = OA × OB
11 9
Now, n ⇒ − + 10 = k ⇒ k =
| OA × OB | 2 2
$i $j k $ 36. The vector equation of the plane passing
Here, OA × OB = 1 8 4 through the points (1, −2, 5), (0, −5, −1) and
2 −3 1 (−3, 5, 0) is [21 April 2019, Shift-II]
(a) r = (1 − λ − 4µ ) $i − (2 + 3λ − 7µ ) $j + ( 5 − 6λ − 5µ ) k$
$ (− 3 − 16)
= $i (8 + 12) − $j(1 − 8) + k (b) r = (1 + λ + 4µ )$i − (2 − 3λ + 7µ )$j + ( 5 − 6λ − 5µ )k$
= 20 i + 7 j − 19k
$ $ $ (c) r = (1 − λ + 4µ ) $i − (2 + 3λ + 7µ ) $j + ( 5 − 6λ + 5µ )k$
(d) r = (1 + λ − 4µ ) i$ + (2 + 3λ − 7µ ) $j + ( 5 + 6λ − 5µ )k$
and | OA × OB | = (20)2 + 72 + (− 19)2
Sol. (a)
= 400 + 49 + 361 = 810 = 9 10 $
Given, a = $i − 2$j + 5k
$
20i$ + 7$j − 19k
∴ $ =
n b = − 5j − k
$ $
9 10
and c = − 3$i + 5$j
20 7 − 19
∴ Direction cosines are , , The vector equation of plane passing through
9 10 9 10 9 10 a , b, c is r = a + λ(b − a) + µ (c − a)
2 10 7 10 − 19 10 = $i − 2$j + 5k$ + λ {(−5$j − k $ ) − ($i − 2$j + 5k
$ )}
i.e. , ,
9 90 90 $ )}
+ µ {(−3$i + 5$j) − ($i − 2$j + 5k
x −1 y +1 z −1
35. If the two lines= = and = $i − 2$j + 5k$ + λ(− $i − 3$j − 6k $ ) + µ (−4$i + 7$j − 5k
$)
2 3 4 $
x −3 y− k z = (1 − λ − 4µ)$i − (2 + 3λ − 7µ)$j + (5 − 6λ − 5µ)k
= = have a point in common,
1 2 1 37. The angle made by the vector 2$i − $j + k$ with
then k = [21 April 2019, Shift-I] the plane represented by r ⋅ (i$ + $j + 2 k$ ) = 7 is
2 2 9
(a) (b) − (c) (d) 0 [21 April 2019, Shift-II]
9 9 2
(a) 30° (b) 60° (c) 45° (d) 75°
Sol. (c)
Sol. (a)
Given line is Given, equation of plane is
x −1 y + 1 z −1
= = =λ $) = 7
r ⋅ ($i + $j + 2k
2 3 4
$ = $i + $j + 2k$
So, (x , y, z) is (2λ + 1, 3λ − 1, 4λ + 1) and this point Here, n
is lies on given line. This point also lies on line. $ ) with the
∴Angle made by the vector (2i$ − $j + k
x−3 y−k z $
normal (i + j + 2k) is
$ $
= =
1 2 1 1 + (−1)()
(2)() 1 + ()(
1 2) 2−1 + 2
So, this point satisfies equation = =
2λ + 1 − 3 3λ − 1 − k 4λ + 1 22 + 12 + 12 12 + 12 + 22 6 6
= = 3 1
1 2 1 ⇒cosθ = = ⇒ cosθ = cos 60° ⇒θ = 60°
2λ − 2 4λ + 1 6 2
⇒ = ⇒ 2λ − 2 = 4λ + 1 So, angle made by the plane with the vector
1 1 = 90° − 60° = 30°
3
⇒ 2λ = − 3 ⇒ λ = −
2 38. In a triangle ABC, if the mid-points of sides
3λ − 1 − k 2λ − 2 AB, BC, CA are (3, 0, 0), (0, 4, 0), (0, 0, 5)
Now, =
2 1 respectively, then AB2 + BC 2 + CA 2 =
3 3
3× − −1 − k 2× − − 2 [21 April 2019, Shift-II]
⇒ 2 = 2
(a) 50 (b) 200 (c) 300 (d) 400
2 1
Three Dimensional Geometry 313

Sol. (d) 40. The equation of the plane in normal form


Given, mid-point of sides AB, BC, CA are (3, 0, 0), which passes through the points (−2 , 1, 3),
(0, 4, 0), (0, 0, 5.
) (1, 1, 1) and (2 , 3 , 4) is [21 April 2019, Shift-II]
(a)   x +  −  y +  1 z = 1
2 2
A  
 3  3  3 3
(b)  −  x +   y +  − 1 z = 1
2 2
 
(3, 0, 0) (0, 0, 5)  3  3  3 3
−4   −6  z = 1
(c)  x+
 11  y +
   
 173   173   173  173
(d) 
4   −11  y +  6 z= 1
x+    
B (0, 4, 0) C  173   173   173  173
∴ A = (3, − 4, 5) Sol. (c)
B = (3, 4, − 5) and C = (−3, 4, 5) Given points are A(−2, 1, 3, ) B(1, 1, 1) and C(2, 3, 4).
∴ A = − 2$i + $j + 3k $ , B = $i + $j + k $ and
AB = (x 2 − x1) + (y2 − y1) + (z2 − z1)
2 2 2 2

C = 2$i + 3$j + 4k$


= (3 − 3)2 + (4 + 4)2 + (−5 − 5)2
Now, AB = ($i + $j + k $ ) − (−2$i + $j + 3k $ ) = 3$i − 2k
$
= 82 + 102 = 164
$ $ $ $ $
BC = (2i + 3 j + 4k) − (i + j + k) = i + 2 j + 3k
$ $ $ $
BC 2 = (−3 − 3)2 + (4 − 4)2 + (5 + 5)2
= 36 + 100 = 136 The normal of the plane n = AB × BC
And CA2 = (−3 − 3)2 + (4 + 4)2 + (5 − 5)2 $i $j k $

= (−6)2 + (8)2 = 36 + 64 = 100 ∴ AB × BC = 3 0 −2



 
1 2 3
Now, AB2 + BC 2 + CA2 = 164 + 136 + 100 = 400  
$ (6 − 0)
= $i(0 + 4) − $j(9 + 2) + k
39. The angle between a line with direction ratios
2, 2, 1 and the line joining the points (3, 1, 4) n = 4$i − 11$j + 6k $
and (7, 2, 12) is [21 April 2019, Shift-II] Now, equation of plane
(a) cos −1   (b) cos −1  
2 3
⇒ 4(x − 2) − 11(y − 3) + 6(z − 4) = 0
 3  4
⇒ 4 x − 8 − 11 y + 33 + 6z − 24 = 0
−2
(c) tan−1   (d) cos −1  
1
 3  3 ⇒ 4 x − 11 y + 6z + 1 = 0
As we know that, if ax + by + cz + d = 0 is the
Sol. (a) equation of plane, then the normal form is
Given that,    
 a x +  b y
Direction ratios of first line are 2, 2, 1 and direction
 a2 + b2 + c 2   a2 + b2 + c 2 
ratios of second line joining the points (3, 1, 4)    
and (7, 2, 12) are 7 − 3, 2 − 1, 12 − 4 i.e. 4, 1, 8.  c   d 
Let θ be the angle between the two lines + z +   =0
 a2 + b2 + c 2   a2 + b2 + c 2 
a1 a 2 + b1 b2 + c1 c 2    
∴ cosθ = ∴ Required normal form is
a1 + b12 + c12 a 22 + b22 + c 22
2
 4   −11 
2× 4 + 2×1 + 1 × 8  x +  y
=  42 + 112 + 62   42 + 112 + 62 
   
2 + 22 + 12 42 + 12 + 82
2
 6   1 
8 + 2+ 8 18 +  z +   =0
= =  42 + 112 + 62   42 + 112 + 62 
9 81 3× 9    
2 4x 11 y
⇒ cosθ = ⇒ −
3 16 + 121 + 36 16 + 121 + 36
6z 1
θ = cos−1   + + =0
2

 3 16 + 121 + 36 16 + 121 + 36
314 AP EAMCET Chapterwise Mathematics

4x 11 y 6z 1
⇒ − + + =0
173 173 173 173 Q
M
4x 11 y 6z 1
⇒ − + =− A
173 173 173 173 O
4x 11 y 6z 1
⇒ − + − =
173 173 173 173 r
P
⇒  −
4   11  y +  −6  z = 1 B
x +     R
 173   173   173  173
Then, the coordinates of P , Q and R are
41. The points A(2, − 1, 4), B(1, 0 , − 1), C(1, 2, 3) and
(l1 r , m1 r , n1 r), (l 2r , m2r , n2r) and (− l 2r , − m2r , − n2r)
D(2 , 1, 8) form a [22 April 2019, Shift-I] respectively.
(a) rectangle (b) square If A, B be the mid-points of PQ and PR, then OA
(c) rhombus (d) parallelogram and OB are along the bisectors of the lines
Sol. (d) direction ratios of OA are l1 + l 2 , m1 + m2, , n1 + n2
Given points are DR’s of OB are l1 − l 2, m1 − m2, n1 − n2
A(2, − 1, 4), B(1, 0, − 1), C(1, 2, 3) and D(2, 1, 8). Now, Σ(l1 + l 2)2 =1 + 1 + 2cosθ
∴ AB = 12 + 12 + 52 = 27 θ
( + cosθ) = 4 cos2
= 21
2
BC = 0 + 22 + 42 = 20 ∴DC’s of internal bisector are
CD = 12 + 12 + 52 = 27 l1 + l 2 m1 + m2 n1 + n2
, ,
θ θ θ
and DA = 0 + 22 + 42 = 20 2cos 2cos 2cos
2 2 2
Here, AB = CD and BC = DA
43. The distance of the plane
AC = 12 + 32 + 12 = 11
3 x + 4 y + 5 z + 19 = 0 from the point (1, − 1, 1)
and DB = 12 + 12 + 92 = 83 ∴ AC ≠ DB measured along a line parallel to the line with
direction ratios 2, 3, 1 is [22 April 2019, Shift-I]
∴ A, B, C and D form a parallelogram.
23 71
42. If l1 , m1 , n1 and l2 , m2 , n2 are direction cosines (a) (b) (c) 14 (d) 23
5 2 5 2
of OA and OB such that ∠ AOB = θ, where O
Sol. (c)
is the origin, then the direction cosines of the
According to given information
internal angular bisector of ∠ AOB are
[22 April 2019, Shift-I] (1,–1, 1)
l + l m + m2 n1 + n2
(a) 1 2 , 1 ,
θ θ θ
2 sin 2 sin 2 sin Dc’s (2, 3, 1)
2 2 2
l −l m − m2 n1 − n2
(b) 1 2 , 1 ,
θ θ θ
2 cos 2 cos 2 cos
2 2 2
l − l m − m2 n1 − n2 3x+4y+5z+19=0
(c) 1 2 , 1 ,
θ θ θ
2 sin 2 sin 2 sin Equation of lines is passing through (1, − 1, 1) and
2 2 2 having DC’s is (2, 3, 1)
l +l m + m2 n1 + n2
(d) 1 2 , 1 , x −1 y + 1 z −1
θ θ θ = = =r
2 cos 2 cos 2 cos 2 3 1
2 2 2
Here, (2r + 1, 3r − 1, r + 1) lie on plane.
Sol. (d)
∴These points satisfy the equation of plane.
Q l1 l 2 + m1 m2 + n1 n2 = cosθ
3 (2r + 1) + 4(3r − 1) + 5 (r + 1) + 19 = 0
Through origin O draw two lines parallel to given
lines and take two points on each at a distance r ⇒ 6r + 3 + 12r − 4 + 5r + 5 + 19 = 0
from O and a point R on QO produced so that OR = r ⇒ 23r + 23 = 0 ⇒ r = −1
Three Dimensional Geometry 315

So, point is (−2 + 1, − 3 − 1, − 1 + 1) i.e. (−1, − 4, 0) Now, equation of line II is


Now, required distance = (−2) + (−3) + 1 2 2 2
Y
line II
= 4 + 9 + 1 = 14
45°
44. The direction cosines of the line which is 90°
X′ X
perpendicular to the lines with direction
cosines proportional to (1, − 2 , − 2) and
(0, 2, 1) are [22 April 2019, Shift-II]
2 −2 1 2 −1 −2 Y′
(a) , , (b) , ,
3 3 3 3 3 3
2 −1 2 2 2 −1 y − 0 = tanθ(x − x1)
(c) , , (d) , , y = −1(x)
3 3 3 3 3 3
[Q tanθ = tan(90° − 45°) = − cot 45° = − 1]
Sol. (c)
y=−x …(ii)
Given direction cosines (1, − 2, − 2) and (0, 2, 1)
are perpendicular to line whose d’c is (a , b , c). From Eqs. (i) and (ii), number of lines is two,
whose passing through (0, 0, 0) and making an
∴ a − 2b − 2c = 0 …(i)
angle of 45°.
0 + 2b + c = 0 …(ii) Hence, number of lines = 2
By Eqs. (i) and (ii), we get
a b c a b c 46. The equation of the plane which passes
= = ⇒ = = through the point (2 , 5 , − 8) and perpendicular
− 2+ 4 −1 2 2 −1 2
to each of the planes 2 x − 3 y + 4 z + 1 = 0 and
∴ Required d’c are 4 x + y − 2 z + 6 = 0 is [22 April 2019, Shift-II]
 a b c (a) x + 10 y + 7 z + 4 = 0
= , ,  (b) x + 2 y + 2 z + 4 = 0
 a 2 + b 2 + c 2 a + b + c
2 2 2
a + b + c 
2 2 2
(c) 3 x + 2 y + 2 z = 0
 2 −1 2  2 1 2 (d) x + 10 y + 7 z − 4 = 0
= , , = ,− ,
 4 + 1+ 4 4+1+ 4 4 + 1 + 4 3 3 3 Sol. (a)
The equation of a plane passing through (2, 5, − 8) is
45. The number of lines passing through (0, 0, 0)
a(x − 2) + b(y − 5) + c(z + 8) = 0 …(i)
and making an angle of 45° with each of the
It is perpendicular to 2x − 3y + 4z + 1 = 0
three co-ordinate axes is [22 April 2019, Shift-II]
and 4 x + y − 2z + 6 = 0
(a) 0 (b) 2
∴ 2a − 3b + 4c = 0 and 4a + b − 2c = 0
(c) 4 (d) 8 a b c
⇒ = =
Sol. (b) 1 10 7
Equation of line passing through point (x1 , y1) is Substituting the values of a , b , c in Eq. (i), we get
y − y1 = tanθ(x − x1) x − 2 + 10(y − 5) + 7(z + 8) = 0
Y ⇒ x − 2 + 10 y − 50 + 7z + 56 = 0
line I ⇒ x + 10 y + 7z + 4 = 0

45º 47. The distance between the orthocentre and


45º circumcentre of the triangle formed by the
X′
(0, 0, 0)
X points (1, 2 , 3), (3 , − 1, 5) and (4 , 0 , − 3) is
[23 April 2019, Shift-I]
33 31 27 23
(a) (b) (c) (d)
Y′ 2 2 2 2

Here, tanθ is slope, tanθ = tan 45° = 1 and given Sol. (a)
line passing through (0, 0). Let A = (1, 2, 3)
∴ y − 0 = 1(x − 0) B = (3, − 1, 5)
⇒ y= x …(i) C = (4, 0, − 3)
316 AP EAMCET Chapterwise Mathematics

DR’s of AB = (2, − 3, 2) Required dc’s are


11 −7 3
DR’s of BC = (1, 1, − 8) = , ,
121 + 49 + 9 121 + 49 + 9 121 + 49 + 9
DR’s of AC = (3, − 2, − 6)
−7
= 
11 3 
Here, we noticed that AB ⊥ AC , , 
 179 179 179 
⇒ ∠A = 90° ∴ Hence, option is (a) correct.
C
49. The equation of the plane π through the line
of intersection of the planes
π1 ≡ x + 3 y − 6 = 0 , and π 2 ≡ 3 x − y + 4 z = 0
is π1 + λπ 2 = 0 . If the plane π is at unit
distance from the origin, then an equation of
the plane π is [23 April 2019, Shift-I]
A B
(a) 2 x + y + 2 z − 3 = 0 (b) 2 x − y − 2 z + 3 = 0
Orthocentre of ∆ABC = vertex A (c) 2 x + y + 2 z + 3 = 0 (d) x + 2 y + 2 z + 3 = 0
H = (1, 2, 3) Sol. (a)
Circumcentre of ∆ABC = mid-point of BC Given equation of plane are
7 −1  π1 = x + 3y − 6 = 0
S =  , , 1
2 2  π 2 = 3x − y + 4z = 0
33 Given, π1 + λπ 2 = 0
∴ Required distance (HS) =
2 (x + 3y − 6) + λ (3x − y + 4z) = 0
2 2
(1 + 3λ) x + (3 − λ) y + 4λz − 6 = 0 …(i)
 7 − 1 +  − 1 − 2 + (1 − 3)2 Perpendicular distance from (0, 0, 0) to above
   
2   2  plane is 1.
25 25 66 | ax1 + by1 + cz1 + d|
= + + 4= ∴ =1
4 4 4 a2 + b2 + c 2
Hence, option is (a) correct. |− 6|
=1
48. The direction cosines of the normal drawn to (1 + 3λ) + (3 − λ)2 + (4λ)2
2

the plane passing through the points ⇒ 1 + 9λ2 + 6λ + 9 + λ2 − 6 λ + 16 λ2 = 36


(2 , − 1, 5), (1, − 3 , 4), (5 , 2 , 1) are ⇒ 26λ2 = 26
[23 April 2019, Shift-I]
11 −7 3 9 −7 2 ⇒ λ = ±1
(a) , , (b) , ,
179 179 179 134 134 134 ∴Substitute λ = 1 in Eq. (i), we get
11 7 −3 9 7 −2 4 x + 2y + 4z − 6 = 0
(c) , , (d) , ,
179 179 179 134 134 134 ⇒ 2x + y + 2z − 3 = 0
Hence, option is (a) correct.
Sol. (a)
Let A = (2, − 1, 5;
) B = (1, − 3, 4) 50. If the orthocentre and the centroid of a
and C = (5, 2, 1) triangle are (−3 ,5 ,2) and (3 ,3 ,4) respectively,
∴ Equation of plane passing through A, B, C is then its circumcentre is [22 April 2018, Shift-I]
x − x1 y − y1 z − z1 (a) (6, 2, 5) , ,−5)
(b) (62
(c) (6,−2,5) (d) (6,−2,−5)
x 2 − x1 y2 − y1 z2 − z1 = 0
x 3 − x1 y3 − y1 z3 − z1 Sol. (a)
We know that, in a triangle, if O is orthocentre,
x − 2 y+1 z− 5 G is centroid and S is circumcentre, then SG:G0
⇒ −1 −2 −1 = 0 =1:2 Let circumcentre be (x , y, z).
3 3 −4 1:2
11 x − 7 y + 3z − 26 = 0 S(x, y, z) C,(3, 3,4) O(-3, 5-2)
Three Dimensional Geometry 317

 − 3 + 2x 5 + 2y 2 + 2z  1(x − 1) + 2(y − 2) + 3(z − 3) = 0


∴  , ,  = (3, 3, 4)
 1+ 2 1+ 2 1+ 2 ⇒ x + 2y + 3z − 14 = 0
∴(7, 2, 1) lies on the given plane.
⇒  − 3 + 2x , 5 + 2y , 2 + 2z  = (3, 3, 4)
  53. A (3, 2, − 1), B(4 , 1, 1), C (6 , 2, 5) andD(3, 3, 3) are
 3 3 3 
− 3 + 2x 5 + 2y 2 + 2z four points. G1 , G2 , G3 and G4 respectively are
⇒ = 3, = 3, =4 the centroids of the triangles ∆BCD , ∆CDA ,
3 3 3
∆DAB and ∆ABC. The point of concurrence of
⇒ x = 6 , y = 2, z = 5
the lines AG1 , BG2 , CG3 and DG4 is
∴ S(6, 2, 5.
) [22 April 2018, Shift-II]
51. A plane cuts the coordinate axes X , Y , Z at A, (a) (4, 2, 2) (b) (2, 4, 2)
B, C respectively such that the centroid of the (c) (2, 2, 4) (d) (2, 2, 2)
∆ABC is (6, 6, 3). Then the equation of that Sol. (a)
plane is [22 April 2018, Shift-I]
Given points,
(a) x + y + z − 6 = 0 (b) x + 2 y + z − 18 = 0 A(3, 2, − 1), B(4, 1, 1) , C(6, 2, 5) and D(3, 3, 3,
) So,
(c) 2 x + y + z − 18 = 0 (d) x + y + 2 z − 18 = 0
G1 is centroid of triangle BCD, G1 ≡  , ,   13 6 9 
Sol. (d)  3 3 3
Let the equation of the plane is
G2 is centroid of triangle CDA G2 ≡  , , 
12 7 7
x y z  3 3 3
+ + =1
a b c Q The line AG1 , BG2 , CG3 and DG4 are concurrent,
By the definition of the centroi  , ,  = (6, 6, 3)
a b c so point of concurrence of these four lines is point
 3 3 3 of intersection of lines AG1 and BG2.
Therefore, Equation of line AG1 is
a = 18 x − 3 y− 2 z+1
= = = r1 (let)
b = 18 4/ 3 0 12
c=9 3
So, point on line this AG1 is  3 + r1 , 2, − 1 + r1 
4 12
The equation of the plane becomes
 3 3 
x y z x + y + 2z
+ + =1⇒ =1 and equation of line BG2 is
18 18 9 18
x − 4 y −1 z −1
⇒ x + y + 2z = 18 ⇒ x + y + 2z − 18 = 0 = = = r2 (let)
0 4/ 3 4/ 3
So, point on line BG2 is  4, 1 + r2 , 1 + r2 
52. If the foot of the perpendicular drawn from 4 4
the origin to a plane is (1, 2, 3), then a point  3 3 
on that plane is [22 April 2018, Shift-I] Let the above point is the point of intersection, so
(a) (3, 2, 1) (b) (7, 2, 1) 4 4
3 + r1 = 4 ⇒ 2 = 1 + r2
(c) (7,3,−1) (d) (6,−3,4) 3 3
Sol. (b) 12 4
and −1 + r1 = 1 + r2, from these we are getting
3 3
O (0, 0, 0) 3 3
r1 = and r2 =
4 4
So, required point of concurrence is (4, 2, 2 ) .
P (1, 2, 3) 54. The acute angle between the lines whose
direction cosines are given by the equations
DR ′s of OP = < 1 − 0, 2 − 0, 3 − 0 > = < 1, 2, 3 > l + m + n = 0 and 2 lm + 2 ln − mn = 0 is
Since, OP is perpendicular to the plane, therefore [22 April 2018, Shift-II]
OP is normal to the plane. π π
(a) (b)
6 4
∴ Equation of plane passing through (1, 2, 3) and π 2π
having d ′ r < 1, 2, 3 > of its normal is (c) (d)
3 5
318 AP EAMCET Chapterwise Mathematics

Sol. (c) x y z
+ + =1 …(i)
Given equations are, a b c
l + m+ n= 0 …(i) Q plane Eq. (i) passes through the point (α , β , γ), so
α β γ
2lm + 2ln − mn = 0 …(ii) + + =1
a b c
From Eqs. (i) and (ii),
On taking locus of point (a , b , c) we are getting,
2l(− l) − mn = 0
α β γ
2l 2 + mn = 0 …(iii) + + =1
x y z
On squaring both side of the Eq. (i).
5
l 2 + m2 + n2 + 2lm + 2mn + 2nl = 0 56. If a line makes angles tan − 1 7 , tan − 1
3
Q l 2 + m2 + n2 = 1,
with X -axis, Y -axis respectively, then the
So, 2lm + 2mn + 2nl = − 1 …(iv) angle made by it with Z-axis is
From Eqs. (ii) and (iv), [23 April 2018, Shift-I]
1 π π 5π
3mn = − 1 ⇒ mn = − (a) (b) or
3 2 6 6
From Eq. (iii) π 2π π 3π
(c) or (d) or
6l 2− 1 = 0, let root of this equation is l1 and l 2, so 3 3 4 4
1
l1 l 2 = − . Sol. (d)
6
A line makes angle tan−1 7
Now, from Eqs. (i) and (ii),
5
2lm + (2l − m) (− l − m) = 0 and tan−1 with X-axis
3 √5 √7
⇒ 2l 2 − lm − m2 = 0 and Y-axis respectively.
α = tan−1 7
2
2   −   − 1 = 0
l l So, α

 m  m tan α = 7 2
l1 l 2 −1 ll mm
⇒ = ⇒ 12 = 1 2 1 5
m1 m2 2 −1 / 6 2/ 6 ⇒ cos α = and β = tan−1
8 3
l1 l 2 n1 n2
Similarly, = 5
−1 / 6 2 / 6 tanβ =
3
3 1 π
So, cosθ = l1 l 2 + m1 m2 + n1 n2 = = ⇒ θ = 3
6 2 3 ⇒ cos β = √8 √5
8
55. A variable plane passes through a fixed point Let angle make with
(α , β , γ) and meets the coordinate axes in A, B Z-axis is γ. So,
and C. Let P1 , P2 and P3 be the planes passing √3
cos2 α + cos2 β + cos2 γ = 1
through A, B, C and parallel to the coordinate
2
 3
⇒ 
planes YZ , ZX , XY respectively. Then, the 1 
 +   + cos γ = 1
2
locus of the point of intersection of the  8  8
planes P1 , P2 and P3 is [22 April 2018, Shift-II] 4 1
α β γ ⇒ + cos2 γ = 1 ⇒ + cos2 γ = 1
(a) αx + βy + γz = 1 (b) + + =1 8 2
x y z 1 1
⇒ cos γ =
2
⇒ cos γ = ±
(c) αx2 + βy2 + γz2 = 1 (d) αβx + βγy + αγz = 1 2 2
1 −1
⇒ cos γ = or
Sol. (b) 2 2
Let the point A(a , 0, 0), B(0, b , 0) and C(0, 0, c). So, π π π 3π
⇒ γ = or π − ⇒ γ = or
point of intersection of planes P1 , P2 and P3 is 4 4 4 4
P(a , b , c) π 3π
So, angle made by line with Z-axis is and .
Now, equation of plane ABC is 4 4
Three Dimensional Geometry 319

57. A plane passes through the point (3, 5, 7). If 59. If the direction ratios of the lines L1 and L2
the direction ratios of its normal are equal to are 2 , − 1, 1 and 3 , − 3 , 4 respectively, then the
the intercepts made by the plane direction cosines of a line that is
x + 3 y + 2 z = 9 with the coordinate axes, then perpendicular to both L1 and L 2 are
the equation of that plane is [23 April 2018, Shift-II]
[23 April 2018, Shift-I] 2 1 1 1 5 3
(a) ± ,± ,± (b) ± ,± ,±
(a) x + y + z = 5 (b) 6 x + 2 y + 3 z = 105 6 6 6 35 35 35
(c) 12 x + 4 y + 6 z = 49 (d) 6 x + 2 y + 3 z = 49 3 3 4 1 2 3
(c) ± ,± ,± (d) ± ,± ,±
Sol. (d) 34 34 34 14 14 14
Given equation of plane. Sol. (b)
x + 3y + 2z = 9 Let direction cosines of line that is perpendicular
x y z to both L1 and L2 are l , m, n, then
⇒ + + =1
9 3  9 2l − m + n = 0 and 3l − 3m + 4n = 0
 
 2 l m n
⇒ = =
9 − 4 + 3 3− 8 − 6 + 3
⇒ So, intercepts are 9, 3, .
2 l m n
According to the questions, ⇒ = =
−1 − 5 − 3
9
direction ratio of normal is 9, 3, and plane
2 l m n l 2 + m2 + n2
⇒ = = =±
passes through (3, 5, 7) 1 5 3 1 + 25 + 9
∴Equation of plane is 1 5 3
9 ⇒ l, m, n = ± ,± ,± .
(x − 3)9 + (y − 5)3 + (z − 7) =0 35 35 35
2
9 63 60. If the equation of the plane bisecting the line
⇒ 9 x − 27 + 3y − 15 + z− =0
2 2 segment joining the points P (3 , 2 , 4) and
⇒ 18 x + 6 y + 9z = 54 + 30 + 63
Q (−1, 0 , − 2) and perpendicular to PQ is
ax + by + cz + d = 0, then ac + bd
⇒ 18 x + 6 y + 9z = 147
[23 April 2018, Shift-II]
⇒ 6 x + 2y + 3z = 49
(a) 0 (b) 12
58. The harmonic conjugate of P (−9 , 12, − 15) (c) 6 (d) 1
with respect to the line segment AB, where Sol. (a)
A = (1, − 2 , 3) and B = (−4 , 5 , − 6) is Mid-point of line segment joining the points
[23 April 2018, Shift-II] P(3, 2, 4) and Q(− 1, 0, − 2) is R(1, 1, 1) and direction
(a)  − , , 0
2 1 ratios of line segment PQ is 4, 2, 6, so direction
(b) (6, − 9, 12 )
 3 3  ratios of normal to the plane is
< a , b , c > = < 4, 2, 6 > .
(c)  − , , − 3 (d)  , − , 
7 8 7 8 9
 3 3   3 3 3 So, equation of plane will be
4 x + 2y + 6z + d = 0 … (i)
Sol. (c)
Since, plane (i) bisect the line segment joining PQ.
Let point P(− 9, 12, − 15) divides the line joining
So, d = − 12
A(1, − 2, 3) and B(− 4, 5, − 6) in ratio λ :1.
− 4λ + 1 Therefore, ac + bd = 0.
Then, − 9 = ,λ=−2
λ +1 61. If the mid-points of the sides AB, BC , CA of a
So, harmonic conjugate of point ‘P’ with respect to
triangle are (1, 5 , − 1), (0 , 4 , − 2), (2 , 3 , 4)
the line segment AB will divides the line segment respectively, then the length of the median
AB internally in ratio 2 : 1, so point will be drawn from C to AB is [24 April 2018, Shift-I]
 − 8 + 1 , 10 − 2 , − 12 + 3 =  − 7 , 8 , − 3 . (a) 29 (b) 27
   
 3 3 3   3 3  (c) 5 (d) 5
320 AP EAMCET Chapterwise Mathematics

Sol. (d) π π
l = cosα, m = cos , n = cos
A (x1, y1, z1) 4 3
But l 2 + m2 + n2 = 1
2 2
l 2 +   +   = 1
1 1
D F
 2  2
1 1
l2 + + = 1
B (x2, y2, z2) E C (x3, y3, z3) 2 4
1 1
D(1, 5, − 1), E(0, 4, − 2), F(2, 3, 4) are mid-points l = ⇒l = ±
2
4 2
Then, 1
x1 + x 2 y + y2 z + z2 ⇒ cosα = − (α is obtuse angle)
=1 1 =5 1 = −1 2
2 2 2
⇒ α = π − π / 3 = 2π / 3.
x1 + x 2 = 2, y1 + y2 = 10 , z1 + z2 = − 2
Similarly, 63. The equation of the plane bisecting the line
segment joining the points (2 , 0 , 6) and
x 2 + x 3 = 0, y2 + y3 = 8 , z2 + z3 = − 4
(− 6 , 2 , 4) and perpendicular to it, is
x1 + x 3 = 4 , y1 + y3 = 6 , z1 + z3 = 8
[24 April 2018, Shift-I]
On adding, 2(x1 + x 2 + x 3) = 6,
(a) 2 x − y + 4 z − 15 = 0 (b) 4 x − y + 3 z − 6 = 0
2(y1 + y2 + y3) = 24, 2(z1 + z2 + z3) = 2
(c) 4 x − y + z + 4 = 0 (d) x − 2 y + 3 z − 11 = 0
x1 + x 2 + x 3 = 3, y1 + y2 + y3 = 12,
z1 + z2 + z3 = 1 Sol. (c)
⇒ x 3 = 1, y3 = 2, z3 = 3 Let variable point on the plane is (x , y, z).
length of median drawn from C to AB is Mid-point of the line segment joining the points
(2, 0, 6) and (− 6, 2, 4) = (− 2, 1, 5)
CD = (1 − 1)2 + (2 − 5)2 + (− 1 − 3)2
Direction ratio of the line segment joining the
= 0 + 9 + 16 = 25 = 5 points (2, 0, 6) and (− 6, 2, 4)
π π ⇒(− 6 − 2), (2 − 0), (4 − 6) − 8, 2, − 2 = a1 , b1 , c1
62. If a line makes angles and with Y -axis direction ratios of line joining (x , y, z) and (− 2, 1, 5)
4 3
and Z-axis respectively, then the obtuse angle = (x + 2, ) (y − 1), (z − 5)
made by that line with X -axis is = a 2 , b2 , c 2
[24 April 2018, Shift-I] direction ratio’s a1 , b1 , c1 and a 2 , b2 , c 2 are
π 2π perpendicular so,
(a) (b)
3 3 a1 a 2 + b1 b2 + c1 c 2 = 0
π 5π So, required equation of plane is
(c) (d)
6 6 ⇒ − 8(x + 2) + 2(y − 1) − 2(z − 5) = 0
Sol. (b) ⇒ − 8 x − 16 + 2y − 2 − 2z + 10 = 0
Let line makes angle α with X-axis then, direction ⇒ − 8 x + 2y − 2z − 8 = 0
cosine of the line are ⇒ 4 x − y + z + 4 = 0.
23
Limits and Continuity
1. If a > 0 , n ∈ R , then lim x n = ....  x2   x2 
x→a 32⋅ sin2   ⋅ sin2   × x 8
 4  8 1
[17 Sep. 2020, Shift-I] ⇒ lim =
x→ 0 2 2 32
x 2
x 
(a) nan (b) (n − 1)an  ×  ⋅ sin8 x × (4 × 8)2
(c) nan −1 (d) an  4 8
qx
Sol. (d)  p
3. If lim 1 +  = e 9 where p, q ∈N then
If a > 0, n ∈ R ⇒ lim x n = a n x → ∞ x
x→ a
8 p+q= [17 Sep. 2020, Shift-II]
2. lim (a) 6 (b) 9 (c) 81 (d) 18
x → 0 sin 8 x
  x2   x2   x2   x2   Sol. (a)
1 − cos   − cos   + cos   cos    = qx
  2  4  2  4  Given, lim 1 + 
p
= e 9,
x→ ∞  x
[17 Sep. 2020, Shift-I]
1 1 1 1 1
(a) (b) (c) (d) Let= y ⇒ When x → ∞ ⇒ y → 0
16 32 64 8 x
∴ lim (1 + py)q / y = e 9
Sol. (b) y→ 0

8   x2   x2  q
 lim(1 + py)1 / y  = e 9
lim 1 − cos  − cos  ⇒  y→ 0 
x → 0 sin 8 x
  2  4  
 x2   x2   ⇒ (e p)q = e 9 ⇒ pq = 9
+ cos  ⋅ cos  
 2  4  Let we take p = 3 ⇒ q = 3 and ∴ p + q = 6
⇒ 1 − sin x / 2
4. lim =
8    x2    x2    x 2    x→π  x  x x
lim 8  1 − cos     − cos    1 − cos 2     cos  cos − sin 
x → 0 sin x 
   
2   
4  2     2   4 4
[17 Sep. 2020, Shift-II]
8   x 2    x2   3 −1 1 5
⇒ lim 8 1 − cos   1 − cos   (a) (b) (c) (d)
x → 0 sin x  2    4  2 2 2 2

8   x 
2
 x2   Sol. (c)
⇒ lim 8  2sin2     2sin2    x
x → 0 sin x   4   8  1 − sin
lim 2
 x2   x2  x→ π  cos x   cos x − sin x 
32sin2   ⋅ sin2      
 4  8  2  4 4
⇒ lim
x→ 0 sin8 x Let x = π + h, x → π, h → 0
322 AP EAMCET Chapterwise Mathematics

π
1 − sin 
h | x|
+  7. The function of f (x) =| x | + is
2 2
= lim x
x→ π  π + h   cos π + h  π + h 
h→ 0 cos    − sin  [18 Sep. 2020, Shift-II]
 2 2   4   4   (a) continuous at the origin
h (b) discontinuous at the origin because| x| is
1 − cos
2 discontinuous there
= lim | x|
h→ 0 h  π h π h (c) discontinuous at the origin because is
− sin ⋅  cos cos − sin sin
2  4 4 4 4 x
discontinuous there
π h π h
− sin ⋅ cos − cos ⋅ sin  (d) discontinuous at the origin because both| x|
4 4 4 4 | x|
and are discontinuous
21 − cos 
h x
 2
= lim Sol. (c)
− sin cos − sin − cos − sin 
h→ 0 h h h h h
x
2 4 4 4 4 f ( x) = x +
x
2 1 − cos 
h h
2 ⋅ 2sin2 LHL of x = 0
 2  4
= lim = lim −x
h→ 0
sin .  −2sin 
h h h→ 0 h
2⋅ sin × sin
h limx → 0 – f (x) = limx → 0 – − x +
2 4 2 4 x
= limx → 0 – − x − 1 = −1
h h
sin sin (x)
= lim 2 4 = 2 lim 4 ⋅
2(h / 2) limx → 0 + f (x) = limx → 0 + (x) +
h→ 0 h h→ 0  h  h x
sin 4  sin = limx → 0 + x + 1 = 1
2  4 2
∴ LHL ≠ RHL at x = 0
 sin h   h 
 ∴ f (x) is discontinuous at x = 0
=
2
2 × ⋅ lim  4  2 
→ h   h Hence, option (c) is correct.
4 h 0
   sin 
 4   2 8. A point in the domain that cannot be filled in
2 1 so that the resulting function is continuous is
= 2 × ×1 ×1=
4 2 called [18 Sep. 2020, Shift-II]
(a) Removable Discontinuity
n!
5. lim = (b) Non-Removable Discontinuity
n → ∞ (n + 1)! − n! [18 Sep. 2020, Shift-I] (c) Impossible Discontinuity
(a) 1 (b) − 1 (d) Irrelevant Discontinuity
(c) 2 (d) 0 Sol. (b)
Sol. (d) We have, a point in the domain that can’t be filled
n! 1 in so that resulting function is discontinuous is
Lim = Lim called non-removable discontinuity.
n→ ∞ (n + 1)!− n! n→ ∞ (n + 1) − 1
Hence, option (b) is correct.
1
= Lim =0  x −4
n→ ∞ n
| x + a, x < 4
− 4|
6. lim (1 + 3 x)2/ x = 
x→0
9. f (x) =  a + b, x=4
[18 Sep. 2020, Shift-I]
 x −4
(a) 6 (b) e 6 | x + b, x > 4
(c) e − 6 (d) e1/ 6  − 4|

Sol. (b) If f (x) given above is continuous at x = 4,


6 then find the values of ‘a’ and ‘b’.
2  1

lim (1 + 3x) = lim  (1 + 3x) 3x  [21 Sep. 2020, Shift-I]
x→ 0 x x → 0  
 (a) a = 1, b = − 1 (b) a = − 1, b = 1
= e6 {Q lim (1 + ax)1 / ax = e , (a ≠ 0)} (c) a = 1, b = 1 (d) a = − 1, b = − 1
x→ 0
Limits and Continuity 323

Sol. (a) Sol. (b)


Given function Given differential equation
 x− 4
+ a, x<4
y32/ 3 + 2 + 3y2 + y1 = 0
|x − 4| ⇒ y32 = (− 2 − 3y2 − y1)3
f (x) =  a + b, x=4
 x− 4 having degree 2
 + b, x>4
|x − 4| Hence, option (b) is correct.

− 1 + a, x<4   πx  
13. lim (1 − x)tan   =
=  a + b, x=4 x →1  2  [21 Sep. 2020, Shift-II]

 1 + b, x>4 1 3 4 2
(a) (b) (c) (d)
π π π π
Q Function f is continuous at x = 4, so
LHL (at x = 4) = f(4) = RHL (at x = 4) Sol. (d)
π
⇒ − 1 + a = a + b = 1 + b ⇒ a = 1 and b = − 1 Lim (1 − x) tan x 
Hence, option (a) is correct. x →1 2 
1− x −1
ax − 1 = Lim = Lim
10. lim = x →1  πx  x →1 π
cot  
πx
− cosec2  
x→0 sin(x) [21 Sep. 2020, Shift-I]  2 2  2
1
(a) log(a) (b) log(a) (c) 0 (d) 1 (on applying L′ Hospital rule)
2 2
=
Sol. (a) π
ax − 1 Hence, option (d) is correct.
lim
ax − 1 x → 0 x log e a
lim = = = log e a sin(x m)
x → 0 sin x sin x 1 14. Find the value of lim , given that
lim x → 0 (sin x)n
x →0 x
Hence, option (a) is correct. n< m [21 Sep. 2020, Shift-II]

2 + 4 + 6 + K + (2 n)
2 2 2 2 (a) 2 (b) 1 (c) 0 (d) ∞
11. lim =
n→ ∞ n3 Sol. (c)
[21 Sep. 2020, Shift-I] sin(x m)
Lim ; (n < m)
(a)
2
(b)
4
(c)
3
(d)
8 x→ 0 (sin x)n
3 3 2 7 sin x m m sin x m
x
Sol. (b) = Lim x
m m
= Lim x n x m − n
n
22 + 42 + 62 + K + (2n)2 x → 0 sin x
 x → 0 sin x
lim  x  



n →∞ n3  x   x 
1 + 2 + 32 + .... + n2
2 2 1
×0= {Qm > n}
= 4 lim
n →∞ n3 1
=0
1. 1 +   2 + 
1 1
n(n + 1) (2n + 1)  n   n Hence, option (c) is correct.
= 4 lim = 4 lim
n →∞ 6n3 n →∞ 6
15. f (x) =|log e| x|| is differentiable at
2 4
=4× = [21 Sep. 2020, Shift-II]
6 3
Hence, option (b) is correct. (a) x = 0 only (b) x = 1only
(c) x = −1only (d) R − {0, ± 1}
12. Find the degree of the differential equation
Sol. (d)
y 32 / 3 + 2 + 3 y 2 + y1 = 0 . [21 Sep. 2020, Shift-I] Since the graph of f (x) =|log e|x|| has point of
(a) 4 (b) 2 discontinuity at x = 0 and the sharp edges at
(c) 3 (d) 1 x = −1, 1 where function is not differentiable.
324 AP EAMCET Chapterwise Mathematics

y
Sol. (a)
dt t
= −2 x
dx
x + te t

On reciprocal, we get
−2 x
x −2 x
dx x + te 
=   + e 
t x t
–1 O 1 =
dt t  t
Hence, option (a) is correct.
f (x) = |log e|x||
Hence, option (d) is correct.  x n − 3n 
18. If lim   = 108 and n ∈N, then the
x→3  x − 3 
16. Let f : R → R be a continuous function such
that for any two real numbers x and y, value of ‘n’ is [22 Sep. 2020, Shift-II]
| f (x) − f (y) ≤ 10.| x − y|201 , then (a) 3 (b) 6 (c) 5 (d) 4
[22 Sep. 2020, Shift-II] Sol. (d)
(a) f(2019) = f(2020) + 1 x n − 3n
lim = 108
(b) f(2019) + f(2022 ) = 2 f(2021) x→ 3 x−3
(c) f(2019) = f(2020) + 8
LHS is in ÷from, so apply L-Hospital rule
(d) f(2019) = f(2020) + 2
n ⋅ x n −1 − 0
Sol. (b) lim = 108
x→ 3 1− 0
Given, f : R → R is continuous
lim n ⋅ x n − 1 = 108
| f (x) − f (y)|≤ 10 |x − y|201 x→ 3
| f (x) − f (y)| n ⋅ 3n − 1 = 108
⇒ ≤ 10|x − y|200
|x − y| 3n
n⋅ = 108
Put, y = x + h and taking limit h → 0 on both sides 3
| f (x) − f (x + h)| n ⋅ 3n = 3 × 108
lim ≤ lim | x − (x + h)|200
h→ 0 |x − (x + h)| h→ 0
n ⋅ 3n = 3 × (33 × 4)
f (x + h) − f (x)
lim ≤0 n ⋅ 3n = 4 × 34
h→ 0 h
∴On comparison n = 4
⇒ | f ′(x)|≤ 0 ⇒ f ′(x) = 0
Hence, option (d) is correct.
∴ f (x) = constant 5/ 7
∴ f(2019) = constant  x
1 +  −1
f (2021) = f (2022) = constant  2
19. lim =
So, f (2019) + f (2022) = 2 f (2021) x→0 x [22 Sep. 2020, Shift-II]
{Q boths are constant} 5 10 5 5
(a) (b) (c) (d)
∴ Hence, option (b) is correct. 7 7 14 17
dt t Sol. (c)
17. Express = in the form of
dx (x + te −2 x / t) 1 + x 
5/ 7
  −1
dx  x  2
=φ  lim
dt  t [22 Sep. 2020, Shift-II]
x→ 0 x
 x
−2   −2   x

Here, Limit is ÷ form, so we can apply L-Hospital
x x
(a) + e t 
(b) −e t  rule
t t −2/ 7
5 x d  x
2  x
 2  x
 1 +  .   −0
x
(c) + e  t  x
(d) − e  t  7  2 dx  2 
= lim
t t x→ 0 1
Limits and Continuity 325

−2/ 7
5 x 1 For the existance of limit, coefficient of x 3 = 0
= lim1 +  .
x→ 07  2 2 ∴ α =1
5 −2/ 7 1 5 −βx 2 − x − β + 1
= (1 + 0) × = ∴ lim =2
7 2 14 x→ ∞ x2 + 1
Hence, option (c) is correct 1 β 1
−β − − 2 + 2
 4 x − 5, x ≤ 2 ⇒ lim x x x = 2⇒ − β = 2 ⇒ β = − 2
20. If f (x) =  , then the value of ‘k’ x→ ∞ 1
1+ 2
 x − k, x > 2 x
if lim f (x) may exist is equal to ∴ (α , β) = (1, −2)
x→2
[22 Sep. 2020, Shift-II]
Hence, option (d) is correct.
(a) −1 (b) −2 (c) 1 (d) 2
23. For
Sol. (a) ∞ n− x x
kx n!  k  1
Given, k > 0, ∑ lim
→ ∞
1 −
(n − x)! 

n
  =
 n
= 0 x!
n
4 x − 5, x ≤ 2
x
f (x) =  [20 April 2019, Shift-I]
 x − k, x > 2
(a) 0 (b) k (c) x (d) 1
lim f (x) is exists when LHL at x = 2 = RHL at x = 2
x→ 2 Sol. (d)
∞ n−x x
∴ lim f (x) = lim f (x) kx n!  k 1 
For k > 0, ∑ lim 1 −   
x → 2− x → 2+ x = 0 x ! n → ∞ (n − x)!  n  n
lim 4 x − 5 = lim x − k ∞ n−x x
x → 2− x → 2+ = lim ∑
n! 1 − k   k
   
n → ∞ x =0 x !(n − x)!  n  n
4(2) − 5 = 2 − k n−x x

= lim ∑ n Cx 1 −   k
3 = 2 − k ⇒ k = −1 k
 
n → ∞ x =0  n  n
Hence, option (a) is correct.
n
= lim  1 − +  = lim 1n = 1
3x k k
 2
21. lim 1 +  = n→ ∞  n n n→ ∞
x→∞  x [23 Sep. 2020, Shift-I]
Hence, option (d) is correct.
(a) e 6 (b) e 3 (c) e 2 (d) e
24. Let f : R → R be the function defined by
Sol. (a)
3x  5, if x ≤ 1
lim 1 +  = e x → ∞ 1 + − 1 ⋅ 3x [Q1∞ form]  a + bx ,
2 lim 2
x→ 0 
 if 1 < x < 3
x  x  f (x) =  then f is
b + 5 x , if 3 ≤ x < 5
= e x → ∞  × 3x  = e 6
lim 2
x   30 , if x ≥ 5
[20 April 2019, Shift-I]
Hence, option (a) is correct. (a) continuous if a = 5 and b = 5
 x3 + 1  (b) continuous if a = 0 and b = 5
22. If lim  − (αx + β) exists and equal to (c) continuous if a = − 5 and b = 10
 x +1
x→∞ 2
 (d) not continuous for any values of a and b
2, then the ordered pair (α , β) of real numbers Sol. (d)
is [20 April 2019, Shift-I] Given function f : R → R, such that
(a) (1, − 1) (b) (−2, 1) (c) (−1, 1) (d) (1, − 2 )  5, if x ≤ 1
Sol. (d)  a + bx , if 1 < x < 3

It is given that, f (x) =  b + 5x , if 3 ≤ x < 5
 x3 + 1   30 , if x ≥ 5
lim  2 − (αx + β) = 2 
x→ ∞
 x + 1  
x 3 + 1 − αx 3 − βx 2 − αx − β If f is continuous at x = 1, then
⇒ lim =2
x→ ∞ x2 + 1 a+ b=5 …(i)
326 AP EAMCET Chapterwise Mathematics

If f is continuous at x = 3, then  
a + 3b = b + 15  
2 −1
x
=  lim 
1 1
⇒ a + 2b = 15 …(ii) ×
2 x → 0 x  
2 
and if f is continuous at x = 5, then  sin (1 / )
2 x 1
lim   × 

b + 25 = 30 ⇒ b = 5 …(iii) x→ 0  x/2  4 
From Eqs. (ii) and (iii), we get Q lim sin x = 1
a=5 …(iv)  x → 0 x 
but a = 5and b = 5doesn’t satisfy the Eq. (i).  
ax − 1
so, f : R → R is not continuous for any values of a  x → 0 x = log a 
Q lim
and b.  
1 1 1
Hence, option (d) is correct. = × log 2 × = log 2 × 4
2 1 2

25. Let [ x ] denote the greatest integer less than 4
or equal to x. Then the number of points α = 2.log 2 …(i)
where the function x ⋅ 2x − x
Given, β = lim
y = [ x ] + |1 − x|, − 1 ≤ x ≤ 3 is not x→ 0
1 + x2 − 1 − x2
differentiable, is [20 April 2019, Shift-I]
x(2x − 1)
(a) 1 (b) 2 (c) 3 (d) 4 β = lim
x→ 0
1 + x2 − 1 − x2
Sol. (d)
Rationalise the denominator
The given function y = [x] + |1 − x| have point of
discontinuity at x = 0, 1, 2 and 3 for −1 ≤ x ≤ 3. x(2x − 1) 1 + x2 + 1 − x2
= lim ×
So function y = [x] + |1 − x| is not differentiable at x→ 0
1 + x2 − 1 − x2 1 + x2 + 1 − x2
4 points. Hence, option (d) is correct.
x (2x − 1)( 1 + x 2 + 1 − x 2)
x ⋅ 2x − x = lim
26. If α = lim and x→ 0 (1 + x 2) − (1 − x 2)
x → 0 1 − cos x
x(2x − 1)( 1 + x 2 + 1 − x 2)
x ⋅ 2x − x = lim
β = lim , then x→ 0 2x 2
x→0
1 + x2 − 1 − x2
1 2 −1x

[20 April 2019, Shift-II]


= lim × × ( 1 + x 2 + 1 − x 2)
x→ 0 2 x
(a) α = β (b) 2α = β (c) α = 2β (d) α = 3β 1
= × log 2 × 2
Sol. (c) 2
x ⋅ 2x − x β = log 2 ⇒ 2β = 2⋅ log 2 …(ii)
α = lim
x→ 0 1 − cos x From Eqs. (i) and (ii), we get
α = 2β
x(2x − 1) x(2x − 1)
α = lim = lim Hence, option (c) is correct.
x → 0 1 − cos x x→ 0 x
2sin2
2  x − | x|
 , when x < 0
2x − 1 x
 5x + a,
2
when 0 ≤ x ≤ 1
1 2 −1 1
x
x 
= lim ⋅ = lim 27. If f (x) =   
 b  x − 1  , when 1 < x < 3
x→ 0 2 x 2x → 0 x 2
sin2 sin2
2 2
  x 2 − 3 x + 2
x x2 
[Q divided by x]  −14 , when x ≥ 3
2x − 1 is a continuous function on R, then (a , b) =
1 x [20 April 2019, Shift-II]
= lim
2 x → 0  1 2
(a)  2, − 
7
(b) (2, − 14)
 sin 2 x   2
 
 x  (c)  − , − 14
7
  (d) (2, 7 )
 2 
Limits and Continuity 327

Sol. (a) x 2 (tan 2 x − 2 tan x)2


29. lim =
 x − | x| x→0 (1 − cos 2 x)4
; when x < 0 [21 April 2019, Shift-I]
 x
 1 1
 5x + a ;
2
when 0 ≤ x ≤ 1 (a) 4 (b) 2 (c) (d)
Given, f (x) =  2 4
 x2 − 1 
b  2  ; when 1 < x < 3
  x − 3x + 2 Sol. (d)
 −14 ; when x ≥ 3 x 2(tan 2x − 2tan x)2
Given, lim
Since, given f (x) is continuous at x = 0 and x = 3
x→ 0 (1 − cos 2x)4
f (x) is continuous at x = 0.  
2
(2x)3 2 
⇒ LHL at x = 0 = RHL at x = 0   2x + + (2x)5 + K 
 3 15 
= lim f (x) = lim f (x) x2  
x → 0− x → 0+
x − | x|   x3 2 5 
= lim = lim 5x 2 + a  − 2 x + + x + K 
x → 0− x x → 0+   3 15  
= lim
x − (− x) x→ 0

4
= lim = lim 5x 2 + a  (2x)2 (2x)4 (2x)6 
x → 0− x x→ 0 1 − 1 − + − + K 
  2! 4! 6! 
2x
[as x < 0, then|x| = − x] = lim = 5(0)2 + a 2
4 x 8   −  + (16 x 2 − x 2) + K
x → 0− x 4 1 2
2= a ⇒ a = 2   3 3 15 
= lim 4
x→ 0
f (x) is also continuous at x = 3  x 2

16 x 8 1 + + K
∴ LHL at (x = 3) = RHL at (x = 3)  3 
= lim f (x) = lim f (x) 4 1
x → 3− x → 3+ = =
16 4
 x2 − 1 
= lim b  2  = lim − 14  6 x 2 − cos 3 x 5 x 3 + 3 
x→ 3−
 x − 3x + 2 x → 3 +
30. lim  −  =
x → ∞ x +5 x 6 + 2 
2
 (x + 1)(x − 1)  
= lim b   = − 14
x → 3−  (x − 1)(x − 2)  [20 April 2019, Shift-I]
 x + 1 (a) 11 (b) 0
= lim b   = − 14
x → 3−  x − 2  (c) − 1 (d) 1

 3 + 1 Sol. (a)
⇒ b  = − 14 ⇒ 4b = − 14  6 x 2 − cos 3x
 3 − 2 5x 3 + 3 
lim  −

x → −∞ x + 5
2
x 6 + 2 
⇒ b=−
14
⇒ b=−
7 
4 2
 
 −7   6 − cos 3x 5+ 3
3 
∴ (a , b) =  2, 
= lim  
2
 2 x − x
x → −∞  5  | | 3
 2 
 1+ 2
x
∴Hence, option (a) is correct.   1+ 6
 x  x  3
x 
28. The number of points in the interval (0, 2) at
Q lim cos 3x = 0 and lim |x| = − 1
3
which f (x) = | x − 0 .5| + | x − 1| + tan x is not
 x → −∞ x 2 x → −∞ x 3 
differentiable is [20 April 2019, Shift-II] 
(a) 1 (b) 2 (c) 3 (d) 4 = 6 + 5 = 11
Sol. (c) 31. The number of discontinuities in R for the
Given, f (x) = |x − 0.5| + |x − 1| + tan x x −1
π function f (x) = is
Clearly f (x) is not differentiable at x = 0.5, 1 and x 3 + 6 x 2 + 11 x + 6
2
for x ∈(0, 2). [20 April 2019, Shift-I]

∴Number of Non differentiable points of f (x) are 3. (a) 3 (b) 2


(c) 1 (d) 0
∴Hence, option (c) is correct.
328 AP EAMCET Chapterwise Mathematics

Sol. (a) 2log 2 2log 2


= =
Given, (1 − 0) + (1 + 0) 2
x −1 β = log 2 …(ii)
f (x) =
x 3 + 6 x 2 + 11 x + 6 By Eqs. (i) and (ii), we get
x −1 α = 2β
f (x) =
(x + 1) (x 2 + 5x + 6)  1 1 1 
33. lim  + + + … + (n terms) =
x −1 n→ ∞  3 ⋅ 7 7 ⋅11 11 ⋅15 
f (x) =
(x + 1) (x + 2) (x + 3) [21 April 2019, Shift-II]
For is discontinuous, hence denominator is 0. 1 1 1
(a) (b) (c) (d) 0
⇒ (x + 1) (x + 2) (x + 3) = 0 12 4 3
⇒ x = − 1, − 2, − 3 Sol. (a)
Hence, number of discontinuous in R is 3. Given,
x ⋅ 2x − x lim  + … + (n terms)
1 1 1
32. If α = lim and + +
n→ ∞  3⋅ 7 7 ⋅11 11 ⋅15 
x → 0 1 − cos x
1 4
+ …
4 4
x ⋅ 2x − x = lim  + +
β = lim , then n→ ∞ 4  3⋅ 7 7 ⋅11 11 ⋅15 
x→0
1 + x2 − 1 − x2 1 1 1 1 1 1 1
= lim  − + − + − …
[21 April 2019, Shift-II] n→ ∞ 4  3 7 7 11 11 15
1 1 1 1 
(a) α = 5β (b) α = 2β (c) β = 2α 2 (d) β = α − + − 
6 n n n + 4

Sol. (b) 1 1 1 
= lim  − 
Given,
n→ ∞4  3 n + 4
x ⋅ 2x − x 1 1
= lim − lim
α = lim n→ ∞ 12 n→ ∞ 4(n + 4)
x → 0 1 − cos x
1 1
Applying L’Hospital rule = − lim
12 n→ ∞ 4n1 + 4 
2x + x ⋅ 2x log 2 − 1  
α = lim  n
x→ 0 sin x 1 1
= − 0=
Again, applying L’Hospital rule 12 12
2x log 2 + 2x log 2 + x ⋅ 2x (log 2)2
α = lim 34. lim [ x 2 + ax + b − x ](a < 0 < b)
x→ 0 cos x x→∞
α = log 2 + log 2α = 2log 2 …(i) [21 April 2019, Shift-II]
x ⋅ 2x − x (a) depends on both a and b
and β = lim (b) depends only on b
x→ 0
1 + x2 − 1 − x2
(c) depends only on a
2x + x ⋅ 2x log 2 − 1 (d) does not depend on a and b
= lim

x→ 0
2x 2x  Sol. (c)
 + 
 2 1 + x2 2 1− x  2 Given that, lim[ x 2 + ax + b − x]
 x→ ∞
(By L’Hospital rule)  x 2 + ax + b + x 
Again, applying L’Hospital rule, we get = lim( x 2 + ax + b − x) ×  
x→ ∞  x 2 + ax + b + x 
2x log 2+ 2x log 2 + x ⋅ 2x (log 2)2  
= lim
x→ 0   2x 2   [By rationalisation]
 1 + x − 
2
x 2 + ax + b − x 2
  2 1+ x2   = lim
  x→ ∞
x 2 + ax + b + x
  2x 2   ax + b
+  1 − x2 +   = lim
x→ ∞
  2 1 − x2   x 2 + ax + b + x
 
Limits and Continuity 329

x  a + 
b 1 3
⇒ β= +1 ⇒ β =
 x 2 2
= lim
x→ ∞  a b   1 3
x  1 + + 2 + 1 Hence, (α , β) =  , 
 x x   2 2
(taking x common from both numerator and sin x − e nx
denominator) 36. For A ≠ 0 , x < 0 , lim =
a a
n→ ∞ 1 + Ae nx
= = [22 April 2019, Shift-I]
(1 + 1) 2
1 1
It is clear that it depends only on a. (a) (b) sin x (c) − (d) −sin x
A A
35. If α and β are such that the function f (x) Sol. (c)
αx 2 − β , for| x| < 1
e nx  nx − 1
sin x

defined by f (x) =  −1 sin x − e nx  e 
, for| x| ≥ 1 Given, lim = lim
 | x| n→ ∞ nx  1
e  nx + A
n→ ∞ 1 + Ae nx

e 
is differentiable everywhere, then the ordered
pair (α , β) = sin x
[21 April 2019, Shift-II] −1
nx 0 −1 1
(a)  − , − 
1 3
(b)  , − 
1 3 = lim e = =−
 2 2 2 2 n→ ∞ 1
+ A 0 + A A
e nx
(c)  ,  (d)  − , 
1 3 1 3
2 2  2 2
37. Define
Sol. (c)  1 + px − 1 − px
 , if − 1 ≤ x < 0
Given, that, function is differentiable at all points. x
f (x) = 
Then, according to first principle, 2 x +1
 , if 0 ≤ x ≤ 1
∴ LHD (at x = 1) = RHD (at x = 1)  x −2
f (x) − f (1) f (x) − f (1)
⇒ lim = lim If lim f (x) exists, then p = [22 April 2019, Shift-I]
x →1 − x −1 x →1 + x −1 x→0
1
1 (a) −1 (b) −
− +1 2
αx 2 − β − α + β
⇒ lim = lim x (c)
1
(d) 1
x →1 − x −1 x →1 + x − 1
2
 x − 1 Sol. (b)
 
αx 2 − α  x 
⇒ lim = lim  1 + px − 1 − px
x →1 − x − 1 x →1 + x −1  , if − 1 ≤ x < 0
Given , f (x) =  x
α(x 2 − 1) 1 2x + 1
⇒ lim = lim  , if 0 ≤ x ≤ 1
x →1 − x −1 x →1 + x  x −2
α(x − 1)(x + 1) 1 Now, RHL = LHL (at x = 0)
⇒ lim = lim 2x + 1 1 + px − 1 − px
x →1 − (x − 1) x →1 + x ⇒ lim = lim
x→ 0 + x − 2 x→ 0 − x
1
⇒ lim α(x + 1) = lim Apply L’Hospital rule in LHL
x →1 − x →1 + x
p p
1 +
⇒ α(1 + 1) = 1 ⇒ 2α = 1 ⇒α = 1 2 1 + px 2 1 − px
2 ⇒ − = lim
2 x→ 0 − 1
As we know that, every differentiable function is
1 p p
continuous also. ⇒ − = +
2 2 2
∴ RHL = LHL
1
⇒ lim f (x) = lim f (x) ⇒ − =p
x →1 − x →1 + 2
1 2 1 1 1
⇒ 1 − β = − ⇒ − β = −1
() ⇒ p=−
2 1 2 2
330 AP EAMCET Chapterwise Mathematics

∞ x 2n − 1 40. The positive integer n for which


38. Given, sin x = Σ (−1)n − 1 . If the
n=1 (2 n − 1)! (cos x − 1) (cos x − e x )
lim exists and is finite,
function f (x) given by x→0 xn
cos(sin x) − cos x is [22 April 2019, Shift-II]
f (x) = (x ≠ 0) and f (0) = k, is
x4 (a) 4 (b) 3
continuous at x = 0, then k = (c) 2 (d) 1
[22 April 2019, Shift-I] Sol. (b)
1 1 Let
(a) (b)
6 3 (cos x − 1) (cos x − e x )
1 lim =K
(c) (d) 0 x→ 0 xn
2
(1 − cos x) (e x − 1 − cos x + 1)
Sol. (a) ⇒ lim =K
x→ 0 xn
Given,
cos(sin x) − cos x x
f (x) = 2sin2
 e x − 1 1 − cos x 
x4 ⇒ lim n − 12  +  =K
sin x + x   x − sin x  x→ 0 x  x 
2sin
x
 sin 
 2   2  x
= lim 2sin2
x→ 0 x 4  e x − 1 2sin2 x / 2
⇒ lim n − 12  +  =K
 x3  x→ 0 x  x x 
2sin(x)sin 
 12  x
= lim 4sin4
x→ 0 3 e x − 1  2sin2 x / 2 2 =K

x
× 12 ⇒ lim   + lim
12 x→ 0 x  xn − 1  x → 0 xn
 x3  x
 Applying sin x = x − 3! + K  2sin2 x / 2
4sin2
  ⇒ lim   + lim 2 =K
 x3  x → 0 x n − 1  x→ 0 xn
sin   ex − 1 
= lim 2 
sin x 
×
 12  1 1 1
× = 2 ×1 ×1 × =  x → 0 x = 1
Q lim
x→ 0  x   x3  12 12 6  
  = lim
1
+ lim
1
=K
 12 
2(x n − 3) x → 0 4(x n − 4)
x→ 0
 π To exists and finite put n − 3 = 0 ⇒ n = 3
sin  x − 
 6
39. lim = 41. Let [x] denote the greatest integer not
π 3
x→
6 − cos x exceeding x. If l1 = lim (x 2 + [ x ]),
2 [22 April 2019, Shift-II] x → 2+

(a) 0 (b) 1  cos x 


1 l2 = lim (2 x − [ x ]) and l3 = lim  ,
(c) 2 (d) x → 3− π  x − π / 2
x→
2 2

Sol. (c) then [23 April 2019, Shift-I]

Given, (a) l2 < l3 < l1 (b) l1 < l3 < l2


(c) l1 < l2 < l3 (d) l3 < l2 < l1
π
sin x − 
 6  0 form Sol. (d)
lim 
x→
π 3 0  Given,
6 − cos x
2 l1 = lim (x 2 + [x])
x → 2+
Applying L’ Hospital rule,
π
cos x −  cos − 
π π = lim (x 2 + 2) [Q as x → 2+ , [x] = 2]
   6 6  cos 0 1 x → 2+
= lim 6 = = = =2
π π
sin 
x→ sin x 1 1 =4+ 2
6
 6 2 2 l1 = 6
Limits and Continuity 331

l 2 = lim (2x − [x]) Sol. (b)


x → 3−
 x − 1, −∞ < x < 1
= lim (2x − 2) [Q as x → 3− , [x] = 2] 
We have, f (x) =  0, x =1
x → 3−
 x 3 − 1, 1 < x < ∞
= 2(3) − 2 
l2 = 4 Now, (LHL at x = 1)
 cos x  = lim (x − 1) = 1 − 1 = 0
l 3 = lim   x→1
x → π / 2  x − π / 2
(RHL at x = 1) = lim x 3 − 1
π π x →1
Put x − = y and as x → , then y → 0
2 2 = ()
1 3 − 1 = 0 and f()
1 =0
π
cos  + y ∴ LHL = RHL = f()
1
π 2  sin y
x = + y = lim = lim − So, f (x) is continuous at x = 1
2 y → 0 y y→ 0 y
Now, (LHD at x = 1)
l3 = − 1 (x − 1) − 0
= lim =1
∴ l 3 < l 2 < l1 x →1 x −1
∴ Hence, answer is (d). and (RHD at x = 1)
[(a − n) nx − tan x ] sin nx (x 3 − 1) − 0
42. If lim = 0, (n ≠ 0) = lim
x→0 x 2 x →1 x −1
then the minimum possible positive value of = lim(x 2 + x + 1) = 3
x →1
a is [23 April 2019, Shift-I]
∴ LHD ≠ RHD
(a) 0 (b) − 2 (c) 2 (d) 1
So, f (x) is not differentiable at x = 1
Sol. (c)
((a − n)nx − tan x) sin nx
lim = 0, n ≠ 0
44. If [x] denotes the greatest integer ≤ x, then
x→ 0 x2 1
(a − n) nx tan x  sin(nx)
lim {[12 x ] + [2 2 x ] + [3 2 x ]+…+[ n2 x ]} =
⇒ lim  −  =0 n→ ∞ n3
x→ 0  x x  x [22 April 2018, Shift-I]
Q lim sin nx = n x x
 x → 0 nx  (a) (b)
2 3
1 x
⇒ n(an − n2 − 1) = 0 ⇒ a = n + , n ≠ 0 (c) (d) 0
n 6
1 Sol. (b)
n+
n ≥ n 1 (by AM ≥ GM) 1
Q
2 n lim {[12 x]+ [22 x]+ [32 x]+ …+ [n2 x]}
n→ ∞ n3
a
⇒ ≥ 1 ⇒ a ≥ 2. 1 n
2 = lim 3 ∑ [r 2 x]
n→ ∞ n r =1
∴ Minimum possible positive value of a is 2.
Hence, option (c) is correct. 1 n
= lim 3 ∑ (r 2 x − {r 2 x})
n→ ∞ n r =1
43. If a function f is defined by :
1  n 2 n 2 
f (x) = 0, when x = 1, = lim  x ∑ r − ∑ {r n}
n→ ∞ n3  r =1 r =1 
= x 3 − 1, when 1 < x < ∞,
x × n(n + 1)(2n + 1) n {r 2n}
= x − 1, when − ∞ < x < 1, then at x = 1, f is = lim −∑
n→ ∞ n3 × 6 r =1 n3
[23 April 2019, Shift-I]
x n n + 1 (2n + 1)  n {r 2n}
= lim  × ×
(a) continuous and differentiable
×  − lim ∑
(b) continuous but not differentiable n→ ∞  6 n n n  n→ ∞ r =1 n3
(c) discontinuous and differentiable x x
(d) discontinuous and not differentiable = ×1 × 2− 0 =
6 3
332 AP EAMCET Chapterwise Mathematics

45. If a function f defined by 47. lim n− nk


n→ ∞
1 − 2 sin x π   1  1  1 
n
 , if x ≠ (n + 1)  n +   n + 2  ...  n + k − 1   =
f (x) =  π − 4 x 4 is continuous at   2  2   2 
 π
k , if x = [22 April 2018, Shift-II]
 4  1 
2 1 − 
π  2k 
x = , then k = (a) 2 (b) e
(c) 2  1 − k 
4 [22 April 2018, Shift-I] 1
(d) e 2
1  2 
(a) (b) 1
4 Sol. (b)
−1 n
(c) (d) 2  1 
lim n− nk (n + 1)  n +   n + 2  ......  n + k − 1  
1 1
4
n→ ∞   2  2   2 
Sol. (a) = P Let
We have,  1 
n
 1 1 
1 − 2sin x π  (n + 1)  n + 1   n + 2  ...  n + k−1  
, x≠   2  2   2 
 ⇒ P = lim
f (x) =  π − 4 x 4 n→∞  nk 
π  
 k , x=  
 4  1   1   1   1 
n

Since, f (x) is continuous at x =


π   n + 2°  n+ 1
2
 n+ 2  n+ k − 1
2  .....  2

= lim       
4 n→∞    n   n    
 n n
1 − 2sin x        
∴ f (π / 4) = lim f (x) ⇒ k = lim
π→
π
π→
π π − 4x So,
4 4   1   1  
− 2 cos x  log 1 + 20 n  + log 1 + 21 n  
⇒ k = lim [using L’ hospital Rule] log P = lim n 
π −4 n→∞   1   1 
+ log 1 + 2  + ... + log 1 + k−1  
π→
4
  2 n  2 n  
2 1 1
⇒ k= × ⇒ k= .  1 
log 1 + °  log 1 + 1  log 1 + 2 
1 1
4 2 4   2 n  2 n  2 n 
1 + cos 2 x  + + 
46. lim = = lim 
1/ n 1/ n 1/ n 
x→
π cot 3 x (3sin 2 x − 1) n→ ∞   1  
2 [22 April 2018, Shift-II]  log 1 + k − 1  
 2 n 
1 2  + ...+ 
(a) (b)  1/ n 
3log 9 3log 3
1
1 3 1− k
(c) (d) 1 1 1 1 2 = 2 1 − 1 
3log 3 log 3 = ° + 1 + 2 + .....+ k− 1 =
2 2 2 2 1  2k 
1−
Sol. (c) 2
 1 
1 + cos 2x 2cos2 x 21 − 
lim ⇒ lim So, P=e  2k 
π
x→ cot 3x (3 − 1) x → π cos 3x × (3sin 2x − 1)
sin 2x
2 2
sin 3x 48. If a and b (a > b) are points of discontinuity of
π 2sin2 h the function
Let x = − h, then lim
2 h→ 0 sin 3h sin 2h
(3 − 1)  3 − 2x2, for x≤0
cos 3h 
sin2 h  2 x + 3 , for 0 < x ≤ 1
2 2 × h2(cos 3h) f (x) = 2 x 2 − 3 x , for 1 < x < 2 ,
= lim h
 2 x − 3, for 2 ≤ x < 3
h→ 0
 sin 3h × 3h 3
sin 2h
− 1 sin 2h 
  × × 2h
 3h  sin 2h 2h  | x|, for x≥3
2×1 ×1 1 then 3a − b =
= = [22 April 2018, Shift-II]
(1 × 3) × [log e (3)] × 1 × 2 3log e 3 (a) 3 (b) 7 (c) 5 (d) 1
Limits and Continuity 333

Sol. (c) Sol. (a)


Since, LHL (at x = 1 ) = 5 Given function
and RHL (at x = 1) = −1, so function is sin (a + 1) x + sin x
discontinuous at x = 1,  , x<0
And LHL (at x = 2) = 2  x
f (x) =  b ,x=0
and RHL (at x = 2) = 1, so function is

discontinuous at x = 2.  x+ x − x
2
, x>0
So, a = 2and b = 1 (Q a > b),  x 3/ 2
⇒ 3a − b = 5. Function is continuous so,
49. If f :[0 , 2) → [ R is defined by lim f (x) = lim f (x) = f (0)
x → 0− x → 0+
 2x
1 + for 0 ≤ x < 1
f (x) =  k where k > 0, and ⇒ lim f (x) = lim f (x) = b …(i)
x → 0+ x → 0+
 kx for 1 ≤ x < 2
Now, lim f (x) = lim 
sin (a + 1) x + sin x 

f is such that lim f (x) = lim f (x) , then the x → 0− x → 0−  x 
x → 1− x → 1+
 sin (a + 1) x × (a + 1) sin x 
value of k is 2
[23 April 2018, Shift-I]
= lim  + 
x → 0−  (a + 1) x x 
1
(a) 2 (b) 1 (c) 4 (d)  sin x 
4 = (a + 1) + 1 Q lim =1
 x → 0 − x 
Sol. (c)
Given function =a+ 2 …(ii)
1 + 2x , 0 ≤ x < 1 [from Eqs. (i) and (ii)]
 a + 2= b
f (x) =  k , where k > 0 …(iii)
kx , 1≤ x < 2 x + x2 − x
Now, lim f (x) = lim +
Now, lim f (x) = lim 1 +
2x  2 + +
x 3/ 2
 =1 + x→ 0 x→ 0
x → 1− x → 1−  k k
x ( 1 + x − 1) × 1 + x + 1
and lim f (x) = lim (kx) = k = lim
x→1 +
x→1 + x → 0+ x x × ( 1 + x + 1)
Now, given that 1 + x −1 1
lim f (x) = lim f (x) = lim = … (iv)
x→1 −
x→1 + x → 0+ x( 1 + x + 1) 2
2 k+ 2 [From Eqs. (i) and (iv)]
1+=k ⇒ =k 1
k k b=
⇒ k2 − k − 2 = 0 ⇒ k2 − 2k + k − 2 = 0 2
⇒ k(k − 2) + 1 (k − 2) = 0 Put in Eq. (iii)
1
⇒ (k − 2) (k + 1) = 0 a + 2=
So, k = 2, k = − 1 2
k> 0 −3
but ⇒ a=
So, k = 2 ⇒ k2 = 4 2
−3 1
Now, a+ b= +
50. If the function f : R → R defined by 2 2
 sin(a + 1) x + sin x a + b = −1
 , x<0
 x 51. Let f be defined on D = [ R − { − 1, 1} by
f (x) =  b , x = 0 is | x|
 f (x) = , then
x+x − x
2
1 −| x |
 , x>0 [23 April 2018, Shift-I]
 x 3/ 2 (a) f is differentiable on D
continuous on [R, then a + b = (b) f is differentiable on D except at x = 0
[23 April 2018, Shift-I] (c) f is continuous but not differentiable on D
(a) − 1 (b) 2 (c) 1 (d) 3 (d) f is differentiable but not continuous on D
334 AP EAMCET Chapterwise Mathematics

Sol. (b) [6 2 + 12 2 + 18 2 +....+ (6 n)2 ]2


53. lim =
 −x , x<0 n→ ∞[5 + 10 + 15 +.... + 5 n][2 3 + 4 3 + 6 3 +....+8 n3 ]
|x| 1 + x
We have, f (x) = =
1−|x|  x , x≥0 [23 April 2018, Shift-II]
1 − x 4 144 4 144
(a) (b) (c) (d)
f (0 − h) − f (0) 5 5 25 25
LHD (at x = 0) = lim
h→ 0 −h
Sol. (b)
f (− h) − 0 h
= lim = lim [6 2 + 122 + 18 2 + K + (6n) 2]2
h→ 0 −h h→ 01 − h lim
n→ ∞ [5 + 10 + 15 + K + 5n][23 + 4 3 + 6 3 + K + 8n3]
−h
−1 64 [12 + 22 + 32 + K + n2]2
= lim = −1 = lim
n → ∞ (5) [1 + 2 + 3 + K + n] (23)
h → 01 − h

f (0 + h) − f (0) [13 + 23 + 33 + K + n3]


RHD (at x = 0) = lim 2
h→ 0 h n(n + 1) (2n + 1) 
f (h) − 0 64  
= lim = lim
h  6 
h→ 0 h→ 01− h
= lim
h n→ ∞ n(n + 1) 
2
5  n(n + 1) 
 ×2 ×
3
h 
 2   2 
1
= lim =1 (1 + 1 / n) (2 + 1 / n) 
2
h → 01 − h 64 ×  
 6 
∴ LHD ≠ RHD = lim 2
n→ ∞ 1 + 1/ n  3 1 + 1/ n 
So, f (x) is not differentiable at x = 0. 5 ×   ×2  
 2   2 
Hence, f (x) is differentiable on D except at x = 0.
1
64 × × 22
 2 π x + πx cos (π / 2 cos 2 x) = 6 2
=
144
.
52. cos  lim + lim  1 1 5
x → ∞ x − 3 x 5× × 2 × 2
3
x→0 x2  2 2
[23 April 2018, Shift-II]
(a) 1 (b) −1 54. If [ x ] denotes the greatest integer not
1 exceeding the number x , then f (x) defined by
(c) 0 (d)
2
[ x ], if x < 2
f (x) =  is continuous in the
Sol. (b) [ x ] − 1, if x ≥ 2
 π
cos cos2 x   interval. [23 April 2018, Shift-II]
 2π | x | + πx 2 
cos lim + lim  (a) [1, 2 ) ∪ (2, 3) (b) [1, 3)
x → ∞ | x | − 3x x→ 0 x 2
  (c) (1, 3) (d) R
 
Sol. (b)
 π
cos (1 − sin2 x)  Given function,
 2πx + πx 2 
= cos lim + lim   [x] , if x < 2
x → ∞ x − 3x x→ 0 x 2 f (x) = 
  [x] − 1 , if x ≥ 2
 
At x = 2, f(2) = 1
  sin π sin2 x  π 2 
 3πx   

sin x   LHL (at x = 2) = lim [2 − h] = 1
= cos lim + lim  2 × 2 2  h→ 0
x → ∞ − 2x x → 0  π 2  x 
  sin x  and RHL (at x = 2) = lim ([2 + h] − 1) = 2 − 1 = 1
  2    h→ 0

3π π So, f (x) is continuous at x = 2


= cos − +  = cos(− π) = − 1. And [x] is continuous in [n, n + 1), ∀ n ∈ integer.
 2 2 
So, given function continuous in the interval [1, 3).
Limits and Continuity 335

1 + x sin x − cos x 7 = 2β + 3 ⇒ 2β = 4 ⇒ β = 2
55. lim 2
= So, α 2 + β 2 = 1 + 4 = 5.
x→0 tan 2 x [24 April 2018, Shift-I]
(a) 3 (b)
3
(c)
3
(d)
3 57. If f :[0 , 3] → [0 , 3] is defined by f (x)
2 4 16 1 + x , 0 ≤ x ≤ 2
= , then fof is
Sol. (d) 3 − x , 2 < x ≤ 3 [24 April 2018, Shift-I]
1 + x sin x − cos x  0 
lim  form (a) Continuous at x = 1
x→ 0 2
tan 2x 0 
(b) Continuous at x = 2
On rationalising (c) Discontinuous at x = 1and x = 2
1 + x sin x − cos x 1 + x sin x + cos x (d) Continuous on [0, 3]
= lim ×
x→ 0 tan2 2x 1 + x sin x + cos x Sol. (c)
1 + x sin x − cos x 1  f (1 + x) ; 0 ≤ x ≤ 2
= lim × lim Let g(x) = f ( f (x)) = 
+ x+  f (3 − x) ; 2 < x ≤ 3
x→ 0 2 →
tan 2x x 0 1 x sin cos x
(1 − cos x) + x sin x 1  f (1 + x) ; 0 ≤ x ≤ 1
= lim ×
x→ 0 tan2 2x 2 
=  f (1 + x) ; 1 ≤ x ≤ 2
2 x x x  f (3 −
2sin + 2x sin cos  x) ; 2 < x ≤ 3
= lim 2 2 2 ×1
x→ 0 2
tan 2x 2 x ∈[0, 1] ⇒(1 + x) ∈ [1, 2], x ∈[1, 2] ⇒(1 + x) ∈ [2, 3]
2 x  x  x ∈[2, 3] ⇒(3 − x) ∈ [0, 1]
2sin 1+
2  tan x / 2 1 Hence,
= lim 2
×  f (1 + x) ; for 0 ≤ x ≤ 1 ⇒ 1 ≤ x + 1 ≤ 2
x→ 0 tan 2x 2 
2 g(x) =  f (1 + x) ; for 1 ≤ x ≤ 2 ⇒ 2 ≤ x + 1 ≤ 3

1  sin x / 2 2⋅ x / 2   f (3 − x) ; for 2 < x ≤ 3 ⇒ 0 ≤ 3 − x ≤ 1
⋅ 1 + tan x / 2
 
4  x/2   
= lim Now, if (1 + x) ∈ [1, 2] then,
x→ 0 2
4 tan 2x  f (1 + x) = 1 + (1 + x) = 2 + x …(i)
 2x  [From original definition of f (x)]
On applying limits, we get Similarly, if (1 + x) ∈ (2, 3) then,
1
× ()
1 2 × [1 + 2 × 1] f (1 + x) = 3 − (1 + x) = 2 − x …(ii)
3
= 4 = .
()2
41 16 If (3 − x) ≤ (0, 1),
f (3 − x) = 1 + (3 − x) = 4 − x …(iii)
56. If the function f defined by Using Eqs, (i), (ii), (iii), we get
 cos x , if x≤0 2 + x ; 0 ≤ x < 1

 g(x) =  2 − x ; 1 < x ≤ 2
3 x + α , if 0 < x < 2
f (x) =  4 − x ; 2 < x ≤ 3

 βx + 3 , if 2 ≤ x ≤ 4
 11 Here, as g(x) change the inequality sign at x = 1
, if x>4
and x = 2.
where, α and β are real constants, is Thus, to check continuity at x = 1 and x = 2.
continuous on R, then α 2 + β 2 = Now, we will check the continuity of g(x) at
x = 1, 2.
[24 April 2018, Shift-I]
At x = 1, LHL = lim g(x) = lim (2 + x) = 3
(a) 3 (b) 9 (c) 5 (d) 1 x → 1− x → 1−
RHL = lim g(x) = lim (2 − x) = 1
Sol. (c) x →1+ x →1+
f (x) is continuous on R, so As. LHL ≠ RHL, g(x) is discontinuous at x = 1.
at x = 0, lim f (x) = f (0) At x = 2. LHL = lim g(x) = lim (2 − x) = 0
x → 0+ x → 2− x → 2−
⇒ 0 + α = cos 0 = 1 ⇒ α = 1 RHL = lim g(x) = lim (4 − x) = 2
x → 2+ x → 2+
at x = 2, lim f (x) = f (2) As LHL ≠ RHL, g(x) is discontinuous at x = 2.
x → 2−
6 + α = 2β + 3 Thus, g(x) is continuous for all x ∈ [0, 1) ∪ (1, 2.
)
24
Differentiation
 1  
1. If 3 f (x) − 2 f   = x , then f ′(2) is 3. The derivative of tan −1 
x
 with
 x
1 + 1 − x 2 
[17 Sep. 2020, Shift-I]
7 1 2  1 
(a) (b) (c) (d) 2 respect to sec −1  2  is
2 2 7  2 x − 1
Sol. (b) [17 Sep. 2020, Shift-I]
−1 −1
3 f (x) − 2 f   = x , f ′(2) = ?
1 1 1
(a) (b) (c) (d)
 x 2 4 4 2
3 f ′(x) − 2 f ′    − 2  = 1
1 1 Sol. (c)

 x x 
 x 
Let, y = tan−1  
At (x = 2) ⇒ 3 f ′(2) + f ′   = 11
1 1
…(i) 1 + 1 − x 2 
2  2
u = sec−1  2 
1
And At  x =  ⇒ 3 f ′   + 8 f ′(2) = 1
1 1
…(ii)  2x − 1 
 2  2
Let x = cosθ
On solving Eqs. (i) and (ii), we get
cosθ 
 f ′(2) = 1  y = tan−1  −1
 , u = sec (sec 2θ)
⇒    1 + sinθ 
 2
y = tan−1   , (u = 2θ)
1
dy ⇒ 
2. If y = sin 98(x). cos 39(x), then find  secθ + tanθ 
dx
⇒ y = tan−1 (secθ − tanθ), (u = 2θ)
[17 Sep. 2020, Shift-I]
(a) (98cos 99 x ⋅ sin38 x) + (39 sin40 x ⋅ cos 97
x) ⇒ tan y = secθ − tanθ, u = 2θ
dy du
(b) (99 cos 98 x ⋅ sin39 x) − (40 sin39 x ⋅ cos 98 x) ⇒ sec y ⋅
2
= secθ ⋅ tanθ − sec2 θ, =2
dθ dθ
(c) (98 cos 99 x ⋅ sin38 x) − (39 sin40 x ⋅ cos 97 x)
dy secθ ⋅ tanθ − sec2 θ du
(d) 99 cos 98 x ⋅ sin39 x) + (39sin40 x ⋅ cos 97 x) ⇒ = , =2
dθ sec2 y dθ
Sol. (c)
dy secθ ⋅ tanθ − sec2 θ du
y = sin98 x ⋅ cos39 x ⇒ = , =2
dθ 1 + (secθ − tanθ)2 dθ
dy
= sin98 x ⋅ 39 cos38 x(− sin x)
dx dy 1  secθ ⋅ tanθ − sec2 θ 
⇒ = 
+ cos39x ⋅ 98sin97 x(cos x) du 2 1 + sec2 θ + tan2 θ − 2secθ ⋅ tanθ 
= 98sin97 x ⋅ cos40 x − 39sin99 x ⋅ cos38 x 1 secθ(tanθ − secθ) dy 1
= ⇒ =−
= 98 cos99 x ⋅ sin38 x − 39sin40 x ⋅ cos97 x 2 2sec2 θ − 2secθ ⋅ tanθ du 4
Differentiation 337

3cos−1  
dy dy d x
4. If x 2019 ⋅ y 2020 = (x + y)4039 , then = ∴ =
dx dx dx  3
−1
×  
[17 Sep. 2020, Shift-I] d x
= 3×
x y 2 dx  3
1 −  
(a) 0 (b) (c) (d) 1 x
y x  3
Sol. (c) −3 1
= 3× × = − 3/ 9 − x2
x 2019 ⋅ y 2020 = (x + y)4039 9− x 2 3
On differentiating
dy 6. Differentiation of
⇒ 2020 y 2019 x 2019 + 2019 x 2018 ⋅ y 2020
dx (x 2 − 5 x + 8) × (x 3 + 7 x + 9) can be done by
= 4039(x + y)4038 ⋅ 1 + 
dy [17 Sep. 2020, Shift-II]
 dx  (a) Only by using product rule
(x + y)4039 dy (x + y)4039 (b) Only by obtaining a single polynomial,
⇒ 2020 + 2019 expanding it
y dx x
(c) Only by using logarithmic differentiation
= 4039(x + y)4038 1 + 
dy
(d) All of the options are correct
 dx 
Sol. (d)
 2020 dy 2019   dy 
⇒  +  (x + y) = 40391 +  As given function contains two factors we can
 y dx x  dx apply product rule or we can multiply factors to
2020(x + y) dy 2019
(x + y) = 40391 + 
dy get a single polynomial.
⇒ ⋅ +
y dx x  dx  Another way is to take logarithm and then
dy  2020(x + y)  2019(x + y) differentiate.
⇒ 4039 − =
dx  y  x 7. If log(x + y) − 2 xy = 0, then y′ (0) =
dy  2019 y − 2020 x  2019(x + y) [17 Sep. 2020, Shift-II]
⇒ = − 4039
dx  y  x (a) 2 y + 1 (b) 2 y − 1 (c) 0
2 2
(d) 2 y2
dy y
⇒ = Sol. (b)
dx x
log(x + y) − 2xy = 0
d  − 1  4 x3  Differentiating with respect to ‘x’ we get,
5. cos  − x   =
dx   27 
1 
1 +
dy   dy + y = 0
 − 2 x 
[17 Sep. 2020, Shift-II] x + y dx   dx 
3 1
(a) (b) 1  1  dy
9− x 2
9 − x2 ⇒ +  − 2x  − 2y = 0
x+ y x+ y  dx
−3 −1
(c) (d) 1
2y − 1
9 − x2 9 − x2 2y −
dy x+ y dy y = 2y 2 − 1
⇒ = ⇒ =
Sol. (c) dx 1
− 2x dx x =0
1
 4x  3 x+ y y
Let, y = cos−1  − x
 27 
 π dy π
 x 3
8. If y = 2 x + cos 2 2 x +  , then at x = .
x   4
= cos−1  4   − 3   dx 4
  3   3  [18 Sep. 2020, Shift-I]
x x 2 2 2 2
Let = cos A then, A = cos−1 (a) (b) 2 π + 1 (c) (d)
3 3 π+1 π+1 π+1
Also, y = cos−1 (4 cos3 A − 3cos A)
Sol. (a)
= cos−1 (cos 3A) π
Given, y = 2x + cos2  2x + 
= 3A = 3cos−1  
x  4
 3
338 AP EAMCET Chapterwise Mathematics

π
So, at x = 12. The derivative of
4  1+ x 
π π π π π π 1 f (x) = cos −1 sin  + x with respect to
x
y= + cos2  +  = + sin2 = + 2
2  2 4 2 4 2 2  
π x at x = 1 is equal to [18 Sep. 2020, Shift-II]
Now, as y 2 = 2x + cos2  2x +  1 3 2
 4 (a) 1 (b) (c) (d)
4 4 3
On, differentiating w.r.t. x, we get
π Sol. (c)
= 2 − 2sin  4 x + 
dy
2y 
 2 1+ x
dx Given, f (x) = cos−1  sin + x
x

dy 1 − cos 4 x  2 
= 2 − 2cos 4 x ⇒ =
dx y π  1+ x
= − sin−1  sin + x
x
dy 1 − (−1) 2 2 2  2 
⇒ | π= =
dx x = 4 π 1 π +1
+ π 1+ x
2 2 = − + xx
2 2
9. If f (x) = x 4 − x 3 + 7 x 2 + 14, then what is the −1 1
∴ f ′ ( x) = ⋅ + x x (log e (ex))
value of f ′(5)? [18 Sep. 2020, Shift-I] 2 2 1+ x
(a) 594 (b) 549 (c) 954 (d) 495  d(x x ) 
Q dx = x log e (ex)
x

Sol. (d)  
Since, f (x) = x 4 − x 3 + 7 x 2 + 14 On putting x = 1, we get
Then, f ′(x) = 4 x 3 − 3x 2 + 14 x −1 3
f ′(1) = +1 =
∴ f ′(5) = 500 − 75 + 70 = 495 4 4
Hence, option (c) is correct.
10. If f (x) = x + 2 2 x − 4 + x − 2 2x − 4 ,
then the value of 10 × f ′ (102) = 13. The derivative of
[18 Sep. 2020, Shift-I]  1 + sin x + 1 − sin x 
y = tan − 1   with
(a) 1 (b) 2 (c) 102 (d) − 1  1 + sin x − 1 − sin x 
Sol. (a) respect to x is equal to [21 Sep. 2020, Shift-I]
Given, f (x) = x + 2 2x − 4 + x − 2 2x − 4 1
(a) − 1 (b) 0 (c) ± 2 (d) ±
2
= (x − 2) + 2 + 2 2 x − 2
Sol. (d)
+ (x − 2) + 2 − 2 2 x − 2
It is given that,
=| x − 2 + 2| + | x − 2 − 2|  1 + sin x + 1 − sin x 
y = tan−1  
for x ≥ 4
 1 + sin x − 1 − sin x 
f (x) = x−2+ 2+ x − 2− 2= 2 x − 2
x x x x
1 Q 1 + sin x = sin2 + cos2 + 2sin cos
∴ f ′(x) = 2 2 2 2
x−2
x
So, 10 × f ′ (102) = 10 ×
1
=1 = cos + sin x/2
102 − 2 2
x x x x
d log e 1 + tan 2 x and 1 − sin x = sin2 + cos2 − 2sin cos
11. (e )= 2 2 2 2
dx [18 Sep. 2020, Shift-I]
x x
(a) sec 2 ( x) ⋅ tan( x) (b) sec( x) ⋅ tan2 ( x) = cos − sin
2 2
(c) sec( x) ⋅ tan( x) (d) tan2 ( x)
y = tan−1  cot  , when cos + sin
x x x

Sol. (c)  2 2 2
d loge d x x π x
(e 1 + tan2 x = (sec x) = sec x tan x. and cos − sin = − are positive
dx dx 2 2 2 2
Differentiation 339

π π
tan−1  tan  = , when cos + sin is positive Q Domain of cosec−1 x is  − ,  − {0} and the
x x x x
 2 2 2 2  2 2 
π π
is define for x ∈  − , 0 ∪  0,  . Because
x x dy
and cos − sin is negative
2 2 dx  2   2
−1  x x x x for derivatives we should exclude the end points
tan  tan  = , when cos + sin is negative
 2  2 2 2 of a internal.
x x Hence, option (c) is correct.
and cos − sin is positive
2 2
16. If 2 f (sin x) + f (cos x) = x , then f ′ (x) =
−1  x π x x x
tan  cot  = − , when cos + sin and as [21 Sep. 2020, Shift-I]
 2  2 2 2 2
x x 1 −1 x − x
cos − sin is negative. (a) (b) (c) (d)
2 2 1− x 2
1− x 2
1− x 2
1 − x2
dy 1
So, =± Sol. (a)
dx 2
It is given that,
Hence, option (d) is correct.
2f (sin x) + f (cos x) = x … (i)
y+ ey+K dy π
14. If x = e y + e , then = by replacing x by − x, we get
dx 2
[21 Sep. 2020, Shift-I] π
2 f (cos x) + f (sin x) = − x … (ii)
1− x 1 2
(a) (b)
x x from Eqs. (i) and (ii), we get
x 1+ x π
(c) (d) 3 f (sin x) = 3x −
1+ x x 2
π
Sol. (a) ⇒ 3 f (x) = 3sin−1 x − (on replacing x by sin−1 x)
2
It is given that, Now, on differentiating both sides w.r.t. ‘x ’, we
y + ...
ey+e get
x = ey +
1 1
3 f ′(x) = 3 ⇒ f ′(x) =
⇒ x = ey + x
1− x 2
1 − x2
⇒ log e x = x + y ⇒ y = log e x − x
Hence, option (a) is correct.
On differentiating both sides with respect to ‘x’,
we get  1 + x 2 − 1
1− x 17. The derivate of y = tan −1   is equal to
dy 1
= −1 =  x 
dx x x
 
Hence, option (a) is correct. [21 Sep. 2020, Shift-II]
2 1
dy −1 (a) (b) (c) (1 + x2 ) (d) 2 (1 + x2 )
15. If y = cosec − 1(x) and = , then (1 + x2 ) 2 (1 + x2 )
dx | x | x 2 − 1
Sol. (b)
[21 Sep. 2020, Shift-I]
 1 + x 2 −1
π
(a) y ∈  − , 0 Given, y = tan−1  
 
 2   x 
π Let, x = tanθ
(b) y ∈  − , 2 π 
 2   1 + tan2 θ − 1 
π   π So, y = tan−1  
  tanθ 
(c) y ∈  − , 0 ∪  0,   
 2   2
secθ − 1  −1  1 − cos θ 
(d) y ∈R = tan−1   = tan  
 tanθ   sinθ 
Sol. (c) = tan−1 (tanθ / 2) = θ / 2
It is given that, y = cosec−1 x 1 dy 1  1 
−1 ⇒ y = tan−1 x ⇒ =  
and
dy
= 2 dx 2  1 + x 2 
dx |x| x 2 − 1 Hence, option (b) is correct.
340 AP EAMCET Chapterwise Mathematics

18. If f (x) = x 4 − x 3 + 7 x 2 + 14, then what is the Sol. (a)


value of f ′ ′(5) ? [21 Sep. 2020, Shift-II] Given equation 2x 2 − 3xy + y 2 + x + 2y − 8 = 0
(a) 842 (b) 248 (c) 284 (d) 482 On differentiating w.r.t. x, we get
dy dy dy
Sol. (c) 4 x − 3x − 3y + 2y + 1 + 2 = 0
dx dx dx
Given, f (x) = x 4 − x 3 + 7 x 2 + 14 dy
⇒ (3x − 2y − 2) = 4 x − 3y + 1
⇒ f ′(x) = 4 x 3 − 3x 2 + 14 x dx
⇒ f ′ ′(x) = 12x 2 − 6 x + 14 dy 4 x − 3y + 1 3y − 4 x − 1
⇒ = =
∴ f ′ ′(5) = 300 − 30 + 14 = 284 dx 3x − 2y − 2 2y − 3x + 2
Hence, option (c) is correct. Hence, options (a) is correct.

1 + tan x dy e x log x dy
19. If y = , then = 21. If y = 2
, then =
1 − tan x dx x dx [22 Sep. 2020, Shift-I]
[21 Sep. 2020, Shift-II] e x {1 + ( x + 2 ) log x} e x {1 − ( x − 2 ) log x}
(a) (b)
1  1 − tan x  2 π  x 3
x4
(a)   sec  + x e {1 − ( x − 2 ) log x}
x
e {1 + ( x − 2 ) log x}
x
2  1 + tan x  4 (c) (d)
1  1 − tan x  π  x3 x3
(b)   sec  + x
2  1 + tan x  4 Sol. (d)
 1 − tan x  2 π  It is given that,
(c)   sec  + x e x log x
 1 + tan x  4 y=
1  1 + tan x  2 π
x2

(d)   sec  + x On applying logarithm both sides, we get
2  1 − tan x  4
log e y = x + log e (log x) − 2 log x
Sol. (a) Now, on differentiating both sides w.r.t ‘x’, we
1 + tan x get
Given, y =
1 − tan x 1  dy  1 2
  =1 + −
dy 1 1 − tan x y  dx  x log x x
∴ =
dx 2 1 + tan x dy  1 2
⇒ = y 1 + − 
(1 − tan x)sec2 x + (1 + tan x)sec2 x dx  x log x x 
×
(1 − tan x)2 e x log x  x log x + 1 − 2 log x 
=  
1 1 − tan x 2sec x 2
x2  x log x 
= .
2 1 + tan x (1 − tan x)2 ex
=
[1 + (x − 2) log x]
1 1 − tan x 2 x3
=
2 1 + tan x (cos x − sin x)2 Hence, option (d) is correct.
1 1 − tan x 1  x 2 − y2 
= . dy
2 1 + tan x  π
2 22. If cos −1   = sin −1 (a) then is equal
  x 2 + y2  dx
 cos x +  
 4 
to [22 Sep. 2020, Shift-I]
1 1 − tan x π
= sec2  + x  (a) y / x
2 1 + tan x 4 
(b) − y / x
Hence, option (a) is correct. (c) x / y
dy (d) − x / y
20. Find if 2 x 2 − 3 xy + y 2 + x + 2 y − 8 = 0
dx Sol. (a)
[21 Sep. 2020, Shift-II]
 x 2 − y2 
3y − 4x − 1 3y + 4x − 1 Given, cos− 1  2 −1
 = sin a
(a) (b)  x + y2 
2 y − 3x + 2 2 y − 3x + 2
3y − 4x − 1 3y − 4x − 1 x 2 − y2
(c) (d) ⇒ = cos(sin− 1 a) = c (let)
2 y + 3x + 2 2 y − 3x − 2 x 2 + y2
Differentiation 341

On applying componendo and dividendo law, Sol. (a)


we get y = log(cos hx)
2x 2 c + 1 y2 1 − c Differentiating w.r.t. ‘x ’ on both sides
= ⇒ =
2y 2
1−c x2 1 + c dy 1
= (sin hx)
On differentiating both sides w.r.t ‘x’, we get dx cos h x
dy dy y dy
2x 2 y − 2y 2 x = 0 ⇒ = = tan hx
dx dx x dx
 1+ x − 1− x Again differentiating w.r.t. ‘x’ on both sides
23. The derivative of y = sin −1   d2y
= sech2 x
 2  dx 2
is [22 Sep. 2020, Shift-II] Hence, option (a) is correct.
1 1
(a) (b)  x dy
1+ x 2
1 − x2 25. If log x 2 + y 2 = tan −1   , then is equal
 y dx
1 1
(c) (d) to [22 Sep. 2020, Shift-II]
2 1 + x2 2 1 − x2 y− x x+ y 1 1
(a) (b) (c) (d)
Sol. (d) y+ x x− y y+ x x− y
 1+ x − 1− x Sol. (a)
y = sin−1  
 2   x
log x 2 + y 2 = tan−1  
Put, x = cos 2θ ⇒ 2θ = cos x −1  y
1 Differentiate w.r.t ‘x’ on both sides,
θ = cos−1 x
2 1 d 1 d  x
( x 2 + y 2) = ⋅  
 x  dx  y 
2
 1 + cos 2θ − 1 − cos 2 θ  x 2 + y 2 dx
= sin−1   1+  
 2   y
1 1 d
 2 cos θ − 2 sin θ  . . (x 2 + y 2)
= sin−1  
 2  x 2 + y2 2 x 2 + y 2 dx

cos θ − sin θ  1 1 ⋅ y − x ⋅ y′
= sin−1  =
 y2 + x 2 y2
 2 
2
y
= sin−1 
1 1 
cos θ − sin θ 
 2 2 1 y2 y − xy ′
(2x + 2yy ′) = 2
π π 2(x 2 + y 2) (x + y 2) y 2
= sin−1  sin cos θ − cos sin θ
 4 4   y − xy ′ 
x + y. y ′ = y 2  
π π  y2 
= sin−1 sin  − θ  = − θ
 4   4 x + yy ′ = y − xy ′
π 1 yy ′ + xy ′ = y − x
y = − cos−1 x
4 2 y ′(x + y) = y − x
Now, differentiate w.r.t ‘x’, on both sides y −x
y′ =
dy 1 −1 dy 1 x+ y
=0− . ⇒ =
dx 2 1− x 2 dx 2 1 − x 2 dy y − x
=
Hence, option (d) is correct. dx y + x
Hence, option (a) is correct.
d2y
24. If y = log(cosh x) then = d
dx 2 26. {(1 + x 2) tan −1 (x)} =
[22 Sep. 2020, Shift-II] dx [22 Sep. 2020, Shift-II]
(a) sech x2
(b) −sech x 2 (a) x tan−1( x) (b) 2 tan−1 ( x)
(c) sinh x (d) − sinh x (c) 2 x tan−1( x) + 1 (d) x tan−1 ( x) + 1
342 AP EAMCET Chapterwise Mathematics

Sol. (c) 1
f ′(e) =
d dv dv 2e
We have, (u. v) = u ⋅ + v.
dx dx dx Hence, option (c) is correct.
d
[(1 + x)2.tan−1 (x)] = (1 + x 2) d  x 4 + x 2 + 1
dx 29. If   = ax + b, then a − b =
d d dx  x 2 + x + 1 
. tan−1 x + (1 + x 2) .tan−1 x
dx dx [23 Sep. 2020, Shift-I]
1
= (1 + x 2) . + (2x). tan−1 x (a) 3 (b) 4 (c) 1 (d) 2
1 + x2
Sol. (a)
= 1 + 2 x tan−1 x = 2 x tan−1 x + 1 We have,
Hence, option (c) is correct. x 4 + x 2 + 1 = (x 2)2 + 2x 2 + 1 − x 2
27. Eliminating a and b from the relation = (x 2 + 1)2 − x 2
y = a log x + b, we get = (x 2 + 1 + x) (x 2 + 1 − x)
[23 Sep. 2020, Shift-I]
[Q a 2 − b 2 = (a + b) (a − b)]
(a) xy2 + y1 = 0 (b) xy − y2 = 0
(c) xy1 + y2 = 0 (d) y2 y2 + x = 0 x 4 + x 2 + 1 = (x 2 + x + 1) (x 2 − x + 1)
Consider,
Sol. (a)
d  x4 + x2 + 1 d  (x 2 + x + 1) (x 2 − x + 1) 
y = a log x + b ∴  2 =  
dx  x + x + 1  dx  (x 2 + x + 1) 
differentiate w.r. to ‘x’, we get
1 d 2
y′ = a ⋅ + 0 = (x − x + 1)
x dx
⇒ xy′ = a ⇒ xy′ ′ + 1 ⋅ y′ = 0 d  x4 + x2 + 1
  = 2 x −1
⇒ xy′ ′ + y′ = 0 (or) xy2 + y1 = 0 dx  x 2 + x + 1 
Hence, option (a) is correct. ax + b = 2x − 1 ⇒ a = 2 and b = −1
28. If f (x) = log x 2 (log x), then f ′(e) is equal to ∴a − b = 2 − (−1) = 3
Hence, option (a) is correct.
[23 Sep. 2020, Shift-I]
1 1  4x  2 + 3x 
(a) e (b) (c) (d) 2e 30. If y = tan −1   + tan −1   , then
e 2e 1 + 5 x 2   3 − 2x 
Sol. (c) dy
=
Given, f (x) = log(log
2
x)
dx [23 Sep. 2020, Shift-I]
x
1 2 5 2
loglog x
loglog x (a) + (b) +
f (x) = e
2
⇒ f (x) = e
1 + 25 x2 1 + x2 1 + 25 x2 1 + x2
log xe 2log e x
5 1
(c) (d)
d d 1 + 25 x2 1 + 25 x2
2log xe . loglog
e
x
− loglog
e
x
2. (log xe )
⇒ f ′(x) = dx dx
(2log xe )2 Sol. (c)
 4x  −1  2 + 3x 
2log xe ⋅
1 1
× − loglog x
. 2.
1 y = tan−1   + tan  
log x x
e
x  1 + 5x 2   3 − 2x 
=
4(log xe )2  2+ x
5x − x 
−1  −1  3 
2log xe loglog x = tan   + tan 
− 2. e
 1 + (5x)(x)  2 
1 − x 
f ′(x) = x ⋅ log x x 2 x  3 
4(log e )
= tan−1 (5x) − tan−1 (x) + tan−1   + tan−1 x
2
2.log ee 2  3
− 2.loglog
e
e
− 2.(0)
y = tan−1 (5x) + tan−1  
e.log e e 2 2
f ′(e) = = =
4(log ee)2 4 ⋅()
12 4⋅ e  3
Differentiation 343

differentiate w.r.to ‘x’ 32. If y = f (x) is twice differentiable function


1 d 5
y′ = . (5x) + 0 =
1 + (5x)2 dx 1 + 25x 2 dy d2y
such that at a point P, = 4 , 2 = − 3,
dy 5 dx dx
=
dx 1 + 25x 2  d2x 
then  2  =
Hence, option (c) is correct.  dy  P [20 April 2019, Shift-I]
dy 64 16 3 3
31. If 1 − x + 1 − y = a(x − y ), then y
6 6 3
= 3 2
(a) (b) (c) (d)
dx 3 3 16 64
[20 April 2019, Shift-I] Sol. (d)
1 − y6 1 − y6  
(a) (b) x
1 − x6 1 − x6 d2x d  dx  dx d  1 
Q =   =  
1 − y6 1 − y6 dy 2 dy  dy  dy dx   dy  
1  
(c) x2 (d)   dx  
1− x 6
x 2
1 − x6
 d2y   d2y 
−   2
Sol. (c)  dx   dx  = −  dx 
2
= 
Given equation is  dy    dy  2   dy 
3

1 − x 6 + 1 − y 6 = a(x 3 − y 3)      
 
 dx  dx
1 − x 6 + 1 − y6  d2x  (−3) 3
⇒ =a ∴  2 = − =
x 3 − y3  dy  p (4)3 64
On differentiating both sides w.r.t., x, we get Hence, option (d) is correct.
  −6 x 5 6 y 5 dy 
(x − y ) 
3 3
− −( 1 − x 6 33. If x = sec θ − cos θ, y = sec 10θ − cos10 θ and
  2 1 − x 6 2 1 − y 6 dx 
   dy 
2
(x 2 + 4)   = k (y 2 + 4), then k =
 dx 
+ 1 − y 6) 3x 2 − 3y 2  
dy
 dx   [20 April 2019, Shift-II]
=0
(x 3 − y 3)2 1
(a) (b) 1 (c) 10 (d) 100
 100
y 5(x 3 − y 3)  dy
⇒  y 2 ( 1 − x 6 + 1 − y 6) − Sol. (d)
 1 − y 6  dx

Given, y = sec10 θ − cos10 θ and x = secθ − cosθ
x5
= x ( 1− x + 1− y )+
2 6 6
(x − y )
3 3
So,
dy
= 10 (sec9 θ secθ tanθ + cos9 θ sinθ)
1 − x6 dθ
dy  1 − x 6 + 1 − y 6 + (1 − y 6) − y 3 x 3 + y 6  = 10 (sec10 θ + cos10 θ) tanθ
⇒ y2   dx
dx 
 1 − y6  and = secθ tanθ + sinθ = (secθ + cosθ) tanθ

 (1 − x ) + 1 − y 1 − x + x − x y 
6 6 6 6 3 3
= x2  dy sec10 θ + cos10 θ
 ∴ = 10
 1 − x6  dx secθ + cosθ
2
dy  1 − x 1 − y + 1 − x y 
2
 dy  = 100  sec θ + cos θ 
6 6 3 3 10 10
⇒ y2   ⇒    
 dx   secθ + cosθ 
dx 
 1 − y6 
1 + 1 − y 1 − x − x y 3 
6 6 3 sec20 θ + cos20 θ + 2
= 100
= x2   sec2 θ + cos2 θ + 2
 1 − x6 
(sec10 θ − cos10 θ )2 + 4
= 100
dy 1 − y6 (secθ − cosθ)2 + 4
⇒ y2 = x2
dx 1 − x6  y2 + 4 
= 100  2 
Hence, option (c) is correct.  x + 4
344 AP EAMCET Chapterwise Mathematics

2
⇒ (x 2 + 4)   = 100 (y 2 + 4)
dy ( x 2 + a 2 + x) 1
 dx  = =
2 x + a (x +
2 2
x + a)
2 2
2 x + a2
2

= K (y 2 + 4) (given)
 xx − x− x 
So, K = 100 36. If f (x) = cot − 1   , then f ′ ()
1 =
−1  2 
34. If y = esin x
, then (1 − x 2) y 2 − xy1 =
[21 April 2019, Shift-I]
[20 April 2019, Shift-II]
(a) − log2 (b) log2 (c) 1 (d) − 1
(a) 0 (b) 1 (c) y (d) 2 y
Sol. (d)
Sol. (c) Given,
−1
Given, y = esin x
 xx − x− x 
f (x) = cot − 1  
Differentiating w.r.t. x  2 
d sin−1 x −1 d
y1 = e ⇒ y1 = esin x ⋅ sin−1 x  x 2x − 1 
dx dx Let y = cot − 1  
−1 1  2⋅ x x 
y1 = esin x ⋅ ⇒ y1 1 − x 2 = y …(i)
1 − x2 Put x x = tanθ
Again differentiating w.r. to x,  tan2 θ − 1 
∴ y = cot − 1  
d
(y1 ⋅ 1 − x 2) =
d
(y)  2tanθ 
dx dx = cot − 1 (− cot 2θ)
1
⇒ y2 ⋅ 1 − x 2 + y1 ⋅ (− 2x) = y1 −1
2 1 − x2 y = π − cot (cot 2θ)

⇒ y2 ⋅ 1 − x 2 − y1 ⋅
x
= y1 y = π − 2θ
1 − x2 y = π − 2tan− 1 (x x )
y2(1 − x 2) − xy1 dy 2
⇒ = y1 ∴ =− ⋅ x x (1 + log x)
1 − x2 dx 1 + x 2x
⇒ y2(1 − x 2) − xy1 = y1 1 − x 2 dy −2
So, = ⋅11 (1 + log1)
y2(1 − x 2) − xy1 = y dx x =1 1 + ()
12
2
[Q from Eq. (i)] =− ⋅1 = − 1
∴Hence, option (c) is correct. 2

d    37. If a ≠ 0, x = a(t + sin t) and y = a(1 − cos t),


35.  log  x + x 2 + a2  =
dx    d2y 2π
then at t = is
[21 April 2019, Shift-I] dx 2 3 [21 April 2019, Shift-I]
1 4 1 a
(a) x + a
2 2
(b) (a) (b) (c) 4a (d)
x + a
2 2 a 4a 4

(c)
1
(d)
1 Sol. (a)
2 x 2 + a2 2( x + x 2 + a2 ) Given,
x = a(t + sin t) … (i)
Sol. (c) and y = a(1 − cos t) … (ii)
d   
 log  x + x 2 + a 2   Differentiate Eq. (i) w.r.t. t, we get
dx   dx
= a(1 + cos t)
  dt
× 
1 1
= Differentiate Eq. (ii) w.r.t. t, we get
 2 
x+ x + a
2 2
2 x + x + a 
2
dy
= a sin t
  dx
1 2x
× 1 + ⋅  dy dy / dt a sin t sin t
 2 x 2 + a 2  ∴ = = =
 dx dx / dt a(1 + cos t) (1 + cos t)
Differentiation 345

d2y d  dy  d  dy  dt 39. If a ≠ b, x ≠ nπ, n ∈ Z and


Now, =  =   ×
dx 2 dx  dx  dt  dx  dx
d2y
d  sin t  1 y 2 = a 2 cos 2 x + b2 sin 2 x , then + y=
=   × dx 2
dt  1 + cos t  a(1 + cos t) [21 April 2019, Shift-II]
(1 + cos t) cos t − sin t (− sin t) 1  ab 
2
1  ab 
2
(ab )2
= × (a)  
ab
(1 + cos t)2 a(1 + cos t)
(b)   (c) (d)
 y y y  y y3
2
d y (cos t + 1) 1
= = Sol. (b)
dx 2 a(1 + cos t)3 a(1 + cos t)2
It is given that, if a ≠ b, x ≠ nπ, n ∈ z and
At t = 2π / 3 y 2 = a 2 cos2 x + b 2 sin2 x
d2y 1 4 On differentiating both sides w.r.t. x, we get
= = dy
dx 2 2
a = − a 2 sin 2x + b 2 sin 2x
a  
1 2y
 2 dx
dy
⇒ 2y = (b 2 − a 2)sin 2x
 1+ x dy dx
38. If y = sin 2 cot −1  , then =
 1− x dx On differentiating again both sides w.r.t. x, we
get
[21 April 2019, Shift-II]
 d2y dy 
2
−1 1 1 2  y 2 +    = 2(b 2 − a 2) cos 2x
(a) (b) (c) (d) 1
 dx  dx  
2 1+ x 1− x
2
d 2 y  dy 
Sol. (a) ⇒ y +   = (b 2 − a 2) cos 2x
dx 2  dx 
Given,
On, multiply by y 2 on both sides, we get
 1+ x
y = sin2  cot −1
2
 d2y
y 3 2 + y 2   = y 2(b 2 − a 2) cos 2x
dy
 1− x  dx 
dx
2
x = cos 2θ d2y
= y 2(b 2 − a 2) cos 2x − y 2  
Let …(i) dy
⇒ y3
 1 + cos 2θ  dx 2  dx 
∴ y = sin2  cot −1
 1 − cos 2θ  On adding y 4 on both sides, we get
2
d2y
= y 2(b 2 − a 2) cos 2x − y 2   + y 4
[Q1 + cos 2θ = 2cos2 θ and 1 − cos 2θ = 2sin2 θ] dy
y4 + y3
dx 2  dx 
 2cos2 θ 
= sin2  cot −1 = (a 2 cos2 x + b 2 sin2 x)(b 2 − a 2)

 2sin2 θ 
(cos2 x − sin2 x) − (b 2 − a 2)sin x cos x)2
= sin2[cot −1 (cot θ)] + (a 2 cos2 x + b 2 sin2 x)2
⇒ y = sin θ 2
= a 2b 2 cos4 x − a 2b 2 cos2 x sin2 x
[Differentiating w.r.t. θ, we get] − a 4 cos4 x + a 4 cos2 x sin2 x + b 4 sin2 x cos2 x
dy dy
⇒ = 2sinθ cosθ ⇒ = sin 2θ …(ii) − b 4 sin4 x − a 2b 2 sin2 x cos2 x + a 2b 2 sin4 x
dθ dθ
− b 4 sin2 x cos2 x + 2b 2a 2sin2 x cos2 x
Differentiating Eq. (i) w.r.t. θ, we get
dx − a 4 sin2 x cos2 x + a 4 cos4 x + 2a 2b 2
= − 2sin 2θ …(iii)
dθ sin2 x cos2 x + b 4 sin4 x
dy dy / dθ sin 2θ
Q = = = a 2b 2 cos4 x + a 2b 2 sin2 x + 2a 2b 2 sin2 x cos2 x
dx dx / dθ −2sin 2θ
= a 2b 2(cos2 x + sin2 x)2 = a 2b 2
[by Eqs. (ii) and (iii)] 2

dy
=−
1 d2y d2y 1  ab 
⇒ y4 + y3 = a 2b 2 ⇒ 2 + y =   .
dx 2 dx 2
dx y y 
346 AP EAMCET Chapterwise Mathematics

40. Match for each functions in List - I to its 1 + cosec t π


t =
41. If f () for 0 < t < and
derivative given in List-II [22 April 2019, Shift-I] 1 − cosec t 2
List I List II f ′ ()
t = f () t then g()
t g(), t =
[22 April 2019, Shift-I]
 2x 
(A) sin−1   (I) cos x − sin x (a) −4 cosec 2t (b) 4 cosec 2t
 1 + x2  (c) 2 sin2t (d) 4cosec t
 1− x −1 Sol. (b)
(B) tan−1   (II)
 1 + x 1 + x2 1 + cosec t
t =
Given, f ()
2 1 − cosec t
log(sin x + cos x )
(C) e (III) 1 + x2 Differentiating w.r.t. t, we get
π (1 − cosec t)(− cosec t cot t)
(D) 1 − sin2 x  0 < x <  (IV) cos x + sin x − (1 − cosec t)(cosec t cot t)
 4 f ′()
t =
(1 − cosec t)2
(V) − sin x − cos x
− cosec t cot t + cosec2t cot t
The correct match is − cosec t cot t − cosec2t cot t
A B C D A B C D =
(1 − cosec t)2
(a) III II I V (b) II III V IV
−2 cosec t cot t 1 + cosec t
(c) II III V I (d) III II I IV = ×
(1 − cosec t)2 1 + cosec t
Sol. (d)
 2x   1 + cosec t   2cosec t cot t 
(A) Let y = sin−1  =  − 
  1 − cosec t   1 − cosec2t 
1 + x 2 
 2tanθ  2cosec t cot t 2cosec t
= f ()
t = f ()
t
Again, let x = tanθ = sin−1   cot 2 t cot 2 t
 1 + tan2 θ 
2
= sin−1 (sin 2θ) = 2θ ⇒ y = 2tan−1 x = f ()
t = f ()(
t 4cosec 2t) = f ()
t g()
t
sin t cos t
dy 2
⇒ = (differentiating w.r. t. x) [given]
dx 1 + x 2 t = 4cosec 2t
So, g()
A → III
42. If x = 4 cos 3 θ and y = 3sin 2 θ,
−1  1− x −1
(B) Let tan   = y ⇒ tan () 1 − tan−1 (x) = y d2y π
1 + x  then at θ = , is
1 dx 2 4 [22 April 2019, Shift-I]
0− = y (differentiating w.r.t.x)
1 + x2 1 1 −1 −1
(a) (b) (c) (d)
−1 3 6 6 3
⇒ y=
1 + x2 Sol. (b)
B → II Given, x = 4 cos3 θ …(i)
(c) elog (sin x + cos x ) = sin x + cos x = y
y = sin x + cos x and y = 3sin θ
2
…(ii)
Differentiating w.r.t.x Differentiating Eq. (i) and Eq. (ii) w. r. t. t θ
dy dx
= cos x − sin x = 4 × 3cos2 θ(− sinθ)
dx dθ
C→ I
= −12cos2 θsinθ …(iii)
(D) Let y = 1 − sin 2x
dy
= sin2 x + cos2 x − 2sin x cos x and = 3 × 2sinθ cosθ = 6sinθ cosθ …(iv)

= (sin x − cos x)2 ⇒ y = sin x − cos x dy
Differentiating w.r. t. x, we get dy dθ
Now =
dy dx dx
= cos x + sin x
dx dθ
D → IV 1
= − secθ [by Eqs. (iii) and (iv)]
Hence, option (d) is correct. 2
Differentiation 347

d2y 1 dθ [7sin x + (7 x + 1) cos x ] (e x log x)


So, = − secθ tanθ.
dx 2 2 dx  ex 
−1 − (7 x + 1)sin x  e x log x +
=
1
− × secθ tanθ × [by Eq. (iii)]  x 
2 12cos2 θsinθ ⇒ f ′(x) = x 2
(e log x)
1 1 sinθ 1
= × × × (7sin x + 7 x cos x + cos x) log x
24 cosθ cosθ cos θsinθ 2

− (7 x + 1) sin x  log x + 
1
=
1
=
1 Q θ = π  


 x
24(cosθ)4 π
4
4 =
24 cos 
 e x (log x)2
 4
=
1
×
1
=
1
×4=
1 (7sin x + 7 x cos x + cos x) log x
24  1  4 24 6
− (7 x + 1)sin x  log x + 
1
 
 2 f ′(x)  x
∴ =
dy f (x) (7 x + 1)sin x log x
43. If x = sin h− 1[log(1 + y)], then =
dx  log x + 1 
 
[22 April 2019, Shift-II] 7sin x + 7 x cos x + cos x  x
∴ g(x) = −
(a) 2( y + y )sin hx (7 x + 1)sin x log x
(b) 2( y + y ) 1 − (log(1 + y ))2 7 1
= + cot x − 1 −
(c) 2( y + y )cos hx 7x + 1 x log x
(d) 2 ( y + y )log(1 + y) − 49  −1 
∴ g′(x) = − cosec2 x −   (log x + 1)
Sol. (c) (7 x + 1) 2
 (x log x)2 
Given, − 49 1 1
= − cosec2 x + 2 +
x = sin h− 1 [log(1 + y)] (7 x + 1)2 x (log x) (x log x)2
sin hx = log(1 + y) cos x 1 0
1 1 dy 1
cos hx = . 45. If f (x) = 1 2 cos x 1 , then
(1 + y) 2 y dx 9
0 1 2 cos x
dy
⇒ = 2(y + y) cos hx
dx d2 f
=
(7 x + 1)sin x dx 2 [22 April 2019, Shift-II]
44. If f (x) = and f ′ (x) = f (x) g(x),
e log x x (a) cos 3x (b) cos( π + 3x)
(c) sin3x (d) sin( π + 3x)
then g ′ (x) = [22 April 2019, Shift-II]
1 1 49 Sol. (b)
(a) + − cosec 2 x –
2
x log x ( xlog x) 2
(7 x + 1) 2 Given,
1 1 49 cos x 1 0
(b) 2 + − cosec x –
2
1
x log x log x (7 x + 1)2 f (x) = 1 2cos x 1
9
1 x 49 0 1 2cos x
(c) + − cosec 2 x –
( xlog x)2 log x (7 x + 1)2 1
= [cos x (4 cos2 x − 1) − 1(2cos x)]
1 1 49 9
(d) 2 + + cosec 2 x +
x log x ( xlog x) 2
(7 x + 1)2 1
= [4 cos3 x − cos x − 2cos x]
9
Sol. (a)
1
We have, = [4 cos3 x − 3cos x]
(7 x + 1)sin x 9
f (x) = 1
e x log x f (x) = cos 3x
9
348 AP EAMCET Chapterwise Mathematics

1 1 Sol. (d)
⇒ f ′(x) = (− sin 3x) 3 ⇒ f ′(x) = − sin 3x
9 3 1
Given, x 4 + y4 = t 2 + …(i)
1 t2
⇒ f ′′(x) = − (cos 3x) (3) = − cos 3x
3 1
x 2 + y2 = t −
d2 f t
⇒ f ′′(x) = cos(π + 3x) ⇒ 2 = cos(π + 3x)
dx On Squaring both sides, we get
1
1 − x2 x 4 + y 4 + 2x 2 y 2 = t 2 + 2 − 2
46. If cos ( f (x)) = and t
1 + x2 1 1
t 2 + 2 + 2x 2 y 2 = t 2 + 2 − 2 [Q from Eq. (i)]
3x − x3 df t t
tan (g(x)) = , then =
1 − 3x 2
dg x 2 y2 = − 1
−1
[23 April 2019, Shift-I] y2 = 2
3 1 + x2 + 2 x3 x
(a) (b) Differentiating w.r.t. x, we get
2 (1 − 3 x2 )2
2
2 x2 + x3 2yy′ = 3
(c) (d) x
3 (1 + x2 ) (1 − 3 x2 ) 1
⇒ y′ = 3
Sol. (c) x y
1 − x2 ∴Hence, answer is (d).
Given, cos ( f (x)) =
1 + x2 48. If x = sinθ and y = cos pθ, then (1 − x 2) y 2 =
Put x = tanθ [23 April 2019, Shift-I]
1 − tan2 θ (a) xy1 − p2 y (b) p2 y − xy1
cos( f (x)) =
1 + tan2 θ (c) xy1 (d) p2 y
cos ( f (x)) = cos 2θ
Sol. (a)
f (x) = 2 θ = 2tan−1 x
Given, x = sinθ and y = cos pθ
3x − x 3 dy
tan(g(x)) = ∴ = − p sin pθ
1 − 3x 2 dθ
Put x = tanθ and
dx
= cos θ
3tan θ − tan3 θ dθ
tan(g(x)) =
1 − 3 tan2 θ dy p sin pθ 1 − y2
⇒ =− =−p
tan(g(x)) = tan 3θ dx cosθ 1 − x2
∴ g(x) = 3θ = 3 tan−1 x dy
⇒ 1 − x2 = − p 1 − y2
f ′(x) 2 dx
∴ =
g′(x) 3 2
(1 − x 2)   = p 2 (1 − y 2)
dy

df 2  dx 
∴ =
dg 3 (On squaring both sides)
2
dy  d 2 y 
⇒ − 2x   + (1 − x 2) 2  2  = − p 2 2y
Hence, answer is (c). dy dy
1 1  dx  dx  dx  dx
47. If x 2 + y 2 = t − and x 4 + y 4 = t2 + ,
t t2 [On differentiating both sides]
dy dy d2y
then = ⇒ −x + (1 − x 2) 2 = − p 2 y
dx [23 April 2019, Shift-I] dx dx
(a)
2
(b)
2
(c)
1
(d)
1 ⇒ (1 − x 2) y2 = xy1 − p 2 y
3 3 3
x x y x x3 y Hence, option (a) is correct.
Differentiation 349
−1
49. The derivative of f (x) = x tan x
with respect Sol. (d)
We have,
 1 
to g(x) = sec −1  2  is x = 3cos t and y = 4sin t
 2 x − 1 [22 April 2018, Shift-I] x2 y2
1 −1 x  log x tan−1 x  ⇒ + = cos2 t + sin2 t
(a) 1 − x2 xtan  + 9 16
2 1 + x
2
x 
x2 y2
1 − 1 x ⇒ + =1
(b) − 1 − x2 xtan [log(tan−1 x) + x(1 + x2 )tan−1 x] 9 16
2 2x 2y dy
x  log x tan−1 x  ⇒ + =0
−2 tan−1  + 9 16 dx
1 + x
2
x  dy − 16 x
(c) ⇒ =
1 − x2 dx 9 y
1 −1  log x tan−1 x  1 ⋅ y − x dy 
(d) − 1 − x2 xtan x  +
2 1 + x
2
x  ⇒
d 2 y − 16 
= dx 
dx 2 9  y2 
Sol. (d)  
 
We have,
  − 16 x 
f (x) = xtan
−1
 y− x− 
y 
x
d y − 16
2
 9
⇒ =  
⇒ log f (x) = tan− 1 x log x dx 2
9  y2 
1 d 1 tan− 1 x  
∴ ⋅ f (x) = log x +
f (x) dx 1+ x 2
x − 16  9 y 2 + 16 x 2  − 16 144  x2 y2 
=   = 9 × 9 y3 Q 9 + 16 = 1
9  9 y3   
df (x) − 1  log x tan− 1 x 
⇒ = xtan x  +
dx 1 + x
2
x  − 16 × 144  3 2 
= Q (x 0 , y0) =  2 , 2 2 
 1  81 × (2 2)3  
Also, g(x) = sec− 1  2 
 2x − 1  − 16 × 144 − 144 − 16 − 8 2
= = = =
= cos− 1 (2x 2 − 1) = 2cos− 1 x 81 × 16 2 81 × 2 9 2 9
d −2 2  a−b x
∴ g(x) =
51. If y = tan −1  tan , then
dx 1 − x2
a −b  a+b 2
2 2
df (x)
df (x) d2y
∴ = dx
d ⋅ g(x) dg(x) = [22 April 2018, Shift-I]
dx
dx 2 x=
π
2
−1  log x tan− 1 x  b b 2b b2
xtan 1 + x 2 +
x
(a) (b) (c) (d)
 x  2a 2
a 2
a 2a
=
−2 Sol. (b)
1 − x2 We have,
−1 − 1  log x tan− 1 x  2  a−b x
= 1 − x 2 ⋅ xtan x  + y= tan− 1  tan 
2 1 + x
2
x  a2 − b2  a + b 2

d2y dy  2 1
50. If x = 3cos t and y = 4 sin t, then ⇒ = ⋅
2
at the dx  a 2 − b 2  a − b 2 x
dx 1+   tan
3   a + b 2
point (x 0 , y 0) =  2 , 2 2  , is
2  ⋅
a−b
⋅ sec2 ⋅ 
x 1
[22 April 2018, Shift-I] a+ b 2 2
4 2 4 2 8 2 8 2 sec2 x / 2 a+ b
(a) (b) − (c) (d) − =
9 9 9 9 a + b (a + b) + (a − b) tan2 x / 2
350 AP EAMCET Chapterwise Mathematics

sec2 x / 2 Now, put y = 0, then we are getting


=
x
a + b + a tan2 − b tan2 x / 2 1 = f (x). f (0) − f (0) − x + 2
f ()
2 ⇒ 1 = f (x) − 1 − x + 2
f () {Q f (0) = 1}
sec2 x / 2 On differentiating with respect to ‘x’
=
df (x)
a 1 + tan2  + b 1 − tan2 
x x 0= − 0 −1 + 0
 2  2 dx
 df (x)
sec2 x / 2  1 − tan2 x / 2  ⇒ =1
= a + b  dx
1 + tan2 x    1 + tan2 x / 2 
  df (x)
 2 So, (at x = e) = 1
dx
dy 1
=
dx a + b cos x 54. y = sin(log(x 2 + 2 x + 1))
d2y −1 d2y dy
∴ = ⋅ (− b sin x) ⇒ (x + 1)2 + (x + 1) =
dx 2 (a + b cos x)2 dx 2 dx
d2y b sin x [22 April 2018, Shift-II]
⇒ =
dx 2 (a + b cos x)2 (a) y (b) − 4y
(c) 4y (d) − y
d2y b sin π / 2 b
∴ = = Sol. (b)
dx 2 x =
π  a + b cos π 
2
a2
  Given, y = sin(log(x 2+ 2x + 1))
2
 2
⇒ y = sin[2log(x + 1)]
 1− x  2 
52. For − 1 < x < 1, if f (x) = cos  tan −1 2
, So,
dy
= [cos(2log(x + 1)] ×  
 1+ x dx  x + 1
then f ′ (x) = [22 April 2018, Shift-II]

dy
(x + 1)
= 2cos(2(log(x + 1))
1 −1 dx
(a) (b) 1 (c) − 1 (d)
2 2 On differentiating both side with respect to ‘x’
Sol. (a) d 2 y dy  2 
(x + 1) 2 + = − 2(sin(2log(1 + x)))  
 1− x dx dx  x + 1
Given function f (x) = cos2  tan1  , for
 1 + x d2y dy
⇒ (x + 1)2 + (x + 1) = − 4y
−1 < x < 1. dx 2 dx
Let x = cos 2θ
55. If x = sec θ − cos θ, y = sec n θ − cos n θ,
1 − x 1 − cos 2θ
Then, = = tan2 θ dy
1 + x 1 + cos 2θ then =
dx [23 April 2018, Shift-I]
1 + cos 2θ 1 + x
So, f (x) = cos2 θ = = y + 4
2
y2 + 4
2 2 (a) (b) n
1 x + 4
2
x2 + 4
So, f ′(x) = .
2 x2 + 4 x2 + 4
(c) (d) n
53. f : R → R is a function such that f (0) = 1 and y + 4
2
y2 + 4

for all x , y ∈ R f (xy + 1) = f (x) f (y) − f (y), Sol. (b)


df We have,
− x + 2, then at x = e is
dx x = sec θ − cos θ
[22 April 2018, Shift-II] differential w.r.t θ
(a) 0 (b) − 1 dx
= sec θ tan θ + sin θ
(c) e (d) 1 dθ
Sol. (d) sin θ
= sec θ . + sin θ
Given functional relation is, cos θ
f (xy + 1) = f (x) f (y) − f (y) − x + 2 = tan θ (sec θ + cos θ)
Differentiation 351

Now, y = secn θ − cosn θ a + 1 + 2x


 a + x
differential w.r.t ‘θ’ 57. If a > 0 and f (x) =   , then
1 + x 
dy
= n secn −1 θ. sec θ tanθ − n cosn − 1 θ (− sin θ) f '(0) = [23 April 2018, Shift-II]
dθ a+1
sin θ (a) a
= n secn θ + n cosn − 1 θ . sin θ 1 − a2 
cos θ (b) aa + 1  + 2log a
sin θ  a 
=n (secn θ + cosn θ)
cos θ (c) 2log a
= n tan θ (secn θ + cosn θ)  (1 + a)2 
(d) aa + 1  
Now,  a − 2 log a 
dy n tan θ (secn θ + cosn θ) Sol. (b)
=
dx tan θ (sec θ + cos θ) Given function, for a > 0
a + 1 + 2x
dy n(secn θ + cosn θ)  a + x
= f (x) =   ⇒ f (0) = a a+ 1
dx (sec θ + cosθ) 1 + x 
Squaring both side  a + x
2 ⇒ log f (x) = (a + 1 + 2x) log  
 dy  = n (sec θ + cos θ)
2 n n 2
  1 + x 
 dx  (sec θ + cos θ)2
On differentiating both side with respect to ‘x’,
2
 dy  = n (sec θ + cos θ + 2)
2 2n 2n we are getting
 
 dx  (sec θ + cos θ + 2)
2 2 f ′(x)  a + x  1 − 1 
= 2log   + (a + 1 + 2x) 
n2 (sec2n θ + cos2n θ − 2 + 4) f (x) 1 + x   a + x 1 + x 
=
(sec2 θ + cos2 θ − 2 + 4) So, f ′(0) = f (0)  2log(a) + (a + 1) − 1 
1
 a  
n [(sec θ − cos θ) + 4]
2 n n 2
= a+1
(secθ − cosθ)2 + 4 Q f (0) = a
2 1 − a 2 
 dy  = n (y + 4) So, f ′(0) = a a + 1 
2 2
+ 2log a .
 
 dx  (x 2 + 4)  a 
dy

dy y2 + 4
=n 2
58. If y = log 2(log 2 x) , then =
dx
dx x + 4
[23 April 2018, Shift-II]
56. If y = a sin x + (5 + 2 x)cos x , then y ′ ′ + y = log 2 e 1
(a) (b)
2 xloge x xloge x loge 2
[23 April 2018, Shift-I]
1 1
(a) 4cos x (b) − 4cos x (c) (d)
(c) 4sin x (d) − 4sin x loge (2 x) x log 2 e loge x

Sol. (d) Sol. (b)


y = a sin x + (5 + 2x) cos x Given, y = log 2(log 2 x)
Differential w.r.t ‘x’  log e x 
log e  
y′ = a cos x + (5 + 2x) (− sin x) + (0 + 2) cos x  log e 2 
⇒ y=
y′ = a cos x − (5 + 2x) sin x + 2 cos x log e 2
Again differential w.r.t ‘x’
On differentiating w.r.t. ‘x’, we are getting
y ′′ = − a sin x − [(5 + 2x) cos x
+ (0 + 2) sin x] + 2 (− sin x) dy 1 1
= ×
⇒ y ′′ = − a sin x − (5 + 2x) cos x − 2 sin x − 2 sin x dx log log e x x ⋅ log e 2
e2
⇒ y ′′ = − (a sin x + (5 + 2x) cos x) − 4 sin x log e 2
⇒ y ′′ = − y − 4 sin x dy 1
⇒ = .
⇒ y ′′ + y = − 4 sin x dx x ⋅ log e x log e 2
352 AP EAMCET Chapterwise Mathematics

 x  2 d2y Let U = xsin x , V = (sin x)x


59. If y = x log   for 0 < x < , then 2
2 − 3x  3 dx log U = sin x log x
1 1 dU sin x
at x = is ⋅ = + cos x log x
2 [23 April 2018, Shift-II] U dx x
= xsin x  + cos x log x 
(a) 4 (b) 16 dU sin x
(c) 32 (d) 2 dx  x 

Sol. (c) and V = (sin x)x


 x  2 log V = x log sin x
Given, y = x log   , for 0 < x < 1 dV 1
 2 − 3x  3 ⋅ = log sin x + x ⋅ cos x
V dx sin x
On differentiating w.r.t. to ‘x’, we are getting
dV
dy 1 −3   x  = (sin x)x [log sin x + x cot x]
= x −  + log   dx
dx  x 2 − 3x   2 − 3x  d sin x dU dV
⇒ [x + (sin x)x ] = +
dy 2  x  dx dx dx
⇒ = + log  
dx 2 − 3x  2 − 3x  = x sin x  sin x
+ cos x log x 
 x 
Again differentiating w.r.t. to ‘x’, we are getting
d2y −2 1 −3  + (sin x)x [log sin x + x cot x].
= (− 3) +  − 
dx 2 (2 − 3x)2  x 2 − 3x  There is no option match.
6 2 61. If x = a (t + sin t) and y = a (1 − cos t), then
= +
(2 − 3x)2 x(2 − 3x)
d2y
2
d y 1 6 2
So,  at x =  = + dx 2 [24 April 2018, Shift-I]
dx 2  2 (1 / 2)2 1 / 2 × 1
1 1
2 (a) (b)
4a sin   4a cos 4  
4 t t
= 24 + 8 = 32. 2 2
d
(c) 4a cosec 4   (d) 4a sec 4  
60. [ x sin x + (sin x)x ] = t t
dx [24 April 2018, Shift-I] 2 2

(a) xsin x  + cos x log x + (sin x) x


sin x
Sol. (b)
 x 
x = a(t + sin t) and y = a(1 − cos t)
[ x tan x + log (sin x)] dx dy
(b) xsin x [ x tan x + cos x log x] + (sin x) x = a(1 + cos t), = a sin t
dt dt
 sin x + log (sin x) dy dy / dt a sin t
 x  = =
dx dx / dt a(1 + cos t)
sin x  x 
 sin x + cos x log x + (sin x)
x
(c) x 2sin t / 2cos t / 2
  =
2cos2 t / 2
[ x cot x + log (sin x)]
dy t
= tan
(d) xsin x  x + (sin x) x
sin x dx 2
+ sin x log
 x  d2y 1 dt
= sec2 t / 2⋅ ⋅
[ x cot x + log (cos x)] dx 2 2 dx
Sol. (*) sec2
t
d sin x 2 1
Given, [x + (sin x)x ] = = .
dx 2a(1 + cos t) 4a cos4 (t / 2)
25
Applications of Derivatives
1. The values of x at the stationary points of Sol. (a)
n n
f (x) = x + 3 x − 2 are
3 2
[17 Sep. 2020, Shift-I]  x + y  =2
   
 31   1209 
(a) 0, 2 (b) 1, 2
(c) 0, − 2 (d) 1, 1 On differentiating
nx n −1 ny n −1 dy
Sol. (c) ⇒ + =0
31n 1209n dx
Let y = f (x) = x 3 + 3x 2 − 2
dy  nx n −1 × 1209n 
dy d2y ⇒ = − 
= 3x 2 + 6 x ⇒ 2 = 6 x + 6 dx  ny n −1 ⋅ 31n 
dx dx n −1
dy  x
⇒ = −  × (39n)
dy dx  y
For stationary points =0
dx Slope of tangent at (31, 1209)
⇒ 3x(x + 2) = 0 ⇒ x = 0, − 2 n −1
= −  
1
× (39)n = −39
2. The function f (x) = (1 / 2)x on R is  39 
[17 Sep. 2020, Shift-I] 4. If y = 5 x 2 + 6 x + 6 , x = 2, ∆x = 0.001, then
(a) Strictly decreasing
value of dy is [17 Sep. 2020, Shift-I]
(b) Strictly increasing
(a) 0.026 (b) 0.0026 (c) 0.062 (d) 0.0062
(c) Decreasing
(d) Neither increasing nor decreasing Sol. (a)
y = 5x 2 + 6 x + 6, x = 2
Sol. (a)
x ∆x = 0.001
Let y =   on R
1
 2 dy = (10 x + 6)dx
x x dy = (10 × 2 + 6) × 0.001
dy  1 
=   log = −   log 2
1 1
dy = 0.026
dx  2 2  2
 dy < 0 so strictly decreasing
2
  5. A man 2 m tall, walks at the rate of 1 m/s
 dx  3
1
3. The slope of the tangent of the curve towards a street light which is 5 m above
3
n n
 x  y  the ground. The rate at which the length of
  +  = 2 at (31, 1209) ………… 1
 31 1209  his shadow is changing when he is 3 m
[17 Sep. 2020, Shift-I] 3
(a) − 39 (b) 39 away from the base of the light is .......
1 −1 [17 Sep. 2020, Shift-II]
(c) (d)
39 39 (a) − 1m/s −1 (b) 2 m/s (c) − 2 m/s (d) 1 m/s
354 AP EAMCET Chapterwise Mathematics

Sol. (a) Sol. (b)


Given,
B
dN
= t 3,
dt
D
When, rate of growth is 1200 we have,
d  dN  d 3
  = 1200 ⇒ t = 1200
dt  dt  dt
A C E ⇒ 3t 2 = 1200 ⇒ t 2 = 400

AB = street light, C = Man. ⇒ t = 20 sec


From ∆ ABE and ∆ DCE, 8. The sides of an equilateral triangle are
AB AE AC + CE increasing at the rate of 2 cm s − 1 . How fast
= =
CD CE CE does its area increase when its side is 10 cm?
AB AC AC AB
⇒ = +1 ⇒ = −1 [17 Sep. 2020, Shift-II]
CD CE CE CD (a) 10 3 cm 2 , s − 1 (b) 5 3 cm 2 , s − 1
1
5 (c) 3 cm 2 , s − 1 (d) 2 3 cm 2 , s − 1
AC 5 3
⇒ = 3 −1 = ⇒ CE = AC
CE 2 3 5 Sol. (a)
Differentiating with respect to time we get,
CE =  AC 
d 3 d
…(i)
dt 5  dt 
d 2 d 5
None AC = −1 ms −1 ⇒ AC = − ms −1 60º
dt 3 dt 3 a
Negative sign as AC decreases with time. Area of an equilateral triangle is,
substituting in Eq. (i) we get. 3 a2
Rate of change of shadow length A=
4
=  −  = − 1 ms −1
3 5
Differentiating with respect to time we get,
5  3
dA 3 da
⇒ = × 2a ⋅
6. The equation of tangent to the curve dt 4 dt
y = 5 x 2 − 3 x + 7 at the point (− 1, 4) is ...... Here,
da −1
= 2 cm when a = 10 cm
[17 Sep. 2020, Shift-II] dt
(a) 13 x − y − 9 = 0 (b) 13 x + y − 9 = 0 So,
dA
=
3
× 2 × 10 × 2 = 10 3 cm 2.s −1
(c) 13 x + y + 9 = 0 (d) 13 x + 2 y + 5 = 0 dt a=10 cm 4
Sol. (c)
9. The curve 3 y 2 = 2ax 2 + 6 b passes through the
Curve is, y = 5x 2 − 3x + 7
point P(3 , − 1) and the gradient of the curve
Slope of tangent to curve at (−1, 4) is, at P is “− 1”, then the values of a and b are
dy
mT = = (10 x − 3)|x = −1 = − 13 [18 Sep. 2020, Shift-I]
dx ( −1 , 4) 1 −1
(a) a = , b = − 1 (b) a = ,b=1
Equation of tangent is, 2 2
y − y1 = m T (x − x1) 1 −1
(c) a = , b = 1 (d) a = ,b= −1
⇒ y − 4 = − 13(x + 1) 2 2
⇒ 13x + y + 9 = 0 Sol. (a)
3 Equation of given curve is
7. A kind of bacteria grows by t in t s. Time
taken for the rate of growth of the bacteria 3y 2 = 2ax 2 + 6b …(i)
to become 1200 per s is ......... Q Curve (i) passes through point P(3,−1), so
[17 Sep. 2020, Shift-II] 3 = 18a + 6b
(a) 10 s (b) 20 s (c) 40 s (d) 400 s ⇒ 6a + 2b = 1 …(ii)
Applications of Derivatives 355

dy dy
and the (3,−1) = −1 (given), so on = 8x + 2 …(i)
dx dx
differentiating the Eq. (i), we get y = x 3 − x + 13
dy differentiate w.r.t. ‘x’ on both sides,
6y = 4ax
dx dy
= 3x 2 − 1 …(ii)
⇒ 6 × (−1) × (−1) = 4a (3) ⇒ a = 1 / 2 dx
∴b = −1{from Eq. (ii) on putting value of ‘a’} Since, curves are touch each other
⇒ 8 x + 2 = 3x 2 − 1
10. The value of ‘k’ for which the function ⇒ 3x 2 − 8 x − 3 = 0
f (x) = k(x + sin x) + k is increasing, is equal ⇒ (x − 3)(3x + 1) = 0
to [18 Sep. 2020, Shift-I] −1
⇒ x = 3 (or)
(a) k < 0 (b) k > 0 3
(c) k = 0 (d) Data Insufficient Put, x = 3 in y = x 3 − x + 13
Sol. (b) y = 33 − 3 + 13
Given function f (x) = k(x + sin x) + k y = 37
So, f ′(x) = k(1 + cos x) ∴Point of contact P = (3,37)
Q f (x) is an increasing function, so (*) No option is correct.
f ′(x) ≥ 0
⇒ k(1 + cos x) ≥ 0 13. If the subnormal at any point on the curve
⇒ k> 0 {Q1 + cos x ≥ 0} y n = ax is constant then the value of n is
11. Find the equation of the normal to the curve [18 Sep. 2020, Shift-II]
(x − 7) (a) 1 (b) 2 (c) 3 (d) 0
y= at the point where it cuts
(x − 2)(x − 3) Sol. (b)
the X -axis. [18 Sep. 2020, Shift-I] Equation of given curve y n = ax, so
(a) 20 x + y + 140 = 0 (b) x − 20 y − 140 = 0 ny n−1
dy
=a ⇒
dy a 1 − n
= y
(c) x + 20 y + 140 = 0 (d) 20 x + y − 140 = 0 dx dx n
dy a
Sol. (d) ∴The length of subnormal = y = y y1 − n
(x − 7) dx n
Given curve, y = ,cuts the X −axis
(x − 2)(x − 3) = y
a 2− n
at point P(7,0). n
dy (x − 2)(x − 3) − (x − 7)[(x − 2) + (x − 3)] It is given that length of subnormal is constant,
Now, =
dx (x − 2)2(x − 3)2 means it must be independent of x and y terms,
so n = 2.
dy 5× 4 1
∴ = = Hence, option (b) is correct.
dx x =7 52 × 42 20
So, the slope of normal to given curve at point 14. If the radius of a sphere is mentioned as 7 m
P is −20. with an error of 0.02 m, then the
Therefore equation of required normal is approximate error in calculating its volume
y − 0 = −20(x − 7) ⇒ 20 x + y − 140 = 0 is [18 Sep. 2020, Shift-II]
(a) 1.83 πm 3 (b) 2.25 πm 3
12. The curves y = 4 x 2 + 2 x − 8 and (c) 4.39 πm 3 (d) 3.92 πm 3
y = x 3 − x + 13 touch each other at the point Sol. (d)
[18 Sep. 2020, Shift-II] Radius (r) = 7m
(a) (34, 3) (b) (3, 34) (c) (− 3, 34) (d) (− 34, 3) Error in radius (dr) = 0.02m
Sol. (*) 4
Volume of sphere (v) = πr 3
y = 4 x 2 + 2x − 8 3
differentiate w.r. to ‘r’ on both sides,
differentiate w.r.t. ‘x’ on both sides,
dv 4 π 2
dy
= 4(2x) + 21
() = (3r )
dx dr 3
356 AP EAMCET Chapterwise Mathematics

dv = 4 π(r 2). dr = 4 π(49 ) ⋅(0.02) A = x  − x 


p

2 
dv = 3. 92πm 3
dA p p
Hence, option (d) is correct. for maxima = 0 ⇒ − 2x = 0 ⇒ x = cm and
dx 2 4
p
15. The tangent at the point (1, 2) to the curve y = cm
4
y 2 = 4 x makes an angle θ with the positive ∴For given perimeter of rectangle p, the
direction of X -axis. Then θ = p p p2
maximum possible area A = × = cm2
[18 Sep. 2020, Shift-II] 4 4 16
(a) 60° (b) 30° (c) 90° (d) 45° Hence, option (b) is correct.
Sol. (d) 18. Find the value of ‘p ’ and ‘q ’ if the function
y 2 = 4 x ; P = (1, 2)
t = t3 − 6 t2 + pt + q defined on [1, 3]
f ()
differentiate w.r. to ‘x’
2 3 +1
2y ⋅ y′ = 4 ⋅1 ⇒ y′ =
4 satisfies the Rolle’s theorem for c =
2y 3
2 [21 Sep. 2020, Shift-II]
y′ =
y (a) p ∈ R,q = 11 (b) p = 11,q ∈ R
(c) p ∈ R ,q ∈ R (d) p = 11,q = 11
2
Slope of tangent = y′ =
P(1 , 2)
y P(1 , 2)
Sol. (b)
t = t 3 − 6t 2 + pt + q
It is given that, function f ()
tan θ = 1 ⇒ θ = 45°
defined on [1, 3] satisfied the Rolle’s theorem, so
Hence, option (d) is correct.
f (1) = f (3)
16. The interval in which y = ln (ln(x)), x > 1 is ⇒ 1 − 6 + p + q = 27 − 54 + 3p + q
decreasing is [21 Sep. 2020, Shift-I] ⇒ 2p = 22 ⇒ p = 11 and q ∈ R.
(a) (− ∞, 0) ∪ (2, ∞ ) (b) (0, 2 ) Hence, option (b) is correct.
(c) (0, 1) (d) (− 1, 0)
19. If the tangent to the curve x 2/ 3 + y 2/ 3 = a 2/ 3
Sol. (*) meets the X -axis at A and Y -axis at B, then
Given function y = l(ln(x)), x > 1 AB = [21 Sep. 2020, Shift-II]
dy 1
∴ = , x >1 (a) 2a (b) 3 a (c) a (d) 4 a
dx x ln x
Q y is a decreasing, then Sol. (c)
dy 1 Given curve is x 2/ 3 + y 2/ 3 = a 2/ 3 …(i)
< 0⇒ <0
dx x ln x Now, let a point on curve P(a cos3 θ, a sin3 θ)
Q x > 1 ⇒ ln x < 0 ⇒ x < 1, which is not possible as On differentiating the curve (i) w.r.t x, we get
x >1 2 −1 / 3 2 −1 / 3 dy
x + y =0
∴ x ∈ φ,for decreasing 3 3 dx
(*) No option is correct. 1/ 3
= −  
dy y

17. Maximum area of the rectangle that can be dx  x
formed with the fixed perimeter ‘p’ cm ......... ∴Slope of tangent at point P is
[21 Sep. 2020, Shift-I] dy
m= = − tanθ
p2 p2 dx P
(a) cm 2 (b) cm 2
8 16 ∴Equation of the tangent of the curve at point P
p2 p2 is
(c) cm 2 (d) cm 2
64 32 y − a sin3 θ = − tanθ(x − a cos3 θ) …(ii)
Sol. (b) Q The tangent (ii) meets the axes at A and B, so
A(a cosθ, 0) and B(0, a sinθ)
Let length of adjacent sides of rectangle is
x cm and y cm so perimeter of rectangle is ∴ AB = a 2 cos2 θ + a 2 sin2 θ = a
p = 2(x + y) cm and the area A = xy cm2 Hence, option (c) is correct.
Applications of Derivatives 357

20. The equation of the normal to the curve 23. What is an approximate value of 199
y = sin x at the point (0 , 0) is corrected to 4 decimal places?
[22 Sep. 2020, Shift-I] [22 Sep. 2020, Shift-I]
(a) x = 0 (b) y = 0 (a) 14.1608 (b) 14.0168
(c) x + y = 0 (d) x − y = 0 (c) 14.1086 (d) 14.1068
Sol. (c) Sol. (*)
Given curve y = sin x Let a function f (x) = x
∴ Slope of normal to the curve at point (0, 0)
and x = 196 and ∆x = 3
 dx   1 
=−  =−  = −1 ∆y =
dy
× ∆x
 dy  ( 0 , 0)  cos x  ( 0 , 0) Q
dx x = 196
∴ Equation of normal to the curve at point (0, 0) 1 3
is = × 3= = 0.1071
2 x x = 196 28
y − 0 = − 1(x − 0) ⇒ x + y = 0
∴ 199 = 14 + 01071
. = 14.1071
21. Displacement ‘s’ of a particle, in meters, at
24. A point on the curve x = 3 cos θ , y = 2 sin θ at
t3
any time ‘t’ s in expressed as s = − 6 t. Find which the tangent is perpendicular to X -axis is
3
[22 Sep. 2020, Shift-I]
the acceleration at a time when the velocity
vanishes, [22 Sep. 2020, Shift-I]
(a) (3, 0) (b) (0, 3)
(c) (0, − 3) (d) (−3, 3)
(a) 6 m⋅ s −2 (b) 2 6 m⋅ s −2
(c) 12 m⋅ s −2
(d) 6 6 m⋅ s −2
Sol. (a)
Given curve is x = 3 cos θ, y = 2 sin θ is represent
Sol. (b) x2 y2
ellipse, + = 1 and the tangent perpendicular
Given, displacement of a particle at time t as 9 4
t3 to X-axis can be drawn at end points of the major
s= − 6t
3 axis means either at (3, 0 )
ds or (− 3, 0 ).
∴Velocity = = t2 − 6 = v (let)
dt 25. If displacement s = 5sin(2t), then the velocity
and as velocity vanishes, so v = 0 ⇒ t = 6 sec
π
dv at the end of s is
Now, the acceleration a = = 2t 3 [22 Sep. 2020, Shift-II]
dt
(a) 5 (b) −5 3 (c) 5 3 (d) −5
∴Acceleration a at t = 6 sec. is 2 6 m/sec 2.
Sol. (d)
22. The radius of a circle is increasing at a rate of s = 5 sin (2t)
0.1 cm s −1 . Then the rate of change of area, differentiate w.r.t. ‘t’
when its radius is 5 cm, is ……… ds d
= 5 . sin (2t)
[22 Sep. 2020, Shift-I] dt dt
(a) π 2 cm2s −1 (b) π cm2s −1 ds
π = 5. cos (2t). 2 = 10.cos 2t
(c) 2 π cm2s −1 (d) cm2s −1 dt
2 π π
= 10.cos 2  
ds
Velocity when t = sec =
Sol. (b) 3 dt t = π / 3  3
Let the circle have radius r cm and area A cm 2 1
dr = 10 × −
=−5
and it is given that = 0.1 cm. s − 1 and as we 2
dt
dA dr Hence, option (d) is correct.
know area A = πr 2 ⇒ = 2πr
dt dt 26. What is the value of 3 26 corrected up to
∴Rate of change of area, when radius is 5 cm is three decimal places? [22 Sep. 2020, Shift-II]
dA
= 2π(5) (0.1) = π cm 2. s − 1 (a) 2.998 (b) 2.844
dt r = 5 cm (c) 2.962 (d) 2.823
358 AP EAMCET Chapterwise Mathematics

Sol. (c) ∴ P = (2, 2)


Let y = f (x) = x 1/ 3 Equation of normal is y − y1 = m(x − x1)
−1
x + ∆ x = 26 y − 2= (x − 2)
9
27 + ∆ x = 26 [Q let x = 27]
x + 9 y = 20
∆ x = −1
Case (ii) If x = − 2
We have, y = x1 / 3
y = (−2)3 − 3(–2) = − 2
Differentiate w.r.to ‘x’
1 ∴ Q = (−2, −2)
dy 1 3 − 1
= x Equation of Normal is y − y1 = m(x − x1)
dx 3 −1
2 y + 2= (x + 2)
1 − 9
dy = x 3 dx
3 x + 9 y = − 20
1 1 Required Equation of normal is x + 9 y = ± 20
dy = dx = (−1)
3. x 2/ 3 2
Hence, option (c) is correct.
3.(27) 3
−1 1 28. If the increase in the side of square is 6%,
∆y = =− ≈ − 0.037
3(9) 27 then the approximate percentage increase in
3 26 = y + ∆y = 3 + (−0.037) = 2. 962 its area …… [22 Sep. 2020, Shift-II]
Hence, option (c) is correct. (a) 36% (b) 12% (c) 3% (d) 4%

27. Find the equation of normal to the curve Sol. (b)


y = x 3 − 3 x , which is parallel to the line Let side of square = x
Given, percentage change in side = 6%
2 x + 18 y = 9 ? [22 Sep. 2020, Shift-II]
dx
(a) x + 9 y = 20 only (b) x + 9 y = 40 only × 100 = 6% … (i)
x
(c) x + 9 y = ± 20 (d) x + 9 y = ± 40
We have, A = x 2
Sol. (c)
Differentiate w.r.to ‘x’
Given, line is 2x + 18 y = 9
dA
−a = 2x ⇒ dA = 2x dx
slope (m) = dx
b
−2 1 Dividing by x 2 on both sides
m= =− dA x dx
18 9 2
= 2⋅ 2
x x
Given equation of curve is
dA dx
y = x 3 − 3x … (i) =2 [Q A = x 2]
A x
Differentiate w.r.t. ‘x’, we get Now, multiply by 100
dy
= 3x 2 − 3 × 100 = 2 ×  × 100
dA dx
dx A  x 
−1 −1
∴Slope of normal = = 2 Percentage change in Area = 2 × 6% [from Eq. ()]
i
dy 3x − 3
= 12%
dx
Hence, option (b) is correct.
Since, normal is parallel to given line
∴Slope of line = slope of normal 29. Tangent at any point θ on the curve
−1 −1 x = 35 sec θ , y = 35 tan θ is
=
9 3(x 2 − 1) [22 Sep. 2020, Shift-II]
⇒ x 2 −1 = 3 ⇒ x = ± 2 (a) y sinθ = x + 35 cos θ
(b) y sinθ = x − 35 cos θ
Case (i) if x = 2
(c) ycos θ = x − 35 sinθ
from Eq. (i) (d) ycos θ = x + 35 sinθ
y = (2)3 − 3(2) Sol. (b)
y=2 Given, x = 35secθ … (i)
Applications of Derivatives 359

y = 35 tan θ …(ii) differentiate w.r.t. ‘x’ on both sides,


∴ Point P = (35sec θ, 35 tanθ) 1 x 2 − 5x + 4) − (ax − b) (2x − 5)
a()(
Differentiate Eq. (i) w.r.to ‘θ’ y′ =
(x 2 − 5x + 4)2
dx
= 35(sec θ. tan θ) …(iii) Since, P(2, − 1) is a turning point then y′P( 2, −1) = 0

Differentiate Eq. (ii) w.r.to ‘θ’ a [22 − 5(2) + 4] − (2a − b) (2(2) − 5) = 0
dy
= 35 (sec2 θ) …(iv) −2a − (2a − b) (−1) = 0
dθ b=0
Eq. (iv) dy 35(sec2 θ) Substitute b = 0 in Eq. (i)
⇒ ⇒ =
Eq. (iii) dx 35 (sec θ . tan θ) 2a − 0 = 2
dy sec θ 1 a =1
= =
dx tan θ sin θ Hence, option (b) is correct.
1
∴ m= 31. If the curved surface area of right circular
sin θ cylinder inscribed in a sphere of radius
Equation of tangent is 22 cm is maximum then height of the cylinder
y − 35 tan θ = m (x − 35 sec θ) will be [23 Sep. 2020, Shift-I]
sinθ 1  1  11
y − 35 =  x − 35.  (a) cm (b) 11 2 cm
cos θ sin θ  cos θ  2
(c) (0.22) 2 m (d) (0.11) 2 m
sin2 θ 35
y sin θ − 35 = x− Sol. (c)
cos θ cos θ
We have,
35 sin2 θ 35
y sin θ = x + − The C.S.A of a right circular cylinder inscribed
cos θ cos θ in a sphere of radius ‘R’ is maximum when
35 (sin2 θ − 1) height is 2R
y sin θ = x +
cos θ Height = 2R= 2 (22 cm)
35 (− cos 2 θ) = 2 
22 
m = 0.22 2 m
y sin θ = x +
cos θ  100 
y sin θ = x − 35 cos θ Hence, option (c) is correct.
Hence, option (b) is correct. 32. The radius of a sphere increases at the rate of
(ax − b) 0.04 cm/sec. The rate of increase in the
30. If y = has a turning point
(x − 1) (x − 4) volume of that sphere with respect to its
P(2 , − 1), then the values of a and b are surface area, when its radius is 10 cm is
[20 April 2019, Shift-II]
[23 Sep. 2020, Shift-I]
(a) a = 0, b = 1 (b) a = 1, b = 0 (a) 16 π (b) 25
(c) a = −1, b = 0 (d) a = 0, b = − 1 (c) 20 (d) 5

Sol. (b) Sol. (d)


ax − b Let r be the radius of the sphere.
Given, curve is y = ; P = (2, − 1) dr
(x − 1) (x − 4) Given, rate of change in radius = 0.04 cm/sec
dt
Since, P(2, − 1) lies on the above curve. 4
a (2) − b Volume of sphere (V) = πr 3
−1 = 3
(2 − 1) (2 − 4) Differentiating w.r.t, t, we get
2a − b dV 4 dr
−1 = = π(3r 2) ⋅
−2 dt 3 dt
⇒ 2a − b = 2 …(i) dV 2 dr
= 4 πr ⋅ …(i)
ax − b ax − b dt dt
y= ⇒ y= 2
(x − 1) (x − 4) x − 5x + 4 Surface area of sphere (S) = 4 πr 2
360 AP EAMCET Chapterwise Mathematics

dS dr
= 4 π(2r) ⋅ 34. If f (x) = x 3 + px 2 + qx is defined on [0, 2]
dt dt
 1 
dS
= 8πr ⋅
dr
…(ii) such that f (0) = f (2) and f ′ 1 +  = 0, then
dt dt  3
Eq. (i) divided by Eq. (ii), we get p 2 + q2 = [20 April 2019, Shift-II]
dV dr
4 πr 2 ⋅ (a) 13 (b) 5 (c) 2 +
1
(d) 1
dt = dt 3
dS dr
8 πr ⋅
dt dt Sol. (a)
dV r 10
= = [Q r = 10 cm] Given, f (x) = x 3 + px 2 + qx is defined on [0, 2]
dS 2 2 f (0) = f (2)
dV
= 5cm 0 = 23 + p(2)2 + q(2)
dS ⇒ 4p + 2q + 8 = 0
Hence, option (d) is correct. ⇒ 2p + q + 4 = 0 …(i)
33. If f (x) = a log| x| + bx 2 + x has extreme d
Now f ′(x) = (x 3 + px 2 + qx)
values at x = − 1 and x = 2, then the ordered dx
pair (a , b) = [20 April 2019, Shift-II] f ′(x) = 3x 2 + 2px + q

Given, f ′ 1 +
1 
(b)  2, − 
1
(a) (2, − 1)  =0
 2  3
(d)  − , 2 
1 2
31 +
(c) (−1, 2 ) 1   1 
 2   + 2p 1 +  + q=0
 3  3
Sol. (b)
∴2p 1 +
1   1 2
 + q + 31 + +  =0
Given, f (x) = a ⋅ log|x| + bx 2 + x  3   3 3

2p 1 +
f ′(x) = a ⋅
1

x
+ b(2x) + 1 1  6
 + q+ 4+ =0 …(ii)
| x| | x|  3 3
ax Eq. (ii) Subtracting by Eq. (i), we get
f ′(x) = + 2bx + 1
|x|2 ⇒
2p
+
6
=0
Given that x = − 1 is one of extremity of f (x). 3 3
⇒ f ′(−1) = 0 ⇒ 2P + 6 = 0 ⇒ P = − 3
a(−1) Substituting P = − 3 in Eq. (i)
+ 2b (−1) + 1 = 0
1 2(−3) + q + 4 = 0
⇒ − a − 2b + 1 = 0 …(i) ⇒ q=2
Also, given x = 2is the other extremity of f (x). ∴ p 2 + q 2 = (−3)2 + (2)2 = 9 + 4
⇒ f ′(2) = 0 p 2 + q 2 = 13
a(2)
+ 2b (2) + 1 = 0 Hence, option (a) is correct.
4
a 35. The number of tangent to the curve
+ 4b + 1 = 0
2 y 2 (x − a) = x 2 (x + a)(a > 0) that are parallel to
a + 8b + 2 = 0 …(ii)
Adding on Eqs. (i) and (ii), we get the X -axis is [21 April 2019, Shift-I]
⇒ 6b + 3 = 0 (a) infinitely many (b) 0
1 (c) 1 (d) 2
⇒ b=−
2 Sol. (b)
1
Substituting b = − in Eq. (i), we get Given, curve is
2
y 2(x − a) = x 2(x + a)
a=2
Apply log on both sides
(a , b) =  2, − 
1

 2 log{y 2(x − a} = log{x 2(x + a)}
∴ Hence option (b) is correct. ⇒ 2log y + log(x − a) = 2log x + log(x + a)
Applications of Derivatives 361

Differentiating both sides w.r.t. x, we get Sol. (a)


2 dy 1 2 1 Given, f (x) = ax 3 + bx 2 + 11 x − 6 satisfies the
⇒ ⋅ + = +
y dx x−a x x+ a Rolle’s theorem in [1, 3]. So,
2 dy 2 1 1 1 = 0 and f(3) = 0
⇒ = + − f()
y dx x x+ a x−a ⇒ a + b + 5= 0 … (i)
2(x + a) (x − a) + x(x − a) − x(x + a) and 27a + 9b + 27 = 0
=
x(x + a) (x − a) ⇒ 9a + 3b + 9 = 0
2(x 2 − a 2) + x 2 − ax − x 2 − ax ⇒ 3a + b + 3 = 0 … (ii)
= From Eq. (i), we get a + b = − 5
x(x − a) (x + a)
2 dy 2x 2 − 2a 2 − 2ax 38. For a > 0, if the function
⇒ =
y dx x(x − a) (x + a) f (x) = 2 x 3 − 9 ax 2 + 12 a 2 x + 1 attains its
dy x 2 − a 2 − ax x+ a maximum value at p and minimum value at
⇒ = ⋅ x⋅
dx x(x − a) (x + a) x−a q such that p 2 = q, then a =
dy [21 April 2019, Shift-I]
At =0 1
dx (a) (b) 1 (c) 2 (d) 4
∴ x 2 − a 2 − ax = 0 2

Here, p = a 2 + 4a 2 > 0 Sol. (c)


Given,
So, it has two real roots.
f (x) = 2x 3 − 9ax 2 + 12a 2 x + 1 has maximum
36. If f (x) = (2k + 1) x − 3 − ke − x + 2e x is value at P and minimum value at q.
monotonically increasing for all x ∈ R, then So, f ′(p) = 0 and f ′(q) = 0
the least value of k is [21 April 2019, Shift-I] Now, f ′(x) = 6 x 2 − 18ax + 12a 2 has roots p and q
(a) 1 (b) 0 18a
1 ∴ p+ q= = 3a
(c) − (d) − 1 6
2
12a 2
and pq = = 2a 2
Sol. (b) 6
Given, on solving both equations, we get
f (x) = (2k + 1) x − 3 − ke − x + 2e x p = a and q = 2a
Since, f (x) is monotonically increasing for all Given, p2 = q
x ∈ R. ∴ a 2 = 2a
So, f ′(x) ≥ 0
⇒ a 2 − 2a = 0
(2k + 1) + ke − x + 2e x ≥ 0
⇒ a(a − 2) = 0
⇒ e − x ((2k + 1)e x + k + 2e 2x ) ≥ 0
⇒ a=2 (Q a ≠ 0)
or (2k + 1)e x + k + 2e 2x ≥ 0
⇒ 2e x (e x + k) + 1(e x + k) ≥ 0 39. If 2 y = 3 x − 1 is a tangent drawn to the curve
⇒ (2e x + 1) (e x + k) ≥ 0 y 2 = ax 3 + b at (1, 1) where a , b are constants,
⇒ e x + k ≥ 0 or k ≥ 0 then (a , b) = [21 April 2019, Shift-II]
(a) (1, 0) (b) (0, 1) (c) (1, − 1) (d) (−1, 1)
Hence, least value of k is zero.
Sol. (a)
37. If the function f (x) = ax 3 + bx 2 + 11x − 6
Given that 2y = 3x − 1 is tangent to y 2 = ax 3 + b
satisfies the conditions of Rolle’s theorem in 3
 1  at point (1, 1) with slope .
[1, 3] and f ′ 2 +  = 0, then a + b = 2
 3 Differentiate y 2 = ax 3 + b w.r.t. x, we get
[21 April 2019, Shift-I]
dy dy 3ax 2
(a) − 5 (b) − 3 (c) 4 (d) 7 2y = 3ax 2 ⇒ =
dx dx 2y
362 AP EAMCET Chapterwise Mathematics

 dy  1 2 3a
3a() ⇒ 25y 2 = 225 ⇒ y 2 = 9
⇒   = =
 dx  (1 , 1) 21
() 2 ⇒ y=3 (Q y ≥ 0)
Since, slope   =
dy 3
41. Suppose f ′ ′(x) exists for all real x. If f (2) = 2,
 dx  2
3a 3
f (3) = 5 and f (4) = 10 , then which one among
∴ = ⇒ a =1 the following statements is definitely true?
2 2
[21 April 2019, Shift-II]
Q Point (1, 1) lies on y 2 = ax 3 + b
(a) f ′ ′( x) < 1for some x ∈(2, 4)
∴ 1 2 = ()()
() 1 13 + b (b) f ′ ′( x) > 1for some x ∈(2, 4)
⇒ 1 =1 + b ⇒ b = 0 (c) f ′ ′( x) = 1for some x ∈(2, 4)
∴ Point (a , b) are (1, 0). (d) f ′ ′( x) = 0 for some x ∈(2, 4)
Sol. (b)
40. A ladder of 5 meters long rests against a
vertical wall with the lower end on the Let f (x) = ax 2 + bx + c
horizontal ground. ∴ f(2) ⇒ 4a + 2b + c = 2 …(i)
The lower end of the ladder is pulled along f(3) ⇒ 9a + 3b + c = 5 …(ii)
the ground away from the wall at the rate and f(4) ⇒16a + 4 b + c = 10 …(iii)
3 m/sec. The height of the upper end (in Subtracting Eq. (i) from Eq. (ii), we get
meters) while it is descending at the rate of 5a + b = 3 …(iv)
4 m/sec, is [21 April 2019, Shift-II] Subtracting Eq. (ii) from Eq. (iii), we get
(a) 1 (b) 2 (c) 3 (d) 4 7a + b = 5 …(v)
Sol. (c) Subtracting Eq. (iv) from Eq. (v), we get
2a = 2 ⇒ a = 1
A Put a = 1 in Eq. (iv), we get
5×1 + b = 3
5m ⇒ b = 3− 5= − 2
y Put the value of a and b in Eq. (i),
4−4+ c=2
O x B ⇒ c=2
∴ f (x) = x 2 − 2x + 2
Given, length of ladder (AB) = 5 m Now, f ′(x) = 2x − 2
In ∆OAP, Again, f ′ ′(x) = 2
x 2 + y 2 = 52 …(i) Since, f ′ ′(x) > 1 for some x ∈(2, 4).
∴ Option (b) is correct.
dx dy
and = 3m/sec, = − 4 m/s …(ii)
dt dt 42. If p and q are respectively the global
(since y is decreasing) maximum and global minimum of the
Differentiating Eq. (i) w.r.t. x, we get function f (x) = x 2 e 2 x on the interval [ −2 , 2],
dx dy
2x + 2y = 0 then pe −4 + qe 4 = [21 April 2019, Shift-II]
dt dt
dx dy (a) 0 (b) 4e 8
⇒ x + y =0
dt dt (c) 4 (d) 4e 8 + 1
⇒ 3x − 4 y = 0 [by Eq. (ii)] Sol. (c)
⇒ 3x = 4 y Given, f (x) = x 2e 2x
⇒ 3 25 − y = 4 y
2
Differentiating w.r.t. x, we get
By squaring both sides, we get f ′(x) = 2e 2x x 2 + 2xe 2x = 2e 2x (x 2 + x)
9(25 − y 2) = 16 y 2 ⇒ f ′(x) = 0 ⇒ 2e 2x (x 2 + x) = 0
⇒ 225 − 9 y = 16 y
2 2
⇒ e 2x = 0 or x 2 + x = 0 ⇒ x = 0, − 1
Applications of Derivatives 363

So, maxima and minima can be attain at either B


of −2, −1, 0, 2 as the function bound.
∴ f (−2) = (−2)2 e −4 = 4e −4
f (−1) = (−1)2 e −2 Q
9m
f(0) = 0
f (2) = (2)2 e 4 = 4e 4 2m

∴p = 4e and q = 0
4
C
Therefore, pe −4 + qe 4 = 4e 4 (e −4) + 0 = 4e 0 A x P y
= 4 ×1 = 4 PC PQ
∴ =
AC AB
43. Tangents are drawn to the curve y = sin x y 2
from the origin. The locus of the points of ⇒ =
x+y 9
contact is [22 April 2019, Shift-I]
⇒ 9 y = 2x + 2y
(a) xy = x + y (b) x2 y2 = x2 − y2
⇒ 7 y = 2x
(c) xy = x − y (d) x y = x + y
2 2 2 2
7
⇒ x= y
Sol. (b) 2
Given, y = sin x dx 7 dy
⇒ = [differentiating w.r.t. t]
Differentiating w.r.t.x, we get dt 2 dt
dx
dy
= cos x But = 7 m/min
dx dt
7 dy
If tangent to y =sin x meet at (h, k) ∴ 7=
 dy  2 dt
  = cos h dy
 dx  ( h ,k) ⇒ = 2 m/min
dt
∴Equation of tangent is cos h(x − h) = y − k
∴Length of shadow is increases at 2 m/min.
Since, tangent is passing through (origin).
∴ h cos h = k 45. The maximum volume (in cubic units) of the
But given curve is passing through (h, k) cylinder which can be inscribed in a sphere
of diameter 6 units is [22 April 2019, Shift-I]
k = sin h ⇒ cos h = 1 − k2
(a) 12 3π (b) 4 3π (c) 3 3π (d) 8 3π
k2 k2
⇒ cos h = 1 − k = 2 ⇒1 − 2 = k2
2 2
Sol. (a)
h h
If h is the height and V be the volume, required
⇒ h2 − k2 = h2k2 cylinder then from the given figure.
Hence, locus of point of contact is x 2 − y 2 = x 2 y 2
D C
44. A man of height 2 m walks at a uniform L
speed of 7 m/min away from a lamp post of
height 9 m. The rate (m/min) at which the O
length of his shadow increases is h/2
[22 April 2019, Shift-I]
(a) 2 (b)
5
(c) 3 (d)
7 θ M
2 2 A B
Sol. (a) In ∆OAM,
2
r 2 +   =(3)2
Let AB be the lamp-post and PQ the man, CP be h
his shadow at time t.  2
Let AP =`x, PC = y, Also AB = 9 m, PQ = 2 m
h2 h2
Now, ∆CAB and ∆CPQ are equiangular and hence ⇒ r2 + = 9 ⇒ r2 = 9 −
4 4
similar.
364 AP EAMCET Chapterwise Mathematics

 h2  47. The function f (x) = x 1/ x for x > 0, is


Now, volume = πr 2h = π  9 −  h
 4 [22 April 2019, Shift-II]
 h3  dV  3h2  (a) increasing in (1, ∞ )
V = π  9h −  ⇒ =9 − π
 4 dh  4  (b) decreasing in (1, ∞ )
dV (c) increasing in (1, e ) and decreasing in (e, ∞ )
For maximum or minimum, =0 (d) decreasing in (1, e ) and increasing in (e, ∞ )
dh
3h2 3h2 Sol. (c)
⇒ 9− =0⇒ =9 1
4 4 Given, f (x) = y = x1 / x ⇒ log y = log x
⇒ h2 = 12 ⇒ h = ± 2 3 x
Differentiate w.r.t. x, we get
∴ r2 = 9 − 3⇒ r2 = 6 1 − log x 
= y
1 dy 1 log x dy
= 2 − ⇒
 d 2V 3h  y dx x x 2
dx  x 2 

The maximum value, Q 2 = − < 0
2
  d  x 
dh 1
1 − log x 
⇒ x = x1 / x 
= π(6)(2 3) = 12 3π sq units dx    x 2 

46. The angle between the curves y 2 = 8(x + 4) It is clear that, it is increasing in (1, e) and
and y 2 = 24 (4 − x) is decreasing in (e , ∞).
[22 April 2019, Shift-II]
 1
(a) tan  
− 1
1 48. Let f : 0 ,  → R be given by
(b) tan− 1(3)
 6  2
π π f (x) = x (x − 1) (x − 2). The value ‘c’, when
(c) (d)
2 4 Lagrange’s mean-value theorem is applied
Sol. (c) for f (x), is [22 April 2019, Shift-II]
Given, curves are 21 1
(a) (b)
y 2 = 8(x + 4) …(i) 6 6
21 21
and y = 24(4 − x)
2
…(ii) (c) 1 − (d) 1 ±
6 6
By Eqs. (i) and (ii), we get
Sol. (d)
8(x + 4) = 24(4 − x)
Given, f (x) = x(x − 1) (x − 2)
x + 4 = 12 − 3x
f (x) = x 3 − 3x 2 + 2x ⇒
4x = 8 ⇒ x = 2
∴ y 2 = 8(6) ⇒ y = ± 4 3 f ′(x) = 3x 2 − 6 x + 2
1−6+ 8 3
Now, f   = − + 1 =
1 1 3
For Eq. (i), 2y
dy
= 8 ⇒ 2ym1 = 8 =
 2 8 4 8 8
dx
8 4 4 1 f(0) = 0
m1 = ⇒ m1 = = = and
2y y 4 3 3 For Lagrange’s theorem,
dy
= − 24 ⇒ y m2 = − 12 f (b) − f (a)
For Eq. (ii), 2y f ′(c) =
dx b−a
− 12
m2 = =− 3 ⇒ 3c − 6c + 2 =
2 3/ 8
4 3 1/ 2
1
Here, m1 m2 = × − 3= −1 3
3 ⇒ 3c − 6c + 2 =
2

 m − m2  4
So, angle between the curves tanθ =  1  ⇒ 12c 2 − 24c + 8 = 3
 1 + m1 m2 
1
− (− 3) ⇒ 12c 2 − 24c + 5 = 0
⇒ tanθ = 3 ⇒ tanθ = ∞ 24 ± 576 − 240 24 ± 336
1 + (−1) ⇒ c= =
24 24
π π
⇒ tanθ = tan ⇒θ = 16 × 21 4 21 21
2 2 =1 ± =1 ± =1 ±
24 24 6
Applications of Derivatives 365

49. If T is the length of the subtangent drawn at So, according to Lagranage’s mean value
theorem
any point on the curve 3 y = 4 x and N is the
2 3
f (7) − f (2)
length of the subnormal at the same point, = f ′(x) ≤ 5 (given)
7−2
then (3T)2 = [23 April 2019, Shift-I] ⇒ f (7) − f (2) ≤ 25 ⇒ f(7) ≤ 28
(a) 4 N2 (b) 4 N (c) 2 N (d) 8 N2 Hence, option (d) is correct.
Sol. (c) 52. In the interval [ −2, 4], the absolute
The equation of given curve is maximum of f (x) = 2 x 3 − 3 x 2 − 12 x + 5
3y 2 = 4 x 3 …(i)
occurs x = [23 April 2019, Shift-I]
Let a point P(h, k) on the curve (i), so (a) 4 (b) − 2 (c) − 1 (d) 2
dy 2h2
= Sol. (a)
dx ( h , k) k
Given, function f (x) = 2x 3 − 3x 2 − 12x + 5
k 1 k2
Now, length of sub-tangent, T = k 2 = So, f ′(x) = 6 x 2 − 6 x − 12 = 0
2h 2 h2 [for maxima and minima f ′(x) = 0]
2h2 ⇒ x 2 − x − 2 = 0 ⇒ (x + 1) (x − 2) = 0
and length of sub-normal, N = k = 2h2
k ⇒ x = − 1, 2 ∈ [−2, 4]
9 k4 16h6 Q f(−2) = − 16 − 12 + 24 + 5 = 1
Q (3T)2 = = = 4h2 = 2N
4 h4 4h4 f(−1) = − 2 − 3 + 12 + 5 = 12
Hence, option (c) is correct. f(2) = 16 − 12 − 24 + 5 = − 15
and f(4) = 128 − 48 − 48 + 5 = 37
50. The interval in which the function The absolute maximum of f (x) occurs at x = 4.
log (7 + x)
f (x) = (x > 0) decreases is Hence, option (a) is correct.
log (3 + x)
[23 April 2019, Shift-I] 53. If f (x) = x 3 + ax 2 + bx + 5sin 2 x is an
increasing function on R, then
(a)  0,  (b)  0, 
7 3
 3  7 [22 April 2018, Shift-I]
(c) (0, 1) (d) (0, ∞ ) (a) a2 − 3b − 15 < 0 (b) a2 − 3b + 15 > 0
Sol. (d) (c) a2 − 3b − 15 > 0 (d) a2 + 3b + 15 > 0
log (7 + x) Sol. (*)
If the given function f (x) = , x > 0 is
log (3 + x) Q f ′(x) > 0
decreasing function then f ′(x) < 0 ⇒ 3x 2 + 2ax + b + 10sin x cos x > 0
1 1
log (3 + x) − log (7 + x) ⇒ 3x 2 + 2ax + b + 5sin 2x > 0
(7 + x) (3 + x)
⇒ <0 ⇒ 3x 2 + 2ax + b − 5 > 0
[log (3 + x)]2
⇒ (3 + x)( 3 + x ) < (7 + x)( 7 + x)
, it is true for ⇒ 3x 2 + 2ax + (b − 5) > 0
every value of x > 0. Here, a > 0, a < 0
⇒ x ∈ (0, ∞) ∴ (2a)2 − 4 × 3 × (b − 5) < 0
Hence, option (d) is correct. 4a 2 − 12(b − 5) < 0
a 2 − 3(b − 5) < 0
51. Let f be a polynomial function defined on
[2 , 7 ]. If f (2) = 3 and f ′ (x) ≤ 5 for all x in (2 , 7), a 2 − 3b + 15 < 0.
then the maximum possible value attained
by f at x = 7 is [23 April 2019, Shift-I]
54. The approximate value of cos31° is
(Take 1° = 0.0174) [22 April 2018, Shift-I]
(a) 7 (b) 14 (c) 18 (d) 28
(a) 0.7521 (b) 0.866 (c) 0.7146 (d) 0.8573
Sol. (d)
Sol. (d)
Since, the polynomial function are continuous
and differentiable in interval R. Let y = cos(x), x = 30° and x + ∆x = 31°
366 AP EAMCET Chapterwise Mathematics

π
= cos  =
3 56. The constant ‘c’ of Lagrange’s mean value
Let us find (y) π
x =  6 2 2x + 3
6 theorem for the function f (x) =
Then, ∆x = 1° = 0 ⋅ 0174 radian. 4 x −1
Consider the given function,
defined on [1, 2] is [22 April 2018, Shift-I]

y = f (x) = cos(x) 1 + 15 1 + 21
(a) (b)
Differentiating w.r.t. x, 3 4
5 3
dy (c) (d)
= − sin(x) 3 2
dx
π Sol. (b)
⇒   = − sin ⇒  
dy dy 1
=−
 dx  x = π 6  dx  x =
π 2 Using mean value theorem, we have
6 6 f (2) − f ()
1
Let ∆y be the change in y due to the change ∆x in f ′(c) =
x. 2−1
dy 5
∴ ∆y = × ∆x 1−
− 14 3 ⇒ − 14 2
dx ⇒ = =−
=  −  × 0 ⋅ 0174
1 (4c − 1)2 1 (4c − 1)2 3
 2
⇒ (4c − 1)2 = 21 ⇒ (4c − 1) = ± 21
= (− 0 ⋅ 5) × 0 ⋅ 0174 ≈ − 0 ⋅ 0087
∴ f (36°) = y + ∆y ⇒ 4c = 1 + 21 or 4c = 1 − 21
3 1+ 21  1 − 21 
= − 0 ⋅ 0087 ⇒ c= Q c = ∉[1, 2]
2 4  4 
1 ⋅ 732
= − 0 ⋅ 0087
2 57. The acute angle between the tangents drawn
= 0 ⋅ 8660 − 0 ⋅ 0087 at the point of intersection (other than the
= 0 ⋅ 8573 origin) of the curves x 2 = 4 y and y 2 = 4 x is
[22 April 2018, Shift-II]
55. If x and y are two positive numbers such
(a) tan   (b) sin−1  
−1 1 3
that x + y =32, then the minimum value of 2  5
x 2 + y 2 is,
(c) cos −1   (d) tan−1  
[22 April 2018, Shift-I] 1 2
(a) 500 (b) 256 (c) 1024 (d) 512  3  3
Sol. (d) Sol. (b)
Let, s = x 2 + y2 Given curves, x2 = 4y …(i)
= x + (32 − x)
2 2
[Q x + y = 32] and y = 4x
2
…(ii)
ds
∴ = 2x + 2(32 − x) (− 1) Point of intersection other then origin is (4, 4).
dx
Now, slope of fangents to curve (i) at (4, 4) is
= 2x − 2(32 − x)
dy 2x
= 2x − 64 + 2x m1 = = =2
dx ( 4 , 4) 4 ( 4 , 4)
= 4 x − 64
For maxima or minima, and slope of tangent to curve (ii) at (4, 4) is
ds dy 4 1
=0 m2 = ( 4 ,4) = =
dx dx 2y ( 4 , 4) 2
⇒ 4 x − 64 = 0 ⇒ x = 16 So, angle (θ) between the tangents is
and y = 32 − x = 32 − 16 = 16
1
2−
d 2s m1 − m2 2 = 3/ 2 = 3
Again, = 4 = + ive tanθ = =
dx 1 + m1 m2 1 + 2 1  2 4
 
∴ s is minimum when x = y = 16  2
∴ Minimum value of θ = tan−1   = sin−1  
3 3
So,
s = 162 + 162 = 256 + 256 = 512.  4  5
Applications of Derivatives 367

x Sol. (b)
58. If x > 0, then − log (1 + x)
1+ x Let V > C the speed of motor boat, relative to
[22 April 2018, Shift-II] water. In fact, relative to land, the motor boat
moves with the speed V − C.
(a) is less than zero S
(b) is greater than zero The motor boat covers a distance S in h
V−C
(c) is equal to zero
(d) takes all the real values and consumption is
S
Sol. (a) f (v) = kv3. , where k is a constant.
x V− C
Let a function f (x) = − log(1 + x), for x > 0
1+ x The minimum of consumption is attained for
(x + 1) − x 1 f ′(v) = 0
Q f ′(x) = − (v − C)3v2 − v3() 1
(1 + x)2 (1 + x) ⇒ kS =0
1 1 1 − (1 + x) (v − C)2
= − =
(1 + x)2 1 + x (1 + x)2 ⇒ 3v3 − 3v2C − v3 = 0
−x ⇒ 2v3 − 3v2C = 0 ⇒ v =
3C
= < 0, for x > 0 .
(1 + x)2 2
Q f ′(x) < 0, ∀ x > 0 61. The area (in sq. units) of the triangle formed
∴ f (x) is a strictly decreasing function for x < 0. by the tangent and the normal at the point
Now, f(0+ ) = 0  a b x 2 y2
 ,  to the curve 2 + 2 = 1 and the
So, f (x) < 0.  2 2 a b
59. On the curve y = x 3, the point at which the X -axis is [23 April 2018, Shift-I]
a 2
tangent line is parallel to the chord joining (a) (a + b 2 ) (b) 4ab
b
the points(−1, − 1) and (2, 8) is b
(c) (a2 + b 2 ) (d) 2ab
[22 April 2018, Shift-II] 4a
(a) (1, − 1) (b) (2, 8) (c) (1, 1) (d) (3, 27) Sol. (c)
Sol. (c) x2 y2
Given curve, 2
+ 2 =1
According to Lagrange’s Mean value theorem, a b
Q y = x 3 is a continuous and differentiable Differentiate w.r.t x, we get
function 2x 2y ⋅ y′ − b2x
⇒ + = 0 ⇒ y′ =
So, there exist x = c, such that a 2
b 2
ya 2
f (2) − f (−1)
− b 2  
f ′(c) = a
2 − (−1)  a b   2 b
At  ,  , y′ = b =−
8 − (−1)  2 2 2 a
⇒ 3c =2
=3 (a )
3 2
⇒ c2 = 1 b
So, slope of tangent, m1 = −
a
⇒ c = ±1
a
So, point is (1, 1). and slope of normal, m2 =
b
Equation of tangent at 
60. If the petrol burnt in driving a motor boat a b 
,  is
varies as the cube of the velocity, then the  2 2
speed (in km/hour) of the boat going against y − b  = − b x − a 
   
a water flow of C kms/hour so that the  2 a  2
quantity of petrol burnt is minimum is
And equation of normal at 
a b 
,  is
[22 April 2018, Shift-II]  2 2
2C 3C 4C 3C  y − b  = a  x − a .
(a) (b) (c) (d)    
3 2 3 4  2 b  2
368 AP EAMCET Chapterwise Mathematics

Now, Given function is algebraic function so. it is


a , b continuous and difference in [0, 4]
√2 √2 Now,
2a ,
0 f(4) = (4)3 − 6(4)2 + 11 (4) − 6
√2
= 64 − 96 + 44 − 6 = 6
f(0) = − 6
Now, f ′(x) = 3x 2 − 12x + 11
a − b2 ,0
√2 √2a by Lagrange’s mean value theorem
f (4) − f (0)
1
Area of ∆ = × base × height f ′(c) =
2 4−0
1  a2 + b2  6 − (− 6)
= 
b b 2 ⇒ 3c 2 − 12c + 11 =
 × = (a + b 2). 4
2 2a  2 4a
12
⇒ 3c 2 − 12c + 11 =
62. The interval in which the function 4
f (x) = 2 x 2 − log x , for x > 0 decreases, is ⇒ 3c 2 − 12c + 11 − 3 = 0
[23 April 2018, Shift-I] ⇒ 3c 2 − 12c + 8 = 0
(b)  0,
1
(c)  , ∞  (d)  0, 
1 1
(a) (2, 4)  12 ± (−12)2 − 4 (3)(8)
 4 2   2 ⇒ c=
2× 3
Sol. (d) 12 ± 144 − 96 12 ± 4 3
We have, = =
6 6
f (x) = 2x 2 − log x, x > 0 2 3
c = 2± ∈ [0, 4)
Differentiate w.r.t ‘x’ 3
1 4x2 − 1
f ′(x) = 4 x − = 64. The curve f (x) = e x sin x is defined in the
x x
Now, f ′(x) = 0 ⇒ 4 x 2 − 1 = 0 interval [0 , 2π]. The value of x for which the
slope of the tangent drawn to the curve at x
1
4x2 = 1 ⇒ x = ± is maximum, is [23 April 2018, Shift-I]
2 π π π π
+ – + (a) (b) (c) (d)
4 2 6 3
–1/2 1/2
Sol. (b)
for decreasing
We have,
f ′(x) < 0
f (x) = e x . sin x
− 1 1
x ∈  ,  but x > 0
 2 2 difference w.r.t ‘x’

So, x ∈  0, 
1 f ′(x) = e x cos x + e x sin x
 2 For f ′(x) is maximum we find f ′′(x)
63. If f (x) = (x − 1) (x − 2) (x − 3) for x ∈[0 , 4], f ′′(x) = − e x sin x + e x cos x + e x cos x
then the value of c ∈(0 , 4) satisfying + e x sin x = 2e x cos x
Lagrange’s mean value theorem, is ∴ f ′(x) is maximum at f ′′(x) = 0
[23 April 2018, Shift-I] Now, f ′′(x) = 0
2 2 3 3 3
(a) 3 ± (b) 2 ± (c) 2 ± (d) 3 ± ∴ 2e cos x = 0
x
3 3 2 3
π
Sol. (b) ⇒ x=
2
We have, So, slope of tangent drawn to curve is maximum
f (x) = (x − 1) (x − 2) (x − 3) π
at x = .
f (x) = x 3 − 6 x 2 + 11 x − 6 2
Applications of Derivatives 369

65. An approximate value of 4 18 is Now, according to Lagrange’s mean value


π π
[23 April 2018, Shift-II] theorem, there exist c ∈  − ,  , such that
 2 2
π π
f   − f  − 
(a) 2.0512 (b) 2.0425 (c) 2.0625 (d) 2.0834
Sol. (c)  2  2
f ′(c) =
Let y = f (x) = x1 / 4 and x = 16 and ∆x = 2 π  π
− − 
2  2
So, ∆y =   ∆x = (x − 3/ 4)∆x
dy 1
 dx  4 ⇒ − sin c − 2cos(2c) = 0 ⇒sin c + 2 − 4sin2 c = 0
at x = 16 and ∆x = 2 ⇒ 4sin2 c − sin c − 2 = 0
1 1 1 1 ± 1 + 32
∆y = × × 2 = and y = (16)1 / 4 = 2 So, sin c =
4 8 16 8
1 1±  1 ± 33 
So, y + ∆y = 2 + = 2. 0625 ⇒ sinc =
33
⇒ c = sin− 1 
16 
8  8 
66. The sum of the maximum and the minimum
values of 3 x 4 − 2 x 3 − 6 x 2 + 6 x + 4 , in (0 , 2) is 68. Air is discharging from a large spherical
balloon at the rate of 4 cubic meters per
[23 April 2018, Shift-II]
167 minute. Then, the rate at which the surface
(a) 28 (b) area is shrinking when the radius of the
16
134 87 balloon is 8 m, is [23 April 2018, Shift-II]
(c) (d)
15 16 (a) 2m 2 /min (b) 1m 2 /min
(c) 4m 2 /min (d) 8m 2 /min
Sol. (b)
Let a function f (x) = 3x 4 − 2x 3 − 6 x 2 + 6 x + 4 Sol. (b)
So, f ′(x) = 12x − 6 x − 12x + 6
3 2 Let air is discharging from a large spherical
dv
= 6(x − 1) (x + 1) (x − 1 / 2) balloon, at the rate = 4 m 3 / min
dt
For maxima and minima 4
Q V = πr 3, where r is the radius of spherical
f ′(x) = 0 3
1 balloon.
⇒ x = 1, , [Q − 1 ∉(0, 2)] dv dr
2 So, = 4 πr 2 at r = 8 m
dt dt
Now, 1 = 5 (minimum)
f()
dr dr 1
and f(1 / 2) =
87
(maximum) 4 = 4 π(64) ⇒ = m/min
16 dt dt 64π
ds dr
1 + f   = Q Area s = 4 πr 2 ⇒ = 4 π(2r) at r = 8 m
1 167
So, f () .
 2 16 dt dt
ds 1
= 64 π × = 1 m 2 / min.
67. The constant c of Lagrange’s mean value dt 64 π
 π π 69. If θ is an angle between the curves
theorem for f (x) = cos x − sin 2 x in − ,  is
 2 2 x 2 + 4 y = 0 and xy = 2 , then tanθ =
[23 April 2018, Shift-II] [24 April 2018, Shift-I]
 1 ± 33 
(a) 0 (b) sin−1   (a) − 1 (b)
1
(c)
1
(d) 3
 8  3 2
 1 ± 33  π
(c) cos −1   (d) ± Sol. (d)
 8  4 Point of intersection of the curves
Sol. (b) x 2 + 4 y = 0 and xy = 2
The given function f (x) = cos x − sin 2x, is a 2
⇒ x 2 = − 4 y and y =
π π
continuous function in interval  − ,  and
x
 2 2  ⇒ x =
2 −8
π π
differentiable in interval  − ,  .
x
 2 2 x3 = − 8 ⇒ x = − 2
370 AP EAMCET Chapterwise Mathematics

Slope of first curve at x = − 2 Sol. (c)


dy dy x Given, f (x) = x1 / 2
2x + 4 = 0⇒ =−
dx dx 2 1
 dy  − (− 2) 1 −2
m1 =   = =1 f ′(x) = x
 dx  x = − 2 2 2
1 1
Slope of the second curve xy = 2 1 −2 −
f ′(c) = c , g(x) = x 2
2 dy − 2 2
y= ⇒ = 2 3 3
x dx x −1 − 2 1 −2
g′(x) = x g′(c) = − c
 dy  −2 −1
m2 =   = = 2 2
 dx  x = − 2 (− 2)2 2 f ′(c) f (12) − f (3)
Now, =
If angle between them θ, then g′(c) g(12) − g(3)

1 −  − 
1 1
1 −2
m1 − m2  2 3/ 2 c
12 − 3
tanθ = = = = 3. ⇒ 2 =
1 + m1 m2 1 + () 
1 − 
1 1 /2 1 − 3/ 2 1 1
 2 − c −
2 12 3
2 3− 3
70. If the function f (x) = x 3 + 2px 2 + 27 x + 16 is =−c=
1 1
strictly increasing for all x ∈ R, then the −
2 3 3
range of p is [24 April 2018, Shift-I]
3
−9 ⇒ −c = ⇒c = 6.
(a)  −∞ ,  ∪  , ∞ 
9
(b) (−∞, − 9) ∪ (9, ∞ ) −
1
 2   2  2 3
− 9 9
(c)  ,  (d) (−9, 9)
 2 2 72. If the distance s described in time ‘t’ by a
particle moving on a straight line is given by
Sol. (c) s = t5 − 40 t3 + 30 t2 + 80 t − 250, then its
f (x) = x 3 + 2px 2 + 27 x + 16
minimum acceleration is
f ′(x) = 3x 2 + 4px + 27 [24 April 2018, Shift-I]
f (x) is strictly increasing function, so f ′(x) > 0 (a) 260 (b) − 260
⇒ 3x 2 + 4px + 27 > 0 (c) 130 (d) − 130

Possible only when its D < 0 Sol. (b)


(4p)2 − 4(3) (27) < 0 Given that, S = t 5 − 40t 3 + 30t 2 + 80t − 250
dS
16p 2 − 4 ⋅ 81 < 0 V= = 5t 4 − 120t 2 + 60t + 80
dt
81
p2 − <0 dV d 2S
4 a= = 2 = 20t 3 − 240t + 60
dt dt
⇒  p + 9  p − 9 < 0
    a = f ()
t = 20t 3 − 240t + 60
 2  2 Let

p ∈  − ,  . f ′()
t = 60t 2 − 240
9 9

 2 2
f ′()
t = 0 ⇒ 60t 2 − 240 = 0 ⇒ t 2 = 4
1 t = ± 2 ⇒ t = 2, t = − 2
71. If f (x) = x and g(x) = for x ∈[3 , 12],
x f ′′()
t = 120t
then the value of c ∈(3 , 12) for which at t = 2 , f ′′()
t >0
f ′ (c) f (12) − f (3) So, at t = 2 , f () t is minimum
= holds, is
g ′ (c) g(12) − g(3) So, minimum acceleration is
[24 April 2018, Shift-I] amin = f (2) = 20(2)3 − 240 ⋅ 2 + 60
(a) 7.5 (b) 4.8 (c) 6 (d) 9 = 160 − 480 + 60 = − 260.
26
Indefinite Integrals
x3 − 1  x2 + x + 1 2 2 x + 1
tan−1 
1. ∫ x3 + x dx = (b) log 
 | x − 1| 
 −
3  3 
 +c
[17 Sep. 2020, Shift-I]
 x2 + x + 1 2 x + 1
tan−1 
1 1
(a) x + log| x| + log ( x2 + 1) + sin−1( x) + c (c) log   −  +c
2  | x − 1|  3  3 
1
(b) x − log| x| + log ( x2 + 1) − sin−1( x) + c  x2 + x + 1 2 x + 1
tan−1 
2
2 (d) log   +  +c
1  | x − 1|  3  3 
(c) x + log| x| − log ( x2 + 1) + tan−1( x) + c
2 Sol. (d)
1
(d) x − log| x| + log ( x2 + 1) − tan−1( x) + c f (0) = f ()
1 = 3 f (2) = −3
2
Let f (x) = ax 2 + bx + c
Sol. (d)
⇒ c = −3, a + b + c = −3 ⇒ (a + b = 0)
 x 3 −1   x +1 
I= ∫  x 3 + x  dx = ∫ 1 − x 3 + x  dx and 4 a + 2b + c = −1 ⇒ 4 a + 2b = 2
So, a = 1, b = −1
(x + 1) (x + 1)
⇒ ∫ 1 ⋅ dx − ∫ x3 + x
dx = x − ∫
x(x 2 + 1)
dx f (x) x2 − x − 3
∫ x 3 − 1 dx = ∫ x 3 − 1 dx
x +1 A Bx + C
= + 2 x2 − x − 3 Bx + C 
+ 
Now, A
x(x 2 + 1) x x +1 = 
x 3 −1 (x − 1)  x 2 + x + 1 
⇒ (x + 1) = A(x 2 + 1) + (Bx + C) x
x 2(A + B) + x(A − B + C) + (A − C)
=
[using partial fractions] (x 3 − 1)
⇒ (x + 1) = (A + B) x 2 + Cx + A
On comparing A + B = 1, A − B + C = −1, A − C = −3
On comparing A + B = 0, C = 1, A = 1 ⇒ B = −1 ⇒ A = −1, B = 2, C = 2
1 (1 − x)
∴ I = x − ∫ dx − ∫ 2 dx  x 2 − x − 3 −1 (2x + 2)
x x +1 ∫  x 3 − 1  dx = ∫ (x − 1) dx + ∫ (x 2 + x + 1) dx
1 1 2x
⇒ I = x − log|x|− ∫ 2 dx + ∫ 2 dx (2x + 1)dx 1. dx
x +1 2 x +1 = − log|x − 1| + ∫ x2 + x + 1 + ∫ x2 + x + 1
1
⇒ I = x − log|x|− tan−1 x + log(x 2 + 1) + C
 x2 + x + 1 2x + 1 
tan−1 
2 2
= log   +  + C
 |x − 1|  3  3 
2. If f (x) is a polynomial of the second degree
in x such that f (0) = f ()
1 = 3 , f (2) = − 3. Then,  x x2 
f (x) 3. ∫ 1 + 1! + 2! + ... ∞ dx =
∫ x 3 −1 dx =  [17 Sep. 2020, Shift-I]
[17 Sep. 2020, Shift-I] 1
(a) log ( x + 1) + c (b) +c
 x2 + x + 1 2 x + 1 x+1
tan−1 
1
(a) log   +  +c
 | x − 1|  3  3  (c) e x + c (d) − e − x + c
372 AP EAMCET Chapterwise Mathematics

Sol. (c) x4 + x2 + 1
 x x 2 6. ∫ x2 − x + 1
dx =
∫ 1 + +
 1! 2!
+ ...... ∞  dx =
 ∫ e x ⋅ dx = (e x + c) [17 Sep. 2020, Shift-II]
(a)   x3 +   x2 + x + c
1 1
2 tan(x)  3 2
4. ∫ 1 + 2 tan2(x) dx = (b)   x3 −   x2 + x + c
1 1
[17 Sep. 2020, Shift-I]
 3 2
(a) log|cos 2 x + sin2 x| + c
(c)   x3 +   x2 − x + c
1 1
cos 2 x
(b) log + sin2 x + c  3 2
2
(d)   x3 −   x2 − x + c
1 1
sin2 x  3 2
(c) log cos 2 x + +c
2
Sol. (a)
cos 2 x sin2 x We have,
(d) log + +c
2 2 x4 + x2 + 1
I= ∫
x2 − x + 1
dx
Sol. (b)
  2sin x ⋅ cos x (x 2 + x + 1)(x 2 − x + 1)
2tan x I=∫ dx
∫ 1 + 2tan2 x  dx = ∫ cos2 x + 2sin2 x
dx x2 − x + 1
x3 x2
2sin x ⋅ cos x So, I = ∫ (x 2 + x + 1) dx = + + x+ c
⇒∫ dx 3 2
1 + sin2 x
Let1 + sin2 x = t ⇒ 2sin x ⋅ cos x dx = dt cos(13 x) − cos(14 x) sin(4 x) sin(5 x)
7. If ∫ dx = −
dt 1 + 2 cos(9 x) a b
⇒∫ = log|t|+ c1 = log|1 + sin2 x|+ c1
t + c, then a b = [17 Sep. 2020, Shift-II]
= log|2sin2 x + cos2 x|+ c1 (a) 45 (b) 54 (c) 44 (d) 55
2
cos x Sol. (a)
log 2 sin2 x + + c1
2 cos13x − cos14 x
2
Given, I = ∫ 1 + 2cos 9 x
dx
cos x
log 2 + sin2 x + + c1 cos 13x − cos14 x
2 Let, f (x) =
1 + 2cos 9 x
cos2 x sin 9 x (cos13x − cos14 x)
∴ log + sin2 x + c [Q log 2 + c1 = c] =
2 sin 9 x + sin18 x
x ⋅ sin − 
27 x
5. ∫ sin 3(x) ⋅ cos 3(x) dx = [17 Sep. 2020, Shift-I]
sin 9 x − 2sin
 2
= 2
(a) sin4 ( x) − sin6 ( x) + c ⋅ cos − 
27 x 9x
2sin
(b) cos 4 ( x) − cos 6 ( x) + c 2  2
1 1 x
(c) sin4 ( x) − sin6 ( x) + c sin 9 x ⋅ sin
4 6 = 2 = 2sin 9 x ⋅ sin x
cos 
1 1 9x 2 2
(d) cos 4 ( x) − cos 6 ( x) + c
4 6  2

= cos
9x x 
−  − cos + 
Sol. (c) 9x x
 2 2  2 2
∫ sin x ⋅ cos x dx = ∫ sin x ⋅ cos x (1 − sin x)dx
3 3 3 2
= cos 4 x − cos 5x
Let t = sin x ⇒ dt = cos x ⋅ dx So, I = ∫ (cos 4 x − cos 5x) dx
⇒ ∫t ⋅ (1 − t 2)dt = ∫ (t − t 5)dt
3 3
sin 4 x sin 5x
= − + C
t4
t 6
sin x sin x 4 6 4 5
⇒ − +C= − +C
4 6 4 6 Hence, a b = 45
Indefinite Integrals 373

8. Solve I n+ nI n − 1, if I n = ∫ (ln x)ndx I = ∫ e t  log()t + 2  dt


1

 t 
[17 Sep. 2020, Shift-II]
= ∫ e t   log t +  −  − 2   dt
1 1 1
(a) x(ln x)n − 1 + k (b) x(ln x)n + k   t   t t  
(ln x)n (ln x)n −1
+ k + k
= ∫ e t  log t +  dt − ∫ e t  − 2  dt
(c) (d) 1 1 1
x x  t t t 
Sol. (b) As, we know that,
In = ∫ (log x)n
∫ e ( f (x) + f ′(x)) dx = ∫ e f (x) + c ,
x x

 d(log x)n  et
I n = (log x)n ∫ dx − ∫  ∫ dx  dx + k So, I = e t log t − +c
 dx t
n(log x)n −1 x
I n = x(log x)n − ∫ ×x+k = x log(log x) − + c {Q t = log x}
x log x
I n = x(log x)n − nI n −1 + k  1 
= x  log (log x) − + c
⇒ I n = nI n −1 = x(log x) + k
n  log x 

cos 3 (x) 11. ∫ x − 1 (x x + 1)− 1 dx =


9. ∫ sin2(x) + sin(x) dx =
−1
[18 Sep. 2020, Shift-I]
[17 Sep. 2020, Shift-II] (a) ln| x + x − 1| − sec ( x) + c
2

(a) log| sin( x)| + sin( x) + c (b) ln| x − x2 − 1| − tan− 1( x) + c


(b) log| sin( x)| + cos( x) + c
(c) log| cos( x)| − sin( x) + c (c) ln| x + x2 − 1| + sec − 1( x) + c
(d) log| sin( x)| − sin( x) + c (d) ln| x + x2 − 1| − tan− 1( x) + c
Sol. (d)
Sol. (a)
cos3 x (1 − sin2 x) cos x
I= ∫ sin2 x + sin x . dx = ∫ sin2 x + sin x
. dx
I= ∫ x − 1(x x + 1)−1 dx =
1
∫ x⋅
x −1
dx
x +1
sin x = t ⇒ cos xdx = dt x −1 1 + t2
Put = t2 ⇒ x =
1 − t2 (1 − t) (1 + t) x +1 1 − t2
∴ I=∫ 2 . dt = ∫ dt
t + t t (1 + t) (1 − t 2)(2t) − (1 + t 2)(−2t)
So, dx = dt
(1 − t 2)2
= ∫  − 1 dt = log t − t + c
1
t  ⇒ dx =
4t
dt
= log|sin x| − sin x + c (1 − t 2)2

  1 − t2  4t 4t 2
1  So, I = ∫  2
t dt = ∫ dt
10. ∫ log(log x) +  dx =  1 + t  (1 − t )
2 2
(1 + t ) (1 − t 2)
2

 (log x)2 
 1 1 
[18 Sep. 2020, Shift-I] = 2∫  −  dt
 1 − t 2 1 + t 2) 
(a) x [log(log x) + log x] + c
1− t
= 2  − log  −1 
x 1
(b) +c  − tan () t +c
log(log x)  2 1 + t  
(c) x log(log x) + c  x −1 
  1 − 
(d) x log (log x) −
1
+c = − log e  x + 1  − 2 tan−1 x − 1 + c
 log x  1 + x − 1  x +1
 
 x +1 
Sol. (d)
 x −1 
 1  2 
I = ∫  log(log x) +  dx  x + 1 − x −1  −1 x +1  + c
(log x)2  = − log e   − tan 
  x + 1 + x −1  1 − x − 1 
Let, log x = t ⇒ dx = e t dt  
 x +1
374 AP EAMCET Chapterwise Mathematics

 x + 1 + x −1 − 2 x 2 −1  x / 22 + sin x 
= − log e 
 (x + 1) − (x − 1)
 − tan−1

x 2 −1 + c 14. ∫e   dx =
1 + cos x 
  [18 Sep. 2020, Shift-II]
cosec   + c (b) 2e x/ 2 tan   + c
−1 x x
= − log e (x − x − 1) − sec
2
x+c (a) 2e x/ 2
 2 2
−1
= log e (x + x − 1) − sec
2
x+c
cos  + c (d) 2e x/ 2 sin   + c
x x
(c) 2e x/ 2
cos 7 x − cos 8 x 2 2
12. ∫ dx =
1 + 2 cos 5 x [18 Sep. 2020, Shift-I] Sol. (b)
1  2 tan x / 2 
(a) sin 2 x − sin 3 x + c x x 2+ 
3 2 + sin x  1 + tan2 x / 2 
1 1 ∫ e2   dx ⇒ ∫ e2 dx
(b) sin 2 x − sin 3 x + c  1 + cos x   1 − tan2 x / 2 
2 3  1 + 
1  1 + tan2 x / 2 
(c) sin 2 x − sin 3 x + c x
2  2 + 2 tan2 x / 2 + 2tan x / 2 
1 1
(d) sin 2 x − sin 3 x + c ∫ e 2 1 + tan2 x / 2 + 1 − tan2 x / 2 dx
3 2
x
Sol. (b)  1 + tan2 x / 2 + tan x / 2
cos 7 x − cos 8 x cos 7 x − cos 8 x
∫ e 2 ⋅ 2  2
 dx

I= ∫ 1 + 2 cos 5x
dx = ∫ dx
1 + 2 − 4 sin2  
5x x
 x
+ tan  dx
x
 2 ∫ e 2  sec
2
2 2
15x x
2 sin sin x
=∫ 2 2 dx Put, = t ⇒ dx = 2dt
2
3 − 4 sin2  
5x
∫ e (sec t + tan t)⋅ 20dt
t 2
 2
15x x 5x 2⋅ e t tan t + c
2 sin sin sin
=∫ 2 2 2 dx
x
x
5x 3 5x 2⋅ e 2 tan
+c
3 sin − 4sin
2 2 2
x 5x Hence, option (b) is correct.
= ∫ 2 sin sin dx = ∫ cos 2x − cos 3x) dx
2 2  1 1 
1 1
= sin 2x − sin 3x + c
15. ∫  log x − (log x)2  dx =
2 3   [18 Sep. 2020, Shift-II]
(a) x log x + c (b) − x log x + c
13. Evaluate ∫ sin( k) dk on (0 , ∞) (c)
log x
+c (d)
x
+c
[18 Sep. 2020, Shift-I]
x log x
(a) 2 [cos( k ) − k sin ( k )] + c Sol. (d)
(b) 2 [cos( k ) + k sin ( k )] + c  1 1 
(c) 2 [ k cos( k ) − k sin ( k )] + c ∫  −  dx
 log x (log x)2 
(d) 2 [sin ( k ) − k cos ( k )] + c Put, log x = t
Sol. (d) x = et
I = ∫ sin( k) dk on k ∈ (0. ∞) dx = e t dt
1 −1  t
Put k = t 2 ⇒ dk = 2t dt ∫  t +  e dt
t2 
∴ I = 2 ∫ t sin t dt
We have,
= −2t cos t − 2 ∫ 1.(− cos t) dt ∫ e ( f (x) + f ′(x) dx = e . f (x) + C)
x x

(Integration by parts) 1 elog x x


= e t. + c = +c= +c
= −2t cos t + 2 sin t + c t log x log x
= 2[sin ( k) − k cos ( k)] + c Hence, option (d) is correct.
Indefinite Integrals 375

sin 3 (x) + cos 3 (x) 1 + cos(4 x)


19. If ∫ dx = k cos(4 x) + c, then
16. ∫ sin (x) ⋅ cos (x)
2 2
dx =
cot(x) − tan(x)
[21 Sep. 2020, Shift-I]
[21 Sep. 2020, Shift-II]
(a) sec( x) − cosec( x) + c
1 1 −1 −1
(b) tan( x) + cot( x) + c (a) k = (b) k = (c) k = (d) k =
(c) cosec( x) − cot( x) + c 8 4 8 4
(d) tan( x) − cot( x) + c Sol. (c)
Sol. (a) 1 + cos 4 x
I= ∫ cot x − tan x dx
sin x + cos x
3 3

∫ sin2 x cos2 x
dx 2cos2(2x)
= ∫ cos2 x − sin2 x dx = ∫ 2cos 2x sin x cos xdx
= ∫ (sec x tan x + cot xcosec x) dx
sin x ⋅ cos x
= sec x − cosec x + C 1 −1
= ∫ sin 4 xdx = cos(4 x) + c = k cos(4 x) + c
Hence, option (a) is correct. 2 8
(given)
dx
17. ∫ cos 2(x) + sin (2 x) = So, k = −
1
8
[21 Sep. 2020, Shift-I]
1 Hence, option (c) is correct.
(a) log| 1 + 2 cos( x)| + c
∫ x (tan
2 20. 2
x) dx =
1 [21 Sep. 2020, Shift-II]
(b) log| 1 − 2 tan( x)| + c 2
2 x
(a) x tan( x) − loge (sec x) − +c
1 2
(c) log| 1 + 2 tan( x)| + c 2
2 x
(b) x tan( x) + loge (sec x) − +c
1
(d) log| 1 + 2 cot( x)| + c 2
2
x2
(c) x tan( x) − loge (sec x) + +c
Sol. (c) 2
dx sec2 x dx x 2
I= ∫ cos2 x + sin 2x = ∫ 1 + 2tan x (d) x tan( x) + loge (sec x) +
2
+c

dt Sol. (a)
Put1 + 2tan x = t ⇒ sec2 x dx =
2 I= ∫ x(tan x)dx = x ∫ (sec2 x − 1)dx
2
1 dt 1
So, I = ∫ = log e|t| + C
2 t 2 − ∫ (1 ∫ (sec2 x − 1)dx)dx
1
= log e |1 + 2tan x| + C = x (tan x − x) − ∫ (tan x − x)dx
2
Hence, option (c) is correct. x2
= x tan x − x 2 − log e (sec x) + +C
2
(x + 1)2 x2
18. ∫ x(x 2 + 1) dx = = x tan x − log e (sec x) −
2
+C
[21 Sep. 2020, Shift-I]
Hence, option (a) is correct.
(a) log[ x( x + 1)] + c
2

∫x (tan −1 x + cot −1 x) dx =
2020
(b) log| x| + c 21.
(c) log| x| + 2 tan− 1( x) + c [21 Sep. 2020, Shift-II]
(d) 2 log| x| + tan− 1( x) + c x2021
(a) (tan−1 x + cot −1 x) + c
Sol. (c) 2020
x2021
(x + 1)2 (x 2 + 1) + 2x (b) (tan−1 x + cot −1 x) + c
I= ∫ x(x 2 + 1) dx = ∫ x(x 2 + 1)
dx 2021
πx2021 π
1 dx −1
(c) + +c
= ∫ x dx + 2∫ x 2 + 1 = log e|x| + 2tan x+ C 2021 2
x52 π
(d) + +c
Hence, option (c) is correct. 52 2
376 AP EAMCET Chapterwise Mathematics

Sol. (b)
−1 −1
25. ∫ ( 1 + sin(2 x)) dx =
[22 Sep. 2020, Shift-I]
I= ∫ x (tan x + cot x)dx
2020
(a) cos( x) + sin( x) + c
2020  π  Q tan−1 x + cot −1 x = π  (b) cos( x) − sin( x) + c
= ∫ x  2  dx 
 2

(c) sin( x) − cos( x) + c
π x 2021 x 2021 (d) sin( x) − cosec( x) + c
= + C= (tan−1 x + cot −1 x) + C
2 2021 2021 Sol. (b,c)
Hence, option (b) is correct. I= ∫ 1 + sin 2x dx = ∫ |sin x + cos x | dx

 (sin x + cos x) dx , if sin x + cos x ≥ 0


= ∫
22. Suppose that f and g are integrable on [ a , b],
then f + g is integrable on ……… .
− ∫ (sin x + cos x) dx , if sin x + cos x < 0
[21 Sep. 2020, Shift-II]
(a) (a, b ) (b) Cannot comment ∴ ∫| sin x + cos x| dx
(c) [a, b ] (d) Range of f + g sin x − cos x + C, if sin x + cos x ≥ 0
=
Sol. (c) cos x − sin x + C, if sin x + cos x < 0
As f and g are integrable on [a, b], then cos x − sin x
x2 x2
26. ∫ 5 + sin(2 x)
dx =
∫ f (x) dx and ∫ g(x) dx exists for every value of [22 Sep. 2020, Shift-I]
(a) cot −1  (sin x + cos x) + c
x1 x1 1 1
x2 x2
2  2 
x1 , x 2 ∈[a , b], then ∫ f (x) dx + ∫ g(x) dx −1  1
(sin x + cos x) + c
1
x1 x1 (b) tan
2  2 
x2
1 −1  1
= ∫ ( f + g)(x) dx Also exists for ∀x1 , x 2 ∈[a , b] (c) sin (sin x + cos x) + c
x1 2  2 
−1  1
(sin x + cos x) + c
1
Hence, option (c) is correct. (d) cos
2  2 
 2 x 3 − 3 x + 5
23. Integral ∫   dx is valid for Sol. (b)
 2x2  cos x − sin x
[21 Sep. 2020, Shift-II]
I= ∫5 + sin 2x
dx

(a) x ∈ R − {0} (b) x > 0 cos x − sin x


=∫ dx
(c) x < 0 (d) x ∈R 4 + (sin x + cos x)2
Sol. (b) Let sin x + cos x = t ⇒(cos x − sin x)dx = dt
 2x 3 − 3x + 5
= tan− 1   + c
dt 1 t
I = ∫  dx ∴ I=∫
 2x 2  4 + t2 2  2
x2 3 sin x + cos x 
= ∫  x − + 2  dx = tan− 1 
3 5 51 1
− log e x − +c =  + c
 2x 2x  2 2 2x 2  2 
is valid for x > 0, because log e x valid, if x > 0
Hence, option (b) is correct.
27. ∫ e 3 log x (x 4 + 1)−1 dx =
[22 Sep. 2020, Shift-I]
1
3log x
+c (b) log( x4 + 1) + c
∫ (1 + e − x )−1 dx =
24. (a) e
[22 Sep. 2020, Shift-I] 4
(a) log (1 + e − x ) + c (b) log(1 + e x ) + c 1 x4
(c) log( x4 + 1) + c (d) 4
(c) log(1 − e x ) + c (d) log(e x − 1) + c 3 x +1
Sol. (b) Sol. (b)
I = ∫ (1 + e − x −1
) dx = ∫ x
ex
dx I = ∫ e 3 log x (x 4 + 1)− 1 dx = ∫ x (x
3 4
+ 1)− 1 dx
e +1
Let x 4 + 1 = t ⇒ 4 x 3dx = dt
Put e x + 1 = t ⇒ e x dx = dt
1 dt 1 1
dt ∴ I = ∫ = log e () t + c = log e (x 4 + 1) + c
So, I = ∫ = log e | t | + C = log e (1 + e x ) + C 4 t 4 4
t
Indefinite Integrals 377

sin (2 x) = ∫ 1 ⋅ dx + 2 log|sin x − 2cos x| + c


28. ∫ sin (x) + 2 cos 2 (x)
2
dx =
 
[22 Sep. 2020, Shift-II] f ′(x)
(a) log|1 + cos ( x)| + c
2 Q ∫ f (x) dx = log| f (x)|
 
(b) − log|1 + sin2 ( x)| + c = x + 2 log |sin x − 2cos x| + c
(c) log|1 + tan2 ( x)| + c On comparison with given RHS, we get
(d) − log|1 + cos 2 ( x)| + c α = 1, β = 2 ⇒ α − β = −1
Hence, option (a) is correct.
Sol. (d)
sin 2x
∫ sin2 x + 2 cos2 x dx
30. ∫ e x cosec x ⋅ cosec x ⋅ (1 − x cot x) dx =
[22 Sep. 2020, Shift-II]
sin 2x sin 2x (a) e x cot x + c (b) e x cosec x + c
=∫ dx = ∫ dx
1 − cos x + 2 cos x
2 2
1 + cos2 x (c) e − x cosec x + c (d) e − x cot x + c
Put, cos2 x = t Sol. (b)
2cos x ( − sin x) dx = dt
∫e x cosec x(1 − x cot x) dx
x cosec

− sin 2x dx = dt
− dt Put, x cosec x = t
=∫ = − log |1 + t| + c [Q t = cos2 x] Differentiate w.r.to ‘x’
1+ t
d d d
= − log |1 + cos2 x| + c x. cosec x + cosec x. . x = t
dx dx dx
Hence, option (d) is correct. dt
x (− cosec x. cot x) + cosec x ()
1 =
5 tan x dx
29. If ∫ dx = αx + β log |sin x − 2 cos x| cosec x(1 − x cot x) dx = dt = ∫ e t . dt
(tan x) − 2
+ γ, then α − β = [22 Sep. 2020, Shift-II] = e t + c = e x cosec x + c
(a) – 1 (b) 2 Hence, option (b) is correct.
(c) 0 (d) 1
(1 + x) e x
Sol. (a) 31. ∫ cot (xe x )
dx =
5 tan x 5 sin x [22 Sep. 2020, Shift-II]
∫ tan x − 2 dx = ∫ sin x − 2cos x dx (a) log(cos( xe )) + c x
(b) log(cot ( xe x )) + c
(c) log(sec ( xe x )) + c (d) log (cosec ( xe x )) + c
Put, 5 sin x = α(sin x − 2cos x) +
d Sol. (c)
β (sin x − 2 cos x)
dx (1 + x) ⋅ e x
5 sin x = α(sin x − 2cos x) + β(cos x + 2 sin x) ∫ cot (x ⋅ e x) dx
On comparison of coefficients of sin x , cos x we
Put, xe x = t
get
sin x coefficients → 5 = α + 2β … (i) Differentiate w.r.to, x
cos x coefficients → 0 = − 2α + β … (ii) ⇒ e x (1 + x) dx = dt
On solving Eqs. (i) and (ii), weget dt
=∫
cot t ∫
= tan t dt
α = 1 and β = 2
∴ 5 sin x = 1 (sin x − 2 cos x) + 2 (cos x + 2 sin x) = log(sec t) + c
Now, substitute 5sin x value in the given = log (sec (xe x )) + c
question. Hence option (c) is correct.
5 sin x
∫ (sin x − 2 cos x) dx 3x
1(sin x − 2 cos x) + 2(cos x + 2 sin x)
32. ∫ 1 − 9x
dx =
=∫ dx [23 Sep. 2020, Shift-I]
(sin x − 2 cos x) (a) sin−1 (3 x ) ⋅ (log 3)−1 + c (b) − sin−1 (3 x ) ⋅ log 3 + c
1(sin x − 2 cos x) cos x + 2 sin x
=∫ dx + 2 ∫ dx 1
(c) sin−1(3 x ) + c
1
(d) sin−1 (3 x ) + c
sin x −2 cos x (sin x − 2 cos x) 3 9
378 AP EAMCET Chapterwise Mathematics

Sol. (a) Sol. (d)


3x 3x 1 + tan2 x cos2 x + sin2 x
∫ 1−9 x
dx = ∫ 1 − (3 ) x 2
dx ∫ 1 − tan2 x dx = ∫ cos2 x − sin2 x dx
1
Put 3 = t = ∫ cos 2x dx = ∫ sec2x dx
x

differentiate w.r. t. ‘x’ on both sides 1


= log|sec 2x + tan 2x| + c
3x log 3 dx = dt 2
1 1 1 1
dt = log + + c
1 log 3 2 cos 2x sin 2x
3 dx =
x
dt = ∫
log 3 1 − t2 1 1 + sin 2x
= log +c
1 1 1 2 cos 2x
sin−1 ()
log 3 ∫ 1 − t 2
= dt = t + c
log 3 2tan x
1+
1 1 + tan2 x + c
−1
= sin (3 ) ⋅ (log 3)
x −1
+ c = log
2 1 − tan2 x
Hence, option (a) is correct. 1 + tan2 x
x n −1 1 1 + tan2 x + 2tan x
33. ∫ x 2n
+4
dx = =
2
log
1 − tan2 x
[23 Sep. 2020, Shift-I]
1  xn  n  xn  1 (1 + tan x)2
(a) tan−1   + c (b) tan−1   + c = log + c
2n 2  2 2  2 (1 − tan x) (1 + tan x)
n −1  x n  1  xn  1 1 + tan x
(c) sin   + c (d) tan−1   + c = log + c
2 2  n 2  2 1 − tan x
Hence, option (d) is correct.
Sol. (a)
x −1
∫ (x + 1)
n −1 n −1
x x 35. dx =
Let I = ∫ x 2n + 4 dx = ∫ (x n)2 + (2)2 dx x3 + x2 + x [20 April 2019, Shift-I]
Put x n = t  1 + x + x2 
(a) 2 tan−1   +c
differentiate w.r.to ‘x’  x 
 
dt
n. x n −1 dx = dt ⇒ x n−1 . dx =  1 + x + x2 
n (b) tan−1   +c
 x 
dt / n 1 1  
∴ I= ∫ t 2 + 22 =
n ∫ t 2 + 22
dt
 x 
(c) tan−1   +c
= . tan−1   + c  1+ x + x 
1 1 t 2

n 2  2  1 + x2 
  (d) tan−1   +c
dx = tan−1   + c   
1 1 x
Q ∫ 2  x 
 a + x 2
a  a  
Sol. (a)
−1  x 
n
1
= tan   + c x −1 x2 − 1
 2
2n
∫ (x + 1) x3 + x2 + x
dx = ∫ 1
dx
Hence, option (a) is correct. (x + 1)2 x 1 + + x
x
1 + tan 2 x x2 − 1
34. ∫ 1 − tan x 2
dx = = ∫ 1  x2 1 + 1 + x
dx
[23 Sep. 2020, Shift-I] + + +
1 1 x 
 1 − tan x   1 + tan x   x  x
(a) log  +c (b) log   +c 1
 1 + tan x   1 − tan x  1− 2
1  1 − tan x  1  1 + tan x  =∫ x dx
(c) log   + c (d) log   +c 1 + 1 + 1 + x   1 + 1 + x
 
2  1 + tan x  2  1 − tan x     
x x
Indefinite Integrals 379
1 Sol. (b)
Let1 + + x = t2
x Given integral
⇒ 1 − 2  dx = 2t dt = ∫ x 5 dx
1 2t dt dt
= 2∫
 x  (1 + t 2)t 1 + t2 ∫ (x 2 + x + 1) (x + 1)(x 4 − x 3 + x − 1)
6

 1 + x + x2  Q(x + x + 1)(x 4 − x 3 + x − 1) = (x − 1)(x 3 + 1)


2
t + c = 2tan−1 
= 2tan−1 ()  + c
  (x 2 + x + 1) = (x 3 − 1)(x 3 + 1) = x 6 − 1
 x 
x 5 dx
Hence, option (a) is correct. ∴ Given integral = ∫ 6
(x + 1)(x 6 − 1)
36. If I (x) = ∫ x 2(log x)2 dx and I()
1 = 0 , then I (x) put x = t ⇒ 6 x dx = dt
6 5

1 dt
[20 April 2019, Shift-I] then given integral = ∫
x3 7 6 (t + 1)(t − 1)
(a) [8(log x)2 − 3log x] +
18 18 1 t −1 1 x 6 −1
= log e + c = log e 6 + c
x3 2 12 t +1 12 x +1
(b) [9(log x)2 + 6log x] −
27 27 Hence, option (b) is correct.
x3 2 dx
(c) [9(log x)2
27
− 6 log x + 2 ] −
27
38. ∫x+ x −1
=
[20 April 2019, Shift-I]
x3 2
(d) [9(log x)2 − 6 log x − 2 ] + 1 2 x − 1 + 1
27 27 (a) loge x + x − 1 − tan−1   +c
3  3 
Sol. (c) 1 2 x − 1 + 1
Given integral (b) loge x + x − 1 − tan−1   +c
3  3 
x3 x 3 2log x
I(x) = ∫ x 2 (log x)2 dx = (log x)2 − ∫ dx 2 −1  2 x − 1 + 1
3 3 x (c) loge x + x − 1 − tan   +c
3  3 
[by integration by parts]
2 2 x − 1 + 1
3
 3
x3 1  (d) loge x + x − 1 − tan−1   +c
(log x)2 −  (log x) − ∫   dx 
x 2 x
= 3  3 
3 3 3 
3 x  
x3 2 x3 1 x3  Sol. (d)
= (log x)2 −  (log x) − + c dx
3 3 3 3 3 Given integral, I = ∫x+ x −1
x3
= [9(log x)2 − 6(log x) + 2] + c put x − 1 = t 2 ⇒ dx = 2t dt
27 2t (2t + 1) − 1
then I = ∫ 2 dt = ∫ 2 dt
Q 1 =0
I() (t + 1) + t t + t +1
2 2t + 1 dt
∴ + C=0 =∫ 2 dt − ∫ 2
27 t + t +1 t + t +1
2 dt
⇒ C=− = log e|t 2 + t + 1| − ∫
27 2
t +  + 3
1
 
x3 2  2 4
∴ I(x) = [9(log x)2 − 6(log x) + 2] −
27 27  1
2 t + 
Hence, option (c) is correct. = log e|t + t + 1| −
2 −1
tan  2 + c
3  3 
x 5 dx
37. ∫ (x 2 + x + 1) (x 6 + 1)(x 4 − x 3 + x −1) =  2 
2t + 1 
tan−1 
2
= log e|t 2 + t + 1| −  + c
[20 April 2019, Shift-I] 3  3 
x −1
6
1 x6 − 1 On putting value of t = x − 1, we get
(a) loge +c (b) loge 6 +c
x +1
6
12 x +1 2  2 x − 1 + 1
I = log e |x + x − 1| − tan−1   + c
1 x +1
4
x + 4
8 3  3 
(c) loge 4 +c (d) loge +c
12 x −1 x6 − 1 Hence, option (d) is correct.
380 AP EAMCET Chapterwise Mathematics

−3 x2 x 3 2 4 x + 6 2 dx 9 1
39. If x ≠ , then ∫ dx = 2
− ∫ + ∫ 2
4 2x + 6 2x + 9 2 2x + 6 2x + 9
2
dx
2 2x + 6 2x + 9
2
x 3 2 9 1
[20 April 2019, Shift-II] − log|2x 2 + 6 2x + 9| + ∫ dx
  2 4 2 ( 2x + 3)2
1 9
(a)
2 2 ( 2 x + 3) − 6log 2 x + 3 − 2 x + 3  + c  f ′(x) 
  Q ∫ f (x) dx = log| f (x)|
1  9   
(b) 2 x + 3 − 6log ( 2 x + 3) + +c
2 2  2 x + 3  x 3 2 9 ( 2x + 3)−2+ 1 1
− log|( 2x + 3)2| + × + c′
(c) 2 x + 3 − 6log( 2 x + 3) + c 2 4 2 − 2+ 1 2
(d) log(2 x2 + 6 2 x + 9) + c x 3 2 9 ( 2x + 3)−1 1
− × 2log| 2x + 3|+ × + c′
2 4 2 −1 2
Sol. (a)
x 3 2 9 1
x2 − log| 2x + 3| − × + c′
Given, ∫ 2x 2 + 6 2x + 9 dx 2 2 2 2 2x + 3
1  9 
(2x 2 + 6 2x + 9) −  3 2x + 
1 9 2x − 6 log| 2x + 3| − + c′
2  2 dx 2 2  2x + 3
∫ 2x 2 + 6 2x + 9 1  9 
= ( 2x + 3) − 6 log| 2x + 3| − + c
∴ x 2 = (2x 2 + 6 2x + 9) 1 −  3 2x + 9   2 2  2x + 3
  
2  2  Hence, option (a) is correct.
  3 2x + 9  
1    1 − x 2 sin −1 x + x
 2
∫  2 − 2x 2 + 6 2x + 9  dx 40. ∫ 1 − x2
dx =
  [20 April 2019, Shift-II]
 
−1
9 (a) x sin x + 1 − x + c (b) sin−1 x +
2
1 − x2 + c
3 2x + −1
1 x sin x
∫ 2 dx − ∫ 2x 2 + 6 2x2+ 9 dx (c) x sin−1 x + c (d) +c
1 − x2
9
3 2x + Sol. (c)
x
−∫ 2 2 dx …(i)
2 2x + 6 2x + 9 1 − x 2 ⋅ sin−1 x + x
Given, ∫ dx
9 d
3 2x + = A (2x 2 + 6 2x + 9) + B 1 − x2
2 dx
 1 − x 2 ⋅ sin−1 x x 
∫   dx
9 +
3 2x + = A(4 x + 6 2) + B …(ii) 2
2  1− x 2
1− x 
On comparison, we get
 x 
4A = 3 2 −1
∫  sin x+  dx
1 − x 2 
9 3 2 
6 2A + B = ⇒ A=
2 4 −1 x
3 2 9 9 ∫ sin x dx + ∫ 1 − x2
dx
6 2× + B = ⇒B = −
4 2 2
sin −1 x ⋅ ∫ 1 dx − ∫  sin −1 x ⋅ ∫ 1 dx  dx +
d x
Substituting values of A, B Eq. (ii)  dx  ∫ dx
1 − x2
9 3 2 9
3 2x + = (4 x + 6 2) − …(iii) [Q By using by parts with f (x) = sin−1 x; g(x) = 1]
2 4 2
1 x
Substituting Eq. (iii) in Eq. (i) sin−1 x ⋅ x − ∫ ⋅ x dx + ∫ dx + c
3 2 9 1− x 2
1 − x2
 (4 x + 6 2) −  dx
x  4 2 x ⋅ sin−1 x + c
2 ∫

2x 2 + 6 2x + 9 Hence option (c) is correct.
Indefinite Integrals 381

e 2x
41. If ∫ x 3 e 2x dx = f (x) + c, then the sum of
42. ∫ sin 5x ⋅ cos 5 x dx = [20 April 2019, Shift-II]
8 6
cos x
all the complex roots of f (x) = 1 is (a) (6sin4 x + 3sin2 x + 1) + c
60
[20 April 2019, Shift-II]
sin6 x
(a)
1
(b) 3 (c) 1 (d) 2 (b) − (6cos 4 x + 3cos 2 x + 1) + c
2 60
cos 6 x
Sol. (a) (c) − (6sin4 x + 3sin2 x + 1) + c
60
e 2x sin6 x
Given, ∫ x 3 ⋅ e 2x dx = f (x) + c (d) (6cos 4 x + 3cos 2 x + 1) + c
8 60
By applying integration by parts, we get
Sol. (c)
e 2x
x 3 ⋅ ∫ e 2x dx − ∫  x 3 ∫ e 2x dx) dx  =
d
+ f (x) + c I = ∫ sin5 x cos5 x dx = ∫ cos5 x sin4 x sin x dx
 dx  8
e 2x e 2x e 2x = ∫ cos5 x (1 − cos2 x)2 sin x dx
x3 ⋅ − ∫ 3x 2 ⋅ dx = f (x) + c
2 2 8 Let cos x = t ⇒ − sin x dx = dt
1 3 2x 3  2 e 2x e 2x  e 2x So, I = ∫ t 5(1 − t 2)2(− dt)
x e − x ⋅ − ∫ 2x ⋅ dx  = f (x) + c
2 2 2 2  8
= − ∫ t 5(t 4 + 1 − 2t 2)dt = − ∫ (t 9 − 2t 7 + t 5) dt
[Again applying integration by parts]
1 3 2x 3 2 2x 3 e 2x  t10 t8 t6 
x e − x ⋅ e + ∫ x ⋅ e 2x dx = f (x) + c = − −2 + + c
2 4 2 8  10 8 6
1 3 2x 3 2 2x 3  e 2x e 2x  t6 4
x e − x ⋅ e + x − ∫1 ⋅ dx  = − [6t − 15t 2 + 10] + C
2 4 2 2 2  60
e 2x cos6 x
= f (x) + c =− [6 cos4 x − 15cos2 x + 10] + c
8 60
3 e 2x e 2x cos6 x
1 3 2x 3 2 2x
x e − x e + xe 2x − ⋅
3
= f (x) + c =− ( − sin2 x)2 − 151
[61 ( − sin2 x) + 10] + c
2 4 4 4 2 8 60
[put the value of t]
e 2x e 2x
[4 x 3 − 6 x 2 + 6 x − 3] + c1 = f (x) + c cos6 x
8 8 =− [6sin x − 12sin x + 6 − 15
4 2
60
∴On comparison, we get
+ 15sin2 x + 10] + c
f (x) = 4 x 3 − 6 x 2 + 6 x − 3
cos6 x
But given, =− [6sin4 x + 3sin2 x + 1] + c
60
f (x) = 1
Hence, option (c) is correct.
4x3 − 6x2 + 6x − 3 = 1
4x3 − 6x2 + 6x − 4 = 0 43. If ∫ cos x ⋅ cos 2 x ⋅ cos 5 x dx
Divided by 2, we get = A sin 2 x + Bsin 4 x + C sin 6 x + D sin 8 x + k
2x 3 − 3x 2 + 3x − 2 = 0 (where k is the arbitrary constant of
⇒ 2(x − 1) − 3x(x − 1) = 0
3 1 1
integration), then + =
⇒ 2(x − 1) (x 2 + x + 1) − 3x(x − 1) = 0 B C
[21 April 2019, Shift-I]
⇒ (x − 1)[2x 2 + 2x + 2 − 3x] = 0 1 1 1 1
(a) − (b) + (c) 1 (d) 0
⇒ (x − 1)[2x 2 − x + 2] = 0 A D A D
x = 1 is real root. Sol. (b)
So, sum of non-real complex root from the Given,
−1
quadratic equation 2x 2 − x + 2 = 0 is −   =
1
 2 2 ∫ cosx ⋅ cos 2x ⋅ cos 5x dx
1
2∫
Hence, option (a) is correct. = 2cos x cos 5x cos 2x dx
382 AP EAMCET Chapterwise Mathematics

1 dx
2∫
= {cos(5x + x) + cos(5x − x)} cos 2x dx 45. ∫ sin x + sin2 x = [21 April 2019, Shift-I]
1
= ∫ (cos 6 x + cos 4 x) cos 2x dx 1 1 2
2 (a) loge | 1 + cos x| + loge | 1 − cos x| − loge |
2 6 3
1 1 + 2 cos x| + c
= ∫ (2cos 6 x cos 2x + 2cos 2x cos 4 x) dx
4 1 2 1
(b) loge | 1 + cos x| − loge | 1 − cos x| + loge |
1
= ∫ (cos 8 x + cos 4 x + cos 6 x + cos 2x) dx 2 3 2
4 1 + 2 cos x| + c
1  sin 8 x sin 4 x sin 6 x sin 2x  1 1 1
= + + + + k (c) loge | 1 + sin x| − loge | 1 − sin x| − loge |
4  8 4 6 2  2 3 3
sin 2x sin 4 x sin 6 x sin 8 x 1 + cos x| + c
= + + + + k 1 1 2
8 16 24 32 (d) loge | 1 − sin x| + loge | 1 + cos x| − loge |
3 2 3
On comparing,
1 − 2 cos x| + c
1 1 1 1
A= ,B= ,C= and D = Sol. (a)
8 16 24 32
1 1 Given,
∴ + = 16 + 24 = 40 dx dx
B C
1 1
∫ sin x + sin 2x = ∫ sin x + 2sin x cos x
Now, + = 8 + 32 = 40
A D dx
1 1 1 1
= ∫ sin x (1 + 2cos x)
∴ + = +
B C A D sin x dx
2
= ∫ sin2 x (1 + 2cos x)
 x + 2
44. If ∫ e x   dx = f (x) + arbitrary constant, − sin x dx
 x + 4 = ∫ (cos2 x − 1) (1 + 2cos x)
then f (x) = [21 April 2019, Shift-I]
Let cos x = t
xe x ex dt
(a) (b)
x+ 4 x+ 4 ⇒ − sin x dx = dt = ∫ (t 2 − 1) (1 + 2t)
xe x ex
(c) (d) By partial fraction,
( x + 4)2 ( x + 4)2 1 A B C
= + +
Sol. (a) (t − 1) (t + 1) (2t + 1) (t − 1) (t + 1) (2t + 1)
2
 x + 2 ⇒ 1 = A(t + 1) (2t + 1) + B(t − 1) (2t + 1) + C(t 2 − 1)
Given,∫ e x   dx = f (x) + c
 x + 4 At t = −1
2 ⇒ 1 = A(0) + B(− 2) (− 1) + C(0)
 x + 2 x  x + 4 + 4x 
2

∫e  dx = ∫ e 
x
Now,   dx ⇒ 1 = 2B ⇒ B =
1
 x + 4  (x + 4)2  2
 x 4 
At t = − ⇒1 = A(0) + B(0) + C  − 
1 3
= ∫ ex  +  dx
 x + 4 (x + 4)2  2  4
x 4 4
Let g(x) = , then g′(x) = ⇒C = −
(x + 4) (x + 4)2 3
At t = 0,
= ∫ e x {g(x) + g′(x)} dx
1 4
⇒ 1= A− B− C ⇒1= A− +
= e x g(x) + c 2 3
 x  1 4 1
= ex  ⇒ A =1 + − =
 + c 2 3 6
 x + 4
1 1 4
xe x ∴ A = , B = and C = −
∴ f (x) = 6 2 3
x+ 4
Indefinite Integrals 383
dt 1/ 6 1 dt sin 5x sin 3x
Now, ∫
(t 2 − 1) (1 + 2t) ∫ (t − 1)
= dt + ∫ =2 + 2 + 2sin x + c
2 t +1 5 3
4 dt 2sin 5x 2
− ∫ = + (3sin x − 4sin3 x) + 2sin x + c
6 t + 1 5 3
 
 2 2 8
I 6 = sin 5x − sin3 x + 4sin x + c
ln(t − 1) 1
+ ln(t + 1) − ln t +  + c
4 1 5 3
=
6 2 6  2 x + sin x
=
ln(cos x − 1) 1
+ ln(cos x + 1) − ln cos x +  + c
2  1 47. ∫ 1 + cos x dx = [21 April 2019, Shift-II]
6 2 3  2
x
1 1 (a) loge (1 + cos x) + c (b) xsin2 +c
= ln (1 + cos x) + ln (1 − cos x)| 2
2 6 x x
2 1 (c) tan +c (d) x tan + c
− ln cos x + +c 2 2
3 2
1 1 Sol. (d)
= ln |(1 + cos x)| + ln |(1 − cos x| x + sin x
2 6
−2
Let I = ∫ 1 + cos x dx
ln |(1 + 2 cos x)| + c
3 x sin x
sin nx
= ∫ 1 + cos x dx + ∫ 1 + cos x dx = I1 + I2
46. In I n = ∫ dx for n = 1, 2 , 3 , ..., then I 6 =
sin x x xdx
[21 April 2019, Shift-I]
∴ I1 = ∫ 1 + cos x dx = ∫  1 − tan2 x 
3 8 5  2
1+ 
(a) sin 3 x + sin x − sin x + c x
5 5  1 + tan2 
2 5 3  2
(b) sin 5 x − sin x − 2 sin x + c
5 3 x
x sec2 dx
2 8 5 x x

2 =
= ∫ 2 sec
2
(c) sin 5 x − sin x + 4sin x + c dx
3 3 2 2
2 8 3 x x x
(d) sin 5 x − sin x + 4sin x + c = 2× tan − ∫ tan [using by parts]
5 5 2 2 2
= x tan − 2logsec  + c1
x x
Sol. (d)
sin nx 2  2
Given, In = ∫ sin x dx … (i)
= x tan − logsec2  + c1
x x
…(i)
sin(n − 2) x 2  2
In − 2 = ∫ sin x
dx … (ii)
I2 = ∫
sin x
dx
Subtracting Eq. (ii) from Eq. (i), we get 1 + cos x
{sin nx − sin(n − 2) x} Let1 + cos x = t = − sin xdx = dt = sin xdx = − dt
In − In − 2 = ∫ dx
dt
sin x ∴ I 2 = − ∫ = − log t + c 2
2cos(n − 1) x sin x t
=∫ dx = ∫ 2cos(n − 1) x dx
sin x = − log|1 + cos x| + c 2
2sin(n − 1) x  1 
= ⇒ log  + c2
(n − 1) 1 + cos x
2sin 5x 2sin 3x
∴I 6 − I 4 = and I 4 − I 2 =  
5 3
⇒ log  + c ⇒ logsec2 x + c
1
sin 2x 2sin x cos x …(ii)
Now, I2 = ∫ dx = ∫ dx cos2 x 2  2
2

sin x sin x  2
= 2∫ cos x dx = 2sin x + c Now, I = I1 + I 2
= x tan − logsec2  + logsec2  + c
sin 5x x x x
and I6 = I4 + 2
5 2  2  2
sin 3x sin 5x x
= I2 + 2 + 2 = x tan + c
3 5 2
384 AP EAMCET Chapterwise Mathematics

2 π  x ∴ 1 = A(x 2 + 1) + B(x + 1)(x 2 + 1) + (Cx + D)(x + 1)2


48. ∫x  2 sin  4 + x  + e  dx =
  Put x = −1 ⇒ 1 = 2A ⇒ A =
1
[21 April 2019, Shift-II] 2
Put x 2 = − 1
(a) ( x2 + 2 x − 2 )sin x + (− x2 + 2 x + 2 )cos x
∴ 1 = (Cx + D)(x 2 + 2x + 1) = (Cx + D)2x
+ ( x2 − 2 x + 2 ) e x + c
(b) (− x2 + 2 x − 2 )sin x + ( x2 + 2 x − 2 )cos x or 1 = 2Cx 2 + 2Dx = − 2C + 2Dx
+ ( x − 2 x + 2) e + c
2 x Q x 2 = −1
(c) ( x2 + 2 x + 2 )sin x + (− x2 − 2 x − 2 )cos x 1
∴ C=− ,D=0
+ ( x2 − 2 x + 2 ) e x + c 2
Comparing coefficient of x 3, we get
(d) ( x − 2 x − 2 )sin x + (− x + 2 x − 2 )cos x
2 2
1
+ ( x2 − 2 x + 2 ) e x + c O= B+ C∴B= −C=
2
1
Sol. (a) ∴∫ dx
2 π (x + 1)2(x 2 + 1)
Let I = ∫x 2 sin + x  + e x  dx
  4   1 1 1 1 1 x 
= ∫ ⋅ + ⋅ − ⋅ 2  dx
π π π  2 ( x + 1) 2
2 x + 1 2 x + 1
Q 2 sin + x  = 2sin cos x + cos sin x 
 4   4 4  1 1 1 
= − + log(x + 1) − log(x 2 + 1) + c
2  x + 1
2 sin x  = (cos x + sin x)
1 1 2 
= cos x +
 2 2  1 1
= log x + 1 − log x 2 + 1 − + c
2(x + 1)
∴ I= ∫ x [(cos x + sin x) + e ]dx
2 x 2

= ∫ x 2(cos x + sin x)dx + ∫ e x x 2dx 50. For n ≥ 2, if I n = ∫ (sin x + cos x)n dx then
Now, ∫ x 2e x dx = x 2e x − 2xe x + 2e x nI n − 2(n − 1)I n − 2 = [21 April 2019, Shift-II]
(a) (sin x + cos x)n + 1(sin x − cos x) + c
= e x [x 2 − 2x + 2] + c
(b) (sin x + cos x)n (sin x − cos x) + c
and ∫ x 2(cos x + sin x)dx (c) (sin x + cos x)n − 1(sin x − cos x) + c
= (x 2 + 2x − 2)sin x − (x 2 − 2x − 2) cos x + c (d) (sin x − cos x)n − 1(sin x + cos x) + c

∴ I = (x 2 + 2x − 2)sin x + (− x 2 + 2x + 2) cos x Sol. (c)


+ (x − 2x + 2)e + c
2 x We have,
In = ∫ (sin x + cos x)
n
dx
dx
49. ∫ (x + 1)2(x 2 + 1) =
n −1
[21 April 2019, Shift-II] = ∫ (sin x + cos x) ⋅ (sin x + cos x)dx
1 1
(a) loge x + 1 + loge x2 + 1 − +c n −1
2 x+1 = ∫ (sin x + cos x) (sin x − cos x)
1 1
(b) loge x + 1 − loge x2 + 1 − +c − ∫ (n − 1)(sin x + cos x) n − 2(cos x − sin x)
2 2( x + 1)
1 1
(c) loge x + 1 − loge x2 + 1 +
1
+c (sin x − cos x)dx
2 4 2( x + 1) Integration by parts,
1 1
(d) loge x + 1 + loge x2 + 1 +
1
+c ∴ I n = (sin x + cos x)n −1 (sin x − cos x)
4 2 x+1
+ (n − 1)∫ (sin x + cos x)n − 2 {(sin x − cos x)2}dx
Sol. (b)
dx As we know that,
Given, ∫
(x + 1)2(x 2 + 1) (sin x + cos x)2 + (sin x − cos x)2 = 2
1 A B Cx + D I n = (sin x + cos x)n −1 (sin x − cos x) + (n − 1)
Let = + + 2
(x + 1)2(x 2 + 1) (x + 1)2 (x + 1) x +1 n−2
∫ (sin x + cos x) 2 − (sin x + cos x)2 dx
Indefinite Integrals 385

= (sin x + cos x)n −1 (sin x − cos x) + (n − 1) ∴I=


1 1
I1 − I 2
n−2 2 2
∫ 2(sin x + cos x) dx − (n − 1)∫ (sin x + cos x)dx
(3x − 2) (3x + 2)
3 3

= (sin x + cos x) n −1
(sin x − cos x) + 2(n − 1) = (x + 1) 2 − (x −1) 2 + c
15 15
I n − 2 − (n − 1)I n [where c = c1 + c 2]
⇒ nI n − 2(n − 1)I n − 2 = (sin x + cos x)n −1 1 −1
∴ A(x) = (3x − 2) and B(x) = (3x + 2)
(sin x − cos x) + c 15 15
1 1 −4
3 3 ∴ A(x) + B(x) = (3x − 2) − (3x + 2) =
x 15 15 15
51. ∫ x +1 + x −1
dx = A(x)(x +1) 2 + B(x)(x − 1) 2
x .log x
+ c, then A(x) + B(x) = [22 April 2019, Shift-I]
52. ∫( x 2 −1)3
dx =
[22 April 2019, Shift-I]
4 4 2x 2x
(a) (b) − (c) (d) − −1 log x log x
15 15 5 5 (a) sec x+ + c (b) sec −1 x − +c
x −1
2
x2 − 1
Sol. (b)
log x − log x
x (c) − sec −1 x + c (d) − sec −1 x + c
Let I = ∫ x +1 + x −1
dx
x −1
2
x −1
2

x[ x + 1 −
x −1] Sol. (b)
= ∫x +1 − x +1
dx
x log x
Let I= ∫( x 2 − 1)3
dx
1 1 1 1
= ∫ x x + 1 dx − ∫ x x −1 dx = I1 − I 2
2 2 2 2 2x dx xdx
Let x 2 −1 = t ⇒ = dt ⇒ = dt
Now, I1 = ∫ x x + 1dx 2 x −1 2
x 2 −`1
x + 1 = u ⇒ dx = du log x dt
Put ∴ I =∫
3 1 t2
∴ I1 = ∫ (u −1) udx = ∫ (u 2 − u 2)dx log( 1 + t 2)
dt = ∫ log (1 + t 2).  2  dt
1
5 3 5 3
= ∫ t 2 t 
2 2 2 2 2 2
= u − u + c1 = (x + 1) 2 − (x + 1) 2 + c1
5 3 5 3 [Q x 2 − 1 = t] ⇒ x 2 = t 2 + 1 ⇒ x = 1 + t 2 ]

.  −  .
3 1 2t 1 1
= − log 1 + t 2 − ∫
= 2(x + 1) 2  (x + 1) −  + c1
1 1 dt
 5 3
t 2 1 + t2  t  1 + t2
3 1 dt
3x + 3 − 5 = − log 1 + t 2 + ∫
= 2(x + 1) 2  + c1 t 1 + t2
 15 
1
2(3x − 2)
3 = − log 1 + t 2 + tan−1 () t +c
= (x + 1) 2 + c1 t
15 log x log x
tan−1 ( x 2 − 1) − + c = sec−1 (x) − +c
Again, I 2 = ∫ x x −1dx x 2 −1 x 2 −1
Put x −1 = v ⇒ dx = dv cos 2 x . sin 4 x
∴ I 2 = ∫ (v + 1) vdv = ∫ (v 2 + v 2)dv
3 1 53. ∫ cos 4 x(1 + cos 2 2 x)dx =
[22 April 2019, Shift-I]
5 3 5 3  1 + cos 2 x 
 + sec x + c
2
2 2 2 2 2 2 (a) log
= v + v + c 2 = (x −1) 2 + (x −1) 2 + c2  1 + cos 2 2 x 
5 3 5 3
3 (1 + cos 2 x)2
= 2(x −1) 2  (x −1) +  + c 2
1 1 (b) log + sec x + c
 5 3 (1 + cos 2 x)
3
3x − 3 + 5 (1 + cos 2 x)2
= 2(x −1) 2  + c2 (c) log + sec 2 x + c
 15  (1 + cos 2 2 x)
3 1 + cos 2 2 x
2(3x + 2) (d) log + sec x + c
= (x −1) 2 + c 2 (1 + cos 2 x)2
15
386 AP EAMCET Chapterwise Mathematics

Sol. (c) Sol. (c)


cos 2x ⋅ (2sin 2x cos 2x) Given,
I = 4∫ dx
x15  3 + 6 
(1 + cos 2x)2 (1 + cos2 2x) 2 5
2x12 + 5x 9 x x 
Put cos 2x = t ⇒ −2sin 2xdx = dt ∫ (1 + x 3 + x 5)3 dx = ∫ 15  1 1  3 dx
t2 x  5 + 2 + 1
∴ I = − 4∫ dt x 
(1 + t) (1 + t 2)
2 x
 2 5 
Split into partial fractions  3 + 6
x x 
 1 1 t  =∫ 3
dx
I = − 4∫  − + 2 
dt  1 + 1 + 1
( + t) 2
( + t) 21
21 ( + t )  5 
 21 x x2 
 1  Let, 5 + 2 = t ⇒  − 6 − 3  dx = dt
1 1 1 5 2
= − 2 − − log(1 + t) + log(1 + t 2)  x x 
 1+t 2 x x

 5 + 2  dx = − dt = − dt 1 1
=
2
+ 2log(1 + cos 2x) − log(1 + cos2 2x) + c
 6
x

x3  ∫ (t + 1)3 = 2 (t + 1)2 + c
2cos2 x
x10
+ c Q t = 5 + 2 
1 1 1
= sec2 x + 2log(1 + cos 2x) − log(1 + cos2 2x) + c =
2 (1 + x 3 + x 5)2  x x 
(1 + cos 2x)2
= sec2 x + log +c
(1 + cos2 2x) x 2 + cos 2 x
56. ∫ (1 + x 2)sin2 x dx =
[22 April 2019, Shift-II]
sin x − cos x
8 8
54. ∫ 1 − 2sin2 x + 2sin4 x dx = (a) cot x + tan −1
x+c (b) cot x − tan− 1 x + c
[22 April 2019, Shift-II] −1
(c) − cot x + tan x+c (d) − cot x − tan− 1 x + c
1
(a) − sin2 x + c (b) − sin2 x + c
2 Sol. (d)
1 x 2 + cos2 x
(c) sin2 x + c (d) sin2 x + c
2
Given, ∫ (1 + x 2)sin2 x
dx

Sol. (a) x2 cos2 x


Given, = ∫ (1 + x 2)sin2 x
dx + ∫ (1 + x 2)sin2 x
dx
sin8 x − cos8 x
∫ 1 − 2sin2 x + 2sin4 x dx (x 2 + 1 − 1) (cos2 x − 1 + 1)
= ∫ (x 2 + 1) sin2 x dx + ∫ (1 + x 2) sin2 x
dx
(sin4 x − cos4 x) (sin4 x + cos4 x)
= ∫ dx 1 1 1 − cos2 x
1 − 2sin2 x(1 − sin2 x) = ∫ sin2 x dx − ∫ (x 2 + 1)sin2 x dx − ∫ (1 +x 2)sin2 x
(sin2 x − cos2 x) (sin2 x + cos2 x) 1 1
dx + ∫ dx = ∫ cosec2 x dx − ∫ dx
{(sin2 x + cos2 x)2 − 2sin2 x cos2 x} (1 + x 2)sin2 x 1 + x2
= ∫ 1 − 2sin2 x cos2 x
dx
= − cot x − tan− 1 x + c
(sin x − cos x) (1 − 2sin x cos x)
2 2 2 2
= ∫ (1 − 2sin2 x cos2 x)
dx
57. If I n = ∫ sin n xdx , for n = 1, 2, 3,K then
= ∫ (sin2 x − cos2 x) dx = − ∫ cos 2x dx 8 I 8 + 7 (I 7 − I 6) − 6 I 5 = [22 April 2019, Shift-II]

1 (a) − sin x cos x(1 + sin x) + c


6

=− sin 2x + c
2 (b) sin8 x cos x + sin5 xcos x + c
(c) − sin7 xcos x (1 − sin x) + c
2 x 12 + 5 x 9
55. ∫ (1 + x 3 + x 5)3 dx = (d) − cos 7 xsin x (1 + cos x) + c
[22 April 2019, Shift-II]
x8 x10
Sol. (a)
(a) +c (b) +c Given,
(1 + x + x )
3 5 2
(1 + x + x )
3 5 2

10 8 I n = ∫ sinn x dx = ∫ sinn −1 x.sin xdx


x x
(c) +c (d) +c
2(1 + x3 + x5 )2 2(1 + x3 + x5 )2 = − cos x sinn − 1 x + ∫ (n − 1)sinn − 2 x cos2 x dx
Indefinite Integrals 387

= − cos x sinn − 1 x + (n − 1) ∫ sinn − 2 x(1 − sin2 x) dx Sol. (a)


∫e [cos (3x + 4) + 5x 2] dx
2x
n −1 n−2
= − cos x ⋅ sin x + (n − 1) ∫ sin xdx − (n − 1)
= ∫ e 2x cos (3x + 4) dx + 5 ∫ e 2x x 2 dx
∫ sin
n
x dx
 e ax
= − cos x sinn − 1 x + (n − 1)I n − 2 − (n − 1) I n Q ∫ e cos (bx + c) dx = 2
ax

 a + b2
⇒ I n (1 + n − 1) = − cos x sinn − 1 x + (n − 1) I n − 2 
(a cos (bx + c) + b sin (bx + c)) + c
cos x sinn − 1 x (n − 1) 
⇒ In = − + In − 2
n n 5x 2 2x
= e 2x  cos (3x + 4) + sin (3x + 4) +
2 3
7
cos x sin x 7 e
⇒ I8 = − + I6 13 13  2
8 8
− 5∫ xe 2x dx
⇒ 8I 8 = − cos x sin7 x + 7I 6
⇒ 8I 8 − 7I 6 = − cos x sin7 x = e 2x  cos (3x + 4) +
2 3
sin (3x + 4)
 13 13 
⇒ 7I 7 = − cos x sin x + 6I 5
6
2
5x 2x 5 2x 5
⇒ 7I 7 − 6I 5 = − cos x sin6 x + e − xe +
2 2 2
∴ 8I 8 + 7(I 7 − I 6) − 6I 5 2x  2 3 
∫ = + + +
2x
= − cos x sin6 x (1 + sin x) + c e dx e cos (3x 4) sin (3 x 4)
13 13 
cos 4 x + 1 5 2 2x 5 2x 5 2x
+ x e − xe + e = e 2x
58. If ∫ dx = k cos 4 x + c, then k is 2 2 4
cot x − tan x
2 3 5x 2 5 5
[23 April 2019, Shift-I]  cos (3x + 4) + sin (3x + 4) + − x+ 
13 13 2 2 4 
1 1 1
(a) − (b) − (c) − (d) − 1 + c
2 4 8
Hence, option (a) is correct.
Sol. (c)
5 cot x + 1
cos 4 x + 1 2cos2 2x 60. If ∫ dx
∫ cot x − tan x dx = ∫ cos2 x − sin2 x × sin x cos x dx (cot x − 1) (cot x − 2) sin 2 x
cos2 2x = 6 log | f (x)| + 11 log | g(x)| + c,
=∫ sin 2x dx = ∫ sin 2x cos 2x dx then ( f (x), g(x)) = [23 April 2019, Shift-I]
cos 2x
1 1 (a) (cot x − 1, (cot x − 2 )−1 )
= ∫ sin 4 x dx = − cos 4 x + c
2 8 (b) ((cot x − 1)−1, cot x − 2 )
So, k=−
1 (c) ((cot x − 1)−1, (cot x − 2 )−1 )
8 (d) (cot x − 1, cot x + 2 )
Hence, option (c) is correct.
Sol. (a)
∫e [cos (3 x + 4) + 5 x 2 ] dx =
2x
59. 5 cot x + 1
[23 April 2019, Shift-I]
I= ∫ (cot x − 1) (cot x − 2) sin2 x dx
2 3 5x 2 5x 5  Let cot x = t ⇒ – cosec2 x dx = dt, so
(a) e 2x  cos( 3 x + 4 ) + sin ( 3 x + 4 ) + − + 
 13 13 2 2 4 5t + 1
I=−∫ dt
+c (t − 1) (t − 2)
2 3 5x 2 5x 5 
(b) e 2x
 cos( 3 x + 4 ) − sin ( 3 x + 4 ) + + +  5t + 1 A B
 13 13 2 2 4 Let = +
(t − 1) (t − 2) t − 1 t − 2
+c
2 3 5x 2 5x 5  ⇒ 5t + 1 = (A + B)t − (2A + B)
(c) e 2x  cos( 3 x + 4 ) − sin ( 3 x + 4 ) − − − 
 13 13 2 2 4 So, A + B = 5 and 2A + B = − 1
+c ⇒ A = − 6 and B = 11
2 3 5x 2 5x 5  dt dt
(d) e 2x
 cos( 3 x + 4 ) − sin ( 3 x + 4 ) + − +  ∴ I=6∫ − 11 ∫
 13 13 2 2 4 t −1 t−2
+c = 6 log |t − 1| − 11 log |t − 2| + c
388 AP EAMCET Chapterwise Mathematics

= 6 log |cot x − 1| + 11 log |(cot x − 2)−1| + c


2
 log x − 1 
63. ∫   dx =
On comparing, we get 1 + (log x)2  [22 April 2018, Shift-I]
( f (x), g(x)) = ((cot x − 1), (cot x − 2)−1) log x x
(a) +c (b) +c
Hence, option (a) is correct. 1 + (log x)2 x2 + 1
n −x
61. If I m,n = ∫ e mx ⋅ x ndx , then I m ,n + I m ,n−1 = (c)
x
+c (d) +c
m 1 + (log x)2 1 + (log x)2
[23 April 2019, Shift-I]
Sol. (c)
xne mx
n
(a) x . e mx
+c (b) +c We have,
n 2
n
x .e mx
− xn ⋅ e mx  log x − 1  (t − 1)2
(c) +c (d) +c I = ∫ 2
dx = ∫ e t 2 dt,
m m 1 + (log x)  (t + 1)2
Sol. (c) where t = log x
Q I m , n = ∫ e mx ⋅ x n dx t 2 + 1 − 2t t 1 − 2t 
= ∫e t
dt = ∫ e  2 + 2 2
dt
(t 2 + 1)2  t + 1 (t + 1) 
1 n mx  e mx 
= x e − ∫ (nx n − 1)   dx
 m  et x
m = + c= + c.
t +1
2
(log x)2 + 1
x n e mx n
e mx x n − 1 dx
m∫
= −
m dx
=
x n e mx

n
I m ,n − 1 + c
64. ∫ x3 + 3x2 + 2x = [22 April 2018, Shift-I]
m m x+2
n x n e mx (a) log x + log +c
⇒ I m ,n + I m ,n −1 = + c x+1
m m
(b) log x − log x + 1 + log x + 2 + c
Hence, option (c) is correct.
1
(c) [log x + log x + 1 + log x + 2 ] + c
sin 2 xdx  π
∫ sin4 x + cos 4 x = tan
−1 2
62. ( f (x)) + c, then f  
 3 x2 + 2 x
1
= [22 April 2018, Shift-I] (d) log +c
2 ( x + 1)2
1
(a) 1 (b) 2 (c) 3 (d)
3 Sol. (d)
dx dx
Sol. (c) Let I= ∫ x 3 + 3x 2 + 2x = ∫ x(x 2 + 3x + 2)
sin 2x dx 2sin x cos x
Let I= ∫ sin4 x + cos4 x = ∫ sin4 x + cos4 x dx dx
2
= ∫ x(x + 1) (x + 2)
Dividing by cos x in numerator and
denominator, we get 1 A B C
Let = + +
2tan x sec2 x x(x + 1) (x + 2) x x +1 x + 2
=∫ dx
1 + (tan2 x)2 ⇒ 1 = A(x + 1) (x + 2) + Bx(x + 2) + Cx(x + 1)
Again let tan x = t 2 Put x = 0, we get
1
dt A=
∴ I= ∫ 1 + t2 2
Put x = − 1, we get
= tan− 1 t + C
B = −1
= tan− 1 (tan2 x) + C
Put x = − 2, we get
By comparing with tan− 1 f (x) + C, we get 1
C=
f (x) = tan2 x 2
2 1 1 1 
π π ∴ I = ∫
∴ f   = tan   = ( 3)2 = 3. − +  dx
 2x x + 1 2(x + 2) 
 3   3
Indefinite Integrals 389
1 1 Sol. (d)
= log x − log(x + 1) + log(x + 2) + C
2 2 x − x2 x(1 − x)
1
= [log x − 2log(x + 1) + log(x + 2)] + C
For x < 1, I = ∫ 1− x
dx = ∫
1− x
dx
2

=
1
log
x(x + 2)
+ C
= ∫x 1 − xdx
2 (x + 1)2 Let 1− x =t 2

1 x + 2x 2
⇒ dx = − 2tdt
= log + C.
2 (x + 1)2 So, I = ∫ (1 − t 2)t (−2t)dt = 2∫ (t 4 − t 2)dt
2
65. For n ≥ 2, If I n = ∫ sec n xdx , then I 4 − I 2 =  t5 t3 
3 ⇒ I = 2 −  + c
[22 April 2018, Shift-I] 5 3
1 2
(a) sec x tan x + c
2
(b) sec 2 x tan x + c ⇒ I = t 3 (3t 2 − 5) + c
3 15
2 1
(c) sec 2 x tan x + c (d) logsec x + tan x + c ⇒
2
I = (1 − x)3/ 2 [3 (1 − x) − 5] + c
3 2 15
Sol. (b) −2
⇒ I= (1 − x)3/ 2 (3x + 2) + c.
We have, 15
I n = ∫ secn x dx
dx
∫ (2sin x + sec x)4 = A(1 + tan x)
−5
67.
∴ I 2 = ∫ sec x dx = tan x + c1
2

+ B (1 + tan x)−6 + C(1 + tan x)−7 + k, then


and I 4 = ∫ sec4 x dx
A+ B+ C = [22 April 2018, Shift-II]
= ∫ sec2 x ⋅ sec2 x dx − 86 −1 −26 −16
(a) (b) (c) (d)
105 105 105 105
= ∫ (tan2 x + 1)sec2 x dx
Sol. (d)
tan3 x dx sec4 x
=
3
+ tan x + c 2 I= ∫ (2sin x + sec x)4 = ∫ (2tan x + sec2 x)4 dx

2 tan3 x 2 2c (1 + tan2 x) 2
∴ I4 −
3
I2 =
3
+ tan x + c 2 − tan x − 1
3 3
= ∫ (1 + tan x)8 sec x dx
1 1 Let tan x = t ⇒ sec2 x dx = dt
= tan3 x + tan x + c
3 3 1 + t2 (1 + t)2 − 2t
 where c − 2 c = c  So, I =∫ dt = ∫ dt
(1 + t) 8
(1 + t)8
 
2 1
3
dt 1 + t −1
1
= tan x(tan2 x + 1) + c
= ∫ (1 + t)6 − 2∫ (1 + t)8 dt
3
1 dt dt dt
= tan x sec2 x + c.
3
= ∫ (1 + t)6 − 2∫ (1 + t)7 + 2∫ (1 + t)8
x − x2 1 2 2
= − (1 + t)−5 + (1 + t)−6 − (1 + t)−7 + K
66. For x < 1, ∫ dx = 5 6 7
1− x [22 April 2018, Shift-II] −1 1
4 2 = (1 + tan x)−5 + (1 + tan x)−6
(a) (1 − x)3 / 2 − (1 − x)5 / 2 − 2 1 − x + c 5 3
3 5 2
4 2 − (1 + tan x)−7 + k
(b) (1 − x)3 / 2 − (1 − x)5 / 2 − 2 1 − x + c 7
3 3
−1 1 2
2
(c) (1 − x)3 / 2 − 2 1 − x + c So, A = , B = and C = −
3 5 3 7
2 16
(d) − (1 − x)3 / 2 (2 + 3 x) + c ∴A + B + C = − .
15 105
390 AP EAMCET Chapterwise Mathematics

2x2 − 1 + x2 x2 + 4 1 − 1 
 
68. ∫ x 2 (x 2 + 4)
dx = = ∫  1 2
 x2  dx
1
1 + 2 +  x 1 + x +
[22 April 2018, Shift-II]  x x x
9 x 1 x
(a) tan−1 + + cosh−1 + c 1− 2
1
8 2 4x 2
∫ 1 
= x dx
9 x 1 x
(b) tan−1 + + sinh−1 + c 1
8 2 4x 2  2 + + x 1 + x +
 x  x
9 x+2 1 x + x2 + 4
(c) log + + log +c 1 − 1 
16 x−2 4x 2  
 x2 
9 2− x 1 x
= ∫   1  1
dx
(d) log + + cosh−1 + c 1 +  x + + 1  1 + x +
16 2+ x 4x 2  x  x
1
Sol. (b) 1− 2
2x 2 − 1 + x 2 x 2 + 4 9
= ∫ x
2
dx
∫ x 2(x 2 + 4)
dx =
4
  1  
1 +  x + + 1  1 + x +
1

 
x x
dx 1 dx dx
⇒ ∫ x2 + 4 4 ∫ x2
− + ∫ x2 + 4 Let x+
1
+1 = t
x
tan−1   + + sinh−1   + c.
9 x 1 x
= 1 − 1 
8  2 4x  2  
 x 2  dx = dt

x −1 1
69. ∫ (x + 1) x (x 2 + x + 1)
dx = 2 x + +1
x
[23 April 2018, Shift-I] 2 dt
∴ 1=∫ = 2 tan−1 t + c
1 + t2
 x2 + x + 1 
(a) tan− 1  +c  1 
 x  = 2 tan−1  x + + 1  + c
   x 
 x2 + x + 1
(b) 2 ⋅ tan− 1   +c cos 3 x + cos 5 x
 x  70. ∫ sin2 x + sin4 x dx =
 x2 + x + 1 [23 April 2018, Shift-I]
(c) tan− 1   +c (a) sin x − 6 tan− 1(sin x) + c
 x 
(b) sin x − 2(sin x)− 1 + c
 1 
(d) 2 ⋅ tan− 1  x + + 1 + c (c) sin x − 2(sin x)− 1 − 6 tan− 1(sin x) + c
 x 
(d) sin x − 2(sin x)− 1 + 5 tan− 1(sin x) + c
Sol. (d)
x −1 Sol. (c)
Let, I = ∫ (x + 1) x (x 2 + x + 1)
dx
We have,
cos3 x + cos5 x
(x − 1) (x + 1) =∫ dx
= ∫ (x + 1)2 x (x 2 + x + 1)
dx sin2 x + sin4 x
cos2 x. cos x (1 + cos2 x)
x2 − 1 = ∫ sin2 x (1 + sin2 x)
dx
= ∫ 1
dx
(x + 1)2 x. 1 + x + (1 − sin2 x) (2 − sin2 x) cos x
x = ∫ sin2 x (1 + sin2 x)
dx
x2 − 1
Substitute sin x = t
x2
= ∫  x + 1 2 1
dx
⇒ cos x dx = dt
  . x 1+ x + (1 − t 2) (2 − t 2) 2 − 3t 2 + t 4
 x  =∫
x
t 2 (1 + t 2)
dt = ∫ t 2 (1 + t 2) dt
Indefinite Integrals 391

t 2 (1 + t 2) − 4 (t 2 + 1) + 6 =
1
[− cos x + sin x − x] + c
= ∫ t 2 (1 + t 2)
dt
2
1
 4 6  = [sin x − cos x − x] + c
= ∫ 1 − 2 + 2 2 
dt 2
 t t (t + t ) 
 π
 1 1 
72. If f (x) = ∫ cosec 5 x dx , then f   =
4
= 1 − 2 + 6  2 −   dx  4
 t t 1 + t2   [23 April 2018, Shift-I]
 2 6  (a) −
1
[3 2 − 5log( 2 + 1)] + c
= ∫ 1 + 2 −  dt
 t 1 + t2  4
1
(b) − [5 2 − 3log( 2 + 1)] + c
= t − 2t −1 − 6 tan−1 t + c 8
= sin x − 2 (sin x)−1 − 6 tan−1 (sin x) + c 1
(c) − [7 2 + 3log( 2 + 1)] + c
8
dx 1
71. ∫ tan x + cot x + sec x + cosec x = (d) [5 2 + log( 2 + 1)] + c
8
[23 April 2018, Shift-I] Sol. (c)
1 f (x) = ∫ cosec5 x dx = ∫ cosec2 x ⋅ cosec3 x dx
(a) (sin x − cos x + x) + c
2
1 = cosec3 x (− cos x) − ∫ − cot x ⋅ 3cosec2 x
(b) (sin x − cos x − tan x + cot x) + c
2 (− cosec x cot x) dx
1
(c) (sin x − cos x − x) + c = − cot x cosec x − 3∫ cot x cosec3 x dx
3 2

2
1 = − cot x cosec3 x − 3∫ (cosec2 x − 1) cosec3 x dx
(d) (sin x + cos x − tan x − cot x) + c
2
= − cot x cosec3 x − 3∫ cosec5 x dx
Sol. (c)
+ 3 ∫ cosec3 x dx
We have,
dx f (x) = − cot x cosec3 x − 3I + 3I1
∫ tan x + cot x + sec x + cosec x
4 f (x) = cot x cosec3 x + 3I1
dx
= ∫ sin x cos x 1 1
I1 = ∫ cosec3 x dx
+ + +
cos x sin x cos x sin x = ∫ cosec x ⋅ cosec2 x dx
sin x cos x dx
=∫ 2 = cosec x(− cot x) − ∫ − cot x ⋅ (− cosec x cot x) dx
sin x + cos2 x + sin x + cos x
sin x cos x dx = − cosec x cot x − ∫ cot 2 x cosec x dx
=∫
1 + sin x + cos x
= − cosec x cot x − ∫ (cosec2 x − 1) cosec x dx
multiply and divide by 2
1 2 sin x cos x = − cosec x cot x − ∫ cosec3 x dx + ∫ cosec x dx
= ∫ dx
2 1 + sin x + cos x 2I1 = − cosec x cot x + log(cosec x − cot x)
1 2 sin x cos x + 1 − 1  3 
1 − cot x cosec x + (− cosec x cot x 
3
= ∫ dx
2 1 + sin x + cos x f (x) =  2
4
1 (sin2 x + cos2 x + 2sin x cos x) − 1  + log(cosec x − cot x)
= ∫ dx
f(π / 4) =  − 2 2 + (− 2 + log( 2 − 1)
2 1 + sin x + cos x 1 3
4  2 
1 (sin x + cos x)2 − 1
2 ∫ 1 + sin x + cos x
= dx 1
= [− 4 2 − 3 2 + 3log( 2 − 1)]
8
1 (sin x + cos x + 1) (sin x + cos x − 1) 1
= ∫ dx = − [7 2 − 3log( 2 − 1)]
2 1 + sin x + cos x 8
1 1
= ∫ (sin x + cos x − 1) dx = − [7 2 + 3log( 2 + 1)]
2 8
392 AP EAMCET Chapterwise Mathematics

x 1 dx
Let 1 + = t2 ⇒ −2 = 2t dt
73. ∫ x 3 − 3 x + 2 dx = x2 x3
[23 April 2018, Shift-II]
So, I = − ∫ t 2 log t 2 dt
2 x−1 2 x+2
(a) log +c (b) log +c
9 x+2 9 x−1  t3  t3 1 
= − 2∫ t 2 log t dt = − 2 log t − ∫  ×  dt 
1 1 2 x−1 3  3 t 
(c) + log +c
3 x−1 9 x+2 2 3 2 3 1 3
= − t log t + t + c = t [2 − 3log t 2] + c
1 1 2 x−1 3 9 9
(d) − + log +c 3/ 2
1 1  1 
3 ( x − 1) 9 x+2 = 1 + 2  2 − 3log 1 + 2  + c
9 x    x  
Sol. (d)
dx
∫ sin x + sin2 x =
x x dx
∫ x3 − 3 x + 2 dx = ∫ ( x − 1)2 (x + 2 )
75.
[23 April 2018, Shift-II]
Now, by partial fraction method 1 1
(a) log(1 − cos x) + log(1 + cos x)
x A B C 6 2
= + + 2
(x − 1) (x + 2)
2
( x − 1) (x − 1) 2
( x + 2) + log 1 + 2 cos x + c
3
⇒ x = A(x − 1) (x + 2) + B(x + 2) + C(x − 1)2 1 1
(b) log(1 − cos x) − log(1 + cos x)
On comparing the coefficient of different terms, 6 2
2
we are getting
2 1 2 − log 1 + 2 cos x + c
A = , B = and C = − 3
9 3 9 1 1
x 2 dx 1 dx (c) log(1 − cos x) + log(1 + cos x)
So, ∫ 3 dx = ∫ + ∫ 6 2
x − 3x + 2 9 x − 1 3 (x − 1)2 2
2 dx − log 1 + 2 cos x + c
− ∫ 3
9 x+ 2 1
(d) log[(1 − cos x) (1 + cos x)|1 + 2 cos x|] + c
2 1 1 2
= log| x − 1 | − ⋅ − log| x + 2| + c 6
9 3 (x − 1) 9
Sol. (c)
1 1 2 x −1 dx
=− ⋅ + log + c
3 (x − 1) 9 x+ 2
I= ∫ sin x + sin 2x
dx sin x dx
x 2 + 1 [log(x 2 + 1) − 2 log x ] = ∫ sin x (1 + 2cos x) = ∫ sin2 x(1 + 2cos x)
74. ∫ x4
dx =
sin x dx
[23 April 2018, Shift-II] = ∫ (1 − cos x) (1 + cos x) (1 + 2cos x)
3
1 1 2  1  Let cos x = t ⇒ − sin x dx = dt
(a) 1 + 2  2 − 3log 1 + 2  + c
9 x    x   dt
 So, I=−∫
1 (1 − t) (1 + t) (1 + 2t)
1 2  1 
2
(b)  1 + 2   6 − log 1 +
1
 +c By partial fraction method
3 x    x2  
1 A B C
 1 = + +
(1 − t) (1 + t) (1 + 2t) 1 − t 1 + t 1 + 2t
(c)  1 + 2   3 − 2 log 1 +
1 1 1 2
 +c
9 x    x2   ⇒1 = A(1 + t) (1 + 2t) + B(1 − t)
 
3 (1 + 2t) + C(1 + t) (1 − t)
1 2  1  1 −1 4
3 + log 1 + 2  + c
1 A= , B= and C =
(d) 1 + 2  
3 x    x   6 2 3
1 dt 1 dt 4 dt
So, I = − ∫
6 1 − t 2 ∫ 1 + t 3 ∫ 1 + 2t
Sol. (a) + −

x 2 + 1 [log(x 2 + 1) − 2log x] 1 1
I= ∫ x4
dx = log(1 − cos x) + log(1 + cos x)
6 2
2
log 1 + 2  dx
1 1 1 − log(1 + 2cos x) + c.
= ∫ x3 1+
x2  x  3
Indefinite Integrals 393

76. ∫ (sec 4 x + tan 4 x) dx = 1 1 


2  ∫
[23 April 2018, Shift-II] = 1 ⋅ dx − ∫ dx
2 2 1 − 2x 
(a) tan3 x − tan x + x + c
=  x + log | 2x − 1 |
3 3 1 1
1 5 tan3 x 2  2 
(b) sec 2 x tan x + tan x + + x+c
3 3 3 x 1
= + log e | 2x − 1 | + C.
2 2 4
(c) tan3 x + x + c
3
x 8 − 9 x 2 + 18
1 5
(d) sec 2 x tan x − tan x +
tan3 x
+ x+c
78. ∫ x4 − 3x2 + 3
dx =
3 3 3 [24 April 2018, Shift-I]
Sol. (c) x5 x5 x4
(a) + x3 + 6 x2 + c (b) + + 6x + c
I = ∫ (sec4 x + tan4 x)dx 4 5 4
x5 x 5
x 3
(c) + x3 + 6 x + c (d) − + 6 x2 + c
= ∫ [sec4 x + (sec2 x − 1)2]dx 5 5 2
= ∫ (2sec4 x − 2sec2 x + 1) dx Sol. (c)
= ∫ [21
( + tan x)sec x − 2sec x + 1] dx
2 2 2 x 8 − 9 x 2 + 18
∫ x 4 − 3x 2 + 3
dx
2
= ∫ (2 tan2 xsec 2 x + 1) dx = tan3 x + x + c.
3 degree of numerator is greater than degree of
denominator so,
 t +1 x 8 − 9 x 2 + 18 is divided by x 4 − 3x 2 + 3
77. If g   = t + 1, then ∫ g(x) dx =
 2 t + 1 We get quotient x 4 + 3x 2 + 6 and 0 remainder,
[24 April 2018, Shift-I] so
x2 x 8 − 9 x 2 + 18
+c
(a)
∫ x 4 − 3x 2 + 3 dx = ∫ (x + 3x + 6) dx
4 2
2
1
(b) loge (2 x − 1) + loge | ( x + 1)| + c x5 3 x3 x5
2 = + + 6x + c = + x3 + 6 x + c.
 x+ 1 5 3 5
1
(c) loge   +c
2  2 x + 1 x 4 +1
x 1
79. ∫ 1 + x 6 dx =
(d) + loge | 2 x − 1| + c [24 April 2018, Shift-I]
2 4
(a) tan−1 ( x3 ) + tan−1 x + c
Sol. (d) 1
(b) tan−1 x + tan−1 x3 + c
 t +1 3
g  = t +1 (c) 3 tan−1 x3 + tan−1 x + c
 2t + 1 
1
t +1 (d) tan−1 x + tan−1 x3 + c
replacing by x, we get, 3
2t + 1
t +1 Sol. (d)
= x ⇒ t + 1 = 2tx + x
2t + 1 x4 + 1 x4 − x2 + 1 + x2
x −1
∫1 + x6
dx = ∫ 1 + x6
dx
t(1 − 2x) = x − 1 ⇒ t =
1 − 2x x4 − x2 + 1 x2
x −1 x − 1 + 1 − 2x
= ∫ 1+ x 6
dx + ∫
1 + x6
dx
g(x) = + 1=
1 − 2x 1 − 2x I1 I2
− x x4 − x2 + 1 x4 − x2 + 1
g(x) = I1 = ∫ dx = ∫ dx
1 − 2x 1 + (x 2)3 (1 + x 2) (x 4 − x 2 + 1)
− x =∫
1
dx = tan− 1 x
∴ ∫ g(x) dx = ∫ 1 − 2x dx 1 + x2
1 1 − 2x − 1 x2 x2
= ∫
2 1 − 2x
dx I2 = ∫1 + x 6
dx = ∫
1 + (x 3)2
dx
394 AP EAMCET Chapterwise Mathematics

Let x 3 = t x6 1
dt = (log x)3 − ∫ (log x)2⋅ x 5 dx
3x dx = dt ⇒ x dx =
2 2
6 2 I II
3
x6 1
I2 = ∫
1 dt 1
= tan− 1 ()t = (log x)3 −
3 1 + t2 3 6 2
(log x)2 x 5 dx −  d (log x)2 x 5 dx  dx
Replacing t,
1  ∫ ∫  dx ∫ 
I 2 = tan− 1 (x 3)
3
x6 1 1 2log x x 6
1 = (log x)3 − x 6(log x)2 + ∫ ⋅ dx
∴ I1 + I 2 = tan− 1 x + tan− 1 (x 3) + C. 6 12 2 x 6
3
x6 x6 1
80. ∫ (log x)3 x 5dx = = (log x)3 − (log x)2 + ∫ (log x)⋅ x 5 dx
[24 April 2018, Shift-I] 6 12 6 I II

 (log x)3 1 1 1 x6 x6
(a) x6  − (log x)2 + log x −  +c = (log x)3 − (log x)2
 12 6 6 36  6 12
+  log x ∫ x 5 dx − ∫  (log x) ∫ x 5dx  dx
 (log x)3 1 d
1 log x 1
(b) x6  − (log x)2 + −  +c 6   dx 
 6 18 12 36 
x6 x6 1 x6
6  (log x) 1 = (log x)3 − (log x)2 + (log x) ⋅
3
1 log x
(c) x  + (log x) −
2
+  +c 6 12 6 6
 6 12 12 36 
1 1 x6
 (log x)3 (log x)2 log x 1  − ∫ ⋅ dx
(d) x6  − + −  +c
6 x 6
 6 12 36 216  x6 x6 x6 1
36 ∫
= (log x)3 − (log x)2 + log x − x 5 dx
Sol. (d) 6 12 36
x6 x6 x6
∫ (logI x)
3
x 5 dx = (log x)3 − (log x)2 + (log x)
II
6 12 36
Using by parts 1  x6 
d −   + c
= (log x)3 ∫ x dx − ∫  dx (log x) ∫ x
5 3 5
dx  dx 36  6 

 (log x)3 (log x)2 log x 1 
x6 3⋅ (log x)2 x 6 = x6  − + −  + c.
= (log x) ⋅ − ∫ x ⋅ 6 dx
3
 6 12 36 216 
6
27
Definite Integrals
and Its Applications
e2 dx Sol. (c)
1. ∫1 x (1 + log x) 2
= [17 Sep. 2020, Shift-I] π
∫− π x (sin x)dx
2

2 1 3
(a) (b) (c) (d) log 2 x 2(sin x) is an odd function
3 3 2
π
∫− π x (sin x) dx = 0
2
Sol. (a) So,
e2 dx
I= ∫1  2x 
1
x(1 + log x)2 4. ∫ sin − 1   dx = [17 Sep. 2020, Shift-II]
1 + x 2 
Let 1 + log x = t 0
1 (a) π − log 2 (b) π + log 2
dx = dt π π
x (c) − log 2 (d) + log 2
3 2 2
3dt  t −1   3−1 1−1 
⇒ I= ∫1 t 2 =  −1  = 
 −1
+
1 
 Sol. (c)
−1  2x 
1
1
1
I = − +1 =
2 I= ∫0 sin 
1 + x 
2
 dx
3 3
When, x = tanθ,
2. The value of x that satisfies the equation 2x 2tanθ
= = sin 2θ
x dt π 1 + x 2 1 + tan2 θ
∫ 2
t t −1 2
=
12
is [17 Sep. 2020, Shift-I]
|| Also, dx = sec2 θ d θ
(a) 1 (b) 0 (c) − 2 (d) 2 and, x = 0, θ = 0
π
Sol. (d) When x = 1, θ =
4
x dt π
∫ 2
|t| t 2 − 1
=
12 So, I=
π /4
∫0 sin−1 (sin 2θ) ⋅sec2 θ. d θ
π π π /4 π /4
⇒ [sec−1 t]x 2 =
12
⇒ sec−1 x − sec−1 2 =
12
= ∫0 2θ ⋅ sec2 θ dθ = 2∫
0
θ ⋅ sec2 θ ⋅ d θ

π π π π By parts we get,
⇒ sec−1 x= + = ⇒ x = sec  = 2 π
12 4 3  3 I = 2[θ,tanθ − log secθ] π0 / 4 ⇒ I = − log 2
2
π
3. ∫− π x 2 (sin x) dx = [17 Sep. 2020, Shift-I] 1
8 log(1 + x)
π 2
5. ∫ 1 + x2
dx = [17 Sep. 2020, Shift-II]
(a) π 2 (b) 0
2 π π
(a) π log 2 (b) log 2 (c) log 2 (d) log 2
(c) 0 (d) 2 π 2 2 4
396 AP EAMCET Chapterwise Mathematics

Sol. (a) 1 (1 + t 4)t dt 1


I= ∫0 = ∫0 (1 + t ) dt
4
So,
1 8 log (1 + x) 1 log (1 + x) t
I= ∫0 1 + x2
dx = 8∫
0 1 + x2
. dx
1
 t5  1 6
= t +  = 1 + =
Let x = tanθ dx = sec θ dθ 2
 5 0 5 5
When x = 0, θ = 0 2
and when x = 1 , θ =
π 8. ∫−1| x | dx = [18 Sep. 2020, Shift-I]
4 5 3
π / 4 log (1 + tan θ) (a) 1 (b) 2 (c) (d)
So, I = 8∫ ⋅ sec2 θ. dθ 2 2
0 1 + tan2 θ
π /4
Sol. (c)
= 8∫ log (1 + tanθ) dθ 2 0 2
0 I= ∫−1|x| dx = ∫−1 (− x) dx + ∫0 (x) dx
Now using 0 2
 − x2   x2   1  4  5
a a
=  +   =  0 +  +  − 0 =
I= ∫0 f (x) dx = ∫0 f (a − x) dx  2  −1  2  0 2 2 2
and adding both equations we have,
π /4 9. Find the value of ‘k’, if it is given that
2I = 8∫ log 2dx b−c b

∫ f (x + c)dx = k ∫ f (x) dx
0
[18 Sep. 2020, Shift-I]
π
I=8× log 2 = π log 2 0 c
8
π /4
(a) 1 (b) 2 (c) 0 (d) − 2
∫− π / 4 x sin 4 (x) dx =
3
6. [17 Sep. 2020, Shift-II] Sol. (a)
b− c
(b) π (d) 2 π
(a) 0 (c) 1 I= ∫0 f (x + c)dx
Sol. (a)
π /4 Put x + c = t, then at x = 0, t = c and
I= ∫− π / 4 x ⋅ sin4 x dx
3
at x = b − c , t = b and dx = dt, so
a b− c
∫− a f (x)dx = 0, when
b b b
As, I= ∫c f ()t dt = ∫c f (x)dx ⇒ ∫0 f (x + c)dx = ∫c f (x)dx
f (− x) = − f (x),
Therefore k = 1
We have,
π / 2 sin x
f (− x) = (− x)3 sin4 (− x) 10. ∫0 e ⋅ cos x dx =
[18 Sep. 2020, Shift-I]
= − (x ⋅ sin x) = − f (x)
3 4
(a) 1 − e (b) 1 + e (c) e − 1 (d) e
π /4
∴ ∫− π / 4 x ⋅ sin x. dx = 0.
3 4 Sol. (c)
π / 2 sin x
π /4 dx I= ∫0 e cos x dx
7. ∫0 cos 3 (x) ⋅ 2 sin(2 x)
=
π
[18 Sep. 2020, Shift-I] Put sin x = t, then at x = 0, t = 0 and at x = , t =1
6 3 4 8 2
(a) (b) (c) (d) and cos x dx = dt
5 5 5 5 1 t
I= ∫0 e dt = [e ]0
t 1
So,
Sol. (a)
π/4 π/4
dx dx = e1 − 1 = e − 1
I= ∫ 3
cos x 2sin 2x
= ∫ 3
cos x 4 sin x cos x π /2

π/4
0 0
π/4
11. If I n= ∫0 sin n(x) dx and I n = (k)I n − 2 , then
dx 1 sec2 x(1 + tan2 x)
= ∫ 4
=
2cos x tan x 2 ∫ tan x
dx what will be the value of k?[18 Sep. 2020, Shift-II]
n n−1 n+1 n
0 0
(a) (b) (c) (d)
Let, tan x = t , then at x = 0, t = 0 and at
2 n−1 n n n+1
π
x = ,t = 1 Sol. (b)
4 π/2
and sec2 x dx = 2 t dt In = ∫0 sinx ⋅sinn –1 x dx
Definite Integrals and Its Applications 397
π /2
= sinn –1 x (− cos x)]0π / 2 + ∫0 (n − 1). sinn – 2 x Sol. (c)
k k
2
.cos x dx ∫0 ( k − t) dt = ∫ (k + t − 2 k t) dt
0
π /2 k
= 0 + (n − 1) ∫ sinn – 2 x (1 − sin2 x) dx  t2 
k
 t 12 +1 
0 =  kt +  − 2 k  1 
π /2 π /2  20  2 +1 
I n = (n − 1) ∫ sinn – 2 x dx − (n − 1) ∫ sinnx dx  0
0 0
 k2  4 k 32 k
nI n = (n − 1). I n – 2 =  k⋅k +  − (k )0
n –1  2 3
In = . In – 2
n 3k2 4 k 3/ 2 k2
− =
.k =
Hence, option (b) is correct. 2 3 6
π /4 Hence, option (c) correct.
12. ∫0 (tan 2 x − tan 4 x) dx = π /4
∫0
[18 Sep. 2020, Shift-II]
15. tan 2 (x) dx =
1 [21 Sep. 2020, Shift-I]
(a) 3 (b) 2 (c) (d) 0
π π π
3 (a) (b) − 1 (c) 1 − (d) 0
4 4 4
Sol. (c)
π Sol. (c)
∫04 (tan x + tan4 x) dx
2
π /4 π/2

π
I= ∫0 tan2 x dx = ∫0 (sec2 x − 1)dx

∫ 0
4 (1 + tan 2 x) tan 2 x dx π
= [tan x − x]0π / 4 = 1 −
π 4
∫ 0
4 sec2 x . tan 2 x dx Hence, option (c) is correct.
π /2
∫0
Put, tan x = t 16. |sin t − cos t|dt =
2 [21 Sep. 2020, Shift-II]
sec x dx = dt
π (a) 2 ( 2 + 1) (b) 2 ( 2 − 1)
when x = , then t = 1 and when x = 0, then t = 0
4 (c) 2 + 1 (d) 2 − 1
1
 t3  Sol. (b)
t . dt =   =  − 0 =
1 2 1 1
= ∫0  0
3  3  3
π/2 π/4
I= ∫|sin t − cos t|dt = 2 ∫ (cos t − sin t)dx
Hence, option (c) is correct. 0 0

 2a a

13. If f is integrable on [0 , a], then the function h Q ∫ f (x)dx = 2∫ f (x)dx if f (2a − x) = f (x)
defined on [0 , a] as h(x) = ....∀ x ∈[0 , a] is  0 0 
integrable on [0 , a] [18 Sep. 2020, Shift-II] = 2[sin t + cos t]π0 / 4
(a) f(a − x) (b) f( x − a) (c) f( x) (d)f(a)
= 2 − 0 − 1 = 2( 2 − 1)
1 1
+
Sol. (a)  2 2 
Given, f is integrable an [0, a] Hence, option (b) is correct.
a
∴ h(x) = ∫0 f (x) dx π /2 1
a
17. ∫0 1 + tan 2020 (x)
dx = [22 Sep. 2020, Shift-I]
h(x) = ∫0 f (a − x) dx
π π
Hence, option (a) is correct. (a) π (b) (c) (d) 0
2 4
k
14. ∫ ( k − t)2 dt = Sol. (c)
π/2
[18 Sep. 2020, Shift-II] dx

0
I=
k 2
k 2
0
1 + tan2020 x
(a) (b)
2 4 π/2
cos2020 x
(c)
k2
(d)
k2 = ∫ cos2020 x + sin2020 x
dx ...(i)
0
6 8
398 AP EAMCET Chapterwise Mathematics

a a 1
On applying property ∫ f (x) dx = ∫ f (a − x) dx, we get 20. ∫0 (1 + x) log(1 + x) dx = [22 Sep. 2020, Shift-I]
0 0 −3 3
π/2 (a) + log 2 (b) + 2 log 2
sin2020 x 4 4
I= ∫ sin 2020
x + cos2020 x
dx ...(ii)
(c) 2 log 2 (d)
−3
+ 2 log 2
0
4
On adding Eqs. (i) and (ii), we get
π/2
π π
Sol. (d)
2I = ∫ dx = 2
⇒I =
4
1
I = ∫ (1 + x) log(1 + x) dx
0
0
Hence, option (c) is correct.
 x2 
π /2 x +1 
∫− π / 2(2sin| x| + cos| x|) dx =   2 1
18. x  2
2   0 ∫0 1 + x
[22 Sep. 2020, Shift-I] = (log(1 + x))  x +  − dx
 
(a) 3 (b) 6 (c) 8 (d) 2
1  1 
1
3
Sol. (b) = log e 2 − ∫ (x + 1) − dx
π/2 2 2 0 1 + x 
I= ∫ (2 sin| x | + cos | x |) dx 3 1  x2 
1
− π/2
= log e 2 −  + x − log e (1 + x)
π/2 2 2 2 0
=2 ∫ (2 sin x + cos x) dx 3
= log e 2 −
1 1
+ 1 − log 2
2  2 
0
(Q 2sin | x | + cos | x | is even function) 2
3 3
= 2[− 2 cos x + sin x]0π / 2 = 2 (loge 2 ) − = − + 2 loge 2
4 4
= 2[(− 2 × 0) + 1 − (−2 × 1) + 0]
= 21
[ + 2] = 6 21. Choose the correct option regarding the
following definite integrals
2π x cos(x)
19. ∫0 1 + cos(x)
dx =
[22 Sep. 2020, Shift-I] π /2 1
[22 Sep. 2020, Shift-II]

π π (i) ∫ sin (x) cos(x) dx =


m
(a) (b) π 2 (c) (d) 2 π 2 0 m +1
6 4
π /2 1
Sol. (*) (ii) ∫ sin(x) cos n(x) dx =

0 n +1
x cos x (a) (i) is true, (ii) is false
I= ∫ 1 + cos x dx ...(i)
(b) (i) is false, (ii) is true
0
a a (c) Both (i) and (ii) are false
On applying property ∫ f (x)dx = ∫ f (a − x)dx, (d) Both (i) and (ii) are true
0 0
Sol. (d)
we get π/2

(2π − x) cos x (i) ∫ sinm x. cos x dx
I= ∫ dx ...(ii) 0

0
1 + cos x Put, sin x = t
π/2 m
{Q cos(2π − x) = cos x}
On adding Eqs. (i) and (ii), we get
cos x dx = dt = ∫0 t dt
π/2
2π t m + 1 π / 2
 (sin x)m + 1 
2I = 2π ∫
cos x
dx =  = 
1 + cos x  m +1 0  m + 1 0
0
1
π
cos x
π
 1  = [(sin x)m + 1 ]π0 / 2
⇒ I = 2π ∫ 1 + cos x dx = 2π ∫ 1 − 1 + cos x  dx m+1
0 0
1  m + 1 π m+1 
 = sin   − sin .(0)
sec2  m + 1  
x  2
π π
= 2π ∫ 1 − 2  dx = 2π  x − tan x 
 1 1
0
2  2  0 = [()1 − (0)] =
  m+1 m+1
Definite Integrals and Its Applications 399
π/2
(ii) ∫ sin x. cosn (x) = dx 23. The value of f ()
1 , given the equation
0
x2
cos x = t
− sin x dx = dt
∫0 t dt = x 5 − x 3 is
xf () [22 Sep. 2020, Shift-II]

sin x. dx = − dt (a) 4 (b) 3 (c) 2 (d) 1


π/2 n π/2 n Sol. (d)
∫0 t .(− dt) = − ∫0 t . dt
x2

t n + 1 π / 2  cos x n + 1 π / 2 ∫0 t dt = x 5 − x 3
x. f ()
=−  =−  Differentiate w.r.t. x on both sides,
 n + 1 0  n + 1 0
d 2
−1 x 2. f (x 2). x − 0 = 5x 4 − 3x 2
= [cosn + 1 x]π0 / 2 dx
n+1
x 2 f (x 2) (2x) = 5x 4 −3x 2
−1  n + 1 π
= cos − cosn + 1 (0) 2x 3 f (x 2) = 5x 4 − 3x 2
n + 1  2 
Put x = 1
−1 1 2⋅1 ⋅ f() 1 = 5() 1 − 3()1 ⇒ f() 1 =1
= [0 − 1] =
n+1 n+1 Hence, option (d) is correct.
Hence, option (d) is correct. π /2x 1 tan −1 x
π /3 1 24. If I1 =∫0 sin x dx , and I 2 = ∫0 x dx ,
22. ∫π / 6 1 + cot x
dx =
then I1 : I 2 is [22 Sep. 2020, Shift-II]
[22 Sep. 2020, Shift-II]
π π π π (a) 1 : 1 (b) 2 : 1 (c) 3 : 1 (d) 4 : 1
(a) (b) (c) (d)
12 6 4 13 Sol. (b)
Sol. (a) π/2 x
π/3 1
Given, I1 = ∫0 sin x
dx
∫π / 61 + cot x dx = I (say)
1 tan −1 π/4
x t
π/3 cos x
I2 = ∫0 x
dx = ∫0 tan t
. sec2 t dt
I= ∫π / 6 sin x + cos x
dx …(i)
Put, tan−1 x = t
b b
We have, ∫ f (x) dx = ∫ f (a + b − x) dx x = tan t
a a
π dx = sec2 t dt
a+ b−x= − x
2 π
U. L →
π
cos  − x 
4
π/3 2  L. L → 0
I =∫ dx π/4 π/4
π/6 π π cos t 1 t
sin  − x  + cos  − x  = ∫0 t.
.
sin t cos t2
dt = ∫
0 sin t.cos t
dt
2  2 
π/4 2t π / 4 2t
π/3 sin x =∫ dt = ∫ dt
I= ∫π / 6 sin x + cos x
dx … (ii) 0 2sin t.cos t 0 sin 2t
Now, put 2t = x
i + (ii) ⇒ 2I
Eqs. ()
π/3  2dt = dx
cos x sin x
=∫  +  dx dx
π / 6  sin x + sin x + cos x  dt =
 cos x
2
π / 3 sin x + cos x  π
U. L →
= ∫π / 6  sin x +
 dx
cos x  2
π π L. L → 0
= (x)ππ // 36 =  − 
π/3
= ∫π / 6 1. dx  3 6 = 2∫
π/2 x
dx
0 sin x
π π
2I = ⇒ I=
6 12 I 2 = 2⋅ I1 ⇒ I1 : I 2 = 2 : 1
Hence, option (a) is correct. Hence, option (b) is correct.
400 AP EAMCET Chapterwise Mathematics

π /2 x + sin x 2 1 
25. ∫0 1 + cos x
dx = [23 Sep. 2020, Shift-I] Required area = 2  ∫ 2 − y dy + ∫ y dy
 1 0 
π π π π
(a) (b) (c) (d)   (2 − y)3/ 2  2  y 3/ 2 1 
4 3 2 6 = 2   +   
  −3 / 2  1  3 / 2  0 
Sol. (c)  
π /2 π /2
x + sin x  x sin x   0 − 1 1 − 0  4 8
=2 + = 2  =
∫ 1 + cos x
dx = ∫  +  dx
 1 + cos x 1 + cos x 
 −3 / 2 3 / 2   3 3
0 0
x x Hence, option (c) is correct.
π /2 2 sin .cos
x
= ∫ x
+ 2
x
2 27. If the value of
π /2 π
2cos2 2cos2
0
2 2 ∫0 sin 4 (x) ⋅ cos 2 (x) dx =
32
, then the value
π /2 π /2
= ∫
1 x
x sec2 + tan
x
of ∫ cos 4 (x) ⋅ sin 2 (x)dx = [23 Sep. 2020, Shift-I]
2 2 2 0
0
π π π π
π /2 (a) (b) (c) (d)
 x sec2 x + tan x  dx 32 64 4 8
= ∫ 
2 2 2

0 Sol. (a)
π /2
We have ∫ (x. f ′(x) + f (x)) dx = x f (x) + C π
π /2
Given, ∫ sin4 x ⋅ cos2 xdx =
32
π π π π
=  x tan  =  .tan − 0 = (1) =
x 0

 2 0 2  2 a
We have, ∫ f (x) dx =
a
4 2
0 ∫0 f (a − x) dx
Hence, option (c) is correct.
π π π
sin4  − x  ⋅ cos2  − x  =
π /2

26. Calculate the area enclosed by the curves


∴ ∫0 2  2  32
x 2 = 2 − y, x 2 = y π /2 π
∫0 cos x.sin x = 32
[23 Sep. 2020, Shift I] 4 2

2 4
(a) (b)
3 3 Hence, option (a) is correct.
8 11
(c) (d) a a− x
3 3 28. ∫− a a+ x
dx = [23 Sep. 2020, Shift-I]
Sol. (c) aπ
Given, curves are x 2 = y …(i) (a) (b) 1 (c) 2aπ (d) aπ
2
x2 = 2− y …(ii) Sol. (d)
a
From Eqs. (i) and (ii), a− x
y=x 2
Let I = ∫ a+ x
dx …(i)
−a

a a − (− a + a − x)
(0, 2)
= ∫− a a + (− a + a − x)
dx

B
 b b 
A Q ∫ f (x)dx = ∫ f (a + b − x) dx 
 a a 
–√2 O √2
a
a+ x
y=2x2
I= ∫ a− x
dx …(ii)
−a

y = 2− y
a
 a− x a+ x
Eqs. (i) + (ii) ⇒ 2I = ∫ a+ x
+  dx
a− x
⇒ 2y = 2 ⇒ y = 1 −a
a
From Eq. (i), x2 = 1 ⇒ x = ± 1 (a − x) + (a + x)
∴ Point of intersection are A(−1, 1), B(1, 1)
= ∫ (a + x) (a − x)
dx
−a
Definite Integrals and Its Applications 401
a π
2a at x = 1,θ =
= ∫ a −x
2 2
dx
4
−a
and dx = sec2θ dθ
 −a 
= 2a sin−1   − sin−1  
a
π
  a  a   log e (1 + tanθ)
−1 −1
Now, I = ∫04 sec2θ
sec2θ dθ
= 2a[sin ()
1 − sin (−1)]
π
 π  −π 4
= 2a −
 2  2   = ∫ log e (1 + tanθ) dθ …(i)
0
2I = 2a(π) a a
I = πa On, applying property, ∫ f (x) dx = ∫ f (a − x)dx,
Hence, option (d) is correct. 0 0
we get
x dt π
29. ∫log = ⇒x= π
 π 
e 2
e −1
t 6
[20 April 2019, Shift-I]
I= ∫04 log e 1 + tan  4 − θ  dθ
(a) 2 ⋅ loge 2 (b) 3 ⋅ loge 2 (c) 4 ⋅ loge 2 (d) 8 ⋅ loge 2 π
 1 − tanθ 
Sol. (a) = ∫04 log e 1 + 1 + tanθ  dθ
Given integral π
 2 
= ∫04 log e 1 + tanθ  dθ
x
dtπ
I= ∫ =
e −1 6
t
(given)
loge 2 π π

x −
t = ∫04 log e (2) dx − ∫04 log e (1 + tanθ) dθ
e 2 π
⇒ ∫ −t / 2
2
dt =
6 ⇒
π
I=log e (2) − I [from Eq. (i)]
loge 2 1 − (e ) 4
π π
put e − t / 2 = u, at t = log e 2, u =
1
and at t = x, ⇒ 2I = log e (2) ⇒ I = log e (2)
2 4 8
Hence, option (d) is correct.
u = e − x / 2 and e − t / 2dt = − 2 du
e−x /2
31. If the area of the circle x 2 + y 2 = 2 is divided
−2 du π
so, I= ∫ 1 = into two parts by the parabola y = x 2 , then
2 1−u 2 6 the area (in sq units) of the larger part is

x [20 April 2019, Shift-I]
e 2
π π π 3π 1 4 4π 2 1
−1
= ⇒sin−1 (e − x / 2) − = − − (b) 6 π − − (d) 4 π −
⇒ [−2 sin u] ∫ 6 4 12
(a)
2 3 3
(c)
3 3 4
1
2 Sol. (a)
−1 π−x / 2 π 1
⇒ sin (e ) = ⇒ e − x / 2 = sin = Given equation of curves
6 6 2 x 2 + y 2 = 2 and y = x 2

x  1
− = log e   = − log e (2) ⇒ x = 2⋅ log e (2). Y
y=x 2
2  2
Hence, option (a) is correct.
1 log e (1 + x) P Q
30. ∫0 1+ x 2
dx =
[20 April 2019, Shift-I] X′ X
π π π π O
(a) loge 2 (b) loge 6 (c) loge 8 (d) loge 2
4 6 2 8 x2+y2=2
Sol. (d)
Given integral
Y′
1 log + x)
e (1
I= ∫0 1 + x2
dx For point of intersection, on solving the given
curves, we get
put x = tanθ, so at x = 0, θ = 0 and y 2 + y − 2 = 0 ⇒ y 2 + 2y − y − 2 = 0
402 AP EAMCET Chapterwise Mathematics

⇒ y(y + 2) − 1(y + 2) = 0 0 2
 −t2 
0
 t2 
2

⇒ y =1 [Q y > 0] = − ∫ t dt + ∫ t dt = 
 +  
−1 0
2  −1  2  0
So, the required area
1 1 1 5
1 = − (0 − 1) + (4 − 0) = + 2 =
= π + 2∫ ( 2 − y 2) − y) dy 2 2 2 2
0
1 Hence, option (d) is correct.
= π + 2 2 − y 2 + sin−1 − y 3/ 2 
y 2 y 2
 2 2 2 3  0 34. The area (in square units) bounded by
= π + 2 +
1 π 2 3π
− = +1−
4 y = tan −1 x , y = cot −1 x and the Y -axis, is
 2 4 3 2 3 [20 April 2019, Shift-II]
3π 1 (a) loge 4 (b) loge 2 (c) loge 3 (d) loge 5
= − sq. units
2 3
Sol. (b)
Hence, option (a) is correct. On drawing the graph of y = tan−1 x and
1 n n 2
n 2 2  y = cot −1 x
 + + +  Y
32. lim  n (n + 1) (n + 2) (n + 3)3
3 3
=
n→ ∞  
1
+ … + 125 n 
[20 April 2019,, Shift-II] π y=tan–1x
3 15 12 35 2
(a) (b) (c) (d) π
8 32 25 72 4 y=cot–1x
X′ X
Sol. (c) O
Given,
1 n2 n2 n2 1 
lim  + + + +… 
n→ ∞ n
 (n + 1) 3
(n + 2) 3
(n + 3) 3
125 n
 n2 n2 n2 n2 
 + + + 3 
(n + 0) 3
(n + 1 ) 3
(n + 2) 3
(n + )
3
= lim   Y′
n→ ∞  n2 
+ … +  π 1 
 (n + 4n) 3
 The required area = 2 − ∫ tan−1 x dx 
 4 0 
4n
n2
= lim Σ , 1
r = 0 (n + r) 3 π
n→ ∞
= − 2 ∫ tan−1 x dx
4 2
r 1
4
dx  (1 + x)−2  0
Let = x and = dx = ∫ =   1 
π x
(1 + x)  −2  0
3
n n 0 = − 2 x tan−1 x)10 − ∫ dx 
2  1+ x 2

= −  2 − 1 = 1 −  = .
1 1 1 1 12 0

2  5  2  25 25 π
= −2  π 1 2 1
− [log e (1 + x )]0
2  4 2 
Hence, option (c) is correct.
π π
e2 log x = − + [log e (1 + x 2)]10 = log e 2
33. ∫e −1
x
dx =
[20 April 2019, Shift-II]
2 2
Hence, option (b) is correct.
2 5 1/ n
(a) (b) 2 (c) 5 (d) 
5 2 1  22   n2  
35. If lim 1 + 2  1 + 2  ... 1 + 2   = k,
n→ ∞  n   n   n  
Sol. (d) 
e2
log x
1
 − log x  dx +
e2
 log x  dx then log k = [21 April 2019, Shift-I]
I= ∫
−1 x
dx = ∫
−1

 x 
 ∫  x 

(a) log 4 +
π
−1 (b) log2 +
π
+1
e e 1
2 2
dx π π
On putting log x = t ⇒ = dt (c) log2 + −2 (d) log2 + −1
x 2 2
Definite Integrals and Its Applications 403

Sol. (c) dt
⇒ tan x sec2 x dx =
2
Given,
1 ∞ dt 1

1/ n ∴ 2I = ∫ 2 = [tan− 1 t]∞0
1  22   n2  
k = lim  1 + 2  1 + 2  ... 1 + 2   2 0 t +1 2
n→ ∞  n   n   n 
 1 1 π
⇒ 2I = [tan− 1 (∞) − tan− 1 (0)] = ×
1/ n 2 2 2
 1   2 2
log k = lim log  1 + 2  1 + 2  ... 1 + 2  
2 n π π
  ⇒ 2I = ⇒ I=
n→ ∞
 n  n   n  4 8
1 ν  r2  1
37. The curve y = ax 2 + bx passes through the
= lim Σ log 1 + 2  = ∫ log(1 + x 2) dx
n → ∞ n r =1  n  0
point (1, 2) and lies above the X -axis for
= [log(1 + x ) ⋅ x]0 − ∫
2 1 1 2x
⋅ x dx 0 ≤ x ≤ 8 . If the area enclosed by this curve,
01 + x 2 the X -axis and the line x = 6 is 108 square
1 x2 1 1  units, then 2b − a = [21 April 2019, Shift-I]
= log 2 − 2∫ dx = log 2 − 2∫ 1 −  dx
01 + x 2 0 1 + x2  (a) 2 (b) 0 (c) 1 (d) − 1

= log 2 − 2[x − tan −1


x]10 Sol. (b)
π π π Given, curve y = ax 2 + bx passes through
= log 2 − 21 −  = log 2 − 2 + = log 2 + −2
 4  2 2 point (1, 2).
∴ 2 = a()
1 2 + b()
1
π /2 sin 3 x cos x dx
36. ∫0 sin 4 x + cos 4 x
= ⇒ a+ b=2
6
… (i)
[21 April 2019, Shift-I]
π π π
Given, area under curve = ∫0 (ax 2 + bx) dx = 108
(a) π (b) (c) (d)
2 4 8  ax 3 bx 2 6
⇒  +  = 108
Sol. (d)  3 2 0
π /2 sin3 x cos x dx ⇒ 72a + 18b = 108
Let I= ∫0 sin4 x + cos4 x
…(i)
⇒ 4a + b = 6 …(ii)
π π By solving Eqs. (i) and (ii), we get
sin3  − x  cos − x  4 2
π /2 2  2  a = and b =
= ∫0 π π
dx 3 3
sin4  − x  + cos4  − x  2 4 4 4
2  2  ∴ 2b − a = 2 × − = − = 0
3 3 3 3
π /2 cos3 x sin x
I= ∫0 cos4 x + sin4 x
dx …(ii)
38. lim
1 + 2 +…+ n
= [21 April 2019, Shift-II]
n→ ∞ n3 / 2
Adding Eqs. (i) and (ii), we get
2
π / 2 (sin 3 x cos x + cos 3 x sin x) (a) 0 (b)
2I = ∫ dx 3
0 cos4 x + sin4 x 3
(c) 1 (d)
x + cos2 x)
π / 2 sin x cos x(sin 2 2
=∫ dx
0 cos x + sin4 x
4 Sol. (b)
π /2 sin x cos x Given,
=∫ dx
0 sin4 x + cos4 x 1+ 2+ …+ n
= lim
π /2 sin x cos x n→ ∞ n3/ 2
=∫ dx
0 cos4 x(tan4 x + 1) 1 + 2+ …+ n
= lim
n→ ∞ n⋅ n
π/2 tan x sec2 x
= ∫0 (tan4 x + 1)
dx
= lim 
1 1
+
2
+ …+
n
n→ ∞ n
 n n n 
Let tan2 x = t ν 1 r 1 2 3/ 2 1 2
Differentiating w.r.t. x, we get = Σ lim = ∫ xdx = [x ]0 =
r =1 n→ ∞ n n 0 3 3
2tan x sec2 x dx = dt
404 AP EAMCET Chapterwise Mathematics

∞ −x 1 1
39. ∫0 e sin 6 x dx = [21 April 2019, Shift-II] Required area = ∫0 ydx − 2 × OA × AP
24 124 136 144 1 1
= ∫0 (x + 2x + 1)dx − ×1 × 4
2
(a) (b) (c) (d)
85 285 529 629 2
1
 x3 
+ x 2 + x  − 2 =  + 1 + 1 − 0 − 2
Sol. (d) =
1
We know that,  3 0 3 
∞ −x 7 1
If Im = ∫0 e sinm xdx , where m > 2 = − 2=
3 3
m(m − 1)
then, Im = Im− 2 ∞ A4 − A6
1 + m2
∞ −x
41. If An = ∫π e − x . cos n x dx , then
A4
=
2
Now, let I6 = ∫0 e sin6 xdx
[22 April 2019, Shift-I]
6(6 − 1) 30 3 7 5 2
⇒ I6 = I4 = I4 (a) (b) (c) (d)
1 + 62 37 2 37 37 7
4(4 − 1) 12 Sol. (b)
also, I4 = , I2 = I2
1 + 42 17 We have,

2 (2 − 1) 2 e − x cos6 xdx
and I2 =
1 + 22
I0 = I0
5
A6 = ∫π
2
30 2 ∞
⇒ I6 = × 12 × I 0 = [− e − x cos6 x]∞π − ∫π ( − e
−x
) 6 cos5 x(− sin x)dx
37 5 2 2
∞ −x ∞ −x ∞
Here, I0 = ∫0 e sin0 xdx = ∫0 e dx = 0 − 6∫π e − x cos5 x sin xdx
= (− e − x )∞0 = 1
2
A6 ∞ −x

⇒ I6 =
30 12 2 144
× × =

−6
= ∫π e cos5 x sin xdx
2
37 17 5 629
−1
⇒ A6 = [− e − x cos5 x sin x]∞π
40. The area (in sq. units) bounded by the curve 6 2
y = x 2 + 2 x + 1 and the tangent to it at (1, 4) ∞ −x

and the Y -axis is [21 April 2019, Shift-II]


− ∫π ( − e ) [5cos4 x(− sin x)sin x + cos5 x ⋅ cos x] dx
2
1 2 7 −1 ∞
(a) (b) (c) 1 (d) ⇒ A6 = 0 + ∫ π e − x [−5 cos 4 x(1 − cos 2 x) + cos 6 x] dx
3 3 3 6 2
−1 ∞ ∞
Sol. (a) ⇒ A6 = − 5∫π e − x cos4 x dx + 6∫π e − x cos6 x dx
6
Given, y = x 2 + 2x + 1 2 2
−1
Differentiating w.r.t. x, we get ⇒ A6 = − 5A4 + 6 A6
6
dy dy
= 2x + 2 ⇒ = 2+ 2= 4 ⇒ 5A4 =
37
A6 ⇒ A6 =
30
A4
dx dx P(1 , 4) 6 37
∴Equation of tangent at P(1, 4) 30
A4 − A4
A4 − A6 37 7
y − 4 = 4(x − 1) ∴ = =
A4 A4 37
y − 4 = 4x − 4 ⇒ y = 4x
1 1 1
y=4x 42. lim  + +
n → ∞ n + m n + 2m n + 3m
(0, 4) P (1, 4) 1 
+K+  = [22 April 2019, Shift-I]
n + nm 
(0, 1)
log (m) loge (1 + m)
(a) e (b)
m 1+ m
A (1, 0) loge (1 + m) loge (1 + m)
(c) (d)
m 1− m
Definite Integrals and Its Applications 405
π π
Sol. (c)
= ∫ 2π cos x dx = [sin x] 2π
Given, − −
2 2
 1 1 1 1  ⇒ 2I = [1 + 1] = 2 ⇒ I = 1
lim  + + + .... + 
n→ ∞ n + m + 2 + 3 +
 n m n m n nm   π cos x 
∴ tan−1  ∫ 2π dx  = tan−1 ()
1
 1 + 1
+
1
+ ....  − + x
   2 1 e 
m  m   m 
1 + 1 + 2  1 + 3   π π
1
= lim 
n  n   n   = tan−1  tan  =
n→ ∞ n
  4 4
 1 
+
  m 
 1 + n   44. The area (in sq. units) bounded by
  n  
x 2 = y , y = x + 2 and the X-axis is
 
  [22 April 2019, Shift-I]
1 n 1 
= lim Σ (a)
2
(b)
3
n→ ∞ n  k = 1
1 + m  
k
  n 
3 5
 5 4
(c) (d)
1 1 1 1 m 6 5
= ∫0 1 + mx dx = m ∫0 1 + mx dx
Sol. (c)
1 1
= [log e (1 + mx)]10 = [log e (1 + m) − log()]
1 Indicate the region bounded by the curves
m m x 2 = y, y = x + 2 and X-axis and obtain the area
log e (1 + m)
= enclosed by them. The parabola and line meet in
m points P(−1, 1) and Q(2, 4).
 π /2 cos x  The line cuts the axes in (−2, 0) and B(0, 2.
)
43. tan −1 ∫ dx  = Here, we have to find the area bounded by the
−π / 2
 1+e 
x
[22 April 2019, Shift-I] curves and X-axis i.e. area ORAPQ
π π
(a) (b) i.e. area APR + area PRO
4 3 Y
π π
(c) (d) Q
6 2 (0,2)
Sol. (a) B
 π
cos x  (–1, 1) P
Given, tan−1  ∫ 2
π
dx 
 −
2
1 + e x 
A(–2, 0) X
π R O
cos x
Let I = ∫ 2
π
dx …(i)
− 1 + ex
2 = ∫ −−12 y line dx + ∫ −−01 y parabola dx
π π
π cos − − x  = ∫ −−12 ( x + 2 )dx + ∫ 0−1 x 2 dx
2 2 
⇒ I=∫ 2
π
dx −1
− π π
 − − x  x 2  1 
0
2
1 + e 2 2 
= + 2x  +  x 3 
Q b f (x)dx =  2  −2 3  −1
f (a + b − x)dx 
b
 ∫a ∫a  1 1 5
= + = sq units
π
cos x e x cos x
π 2 3 6
= ∫ 2π dx = ∫− 1 + e x dx
2 …(ii)
− 1 + e −x
π
1 + 32 + 243 + K + n5
2 2 45. lim =
Adding Eqs. (i) and (ii), we get
n→ ∞ n6
π [22 April 2019, Shift-II]
(e x + 1) cos x
2I = ∫ 2π dx …(iii) 1 1 1 1
− (1 + e x ) (a) (b) (c) (d)
2 5 11 6 2
406 AP EAMCET Chapterwise Mathematics

Sol. (c) Sol. (b)


Given, Y
1 + 32 + 243 + K + n5 1.5
lim
n→ ∞ n6
15 + 25 + 35 + K + n5 1
= lim ⋅
n→ ∞ n5 n 1
5
S = lim Σ   ⋅ ⇒ S =
r 1n 1
∫0 x dx
5
n → ∞ r =1  n  n 0.5
1
 x6  1
S =   ⇒S =
 6 0 6
X
π/10 π/5 3π/10 2π/5 π/2
46. Let a > 1 and b > 1. If f ()
t is a periodic function
π /2
∞ π /2
Let I1 = ∫0 (sin x + cos x) dx = [− cos x + sin x]0
T
of period T and ∫ a − bt f ()
t dt = k ∫ a − bt f ()
t dt,
0 0
π π
then k = [22 April 2019, Shift-II] =  − cos + sin − sin 0 + cos 0 = 2
bT − bT bT bT  2 2 
a a a a
(a) (b) (c) (d) π /2
T+1 a− bT + 1 b aT + 1 abT − 1 and I 2 = ∫0 | cos x − sin x | dx
Sol. (d) π
For x = 0 to x =, cos x − sin x ≥ 0
∞ 4
Let I = ∫0 a − bt f ()
t dt , given a , b > 1 and
π π
T − bt ∞
and for x = to , cos x − sin x ≤ 0
f (T + t) = f ()
t = ∫0 a t dt + ∫ a − bt f ()
f () t dt ...(i) 4 2
T π /4 π /2

Again, let I1 = ∫T

a − bt f ()
t dt
∴ I2 = ∫0 (cos x − sin x) dx + ∫π / 4 (− cos x + sin x) dx
Put, t= x+T = [sin x + cos x]π0 / 4 + [− sin x − cos x]ππ // 42

∴ I1 = ∫ a − b( x + T) f (x + T)dx π π
0 = sin + cos − sin 0 − cos 0
∞ 4 4
= ∫0 a − bx a − bT f (x)dx π π π π
− sin − cos + sin + cos
2 2 4 4
[Q f (x + T) = f (x)]
∞ 1 1 1 1
= a − bT ∫ a − bT f ()
t dt = + − 0 −1 −1 + +
0 2 2 2 2
I1 = a − bT (I) ...(ii) =2 2−2
From integrals Eqs;(i) and (ii), we get
T − bt
∴ Required area (A) = I1 − I 2 = 2 − (2 2 − 2)
I= ∫0 a t dt + a − bT I
f ()
= 4 − 2 2 sq. unit
T − bt
⇒ I(1 − a − bT) = ∫0 a f ()
t dt 1
1
a bT T − bt 48. lim [(n + 1)(n + 2) ...... (2 n)] n =
n→ ∞
⇒ I=
a bT −1 ∫0 a f ()
t dt n
[23 April 2019, Shift-I]
T − bt a bT 2 4
= k∫ a t dt = k =
f () (a) 1 (b) 0 (c) (d)
0 a bT − 1 e e

47. The area (in sq units) enclosed by the curves Sol. (d)
y = sin x + cos x and y = |cos x − sin x |over Let P = lim
1
[(n + 1) (n + 2) ......(2n)]1 / n
 π n→ ∞ n
the interval 0 ,  is [22 April 2019, Shift-II]
 2 On applying ‘log’ both sides, we get
1  n + 1   n + 2  n + n 
(a) 4 + 2 2 (b) 4 − 2 2 log P = lim log     ......  
(c) 2 + 2 3 (d) 6 − 3 2 n→ ∞ n   n   n   n  
Definite Integrals and Its Applications 407

1 n 1 + r  = 1 log (1 + x) dx Sol. (d)


n  ∫0
= lim ∑ log
 
n→ ∞ 
n r =1 Given curve is
1 x
= [x log (1 + x)]10 − ∫ dx ay 2 = x 2 (a − x) , (a > 0)
01 + x
Y
= log 2 − (x − log (1 + x))10
= log 2 − 1 + log 2
= log 4 − 1 = log   ⇒ P =
4 4
e e
X′ X
Hence, option (d) is correct. (a, 0)
O
π
sin 2 x
49. ∫02 sin x + cos x dx = [23 April 2019, Shift-I]
1
3 1 Y′
(a) log ( 2 + 1)2 (b) log ( 2 + 1)
2 2 The required area
2 2 a −x
(c) log ( 3 + 1) (d) log ( 2 − 1) = 2∫ x
a
dx
3 3 0 a
Sol. (b) 2 a

Let
=
a ∫0 x a − x dx
π
sin2 x Put a − x = t 2, at x = 0, t = a and ,
I= ∫02 sin x + cos x dx …(i)
at x = a, t = 0
On applying property ∫ f (x) dx =
a a
and − dx = 2t dt
0 ∫0 f (a − x) dx,
0 4
we get, So, required area = − ∫ a a
× t 2 (a − t 2) dt
π
cos2 x
I= ∫
a
2 dx …(ii) 4 a 4  at 3 t 5 
0 cos x + sin x =
a ∫0 (at 2 − t 4) dt =
a

 3
− 
5 0

On adding Eqs. (i) and (ii), we get 4  a2 a a2 a  8 2


=  −  = a
π
sin2 x + cos2 x a  3 5  15
2I = ∫ 2
0 sin x + cos x
dx
Hence, option (d) is correct.
π
dx  
∫ 1 + 2 2 + 3 3 +…+ n n 
51. lim 
= 2
x + cos x  =
0 sin
n→ ∞ 5
π
π  
x  dx
1 n2
=
2 ∫02 cosec  4 +  [22 April 2018, Shift-I]
π /2 5
1  π  π   (a) 1 (b)
=  log cosec  4 + x  − cot  4 + x   2
2  0 2
(c) 0 (d)
1 5
⇒ I= [log ( 2 + 1) − log ( 2 − 1)]
2 2
Sol. (d)
1 1
= [2 log ( 2 + 1)] = log ( 2 + 1) We have,
2 2 2  1 + 2 2 + 3 3 + K + n n
lim
x → ∞ 
Hence, option (b) is correct.
 n5/ 2 
50. The area (in sq units) enclosed by the loop of 3/ 2
= lim ∑  
n r 1

the curve ay = x (a − x), (a > 0) is
2 2
x → ∞ r =1  n  n
[23 April 2019, Shift-I] 1
1  x 5/ 2  2
π
(a) 2 πa 2
(b) a2
4
(c) a2
8
(d) a2 = ∫0 x 3/ 2dx =   = .
 5 / 20 5
3 15 15
408 AP EAMCET Chapterwise Mathematics

α/3 f (x) 2
∫0 y = − 6  x +  +
dx = 1 25
52. ⇒
α − 3x   12 24
f (x) + f  
 3  2
6  x +  = −  y −
[22 April 2018, Shift-I] 1 25
2α α α α ⇒ 
(a) (b) (c) (d)  12  24 
3 2 3 6 2
⇒ x + 1  = − 1  y − 25
Sol. (d)    
 12 6  24 
Let Y
α/ 3 f (x) –1 , 25
I= ∫0 α − 3x 
dx …(i) 12 24
f (x) + f  
 3 
α
f  − x 
X′ X
–1 , (0, 0) 1,
α/ 3 3  0 0
⇒ I= ∫0  α 
dx 2 3
 α − 3 − x  
α 3 
f  − x  + f  Y′
3   3 
  1/ 3
  ∴Required area = ∫− 1 / 2 y dx
Q f (x) dx =
a a 
 ∫0 ∫0 f (a − x) dx 
1/ 3
1/ 3  x2 
= ∫− 1 / 2 − − = − − 2x 3 
2
(1 x 6 x ) dx  x
α − 3x   2 − 1 / 2
f  
α/ 3  3 
⇒ I= ∫0 α − 3x 
dx … (ii) =  −
1 1

2   1 1 1
 − − − + 
f   + f ( x)  3 18 27   2 8 4 
 3 
18 − 3 − 4   − 4 − 1 + 2
On adding Eqs. (i) and (ii), we get =   − 
α/ 3  54   8 
2I = ∫0 1 dx
11 3 44 + 81 125
= + = = sq unit
⇒ 2I = [x]α0 / 3 54 8 216 216
α π /4
⇒ 2I =
3 54. For n ≥ 2 , let I n = ∫0 tan n x d x and
α Fn = I n + I n − 2 . Then, Fn − Fn + 1 =
⇒ I= .
6 [22 April 2018, Shift-II]
1 1 1
53. The area (in sq. units) of the region bounded (a) (b) (c) (d) 1 + n
n n−1 n (n − 1)
by the X -axis and the curve y = 1 − x − 6 x 2 is
[22 April 2018, Shift-I] Sol. (c)
125 125 25 25 π/4
(a)
216
(b)
512
(c)
216
(d)
512
Q In = ∫0 tann x dx, n ≥ 2.

Sol. (a) and Fn = I n + I n − 2


π/4
We have, = ∫0 (tann x + tann − 2 x) dx
y = 1 − x − 6x2 π/4
= ∫0 (tan2 x + 1) tann −2 x dx
⇒ y = − [6 x + x − 1]
2

π/4
⇒ y = − 6 x2 + − 
x 1 = ∫0 tann − 2 x sec2 x dx
 6 6 
Let tan x = t
 2
1
y = − 6   x +  −
1 1 ⇒ sec x dx = dt
2
⇒ − 
 12 144 6 
 π
2 Then, at x = ⇒ t =1
y = − 6  x +  +
1 1 4
⇒ +1
 12 24 and x=0 ⇒ t=0
Definite Integrals and Its Applications 409
1 1 2
1 n−2 tn − 1 So, I= ∫0 (x
2
− 3x + 2)dx − ∫1 (x
2
− 3x + 2) dx
So, Fn = ∫0 t dt =
n −10 3
+ ∫2(x − 3x + 2) dx
2
1 1
= 1 n −1 =
()
n −1 n −1  x 3 3x 2   x 3 3x 2
1

2

1 = − + 2x  −  − + 2x  +
∴ Fn + 1 =  3 2 0  3 2 1
n 3
1 1  x 3 3x 2 
So, Fn − Fn + 1 = −  − + 2x 
n −1 n  3 2 2
n − (n − 1) 1  1 3    8 12   1 3  
= = =  − + 2 −  − + 4 −  − + 2
n(n − 1) n(n − 1) 3 2    3 2  3 2  
 27 27  11
 +  + 6 −  − + 4 = .
8 12
1 1 −
55. lim n  + +K   3 2   3 2   6
(3 n + 8 n + 4) 3 n + 16 n + 16
n→ ∞ 2 2

1  57. OABC is a unit square where O is the origin


+ = [22 April 2018, Shift-II]
15 n2  and B = (1, 1). The curves y 2 = x and x 2 = y
1 9 1 9 9 1 5 divide the area of the square into three parts
(a) log (b) log (c) 2 log (d) log
2 5 4 5 5 4 9 OABO, OBO and OBCO. If a1 , a 2 , a 3 are the
areas (in sq units) of these parts respectively,
Sol. (b)
then a1 + 2 a 2 + 3 a 3 = [22 April 2018, Shift-II]
 1 1 1 
lim n 2 + 2 + ....+ 
(a) 1 (b) 2
n→ ∞
 3n + 8n + 4 3n + 16n + 16 15n2  (c) 6 (d) 64
n 1
= lim n ∑ 2 Sol. (b)
n→ ∞ r =1 3n + 8nr + (2r) 2
Q a1 + a 2 + a 3 = 1 …(i)
= lim
1 n

1 and due to symmetry a1 = a 3 …(ii)
n→ ∞ n r =1 2 1
3 + 8  + 4  ∫0 (
r r Now, a 2 = x − x 2) dx
 n  n
1
1 dx 1 dx 2 x3  2 1 1
=∫ =∫ =  x 3/ 2 −  = − =
0 3 + 8x + 4x2 0 4(x + 1) 2 − 1
3 3 0 3 3 3
1
 ( x + 1) − 1  Y
1 1 dx 1 1  2
=
4 ∫0 2
=
4
×
1  ln 1 
( x + 1) 2 −   2 ×  ( x + 1) + 
1 (0, 1) C B (1, 1)
 2 2 20 a3
1  ln 3 / 2 − ln 1 / 2  = 1 ln 9 . y2=x a2
=
4  5 / 2 3 / 2 4 5
a1

3
X
O A (1, 0)
56. ∫0 | x 2 − 3 x + 2| dx = [22 April 2018, Shift-II]
y=x2
3 1 11 11
(a) (b) (c) (d) 1
2 6 6 2 So, a1 + a 3 +=1
3
Sol. (c) 2
⇒ a1 + a 3 = …(iii)
3 3
Let I = ∫0 x 2 − 3x + 2 dx
From Eqs. (ii) and (iii),
Q x − 3x + 2 = (x − 2) (x − 1)
2
1
a1 = a 3 = = a 2
So, x 2 − 3x + 2 < 0 for x ∈(1, 2) 3
1 2 3
and x 2 − 3x + 2 ≥ 0 for x ∈ R − (1, 2) So, a1 + 2a 2 + 3a 3 = + + = 2
3 3 3
410 AP EAMCET Chapterwise Mathematics

π sin x π sin x + 1 − 1
58. If a and b are positive integers such that b > a, = π ∫0 dx = π ∫0 dx
1 + sin x 1 + sin x
1 1 1 1
then lim  + + +K+  =  π (1 + sin x) π 1 
n → ∞  na na + 1 na + 2 nb = π ∫ dx − ∫ dx 
0 1 + sin x 0 1 + sin x
 
[23 April 2018, Shift-I]
 π π 1 1 − sin x 
(a) log  (b) log 
b a = π  ∫ 1 dx − ∫ × dx
 a  b  0 0 1 + sin x 1 − sin x 
(d) log(a + b ) π 1 − sin x
= π [x]π0 − ∫ dx  = π [(π − 0) −
(c) log(ab )
Sol. (a)  0 cos 2 x 
π π
∫0 (sec x − sec x tan x) dx = π [π − (tan x − sec x)0 ]
2
The given limit
1 1 1 1 
lim + + + ... + = π [π − [tan π − sec π − tan 0 + sec 0]]
x → ∞  na na + 1 na + 2 nb 
 = π [π − { 0 + 1 − 0 + 1}]
1 1 1 1  π
= lim  + + + ... + 2I = π [π − 2] ⇒ I = [π − 2]
x → ∞ na
 na + 1 na + 2 na + n(b − a)  2
( b − a) n
1 1 ( b− a) n 1 60. The area (in sq. units) of the region lying in
= lim ∑ = lim ∑
x→ ∞ r =0 na + r x → ∞ n r =0 a + r the first quadrant and enclosed by the
n X -axis, the straight line x − 3 y = 0 and the
( b − a) dx Q r = x  circle x 2 + y 2 = 4 , is
= ∫0 a+ x  n  [23 April 2018, Shift-I]
π 2π π 2π
(a) (b) (c) (d)
= [log (a + x)]b0 − a = log (a + b − a) − log (a + 0) 3 3 2 3 3 2
b
= log b − log a = log Sol. (a)
a
Equation of given circle
π x tan x
59. ∫0 sec x + tan x
dx =
[23 April 2018, Shift-I]
x 2 + y2 = 4 … (i)
⇒ x + y =2
2 2 2
π −2 π+2
(a) (b) ∴ radius = 2
2 2
π( π + 2 ) π( π − 2 ) (0, 2)
(c) (d) x=√3, y
2 2
C (√3, 1)
Sol. (d)
Let
π
x tan x O A (2, 0)
I= ∫0
sec x + tan x
dx …(i)

π (π − x) tan (π − x)
I=∫ dx
0 sec (π − x) + tan (π − x)

Q using
 a a and line x− 3y=0
 ∫ f (x) dx = ∫0 f (a − x) dx
 0 ⇒ x= 3y … (ii)
− (π − x) tan (x)
π
∫0 − sec x − tan x dx
= [From Eqs. (i) and (ii)]
( 3y)2 + y 2 = 4
π (π − x) tan x
I=∫ dx … (ii)
0 sec x + tan x ⇒ 4 y2 = 4 ⇒ y2 = 1 ⇒ y = ± 1

Adding Eqs. (i) and (ii) If y= +1


sin x
∴ x= 3
π π tan x π cos x
2I = ∫0 sec x + tan x
dx = π ∫0 1 sin x
dx If y = −1
+
cos x cos x ∴ x=− 3
Definite Integrals and Its Applications 411

5 π
x  dt = ∫ sin5()
As point C is in first quadrant 3 6 π/2
∴ C is ( 3, 1)
So, ∫0 sin 
6  π 0
t dt

6 4×2 16
Area of shaded region = × =
3 x 2 π 5 × 3 × 1 5π
=∫ dx + ∫ 4 − x 2 dx
0 3 3 π /2
1 3 2
62. ∫0 log e (sin 2 x) dx
[23 April 2018, Shift-II]
3 ∫0
= x dx + ∫ 22 − x 2 dx π π
3 (a) πlog2 (b) − πlog2 (c) log 2 (d) − log 2
3 2
2 2
1  x2  x 22 x
=   + 22 − x 2 + sin−1  Sol. (d)
3  2 0 2 2 2 3 π /2

Q a 2 − x 2 dx = x a 2 − x 2 + a sin−1
2
x 
I= ∫0 log e (sin 2x)dx
 ∫
+ c
2 2 a  Let 2x = t
For lower limit at x = 0, t = 0
1 3   3
= . + (0 + 2sin−1 1) −  and upper limit at x = π /2, t = π and dx =
1
dt
3 2   2 2
3   1 π π/2
4 − 3 + 2 sin−1
2 ∫0 ∫0
 So, I = t dt =
log esin() log esin()
t dt
2 
π
3  π 3 π = − loge 2
= + 2 × − − 2.  2
2  2 2 3
63. The area (in sq units) between the curve
3 3 2π 2π π
= + π− − = π− = y 2 = 8 x and its latus rectum is
2 2 3 3 3
π [23 April 2018, Shift-II]
∴Required area = sq. units 32 64 16 8 2
3 (a) (b) (c) (d)
3 3 3 3
1  5 π  5 2π  5 3π 
61. lim sin   + sin   + sin   + Sol. (a)
n→ ∞ n  6n   6n   6 n
2
The required area = 2 ∫ 8 x dx
 π
..... + sin 5    = 0
 2 [23 April 2018, Shift-II] 2 x3 / 2
2
8 2 32
=4 2 = (2 2 ) =
8 8 32 16 3 3 3
(a) (b) (c) (d) 0
15π 5π 5π 5π
1
Sol. (d)  1   2 2   n2   n
1  5 π  64. lim 1 + 2  1 + 2  ... 1 + 2   =
5  2π  5  3π  n→ ∞ 
lim sin   + sin   + sin    n   n   n  
n→ ∞ n  6n   6n   6n 
[24 April 2018, Shift-I]
π
+ K + sin    5 π −4 π −2
 2 (a) 3e 6 (b) 2e 4
π −4 π −4
1  5 π  5  2π  5  3π 
= lim sin   + sin   + sin   (c) 2e 2 (d) 4e 4
n→ ∞ n  6n   6n   6n 
3nπ   Sol. (c)
+ K + sin5   1
 6n    1  22   n2   n
Let A = lim  1 + 2  1 + 2  ... 1 + 2  
1 3n 5  rπ  π
∑ sin   = ∫ sin5  x  dx n→ ∞  n  
3
= lim  n   n 
n→ ∞
n r =1  6n  0 6 
taking log on both side
π
Let x=t 1
6 log e A = lim
n→ ∞ n
For upper limit at x = 3, t = π /2   1  22   n2  
and lower limit at x = 0, t = 0 and dx =
6
dt  log 1 + 2  + log 1 + 2  ... log 1 + 2  
 n  n   n 
π
412 AP EAMCET Chapterwise Mathematics

π
n  r2  1 1 2 (cos x + sin x)
log A = lim ∑ log 1 + 2  = ∫ log(1 + x 2) dx
n → ∞ r =1  n  n 0
= ∫02 2 + 2cos x sin x

[Applying formula lim ∑ f   ⋅ = ∫ f (x) dx


n r 1 1 π
2(cos x + sin x)
n → ∞ r =1  n  n 0 = ∫02 3 − (sin x − cos x)2
dx
5
= ∫0 log(1 I+ x 2)⋅1 dx
II Let sin x − cos x = t ⇒(cos x + sin x) dx = dt
using by parts, Now, changing the limit
x → 0 ⇒ t → − 1; x → π /2 ⇒ t → 1
= log(1 + x 2) ∫ 1 dx − ∫  (log(1 + x 2) ∫ 1 dx  dx
d
 dx  1 2 dt
2I = ∫
2 −1
= x log(1 + x 2) − ∫
x
⋅ x dx 3 − t2
1 + x2
 a f (x) dx = 2 a f (x) dx if f (x) is even function]
= x log(1 + x ) − 2∫
x2 2
dx  ∫− a ∫0
1 + x2 1 2 dt
2I = 2∫
1 + x2 1 0
( 3)2 − t 2
= x log(1 + x 2) − 2∫ + 2∫ dx
1+ x 2
1 + x2 1
 t 
2 sin− 1    = 2 sin− 1   .
1
= x log(1 + x 2) − 2∫ 1 dx + 2tan− 1 (x) I=
  3 0  3
= [x log(1 + x 2) − 2x + 2tan− 1 (x)]10
66. The area (in square units) bounded by the
⇒ log e A = [1 ⋅ log 2 − 2 + 2tan− 1 ()
1 − 0 + 0 − 0]
curves y = 2 x 2 and y = max { x − [ x ], x +| x|}
π π π −4
log 2− 2+ 2⋅ −2
A= e 4 = elog 2 ⋅ e 2 = 2e 2 . in between the lines x = 0 and x = 2 is
[24 April 2018, Shift-I]
π
cos xdx 4 1
65. ∫ 0
2
1 + cos x sin x
= (a)
3
(b)
2
(c) 1 (d) 2
[24 April 2018, Shift-I]
 1  Sol. (d)
sin−1 
−1 1 1 
(a) 2 cos   (b)  y = max⋅ {x − [x], x + | x |}
 3 2  3
−1  1  −1  2x , x ≥ 0
(c) 2 sin   (d) 2 sin ( 3 ) ⇒ y=
 3 {x}, x < 0
Sol. (c) Shaded region is required Area
π 1 2
= ∫0 (2x − 2x )dx + ∫1 (2x − 2x) dx
2 2
cos xdx
∫0
2
1 + cos x sin x
y=2x2
π
cos x
let I = ∫ 0
2
1 + cos x sin x
… (i)
y=2x
 a f (x) dx = a f (a − x) dx 
 ∫0 ∫0 
π
π cos − x 
2  O 1 2
I= ∫ 2
π π
dx
 
1 + cos − x  sin − x 
0

2  2  1 1 2 2
= ∫0 2x dx − ∫0 2x dx + ∫1 2x dx − ∫1 2x dx
2 2
π
sin x
I= ∫02 1 + cos x sin x
dx … (ii)
 2x 2 
1
 2x 3   2x 3 
1
 2x 2 
2 2
=   −  +   − 
On adding Eqs. (i) and (ii), we get  2 0  3 0  3 1  2 1
π
cos x + sin x 2 2 16 2 1 5
2I = ∫02 1 + cos x sin x
×
2
dx = 1−
3
+ − − 4 + 1 = + = 2 sq. units.
3 3 3 3
28
Differential Equations
1. Find the solution of the differential equation Sol. (c)
(e y − x ) dy = (e x − e y)dx [17 Sep. 2020, Shift-I]  x cos y  + y sin y   ydx
    
x2   x  x
(a) e e = e
y x 2x
−e +c
x x
(b) e ye x = e xee − ee + c
=  y sin  − x cos   xdy
y y
y e
(c) e e
x
=e e
x e x
−e ex
+c   x   x 
y x x
(d) ee e x = e xee − ee + c  x cos y  + y sin y  
   
dy   x  x y
Sol. (c) ⇒ = ⋅
dx  y sin y  − x cos y   x
(e y− x )dy = (e x − e y)dx    
  x  x  
dy
⇒ ey⋅ = e 2x − e x ⋅ e y
dx y dy dV
Let =V ⇒ =V+ x
dy x dx dx
⇒ ey⋅ + e x ⋅ e y = e 2x
dx  1 cos V + sin V 
Let e y = z ⇒ e y ⋅
dy dz dz dV  V 
= ⇒ + e x ⋅ z = e 2x ⇒ V+ x = ⋅V
dx dx dx dx  sin V − 1 cos V 
 
∫ e x ⋅ dx V
=e = ee
x
IF
dV  cos V + V sin V 
∴ Solution is ⇒ V + x⋅ = V
dx  V sin V − cos V 
Z ⋅ ee = ∫ e 2 x ⋅ e e ⋅ dx + c
x x

dV  V cos V + V 2 sin V − V 2 sin V + V cos V 


⇒ x⋅ = 
Let e =u x
dx  V sin V − cos V 
e x ⋅ dx = du dV 2V cos V
⇒ x⋅ =
⇒ Z ⋅ ee = ∫ u ⋅ e u du + c = ue u − e u + c
x

dx V sin V − cos V
(V sin V − cos V) dx
⇒ e y ⋅ e e = e e (e x − 1) + c ⇒ dV = 2⋅
x x

V cos V x
2. Find the solution of the following differential Let V cos V = t
  y  y ⇒ (cos V − V sin V)dV = dt
equation  x cos   + y sin    y
  x   x  dt dx
⇒ ∫ − t = 2∫ x
  y  y
dx =  y sin   − x cos    x dy ⇒ 2ln x + c = − ln t
  x   x 
2ln x + c = − ln cos 
y y
[17 Sep. 2020, Shift-I] ⇒
x x
 x
(b) x cos   = ± e − c
y
(a) y cos   = ± e − c ⇒ ln x 2 ⋅ cos  = − C ⇒ ln xy cos  = − C
y y y
 y  x  x x  x
(c) xy cos   = ± e − c (d) xy sin   = ± e − c
y y y
 x  x ⇒ xy ⋅ cos = ± e − C
x
414 AP EAMCET Chapterwise Mathematics

3. If c1 , c2, c3, c4 , c5 are arbitrary constants, then d2y


6. The solution of = 0 represents
the order of the differential equation whose dx 2
general solution is [18 Sep. 2020, Shift-I]
y = (c1 + c2) sin(x + c3) + c4 e x + c5 is (a) straight lines (b) a circle
[17 Sep. 2020, Shift-II] (c) a parabola (d) a point
(a) 3 (b) 5 Sol. (a)
(c) 4 (d) Not defined Given differential equation
Sol. (a) d2 y dy
=0 ⇒ =a
General solution of differential equation is, dx 2 dx
y = (C1 + C2) sin(x + C3) + C4 e x + C 5 ⇒ y = ax + b, is a equation of straight line.
Here solution is of form; 7. The differential equation of the family of all
y = Asin (x + C3) + Be x [Q C1 + C2 = A, C4 e c = B] straight lines passing through the origin is
So, there are 3-arbitrary constants, A1 C3 and B [18 Sep. 2020, Shift-I]
dy dy
Now, order of differential equation = number of (a) x = y (b) =0
dx dx
arbitrary constants in general solution.
dy d2y y
(c) y = x (d) 2 =
So, order of differential equation is 3. dx dx x
4. The product of the degree and order of the Sol. (c)
2
 d2 y
3
 dy  Equation of family of lines passing through
differential equation  2  −   = y 3 is
 dx   dx  origin is
y
[17 Sep. 2020, Shift-II] y = mx ⇒ =m
x
(a) 3 (b) 4 (c) 6 (d) 5 dy
x −y
Sol. (b) ⇒ dx =0 {by differentiation}
2
2 x
 d2 y 
3
 dy 
 2 −   = y
3
dy
dx   dx  ⇒ y= x .
dx
Order = 2, Degree = 2
8. The solution of the differential equation
Product = 2 × 2 = 4. cos(x + y)dy = dx given that y(0) = 0 is
dy [18 Sep. 2020, Shift-I]
5. The integrating factor of x + 3 y = x 2 is x + y x + y
dx (a) y = tan   (b) y = sin  
[17 Sep. 2020, Shift-II]  2   2 
(c) y = tan   (d) y = tan  
3 y x
(a) (b) log x (c) x3 (d) x 2 2
x
Sol. (c) Sol. (a)
Given equation is, Given differential equation
dy dy 3 dy
x + 3y = x 2 or + ⋅y= x cos(x + y)dy = dx ⇒ = sec (x + y)
dx dx x dx
Comparing with, dy dt
Put x + y = t ⇒1 + =
dy dx dx
+ Py = Q ,
dx dt dt
t ⇒ ∫
1 + sec t ∫
⇒ − 1 = sec() = dx
3 dx
We have p = and So, the integrating factor is,
x cos t
∫ dx
3 ⇒ ∫ 1 + cot t dt = ∫ dx
IF = e ∫ = e x = e 3log x = elog x = x 3
pdx 3
Differential Equations 415

 1  10. The particular solution of the differential


⇒ ∫ 1 − 1 + cos t  dt = ∫ dx dy
equation = sec y, y(0) = 0 is
⇒ ∫ 1 − sec2    dt = ∫ dx
1 t dx [18 Sep. 2020, Shift-II]
 2  2  (a) x = cos y (b) x = sin y + q
(c) y = sin x (d) x = sin y
t − tan   = x + c
t

 2 Sol. (d)
x + y
⇒ x + y − tan   = x+ c Given, differential equation,
dy
= secy
 2  dx
x + y
⇒ y = tan   +c ⇒ ∫ cos y dy = ∫ dx
 2 
⇒ sin y = x + c
Q f(0) = 0 ⇒ c = 0
x + y ∴ y(0) = 0
∴ y = tan   ⇒ c=0
 2 
∴ The required solution of differential equation
9. The general solution of the differential is x = sin y
dy Hence, option (d) is correct.
equation + yg ′(x) = g(x) g ′ (x) is
dx dy
[18 Sep. 2020, Shift-II] 11. The solution of x = y(log y − log x + 1) is
dx
(a) g ( x) + log(1 + y + g ( x)) = c [18 Sep. 2020, Shift-II]
(b) g ( x) + log(1 + y − g ( x)) = c
(a) y = xecx (b) y2 = cx2
(c) g ( x) − log(1 + y + g ( x)) = c
(c) y2 = cx log ( x) (d) log( y) = cx
(d) g ( x) − log(1 + y − g ( x)) = c
Sol. (a)
Sol. (b)
dx
dy x = y (log y − log x + 1)
+ y g′(x) = g(x). g′(x) dx
dx
dy y y
∴ P = g′(x) and Q = g(x). g′(x) = (log + 1) …(i)
dx x x
IF = e ∫
pdx

f (x , y) =  log + 1
y y
Let,
= e∫ x 
g ′( x ) dx
x
IF = e g( x ) f (kx , ky) =
ky  ky
+ 1
 log
General Solution is kx  kx 
y (IF) = ∫ Q(IF) dx + c f (kx , ky) =  log + 1 = f (x , y)
y y
x x 
y (e g( x )) = ∫ g(x). g′(x). e dx + c
g( x )

∴Given differentiate Equation is homogeneous


Put, g(x) = t ∴put , y = vx 
 
g′(x)dx = dt  dy dv 
 dx = v + x
ye g( x ) = ∫ t e t dt + e c dx 
From Eq. (i),
ye g( x ) = e t (t − 1) + e c
dv vx  vx 
ye g( x ) = e t . t − e t + e c v+ x =  log + 1
dx x  x 
ye g( x ) = e g( x ) . g(x) − e g( x ) + e c dv
v+ x = v(log v + 1)
e [y + 1 − g(x)] = e c
g( x )
dx
Taking log on both sides, dv
v+ x = v log v + v
log e e g( x ) [y + 1 − g(x)] = log e e c dx
g(x) + [y + 1 − g(x)] = C 1 1
dv = dx
Hence, option (b) is correct. v log v x
416 AP EAMCET Chapterwise Mathematics

Integrating on both sides,


1 1 1
∫ v ⋅ log v dv = ∫ x dx Sol. (b)
∫x e
4
It is given that, y = 3 x
dx
Put, log v = t
1
dv = dt Let x 4 = t ⇒ 4 x 3 dx = dt
4
v 1 et ex
∴So, y = ∫ e t dt = + C= + C
1 1 4 4 4
∫ t dt = ∫ x dx ex
4

Q The curve y = + C passes through point


log t = log x + log c 4
4
log(log v) = log xc 3
(0, 1), so C = ⇒ y =
ex 3 4
+ ⇒ ex = 4y − 3
log e v = xc 4 4 4
v = e xc ⇒ x 4 = log e|4 y − 3| ⇒ x = (log e|4 y − 3|)1 / 4
y Hence, option (c) is correct.
= e xc
x
14. Find the sum of the order and degree of the
y = x. e xc 3
 dy  d2y
Hence, option (a) is correct. differential equation y = x   + 2
 dx  dx
12. In a bank, the principal increases [21 Sep. 2020, Shift-I]
continuously at the rate of 6% per year. Then
(a) 2 (b) 3 (c) 4 (d) 5
the time required to double ` 6000 rupees is
........ (in years) [21 Sep. 2020, Shift-I] Sol. (b)
50 50 The order and degree of the given differential
(a) log 2 (b) log 6 3
d2 y
equation y = x   + 2 are 2 and 1
3 3 dy
50 50  dx  dx
(c) log 3 (d) log 12
3 3 respectively.
Sol. (a) So, the sum of the order and degree of given
differential equation is 3.
According to given information, let the principal
P =`6000 getting double in time ‘t’ with rate of Hence, option (b) is correct.
6% per year, so dy x − y +3
dP 6 12000 dp 3 t 15. On solving = , the solution
P ⇒∫ dx 2 x − 2 y + 5
50 ∫0
= = dt
dt 100 6000 P
obtained is x = 2(x − y) + log ()
t + c, find t
3
⇒ 6000 =
[log P]12000 t [21 Sep. 2020, Shift-II]
50
(a) x − y + 2 (b) x + y − 2
3 (c) x + y + 2 (d) x − y − 2
⇒ log12000 − log 6000 = t
50
Sol. (a)
3 50
⇒ log 2 = t ⇒ t= log 2 Given differential equation,
50 3
dy x− y+ 3
=
Hence, option (a) is correct. dx 2(x − y) + 5
13. Let the equation of the curve passing dy dt
Let x − y = t ⇒ 1 − = , so
through the point (0, 1) be given by dx dx
y = ∫ x 3e x dx . If the equation of the curve is t+ 3 dt 2t + 5 − t − 3 t + 2
4
dt
1− = ⇒ = =
dx 2t + 5 dx 2t + 5 2t + 5
written in the form x = f (y), then f (y) =
2t + 5
[21 Sep. 2020, Shift-I] ⇒ ∫ t + 2 dt = ∫ dx
(a) log| 4 y − 3| (b) (log| 4 y − 3|)1 / 4
1/ 4  1 
(c)  log
3 − 4y 
 (d) log
4y − 3 ⇒ ∫  2 + t + 2 dt = x + C′
 4  4
Differential Equations 417
−1 −1 −1
⇒ 2t + log e|t + 2|= x + C ′ ⇒ xetan y
= (tan−1 y)etan y
− etan y
+c
⇒ 2(x − y) + log|x − y + 2|+ C = x ⇒ xe =e
tan −1 −1
((tan y) − 1) + c
y tan y −1

⇒ 2(x − y) + log()
t +C= x (given) Hence, option (b) is correct.
On comparing, we get
∴ t = x− y+ 2 18. Solve the differential equation given below
Hence, option (a) is correct. x dy
= y+ x 2 + y2
dx [22 Sep. 2020, Shift-I]
16. The general solution of the differential
 dy  (a) x = c[ y +
2
y + x ]
2 2

equation log   = ax + by is
 ax  [21 Sep. 2020, Shift-II] (b) y2 = c[ x + y2 − x 2 ]
(a) ae − by + be ax = c (b) ae ax + be − by = c (c) y2 = c[ x + tan−1 ( 1 + y2 )]
(c) ae − by − be ax = c (d) ae by + be − ax = c (d) y2 = c [ x − y2 + x 2 ]
Sol. (a)
Sol. (a)
Given differential equation
Given differential equation
log   = ax + by
dy
dy
 dx  x = y + x 2 + y2
dx
dy
⇒ = e ax + by = e ax . e by dy y + x + y
2 2
dx ⇒ =
1 1 dx x
∫e
dy = ∫ e ax dx ⇒ − e − by = e ax + c1
− by

b a Now, put y = v ⋅ x
⇒ be ax + ae − by = c dy dv
∴ =v+ x
Hence, option (a) is correct. dx dx
On doing substitution, we get
17. Solve of the differential equation dv
v+ x = v + 1 + v2
dy 1 + y2 dx
=
dx (tan −1 y) − x dv dx
⇒ ∫ 1 + v2 = ∫ x
[21 Sep. 2020, Shift-II]
−1 −1
(a) xe tan y
= e − tan y
((tan−1 y) − 1) + c ⇒ log e C + log e ( 1 + v2 + v) = log e x
−1 −1
(b) xe tan y
= e tan y
((tan−1 y) − 1) + c ⇒ log [(y + x 2 + y 2)C ] = 2 log e x
tan −1 y tan −1 y
(c) xe =e ((tan−1 y) + 1) + c
−1 −1 ⇒ x 2 = C [y + y2 + x 2 ]
(d) xe tan y
= e − tan y
((tan−1 y) + 1) + c
Hence, option (a) is correct.
Sol. (b)
19. If it is mentioned that for a curve passing
Given differential equation
through (3, 4), the slope of the curve at any
dy 1 + y2 dx tan−1 y − x
= −1
⇒ = point is the reciprocal, of twice the ordinate
dx tan y − x dy 1 + y2 of that point, then that curve is a ………
dx x tan−1 y [22 Sep. 2020, Shift-I]
⇒ + = is a linear
dy 1 + y 2 1 + y2 (a) Ellipse (b) Parabola
differential equation, so. (c) Hyperbola (d) Circle

dy
= e tan −1 y Sol. (b)
IF = e 1 + y2
According to given information the slope of
So, the solution is curve at point (x , y) is
−1 −1 tan−1 y dy 1
= ∫ etan
dx 2y ∫
xetan y y
dy + c = ⇒ 2y dy = ∫ dx
1 + y2
⇒ y2 = x + c
−1
⇒ xetan y
= ∫ t. e t dt + c, {where t = tan−1 y } ...(i)
−1 Q Curve (i) passes through point (3, 4), so c = 13.
⇒ xetan y
= te t − ∫ e t + c
∴Required curve y 2 = x + 13 and it is a parabola.
418 AP EAMCET Chapterwise Mathematics

20. The equation of the curve passing through Put ‘c’ value in Eq. (i)
y = −  x +  +
(1, 2) and whose tangent at any point (x , y) 2 1 26 −3 3 x
e .e
makes an angle tan −1 (2 x + 3 y) with the 3 3 9
X -axis is ……… [22 Sep. 2020, Shift-II] 9 y = − 2 (3x + 1) + e 3 x − 3 (26)
(a) 6 x + 9y + 2 = 26 e 3x − 3 6 x + 9 y + 2 = 26 ⋅ e 3 x − 3
(b) 6 x + 9y − 2 = 26 e 3 x − 3 Hence, option (a) is correct.
(c) 6 x + 9y + 2 = 26 e 3 x + 3
(d) 6 x + 9y − 2 = 26 e 3 x + 3
21. The general solution of the differential
equation tan(y) dx + sec 2 (y) ⋅ tan(x) dy = 0 is
Sol. (a) [23 Sep. 2020, Shift-I]
P= (1, 2) (a) sin( y) ⋅ tan( x) = c (b) sin( x) ⋅ tan( y) = c
Inclination (θ) = tan−1 (2x + 3y) (c) sin( x) + tan( y) = c (d) sin( x) − sin( y) = c
tanθ = 2x + 3y
Sol. (b)
∴ Slope (m) = tanθ
Given differential equation is,
dy tan ydx + sec2 y.tan xdy = 0
= 2x + 3y
dx tan ydx = − sec2 y tan xdy
dy
− 3y = 2x 1 − sec2 y
dx dx = dy
tan x tan y
∴ P = − 3 and Q = 2x
sec2 y
IF = e ∫ ⇒ cot xdx = −
= e∫
P dx −3 dx dy
= e −3 x tan y
General soution is,
Integrating on both sides,
y(IF) = ∫ Q (IF) dx + c sec2 y
∫ cot xdx = − ∫ tan y dy
y(e −3 x ) = ∫ 2x. e −3 x dx + c
⇒ log sin x = − log tan y + log c
y(e −3 x ) = 2x ⇒ logsin x + logtan y = log c
dx − ∫  (2x)∫ e −3 x dx  dx + c
d
∫e
−3 x
 dx  ⇒ logsin x ⋅ tan y = log c
e −3 x
e −3 x ∴ sin x ⋅ tan y = c
−3 ∫ −3
= 2x. − 2. dx + c
22. Find the solution of the differential equation
− 2 −3 x 2 −3 x
= xe + ∫ e dx + c given below
3 3
dy
− 2 −3 x 2 e −3 x + y ⋅ cosec 2(x) = cosec 2(x) ⋅ cot(x)
= xe + . + c dx
3 3 −3
[23 Sep. 2020, Shift-I]
− 2 −3 x 
e  x +  + c
1
ye −3 x = (a) yecot x = (1 + cot x) e − cot x + c
3  3
(b) ye − cot x = (1 − cot x)e − cot x + c
Multiply by e 3 x on both sides (c) yecot x = (1 + cot x) ecot x + c
y = −  x +  + c. e 3 x (d) ye − cot x = (1 + cot x) e − cot x + c
2 1
… (i)
3 3
Sol. (d)
Since, it is given that Eq. (i) passing through dy
(1, 2) + y cosec2 x = cosec2 x.cot x
dx
−2  1
2= 1 +  + c. e
3(1 )

3  3 We have,
−8  dy + Py = Q then, IF = e ∫ Pdx 
2= + c ⋅ e3  
9  dx 
26 Solution is y (IF) = ∫ Q(IF) dx + c
= ce 3
9
26 Here, P = cosec2 x , Q = cosec2 x ⋅ cot x
⇒ c = e −3
IF = e ∫ = e ∫
cosec 2 x dx
⇒ IF = e − cot x
Pdx
9
Differential Equations 419

Solution is y (IF) = ∫ Q(IF) dx + c Sol. (a)


f (x) f ′(x)
y e − cot x = ∫ (cosec2 x.cot x). e − cot x dx + c Since, =0
f ′(x) f ′′ (x)
Put − cot x = t ⇒ f (x) f ′′ (x) − ( f ′(x))2 = 0
cosec2 x dx = dt ⇒ f (x) f ′′ (x) = ( f ′(x))2
y. e − cot x = ∫ − te t dt + c f ′′ (x) f ′(x)
⇒ =
ye − cot x = − ∫ t. e t dt + c f ′(x) f (x)
On, integrating both sides, we get
ye − cot x = − (t − 1) e t + c
f ′′ (x) f ′(x)
y e − cot x = − (− cot x − 1) e − cot x + c ∫ f ′(x) dx = ∫ f (x) dx
ye − cot x = (cot x + 1). e − cot x + c
⇒ log e ( f ′(x)) = log e ( f (x)) + c
23. If c is a parameter, then the differential
equation of the family of curves x = c(y + c) is 2 2 Q f(0) = 1 and f ′(0) = 2
[20 April 2019, Shift-I] ∴ c = log e 2
3 2 log e ( f ′(x)) = log e ( f (x)) + log e 2
(a) x   + y  − 1 = 0
dy dy
 dx   dx  ⇒ f ′(x) = 2 f (x)
3 2 f ′(x)
(b) x   − y  + 1 = 0 ⇒ =2
dy dy
 dx   dx  f (x)
3 2
f ′(x)
(c) x   + y  + 1 = 0
dy dy
 dx   dx 
⇒ ∫ f (x)
dx = ∫ 2 dx ⇒ log e f (x) = 2x + c
3 2
f(0) = 1, so c = 0
(d) x   − y  − 1 = 0
dy dy Q
 dx   dx  Q log e f (x) = 2x ⇒ f (x) = e 2 x
Sol. (d) Hence, option (a) is correct.

Given equation of the family of curves


x 2 = c(y + c)2 25. The order of the differential equation
⇒ x = c (y + c) …(i) corresponding to the family of parabolas
whose axes are along the X -axis and whose
[on taking square root both sides]
foci are at the origin, is
Now, differentiate both sides w.r.t. x, we get
dy dx [20 April 2019, Shift-II]
1= c ⇒ c = …(ii) (a) 4 (b) 3 (c) 2 (d) 1
dx dy
From Eqs. (i) and (ii) on eliminating ‘c’, we get Sol. (d)
 dx    dx  
2
The equation of family of parabolas with focus
x =   y +   
 dy    dy   and the X-axis as axis is
 
3 2 y 2 = 4a (x – a), where a is a parameters …(i)
x   = y   + 1
dy dy
⇒ Differentiating w.r.t.x, we get
 dx   dx 
dy y dy
3 2 2y = 4a ⇒ a =
x   − y   − 1 = 0
dy dy dx 2 dx

 dx   dx  Substituting the value of a in Eq. (i), we get
Hence, option (d) is correct. dy  y  dy
y 2 = 2y x– 
24. If f (x), f ′ (x) f ′′(x), are positive functions and dx  2 dx
f (0) = 1, f ′ (0) = 2, then the solution of the ⇒ y2 = y
dy  dy 
 2x – y 
f (x) f ′ (x) dx  dx 
differential equation = 0 is
f ′ (x) f ′′(x) 2
y  – 2x
dy dy
⇒ + y=0
[20 April 2019, Shift I]  dx  dx
(a) e 2 x (b) 2 sin x + 1 It is clear from the differential equation is that
(c) sin2 x + 2 x + 1 (d) e 4 x the orders of equation is 1.
420 AP EAMCET Chapterwise Mathematics

26. The general solution of the differential Sol. (a)


  y2   Let y = Ax 2 + Bx + C
 2 φ 2 
y x   Differentiate w.r.t. x, we get
equation yy ′ = x  2 + , where φ is an
x  y2   dy
= 2Ax + B
 φ′  2   dx
  x  
Again, differentiate w.r.t. x, we get
arbitrary function, is [20 April 2019, Shift-II] d2 y
= 2A
 y2   y2  dx 2
(a) x φ  2  = cy (b) x2 φ  2  = c
x  x  Again, differentiate w.r.t. x, we get
 y2   y2  d3 y
(c) x2 φ  2  = cy2 (d) φ  2  = cx2 =0
x  x  dx 3
This is required equation.
Sol. (d)
Given, differential equation 28. The solution of the equation
dy
 y  2
+ 2 y tan x = sin x satisfying y = 0 when
 2 φ 2   dx
y x 
y y′ = x  2 + , π
x  y2   x = , is
 φ′   3 [21 April 2019, Shift-I]
  x 2  
(a) y = 2 sin2 x + cos x − 2
where φ is an arbitrary function. (b) y = 2 sin2 x − cos x − 2
2
(c) y = 2 cos 2 x − sin x + 2
φ  
y
y dy y 2
 x (d) y = 2 cos x − sin2 x − 1
⇒ = + 2
x dx x 2 Sol. (a)
φ′  
y
 x Given, differential equation is
Let y = vx, where v is function of x. dy
+ 2y tan x = sin x
dy dv dx
⇒ =v+ x
dx dx Here, P = 2tan x and Q = sin x
∴ IF = e ∫
2tan x dx
φ(v2)
v  v + x  = v2 +
dv
So,
 dx  φ′(v2) = e 2ln| sec x | = eln sec
2
x
= sec2 x
dv φ(v ) 2
Now, required solution is,
⇒ vx =
dx φ′(v2) y × IF = ∫ Q ⋅ IF dx + c
φ′(v2) dx
⇒ ∫v dv = ∫ ⇒ y × sec2 x = ∫ sin x ⋅ sec2 x dx + c
φ(v2) x
1 ⇒ y sec2 x = ∫ secx tan x + c
log e φ(v2) = log e x + log e ( c) = log e ( c x)
2 ⇒ y sec2 x = sec x + c
y  2 π
⇒ φ(v2) = cx 2 ⇒ φ  2  = cx 2 Given, y = 0 when x =
x  3
Hence, option (d) is correct. ⇒ 0 = 2+ c
⇒ c=−2
27. The differential equation of all parabolas ∴ y sec2 x = sec x − 2
whose axes are parallel to Y -axis is sec x 2
[21 April 2019, Shift-I] ⇒ y= −
sec2 x sec2 x
d3y d2y ⇒ y = cos x − 2cos2 x
(a) 3 = 0 (b) 2 = 0
dx dx ⇒ y = cos x − 21 ( − sin2 x)
2
d 2 y dy d2y
(d) y 2 +   = 0
dy
(c) 2 + =0 ⇒ y = cos x − 2 + 2 sin2 x
dx dx dx  dx 
⇒ y = 2 sin2 x + cos x − 2
Differential Equations 421

As mentioned above, ye ∫
P ( x ) dx
29. The differential equation formed by = A(x)
eliminating a and b from the equation By Eq. (i), we get
y = e x (a cos x + bsin x) is [21 April 2019, Shift-II]
A(x) = ∫ Q(x)e ∫
P ( x ) dx
dx
d 2 y dy d2y dy
(a) 2 2 + − 2y = 0 (b) 2 + 2 − 2y = 0
A′(x) = Q(x)e ∫
P ( x ) dx
dx dx dx dx Then,
d 2 y dy d2y dy
(c) 2 2 − + 2y = 0 (d) 2 − 2 + 2y = 0
dx dx dx dx 31. Statement (I) The elimination of arbitrary
Sol. (d) constants from α ,β and γ from y = (α + β + γ) x
results in a differential equation of order
Given, three.
y = e x (a cos x + b cos x) …(i)
Statement (II) The elimination of arbitrary
Differentiating w.r.t. x, we get
constants α, β and γ from
dy
= e x (a cos x + b sin x) + e x (− a sin x + b cos x) y = αx + β sin x + γe x results in a differential
dx
dy equation of order three. [22 April 2019, Shift-I]
⇒ = y + e x (− a sin x + b cos x) …(ii) [by Eq. (i)]
dx (a) I is true and II is false (b) I is false and II is false
(c) I is true and II is true (d) I is false and II is true
Again differentiating w.r.t. x, we get
d 2 y dy Sol. (d)
= + e x (− a cos x − b sin x)
dx 2 dx Statement I
+ e x (− a sin x + b cos x) ⇒ y = (α + β + γ) x
2
d y dy dy Differentiate both sides w.r.t.x, we get
⇒ = − e x (a cos x + b sin x) + − y
dx 2 dx dx dy
= (α + β + γ) = k
[by Eq. (ii)] dx
d2 y dy Here, order = 1
⇒ = 2 − y − e (a cos x + b sin x)
x

dx 2 dx So, statement I is false.


d2 y dy Statements II : y = αx + β sin x + γe x
⇒ =2 − y− y [by Eq. (i)]
dx 2 dx Differentiate both sides w.r.t.x, we get
d2 y dy d2 y dy dy
⇒ 2
= 2 − 2y ⇒ 2 + 2y − 2 = 0 = α + β cos x + γe x …(i)
dx dx dx dx dx
Again, differentiating both sides w. r. t. x, we get
30. If y = A(x)e ∫ is a solution of
− Pdx

d2 y
dy = − β sin x + γe x …(ii)
+ P(x) y = Q(x), then A ′ (x) = dx 2
dx d3 y
[21 April 2019, Shift-II] ⇒ = − β cos x + γe x …(iii)
dx 2
(a) e ∫ (b) Q( x)e ∫
Pdx − P dx
Subtracting Eq. (ii) from Eq. (iii), we get
(c) ∫ Q( x)e ∫ (d) Q( x)e ∫ d3 y d2 y
P dx P dx
dx − = − β cos x + γe x + β sin x − γ e x
dx 2 dx 2
Sol. (d) = β sin x − β cos x
Given, y = A(x)e ∫
− Pdx = β(sin x − cos x)
Hence, order = 3
Since, given differential equation is linear
differential equation. So, Statement II is true.

IF = e ∫
P ( x ) dx
∴ 32. The general solution of
dy
Now, solution of differential equation is sin y. = cos y(1 − x cos y) is
dx [22 April, 2019 Shift I]
y ⋅ (IF) = ∫ Q(x) × (IF)dx
(a) sec y = x − 1 − ce x (b) sec y = x + 1 + ce x
⇒ y × e∫ = ∫ Q(x) ⋅ e ∫ (c) sec y = x + e x + c (d) sec y = x − e x + c
P ( x ) dx P ( x ) dx
⋅ dx …(i)
422 AP EAMCET Chapterwise Mathematics

Sol. (b) ⇒ y = 2y′(x + 2yy′)


2
Given differential equation is  y 
dy ⇒  − x  = 2yy′
sin y = cos y(1 − x cos y)  2y′ 
dx
dy ⇒ (y − 2y′ x)2 = 8(y′)3 (y)
⇒ sin y = cos y − x cos2 y Hence, option (c) is correct.
dx
sin y dy 1
⇒ = −x 34. The solution of the differential equation
cos2 y dx cos y x + y − 1 dy x + y +1
dy = , given that y = 1 when
⇒ sec y tan y = sec y − x x + y − 2 dx x + y + 2
dx
Let sec y = t
x = 1; is [22 April 2019, Shift-II]
( x + y)2 − 2
⇒ sec y tan y
dy dt
= (a) 2 ( y − x) + log =0
dx dx 2
dt ( x + y)2 − 2
∴ =t − x (b) log = ( x − y)2
dx 2
dt ( x − y)2 + 2
⇒ + (− t) = − x (c) log + 2( y − x) = 0
dx 2
IF = e ∫ = e − x
− dx
∴ ( x + y)2 − 2
(d) ( x − y) + log =0
Now, required solution 2
⇒ t(IF) = ∫ (− x) (IF)dx + c Sol. (a)
sec y (e − x ) = ∫ (− x)e − x dx + c Given,
x + y − 1 dy x + y+1
= (xe − x ) − ∫ e − x dx + c =
x + y − 2 dx x + y + 2
= xe − x + e − x + c Put x+ y=v
⇒ sec y = x + 1 + ce x dy dv
∴ 1+ =
33. The degree and order respectively of the dx dx
dv (v + 1) ( v − 2)
differential equation of the family of the ∴ −1 =
dx (v + 2) (v − 1 )
curves represented by y = c (x + c) are
dv (v + 1) (v − 2)  (v2 − 2) 
(Here, C is a parameter) ⇒ = + 1 = 2 2 
[22 April 2019, Shift-II] dx (v − 1) (v + 2)  v + v − 2
(a) 1, 3 (b) 2, 3 (c) 3, 1 (d) 2, 2 v2 − 2 + v  1 2 (x + y) 
∴ dv = 2dx or 1 + .  dν = 2dx
Sol. (c) v2 − 2  2 (x + y)2 − 2
Given, On integrating,
y= c(x + 1
c) v + log{(x + y)2 − 2} = 2x + c
1 2
∴ y′ = c⋅ 1
2 x+ c x + y + log{(x + y)2 − 2} = 2x + c
2
1 When x = 1, y = 1
Now, yy′ = c⋅ ⋅ c x+ c
2 x+ c 1 1
⇒ 2 + log{2} = 2 + c ⇒ c = log {2}
c 2 2
yy′ =
2 1 1
∴ x + y + log{(x + y)2 − 2} = 2x + log(2)
⇒ c = 2yy′ 2 2
Since, y 2 = c(x + c) (x + y) − 2
2
⇒ 2(y − x) + log =0
y = 2yy′(x +
2
c) 2
Differential Equations 423

35. The differential equation corresponding to the 37. If m and n are respectively the order and
family of curves y = e x (A cos x + Bsin x) is degree of the differential equation of the
[23 April 2019, Shift-I] family of parabolas with focus at the origin
(a) y′′ + y′ + y = 0 (b) y′′ + 2 y′ + 2 y = 0 and X-axis as its axis, then mn − m + n =
(c) y′′ − 2 y′ + 2 y = 0 (d) y′′ − 2 y′ − 2 y = 0 [22 April 2018, Shift-I]
(a) 1 (b) 4
Sol. (c)
(c) 3 (d) 2
Given, curve y = e x (A cos x + Bsin x) …(i)
So, y′ = e x (− A sin x + B cos x) + e x
Sol. (c)
(A cos x + Bsin x) The equation of the family of parabolas with
⇒ y′ − y = e x (− A sin x + B cos x) …(ii) focus at the origin and X-axis as its axis is given
by
So, y ′′ − y′ = e x (− A cos x − Bsin x)
y 2 = 4a(x + a) = 4ax + 4a 2 … (i)
+ e x (− Asin x + B cos x) ∴ 2y
dy
= 4a
⇒ y ′′ − y′ = − y + y′ − y dx
[from Eqs. (i) and (ii)] 1 dy
⇒ a= y … (ii)
⇒ y ′′ − 2y′ + 2y = 0 2 dx
Hence, option (c) is correct. From Eqs. (i) and (ii), we have
2

+ y 2  
dy dy
36. The solution of the differential equation y 2 = 2xy
dx  dx 
dy  y
x = y − x tan   is (Here, k is an arbitrary ∴ order = m = 1 and degree = n = 2
dx  x
constant) [23 April 2019, Shift-I] ∴ mn − m + n = 1 × 2 − 1 + 2 = 3.

(a) x = ysin −1  k (b) y = x sin −1  k


    38. The general solution of
 x  x
 x
 x
 x
(c) x sin y + k = 0 (d) y = x cos (kx) 1 + e y  dx + e y 1 −  dy = 0 is
   y
Sol. (b)   [22 April 2018, Shift-I]
y x
Given differential equation is (a) ye x + x = c (b) ye y − x = c
dy y x x
x = y − x tan (c) ye y + y = c (d) ye y + x = c
dx x
= − tan  
dy y y Sol. (d)

dx x  x
Given,
Let y = v⋅ x
 x
dy dv (1 + e x / y)dx + e x / y 1 −  dy = 0
⇒ =v+ x  y
dx dx
dv  x
So, v+ x = v − tan v ⇒ (1 + e x / y)dx = − e x / y 1 −  dy
dx  y
dv dx  x
⇒ =− − e x / y 1 − 
tan v x dx  y
 1 ⇒ = … (i)
⇒ ∫ cot v dv = ∫  − x  dx dy (1 + e x / y)
Putting x = vy
⇒ log |sin v | = − log |x | + log k
k Differentiating w.r.t. y,
⇒ sin v = dx d(y) dv
x =v + y
dy dy dy
v = sin−1  
k

 x dx dv
⇒ =v+ y
y = x sin−1  
k dy dy

 x dx
Putting value of and x = vy in Eq. (i)
Hence, option (b) is correct. dy
424 AP EAMCET Chapterwise Mathematics

 x Given family of parabola is


e − x / y 1 − 
dy  y y 2 = 4a(x + a) …(i)
=
dx 1+ e x/y
⇒ 2y
dy
= 4a
dv − e v (1 − v) dx
v+ y = 1 dy
dy 1 + ev ⇒ a= y …(ii)
2 dx
dv e + ve
v v
⇒ y =− −v From Eqs. (i) and (ii),
dy 1 + ev 2
y 2 = 4 x  y  + 4  y 
1 dy 1 dy
dv − e v + ve v − v − ve v  2 dx   2 dx 
⇒ y =
dy 1 − ev 2
y  + 2x
dy dy
dv − [v + e ] v ⇒ − y=0
⇒ y =  dx  dx
dy 1 + ev
40. The general solution of
1 + e v  dy
⇒  v + e v  dv = − y dy
  + y tan x = 2 x + x 2 tan x
dx
Integrating both sides, we get [22 April 2018, Shift-II]
1 + ev dy (a) y − x2 = c sec x (b) ycos x = x2 sec x + c
∫ v + e v dv = − ∫ y (c) ysec x = x + c cos x (d) y = x2 + c cos x
2

1 + ev Sol. (d)
⇒ ∫v+ e v
dv = − log y + log c
Given differential equation,
Put v + ev = t dy
+ y(tan x) = 2x + x 2 tan x …(i)
(1 + e v) dv = dt dx
dt
Thus, ∫ t = − log y + log c Q The differential equation is in linear form, so
Integrating factor (I.F.) = e ∫
tan x dx
= sec x
log t = − log y + log c
So, solution of given differential Eq. (i), is
⇒ log(v + e v) = − log y + log c
y(sec x) = ∫ (2x + x 2 tan x) sec x dx
⇒ log y (v + e v) = log c
Put value of v=
x = ∫ 2x sec x dx + ∫x
2
tan x sec x dx
y
x x / y
= ∫ 2x sec xdx + x 2 sec x − ∫ 2x sec xdx
log y  + e  = log c
y  ⇒ y sec x = x 2 sec x + c
x  ⇒ y = x 2 + c cos x.
⇒ y  + ex / y = c
y 
41. If l and m are the degree and the order
⇒ x + ye x / y = c respectively of the differential equation of the
39. The differential equation corresponding to family of all circles in the XY plane with
radius 5 units, then 2 l + 3 m =
the family of parabolas y 2 = 4 a(x + a), where
[23 April 2018, Shift-I]
a is the parameter, is [22 April 2018, Shift-II]
2
(a) 5 (b) 10 (c) 15 (d) 7
(a) y   + 2 x + y = 0
dy dy
 dx  Sol. (b)
dx
2 Family of all circles in the xy-planewith radius
 dy  dy
(b) y   − 2 x − y = 0
 dx  5 units with center (x1 , y1 ) is
dx
2 (x − x1 )2 + (y − y1 )2 = 52
(c) y   + 2 x   − y = 0
 dy  dy
⇒ (x − x1 )2 + (y − y1 )2 = 25 …(i)
 dx   dx 
dy differential w.r.t ‘x’
(d) y = 2 x ⇒ 2(x − x1 ) + 2(y − y1 ) y ′ = 0 …(ii)
dx
⇒ (x − x1 ) = − (y − y1 ) y ′
Sol. (c)
Now Eq. (ii) diff again w.r.t ‘x’,
Differential Equations 425

we get dt dt
2 + 2(y ′′ (y − y1 ) + (y′)2 ] = 0
I= ∫ t (t + 1) = ∫ 2 2

t + t +   −  
2 1 1
1 − (y′)2  2  2
⇒ (y − y1 ) =
y ′′ dt t
Sub. values in Eq. (i)
= ∫ 1 1 
2 2
= log
t +1
t +  −  
1 + (y′)2 
2
1 + (y′)2   2  2
= . y ′ +   = 25
 y ′′   y ′′  = log
cos 2x
(1 + y′2 ) 1 + (y′)2 1 + cos 2x
⇒ .(y′)2 + = 25
(y ′′) 2
(y ′′)2 log
cos 2 x
cos 2x cos 2x
1 + cos 2 x
Now, IF = e = =
(1 + y′) 3
1 + cos 2x 2. cos2 x
⇒ = 25
(y ′′)2 Now, solution of differential equation is
⇒ 25(y ′′)2 = [1 + (y′)2 ]3 y. IF = ∫ (θ . IF) dx
So, order = 2and degree = 2
cos 2x cos 2x
2 cos2 x ∫
∴ l = 2 and m = 2 ⇒ y = cos2 x . dx
2 cos2 x
Now, 2l + 3m = 2 × 2 + 3 × 2
⇒ y 
cos 2x 
 cos2 x  ∫
= 4 + 6 = 10  = cos 2x dx
π π  1 − tan2 x  1
42. If − < x < , then the general solution of ⇒y  =
sin 2x
+ c1
4 4  1 + tan2 x  cos2 x 2
the differential equation
(1 − tan2 x) sin 2x
dy ⇒ y = + c1
cos 2 x ⋅ − (tan 2 x) y = cos 4 x is sec2 x. cos2 x 2
dx [23 April 2018, Shift-I] sin 2x + 2 c1
1  tan2 x + c 1 cos 2 x + c  ⇒ y (1 − tan2 x) =
(a) y = (b) y = 2
2  1 − tan2 x  2  1 − tan2 x  sin 2x + c
⇒ y=
1 sin2 x + c  1  sin x + c  2 (1 − tan2 x)
(c) y = (d) y =
2  1 − tan2 x  2  1 − tan2 x 
43. The order and the degree of the differential
Sol. (c) dy
equation y = px + a 2p 2 + b2 , (where p = )
dy dx
cos2 x − tan 2x ⋅ y = cos4 x
dx are respectively . [23 April 2018, Shift-II]
dy tan 2 x (a) 2, 1 (b) 1, 2 (c) 1, 1 (d) 2, 2
⇒ − y = cos2 x
dx cos2 x Sol. (b)
the above differential equation is in the form of.
Given differential equation,
dy
+ py = q,
a 2 p 2 + b 2 ,  where, p = 
dy
dx y = px +
 dx 
− tan 2x
where p = ⇒ a 2 p 2 + b 2 = (y − px)
cos2 x
⇒ a 2 p 2 + b 2 = y 2 + p 2 x 2 − 2xyp
and q = cos2 x
tan 2 x ⇒(x 2 − a 2) p 2 − 2xy p + y 2 − b 2 = 0
∫−
Now, IF = e ∫ = e
p dx dx
cos 2 x So, order and degree of the given differential
equation is 1 and 2 respectively.
tan 2x − sin 2x
Let I = ∫− dx = ∫ cos 2x. cos dx
cos2 x 2
x 44. The solution of the differential equation
dx
− 2 sin 2x + 2 yx = 2 y which passes through the point
=∫ dx dy
cos 2x (1 + cos 2x)
(2 , 0) is [23 April 2018, Shift-II]
put cos 2x = t (a) ( x − 1) = 2e y
2
(b) ( x − 1) = 2e − y
2

− 2 sin 2x dx = dt (c) ( x − 1) = e y
2
(d) ( x − 1) = e − y
2
426 AP EAMCET Chapterwise Mathematics

d 2 y dy  dy
Sol. (d) ⇒ xy + x − y = 0
Given differential equation is, dx 2 dx  dx 
2
dx d2 y
+ 2y ⋅ x = 2y + x   − y
dy dy
⇒ xy = 0.
dy dx 2  dx  dx
dx
⇒ = 2y (1 − x)
dy 46. The general solution of the differential
dx dx x
⇒ ∫ 1 − x = ∫ 2y dy equation + = x 2 is
dy y [24 April 2018, Shift-I]
⇒ − log(x − 1) = y 2 + c …(i) 1
(a) = cx − y log x
1
(b) = cy + x log x
Since, curve (i) passes through the point (2, 0), y x
so 1 1
(c) = cy − y log y (d) = cx + y log x
c=0 x y
So, curve will be (x − 1) = e − y
2

Sol. (c)
45. If a and b are arbitrary constants, then the Given differential equation is
dx x
x 2 y2 + = x2
differential equation having 2 + 2 = 1 as its dy y
a b
general solution is [24 April 2018, Shift-I] On dividing by x 2 , we get
3 1 dx  1   1 
+     =1
2
 d 2 y  dy 
2
(a)  2  = 1 +    x 2 dy  y   x 
 dx   dx 
  1
2 let =t
d y dy x
(b) ( x2 − y2 ) 2 − 2 xy − y=0
dx dx 1 dx dt 1 dx dt
2
− 2 = ⇒ =−
d2y x 2 dy
(c) xy 2 + x   − y
dy dy x dy dy dy
=0
dx  dx  dx dt 1
⇒ − + () t =1
d2y dy dy y
(d) x2 2 + 2 x − 2y = 0
dx dx dt  1 
⇒ +  −  () t = −1
Sol. (c) dy  y 
1
x2 y2 ∫ − dy
1
+ =1 IF =e y
= e − log y =
a2 b2 y
On differentiating, So, solution is
2x 2y dy y dy − b 2 1  1 
+ 2 =0 ⇒ = 2 t ⋅   = ∫ (− 1)  dy + c
a 2
b dx x dx a  y  y
Again differentiating w.r.t. x t
⇒ = − log y + c
 x dy − y  y
y d 2 y dy  dx  ⇒ t = − y log y + cy
+ =0
x dx 2 dx  x 2 
  1
  ⇒ = cy − y log yz
x
29
Miscellaneous
1. If f (x) ∈Q [ x ] be a non-zero polynomial such Subtract Eqs. (i) from (ii)
f (a) − f (b) f (b) − f (a)
that all its roots are irrational, then the f (a) − f (b) = α (a − b); α = =
a−b b−a
degree of f (x) is [17 Sep. 2020, Shift-I]
Put, α in Eq. (i)
(a) an even number (b) an odd number f (b) − f (a) 
(c) 0 (d) can’t determine f (a) =  ×a+β
 b−a 
Sol. (a) f (a)[b − a]= af (b) − a f (a) + β (b − a)
Irrational roots exist in pair bf (a) − af (a) = af (b) − a f (a) + β (b − a)
So, the degree of f (x) will be an even number bf (a) − af (b)
β=
(b − a)
2. What is the remainder of 2 x 3 − 5 x 2 + 7 when
f (b) − f (a) x bf (a) − af (b)
divided with (x − 2)? [17 Sep. 2020, Shift-II] ∴r(x) = α x + β = +
b−a (b − a)
(a) − 3 (b) − 2 (c) 3 (d) 2
xf (b) − xf (a) + bf (a) − af (b)
=
Sol. (c) b−a
Using remainder theorem, remainder is P (x = 2). (x − a) f (b) + (b − x) f (a)
r(x) =
So, remainder is, b−a
P (x = 2) = 2 × 23 − 5 × 22 + 7 (x − a) f (b) − (x − b) f (a)
r(x) =
b−a
= 16 − 20 + 7 = 3
Hence, option (c) is correct.
3. Let f (x) be a polynomial and a, b be distinct
4. What is the quotient of x 3 − 5 x 2 + 2 x + 7
real numbers. Then the remainder in the when divided with (x − 1) ? [21 Sep. 2020, Shift-I]
division of f (x) by (x − a)(x − b) is (a) x2 + 4 x − 2 (b) x2 − 4 x + 2
[18 Sep. 2020, Shift-II] (c) x2 + 4 x + 2 (d) x2 − 4 x − 2
( x − a) f ( a) − ( x − b ) f ( b )
(a)
a− b Sol. (d)
( x − a) f ( b ) − ( x − b ) f ( a) On dividing x 3 − 5x 2 + 2x + 7 with x − 1, we have
(b)
a−b x2 − 4x − 2
( x − a) f ( b ) − ( x − b ) f ( a)
(c) x − 1)x 3 − 5x 2 + 2x + 7
b−a
x3 − x2
( x − a) f ( a) − ( x − b ) f ( b )
(d) – +
b−a
− 4 x 2 + 2x + 7
Sol. (c) − 4x2 + 4x
Let f (x) = (x − a)(x − b). q (x) + r(x) + –
Let r (x) = αx + β [Q deg r(x)< deg. of divisor] − 2x + 7
∴ f (x) = (x − a)(x − b). q (x) + αx + β − 2x + 2
f (a) = α a + β …(i) + –
f (b) = α b + β …(ii) 5
428 AP EAMCET Chapterwise Mathematics

∴ The required quotient is (x 2 − 4 x − 2) 8. The sum of the four digit even numbers that
Hence, option (d) is correct. can be formed with the digits 0, 3, 5, 4 with
out repetition is [22 April 2018, Shift-I]
5. Let f (x) = x − 9 x + 29 x + 54 x − 54 suppose
5 4 3
(a) 14684 (b) 43536 (c) 46526 (d) 52336
f (x) = (x − a)n Q(x), where Q(x) is a polynomial
Sol. (b)
not divisible by (x − a). Then
Four digit numbers which are even and formed
[21 Sep. 2020, Shift-II] with the digits 0, 3, 5, 4 without recepetion are
(a) a = 1, n = 2 (b) a = 3, n = 1 3054, 3504, 5034, 5304, 3450, 3540, 4350, 4530,
(c) a = 3, n = 3 (d) a = −1, n = 2 5340, 5430
Sol. (*) ∴ Required sum
Question is wrong. = 3054 + 3504 + 5034 + 5304 + 3450
+ 3540 + 4350 + 4530 + 5340 + 5430 = 43536
6. The average marks of boys in a class is 40
and that of girls is 45. The average marks of 9. The number of three digit numbers in which
both boys and girls combined is 42. Then the 9 appears only in one place is
percentage of boys in the class is [23 April 2018, Shift-II]
[21 Sep. 2020, Shift-II] (a) 243 (b) 234 (c) 217 (d) 225
(a) 60% (b) 30% (c) 40% (d) 50%
Sol. (d)
Sol. (a) The number of three digit numbers in which
Let the number of boys and girls in the class are 9 appears at hundred place only is
m and n respectively, so according to given = 1 × 9 × 9 = 81
informations, Similarly, the number of three digit numbers in
Total marks of boys = 40 m which 9 appears at ten or unit place only is
and total marks of girls = 45 n = (8 × 1 × 9) × 2 = 144
and total marks of boys and girls = 42(m + n) So, required number = 81 + 144 = 225.
∴ 40m + 45n = 42(m + n)
m n
10. Let m be a natural number such that
⇒ 2m = 3n ⇒ = = λ (let) 20000 < m < 60000 and let k be the sum of all
3 2
∴ m = 3λ and n = 2λ the digits in m. Then the number of numbers
So, the percentage of boys in the class is m for which k is even, is [24 April 2018, Shift-I]
m 3λ 3 (a) 19909 (b) 19989 (c) 18999 (d) 19999
×100 = × 100 = × 100 = 60%
m+ n 3λ + 2λ 5 Sol. (d)
Hence, option (a) is correct. Let us consider 10 successive five digit numbers
a1 a 2a 3a 4 0
7. The population of a city grows at the annual
rate of 3%. What percentage increase is a1 a 2a 3a 41
expected in 5 yr? [21 Sep. 2020, Shift-II] a1 a 2a 3a 4 2
(a) 12.9% (b) 13.9% .................
(c) 14.9% (d) 15.9% a1 a 2a 3a 4 9
Sol. (d) where, a1 , a 2 , a 3 , a 4 are some digits. We see that
Let the population in begining is P0 , then half of these 10 numbers i.e. 5 have an even
according to the question percentage increase in sum of digits.
5 yr expected is The first digit a1 can takes 2, 3, 4, 5 and each of
5 the digits a 2 , a 3 , a 4 can takes 10 different values
P0 1 +
3 
 − P0   103 5  the units place digit can assume only 5 different
 100 
× 100 =    − 1 × 100 values of which the sum of all digits is even.
P0 
 100  So, value of K is = 4 × 103 × 5 − 1
= [(1.03)5 − 1] × 100 = [1.159 − 1] × 100 = 159
. % [Q 20,000 will not include]
Hence, option (d) is correct. = 19999.
Practice Set 1 429

Practice Set 1
1. The value of ∫
dx
is 8. The area of figure bounded by y = e x , y = e − x
x 1 − (log x)2 and the straight line x = 1 is
(a)  e +  sq unit (b)  e −  sq unit
−1 1 1
(a) cos (log x + c ) (b) x log(1 − x ) + c
2
 e  e
1
(c) cos −1(log x + c ) (d) sin−1(log x) + c
(c)  e + − 2  sq unit (d)  e + + 2  sq unit
1 1
2
 e   e 
x2
2. ∫ (x 2 + 2)(x 2 + 3) dx is equal to 9. Match the following columns.
Column I Column II
(a) − 2 tan−1 
x  −1  x 
 + 3 tan   +c (A) If a + b, b + c and c + a are 1. a ⋅ b = c
 2  3
coplanar, then
(b) 2 tan−1 
x  −1  x 
 + 3 tan   +c If a × b, b × c and c × a are
 2  3 (B) 2. a, b, c are
coplanar, then coplanar
(c) − 2 tan−1 x + 3 tan−1 x + c
3. a ⋅c = b
(d) None of the above Codes
3. The degree and and order of the differential A B A B
(a) 1 3 (b) 1 2
equation of the family of all parabolas whose (c) 2 1 (d) 2 2
axis is X -axis, are respectively.
(a) 2, 1 (b) 1, 2 (c) 3, 2 (d) 2, 3
10. If D , E , F be the middle points of the sides
BC, CA and AB of the triangle ABC, then
4. The differential equation satisfied by the AD + BE + CF is
1  (a) a zero vector (b) a unit vector
family of curve y = ax cos  + b , where a , b
x  (c) 0 (d) None of these
are parameters, is 11. Let a , b , c be three vectors such that a ≠ 0 and
(a) x2 y2 + y = 0 (b) x4 y2 + y = 0
a ⋅ b = 2a ⋅ c, |a| = |c| = 1, |b| = 4 and|b × c| =
(c) xy2 − y = 0 (d) x4 y2 − y = 0 15, if b − 2c = λa . Then, λ equals
5. Integrating factor of equation (a) − 1 (b) 1 (c) − 4 (d) 2
dy
(x 2 + 1) + 2 xy = x 2 − 1 is
dx 12. The image of the point with position vector
2x x2 − 1 i$ + 3 k$ in the plane r ⋅ (i$ + $j + k$ ) = 1
(a) (b)
x2 + 1 x2 + 1 (a) $i − 2 $j + k$ (b) $i + 2 $j − k$
(c) x + 1
2
(d) None of these (c) − $i − 2 $j + k$ (d) None of these
π 2
6. The value of ∫|sin x − cos x|dx is 13. If|z|< 3 − 1 then|z 2 + 2z cos α | is
0 (a) less than 2 (b) 3 + 1
(a) 0 (b) 2( 2 − 1) (c) 2 − 1 (d) 2( 2 + 1) (c) 3 − 1 (d) None of these
π −a π −a
7. Let I1 = ∫ xf (sin x)dx , I 2 = ∫ f (sin x)dx then 14. (− 3 + i)53 where i 2 = − 1 is equal to
a a
(a) 2 53 ( 3 + 2 i ) (b) 2 52 ( 3 − 2 i )
I 2 is equal to
π  3 1 
(a) I1 (b) πI1 (c)
2
I1 (d) 2 I1 (c) 2 53  + i (d) 2 53 ( 3 − 1)
2 π  2 2 
430 AP EAMCET Chapterwise Mathematics

15. If 3 + i = (a + ib)(c + id), then 22. The existence of the unique solution of the
−1  b  −1  d 
system x + y + z = λ ,5 x − y + µz = 10 ,
tan   + tan   has the value 2 x + 3 y − z = 6 depends on
 a  c
(a) µ only (b) λ only
π π
(a) nπ + , n∈I (b) + 2 nπ, n ∈ I (c) λ and µ both (d) neither λ nor µ
6 3
π π 1 −1 1  4 2 2
(c) nπ − , n ∈ I (d) 2 nπ − , n ∈ I
6 3 23. Let A = 2 1 −3 and10 B =  − 5 0 α
   
16. Which is the correct order for a given number 1 1 1   1 −2 3 
α in increasing order? If B is inverse of matrix A, then α is
(a) log 2 α, log 3 α, loge α, log10 α
(a) −1 (b) −2 (c) 2 (d) 5
(b) log10 α,log 3 α, loge α, log 2 α
(c) log10 α,loge α, log 2 α, log 3 α 24. The inverse of a skew symmetric matrix (if it
(d) None of the above exists) is
5+ 2 5− 2 (a) a symmetric matrix
17. If x = ,y= , then (b) a skew symmetric matrix
5− 2 5+ 2 (c) a diagonal matrix
3 x 2 + 4 xy − 3 y 2 is equal to (d) None of the above
1 1
(a) (56 10 + 12 ) (b) (56 10 − 12 ) x2 y2
3 3 25. The angle of intersection of ellipse + =1
1 a2 b2
(c) (56 + 12 10 ) (d) None of these
3 and circle x 2 + y 2 = ab is
a + b a − b
18. A dictionary is printed consisting of 7 letters (a) tan−1   (b) tan−1  
 ab   ab 
words only that can be made with a letter of
a + b a − b
the word CRICKET. If the words are printed (c) tan−1   (d) tan−1  
at the alphabetical order, as in an ordinary  ab   ab 
dictionary, then the number of words before 26. The equation of parabola whose vertex and
the words CRICKET is focus are (0, 4) and (0, 2) respectively, is
(a) 481 (b) 480 (c) 530 (d) 531
(a) y2 − 8 x = 32 (b) y2 + 8 x = 32
19. If x ∈[0 , 8], then probability that (c) x2 + 8 y = 32 (d) x2 − 8 y = 32
x − 8 x + 12 ≥ 0 is
2
27. If P ≡ (x , y), F1 ≡ (3, 0), F2 ≡ (−3, 0) and
1 3
(a) (b) 16 x 2 + 25 y 2 = 400 , then PF1 + PF2 is equal to
2 4
1 (a) 6 (b) 8
(c) (d) None of these
4 (c) 10 (d) None of these
20. Four dice are rolled. The number of possible 28. The distance between the foci of a hyperbola
outcomes in which at least one die shows 2 is is double the distance between its vertices
(a) 1296 (b) 625 (c) 671 (d) 1023 and the length of its conjugate axis is 6. The
a1 b1 c1 equation of the hyperbola referred to its axes
as axes of coordinates are
21. If ∆ = a 2 b2 c2 and A , B and C, denote the
(a) 3 x2 − y2 = 3 (b) x2 − 3 y2 = 3
a3 b3 c3 (c) 3 x2 − y2 = 9 (d) x2 − 3 y2 = 9
cofactors of a , b and c, respectively, then the
A1 B1 C1 29. The equation (x + 1) − (x − 1) = (4 x − 1) has
value of the determinant A2 B2 C 2 is (a) no solution
(b) one solution
A3 B3 C 3
(c) two solution
(a) ∆ (b) ∆ 2 (c) ∆ 3 (d) 0 (d) more than two solutions
Practice Set 1 431

α β   a − ib 
30. If the roots of the equation + =1 38. tan  i log    is equal to
x − α x −β   a + ib 
be equal in magnitude but opposite in sign, (a)
ab
(b)
ab
(c)
2 ab
(d)
2 ab
then α + β is equal to a + b
2 2
a −b
2 2
a −b
2 2
a + b2
2

(a) 0 (b) 1
(c) 2 (d) None of these 39. The inequation n! > 2n−1 is true for
(a) for all n ∈ N (b) for all n > 1
31. If 2a + 3b + 6 c = 0, then at least one root of (c) for all n > 2 (d) None of these
the equation ax 2 + bx + c = 0 lies in the
40. For all n ∈ N , cos α cos 2α cos 4αKcos(2n−1α) is
interval
equal to
(a) (1, 2) (b) (0, 1) (c) (2, 3) (d) (3, 4) sin(2 n α ) sin(2 n α )
(a) (b)
32. If α , β and γ are the roots of 2 sinα 2 n sinα
x 3 − 9 x 2 + 26 x − 24 = 0 , then Σ(α + 3)(β + 3) cos(2 n α )
(c) n (d) None of these
2 cos α
is
(a) 107 (b) 108 (c) 128 (d) 182 41. The points A(5, − 1, 1), B(7 , − 4 , 7), C (1, − 6 , 10)
and D(−1, − 3 , 4) are the vertices of a
33. The coefficient of x 4 in the expansion of
(a) square (b) rhombus
(1 + x + x + x ) is
2 3 n
(c) rectangle (d) None of these
(a) n C 4
(b) n C 4 + nC 2 42. The equation of the plane passing through the
(c) n C 4 + nC 2 + n C 4 ⋅ nC 2 line of intersection of the planes x + y + z = 1
(d) C 4 + C 2 + C1 ⋅ C 2
n n n n 2 x + 3 y − z + 4 = 0 and parallel to x-axis is
(a) y − 3 z + 6 = 0 (b) y − z + 1 = 0
34. If sum of the coefficients of the first, second (c) y − 3 z − 6 = 0 (d) y − z − 1 = 0
and third terms of the expansion of
 2 1
m 43. In ∆ABC, match the following columns.
 x +  is 46, then the coefficient of the
 x Column I Column II
a
+
b
=1 1. ∠ C = 75°
term that does not contain x is A.
b+c c+ a
(a) 84 (b) 92
= 0 2. ∠C = 60°
b c
(c) 98 (d) 106 B. +
a2 − c 2 a2 − b 2
 x 6 x 10  y4 y6
35. If 4  x 2 + + +  = y2 + + + .... , C. r : R : r1 = 1 : 2 : 3 3. ∠A = ∠B = ∠C = 60°
 3 5  2 3 4. ∠A = 60°
then Codes
(a) x y = 2 x + y
2
(b) x y = 2 x − y
2 A B C A B C
(a) 2 3 1 (b) 2 4 3
(c) x = 2 y − 1
2
(d) None of these
(c) 4 1 3 (d) None of these
1+ x 1+ x + x 1 + x + x2 + x3 2
b− c c− a a − b
36. 1 + + + + ...... is 44. In ∆ABC , + + is equal to
2! 3! 4! r1 r2 r3
equal to (a) 0 (b) abc
ex + 1 ex + 1 ex −e ex −e (c) a + b + c (d) ab + bc + ca
(a) (b) (c) (d)
x−1 x+1 x+1 x−1
45. ABCD is a rectangular field. A vertical lamp
37. If f (x) = cosh x + sinh x , then post of height 12 m stands at the corner A. If
f (x1 + x 2 + ... + x n) is equal to the angle of elevation of its top from B is 60 °
(a) f( x1 )f( x2 ) K f( xn ) and from C is 45°, then the area of the field is
(b) f( x1 ) + f( x2 ) + ..... + f( xn ) (a) 48 2 m2 (b) 12 2 m2
(c) xf( x) (c) 48 m 2
(d) 12 3 m2
(d) None of the above
432 AP EAMCET Chapterwise Mathematics

46. If sin x + sin 2 x = 1, then the value of 54. In a box containing 100 eggs, 10 eggs are rotten.
cos12 x + 3 cos10 x + 3 cos 8 x + cos 6 x − 2 is The probability 100 eggs, 10 eggs are rotten. The
probability that out of a sample of 5 eggs none
equal to
is rotten, if the sampling is with replacement, is
(a) 0 (b) −1 (c) 1 (d) 2 5 5 5 5
(a)   (b)   (c)   (d)  
9 9 1 1
θ x −1  5  10   10   5
47. If θ is an acute angle and sin = , then
2 2x
tanθ is equal to 55. Let 0 < P(A) < 1, 0 < P(B) < 1 and
P (A ∪ B) = P(A) + P(B) − P(A)P(B). Then,
(a) x − 1
2
(b) x −12

(a) P  = P(B) −P( A)


B
(c) x2 + 1 (d) x2 + 1  A
(b) P( Ac ∪ Bc ) = P( Ac ) + P(Bc )
48. The solution of the equation
(c) P( A ∪ B)c = P( Ac )P(Bc )
cos θ sin θ cos θ
(d) p  = P(B)
A
− sin θ cos θ sin θ = 0 is  B
− cos θ − sin θ cos θ Directions (Q. Nos. 56 to 58) For the following
π questions, choose the correct answers from the codes (a),
(a) θ = nπ (b) θ = 2 nπ ±
2 (b), (c) and (d) defined as follows :
π π
(c) θ = nπ ± (−1)n (d) θ = 2 nπ ± (a) Both (A) and (R) are true and (R) is the correct
4 4 explanation of (A).
49. The number of values of x in the interval (b) Both (A) and (R) are true and (R) is not the
correct explanation of (A).
[0 , 5 π] satisfying the equation
(c) (A) is true but (R) is false.
3 sin 2 x − 7 sin x + 2 = 0 is
(d) (A) is false but (R) is true.
(a) 0 (b) 5
(c) 6 (d) 10 56. Assertion (A) The points (2, 1) and (−3, 5)
lie on opposite side of the line 3 x − 2 y + 1 = 0
50. If tan α tan β + tan β tan γ + tan γ tan α
2 2 2 2 2 2
Reason (R) The algebraic perpendicular
+ 2 tan 2 α tan 2 β tan 2 γ = 1, then the value of distance from the given point to the line have
(a) 0 (b) −1 opposite sign.
(c) 1 (d) None of these
57. Assertion (A) Greatest coefficient in the
51. The number of solutions of the equation expansion of (1 + 5 x)8 is 8 C 4 5 4 .
−1 −1 3π
2 sin x − x + 1 + cos
2
x − x=
2
is Reason (R) Greatest coefficient in the
2
expansion of (1 + x)2 n is the middle term.
(a) 0 (b) 2 (c) 4 (d) ∞
52. In a college, 25% of the boys and 10% of the 58. Assertion (A) 7 + 4 i > 5 + 3i where i = −1
girls offer Mathematics. The girls constitute Reason (R) 7 > 5 and 4 > 3.
60% of the total number of students. If a
59. The circumcentre of a triangle formed by the
student is selected at random and is found to
be studying Mathematics, the probability that lines xy + 2 x + 2 y + 4 = 0 and x + y + 2 = 0 is
the students is a girl, is (a) (0, − 1) (b) (−1, 0) (c) (1, 1) (d) (−1, − 1)
1 3 5 5 60. One diagonal of a square is along the line
(a) (b) (c) (d)
6 8 8 6 8 x − 15 y = 0 and one of its vertex is (1, 2).
53. There of the six vertices of a regular hexagon Then, the equation of the sides of the square
are chosen at random. The probability that passing through this vertex are
the triangle with these three vertices is (a) 23 x + 7 y = 9, 7 x + 23 y = 53
equilateral, is equal to (b) 23 x − 7 y − 9 = 0, 7 x + 23 y − 53 = 0
1 1 1 1 (c) 23 x − 7 y + 9 = 0,7 x + 23 y + 53 = 0
(a) (b) (c) (d) (d) None of the above
2 5 10 20
Practice Set 1 433

61. The three lines lx + my + n = 0, 1 2 π 2π nπ 


69. lim sec + sec 2 + ..... sec 2 
mx + ny + l = 0, nx + ly + m = 0 are n→ ∞ n  4n 4n 4 n
concurrent, if is equal to
(a) l = m + n (b) m = l + n 4 2 3 5
(a) (b) (c) (d)
(c) n = l + m (d) l + m + n = 0 π π π π
62. Distance between the pair of lines 70. The nth derivative of (e 2x + e −2x ) is
represented by the equation
(a) 2 n (e 2 x − e −2 x ) (b) e 2 x + (−1)n e −2 x
x 2 − 6 xy + 9 y 2 + 3 x − 9 y − 4 = 0 is n −2 x
(c) 2 [e x + (−1) e
n 2
] (d) None of these
1 15 5 1
(a) (b) (c) (d) x + e x + ... ∞ dy
2 10 2 10 71. If y = e x + e , then is equal to
dx
63. The two curves x 3 − 3 xy 2 + 2 = 0 and y 1 y y
(a) (b) (c) (d)
3 x 2 y − y3 = 2 1− y 1− y 1+ y y−1
(a) Cut at right angles (b) touch each other
π π 72. If in a greatest integer function, the domain is
(c) cut at an angle (d) cut at an angle a set of real numbers, then range will be set of
3 4
(a) real numbers (b) rational numbers
2x + 3 a b
64. If = + , then a + b is (c) imaginary numbers (d) integers
(x + 1)(x − 3) x + 1 x − 3
73. The inverse of the function
equal to
ex − e− x
(a) 1 (b) 2 f (x) = + 2 is given by
9 1 ex + e− x
(c) (d) − 12 12
x−2 x − 1
(a) loge  (b) loge 
4 4
 
1 2 3 1  x − 1  3 − x
65. The rank of 2 4 6 2 is x − 1
−2 12
(c) loge  (d) loge 
  x 
 
1 2 3 2  x + 1  2 − x

(a) 0 (b) 1 74. If f (x) = |cos x| and g(x) = [ x ], then go f (x) is


(c) 2 (d) 3 equal to
66. If x = r cosθ, y = r sinθ, then (a)|cos[ x]| (b)|cos x|
2 2 (c) [|cos x|] (d)|[cos x]|
 ∂r  +  ∂r  is equal to
   
 ∂x   ∂y  75. The equation of the tangent to the circle
(a) 0 (b) 1 x 2 + y 2 = a 2 parallel to the line
(c) 2r (d) r 3 x + y + 3 = 0 are
67. If V is the volume of a rectangular box with (a) 3 x + y ± a = 0 (b) 3 x + y ± 2 a = 0
edges x , y , z, then Vxyx is equal to (c) 3 x + y ± 4a = 0 (d) None of these
(a) 0 (b) 1 76. From the origin chords are drawn to the circle
(c) x + y + z (d) None of these
(x − 1)2 + y 2 = 1. The equation of the locus of
68. If y = f (x) is given by the middle points of these chords is
x 1 1.5 2 2.5 3 (a) x2 + y2 − 3 y = 0 (b) x2 + y2 − 3 x = 0
y 2 2.4 2.7 2.8 3 (c) x + y − x = 0
2 2
(d) x2 + y2 − y = 0
3
then ∫ f ( x) by suing Trapezoidal rule is
77. A circle touches the x-axis and also touches
the circle with centre at (0, 3) and radius 2.
1
The locus of the centre of the circle is
(a) 5 (b) 5.1
(a) a parabola (b) a circle
(c) 5.2 (d) None of these
(c) an ellipse (d) a hyperbola
434 AP EAMCET Chapterwise Mathematics

f (x) − 1 (a) f( x) is continuous at x = 0


1 = 1, f ′ ()
78. If f () 1 = 2, then lim is (b) lim f( x) does not exits
x →1 x −1 x→ 0
1 (c) lim f( x) = 1
(a) 1 (b) 2 (c) 4 (d)
x→ 0
2
(d) lim f( x) exist but f( x) is not continuous at x = 0
 e1 x − 1 x→ 0

79. For the function f (x) =  e1 x + 1 , x ≠ 0 80. The maximum value of f (x) =
x
on
 0 , x =0 4 + x + x2
[ −1, 1] is
which of the following is correct? 1 1 1 1
(a) − (b) − (c) (d)
4 3 6 5

Answers
1. (d) 2. (a) 3. (b) 4. (b) 5. (c) 6. (b) 7. (c) 8. (c) 9. (d) 10. (c)
11. (c) 12. (c) 13. (a) 14. (c) 15. (a) 16. (b) 17. (a) 18. (c) 19. (a) 20. (c)
21. (b) 22. (a) 23. (d) 24. (b) 25. (b) 26. (c) 27. (c) 28. (c) 29. (a) 30. (a)
31. (b) 32. (a) 33. (d) 34. (a) 35. (d) 36. (d) 37. (a) 38. (c) 39. (c) 40. (b)
41. (b) 42. (a) 43. (b) 44. (a) 45. (a) 46. (b) 47. (b) 48. (b) 49. (c) 50. (c)
51. (b) 52. (b) 53. (c) 54. (b) 55. (c) 56. (a) 57. (d) 58. (d) 59. (d) 60. (b)
61. (d) 62. (c) 63. (a) 64. (b) 65. (c) 66. (b) 67. (b) 68. (c) 69. (a) 70. (c)
71. (a) 72. (d) 73. (b) 74. (c) 75. (b) 76. (c) 77. (a) 78. (b) 79. (b) 80. (c)

Solutions
dx ⇒ yy1 = ± 2a
1. (d) Let I = ∫
x 1 − (log x)2 Again differentiating , we get
1 (y1)2 + yy2 = 0
Put log x = t ⇒ dx = dt
x Hence, order and degree are respectively 1 and 2.

4. (b) Given that, y = ax cos 1 + b 


dt −1 −1
∴I= ∫ 1− t 2
= sin t + c = sin (log x) + c
x 

x2 On differentiating w.r.t.x, we get


2. (a) ∫ dx
(x + 2)(x 2 + 3)
2

y1 = a  cos + b  − x sin + b   − 2  
1 1 1
 3 2 
= ∫ 2 − 2  dx   x  x   x  
 x + 3 x + 2
y1 = a  cos + b  + sin + b  
1 1 1

  x  x x  
tan−1   + c
3 x 2 x
= tan−1 −
3 3 2  2
Again differentiating, we get
= 3 tan−1   − 2 tan−1   + c
x x
y2 = a  − sin + b   − 2  − 2
1 1 1
 3  2  x  x  x

sin + b  − 3 cos + b  
3. (b) Let the equation of parabola whose axis is 1 1 1
X-axis, is x  x x  
y 2 = ± 4a(x − h)
y2 = − 4 cos + b 
ax 1

On differentiating w.r.t. x, we get x x 
2yy1 = ± 4a ⇒ x 4 y2 + y = 0
Practice Set 1 435

∴ [a + b b + c c + a ] = 0
5. (c) Given differential equation can be rewritten as
⇒ (a + b) ⋅ {b ( + c) × (c + a)} = 0
dy 2x x2 − 1
+ y= 2 ⇒(a + b) ⋅ {b × c + b × a + c × c + c × a } = 0
dx 1 + x 2
x +1
⇒ (a + b) ⋅ {b × c − a × b + c × a } = 0
2x x2 − 1 ⇒ a ⋅ (b × c) − a ⋅ (a × b) + a ⋅ (c × a)
Here, P = and Q = 2
1+ x 2
x +1 + b ⋅ (b ⋅ c) − b ⋅ (a ⋅ b) + b ⋅ (c ⋅ c) = 0
2x
∫1 + dx ⇒ [abc] − 0 + 0 + 0 − 0 + [b c a ] = 0
x2 x 2)
∴ IF = e = elog(1 + = 1 + x2 ⇒ [a b c] = 0
π ∴ Vectors a , b and c are coplanar.
2
6. (b) ∫|sin x − cos x | dx (B) Since, a × b, b × c and c × a are coplanar.
0 ∴ [a × b b × c c × a ] = 0
π π
⇒ [a b c]2 = 0
2 2
= ∫ − (sin x − cos x)dx + ∫ (sin x − cos x)dx ⇒ [a b c] = 0
0 π 4 ⇒ a b and c are coplanar.
= − [− cos x − sin x]π0 2
+ [− cos x − sin x]ππ 2
4 10. (c) Now, AD = OD − OA
 1  b+ c b + c − 2a
= − − + 1 + − 0 − 1 + 
1 1 1 = −a=
− + 
 2 2    2 2   2 2
A(a)
= 2 −1 −1 + 2 = 2( 2 − 1)
π −a
7. (c) Since, I1 = ∫ xf (sin x)dx F E
a
π −a
= ∫ (π − x) f (sin(π − x))dx
a
π −a (b)B C(c)
D
= ∫ (π − x) f (sin x) dx Similarly, BE = OE − OB
a
c+ a c + a − 2b
π −a = − b=
2 2 2
∫ πf (sinx)dx − I1 ⇒ 2I1 = πI 2 ⇒ I 2 =
π
I1
a + b − 2c
a and CF =
1 2
8. (c) Required area = ∫ (e x − e − x )dx b + c − 2a c + a − 2b
∴AD + BE + CF = +
0 2 2
a + b −2c
=  e + − 2 sq. unit
1
= [e + x
e − x ]10 + =0
 e  2
y
11. (c) Let angle between b and c is α, then
y=e–x
|b × c| = 15 (given)
y=ex
⇒ |b||c|sinα = 15
15
⇒ sinα =
4
1
x′ x ∴ cosα =
O 4
Also, b − 2c = λ a
y′ ⇒ (b − 2c)2 = λ2(a)2
9. (d) (A) Let A = b + c, B = c + a and C = a + b ⇒ |b|2 + 4|c|2 − 4 b ⋅ c = λ2|a|2
∴ A + B + C = 2(a + b + c) ⇒ 16 + 4 − 4{|b|c|cos a} = λ2
Since a + b, b + c and c + a are coplanar.
⇒ λ2 = 16 ⇒ λ = ± 4
436 AP EAMCET Chapterwise Mathematics

12. (c) Let Q be the image of the point P($i + 3k$ ) in the 15. (a) Given that,
plane r ⋅ ($i + $j + k$ ) = 1. Then, PQ is normal to the 3 + i = (a + ib)(c + id )
plane, Since, PQ passes through P and in normal to 3 + i = (ac − bd) + i(ad + bc)
the given plane, therefore equation of PQ is
$ ) + λ($i + $j + k
r = ($i + 3k $) ∴ ac − bd = 3 and ad + bc = 1

Now, tan   + tan−1  


−1 
b d
P(j+3k)
 a  c
 b + d 
−1  a c  = tan−1  bc + ad 
= tan 
b d  ac − bd 
1 − ⋅ 
R  a c
π
= tan−1   = nπ +
1
n∈I
 3 6

16. (b) Since, 10, 3, e, 2 are in decreasing order.


Q
∴ log10 α , log 3 α , log e α , log 2 α are in increasing
Since, Q lies on the line PQ, so let the position order.
vector of Q be ($i + 3k $ ) + λ($i + $j + k
$)
$ 17. (a) Given that,
i.e., (1 + λ)$i + $j + (3 + λ)k
5− 2 5+ 2
Since, R is the mid point of PQ therefore position y= and x =
5+ 2 5− 2
vector of R is
1
$ + $i + 3k
(1 + λ)$i + λ$j + (3 + λ)k $ ⇒ y= ⇒ xy = 1
x
2 ∴ 3x + 4 xy − 3y 2 = 3(x − y)(x + y) + 4 xy
2
λ + 2 $ λ $  6 + λ  $
=  i + j +  k  5+ 2 5− 2
 2  2  2  = 3 − 
 5− 2 5+ 2
λ λ λ $
=  + 1 $i + $j +  3 +  k
2  2  2  5+ 2 5− 2
× +  + 4
$) = 1
Since, R lies on the plane r ⋅ ($i + $j + k  5− 2 5+ 2
λ λ λ $ $ $ $
∴  + 1 $i + $j +  3 +  k ⋅(i + j + k) = 1  ( 5 + 2)2 − ( 5 −
= 3
2)2 
  2  2  2   (5 − 2)(5 − 2) 
 
⇒ λ + 1 + λ + 3 + λ =1 [( 5 + 2)2 + ( 5 − 2)2] + 4
 2 2 2 
1
⇒ λ = −2 = 4 10.2(5 + 2) + 4
$. 3
Hence, the position vector of Q is − $i − 2$j + k 56 1
= 10 + 4 = (56 10 + 12)
13. (a) Now|z 2 + 2z cosα|≤|z|2 +2|z||cosα|≤ |z|2 + 2|z| 3 3
< ( 3 − 1)2 + 2( 3 − 1) 18. (c) The number of words before the word
= 3 + 1 − 2 3 + 2 3 − 2= 2 CRICKET is 4 × 5! + 2 × 4! + 2! = 530
∴ |z 2 + 2z cosα | < 2 19. (a) Given inequality is x 2 − 8 x + 12 ≥ 0
53
  ∴ (x − 2)(x − 6) ≥ 0
14. (c) (− 3 + i)53 = 253  − 3 + 1 
 2 2 ⇒ x ≤ 2 or x ≥ 6
= 253[cos150° + i sin150° ]53 Hence, length of interval = 8 − 0 = 8
and, length of interval for favorable points
= 253[cos(150° × 53) + i sin(150° × 53)]
= 2− 0 + 8 − 6 = 4
= 253[cos(22π + 30°) + i sin(22π + 30°)] 4 1
∴ Required probabiltiy = =
= 253[cos 30° + i sin 30° ] 8 2
 3 1
= 253  + i  20. (c) The total number of possible outcomes is 64 .
 2 2 The number of possible outcomes in which 2 does
Practice Set 1 437

not appear on any die is 54 . Therefore, the number b a


⇒ x=a and y = b
of possible outcomes in which at least one die a+ b a+ b
shows a 2is 64 − 54 = 1296 − 625 = 671. −b 2 x b2 a
Slope of tangent at ellipse = 2 = − 2
21. (b) We know, a y a b
a1 b1 c1 A1 B1 C1 x a
Slope of tangent at circle = − = −
∆∆′ = a 2 b2 c 2 , A2 B2 C2 y b
a 3 b3 c 3 A3 B3 C3  a b2 a 
 − 2  a − b
Σa1 A1 0 0 ∴ θ = tan−1  b a2 b  ⇒ θ = tan−1  
 ab 
= 0 Σa 2 A2 0  1+ b ⋅a 
 a 2 b 
0 0 Σa 3 A3
∆ 0 0
26. (c) Since, vertex is (0, 4) and focus is (0, 2).
∴ a=2
= 0 ∆ 0 = ∆3 ⇒ ∆′ = ∆ 2
∴ Equation of parabola is
0 0 ∆ (x − 0)2 = − 4 ⋅ 2(y − 4) ⇒ x 2 + 8 y = 32
22. (a) For unique solution of the given system, ∆ ≠ 0 27. (c) Given equation of ellipse can be rewritten as
1 1 1 x2 y2
+ =1
i. e., 5 −1 µ ≠ 0 52 42
2 3 −1 Now, PF1 + PF2 = (x − 3)2 + y 2 + (x + 3)2 + y 2
Hence, it depends only on µ. 400 − 16 x 2
= (x − 3)2 +
4 2 2 25
23. (d) Given,  −5 0 α  = 10 A−1 400 − 16 x 2
  + (x + 3)2 +
 1 −2 3 25
 4 2 2 1 −1 1  10 0 0 1
= { 9 x 2 + 625 − 150 x
∴  −5 0 α ,  2 1 −3 =  0 10 0 5
      + 9 x 2 + 625 + 150 x}
 1 −2 3 1 1 1   0 0 10
1
 10 0 0  10 0 0 = (25 − 3x + 3x + 25) = 10
⇒  −5 + α 5 + α −5 + α  =  0 10 0 5
    28. (c) According to given condition, 2ae = 2⋅ 2a
 0 0 10   0 0 10
⇒ −5 + α = 0 ⇒ α = 5 ⇒ e = 2 and 2b = 6 ⇒ b = 3
3
24. (b) Since, A′ = − A Hence, a = = 3
3
Now AA−1 = A−1 A = I n ∴ Required equation is
⇒ (AA−1)′ = (A−1 A)′ = (I n)′ x 2 y2
− = 1 ⇒ 3x 2 − y 2 = 9
⇒ (A−1) ′ A′ = A′(A−1)′ = I n 3 9
⇒ (A−1)1 (− A) = (− A)(A−1) = I n ⇒ (A−1)′ = − (A−1) 29. (a) Given that, ( x + 1) − (x − 1)
= 4 x − 1 On squaring both sides, we get
25. (b) Given equation of curves are
x 2
y 2 (x + 1) + (x − 1) − 2 x 2 − 1 = 4 x − 1
+ 2 = 1 and x 2 + y 2 = ab
a2 b ⇒ −2 x 2 − 1 = 2x − 1
ab − y 2 y 2
∴ + 2 =1 Again squaring, we get
a2 b
4(x 2 − 1) = 4 x 2 + 1 − 4 x
 a 2
− b 2
 a−b
⇒ y2  2 2  = 5
 a b  a ⇒ 4x = 5 ⇒ x =
4
ab 2 a 2b 5
⇒ y2 = and x 2 = But, x = is not satisfied the given equation.
a+ b a+ b 4
438 AP EAMCET Chapterwise Mathematics

4 1 + x 2 
30. (a) Given equation can be rewritten as ⇒ log e   = − log e (1 − y )
2

x 2 − 2(α + β) x + 3αβ = 0 2 1 − x2 
2
Let roots be α′ and − α ′ 1 + x 2   1 
⇒ log e   = log e  
∴ α ′ + (− α ′) = 2(α + β) ⇒ α + β = 0 1 − x2  1 − y2 
2
31. (b) Let f (x) = ax 2 + bx + c 1 + x 2  1
⇒   =
a 3 bx 2 1 − x2  1 − y2
Again, let f (x) = ∫ f (x)dx = 3
x +
2
+ cx
⇒ (1 + x 4 + 2x 2)(1 − y 2) = (1 − x 2)2
At x = 0, f(0) = 0 ⇒ 4 x − y 2 − x 4 y 2 − 2x 2 y 2 = 0
2

Again, at x =1 n −1
36. (d) ∴ Tn = 1 + x + x + .... + x = x − 1 ⋅ 1
2 n
a b 2a + 3b + 6c
1 = + + c=
f () =0 n! x − 1 n!
3 2 6
(Q 2a + 3b + 6c = 0 given) 1 x n
1
=  − 
⇒ f (0) = f ()
1 =0 1 − x  n! n! 
Therefore, it exist at least one point c in between 0 ∞ 1  xn 1 
and 1 such that f ′(x) = 0 or ax 2 + bx × c = 0 for ⇒ Σ  − 
n = 1 1 − x  n! n! 
some x ∈(0, 1).
1  ∞ xn ∞ 1  1
=  Σ − Σ = (e x − e)
32. (a) Since, α , β and γ are the roots of (x − 1)  n = 1 n! n = 1 n!  ( − 1)
 x
x 3 − 9 x 2 + 26 x − 24 = 0 ⇒ α + β + γ = 9
αβ + βγ + γα = 26 and αβγ = 24
37. (a) Since, f (x) = cosh x + sin hx
∴ Σ(α + 3)(β + 3) = Σαβ + 3(α + β ) + 9) e x + e −x e x − e −x
= + = ex
= 26 + 6(9) + 9(3) = 107 2 2
+ x 2 + .... + x n
∴ f (x1 + x 2 + K + x n) = e x1
33. (d) (1 + x + x + x ) = (1 + x)n (1 + x 2)n
2 3 n
= e x1 ⋅ e x 2 ⋅K e x n = f (x1) ⋅ f (x 2) .... f (x n)
= (1 + n C1 x + n C2 x 2 + .... + n Cn x n)
× (1 + n C1 x 2 + n C2 x 4 + .... + n Cn x 2n) 38. (c) Since, tan(ix) = i tan hx
a − ib    a − ib  
∴ The coefficient of x 4 = n C2 + n C2 ⋅ n C1 + n C4 ∴tan i log  
 = i tanh  log  
  a + ib     a + ib  
= C4 + C2 + C1 ⋅ C2
n n n n
a − ib a + ib 
i  − 
34. (a) According to the question,  a + ib a − ib   (a − ib)2 − (a + ib)2 
= = i 2
m
C0 + m
C1 + m
C2 = 46 a − ib a + ib  (a − ib) + (a + ib) 
2
+
a + ib a − ib
⇒ 2m + m(m − 1) = 90
 −4abi  2ab
⇒ m2 + m − 90 = 0 ⇒ m = 9 as m > 0 = i 2 =
m
 2(a − b 2)  a 2 − b 2
Now, (r + 1)th term of  x 2 + 
1
is
 x 39. (c) Given inequation is n! > 2n −1
r
For n = 1, 2 1! >/ 1 ⇒ 1 >/ 1
Cr (x 2)m − r   =
1
m m
Cr x 2m − 3r
 x 2! >/ 21 ⇒ 2 >/ 2
For independent of x, For n = 3! 3! > 22 ⇒ 6 > 4
2m − 3r = 0 ⇒ r = 6 Hence, it is true for n > 2
∴ Required coefficient of x is 9 C6 = 84
40. (b) Now, taking option (b),
 6 10

35. (d) Since, 4 x 2 + x + x + .... P(n) :
sin(2n α)
 3 5  2n sinα
y4 y6 sin 2α
= y2 + + + .... Put n = 1, P()
1 : = cosα
2 3 2sinα
Practice Set 1 439
sin 4α 4sinα cosα cos 2α AE
Put n = 2, P(2) : = In ∆AEC, tan 45° = ⇒ AC = AE = 12m
4sinα 4sinα AC
= cosα cos 2α and in ∆ABE, tan 60° =
AE
⇒ AB =
AE
=4 3
Hence, option (b) in correct. AB 3

41. (b) Given vertices are A(5, − 1, 1), B(7, − 4, 7), Now, BC = AC 2 − AB2
C(1, − 610
, ) and D(−1, − 3, 4) = 144 − 48 = 96 = 4 6
Now, ∴Area of rectangle ABCD
AB = (2)2 + (−3)2 + (6)2 = 4 + 9 + 36 = 7 = AB × BC = 4 3 × 4 6 = 48 2 m 2
BC = (−6) + (−2) + (3) =
2 2 2
36 + 4 + 9 = 7 46. (b) Since, sin x + sin2 x = 1
CD = (−2)2 + (3)2 + (−6)2 = 4 + 9 + 36 = 7 or sin x = 1 − sin2 x = cos2 x
and DA = (6)2 + (2)2 + (−3)2 = 36 + 4 + 9 = 7 ∴ cos12 x + 3cos10 x + 3cos8 x + cos6 x − 2
Now, AB ⋅ BC = (2, − 3, 6) ⋅ (−6, − 2, 3) = sin6 x + 3sin5 x + 3sin4 x + sin3 x − 2
= − 12 + 6 + 18 ≠ 0
= (sin2 x)3 + 3 (sin2 x)2 sin x
∴ ABCD is a rhombus.
+ 3(sin2 x)(sin x)2 + (sin x)3 − 2
42. (a) Equation of plane passing through intersection
of given planes is = (sin2 x + sin x)3 − 2 = ()
1 3 − 2= −1
(x + y + z − 1) + λ(2x + 3y − z + 4) = 0 …(i)
Plane (i) is parallel to x-axis, then 47. (b) Given, sin θ = x − 1
1 2 2x
(1 + 2λ) 1 = 0 ⇒ λ = −
2 θ θ θ
2sin cos 2tan
1 ∴ tanθ = 2 2 = 2
On putting λ = − in Eq. (i), we get
2θ θ
2 1 − 2sin 1 − tan2
y − 3z + 6 = 0 2 2
x −1
43. (b) (A) Since, a + b = 1 2
b+ c c+ a ∴ tanθ = x +1 = x2 − 1
⇒ ac + a 2 + b 2 + bc = bc + ab + c 2 + ac x −1
1−
x +1
a2 + b2 − c 2 1 1
⇒ = ⇒ cos C = cosθ sinθ cosθ
2ab 2 2
⇒ ∠C = 60°
48. (b) Since, − sinθ cosθ sinθ = 0
b c − cosθ − sinθ cosθ
(B) Since, 2 + = 0 ⇒ ∠A = 60°
a − c 2 a2 − b2 ⇒ cosθ [(cos2 θ + sin2 θ) − sinθ
(C) r : R : r1 = 1 : 2 : 3 (− sinθ cosθ + cosθsinθ) + cosθ(sin2 θ + cos2 θ) = 0
⇒ ∆ABC is an equilateral triangle.
⇒ cosθ + cosθ = 0
44. (a) Now, b − c + c − a + a − b ⇒ cosθ = 0
r1 r2 r3 π
(b − c)(s − a) + (c − a)(s − b) + (a − b)(s − c) ⇒ θ = 2nπ ±
= =0 2

49. (c) Given, 3sin2 x − 7sin x + 2 = 0
45. (a) Let AE is a vertical lamp post.
⇒ (3sin x − 1)(sin x − 2) = 0
D 1
E 45°
C ⇒ sin x = (Qsin x ! ≠ 2)
3
1 π
Let sin−1 = α, 0 < α < are the solutions in
12 m 3 2
[0, 5π]. Then α , π − α , 2π + α , 3π − α , 4 π + α
60° 5π − α are the solutions.
A B
440 AP EAMCET Chapterwise Mathematics

Hence, P(A ∪ B)c = P(Ac ∩ Bc) = P(Ac) P(Bc)


50. (c) sin2 α + sin2 β + sin2 γ
As A is independent of B, hence
tan2 α tan2 β tan2 γ
= + +
P   = P(A)
A
1 + tan α 1 + tan β 1 + tan2 γ
2 2
 B
x y z
= + +
1+ x 1+ y 1+ z 56. (a) (A) The algebraic perpendicular distance from
where, x = tan2 α , y = tan2 β, z = tan2 γ (2, 1) to the line 3x − 2y + 1 = 0 is
(x + y + z) + (xy + yz + zx + 2xyz) 3(2) − 2()
1 +1 5
= = L1 (say)
+ xy + yz + zx + xyz 32 + (−2)2 13
(1 + x)(1 + y)(1 + z) and The algebraic perpendicular distance form
1 + x + y + z + xy + yz + zx + xyz (−3, 5) to the line 3x − 2y + 1 = 0 is
= =1
(1 + x)(1 + y)(1 + z) 3(−3) − 2( 5) + 1 −18
= = 1 (say)
(Q xy + yz + zx + 2xyz = 1 given) 32 + (−2)2 13
−1
51. (b) Since, sin x, cos−1 x are defined for x ≤ 1 Here,
L1
<0
and x ≥ 0 L2
∴ x 2 − x + 1 ≤ 1 and x2 − x ≥ 0 ∴ Given points lie on opposite side of the line.
Hence, option (a) is correct.
⇒ x − x ≤ 0 and x − x ≥ 0
2 2

⇒ x 2 − x = 0 ⇒ x = 0, 1 57. (d) (A) The greatest coefficient in the expansion of


(1 + 5x)8 is 8 C4
Hence, number of solution are two.
∴It is false.
52. (b) Let the total number of students be 100. Then,
boys = 40, girls = 60 Reason (R) is true.
25 Hence, option (d) is correct.
25% of boys offer Mathematics = × 40
100 58. (d) (A) By the properties of complex number
= 10 boys 7 + 4i > 5 + 3i is not defined.
10
10% of girls offer Mathematices = × 60 = 6 girls ∴It is false statement.
100
But, reason (R) is true.
It means, 16 students offer Mathematics.
Hence, option (d) is correct.
6 3
∴ Required probability = =
16 8 59. (d) Given lines are
xy + 2x + 2y + 4 = 0 …(i)
53. (c) In a hexagon, there are two equilateral triangle.
2 and x + y + 2= 0 …(ii)
∴Required probability = 6
C3
2 2 1
= = =
6 × 5× 4 20 10
3× 2×1 (–2, 0)
90 9 x′ x
54. (b) Let P (fresh egg) = = = p (say) (0, 0)
100 10
10 1
and P (rotten egg) = = = q (say)
100 10 (0, –2)
Here, n = 5, r = 5 y′
Using binomial distribution,
The required probability From Eqs. (i) and (ii), we get
5 0 5 xy = 0 ⇒ x = y = 0
= 5C5     =  
9 1 9
 10   10   10  ∴ Vertices of triangle are (−2, 0)(0, 0),(0, − 2)in a
right angled triangle circumcentre is the mid point
55. (c) Since, P(A ∩ B) = P(A)P(B) of hypotenuse.
It means A and B are independent events, so Ac ∴ Point (−1, − 1) is the required coordinate of the
and Bc will also be independent circumcentre.
Practice Set 1 441
On differentiating Eqs. (i) and (ii) with respect to
60. (b) Slope of BD is 8 and angle made by BD with x, we get
15
 dy  = x − y and  dy  = − 2xy
2 2
AD and DC is 45°.    
 dx  c 2xy  dx  c x 2 − y2
A B 1 2

∴  dy  ×  dy  = − 1
   
 dx  c  dx  c
1 2

0
= 2x + 3 a b
5y
64. (b) Given that, = +
–1

(x + 1)(x − 3) (x + 1) (x − 3)
8x

45° ⇒ (2x + 3) = a(x − 3) + b(x + 1)


D C(1, 2)
On comparing the coefficient of x and constant,
So, let slope of DC be m, then we get
8
m− 2 = a + b and 3 = − 3a + b
tan 45° = ± 15 1 9
8 ⇒ a=− and b =
1+ m 4 4
15
1 9
⇒ (15 + 8m) = ± (15m − 8) ∴ a+ b=− + =2
23 7 4 4
⇒ m= and −
7 23 1 2 3 1  1 2 3 1 
Hence the equation of DC and AD are 65. (c) Let A =  2 4 6 2 ~  0 0 0 0
23    
y − 2 = (x − 1) 1 2 3 2  0 0 0 1 
7
7 R2 → R2 − 2R1 , R3 → R3 − R1
⇒ 23x − 7 y − 9 = 0 and y − 2 = − (x − 1)
23 1 2 3 1 
⇒ 7 x + 23y − 53 = 0 ~  0 0 0 1  (R2 → R3)
 
61. (d) Since, given lines are concurrent.  0 0 0 0
l m n
Hence, rank of A is 2.
∴ m n l =0
n l m 66. (b) Since, x = r cosθ, y = r sinθ ⇒ x 2 + y2 = r 2
Applying C1 → C1 + C2 + C3 On differentiating partially w.r.t. x , y , we get
l + m+ n m n ∂ 2 ∂ 2
(r ) = (x + y 2)
⇒ l + m+ n n l = 0 ∂x ∂x
l + m+ n l m ∂r ∂r x
⇒ 2r = 2x ⇒ =
1 m n ∂x ∂x r
∂ 2 ∂
⇒ (l + m + n) 1 n l =0 and (r ) = (x 2 + y 2)
∂y ∂y
1 l m
∂r ∂r y
l + m+ n = 0 ⇒ 2r = 2y ⇒ =
∂y ∂y r
62. (c) The distance between the pair of lines 2 2
 ∂r  +  ∂r  = x + y = r = 1
2 2 2
represented by the equation ∴    
x 2 − 6 xy + 9 y 2 + 3x − 9 y − 4 = 0 is  ∂x   ∂y  r 2
r2
9
− (−4)  67. (b) Since, volume of rectangular box,
g2 − ac 
=2 4 Q 2  V = xyz ⇒ Vx = yz
( + 9)
11  a(a + b) 

⇒ Vxy = z ⇒ Vxyz = 1
25
4 = 5 68. (c) Using Trapezoidal rule.
= 2× 3
10 2 0.5
∫ f (x)dx = 2
[(2 + 3) + 2(2. 4 + 2.7 + 2.8)]
63. (a) Given curves are 1
x 3 − 3xy 2 + 2 = 0 …(i) 1 1
= (5 + 158
. ) = (20.8) = 52
.
and 3x y − y 3 − 2 = 0
2
…(ii) 4 4
442 AP EAMCET Chapterwise Mathematics

69. (a) lim 1 sec2 π + sec2 2π + ..... sec2 nπ  77. (a) Let the centre of a circle be C1 (h, k)
n→ ∞ n  4n 4n 4n 
Since, C1 C2 = r1 + r2 (given)
1 n rπ
= lim Σ sec2 ⇒ (h − 0)2 + (k − 3)2 = |k + 2| ⇒ h2 = 5 (2 k − 1)
n→ ∞ n r = 1 4n
1
πx πx
1 Hence, locus of a point is x 2 = 5 (2y − 1) which
dx =  tan  =
4 4
= ∫ sec2 represents a equation of parabola.
0
4 π  4  0 π
f (x) − 1
70. (c) Let y = e 2x + e −2x 78. (b) Let y = lim
x→1 x −1
dy
⇒ = 2 e 2x − 2 e −2x = 2( e 2x − e −2x )  f (x) − 1  ( f (x) + 1) ( x + 1)
dx = lim   × ×
x→1  x −1  ( x + 1) ( f (x) + 1)
d2y
⇒ = 22(e 2x + e −2x ) f (x) − 1 x +1
dx 2 = lim ×
n x→1 x −1 f (x) + 1
d
Similarly, (e 2x + e −2x ) = 2n (e 2 x + (−1)n e −2x )
dx n f (x) − f ()
1 x +1
= lim × lim
x + e x +... ∞
x→1 x −1 x→1 f (x) + 1
71. (a) Since, y = e x + e ⇒ y = ex + y
2
= lim f ′()
1 . ⇒ y=2
⇒ log y = (x + y) log e x→1 1 +1
f ()
1 dy dy dy y
⇒ =1 + ⇒ =
y dx dx dx 1 − y e  −11/ x

79. (b) Given, f (x) =  e1 / x + 1 , x ≠ 0


72. (d) The range of a greatest integer function is an  0, x=0
integers. 1

−x
73. (b) Given, y = e x − e − x + 2 ⇒ y − 2 = e 2x − 1
x 2x
e h −1
LHL = lim f (x) = lim = −1
e + e e +1 x → 0− h→ 0 −
1
e h +1
1 − y y −1 1  y −1 
⇒ e 2x = = ⇒ x = log e   e −1
1/ x
y − 3 3− y 2  3− y RHL = lim f (x) = lim =1
1 2
x → 0+ x → 0+ e1 / x + 1
1 2
 y −1  x −1 
⇒ f −1 (y) = log e   ⇒ f −1 (x) = log e   1

1
 3− y  3− x  e h −1 1− e h
= lim 1
= lim 1
=1
h→ 0 h→ 0 −
74. (c) Given that, f (x) = |cos x|and g(x) = [x] +1 eh1+ e h

∴ go f (x) = g(|cos x|) = [|cos x|] Hence, limit does not exist at x = 0
x
75. (b) Let equation of line parallel to the 80. (c) Given, f (x) =
4 + x + x2
3x + y + 3 = 0 is 3x + y + k = 0
(4 + x + x 2) − x(1 + 2x)
But, it is a tangent to the circle f ′(x) =
(4 + x + x 2)2
k
x 2 + y 2 = a 2, then = a ⇒ k = ± 2a
1+ 3 Now, f ′(x) = 0
Hence, the required equation is (4 + x + x 2) − x(1 + 2x) = 0
3x + y ± 2a = 0 ⇒ x2 = 4 ⇒ x = ± 2
(x 2 + x + 4)(−2x) + (x 2 − 4)(4 x + 2)
76. (c) Given equation of circle is x 2 + y2 − 2x = 0. Let Now, f ′′ (x) =
(x 2 + x + 4)2
(x1 , y1) be the middle point of any chord of this
circle, then its equation is S1 = T Since, f ′′ (−1) = (1 − 1 + 4)(2) + (1 − 4)(−4 + 2)
∴ x12 + y12 − 2x1 = xx1 + yy1 − (x + x1) = 8 + 6 = 14
It is passes through (0, 0), then and f ′′ ()
1 = (1 + 1 + 4)(−2) + (1 − 4)(4 + 2)
= −12 − 18 = −30
x12 + y12 − 2x1 = − x1 ⇒ x12 + y12 − x1 = 0
1 1 1
Hence, required locus is x 2 + y2 − x = 0 So, max of f (x) at x = 1 ⇒ = =
1 2 4+ 2 6
4 + 1 + ()
Practice Set 2 443

Practice Set 2
1. Let f : (− 1, 1) → B be a function defined by 7. The gradient of the radical axis of the circles
f (x) = tan
2x
−1
, then f is both one-one and x 2 + y 2 − 3 x − 4 y + 5 = 0 and
1 − x2 3 x 2 + 3 y 2 − 7 x + 8 y + 11 = 0 is
onto when B is the interval 1 1 1 2
π π π π π π (a) (b) − (c) − (d) −
(a)  0,  (b)  0,  (c)  − ,  (d)  − ,  3 10 2 3
 2  2   2 2   2 2
8. If the sum of the coefficients in the expansion
2. If f (x) = 2 x 6 + 3 x 4 + 4 x 2, then f ′(x) is of (α 2 x 2 − 2αx + 1)51 vanishes, then the value of
(a) even function (b) an odd function α is
(c) neither even nor odd (d) None of the above (a) 1 (b) − 1 (c) 2 (d) − 2

3. The domain of the function log e (x − 6 x + 6) is2 xn
9. The value of ∑ is equal to
(a) (− ∞, ∞ ) n =1
(2n − 1)!
(b) (− ∞, 3 − 3 ) ∪ (3 + 3, ∞ ) (a) e x
− e− x
(b) x(e x
− e− x
)
(c) (− ∞, 1] ∪ [5, ∞ ) − −
(c) x(e x
−e x
)/2 (d) x(e x
+e x
)
(d) [0, ∞ )
π 3π 5π 10. The sum of the series
4. The value of sin sin sin is 1 1 1
14 14 14 + + + ...... is
1 1 1⋅ 3⋅ 5 3⋅ 5⋅ 7 5⋅ 7⋅ 9
(a) (b)
16 8 (a) 3loge x (b) 1/8 (c) 1/9 (d) 1/12
1
(c) (d) None of these 11. If z = i log(2 − 3), then cos z is equal to
2
(a) i (b) 2i (c) 1 (d) 2
5. If sin A + cos A = m and sin 3 A + cos 3 A = n, then
(a) m3 − 3m + n = 0 (b) n3 − 3n + 2 m = 0
12 The number of non-zero integral solutions of
the equation|1 − i |x = 2x is
(c) m − 3m + 2 n = 0
3
(d) m + 3m + 2 n = 0
3
(a) infinite (b) 1
6. Match the following columns (c) 2 (d) None of these

Column I Column II 13. If 2 log x a + log ax a + 3 log a2x a = 0, then x is


A. sinθ = −
1 1. 5 π equal to
2 6 (a) a1/ 3 or a1/ 2 (b) a− 4 / 3 or a− 1/ 2
B. 1 2. − π (c) a1/ 3 or a− 1/ 2 (d) None of these
cosθ = −
2 6 14. If l , m, n are the pth, qth and rth term of a GP,
C. 3 3. 2 π log l p 1
sinθ = −
2 3
all positive, then log m q 1 equals
D. 3 4. − π
cosθ = − log n r 1
2 3
(a) − 1 (b) 2 (c) 1 (d) 0
5. 7 π n
6 15. If (1 + x n) = ∑ C r x r , then
Codes r=0
A B C D A B C D  C   C   Cn 
1 + 1  1 + 2  ... 1 + 
(a) 5 3 1 2 (b) 2 3 4 1  C0   C1   C n − 1 
(c) 1 3 2 4 (d) None of these
444 AP EAMCET Chapterwise Mathematics
nn − 1 (n + 1)n − 1 πx 
(a)
(n − 1)!
(b)
(n − 1)! 21. The value of lim(1 − x)tan   is equal to
x →1  2
(n + 1)n (n + 1)n + 1 π 2 π 2
(c) (d) (a) (b) (c) − (d) −
n! n! 2 π 2 π
16. The point representing the complex numbers z ax 2 + b, 0 ≤ x < 1
for which| z + 4 |2 − | z − 4 |2 = 8 lie on 
22. If f (x) =  x + 3, 1 < x ≤ 2, then the value of
(a) a straight line parallel to X-axis  4, x =1
(b) a straight line parallel to Y-axis

(c) a circle with centre as origin (a, b) for which f (x) cannot be continuous at
x = 1 is
(d) a circle with centre other than origin
(a) (2, 2) (b) (3, 1) (c) (4, 0) (d) (5, 2)
17. The equation of the circle which passes through 23. If a = $i + 2$j + 2k$ and b = 3i$ + 6$j + 2k$ , then the
the origin and belongs to the coaxial system of
which the limiting points are (1, 2) and (4, 3) is vector in the direction of a and having
magnitude as| b | is
(a) x2 + y2 − x − 7 y = 0
7 7 $
(b) 3( x2 + y2 ) − x − 7 y = 0 (a) ($i + 2 $j + 2k$ ) (b) ( i + 2 $j + 2k$ )
9 3
(c) 2( x2 + y2 ) − x − 7 y = 0
(c) 7($i + 2 $j + 2k$ ) (d) None of these
(d) None of the above
24. Let a = 2i$ + $j − 2k$ and b = $i + $j. If c is a vector
x−4
18. The coefficient of x n in is such that a ⋅ c = |c |,| c − a | = 2 2 and the angle
x2 − 5x + 6
between a × b and c is 30°, then|(a × b) × c | is
1 1 1 1
(a) − (b) + equal to
3n + 1 2n 3n + 1 2n (a) 2/3 (b) 3/2
1 1 1 1
(c) n+1
+ n (d) n+1
+ n (c) 2 (d) None of these
5 2 5 3
25. For any three vectors a, b, c,
19. If the parametric equation of a curve is given by (a − b) ⋅ [(b − c) × (c − a)] equals
θ
x = cos θ + log tan and y = sinθ, then the (a) [a b c ] (b) 2[a b c ]
2 (c) [a b c ] 2
(d) None of these
d 2y
points for which 2 = 0 are given by 26. The equation of the curve in which subnormal
dx
(a) θ = nπ, n ∈ I varies as the square of the ordinate is (λ is
π constant of proportionality)
(b) θ = (2 n + 1) , n ∈ I
2 (a) y = ce 2λx (b) y = ce λx
(c) θ = (2 n + 1)π, n ∈ I 2
y
(d) θ = 2nπ, n ∈ I (c) + λx = c (d) None of these
2
0 1 − 1 dy
27. The solution of the equation = e x − y (e x − e y)
20. If A = 2 1 3 , then [ A(adj A) A − 1 ] is equal dx
  is
3 2 1  x x

to (a) e y = e x − 1 + ce − e (b) e y = e x − 1 + cee


y
 3 0 0 − 6 0 0  (c) e x = e y − 1 + cee (d) None of these
(a) 2  0 3 0 (b) 0 − 6 0 

28. The solution of the equation
   
 0 0 3  0 0 − 6 ( x − y) (2dy − dx) = 3dx − 5dy is
 0 1 / 6 − 1 / 6 (a) 2 y − x = log( x − y + 2 ) + c
(c) 1 / 3 1 / 6 1 / 2  (d) None of these (b) 2 x − y = log( y − x + 2 ) + c
  (c) 2 y + x = log( x − y + 2 ) + c
1 / 2 1 / 3 1 / 6 
(d) None of the above
Practice Set 2 445

1 35. If α , β are the roots of x 2 + px + q = 0 and also


29. ∫ x2 1+ x 2
dx is equal to
α β
of x 2n + p n x n + q n = 0 and if , are the roots
β α
1 + x2 1
(a) − +c (b) − +c of x n + 1 + (x + 1)n = 0, then n is
x 1 + x2
(a) an even integer (b) an odd integer
1 + x2 (c) any integer (d) None of these
(c) +c (d) None of these
x x2 + 4 x + 3
36. Function f (x) = > 0 is
 1 x − 6 x 2 + 11 x − 6
3
30. If ∫ x log 1 +  dx = f (x) ⋅ log(x + 1) + g(x) ⋅ x 2
 x defined in the interval
+ Ax + C, then (a) x ∈ (− 3, 1) ∪ (1, 2 ) ∪ (3, ∞ )
x2 (b) x ∈ (− 3, − 1) ∪ (1, 2 ) ∪ (3, ∞ )
(a) f( x) = (b) g ( x) = log x
2 (c) x ∈ (− 3, 2 ) ∪ (2, ∞ )
1 (d) None of the above
(c) A = (d) None of these
2
37. The number of positive real roots of
31. Area lying in the first quadrant and bounded by x 4 − 4 x − 1 = 0 is
the circle x + y = 4, the line x = 3 y and
2 2
(a) 0 (b) 1
X −axis is (c) 2 (d) 3
π
(a) π sq unit (b) sq unit 38. The sum of the series
2 ∞
π C(n , 0) + C(n , 1) + K + C(n , n)
(c) sq unit (d) None of these ∑ P(n , n)
is
3 n =1

32. The volume of the tetrahedron whose vertices (a) (e 2 − 1) (b) (e 3 + 1)


are (1, − 6, 10), (− 1, − 3, 7), (5, − 1, λ) and (7, − 4 , 7) (c) (2e 2 − 1) (d) None of these
is 11 cu unit, then the value of λ is
(a) 2 or 6 (b) 3 or 4 (c) 1 or 7 (d) 5 or 6
39. 2 log x − log(x + 1) − log(x − 1) is equal to
1 1 1 1 1 1
(a) + + +K (b) + + +...
33. Match the following columns x 2
x4
x 6
x 2
2x 4
3 x6
1 1 1
Column I Column II (c) − + +K (d) None of these
x2 2 x4 3 x6
1+ 2 + K + n 1
A. lim 1. 2x A Bx + C
n→ ∞ n2 4 40. If = + , then
x3 − 1 x − 1 x2 + x + 1
13 + 2 3 + K + n3 1
B. lim 2. (a) A = C ≠ B (b) A = B ≠ C
n→ ∞ n4 2
(c) A ≠ B = C (d) A ≠ B ≠ C
lim  + ... n terms 3.
1 1 1
C. + 41. The 14th term from the end in the expansion of
n→ ∞  2 ⋅ 5 5 ⋅ 8  5
( x− y)17 is
1 (a) C 5 x6 (−
17
y )5
4.
6
(b) 17C 6 (− x )11 y3
Codes 17 13 / 2 2
(c) C 4 x y
A B C A B C
(d) None of the above
(a) 2 1 4 (b) 2 3 4
(c) 1 3 4 (d) None of these 42. If the sum of the coefficients in the expansion
of (1 + 2 x)n is 6561, the greatest term in the
34. The two lines 4 x − 24 xy + 11 y = 0 make with
2 2
1
X-axis angles such that the difference of their expansion for x = is
2
tangents is
2 20 (a) 4th (b) 5th
(a) (b) 20 (c) − 11 (d) (c) 6th (d) None of these
11 11
446 AP EAMCET Chapterwise Mathematics

43. Assertion (A) Area of triangle formed by (0, 0), 1


49. If the primitive of is equal to
(x , y) and reflection of (x , y) in x-axis is| xy |. f (x)
log{ f (x)2} + c, then f (x) is
Reason (R) The reflection of the point (x , y) in
x
X-axis is (x , 0) (a) x + d (b) +d
2
(a) Both (A) and (R) are true and (R) is the correct
x2
explanation of (A) (c) +d (d) x2 + d
2
(b) Both (A) and (R) are true and (R) is not the
1 1 1 1
correct explanation of (A) 50. If I1 = ∫ dx and I 2 = ∫ dx, then
0| x|
(c) (A) is true but (R) is false
0
1 + x2
(d) (A) is false but (R) is true (a) I1 = I2 (b) I1 < I2
44. The point (4, 1) undergoes the following three I
(c) I1 > I2 (d) 1 = 2
transformations successively I2
1. Reflection about the line y = x 3
51. The value of ∫ { x} dx is equal to (where, {}
2. Translation through a distance 2 unit along 2
the positive direction of X-axis. denotes fractional point of x)
3. Rotation through an angle π / 4 about the (a)
1
(b)
5
origin in the clockwise direction. The final 2 2
position of the point is given by the 7
(c) (d) None of these
coordinates. 2
(a) 
1 7 
,  (b) ( 2 , 7 2 ) 52. The value of the expression
 2 2
 1 7  sech 2(tanh − 1 (1 / 2)) + cosech 2(coth − 1 3) is
(c)  − ,  (d) (− 2, 7 / 2 )
 2 2 (a) 35/9 (b) 43/4 (c) 35/4 (d) 43/9
 
45. A straight line passes through the point (1, 1) 53. If 2sinh − 1 
a  = log 1 + x  , then x is
cuts the coordinate axes and intercepted at the  1 − a2  1 − x 
 
point in the ratio 4 : 3, then equation is
(a) 3 x + 4 y = 7 (b) 4 x + 3 y = 7 equal to
(c) 4 x − 3 y = 1 (d) 3 x − 4 y + 1 = 0 (a) a (b) 2a (c) 3a (d) 4a

46. The equation of the bisector of the acute angle 54. The area inside the parabola 5 x − y = 0 but
2

between the lines 3 x − 4 y + 7 = 0 and outside the parabola 2 x 2 − y + 9 = 0 is


5 y + 12 x − 2 = 0 is
(a) 4 3 sq unit (b) 6 3 sq unit
(a) 5 x + 5 y + 3 = 0 (b) 5 x − 2 y + 1 = 0
(c) 10 x − 3 y + 7 = 0 (d) 11x − 3 y + 9 = 0 (c) 12 3 sq unit (d) 18 3 sq unit
x + 1 y −1 55. The number of all five digit numbers which are
47. If the angle θ between the line =
1 2 divisible by 4 that can be formed from the
z−2 digits 0, 1, 2, 3, 4 (without repetition) is
= and the plane 2 x − y + λ z + 4 = 0 is
2 (a) 30 (b) 34
1
such that the sinθ = . The value of λ is (c) 36 (d) None of these
3
4 3 3 5 56. The number of ordered triplets of positive
(a) − (b) (c) − (d)
3 4 5 3 integers which are solutions of the equation
48. A parallelopiped is formed by planes drawn x + y + z = 100 is
through the points (2, 3, 5) and (5, 9, 7), (a) 6005 (b) 4851
parallel to the coordinate planes. The length of (c) 5081 (d) None of these
a diagonal of the parallelopiped is
57. If 12 Pr = 11 P6 + 6 ⋅11 P5, then r is equal to
(a) 7 (b) 38
(c) 155 (d) None of these (a) 5 (b) 6
(c) 7 (d) None of these
Practice Set 2 447

π 2π 3π 4π
58. The value of cos + cos + cos + cos 66. Three numbers are chosen at random from
7 7 7 7 {1, 2, 3, … , 30}. The probability that they are
5π 6π 7π not consecutive, is
+ cos + cos + cos is
7 7 7 144 143
(a) (b)
(a) 0 (b) − 1 145 145
(c) 1 (d) None of these 142
(c) (d) None of these
145
59. The straight roads intersect at an angle of 60°.
A bus on one road is 2 km away from the  1 
67. The derivative of sec− 1   with respect to
intersection and a car on the other road is 3 km  2 x 2 − 1
away from the intersection. Then, the direct 1
distance between the two vehicles is 1 − x 2 at x = is
2
(a) 1 km (b) 2 km
(a) 2 (b) 4 (c) 1 (d) − 2
(c) 4 km (d) None of these
68. If f n (x), g n (x), hn (x), n = 1, 2, 3 are polynomials in
60. If in a ∆ABC, (sin A + sin B + sin C)
x such that f n (a) = g n (a) = hn (a), n = 1, 2, 3 and
(sin A + sin B − sin C) = 3sin A sin B, then
f1 (x) f 2(x) f 3(x)
(a) A = 60° (b) B = 60°
(c) C = 60° (d) None of these f (x) = g1 (x) g 2(x) g 3(x)
h1 (x) h2(x) h3(x)
61. If x 2 + y 2 + z2 = r 2, then tan − 1 
xy 

 zr  Then n f ′(a) is equal to
−1 yz − 1  xz (a) 0 (b) f1(a) g 2 (a)h3 (a)
+ tan   + tan   is equal to
 xr   yr  (c) 1 (d) None of these
(a) 0 (b) π
(c) π /2 (d) None of these
69. The point on the curve 3 y = 6 x − 5 x 3, the
normal at which passes through the origin is
62. In a ∆ABC, if A = 30°, b = 2, c = 3 + 1, then (a) (1, 1/3) (b) (1/3, 1)
C−B (c) (2, − 28 / 3) (d) (− 1, − 1 / 3)
is equal to
2
(a) 15° (b) 30° 70. Consider the circle x 2 + y 2 − 6 x + 4 y = 12. The
(c) 45° (d) None of these equations of a tangent of this circle that is
1 1 7 parallel to the line 4 x + 3 y + 5 = 0 is
63. For, n ∈ N , n 5 + n 3 + n is (a) 4 x + 3 y + 10 = 0 (b) 4 x + 3 y − 9 = 0
5 3 15
(a) an integer (b) a natural number (c) 4 x + 3 y + 9 = 0 (d) 4 x + 3 y − 31 = 0
(c) a positive fraction (d) None of these 71. The maximum value of x 3 − 3 x in the interval
1 1 [0, 2] is
1 +  + 2
64. The sum of nth bracket of ()
  3 3 (a) − 2 (b) 0
1 1 1 (c) 2 (d) 1
+  3 + 4 + 5  + K is
3 3 3 72. If X is a Poisson variate such that
(3n − 1)3 (3n − 1) P(X = 2) = 9P(X = 4), the mean and variance of
(a) (b)
2 ⋅ 4n − 1 2 ⋅ 3( n − 1) ( n + 2 )/ 2
X are
3n + 1 (a) 1, 1 (b) 2, 2
2 2
(c) , (d)
2 2
,
(c) (d) None of these
3⋅7n − 1 3 3 3 3

65. If a party of n persons sit at a round table, then 73. If on an average, out of 10 ships, one is
the odds against two specified individuals drowned, then what is the probability that out
sitting next to each other are of 5 ships atleast 4 reach safely?
(a) 2 : n − 3 (b) n − 2 : 2 (a) 1.4(0.9)2 (b) 1.4(0.9)5
(c) n − 3 : 2 (d) None of these (c) 1.4(0.9)3 (d) 1.4(0.9)4
448 AP EAMCET Chapterwise Mathematics
(a) x2 + y2 − 6 x + 2 y − 18 = 0
74. A chimney of 20 m height, standing on the top
(b) x2 + y2 − 6 x + 2 y − 28 = 0
of a building subtends an angle whose tangent
(c) x2 + y2 − 6 x + 2 y − 38 = 0
is 1/6 at a distance of 70 m from the foot of the
building. The height of the building is (d) None of the above
(a) 50 m (b) 100 m 77. Equation x 2 + y 2 + 4 x + 6 y + 13 = 0 represents
30 (a) a point (b) an ellipse
(c) 13.79 m (d) m
3 (c) a circle (d) None of these

75. Using Trapezoidal rule, the 78. If f (x) = x 2 − 10 x + 25, then the derivative at
x 0 π /8 π/4 x = 5 is
(a) 0 (b) − 1
tan x 0 0.4141 1
(c) 1 (d) None of these
π/4
approximate value of ∫ tan xdx is equal to 79. Let f (x + y) = f (x) ⋅ f (y) for all x , y where,
0 f (0) ≠ 0. If f (5) = 2 and f ′ (0) = 3, then f ′(5) is
π π equal to
(a) (18282
. ) (b) (18278
. )
16 16 (a) 0 (b) 1
π π
(c) (18276
. ) (d) (18272
. ) (c) 6 (d) None of these
16 16
1 2 3 1
76. Equation of the circle having centre at (3, − 1) 80. The rank of 2 4 6 2 is
and cutting the intercept of length 6 unit on  
1 2 3 2
the line 2 x − 5 y + 18 = 0 is
(a) 0 (b) 1 (c) 2 (d) 3

Answers
1. (d) 2. (b) 3. (c) 4. (b) 5. (c) 6. (b) 7. (b) 8. (a) 9. (c) 10. (d)
11. (d) 12. (d) 13. (b) 14. (d) 15. (c) 16. (b) 17. (c) 18. (a) 19. (a) 20. (d)
21. (b) 22. (d) 23. (b) 24. (b) 25. (d) 26. (b) 27. (a) 28. (a) 29. (a) 30. (c)
31. (c) 32. (c) 33. (a) 34. (d) 35. (a) 36. (b) 37. (b) 38. (a) 39. (b) 40. (a)
41. (c) 42. (b) 43. (c) 44. (c) 45. (b) 46. (d) 47. (d) 48. (a) 49. (b) 50. (c)
51. (a) 52. (c) 53. (a) 54. (c) 55. (a) 56. (b) 57. (b) 58. (b) 59. (d) 60. (c)
61. (c) 62. (b) 63. (b) 64. (b) 65. (c) 66. (a) 67. (b) 68. (a) 69. (a) 70. (d)
71. (c) 72. (d) 73. (d) 74. (a) 75. (a) 76. (b) 77. (a) 78. (d) 79. (c) 80. (c)
Solutions
2x m2 − 1
1. (d) Since, tan − 1 = 2 tan − 1 x, for ⇒ sin A cos A =
1 − x2 2
−1 < x <1 ∴ From Eq. (i),
 π π  m2 − 1
∴ Range of f (x) is  − ,  . m3 − 3 m = n
 2 2  2 
 π π ⇒ 2m3 − 3m3 + 3m = 2n
Hence, codomain of B is  − ,  .
 2 2
⇒ m3 − 3m + 2n = 0
2. (b) Since, f (x) = 2 x + 3 x + 4 x
6 4 2
1 π
Now, f (− x) = 2(− x)6 + 3(− x)4 + 4(− x)2
6. (b) (A) sinθ = − ⇒ θ=−
2 6
= 2 x 6 + 3 x 4 + 4 x 2 = f (x) 1 2π
(B) cosθ = − ⇒θ=
2 3
∴ f (x) is an even function and derivative of an
even function is always an odd function. 3 π
(C) sinθ = − ⇒θ=−
2 3
3. (c) Let f (x) = log e (x 2 − 6 x + 6) it is defined, if
3 5π
log(x 2 − 6 x + 6) ≥ 0 (D) cosθ = − ⇒θ =
2 6
⇒ x2 − 6x + 6 ≥ 1
7. (b) Let the equation of the circle be
⇒ (x − 5) (x − 1) ≥ 0
S1 ≡ x 2 + y 2 − 3 x − 4 y + 5 = 0
⇒ x ≤ 1 or x ≥ 5
7 8 11
Hence, the domain of the function will be and S2 ≡ x 2 + y 2 − x+ y+ =0
3 3 3
(− ∞ , 1] ∪ [5, ∞).
The equation of radical axis is S1 − S2 = 0
π 3π 5π π π 
4. (b) sin sin sin = cos  −  ⇒ − 2 x − 20 y − 4 = 0 ⇒ y = −
x 1

14 14 14  2 14 
10 5
 π 3π   π 5π 
cos  −  cos  −  1
 2 14   2 14  ∴ Gradient = −
10
6π 4π 2π
= cos cos cos 8. (a) The sum of the coefficient of the
14 14 14
polynomial (α 2 x 2 − 2αx + 1)51 is obtained by
π 2π 3π
= cos cos cos putting x = 1 in (α 2 x 2 − 2αx + 1)51 .
7 7 7
π 2π 4π Q Sum of the coefficients = 0
= − cos cos cos
7 7 7 ⇒ (α 2 − 2α + 1)51 = 0
− sin(23 π / 7) − sin 8π / 7 1 ⇒ α =1
= = =
23 sin π / 7 8sin π / 7 8 ∞ xn
x x2 x3
9. (c) Σ = + + + ...
5. (c) Since, sin A + cos A = m and n = 1 (2n − 1)! 1! 3! 5!
sin 3 A + cos 3 A = n  x ( x)3  e x
− e− x 
= x + + ... = x  

⇒ (sin A + cos A)3 − 3sin A cos A  
 1! 3!  2
(sin A + cos A) = n
1 1
⇒ m − 3sin A cos A(m) = n
3
…(i) 10. (d) Let S = + +K∞
1⋅ 3⋅ 5 3⋅ 5⋅ 7
Again, sin A + cos A = m ∴ Tn =
1
⇒ sin 2 A + cos 2 A + 2sin A cos A = m2 (2n − 1) (2n + 1) (2n + 3)
450 AP EAMCET Chapterwise Mathematics

1 1 1  1 1 1  14. (d) Let A be the first term and R be the


=  − −  − 
8  2n − 1 2n + 1 8  2n + 1 2n + 3 common ratio of the GP, then
l = AR p − 1
1 1 1 ∞  1 1 
= 1 −  + Σ  −  ⇒ log l = log A + (p − 1)log R …(i)
8 3 8 n = 2  2n − 1 2n + 1
m = AR q − 1
1 ∞  1 1 
− Σ  −  ⇒ log m = log A + (q − 1)log R …(ii)
8 n = 1  2n + 1 2n + 3
and n = AR r − 1
1 1 ∞  1 1  ⇒ log n = log A + (r −1)log R …(iii)
= + Σ  − 
12 8 n = 1  2m + 1 2m + 3 On multiplying Eqs. (i), (ii) and (iii) by (q − r),
1 ∞  1 1  (r − p) and (p − q) and adding all these
− Σ  −  equations, we get
8 n = 1  2n + 1 2n + 3
log l(q − r) + log m(r − p)
1
= + log n(p − q) = 0 …(iv)
12 log l p 1
11. (d) Given that, z = i log(2 − 3) Let ∆ = log m q 1
2
log( 2 − 3) log n r 1
Now, e iz = e i
= e − log( 2 − 3) = log l(q − r) + log m(r − p) + log n(p − q)
=0 [from Eq. (iv)]
⇒ e iz = (2 − 3)− 1 = 2 + 3 n
Cr n − (r − 1)
Similarly, e − iz
=2− 3 15. (c) Since, n
=
Cr − 1 r
We know that,
 C   C   Cn 
e iz + e − iz (2 + 3) + (2 − 3) Let E = 1 + 1  1 + 2  K 1 + 
cos z = = =2  C0   C1   C n − 1 
2 2
 n  n − 1  1
12. (d) Since, |1 − i |x = 2x = 1 +  1 +  K 1 + 
 1  2   n
⇒ ( 2)x = 2x … (i)
(n + 1)n
⇒ 2x / 2 = 2x =
n!
x
⇒ =x ⇒ x≠0 16. (b) Let z = x + iy, Then,
2
Hence, no non-zero solution exist. | z + 4 |2 − | z − 4 |2 = 8

13. (b) Let log a x = y ⇒ 4 Re(z) = 8


1
2 1 3 ⇒ Re(z) =
∴ + + =0 2
y 1+ y 2+ y
2 + 3y 3 17. (c) Equation of circles with limiting point
⇒ =− (1, 2) and (4, 3) are (x − 1)2 + (y − 2)2 = 0
y(1 + y) 2+ y
and (x − 4)2 + (y − 3)2 = 0
⇒ 3y 2 + 8y + y = − 3y − 3y 2
Any circle coaxial with these circles is
⇒ 6 y 2 + 11 y + y = 0 (x −1)2 + (y − 2)2 + λ {(x − 4)2 + (y − 3)2} = 0 … (i)
⇒ (3 y + 4) (2 y + 1) = 0 This will pass through the origin.
4 1 1
⇒ y =− ,− ∴ 1 + 4 + λ(16 + 9) = 0 ⇒ λ = −
3 2 5
4 1
⇒ log a x = − , − ∴From Eq. (i) (x 2 + y 2 − 2 x − 4 y + 5)
3 2
1
⇒ x = a − 4 / 3 or a − 1 / 2 − (x 2 + y 2 − 8 x − 6 y + 25) = 0
5
Practice Set 2 451
⇒ 4 x 2 + 4 y 2 − 2 x − 14 y = 0 πx 
21. (b) lim(1 − x)tan  
⇒ 2(x 2 + y 2) − x − 7 y = 0 x →1  2
(1 − x) −1
x−4 x−4 = lim = lim
18. (a) Now, = x →1  πx  x → 1  πx  π
x 2 − 5 x + 6 (x − 2) (x − 3) cot   − cosec 2  
 2  2 2
2 1
= − (using L’Hospital’s rule)
x−2 x−3
2 2  πx  2
 x
−1
1 x
−1 = lim sin   =
= − 1 −  + 1 −  π x →1  2 π
 2 3 3
22. (d) Given that,
 x  x
2  1 x  x
2 
= − 1 + +   + .. + 1 + +   + .. ax 2 + b, 0 ≤ x < 1
 2  2  3 3  3  
f (x) =  x + 3, 1 < x ≤ 2
n n  4, x =1
 1 1  1 
∴ Coefficient of x n is −   +  
 2 3  3 At x =1
1 1 LHL = lim f (1 − h) = lim a(1 − h)2 + b = a + b
= n+1 − n h→ 0 h→ 0
3 2
RHL = lim f (1 + h) = lim(1 + h) + 3 = 4
19. (a) Given curves are h→ 0 h→ 0
θ 1 = 4.
and f ()
x = cos θ + log tan
2 Since, f (x) is not continuous at x = 1, therefore
and y = sinθ a + b ≠ 4.
On differentiating w.r.t θ respectively, we get Hence, option (d) is correct.
dx sec2 θ / 2 23. (b) The required vector =| b |a
= − sin θ +
dθ 2 tan θ / 2
| b| 32 + 62 + 22 $ $
1 = a= (i + 2 j + 2k$ )
= − sin θ + | a| 12 + 22 + 22
θ θ
2sin cos
2 2 7
= (i$ + 2$j + 2k$ )
1 1 − sin 2 θ 3
= − sin θ + =
sin θ sin θ $i $j k$
cos 2 θ 24. (b) Now, a × b = 2 1 − 2
=
sin θ 1 1 0
dy
and = cos θ
dθ = 2$i + 2$j + k$
dy dy / dθ cos θ sin θ ⇒ |a + b| = 4 + 4 + 1 = 3
∴ = = = tan θ
dx dx / dθ cos 2 θ Also, | c − a |2 = 8
2
d y dθ sin θ sin θ ⇒ | c |2 + | a |2 − 2a ⋅ c = 8
Also, = sec2 θ ⋅ = sec2 θ =
dx 2
dx cos θ cos 4 θ
2
⇒ | c |2 + 9 − 2| c | = 8
2
d y sin θ ⇒ | c |2 − 2| c | + 1 = 0
Since, =0 ⇒ =0
dx 2
cos 4 θ ⇒ (| c | − 1)2 = 0
⇒ θ = nπ , n ∈ I
⇒ | c| = 1
0 1 − 1 1 3
∴|(a × b) × c | =| a × b|| c |sin 30° = 31
() =
20. (d) Since, A = 2 1 3  2 2
 
3 2 1  25. (d) (a − b) ⋅ [(b − c) × (c − a)]
∴ | A | = 0 − 1(2 − 9) − 1(4 − 3) = 7 − 1 = 6 = (a − b) ⋅ (b × c − b × a + c × a)
452 AP EAMCET Chapterwise Mathematics

= a ⋅ (b × c) − a ⋅ (b × a) + a ⋅ (c × a) 1
29. (a) Let I = ∫ dx
− b ⋅ (b × c) + b ⋅ (b × a) − b ⋅ (c × a) x2 1 + x2
=0
1 1
Put = t ⇒ − 2 dx = dt
26. (b) According to the question, x x
dy dy 1 dt tdt
y = λy 2 ⇒ = λy ⇒ dy = λdx ∴ I =−∫ =−∫
dx dx y
1+ 2
1 t2 + 1
On integrating, we get t
log y = λx + log c ⇒ y = ce λx Again, put t 2 + 1 = u 2 ⇒ 2t dt = u du
dy
27. (a) Since = e x − y (e x − e y) ∴ I = − ∫ du = − u + C = − t 2 + 1 + C
dx
⇒ ey
dy
+ e x ⋅ e y = e 2x 1 1 + x2
dx =− + 1 + C = − +C
x2 x
dy dv
Put e y = v ⇒ e y =
30. (c) Let I = ∫ x log 1 +  dx
1
dx dx
 x
dv
∴ + e x ⋅ v = e 2x = ∫ x log(x + 1) − ∫ x log xdx
dx
x x
∴ IF = e ∫ e dx
= ee x2 1 x2
= log(x + 1) − ∫ dx
∴The solution is 2 2 x +1
x2 1 x2
v⋅e ex
=∫e ex
⋅e 2x
dx + c log x + ∫
− dx
2 2 x
x 2
1  1 
= log(x + 1) − ∫  x − 1 +
x
⇒ v ⋅ e e = ∫ e z ⋅ zdx + c  dx
2 2  x + 1
(Put e x = z ⇒ e x dx = dz) x2 x2
x
log x + −
⇒ v ⋅ e e = (z − 1)e z + c 2 4
x2 x2 1  x2 
x x
⇒ e y ⋅ e e = (e x − 1)e e + c ⇒ e y = e x − 1 + ce − e
x = log(x + 1) − log x −  − x
2 2 2 2 
28. (a) Given differential equation can be rewritten 1 x2
as −
log(x + 1) + +C
2 4
dy x−y+3 x 2
x 2
1 x
= = log(x + 1) − log x − log(x + 1) + + C
dx 2(x − y) + 5 2 2 2 2
dy dv But, I = f (x) log(x + 1) + g(x) ⋅ x 2 + Ax + C
Put x − y = v ⇒ 1 − =
dx dx
x2 1 1 1
dy dv ∴ f (x) = − , g(x) = − log x and A =
⇒ =1 − 2 2 2 2
dx dx
∴ The given equation becomes 31. (c)
y
dv v+3
1− =
dx 2v + 5
x=√3y
dv v+2
⇒ =
dx 2v + 5 Q(x2, y)
(x1, y) P
2v + 5  1  x′ x
⇒ dv = dx ⇒  2 +  dv = dx
v+2  v + 2
⇒ 2v + log(v + 2) + c = x
⇒ 2(x − y) + log(x − y + 2) + c = x
⇒ 2 y − x = log(x − y + 2) + c
y′
Practice Set 2 453
∴ Required area ⇒ α 2n + p nα n + q n = 0 and β 2n + p nβ n + q n = 0
1 1
= ∫ (x 2 − x1)dy = ∫ ( 4 − y 2 − 3 y) dy ⇒ α 2n − β 2n + p nα n − p nβ n = 0
0 0 ⇒ (α n + β n)(α n − β n) + p n (α n − β n) = 0
1 2 1
1 y 3y ⇒ (α n − β n)(α n + β n + p n) = 0
=  y 4 − y 2 + 4 sin − 1 −
 2 2 2 2  0 ⇒ α n + βn + p n = 0 …(ii)
3  1 3 α β
= + 2sin − 1   − − 2sin − 1 0 Since, ,
β α
are the roots of
2  2 2
3  π 3 π x n + 1 + (x + 1)n = 0
= + 2  − = sq unit.
2  6 2 3 Therefore, α + β n + (α + β)n = 0
n

α n + β n = − (α + β)n … (iii)
32. (c) Let the vertices of the tetrahedron be
A(1, − 6, 10), B(− 1, − 3, 7), C(5, − 1, λ) and From Eqs. (ii) and (iii), we get
D(7, − 4 , 7) − p n = − (α + β)n
Now, AB = (− 2, 3, − 3), AC = (4 , 5, λ − 10) ⇒ p n = (α + β)n
and AD = (6, 2, − 3) ⇒ p n = (− p)n [from Eq. (i)]
Since, volume of tetrahedron = 11 ⇒ n is an even
−2 3 −3
1 x2 + 4 x + 3
∴ 4 5 λ − 10 = ± 11 36. (b) Now, f (x) =
6 x − 6 x 2 + 11 x − 6
3
6 2 −3 (x + 1) (x + 3)
=
⇒ − 2(− 15 − 2λ + 20) − 3(− 12 − 6λ + 60) (x − 1) (x − 2) (x − 3)
− 3(8 − 30) = ± 66 Critical points are x = − 3, − 1, 1, 2, 3
⇒ − 10 + 4 λ − 144 + 18λ + 66 = ± 66 Q f (x) > 0
⇒ − 88 + 22λ = ± 66 (x + 1) (x + 3)
∴ >0
(x − 1) (x − 2) (x − 3)
⇒ λ = 1 or 7
– + – + – +
Σn n(n + 1) 1 ∞ ∞
33. (a) (A) lim = lim =
n → ∞ n2 n→ ∞ 2n 2 2 –3 –1 1 2 3
2
 n(n + 1) ⇒ x ∈ (− 3, − 1) ∪ (1, 2) ∪ (3, ∞)
 
Σn 3  2  1
(B) lim 4 = lim 4
= 37. (b) Let f (x) = x 4 − 4 x − 1. Then, the number of
n→ ∞ n n→ ∞ n 4
changes of signs in f (x) is 1. Therefore, f (x) can
 1 1 1  have at most one positive real root. We have,
(C) lim  + + .. +
n → ∞  2⋅ 5 5⋅ 8 (3n − 1) (3n + 2)  f (2) > 0 and f (0) < 1. Therefore, f (x) has one
+ ve real root between 1 and 2.
1 1 1  1
= lim − =
n→ ∞ 3  2 3n + 2  6 38. (a) The nth term of the given series is
C(n ,0) + C(n , 1) + C(n , 2)+ K+ C(n , n) 2n
34. (d) Difference of their tangents Tn = =
P(n , n) n!
2 h 2 − ab 2 144 − 44 20
= = = ∞ ∞ 2n
| b| 11 11 ∴ Σ Tn = Σ = e 2 −1
n =1 n =1 n!
35. (a) Since, α and β are the roots of 39. (b) Now, 2 log x − log(x + 1) − log(x − 1)
x 2 + px + q = 0
= log x 2 {log(x + 1) + log(x − 1)}
∴ α + β = − p and αβ = q … (i)
 x2   1
Also, α and β are the roots of = log  2  = − log 1 − 2 
 x − 1  x 
x 2n + p n x n + q n = 0
454 AP EAMCET Chapterwise Mathematics
1 1 1 [Since, X = x cos θ − y sin θ, Y = x sin θ + y cos θ]
+ =
+ + ...
x2 2x4 3x6  1 7
Hence, the coordinates of D are  − , .
2x A Bx + C  2 2
40. (a) Since, 3 = +
x − 1 x − 1 x2 + x + 1 x y
45. (b) Let the equation of line be + = 1 … (i)
⇒ 2 x = A(x 2 + x + 1) + (Bx + C) (x − 1) a b
On equating the coefficient of x 2, x and Since, the line through point (1, 1) intercept in
constant, we get the ratio 4 : 3.
0= A + B  3 ⋅ 0 + 4 ⋅ a 3 ⋅ b + 4 ⋅ 0
∴ (1, 1) =  , 
2 = A − B + C and 0 = A − C  3+ 4 3+ 4 
2 2 7 7
⇒ A = C = and B = − ⇒ a= ,b=
3 3 4 3
41. (c) The 14th term from the end in the On putting the values of a and b in Eq. (i), we
get
expansion of ( x − y)17 is the (18 − 14 + 1)th i.e.
4 x + 3y = 7
5th term from the beginning and is given by
17
C 4 ( x)13(− y)4 = 17C 4 x13/ 2 y 2. 46. (d) Given lines are 3x − 4 y + 7 = 0
and 12 x + 5 y − 2 = 0
42. (b) Since, the sum of the coefficients in the Here, a1 a2 + b1 b2 = 3 × 12 − 4 × 5 = 16 > 0
expansion of (1 + 2 x)n = 6561
∴ Acute angle bisector is
∴ (1 + 2)n = 6561 (put x = 1) 3 x − 4 y + 7 [−12 x + 5 y − 2]
=
⇒ 3n = 38 ⇒ n = 8 5 13
Tr + 1 8
C r (2 x)r 9−r ⇒ 39 x − 52 y + 91 + 60 x + 25 y − 10 = 0
Now, = = × 2x
Tr 8
C r − 1 (2 x)r − 1 r ⇒ 11 x − 3 y + 9 = 0
Tr + 1 9−r  1 47. (d) The angle between the line
⇒ = Q x =  x + 1 y −1 z − 2
Tr r  2 = =
1 2 2
Tr + 1 9−r
Q >1 ⇒ >1 and the plane 2x − y + λ z + 4 = 0 is
Tr r
9 1 1 × 2 − 2 ×1 + 2× λ
⇒ 9−r >r ⇒ r < ⇒ r <4 sinθ =
2 2 12 + 22 + 22 22 + 12 + λ
Hence, 5th term is the greatest term.
1 2 λ
1 x y ⇒ =
43. (c) (A) Area of triangle = 3 3 5+ λ
2 x −y
5
1 ⇒ 5 + λ = 4λ ⇒ λ =
= | − 2 xy | = | xy | 3
2
(R) the reflection of (x , y) w.r.t. X-axis is (x , − y). 48. (a) The length of the edges are
a = 5 − 2 = 3, b = 9 − 3 = 6
44. (c) Let B, C and D be the positions of the point c =7−5=2
A(4 , 1) after the three operations I, II and III
respectively. Then, B is (1, 4), C is (1 + 2, 4) i.e. ∴ Length of the diagonal = 32 + 62 + 22
(3, 4). The point D is obtained from C by = 9 + 36 + 4 = 7
rotating the coordinate axes through an angle
π / 4 in anticlockwise direction. 1
49. (b) Given that, ∫ dx = log{ f (x)} 2 + c
∴The coordinates of D are given by f (x)
π π 1 1 1
X = 3 cos − 4 sin = − ⇒ = 2 f (x) f ′ (x)
4 4 2 f (x) { f (x)} 2
π π 7 1
and Y = 3sin + 4 cos = ⇒ f ′ (x) =
x
⇒ f (x) = + d
4 4 2 2 2
Practice Set 2 455
The number of numbers ending in 04 = 3! = 6
50. (c) Since,| x | < 1 + x 2
The number of numbers ending in12 = 3!− 2!
1 1
⇒ > = 6 − 2= 4
| x| 1 + x2 The number of numbers ending in 20 = 3! = 6
1
1
1
1 The number of numbers ending in 24 = 3!− 2!
∴ ∫ | x | dx > ∫ 1 + x2
dx
=4
0 0
The number of numbers ending in 32 = 3!− 2!
⇒ I1 > I 2
=4
3 3 3 3
51. (a) ∫ { x} dx = ∫ (x − [ x]) dx = ∫ x dx − ∫ [ x] dx The number of numbers ending in 40 = 3! = 6
2 2 2 2 ∴The required five digit numbers
x 2 3 = 6 + 4 + 6 + 4 + 4 + 6 = 30
5 1
=   − [2 x]32 = − 2 =
2
 2 2 2 56. (b) Required number of ways = 100 − 1C 3 − 1
52. (c) sech 2(tanh − 1 (1 / 2)) + cosech 2(coth − 1 3) 99 × 98
= 99C 2 = = 4851
−1
= 1 − tanh (tanh (1 / 2)] + coth (coth
2 2 −1
3) − 1 2 ×1
2
 1 1 35 57. (b) Since, we know
= −   + 32 = − + 9 =
 2 4 4 r ⋅n − 1 Pr − 1 + n −1
Pr = n Pr ⇒ 6 ⋅11 P5 + 11
P6 = 12P6
  ⇒ Pr = 12P6 ⇒ r = 6
 = log 1 + x 
12
a
53. (a) Since, 2sinh − 1 
 1 − a2  1 − x  π 2π 3π 4π
  58. (b) cos + cos + cos + cos
Put a = tanhθ 7 7 7 7
5π 6π 7π
 tanh θ  1 1 + tanh θ  + cos + cos + cos
∴ 2sinh − 1   = log   7 7 7
 sec h θ  2  1 − tanh θ 
 π 6π   2π 5π 
=  cos + cos  +  cos + cos 
⇒ θ = tanh − 1 x  7 7  7 7
54. (c) The point of intersection of the curves  3π 4π 
+  cos + cos  + cos π
5 x − y = 0 and 2 x − y + 9 = 0 are (± , 3 , 15).
2 2  7 7
3  π π  2π 2π 
=  cos − cos  +  cos − cos 
∴Required area = 2 ∫ (9 − 3 x 2) dx = 2[9 x − x 3]0 3  7 7  7 7
0
 3π 3π 
= 2[9 3 − 3 3] = 12 3 sq unit. +  cos − cos  + cos π
 7 7
y
= cos π = − 1
59. (d) Let the two roads intersect at A. If the bus
2x2–y+9=0 and the car at B and C on the two roads
respectively, then c = AB = 2 km, b = AC = 3km
(√3, 15) A

5x2–y=0
x′ x
3k
m
2k

y′

55. (a) The number formed will be divisible by 4, if


B C
the number formed by the two digits on the
extreme right is divisible by 4. i.e. it should be The distance between the two vehicles
04, 12, 20, 24, 32, 40. = BC = a km
456 AP EAMCET Chapterwise Mathematics

b2 + c 2 − a2 n 5 n 3 7n
Q cos A = 63. (b) Let P(n) = + +
2bc 5 3 15
32 + 22 − a 2 1 13 − a 2 Put n =1
⇒ cos 60° = ⇒ =
2×3×2 2 12 1 1 7 3+ 5+ 7
1 = + +
P() = =1
⇒ a 2 = 7 ⇒ a = 7 km 5 3 15 15
Again, put n = 2
60. (c) Since, (sin A + sin B + sin C)
32 8 14
(sin A + sin B − sin C) = 3sin A sin B P(2) = + +
5 3 15
⇒ (sin A + sin B)2 − sin 2 C = 3sin A sin B
96 + 40 + 14 150
⇒ sin 2 A + sin 2 B − sin 2 C = sin A sin B = = = 10
15 15
⇒ sin 2 A + sin(B + C)sin(B − C) = sin A sin B ∴Given expression is a natural number.
⇒ sin 2 A + sin A sin B(B − C) = sin A sin B 1 1
64. (b) Let S = ()
1 +  1 + 2
  3 3
⇒ sin A [sin(B + C) + sin(B − C)] = sin A sin B
⇒ 2sin A sin B cos C = sin A sin B 1 1 1
+  3 + 4 + 5 + K
1 3 3 3
⇒ cosC = (Qsin A sin B ≠ 0) 1 1
2 ∴ Tn = + ... + ( n + 1) n / 2 − 1
⇒ C = 60° 3n( n − 1)/ 2 3
 1 
1− n
61. (c) Now, tan − 1 
xy  − 1  yz 
 + tan   1  3  3(3n − 1)
 zr   xr  = n( n − 1)/ 2   = n n( n − 1)/ 2
 1 −  2⋅ 3 3
3 1
 xz
+ tan − 1    3
 yr 
3n − 1
 xy =
yz xz xyz  2⋅ 3( n − 1) ( n + 2)/ 2
 + + − 3 
= tan − 1  r 
zr xr yr

65. (c) The total number of ways of arranging n
 x 2 + y 2 + z2  
1−   persons at a round table n(S) = (n − 1)!
  r2   Let E = Event of getting two specified person
sitting next to each other.
 xy yz xz xyz 
−1
 zr + xr + yr − r 3  ∴ n(E) = 2!(n − 2)!
= tan   n(E)
 r2  ∴Required probability P(A) =
1− 2 n(S)
 r 
2!(n − 2)! 2 n−3
(Q x 2 + y 2 + z2 = r 2 given) = = and P(A) =
(n − 1)! n −1 n −1
π
= tan − 1 ∞ =
2 ∴ P(A) : P(A) = (n − 3) : 2
C−B c−b A 30 × 29 × 28
62. (b) We know, tan = cot 66. (a) Here, n(S) = 30C 3 = = 4060
2 c+b 2 3 × 2 ×1
 C − B 3 +1− 2 The number of ways in which the three
⇒ tan   = cot15° numbers are consecutive = 28
 2  3 +1+ 2
28 1 144
3 −1 3 +1 ∴ P(A) = 1 − =1 − =
= × 4060 145 145
3+3 3 −1
 1 
=
1
= tan 30°
67. (b) Let y = sec− 1   , z = 1 − x2
 2 x 2 − 1
3
C−B Put x = cosθ, we get
⇒ = 30°
2 y = sec− 1 (sec 2θ) = 2θ = 2 cos − 1 x
Practice Set 2 457
dy 2 dz x
⇒ =− and =− 70. (d) We have,
dx 1− x 2 dx 1 − x2 x 2 + y 2 − 6 x + 4 y = 12
2 ⇒ (x − 3)2 + (y + 2)2 = 25

dy 1− x 2  dy 
2
⇒ (x − 3)2 + (y + 2)2 = (5)2
∴ = = ⇒   =4
dz − x x  dx  x = 1
2 Equation of tangent whose slope m is
1 − x2
y + 2 = m (x − 3) ± 5 m2 + 1 … (i)
68. (a) Given that, Now, this tangent is parallel to line
f1 (x) f 2(x) f 3(x) 4 x + 3y + 5 = 0
f (x) = g1 (x) g 2(x) g 3(x) ∴ Slope of line is −
4
h1 (x) h2(x) h3(x) 3
4
Put the value of m = − is Eq. (i), we get
f1′ (x) f 2′ (x)
f 3′ (x) 3
∴ f ′ (x) = g1 (x) g 2(x)
g 3(x) − 4
2
(x − 3) ± 5 
4
y + 2= −  +1
h1 (x) h2(x) h3(x) 3  3
−4
(x − 3) ± 5  
f1 (x) f 2(x) f 3(x) f1 (x) f 2(x) f 3(x) 5
⇒ y + 2=
3  3
+ g1′ (x) g 2′ (x) g 3′ (x) + g1 (x) g 2(x) g 3(x)
h1 (x) h2(x) h3(x) h1′ (x) h2′ (x) h3′ (x) ⇒ 3y + 6 = − 4 x + 12 ± 25
⇒ 4 x + 3y = 6 ± 25
Since, f n (a) = g n (a) = hn (a), n = 1, 2, 3, therefore
two rows in each determinant become identical ⇒ 4 x + 3y = 31 or 4 x + 3y = − 19
on putting x = a Hence, equation of tangent is
Hence, f ′ (a) = 0 4 x + 3y − 31 = 0 or 4 x + 3y + 19 = 0

69. (a) Let the required point be (x1 , y1) 71. (c) Let f (x) = x 3 − 3 x. Then,
Since, 3y = 6x − 5x 3 f ′ (x) = 3 x 2 − 3
dy For maximum or minimum, put f ′ (x) = 0
⇒ 3 = 6 − 15 x 2 ⇒ x = ± 1. But, x = − 1 ∉ [0, 2]
dx
Therefore, x = 1 is only in the interval.
dy
⇒ = 2 − 5x2 Now, f ′′(x) = 6 x
dx
⇒ 1 = 6 > 0, minima.
f ′′()
 dy 
⇒   = 2 − 5 x12 Now, f (0) = 0, f ()
1 = − 2 and f (2) = 6
 dx  ( x
1 , y1 ) Hence, the maximum value at x = 2.
The equation of the normal at (x1 , y1) is
72. (d) Since, P(X = 2) = 9P(X = 4)
1
y − y1 = − (x − x1) e − λ λ2 e − λ λ4
2 − 5 x12 ⇒ =9
2! 4!
If it passes through the origin, then
2
1 ⇒ λ=
0 − y1 = − (0 − x1) 3
2 − 5 x12
2 2
∴ Mean = , variance =
x1 3 3
⇒ y1 = − … (i)
2 − 5 x12
73. (d) Here, p = 0.9, q = 01
.,n=5
Since, (x1 , y1) lies on the given curve. Therefore,
∴P(X ≥ 4) = P(X = 4) + P(X = 5)
3 y1 = 6 x1 − 5 x13 … (ii) = 5C 4 (0.9)4 (01
. )1 + 5C 5(0.9)5(01
. )0
 1
On solving Eqs. (i) and (ii), we get 1,  . = (0.9)4 (0.5 + 0.9) = (1.4) (0.9)4
 3
458 AP EAMCET Chapterwise Mathematics

1 h In ∆DAC, CA 2 = ( 29)2 + (3)2 = 38


74. (a) Since, tanα = , tanβ =
6 70
20 + h
∴ tan(α + β) =
70
C(3, –1)
D

20 m A 3 D 3 B 2x–5y+18=0

C Thus, equation of circle is


(x − 3)2 + (y + 1)2 = 38

h i.e. x 2 + y 2 − 6 x + 2 y − 28 = 0

α 77. (a) Given equation of curve is


β x 2 + y 2 + 4 x + 6 y + 13 = 0
A B
70 m
Here, centre is (− 2, − 3) and radius
tan α + tan β 20 + h
⇒ = = 4 + 9 − 13 = 0
1 − tan α tan β 70
∴ It is a point circle.
1 h
+
⇒ 6 70 = 20 + h 78. (d) Given that,
1 h 70 f (x) = x 2 − 10 x + 25
1− ⋅
6 70
= (x − 5)2 = | x − 5|
70 + 6h 20 + h
⇒ = x − 5 , x≥5
420 − h 70 =
⇒ 4900 + 420h = 8400 + 420h − 20h − h 2 5 − x , x<5
 1, x >5
⇒ h 2 + 20h − 3500 = 0 f ′ (x) = 
− 1, x <5
⇒ h = 50 m
It is clear that f ′(x) does not exist.
π
75. (a) Here, a = 0, b = , n = 2 f (5 + h) − f (5)
4 79. (c) Now, f ′ (5) = lim
h→ 0 h
π
−0 f (5 + h) − f (5 + 0)
b−a 4 π = lim
∴ h= = = h→ 0 h
n 2 8
Using trapezoidal rule, f (5) ⋅ f (h) − f (5) ⋅ f (0)
= lim
π/4 h→ 0 h
h
∫ tan x dx =
2
(y 0 + y 2 + 2 y1) 
=  lim
f (h) − f (0)
 ⋅ f (5)
0 h→ 0 h 
π
= [0 + 1 + 2(0.4141)] = f ′ (0) ⋅ f (5)
16
= 3⋅ 2 = 6
1.8282π
= 1 2 3 1 1 2 3 1 
16
80. (c) 2 4 6 2 ~ 0 0 0 0
∴[ A (adj A) A − 1 ] A = A(adj A) A − 1 A   
1 2 3 20 0 0 1 
= det AI = 6 I
1 2 3 1 
| 2 × 3 − 5(− 1) + 18|
76. (b) Now, CD = R2 → R2 − 2R1 , R3 → R3 − R1 ~ 0 0 0 1 
4 + 25  
0 0 0 0
29
= = 29
29 Hence, rank is 2.
Practice Set 3 459

Practice Set 3
1. If θ lies in the second quadrant, then the value 9. If f is the greatest integer function and g is
1 − sin θ 1 + sin θ  5
of + is the modulus function, then gof  − 
1 + sin θ 1 − sin θ  3
(a) 2sec θ (b) 2 cosec θ  5
− fog  −  is equal to
(c) − 2 sec θ (d) None of these  3
π   2π  (a) 1 (b) − 1
2. If tan x + tan  + x  + tan  + x  = 3, then
3   3  (c) 2 (d) None of these

(a) tan x = 1 (b) tan2 x = 1 10. The function f : R → R, f (x) = x 2, ∀ x ∈ R is


(c) tan3 x = 1 (d) None of these (a) injective but not surjection.
1 (b) surjection but not injection
3. One root of the equation cos x − x + = 0 lies (c) injection as well as surjection
in the interval 2
(d) neither injection nor surjection
π π π 3π
(a)  0,  (b)  − , 0 (c)  , π (d)  π,  11. Match the following columns
 2   2   2   2 
a cos A + bcos B + c cos C Column I Column II
4. The value of is
a + b+ c A. 1+ 3i 1. 2 cis (− 2 π / 3)
1 r R 1
(a) (b) (c) (d) B. 1− 3i 2. 2 cis (2 π / 3)
r R r R
C. − 1+ 3i 3. 2 cis (− π / 3)
π
5. If sin − 1(1 − x) − 2sin − 1 x = , then x equals D. − 1− 3i 4. 2 cis ( π / 3)
2
(a)  0, −  (b)  , 0
1 1
(c) {0} (d) (− 10
, ) Codes
 2 2  A B C D A B C D
π π (a) 3 4 1 2 (b) 2 1 3 4
6. If − < x < , then the value of log sec x is
2 2 (c) 4 3 1 2 (d) 4 3 2 1
(a) 2 coth− 1  cosec 2 − 1
x
12. If ω is a cube root of unity and p = a + b,
 2 
− 1 q = aω + bω 2 , r = aω 2 + bω, then the value of
(b) 2 coth  cosec 2 x
+ 1
 2  pqr is
− 1 2 x  (a) a2 b + ab 2 (b) a2 + ab + b 2
(c) 2cosech  cot − 1
 2  (c) a3 + b 3 (d) (a + b )3
(d) None of the above
 1  1 13. If z1, z 2 are any two complex numbers such
7. tanh − 1   + tanh − 1   is equal to that | z1 + z 2 | = | z1 | + | z 2 | which one of the
 2  3
following is correct?
(a) tanh− 1   (b) tanh− 1  
5 7
(a) z1 ≥ 0 or z2 ≥ 0 (b) z1 = αz2 with α ∈ R
7  5
(c) z1 = αz2 with α > 0 (d)| z1 | = | z2 |
(c) tanh− 1   (d) tanh− 1  
1 5
 6  6 14. How many roots of the equation
2 2
8. If ABCD is a cyclic quadrilateral, then the value x− =1− have?
of cos A − cos B + cos C − cos D is equal to x −1 x −1
(a) 0 (b) 1 (a) One (b) Two
(c) 2(cos B − cos D) (d) 2(cos A − cos C ) (c) Infinite (d) None of these
460 AP EAMCET Chapterwise Mathematics

(c) A differentiable function is always continuous


15. The set of real values of x satisfying
(d) e x is continuous for all x
log 1 / 2 (x − 6 x + 12) ≥ − 2 is
2

f (5) − f ()
1
(a) (− ∞, 2 ] (b) [2, 4] 23. If f (x) = x 2 − 2 x + 4 and = f ′ (c),
5 −1
(c) [4, ∞ ) (d) None of these
x +2 2 − 2x then value of c will be
 2  3
16. If   =  , then x is equal to (a) 0 (b) 1 (c) 2 (d) 3
 3  2
24. If 0 < x < y, then lim (y + x ) n n 1/ n
is equal to
(a) 0 (b) 1 (c) 3 (d) 4 n→ ∞

(a) e (b) x
17. The value of [12 − (68 + 48 2)] is (c) y (d) None of these
(a) 2 + 2 (b) 2 − 2 25. If p and q be positive, then the coefficient of x p
(c) 2 − 1 (d) None of these
and x q in the expansion of (1 + x)p + q will be
x − 6 x + 10 x − 2
3 2
(a) equal
18. If
x2 − 5x + 6 (b) equal in magnitude but opposite in sign
A B (c) reciprocal to each other
= f (x) + + , then f (x) is equal to (d) None of the above
(x − 2) (x − 3)
(a) x − 1 (b) x + 1 26. The value of 10C1 +10 C 3 +10 C 5 +10 C 7 +10 C 9 is
(c) x (d) None of these (a) 2 9 (b) 210
(c) 210 − 1 (d) None of these
19. Assertion (A) The equation
x 2 + 2 ax − b2 = 0 can have repeated roots, 27. If n ∈ N , then11n + 2 + 122n + 1 is divisible by
where a , b are real numbers. (a) 113 (b) 123
Reason (R) The equation Ax + Bx + C = 0 2 (c) 133 (d) None of these
will have repeated roots when the x 2 − y2 x 4 − y4
discriminant is not zero. 28. The sum of the series +
1! 2!
(a) Both (A) and (R) are true and (R) is the correct x 6 − y6
explanation of (A) + +K ∞ is
(b) Both (A) and (R) are true and (R) is not the 3!
2 2
correct explanation of (A). (a) e x − e y (b) e x − e y
(c) (A) is true but (R) is false. 2 2 e x − ey
(c) 2 + e x − e y (d)
(d) (A) is false but (R) is true. 2
20. If the sum of the squares of the roots 29. The sum of the series log 4 2 − log 8 + log 16 2K is
x 2 − (p − 2) x − (p + 1) = 0 , (p ∈ R) is 5, then (a) e 2 (b) loge 2
what is the value of p? (c) loge 3 − 2 (d) 1 − loge 2
3
(a) 0 (b) 1 (c) − 1 (d) 6i − 3i 1
2
 ax 2 − b, 30. Let 4 3i − 1 = x + iy, then
when 0 ≤ x < 1
 20 3 i
21. If f (x) =  2, when x =1
 x + 1, when 1 < x ≤ 2 (a) x = 3, y = 1 (b) x = 0, y = 0
 (c) x = 0, y = 3 (d) x = 1, y = 3
is continuous at x = 1, then the most suitable
31. If a , b, c are in AP, then the value of
values of a , b are
(a) a = 2, b = 0 (b) a = 1, b = − 1 x +2 x +3 x+a
(c) a = 4, b = 2 (d) All of these x+4 x +5 x + b is
22. Which of the following is not true? x +6 x +7 x+c
(a) A polynomial function is always continuous (a) x − (a + b + c ) (b) 9 x2 + a + b + c
(b) A continuous function is always differentiable (c) a + b + c (d) 0
Practice Set 3 461
32. The system of linear equations x + y + z = 2,
2 x + y − z = 3, 3 x + 2 y + kz = 4 has a unique
40. On the parabola y = x 2, the point least
solution, if distance from the straight line y = 2 x − 4 is
(a) k ≠ 0 (b) − 1 < k < 1 (a) (1, 1) (b) (1, 0)
(c) − 2 < k < 2 (d) k = 0 (c) (1, − 1) (d) (0, 0)

33. If AB = C , then matrices A , B and C are 41. If P(3, 4 , 5), Q(4 , 6 , 3), R(− 1, 2, 4) and S(1, 0 , 5),
(a) A2 × 3 , B3 × 2 , C 2 × 3 (b) A3 × 2 , B2 × 3 , C 3 × 2 then the projection of RS on PQ is
(c) A3 × 3 , B2 × 3 , C 3 × 3 (d) A3 × 2 , B2 × 3 , C 3 × 3 (a) − 2 / 3 (b) − 4 / 3 (c) 1/2 (d) 2
(n + 2)! 42. The length of the perpendicular drawn from
34. Assertion (A) is divisible by b x −1 y z
(n − 1)! the point (5 , 4 , − 1) on the line = = is
2 9 5
Reason (R) Product of 3 consecutive integers 110 2109 2109
(a) (b) (c) (d) 54
is divisible by 3! 2109 110 110
(a) Both (A) and (R) are true and (R) is the correct 43. The equation of the plane which is parallel to
explanation of (A) the plane x − 2 y + 2 z = 5 and whose distance
(b) Both (A) and (R) are true and (R) is not the from the point (1, 2, 3) is 1, is
correct explanation of (A) (a) x − 2 y + 2 z = 3 (b) x − 2 y + 2 z + 3 = 0
(c) (A) is true but (R) is false (c) x − 2 y + 2 z = 6 (d) x − 2 y + 2 z + 6 = 0
(d) (A) is false but (R) is true 2
4 n − 3n 44. The position vectors of the points A , B, C are
35. If the sum to n terms of an AP is ,
4 (2i$ + $j − k$ ), (3 $i − 2 $j + k$ ) and ($i + 4 $j − 3 k$ )
then the nth term of an AP.
5n − 1 8n − 7 respectively. These points
(a) (b) (a) form an isosceles triangle
4 4
3n2 − 2 (b) form a scalene triangle
(c) (d) None of these (c) are collinear
4
(d) None of the above
36. The equation of latusrectum of a parabola is 45. If (a × b) × c = a × (b × c), where a, b and c are
x + y = 8 and the equation of the tangent at
the vertex is x + y = 12, then length of the any three vectors such that a ⋅ b ≠ 0, b ⋅ c ≠ 0,
latusrectum is then a and c are
π
(a) inclined at an angle of between them
(a) 4 2 (b) 2 2 (c) 8 (d) 8 2 6
(b) perpendicular
37. If the normals at two points P and Q of a (c) parallel
parabola y 2 = 4 ax intersect at a third point R (d) None of the above
on the curve, then the product of ordinates of 46. If a, b and c are position vector of vertices of a
P and Q is triangle ABC, then unit vector perpendicular
(a) 2 a2 (b) 4a2 to its plane is
(c) 8a2 a ×b + b ×c + c ×a
(d) None of these (a)a × b + b × c + c × a (b)
| a × b + b × c + c × a|
38. The equation of the ellipse whose centre is a×b
(2 , − 3), one of the foci is (3 , − 3) and the (c) (d) None of these
| a × b|
corresponding vertex is (4 , − 3), is
( x − 2 )2 ( y + 3)2 x3 y2 47. The volume of the parallelopiped whose
(a) + = 1 (b) + =1
3 4 3 4 coterminous edges are i$ − $j + k$ , 2 i$ − 4 $j + 5 k$
( x − 2) 2
( y + 3)2
and 3 $i − 5 $j + 2 k$ , is
(c) + = 1 (d) None of these
4 3 (a) 2 (b) 3 (c) 4 (d) 8
39. The equation13 [(x − 1)2 + (y − 2)2] d  x
48. The value of  x 3 tan 2  is
= 3(2 x + 3 y − 2)2 represents dx  2
(a) parabola (b) ellipse x x x
(a) x3 tan ⋅ sec 2 + 3 x2 tan2
(c) hyperbola (d) circle 2 2 2
462 AP EAMCET Chapterwise Mathematics
x x x 55. The centre of circle passing through the
(b) x3 tan2 ⋅ sec 2 + 3 x2 tan2
2 2 2 points (0, 0), (1, 0) and touching the circle
x x x x 2 + y 2 = 9 , is
(c) x3 tan ⋅ sec 2 − 3 x2 tan2
(a)  ,  (b)  , 
2 2 2 3 1 1 1
(d) None of the above 2 2 2 2
(c)  −
1 1
(d)  , − 2 
1
 1− x 2  , 
49. If y = cos −1   , then the first derivative is  2 2 2 
1 + x 2 
1 1 2 2
56. The greatest distance of the point P(10 , 7)
(a) (b) − (c) − (d)
1+ x 2
1+ x 2
1+ x 2
1+ x 2 from the circle x 2 + y 2 − 4 x − 2 y − 20 = 0 is
(a) 10 (b) 15
y − 1 (c) 5 (d) None of these
50. If u = tan   , then by Euler’s theorem the
 x 57. A tower subtends an angle of 30° at a point
δu δu distant d from foot of the tower and on the
value of x + y is
δx δy same level as the foot of the tower. At a
(a) 0 (b) sinu (c) tanu (d) cos2u second point h vertically above the first, the
depression of the foot of the tower is 60°. The
− x 2 − y2
51. If u = e , then which of the following height of the tower is
option is correct? h h 3h
(a) (b) (c) 3h (d)
(a) xu x = yu y (b) yu x + xu y = 0 3 3d d

(c) x u y + y u x = 0
2 2
(d) None of these 58. There are n letters and n addressed envelopes.
The probability that all the letters are not kept
b
52. Trapezoidal rule for evaluation of ∫a f (x) dx in the right envelope is
1 1
(a) (b) 1 −
requires the interval (a , b) to be divided into n! n!
(a) 2n sub-intervals of equal width 1
(c) 1 − (d) None of these
(b) 2 n + 1 sub-intervals of equal width n
(c) Any number of sub-intervals of equal width
59. A locker can be opened by dialing a fixed
(d) 3n sub-intervals of equal width
three digit code (between 000 and 999). A
b
53. If ∫a f (x) dx is numerically integrated by stranger who does not know the code tries to
open the locker by dialing three digits at
Simpson’s rule, then in any pair of random. The probability that the stranger
consecutive sub-intervals by which of the succeeds at the kth trial is
following curves, the curve y = f (x) is k k
approximated a/an (a) (b)
999 1000
(a) straight line (b) parabola k −1
(c) (d) None of these
(c) circle (d) ellipse 1000
54. Match the following columns 60. A basket contains 5 apples and 7 oranges and
Column I Column II another basket contains 4 apples and
A. x 2 + x + 1 = y 1. Pair of straight 8 oranges. One fruit is picked out from each
lines basket. The probability that the fruits are both
B. x 2 + y 2 + 2 x + 2 y − 6 = 0 2. Circle apples or both oranges is
(a) 24/144 (b) 56/144 (c) 68/144 (d) 76/144
C. 2 x 2 + 3 y 2 + 4 x + 6 y = 0 3. Parabola
D. 3 x 2 − 2 y 2 + 6 x − 4 y = 0 4. Ellipse 61. A sample of 4 items is drawn at random
without replacement from a lot of 10 items
5. Hyperbola
containing 3 defective. If x denotes the
Codes number of defective items in the sample, then
A B C D A B C D P(0 < x < 3) is equal to
(a) 1 2 3 5 (b) 3 2 4 5 1 1 4 3
(a) (b) (c) (d)
(c) 2 3 4 5 (d) None of these 2 6 5 10
Practice Set 3 463

sin 2 x
62. The value of ∫ dx is 69. The solution of the differential equation
sin 4 x + cos 4 x dy
(x 2 − yx 2) + y 2 + xy 2 = 0 is
−1 −1
(a) cot (tan x) + c
2
(b) tan (tan x) + c2 dx
(a) log  = +
y 1 1 1 1
(c) cot − 1(cot 2 x) + c (d) None of these +c (b) log( xy) ++ =c
 x x y x y
63. ∫ [ f (x) g ′′(x) − f ′′(x) g(x)] dx is equal to 1
(c) log( xy) = +
1
+c
 x 1
(d) log  = +
1
+c
f ( x) x y  y x y
(a) (b) f( x))g ′( x) − f ′( x)g ( x)
g ′ ( x) 70. In how many ways can 10 balls be divided
(c) f ′( x)g ( x) − f( x)g ′( x) (d) f( x)g ′( x) + f ′( x)g ( x) between two boys, one receiving two and the
64. The edge of a cube is increasing at the rate of other eight balls?
5 cm/s. How fast is the volume of the cube (a) 45 (b) 75
increasing when the edge is 12 cm long? (c) 90 (d) None of these
(a) 432 cm3 / s (b) 2160 cm3 / s 71. The number of ways to sit 3 men and 2 women
(c) 180 cm3 / s (d) None of these in a bus such that total number of seated men
and women on each side is 3, is
65. The function x 1 − x 2 , (x > 0) has (a) 5! (b) 6! × 6 P5 (c) 6 C 5 × 5! (d) 5! + 6C 5
(a) a local maxima
 1 13 
(b) a local minima 72. Point  , −  divides the line joining the
(c) neither a local maxima nor a local minima 2 4
(d) None of the above points (3 , − 5) and (− 7 , 2) in the ratio of
(a) 3 : 1 internally (b) 3 : 1 externally
66. What is the area bounded by x 2 + y 2 = 9 and
(c) 1 : 3 externally (d) 1 : 3 internally
y 2 = 8 x?
73. The locus of a point whose difference of
(a) 0
distance from points A(3 , 0) and B(− 3 , 0) is 4, is

− 9sin− 1  
2 2 1
(b) + x2 y2 x2 y2
3 2  3 (a) − =1 (b) − =1
(c) 16π 4 5 5 4
2 2
x y
(d) None of the above (c) − =1 (d) None of these
∞ −x ∞ − λx 2 3
∫ ∫
n−1 n−1
67. If I n = e x dx , then e x dx is
0 0 74. If the sum of the slopes of the lines represe
equal to -nted by the equation x 2 − 2 xy tan A − y 2 = 0
(a) λIn (b) λIn
In 1 be 4, then ∠A is equal to
(c) (d) In (a) 0° (b) 45° (c) 60° (d) tan−1(−2 )
λn λ
68. The differential equation for all the straight 75. The equation 8 x 2 + 8 xy + 2 y 2 + 26 x + 13 y
lines which are at a unit distance from the + 15 = 0 represents a pair of straight lines. The
origin is distance between is
2 2 7 7
(a)  y − x  = 1 −  
dy dy (a) (b)
5 2 5
 dx   dx 
7
2 2 (c) (d) None of these
(b)  y − x  = 1 +  dy 
dy 5
 
 dx   dx  76. A line meet X -axis and Y -axis at the points A
2 2
and B respectively. If the middle point of AB
(c)  y + x  = 1 −  
dy dy
 dx   dx  be (x1 , y1), then the equation of the lines is
2 2 (a) y1 x + x1 y = 2 x1 y1 (b) x1 x + y1 y = 2 x1 y1
(d)  y + x  = 1 +  dy 
dy
  (c) y1 x + x1 y = x1 y1 (d) x1 x + y1 y = x1 y1
 dx   dx 
464 AP EAMCET Chapterwise Mathematics

77. The two consecutive sides of a parallelogram 79. The product of the determinants
are 4 x + 5 y = 0 and 7 x + 2 y = 0 . If the
log 3 512 log 43 log 2 3 log 8 3
equation to one diagonal is11 x + 7 y = 9 , then × is
the equation of the other diagonal is log 3 8 log 49 log 3 4 log 3 4
(a) x − y = 0 (b) 2 x + y = 0
(c) x + 2 y = 0 (d) None of these (a) 7 (b) 10
(c) 13 (d) 17
p + q− x q+ r− x
78. The solution of equation + 80. A force of magnitude 5 unit acting along the
r p
r+p− x 4x vector 2 $i − 2 $j + k$ displaces the point of
+ + = 1 is application from (1, 2, 3) to (5, 3, 7), then the
q p+q+r
work done is
(a) x = p − q + r (b) x = p + q + r 50 50 25 25
p+ q p (a) (b) (c) (d)
(c) x = (d) x = + r 7 3 3 4
q+ r q

Answers
1. (c) 2. (c) 3. (a) 4. (b) 5. (c) 6. (a) 7. (a) 8. (a) 9. (a) 10. (d)
11. (d) 12. (c) 13. (b) 14. (d) 15. (b) 16. (d) 17. (b) 18. (a) 19. (c) 20. (b)
21. (d) 22. (b) 23. (d) 24. (c) 25. (a) 26. (a) 27. (c) 28. (b) 29. (d) 30. (b)
31. (d) 32. (a) 33. (d) 34. (a) 35. (b) 36. (d) 37. (c) 38. (c) 39. (c) 40. (a)
41. (b) 42. (b) 43. (c) 44. (c) 45. (c) 46. (b) 47. (d) 48. (a) 49. (d) 50. (a)
51. (d) 52. (c) 53. (b) 54. (b) 55. (d) 56. (b) 57. (a) 58. (b) 59. (b) 60. (d)
61. (c) 62. (b) 63. (b) 64. (b) 65. (a) 66. (b) 67. (c) 68. (b) 69. (d) 70. (c)
71. (c) 72. (d) 73. (a) 74. (d) 75. (b) 76. (a) 77. (a) 78. (b) 79. (b) 80. (b)

Solutions
1. (c) 1 − sinθ + 1 + sinθ 3. (a) Let f (x) = cos x − x + 1
1 + sinθ 1 − sinθ 2
1 3
1 − sin θ + 1 + sin θ 2 At x = 0, f (0) = 1 − 0 +
= >0
= = 2 2
1 − sin 2 θ | cos θ | π  π π 1 1−π
At x = , f  =0− + = <0
2 2  2 2 2 2
=− (Qθ lies in IInd quadrant)
cosθ  π
∴One root lies in the interval 0, .
= − 2secθ  2 

2. (c) Now, tan x + tan π + x  + tan 2π + x  4. (b) Now, a cos A + b cos B + c cos C
3   3  a+ b+ c
tan x + 3 tan x − 3 k(sin A cos A + sin B cos B + sin C cos C)
= tan x + + =
1 − 3 tan x 1 + 3 tan x k(sin A + sin B + sin C)
8 tan x 1 sin 2 A + sin 2B + sin 2C
= tan x + = ⋅
1 − 3 tan 2 x 2 (sin A + sin B + sin C)
3(3 tan x − tan 3 x)  
= = 3 tan 3 x
1 − 3 tan 2 x 1  2sin(A + B) ⋅ cos(A − B) + 2sin C cos C 
=  
2  A + B  A − B C C
⇒ 3 = 3 tan 3 x 2sin   cos   + 2sin cos 
  2   2  2 2 
⇒ tan 3 x = 1 (given)
Practice Set 3 465
  8. (a) Since, ABCD is a cyclic quadrilateral.
1  sin C{cos(A − B) − cos(A + B)} 
=   ⇒ A = 180° − C
2 C   A − B  A + B   ⇒ cos A = cos(180° − C) = − cosC
cos cos   +  
 2   
cos
 2  2 
 ⇒ cos A + cos C = 0 … (i)
  Now, B + D = 180°
1  sin C(2sin A sin B)  then cos B + cos D = 0 … (ii)
=  
2 C A B On subtracting Eq. (ii) from Eq. (i), we get
cos  2 cos cos  
 2 2 2   cos A − cos B + cos C − cos D = 0
 A A B B C C
1
2sin cos ⋅ 2sin cos ⋅ 2sin cos 9. (a) (gof) − 5 − ( fog) − 5
=  2 2 2 2 2 2  3  3
2 A B C 
 cos cos cos    5    5 
 2 2 2  = g f  −   − f  g −  
A B C r   3    3 
= 4 sin sin sin = [Q f (x) = [ x], ag(x) = | x |]
2 2 2 R
 5
= g(− 2) − f   = 2 − 1 = 1
5. (c) Given that, sin− 1 (1 − x) − 2sin− 1 x = π  3
2
π
⇒ sin (1 − x) = + 2sin − 1 x
−1 10. (d) It is clear from the graph that, for two different
2 values of x, we will get the same values of y. And
π  also, graph is not defined below the X-axis.
⇒ 1 − x = sin  + 2sin − 1 x
2 
π −1
⇒ 1 − x = sin cos(2sin x)
2
π
+ cos sin(2sin − 1 x)
2
⇒ 1 − x = cos(2sin − 1 x)
⇒ 1 − x = cos {cos − 1 (1 − 2 x 2)} Hence, it is neither injection nor surjection.
 
11. (d) (a)1 + 3i = 2 1 + 3 i  = 2 cis π 
1
⇒ 2 x 2 − x = 0 ⇒ x = 0, ⇒ x = 0,
2 2 2   3
1
(Q x = does not satisfy the equation) 1 3   π
2 (b)1 − 3i = 2 − i = 2 cis − 
2 2   3
1 e y/ 2
6. (a) Let y = log sec x ⇒ = − y/ 2  1 3   2π 
cos x e (c) − 1 + 3i = 2 − + i = 2 cis 
 2 2   3
Applying componendo and dividendo, we get
1 + cos x e y / 2 + e − y / 2  1 3   2π 
= (d) − 1 − 3i = 2 − − i = 2 cis − 
1 − cos x e y / 2 − e − y / 2  2 2   3

 x  y 12. (c) Given that, p = a + b,


⇒ cot 2   = coth  
 2  2 q = aω + bω 2, r = aω 2 + bω
 x  ∴ pqr = (a + b) (aω + bω 2) (aω 2 + bω)
⇒ y = 2 coth − 1  cosec 2 − 1
 2  = (a + b) (a 2ω 3 + abω 2 + abω 4 + b 2ω 3)
= (a + b) (a 2 + ab(ω 2 + ω) + b 2)
7. (a) tanh− 1  1  + tanh− 1  1 
 2  3 = (a + b) (a 2 − ab + b 2) (Q1 + ω + ω 2 = 0)
 1 1  = a3 + b3
 + 
−1 − 1  5
= tanh  2 3  = tanh   13. (b) Since,| z1 + z2 | = | z1 | + | z2 |
1 1
1 + .   7
 2 3 ⇒ arg(z1) = arg(z2) ⇒ z1 = αz2, where α ∈ R.
466 AP EAMCET Chapterwise Mathematics

2 2 ⇒ (p − 2)2 + 2(p + 1) = 5
14. (d) We have, x − =1 − . If x ≠ 1
x −1 x −1 ⇒ p 2 − 4 p + 4 + 2p + 2 = 5
multiplying each term by (x − 1), the given equation ⇒ p 2 − 2p + 1 = 0
reduces to x(x − 1) = (x − 1) or (x − 1)2 = 0 or x = 1,
which is not possible. ⇒ (p − 1)2 = 0 ⇒ p = 1
Hence, it has no root exist. ax 2 − b , when 0 ≤ x < 1
15. (b) Given that, 21. (d) Given that, f (x) =  2, when x =1
 x + 1, when 1 < x ≤ 2
log1 / 2(x 2 − 6 x + 12) ≥ − 2 … (i) 
For log to be defined x 2 − 6 x + 12 > 0 At x = 1,
⇒ (x − 3) + 3 > 0 which is true ∀ x ∈ R
2 LHL, lim ax 2 − b = a − b
x →1
−2
 1 RHL, lim x + 1 = 2 and f ()
1 =2
From Eq. (i), x 2 − 6 x + 12 ≤   x →1
 2
Since, f (x) is continuous at x = 1
⇒ x 2 − 6 x + 12 ≤ 4 ⇒ x 2 − 6 x + 8 < 0
∴ LHL = f ()
1
⇒ (x − 2) (x − 4) ≤ 0 ⇒ 2 ≤ x ≤ 4 ⇒ a−b=2
x + 2 2 − 2x
16. (d) Given that,  2 =  
3 Hence, all the values of the options are
  3  2 satisfied, therefore option (d) is correct.
x + 2 2x − 2
 2  2 22. (b) A continuous function may or may not be
⇒   = 
 3  3 differentiate.
Hence, option (b) is not true.
⇒ x + 2 = 2x − 2 ⇒ x = 4
23. (d) Given that, f (x) = x 2 − 2x + 4,
17. (b) 12 − 68 + 48 2
⇒ f ′ (x) = 2 x − 2
= 12 − 62 + (4 2)2 + 2 ⋅ 6 ⋅ 4 2 At x = c, f ′ (c) = 2c − 2
= 12 − (6 + 4 2)2 = 12 − 6 − 4 2 Now, f (5) = 52 − 2(5) + 4 = 19

= 6 − 4 2 = 22 + ( 2)2 − 2 ⋅ 2 2 1 = 12 − 21
and f () () + 4 = 3

=2− 2 f (5) − f ()
1
Q = f ′ (c)
5 −1
18. (a) x 2 − 5x + 6) x 3 − 6 x 2 + 10 x − 2(x − 1 19 − 3 16
⇒ = 2c − 2 ⇒ = 2c − 2
x3 − 5x2 + 6x 5−1 4
− + −
⇒ c=3
− x2 + 4 x − 2
24. (c) Since, 0 < x < y
− x2 + 5x − 6
1/ n
+ − +   x 
n
∴ lim (x n + y n)1 / n = y lim 1 +   
−x+4 n→ ∞ n→ ∞
  y 

∴ f (x) = x − 1 n
1 x
 
19. (c) (A) is true, but (R) is false.   y n y
n

 n   x  
 x
20. (b) Let α and β be the roots of = y lim 1 +    
n → ∞   y  
x 2 − (p − 2) x − (p + 1) = 0. Then,  
α + β = (p − 2) and αβ = − (p + 1)  x n 
 x
Q α 2 + β2 = 5 = ye 0 = y Q < 1 ⇒   → 0 as n → ∞ 
 y  y  
⇒ (α + β)2 − 2αβ = 5
Practice Set 3 467
25. (a) Coefficient of x p in (1 + x)p + q is p + q
Cp and
coefficient of x q in (1 + x)p + q is p + q
Cq 32. (a) The given system of linear equations has
p+ q p+ q unique solution
But, Cp = Cq
1 1 1
Hence, option (a) is correct. = 2 1 −1 ≠ 0
26. (a) We know that, 3 2 k
2n − 1 = nC1 + nC 3 + nC 5 + K ⇒ 1(k + 2) − 1(2k + 3) + 1(4 − 3) ≠ 0 ⇒ k ≠ 0
10 − 1
∴ 10
C1 + C3 +
10 10
C5 + K + C9 = 2
10
=2 9
33. (d) Given that AB = C
Hence, option (d) is correct.
27. (c) Let P(n) = 11n + 2 + 122n + 1
Put n = 1, we get 34. (a) (A) (n + 2)! = (n + 2) (n + 1)n(n − 1)!
(n − 1)! (n − 1)!
1 = 113 + 123 = 3059
P()
= (n + 2)(n + 1)n
Which is divisible by 133.
It is a product of 3 consecutive integrals and it
 2 4   2 4  is divisible by 3!.
28. (b) Let S =  x + x + .. −  y + y + ...
 1! 2!   1! 2!  Hence, option (a) is correct.

35. (b) Since, Sn = 4n − 3n ⇒ Tn = Sn − Sn − 1


2 2 2 2 2
= e x − 1 − (e y − 1) = e x − e y
4
29. (d) Let S = log 4 2 − log 8 2 + log16 2 − K 4 n 2 − 3n 4(n − 1)2 − 3(n − 1)
= −
1 1 1 1 4 4
= − + − + K = 1 − log e 2
2 3 4 5 4 n 2 − 3n − 4(n 2 − 2n + 1) + 3n − 3 8n − 7
= =
6i − 3i 1 4 4
30. (b) Given, 4 3i − 1 = x + iy 36. (d) Since, equation of latusrectum is x + y = 8 and
20 3 i vertex of the parabola is x + y = 12

⇒ 6i(− 3 + 3) + 3i(4 i + 20) +112


( − 60i) = x + iy − 12 −8 4
∴ a= − =
⇒ 0 + 60i − 12 + 12 − 60i = x + iy 1+1 1+1 2
4
⇒ x = 0, y = 0 Length of latusrectum = 4a = 4 × =8 2
2
x+ 2 x+ 3 x+ a
31. (d) Let A = x + 4 x + 5 x + b 37. (c) Let the points on the parabola be P(at12 , 2at1)
x+ 6 x+ 7 x+ c and Q(at22 , 2at2).

Applying C 2 → C 2 − C1 , we get We know, t1 × t2 = 2 ⇒ 2at1 × 2at2 = 8a 2


x+2 1 x+a
38. (c) Since, foci = (3, − 3) and centre = (2, − 3)
A= x+4 1 x+b
x+6 1 x+c ∴ ae = 3 − 2 = 1
Also, vertex = (4 , − 3)
Applying R2 → R2 − R1 and R3 → R3 − R1
1
x+2 1 x+a ∴ a=4 −2=2 ⇒ e =
2
= 2 0 b−a
1 2
4 0 c−a Now, b=a 1− = 3= 3
4 2
= − 1(2c − 2a − 4 b + 4 a) Hence, required equation of ellipse is
= 2(2b − c − a) (x − 2)2 (y + 3)2
+ =1
∴ A=0 (Q 2b = a + c) 4 3
468 AP EAMCET Chapterwise Mathematics
39. (c) Given equation of curve is 44. (c) Given that
13[(x − 1)2 + (y − 2)2] = 3(2 x + 3 y − 2)2 OA = 2i$ + $j − k$
⇒ 13(x 2 + y 2 − 2 x − 4 y + 5) OB = 3$i − 2$j + k$ and OC = i$ + 4 $j − 3k$
= 3(4 x 2 + 9 y 2 + 4 + 12 xy − 12 y − 8 x) Now, AB = (3 − 2)$i + (− 2 − 1)$j + (1 + 1)k$
⇒ x 2 − 14 y 2 − 2 x − 16 y + 53 = 0 = i$ − 3$j + 2k$
2 2
Here, coefficient of x and y have opposite ⇒ | AB| = 1 + 9 + 4 = 14
signs. So, it represents a hyperbola
BC = (1 − 3)i$ + (4 + 2)$j + (− 3 − 1)k$
40. (a) Given equation of parabola is y = x 2 … (i)
= − 2i$ + 6$j − 4 k$
And straight line y = 2 x − 4 … (ii)
⇒ | BC| = 4 + 36 + 16 = 56 = 2 14
From Eqs. (i) and (ii), we get
x2 − 2x + 4 = 0 and CA = (2 − 1)$i + (1 − 4)$j + (− 1 + 3)k$
Let f (x) = x 2 − 2 x + 4 = $i − 3$j + 2k$ ⇒| CA | = 1 + 9 + 4 = 14
On differentiating w.r.t. x, we get It also|AB| + |AC| = |BC|
f ′ (x) = 2 x − 2
∴Angle between AB and BC is 180°.
For least distance, f ′ (x) = 0 ⇒ 2 x − 2 = 0
∴ Points A , B and C cannot form an isosceles
⇒ x =1
triangle. Hence, it is a collinear points.
∴From Eq. (i) y =1
Hence, the nearest point is (1, 1). 45. (c) Since, (a × b) × c = a × (b × c)
41. (b) Since, points are P(3, 4, 5, ) R(− 1, 2, 4)
) Q(4, 6, 3, ⇒ (a ⋅ c) b − (b ⋅ c)a = (a ⋅ c)b − (a ⋅ b) c
and S(1, 0, 5) ⇒ (b ⋅ c) a = (a ⋅ b) c
DR’s of RS = 1 + 1, 0 − 2, 5 − 4 = 2, − 2, 1 ⇒ a is parallel to c.
DR’s of PQ = 4 − 3, 6 − 4, 3 − 5 = 1, 2, − 2
46. (b) Unit vector perpendicular to plane
1 2 2
⇒ DC’s of PQ = , , − = (l1 , m1 , n1) (say) AB × AC (b − a) × (c − a)
3 3 3 = =
| AB × AC | | (b − a) × (c − a) |
∴Projection of RS on PQ = Σl(x 2 − x1)
1 2 2 2 4 2 4 a× b+ b× c+ c× a
= (2) + (− 2) − () 1 = − − =− =
3 3 3 3 3 3 3 |a × b + b × c + c × a|
42. (b) Required distance 1 −1 1

(5 − 1) + (4 − 0) + (− 1 − 0)
2 2 2 47. (d) Volume of parallelopiped = 2 − 4 5
2 3 −5 2
= (5 − 1)2 + (4 − 0)9 + (− 1 − 0)5 
−  = 1(− 8 + 25) + 1(4 − 15) + 1(− 10 + 12)
 4 + 81 + 25 
= 17 − 11 + 2 = 8 cu unit.
39 × 39 2109
= 33 −
110
=
110 48. (a) d  x 3 tan2 x 

dx 
2
43. (c) Any equation of plane parallel to x x x 1
= 3 x 2 tan 2
+ 2 x 3 tan sec2 ⋅
x − 2 y + 2z = 5 is x − 2 y + 2z + k = 0 2 2 2 2
According to the given condition x 2x 2 x
= x tan sec + 3 x tan
3 2
1−4 + 6+ k 2 2 2
= ±1
12 + 22 + 22  2
49. (d) Given that y = cos− 1  1 − x 2 
⇒ 3 + k = ± 3 ⇒ k = 0, − 6 1 + x 
∴ x − 2 y + 2z − 6 = 0 or x − 2 y + 2z = 6 Put x = tanθ
Practice Set 3 469

 1 − tan 2 θ  (D) 3 x 2 − 2 y 2 + 6 x − 4 y = 0
∴ y = cos −1  
1 + tan 2 θ  Here, a = 3, b = − 2, c = 0,
−1
= cos (cos 2θ) = 2θ ⇒ y = 2 tan −1
x h = 0, g = 3, f = − 2

dy 2 ∴ ∆ = 0 + 0 − 3(− 2)2 + 2(3)2 − 0 = 6


On differentiating w.r.t.x, we get =
dx 1 + x 2 ⇒ ∆≠0
Now, ab = − 6 and h 2 = 0 ⇒ h 2 > ab
50. (a) Given u = tan− 1 y = x 0 ⋅ tan− 1 y
x x ∴ It is a hyperbola.
Clearly, u is homogeneous in x , y of degree 0. 55. (d) Let the equation of circle passing through
∴ By Euler theorem. origin is
δu δu x 2 + y 2 + 2 gx + 2 fy = 0
x +y = 0. u = 0
δx δy It passes through (1, 0), then
1
51. (d) Given that g=−
2
− y2 2
− y2 2
u = e− x ⇒ ux = e − x (− 2 x) = − 2ux 2
−2  1 1
and u y = e −x − y 2
(− 2 y) = − 2uy ⇒ yux = xu y ∴ Radius, r1 =  −  + f 2 − 0 = + f2
 2 4
52. (c) Trapezoidal rule divides the given interval into 1 
and its centre, C1 =  , − f 
any number of sub-intervals of equal width. 2 
53. (b) Required curve is a parabola. Since, the above circle touches the circle
 2 
x + y = 9
2
54. (b) General equation is 
∴Its centre, C 2 = (0, 0) and radius, r2 = 3 
ax 2 + 2hxy + by 2 + 2 gx + 2 fy + c = 0  
(A) x 2 + x + 1 − y = 0 Since, the circle is touching the required circle,
then
1 1 C1C 2 = r2 − r1
Here, a = 1, h = 0, b = 0, g = , f = − , c = 1
2 2
2
∴ ∆ = abc + 2 fgh − af 2 − bg 2 − ch 2  1 1
∴  0 −  + (0 + f ) = 3 −
2
+ f2
2
 2 4
 1 1
= 0 + 0 − 1 −  − 0 − 0 ⇒ ∆ = − ≠ 0 1 1 9
 2 4 ⇒ + f2 =3 ⇒
2 + f2 =
4 4 4
and h 2 = ab = 0. ⇒ f2 =2 ⇒ f =± 2
∴It is a parabola. 1 
∴Centre, C1 =  , ± 2
(B) x 2 + y 2 + 2 x + 2 y − 6 = 0 2 
Here, a = 1, b = 1, h = 0, g = 1, f = 1, c = − 6 56. (b) Let S ≡ x 2 + y2 − 4 x − 2y − 20 = 0
∴ ∆ = 1 ⋅1 ⋅ (− 6) + 0 − 11
()2 − 11
()2 − 0 At point P(10, 7)
= − 6 −1 −1 = − 8 S1 = 102 + 72 − 4 × 10 − 2 × 7 − 20 > 0
⇒ ∆ ≠ 0 and a = b, h = 0
∴ It is a circle.
B A
(C) 2 x 2 + 3 y 2 + 4 x + 6 y = 0 P
C
Here, a = 2, b = 3, g = 2, f = 3, h = 0, c = 0
∴ ∆ = 0 + 0 − 2(3)2 − 3(2)2 − 0
= − 18 − 12 = − 30 ≠ 0 ∴P lies outside the circle, join P with the centre
C(2, 1) of the given circle. Suppose PC cuts the
Now, h = 0 and ab = 6 ⇒ h < ab
2 2
circle at A and B. Then, PB is the greatest
∴ It is an ellipse. distance of P from the circle.
470 AP EAMCET Chapterwise Mathematics

We have, PC = (10 − 2)2 + (7 − 1)2 = 10 7


C1 8 C1
= 12

and CB = radius = 4 + 1 + 20 = 5 C1 12C1

∴ PB = PC + CB = 10 + 5 = 15 ∴ Required probability
5
C1 ⋅4 C1 7
C1 ⋅8 C1 20 + 56 76
57. (a) Let the height of the tower be BC = h1 . = + = =
C1 ⋅12 C1
12
C1 ⋅12 C1
12
144 144
h1
In ∆ABC, tan 30° = ⇒ d = 3h1 … (i)
d 61. (c) Since, the item are chosen without
D replacement.
60°
C C x × 7C 4 − x
3
∴ P(X = x) = 10
C4
h On putting x = 1, 2, we have
h1
C1 × 7C 3 3C 2 × 7C 2
3
P(0 < X < 3) = +
30° 60° 210 210
A d B 3 × 35 + 3 × 21 168 4
= = =
and in ∆ABD, 210 210 5
h h sin 2x
tan 60° = ⇒ 3= 62. (b) Let I = ∫ dx
d d sin4 x + cos4 x
h
⇒ 3= [from Eq. (i)] 2sin x cos x tan x sec2 x
3h1 =∫ dx = 2∫ dx
sin x + cos x
4 4
1 + tan 4 x
h
⇒ h1 = unit. Put tan 2 x = t ⇒ 2 tan x sec2 x dx = dt
3
dt
58. (b) Required probability ∴ I =∫ = tan − 1 t + c = tan − 1 (tan 2 x) + c
1 + t2
= 1 − P (all letters in right envelope)
1 63. (b) ∫ [ f (x) g′′(x) − f ′′(x) g(x)] dx
=1 −
n! = ∫ f (x) g′′(x) dx − ∫ f ′′(x) g(x) dx
59. (b) Let A denote the event that the stranger = f (x) g′ (x) − ∫ f ′ (x) g′ (x) dx
succeeds at the kth trial. Then,
999 998
P(A C) = × ×K× − [ g(x) f ′ (x) − ∫ g′ (x) f ′ (x) dx]
1000 999
100 − k + 1 1000 − k = f (x) g′ (x) − f ′ (x) g(x)
×
100 − k + 2 1000 − k + 1 64. (b) Let edge of a cube be a, then volume, V = a 3
1000 − k On differentiating w.r.t. x, we get
⇒ P (A ) =
C
1000 dV da
1000 − k k = 3a 2 = 3a 2 ⋅ 5 = 15a 2
∴ P(A) = 1 − P(A C) = 1 − = dt dt
1000 1000
= 15 × (12)2 = 2160 cm3/s
60. (d) P(selecting an apple from both baskets)
65. (a) Let f (x) = x 1 − x 2 ⇒ f ′(x) = 1 − 2x
2
= P(apple from first basket)
⋅P(apple from second basket) 1 − x2
5
C1 4 C1 For maxima or minima, put f ′ (x) = 0
= 12 ⋅ 12
C1 C1 1 − 2x2 1
⇒ =0 ⇒ x=±
and P(selecting a orange from both baskets) 1− x 2 2
= P(orange from first basket) But, x > 0, therefore we take x =
1
⋅P(orange from second basket) 2
Practice Set 3 471
(− x) On multiplying Eq. (ii) by x and subtracting
1 − x 2 (−4 x) − (1 − 2 x 2)
1− x 2 Eqs. (i) and (ii), we get
Now, f ′′(x) =  dy 
(1 − x 2) sinα  y − x  = 1
 dx 
2x3 − 3x  1 
= ⇒ f ′′   = − ve  dy 
(1 − x 2)3/ 2  2 ⇒  y − x  = cosec α … (iii)
 dx 
1
Then, f (x) is maximum at x = . dy
2 From Eq. (ii), = − cotα
dx
66. (b) The point of intersection of x 2 + y2 = 9 and  dy 
2
⇒   = cot α
2
… (iv)
y 2 = 8 x is C(1, 2 2) and D(1, − 2 2)  dx 
y ∴From Eqs. (iii) and (iv), we get
2 2
 dy   dy 
1+   = y − x 
C  dx   dx 

69. (d) The given equation can be rewritten as


x′ x 1− y 1+ x
O A B(3, 0) dy + dx = 0
(1, 0) y2 x2
1 1 1 1
⇒  2 −  dy +  2 +  dx = 0
D y y  x x
 x 1 1
y′ On integrating, we get log   = + + c
 y x y
∴ Required area = 2 ∫ y dx + ∫ y dx 
1 3
 0 1  70. (c) Let two boys be A and B. There are two cases
arise
= 2∫ 2 2 x1 / 2dx + ∫ 32 − x 2 dx 
1 3
(1) A gets 2 and B gets 8
 0 1 
10!
Number of ways = = 45
 1
 x 9 − x2 9  
3
2! 8!
 x 3/ 2  − 1  x 

= 2 2 2×
  + + sin   
 3   (2) A gets 8 and B gets 2
 3 / 2  0  2 2
1  10!
 Number of ways = = 45
4 2 9 π 8 9 − 1  1  8! 2!
= 2 + × − − sin   
3 2 2 2 2  3  ∴ Total number of ways = 45 + 45 = 90

2 2 9π
71. (c) 3 men and 2 women equal to 5. A group of 5
 1
= + − 9sin − 1   members make 5! permutation with each other.
3 2  3
∴The number of ways to sit 5 members = 5!

67. (c) Given, I n = ∫ e − x x n − 1 dx 6 places are filled by 5 members by 6C 5 ways.
0
∴The total number of ways to sit 5 members on

Let I = ∫ e − λx x n − 1 dx 6 seats of a bus = 6C 5 × 5!
0
72. (d) Let the ratio be k :1.
Put λx = t ⇒ λdx = dt
− 7k + 3 1 1
1 ∞ 1 ∞ −x I ∴ = ⇒ k=
∴ I = n ∫ e − t t n − 1 dt = n ∫0 e x n −1
dx = nn k+1 2 3
λ 0 λ λ Hence, ratio is 1 : 3 internally.
68. (b) Since, the equation of lines whose distance 73. (a) Let the point be P(h, k).
from origin is unit, is given by
Given that, PA − PB = 4
x cos α + y sin α = 1 … (i)
dy (h − 3)2 + k2 − (h + 3)2 + k2 = 4
⇒ cos α + sin α = 0 … (ii)
dx ⇒ (h − 3)2 + k2 = 4 + (h + 3)2 + k2
472 AP EAMCET Chapterwise Mathematics
On squaring both sides, we get C B
h 2 + 9 − 6h + k2 = 16 + h 2 + 9
11
+ 6h + k2 + 8 (h + 3)2 + k2 x+

0
7y

7x+2y=
=
⇒ − 6h = 16 + 6h + 8 (h + 3)2 + k2 M
9

⇒ − 8 (h + 3)2 + k2 = 12h + 16
Again squaring, we get O 4x+5y=0 A
64(h 2 + 9 + 6h + k2) = 144 h 2 + 256 + 2 ⋅16 ⋅12h (0, 0)

⇒ 4(h 2 + 9 + 6h + k2) = 9h 2 + 16 + 24 h  1 1
Therefore, the middle point M is  ,  .
 2 2
h 2 k2
⇒ 5h 2 − 4 k2 = 20 ⇒ − =1 Hence, the equation of OB is y = x
4 5
x2 y 2 78. (b) Given that p + q − x + q + r − x + r + p − x
Hence, the locus of a point P is − =1 r p q
4 5 4x
=1 −
74. (d) Given pair of lines is p+q+r
2 tan A p+ q+ r− x p+ q+ r− x p + q + r − x
x 2 − 2 xy tan A − y 2 = 0 ⇒ m1 + m2 = 4 = ⇒ + +
−1 r p q
4x
⇒ tan A = − 2 ⇒ A = tan − 1 (− 2) =4−
p+q+r
75. (b) Now, h2 − ab = 42 − 8(2) = 16 − 16 = 0  1 1 1  p + q + r − x
⇒ (p + q + r − x)  + +  = 4  
(13) − 8 ⋅15
2  p q r  p+q+r 
∴ Required distance = 2
8(8 + 2) 1 1 1 4 
⇒ (p + q + r − x)  + + −  =0
169 − 120 7 7  p q r p + q + r
=2 = 2⋅ =
80 4 5 2 5 ⇒x=p + q+ r
76. (a) Let the equation of line be x + y = 1 log 3 512 log 4 3 log 2 3 log 8 3
a b 79. (b) ×
log 3 8 log 4 9 log 3 4 log 3 4
Y
(0, b)  log 512 log 9 log 3 log 8
B = × − × 
 log 3 log 4 log 4 log 3
(x, y)
 log 3 log 4 log 3 log 4 
× × − × 
X′
A X  log 2 log 3 log 8 log 3
Y′ (a, 0)
 log 29 log 32 log 23   log 22 log 22 
= × −  × − 
 a b  log 3 log 22 log 22   log 2 log 23 
∴ Thus middle point of AB is  ,  .
 2 2
 9 × 2 3  2
a b = −  2 −  = 10
i.e. x1 = , y1 =  2 2  3
2 2
∴ The equation of line AB is  $ $ $ 
80. (b) Force, F = 5 2i − 2 j + k$  = 5 (2$i − 2$j + k$ )
x y  | 2$i − 2$j + k | 3
+ = 1 ⇒ xy1 + yx1 = 2 x1 y1
2 x1 2 y1
∴ Required work done
77. (a) Since, equation of diagonal 11 x + 7 y = 9 does 5
= (2$i − 2$j + k$ ) ⋅ [(5$i + 3$j + 7k$ ) − (i$ + 2$j + 3k$ )]
not pass through origin, so it cannot be the 3
equation of the diagonal OB. Thus, on solving the 5
= [(2$i − 2$j + k$ ) ⋅ (4 i$ + $j + 4 k$ )]
equation AC with the equation OA and OC, we get 3
A , −  and C  − ,  .
5 4 2 7 5 50
= (8 − 2 + 4) = unit
3 3  3 3 3 3

You might also like